You are on page 1of 734

ÑAËNG THAØNH NAM

(Trung taâm Nghieân cöùu vaø phaùt trieån saûn phaåm giaùo duïc Newstudy.vn)

n
.v
om
.c
ok
bo
et

SOAÏN THEO CAÁU TRUÙC MÔÙI AÙP DUÏNG KÌ THI THPT QUOÁC GIA
vi

(PHIEÂN BAÛN MÔÙI NHAÁT)


a ng
kh

Daønh cho hoïc sinh 10, 11, 12 naâng cao kieán thöùc.
Boài döôõng hoïc sinh gioûi luyeän thi Quoác Gia.

NHAØ XUAÁT BAÛN ÑAÏI HOÏC QUOÁC GIA HAØ NOÄI


MUÏC LUÏC
Chöông 1: Baát ñaúng thöùc vaø caùc kyõ thuaät cô baûn
Chuû ñeà 1. Kyõ thuaät bieán ñoåi töông ñöông .............................................. 04
Chuû ñeà 2. Kyõ thuaät minh phaûn chöùng .................................................... 45
Chuû ñeà 3. Kyõ thuaät quy naïp toaùn hoïc ..................................................... 56
Chuû ñeà 4. Kyõ thuaät mieàn giaù trò ............................................................. 60
Chuû ñeà 5. Kyõ thuaät söû duïng nguyeân lí Diricle ....................................... 68
Chuû ñeà 6. Kyõ thuaät tam thöùc baäc hai ..................................................... 73
Chuû ñeà 7. Kyõ thuaät ñaùnh giaù baát ñaúng thöùc tích phaân ......................... 93

n
Chöông 2: Baát ñaúng thöùc vaø phöông phaùp tieáp caän

.v
Chuû ñeà 1. Caùc kyõ thuaät söû suïng baát ñaúng thöùc AM-GM cô baûn .......... 102
Chuû ñeà 2. Kyõ thuaät gheùp caëp trong chöùng minh ñaúng thöùc AM-GM ............. 198

om
Chuû ñeà 3. Kyõ thuaät söû duïng baát ñaúng thöùc AM-GM daïng coäng maãu soá ........211
Chuû ñeà 4. Kyõ thuaät söû duïng baát ñaúng thöùc Cauchy-Schwarz .............. 218
Chuû ñeà 5. Kyõ thuaät söû duïng baát ñaúng thöùc Cauchy-Schwarz daïng
.c
phaân thöùc ................................................................................ 243
Chuû ñeà 6. Kyõ thuaät tham soá hoùa ........................................................... 278
ok
Chuû ñeà 7. Baát ñaúng thöùc Holder vaø öùng duïng ...................................... 291
Chuû ñeà 8. Kyõ thuaät söû duïng baát ñaúng thöùc Chebyshev ....................... 304
bo

Chuû ñeà 9. Baát ñaúng thöùc Bernoulli vaø öùng duïng .................................. 314
Chöông 3: Phöông trình haøm soá trong giaûi toaùn baát ñaúng thöùc vaø
cöïc trò
et

Chuû ñeà 1. Kyõ thuaät söû duïng tính ñôn ñieäu vôùi baøi toaùn cöïc trò vaø baát
ñaúng thöùc moät bieán soá ......................................................... 325
vi

Chuû ñeà 2. Kyõ thuaät söû duïng tính ñôn ñieäu cho baøi toaùn cöïc trò vaø baát
ñaúng thöùc hai bieán soá .......................................................... 351
ng

Chuû ñeà 3. Kyõ thuaät söû duïng tính ñôn ñieäu cho baøi toaùn cöïc trò vaø baát
ñaúng thöùc ba bieán soá............................................................ 379
a

Chuû ñeà 4. Kyõ thuaät söû duïng tính thuaàn nhaát....................................... 427
kh

Chuû ñeà 5. Kyõ thuaät söû duïng baát ñaúng thöùc tieáp tuyeán ........................ 484
Chuû ñeà 6. Kyõ thuaät khaûo saùt haøm nhieàu bieán...................................... 502
Chuû ñeà 7. Kyõ thuaät söû duïng tính chaát cuûa nhò thöùc baäc nhaát vaø tam
thöùc baäc hai .......................................................................... 534
Chuû ñeà 8. Baát ñaúng thöùc phuï ñaâng chuù yù vaø aùp duïng giaûi ñeà thi tuyeån sinh .. 540
Chuû ñeà 9. Baøi toaùn choïn loïc baát ñaúng thöùc vaø cöïc trò ba bieán ............ 617
Chöông 4: Soá phöông phaùp chöùng minh baát ñaúng thöùc khaùc
Chuû ñeà 1. Kyõ thuaät löôïng giaùc hoùa ....................................................... 654
Chuû ñeà 2. Kyõ thuaät söû duïng baát ñaúng thöùc Schur ................................ 684
Chuû ñeà 3. Kyõ thuaät doàn bieán ................................................................. 694
Cty TNHH MTV DVVH Khang Việt

Chương 1:
BẤT ĐẲNG THỨC VÀ CÁC KỸ THUẬT CƠ BẢN
KIẾN THỨC CƠ BẢN VỀ BẤT ĐẲNG THỨC
I. Định nghĩa bất đẳng thức
Giả sử A và B là hai biểu thức bằng chữ hoặc bằng số.
+ A ≥ B (hoặc B ≤ A) , A ≤ B (hoặc B ≥ A) được gọi là các bất đẳng thức.
+ A ≥ B ⇔ A − B ≥ 0; A − B ≥ 0 ⇔ A ≥ B.
+ Một bất đẳng thức có thể đúng hoặc sai và ta quy ước khi nói về một bất đẳng

n
thức mà không nói gì thêm thì ta hiểu đó là bất đẳng thức đúng.

.v
II. Tính chất cơ bản của bất đẳng thức
 ∀a ∈ ; a ≥ a .

om
a ≤ b
  ⇒a≤c.
b ≤ c .c
 ∀a, b, m ∈ ; a ≤ b ⇒ a ± m ≤ b ± m .
a ≤ b
ok
  ⇒a+c≤b+d .
c ≤ d
bo

 a ≥b+c ⇔ a−c≥b.
ma ≤ mb khi m >0
 ∀a, b,∈ ; a ≤ b ⇔  .
ma ≥ mb khi m < 0
et

a b
 m ≤ m khi m >0
vi

+
 ∀a, b,∈  ; a ≤ b ⇔  .
ng

 a ≥ b khi m < 0
 m m
1 1
a

 Nếu a > b > 0 ⇒ < .


a b
kh

a ≥ c
 ∀a, b, c, d ∈  + ;  ⇒ ab ≥ cd .
b ≥ d
a n ≥ b n
 a≥b≥0⇒ , ∀n ∈  .
 n a ≥ n b
a > 1 ⇒ a x > a y
 x > y > 0;  .
0 < a < 1 ⇒ a x < a y

 a > b ⇒ a 2 n +1 > b 2 n +1; 2 n +1 a > 2 n +1 b , ∀n ∈  .


3
Khám phá tư duy Kỹ thuật giải bất ĐT Bài toán Max – Min – Đặng Thành Nam

1. Bất đẳng thức về giá trị tuyệt đối


 − a ≤ a ≤ a , ∀a ∈  .
 a < α ⇔ −α < a < α ( khi α > 0 ) .
a > α
 a >α ⇔  ( khi α > 0 ) .
 a < −α
 a − b ≤ a + b ≤ a + b , ( ∀a, b ∈  ) .
2. Bất đẳng thức liên quan đến hàm số mũ và logarit
a > 1 0 < a < 1
  ⇒ ax > a y ; ⇒ ax < a y .

n
 x > y  x > y

.v
a > 1 0 < a < 1
  ⇒ log a x > log a y;  ⇒ log a x < log a y .
x > y > 0 x > y > 0

om
3. Bất đẳng thức AM – GM
Cho n số thực không âm a1 , a2 ,..., an ta có
a1 + a2 + ... + an n
≥ a1a2 ...an .
.c
n
ok
Đẳng thức xảy ra khi và chỉ khi a1= a2= ...= an .
4. Bất đẳng thức Cauchy – Schwarz
bo

Cho 2 dãy số thực ( a1 , a2 ,..., an ) ; ( b1 , b2 ,..., bn ) ta có

( a1b1 + a2b2 + ... + anbn )2 ≤ ( a12 + a22 + ... + an2 )( b12 + b22 + ... + bn2 ) .
et

Đẳng thức xảy ra khi và chỉ khi= =


ai kbi , i 1, n, k ∈ .
vi
ng

CHỦ ĐỀ 1: KỸ THUẬT BIẾN ĐỔI TƯƠNG ĐƯƠNG


Xuất phát từ bất đẳng thức cơ bản và hết sức tự nhiên x 2 ≥ 0; A − B ≥ 0 với mọi
a

số thực x ta có các bất đẳng thức hết sức đẹp mắt. Nội dung chủ đề này đề cập đến
kh

kỹ năng biến đổi bất đẳng thức về dạng luôn đúng. Các bài toán đề cập đến là các
bài toán trong chủ đề này các bạn chú ý sẽ được sử dụng đến trong các chủ đề khác
ở các chương sau như một bài toán phụ.
A. NỘI DUNG PHƯƠNG PHÁP
I. Các bất đẳng thức cơ bản
Bình phương của một số thực
Với mọi số thực x ta luôn có x 2 ≥ 0. Đẳng thức xảy ra khi và chỉ khi x = 0 .
Từ đó ta có các bất đẳng thức với 2 biến và 3 biến thường sử dụng như sau:
 ( a − b )2 ≥ 0 hay a 2 + b 2 ≥ 2ab. Đẳng thức xảy ra khi và chỉ khi a = b .

4
Cty TNHH MTV DVVH Khang Việt

 ( a − b )2 + ( b − c )2 + ( c − a )2 ≥ 0 hay a 2 + b 2 + c 2 ≥ ab + bc + ca

(
hoặc ( a + b + c ) ≥ 3 ( ab + bc + ca ) hoặc 3 a 2 + b 2 + c 2 ≥ ( a + b + c ) .
2
) 2

Đẳng thức xảy ra khi và chỉ khi a= b= c .


Bất đẳng thức về trị tuyệt đối
 Với 2 số thực x,y ta luôn có x + y ≥ x + y . Đẳng thức xảy ra khi và chỉ khi
xy ≥ 0 .
 Với 2 số thực x,y ta luôn có x − y ≥ x − y . Đẳng thức xảy ra khi và chỉ khi
x( x − y) ≥ 0 .

n
Bất đẳng thức về độ dài cạnh của một tam giác

.v
 a + b > c; b + c > a; c + a > b .
 a > b − c ;b > c − a ;c > a − b .

om
 a 2 + b 2 + c 2 < 2 ( ab + bc + ca ) .
II. Một số hằng đẳng thức cần lưu ý .c
 a3 + b3 + c3 − 3abc = ( a + b + c ) ( a 2 + b2 + c 2 − ab − bc − ca ) .
ok
 ( a + b + c )3 = a3 + b3 + c3 + 3 ( a + b )( b + c )( c + a ) .
 ( a + b )( b + c )( c + a ) = ( a + b + c )( ab + bc + ca ) − abc .
bo


a −b b−c c−a
+ + = −
( a − b )( b − c )( c − a )
.
c a b abc
et

a −b b−c c−a a−b b−c c−a


 + + + . . =0.
a+b b+c a+c a+b b+c c+a
vi

1) Kỹ thuật biến dùng định nghĩa


ng

Để chứng minh bất đẳng thức: A ≥ B . Ta chứng minh bất đẳng thức
A − B ≥ 0 đúng.

(x )
a

2
2
+ y2
≥8.
kh

Ví dụ 1. Cho x > y và xy = 1. Chứng minh rằng


( x − y )2
Lời giải
Ta có x 2 + y 2 =( x − y ) + 2 xy =( x − y ) + 2
2 2
(vì xy = 1)

(x )
2
=( x − y ) + 4. ( x − y ) + 4 .
4 2
⇒ 2
+ y2
Do đó BĐT cần chứng minh tương đương với
( x − y )4 + 4 ( x − y )2 + 4 ≥ 8.( x − y )2

5
Khám phá tư duy Kỹ thuật giải bất ĐT Bài toán Max – Min – Đặng Thành Nam

⇔ ( x − y )4 − 4 ( x − y )2 + 4 ≥ 0
2
⇔ ( x − y ) 2 − 2  ≥ 0 .
 
Bất đẳng thức cuối đúng nên ta có điều phải chứng minh.
Ví dụ 2.
a) Cho x,y là hai số thực thoả mãn điều kiện xy ≥ 1 .
Chứng minh rằng
1 1 2
+ ≥ .
1+ x 2
1+ y 2 1 + xy

n
b) Cho a,b,c là các số thực không nhỏ hơn 1 chứng minh

.v
1 1 1 3
+ + ≥ .
1 + a 1 + b 1 + c 1 + abc
3 3 3

om
c) Cho x, y, z ∈ [ 0;1] . Tìm giá trị lớn nhất của biểu thức
 1 1 1 
P= (1 + xyz )  + + 3
.
.c
1+ x 1+ y 1+ z 
3 3
ok
Lời giải
a) Bất đẳng thức đã cho tương đương với:
bo

 1 1   1 1 
 − +
2 1 + xy  
− ≥0
2 1 + xy 
1+ x  1+ y 
et

xy − x 2 xy − y 2
⇔ + ≥0
(1 + x ).(1 + xy ) (1 + y ).(1 + xy )
2 2
vi

x( y − x) y( x − y)
ng

⇔ + ≥0
( )
1 + x . (1 + xy )
2
( )
1 + y 2 . (1 + xy )
a


( y − x )2 ( xy − 1) ≥0
(1 + x2 ).(1 + y 2 ).(1 + xy )
kh

BĐT cuối này đúng do xy ≥ 1. Vậy ta có đpcm. Đẳng thức xảy ra khi và chỉ khi
x = y hoặc xy = 1 .
b) Sử dụng bất đẳng thức:
1 1 2
+ ≥ , ( xy ≥ 1) .
1+ x 2
1+ y 2 1 + xy
1 1 2
Ta có + ≥
1+ a 3
1+ b 3
1 + a 3b 3
6
Cty TNHH MTV DVVH Khang Việt

1 1 2
+ ≥
1+ c 3 1 + abc 1 + abc 4
.
 1 1  2 4
2 +  ≥ 2. =
  1 + abc
1+ a b 1 + abc 4
3 3
 1+ a3b3 . abc 4
Cộng theo vế ba bất đẳng thức trên ta có đpcm.
Chú ý. Bất đẳng thức này được áp dụng khá phổ biến trong một số bài toán cực trị.
Một số dạng tương tự bất đẳng thức trên như sau
1 1 2
 + ≤ , ( −1 < xy ≤ 1) .
1+ x 2
1+ y 2 1 + xy

n
.v
1 1 2
 + ≥ , ( xy ≥ 1) .
1+ x 2
1+ y 2 1 + xy

om
1 1 2
 + ≤ , ( −1 < xy ≤ 1) .
1 + x2 1 + y2 1 + xy

c) Sử dụng kết quả bài toán trên ta có :


1
+
.c1

2
1+ x 3
1+ y 3
ok
1 + x3 y 3
1 1 2
+ ≤
bo

1+ z 3 1 + xyz 1 + xyz 4

2 2 4 4
+ ≤ =
et

1 + x3 y 3 1 + xyz 4 1 + xyz
1+ x4 y 4 z 4
vi

Cộng theo vế các bất đẳng thức trên ta suy ra


 1 1 
ng

1 1 1 3 1
+ + ≤ (1 + xyz ) 
⇒P= + + ≤3
3
1+ x 3
1+ y 3
1+ z 3 1 + xyz 1+ x 1+ y 1+ z 
3 3
a

Đẳng thức xảy ra khi và chỉ khi x= y= z


kh

Vậy giá trị lớn nhất của P = 3 .


Ví dụ 3. Chứng minh rằng với mọi số thực x,y ta có
3 1 + ( x + y ) 
2

(1 + x )(2
)
1 + y2 ≥ 
4
.

Lời giải
( 2 xy − 1) + ( x − y ) ≥ 1 + x + y 2 . 2 2
4
3
( )(
Chú ý: 1 + x 2 1 + y 2 =1 + ( x + y ) +
2
3
) ( )

7
Khám phá tư duy Kỹ thuật giải bất ĐT Bài toán Max – Min – Đặng Thành Nam

1
Bất đẳng thức được chứng minh. Đẳng thức xảy ra khi và chỉ khi x = y = ±
.
2
Ví dụ 4. Chứng minh rằng với mọi số thực a,b không âm thoả mãn a, b < 1;
3
a + b ≥ , ta có
(1 − 2a )(1 − 2b ) ≥ 4  1 − a − b 2 .
 
2 (1 − a )(1 − b )  2−a −b
Lời giải
Bất đẳng thức đã cho tương đương với:
( a − b )2 ( 2a + 2b − 3) ≥ 0 .
(1 − a )(1 − b )( 2 − a − b )2

n
Bất đẳng thức luôn đúng và ta có đpcm.

.v
om
Bài tập tương tự
1
Chứng minh rằng với mọi số thực a,b không âm thoả mãn a, b < 1; a + b ≥ ta có
.c 2
(1 + 2a )(1 + 2b ) ≥ 4  1 + a + b 2 .
 
(1 − a )(1 − b )
ok
 2−a −b
bo

2) Kỹ thuật phân tích hằng đẳng thức


n
∑ ( xi − yi )
2
Phân tích thành tổng các bình phương ≥0.
et

i =1
Ví dụ 1. Cho a,b,c là các số thực bất kỳ chứng minh
vi

a) a 2 + b 2 + c 2 ≥ ab + bc + ca .
ng

b) ( ab + bc + ca ) ≥ 3abc ( a + b + c ) .
2

c) ( a + b + c ) −
2 1
( )
( b − c )2 + ( c − a )2 + ( a − b )2 ≥ 3 ( ab + bc + ca ) .
a

4
kh

( )( )( )
d) a 2 + 2 b 2 + 2 c 2 + 2 ≥ 3 ( a + b + c ) .
2

Lời giải
a) Bất đẳng thức đã cho tương đương với:
2a 2 + 2b 2 + 2c 2 − 2ab − 2bc − 2ca ≥ 0

( ) ( ) (
⇔ a 2 − 2ab + b 2 + b 2 − 2bc + c 2 + c 2 − 2ca + a 2 ≥ 0 . )
⇔ ( a − b) + (b − c ) + (c − a ) ≥ 0
2 2 2

Bất đẳng thức cuối đúng. Đẳng thức xảy ra khi và chỉ khi a= b= c .

8
Cty TNHH MTV DVVH Khang Việt

b) Thực hiện tương tự câu a) đưa về bất đẳng thức luôn đúng
( ab − bc )2 + ( bc − ca )2 + ( ca − ab )2 ≥ 0 .
c) Ta có:
1 1 1
( a + b + c )2 − ( b − c= )2 ( 2a − b − c )2 + ab + bc + ca ≥ ab + bc + ca .
3 4 12
Tương tự ta có:
1 1
( a + b + c )2 − ( c − a )2 ≥ ab + bc + ca
3 4
.
1 1
( a + b + c ) 2
− ( a − b ) 2
≥ ab + bc + ca
3 4

n
Cộng lại theo vế ba bất đẳng thức trên ta có đpcm.

.v
Đẳng thức xảy ra khi và chỉ khi a= b= c .
d) Chú ý đẳng thức:

om
(a 2
)( )( )
+ 2 b2 + 2 c2 + 2 − 3( a + b + c )
2

=
1 2
2
( )

2 2

3
c + 2 ( a − b ) + 2 ( ab − 1)  + ( ac + bc − 2 ) .
2
2 .c
⇒ ( a + 2 )( b )( )
+ 2 c2 + 2 ≥ 3( a + b + c )
2 2 2
ok
Ta có điều phải chứng minh. Đẳng thức xảy ra khi và chỉ khi a= b= c= 1 .
bo

Ví dụ 2. Cho x, y, z là các số thực thỏa mãn điều kiện x 2 + y 2 + z 2 =


1.
Chứng minh rằng
et

1
a) − ≤ xy + yz + zx ≤ 1 ;
2
vi

8
b) ( xy + yz + 2 xz ) −
2
≥ −3.
( x + y + z ) − xy − yz + 2
2
ng

Lời giải
a) Bất đẳng thức vế trái tương đương với:
a

2 ( xy + yz + zx ) + 1 ≥ 0 ⇔ 2 ( xy + yz + zx ) + x 2 + y 2 + z 2 ≥ 0 ⇔ ( x + y + z ) ≥ 0 .
2
kh

Bất đẳng thức được chứng minh.


 x + y + z =0 1 1
Đẳng thức xảy ra khi và chỉ khi  2 2 2 , chẳng hạn tại x =
− ,y= .
 x + y + z = 1 2 2
Bất đẳng thức vế phải:
1 − ( xy + yz + zx ) = x 2 + y 2 + z 2 − ( xy + yz + zx )

=
1
2
( )
( x − y )2 + ( y − z )2 + ( z − x )2 ≥ 0
⇒ xy + yz + zx ≤ 1

9
Khám phá tư duy Kỹ thuật giải bất ĐT Bài toán Max – Min – Đặng Thành Nam

b) Chú ý điều kiện ta rút gọn vế trái và đưa về chứng minh


8
( xy + yz + 2 xz )2 − ≥ −3
xy + yz + 2 zx + 3
.

( xy + yz + 2 zx + 1)
3
≥0
xy + yz + 2 zx + 3
Vậy ta chỉ cần chứng minh
xy + yz + 2 zx ≥ −1 = − x 2 − y 2 − z 2
2
 1  3
⇔ ( x + z ) + y + y ( x + z ) ≥ 0 ⇔  x + z + y  + y2 ≥ 0
2 2
 2  4

n
Bất đẳng thức cuối đúng và ta có đpcm.

.v
y = 0

om
 1 1 1
Đẳng thức xảy ra khi và chỉ khi  x + z + y =0 ⇒ x = , y =0, z =− .
 2 2 2
 x 2 + y 2 + z 2 =
1 .c
Ví dụ 3. Cho x,y,z là các số thực không âm. Chứng minh:
ok
a) x3 + y 3 + z 3 ≥ 3 xyz .
x3 + y 3 + z 3 3
≥ xyz + ( x − y )( y − z )( z − x ) .
bo

b)
3 4
3
 y+z 
c) x3 + y 3 + z 3 − 3 xyz ≥ 2  − x .
et

 2 
vi

Lời giải
a) Bất đẳng thức đã cho tương đương với:
ng

( x + y + z ) ( x 2 + y 2 + z 2 − xy − yz − zx ) ≥ 0
.
1
⇔ ( x + y + z ) ( x − y )2 + ( y − z )2 + ( z − x )2  ≥ 0
a

2
kh

Bất đẳng thức được chứng minh. Đẳng thức xảy ra khi và chỉ khi x= y= z .
b) Bất đẳng thức đã cho tương đương với:
1 3
( x + y + z ) ( x − y )2 + ( y − z )2 + ( z − x )2  ≥ ( x − y )( y − z )( z − x )
6 4 .
⇔ ( ( x + y ) + ( y + z ) + ( z + x ) ) ( x − y ) + ( y − z ) + ( z − x ) ≥ 9 ( x − y )( y − z )( z − x )
 2 2 2
 
Chú ý x + y ≥ x − y ; y + z ≥ y − z ; z + x ≥ z − x và sử dụng bất đẳng thức đã
chứng minh được ở câu a) ta có

10
Cty TNHH MTV DVVH Khang Việt

( x + y) + ( y + z ) + ( z + x) ≥ x − y + y − z + z − x ≥ 33 ( x − y )( y − z )( z − x )
.
( x − y )2 + ( y − z )2 + ( z − x )2 ≥ 3 3 ( x − y )2 ( y − z )2 ( z − x )2
Nhân theo vế hai bất đẳng thức trên ta có điều phải chứng minh. Đẳng thức xảy
ra khi và chỉ khi x= y= z .
y+z
c) Theo câu a) ta có x3 + y 3 + z 3 − 3 xyz ≥ 0, do đó nếu − x ≤ 0 bất đẳng thức
2
luôn đúng.
+ Ngược lại xét y + z − 2 x > 0 ⇔ ( y − x ) + ( z − x ) > 0 .

n
Đặt y =2a + x, z =2b + x bất đẳng thức trở thành

( )

.v
12 x a 2 − ab + b 2 + 6 ( a + b )( a − b ) ≥ 0.
2

om
y+z
Bất đẳng thức đúng vì a +=
b −x>0.
2
Đẳng thức xảy ra khi và chỉ khi a= b= c hoặc=
b c=
,a 0 .
Bài tập tương tự
.c
1 1
ok
Cho a,b là hai số thực khác 0 thoả mãn điều kiện ab ≥ + +3.
a b
3
 1 1 
bo

Chứng minh rằng ab ≥  3 + 3  .


 a b
Ví dụ 4. Cho x,y,z là các số thực dương chứng minh
et

3( x 2 + xy + y 2 )( y 2 + yz + z 2 )( z 2 + zx + x 2 ) ≥ ( x + y + z )2 ( xy + yz + zx) 2 .
vi

Lời giải
3 1 3
Chú ý x 2 + xy + y 2 = ( x + y )2 + ( x − y )2 ⇒ x 2 + xy + y 2 ≥ ( x + y )2 .
ng

4 4 4
3 3
y 2 + yz + z 2 ≥ ( y + z ) ; z 2 + zx + x 2 ≥ ( z + x )
2 2
a

4 4
kh

27
Do đó ( x 2 + xy + y 2 )( y 2 + yz + z 2 )( z 2 + zx + x 2 ) ≥ ( x + y ) ( y + z ) ( z + x ) .
2 2 2
64
Ta chỉ cần chứng minh
64
[( x + y )( y + z )( z + x)]2 ≥ [( x + y + z )( xy + yz + zx)]2
81
8
⇔ ( x + y )( y + z )( z + x ) ≥ ( x + y + z )( xy + yz + zx)
9
⇔ x( y − z )2 + y ( z − x)2 + z ( x − y )2 ≥ 0
Bất đẳng thức cuối đúng. Ta có điều phải chứng minh.

11
Khám phá tư duy Kỹ thuật giải bất ĐT Bài toán Max – Min – Đặng Thành Nam

Ví dụ 5. Cho a,b,c là các số thực dương thỏa mãn điều kiện 0 < a ≤ b ≤ c ≤ 1 .
Tìm giá trị lớn nhất của biểu thức P= ( )
a + b c − (a + b) c .
Lời giải
Ta có
P= ( )
a + b c − ( a + b ) c= c ( )
ac + bc − a − b ≤ ac + bc − a − b
2 2
 1  1 1 1
≤ a + b −a −b =
− a −  −  b −  + ≤
 2  2 2 2
1
Đẳng thức xảy ra khi và chỉ khi c= 1, a= b= .

n
2

.v
3) Kỹ thuật thêm bớt hằng số
Việc cộng hoặc trừ hai vế của bất đẳng thức cho một số nào đó làm lược bỏ đi

om
phần phức tạp của bất đẳng thức.
Ví dụ 1. Cho x,y,z là các số thực không âm thoả mãn điều kiện x ≥ y ≥ z. Chứng
minh rằng .c
xy + yz + zx x + z
a) 2 ≥ .
y + yz + z 2 y + z
ok

b)
xy + yz + zx

( x + z )( y + z ) .
bo

x + xy + y
2 2
( x + z ) + ( x + z )( y + z ) + ( y + z )2
2

Lời giải
et

a) Bất đẳng thức đã cho tương đương với:


xy + yz + zx y 2 + yz + z 2
vi


x+z y+z
ng

xy + yz + zx y 2 + yz + z 2
⇔ −y≥ −y .
x+z y+z
a


zx

z2
⇔ z  x ( y + z ) − z ( x + z )  ≥ 0 ⇔ z xy − z 2 ≥ 0 ( )
kh

x+z y+z
Bất đẳng thức cuối đúng. Đẳng thức xảy ra khi và chỉ khi x= y= z hoặc
=
z 0,=x y.
b) Bất đẳng thức đã cho tương đương với:
( x + z )2 + ( x + z )( y + z ) + ( y + z )2 − 1 ≥ ( x + z )( y + z ) − 1
x 2 + xy + y 2 xy + yz + zx
3z ( x + y + z )

x 2 + xy + y 2

z2
xy + yz + zx  (  )
⇔ z 3 ( x + y + z )( xy + yz + zx ) − z x 2 + xy + y 2  ≥ 0

12
Cty TNHH MTV DVVH Khang Việt

Bất đẳng thức cuối đúng vì


3 ( x + y + z )( xy + yz + zx ) ≥ 3 ( x + y )  z ( x + = (
y )  3z ( x + y ) ≥ z x 2 + xy + y 2 .
2
)
Bất đẳng thức được chứng minh. Đẳng thức xảy ra khi và chỉ khi z = 0 .
4) Kỹ thuật biến đổi với bất đẳng thức chứa căn
+ Phép bình phương hai vế được ưu tiên.
+ Cần chứng minh A1 + A2 + ... + An ≥ b1 + b2 + ... + bn .

Ta có để chứng minh A1 = b12 + c12 ≥ b12 = b1 .


Rồi cộng lại theo vế các bất đẳng thức trên ta có đpcm.

n
Ví dụ 1. Chứng minh với mọi số thực x,y cùng dấu và số thực k, ta có

.v
k2 + x + k2 + y ≥ k + k2 + x + y .

om
Lời giải
Bất đẳng thức đã cho tương đương với
k 2 + x + k 2 + y + 2 (k 2 + x)(k 2 + y ) ≥ 2k 2 + x + y + 2k k 2 + x + y
.c .
⇔ k + k ( x + y ) + xy ≥ k k + x + y ⇔ xy ≥ 0
4 2 2
ok
Bất đẳng thức cuối đúng ta có đpcm. Đẳng thức xảy ra khi và chỉ khi hoặc x
bằng 0 hoặc y bằng 0.
bo

Bài 2. Chứng minh rằng với x,y là hai số thực không âm thỏa mãn x + y ≥ 1, ta luôn

có x2 + x + 4 + y 2 + y + 4 ≤ 2 + ( x + y )2 + x + y + 4 .
et

Lời giải
Bất đẳng thức đã cho tương đương với
vi

x2 + y 2 + x + y + 8 + 2 (x 2
)(
+ x + 4 y2 + y + 4 ≤ )
ng

4+4 ( x + y )2 + x + y + 4 + ( x + y )2 + x + y + 4 .
a

⇔ (x 2
)(
+ x + 4 y 2 + y + 4 ≤ xy + 2 ) ( x + y )2 + x + y + 4 .
kh

( )( )
⇔ x 2 + x + 4 y 2 + y + 4 ≤ x 2 y 2 + 4 xy ( x + y )2 + x + y + 4 + 4 ( x + y )2 + x + y + 4 .

⇔ xy  4 ( x + y ) + x + y + 4 + x + y − 7  ≥ 0 (luôn đúng do x + y ≥ 1 ).
2
 
Tổng quát. Tương tự ta có các bất đẳng thức cùng dạng sau
+ Với mọi số thực không âm x,y ta luôn có
x2 + x + k 2 + y 2 + y + k 2 ≤ k + ( x + y )2 + x + y + k 2 .

13
Khám phá tư duy Kỹ thuật giải bất ĐT Bài toán Max – Min – Đặng Thành Nam

Đẳng thức xảy ra khi và chỉ khi x = 0 hoặc y = 0 .


+ Với mọi số thực không âm ta luôn có
x2 − x + 1 + y 2 − y + 1 ≤ 1 + ( x + y )2 − x − y + 1 .
Đẳng thức xảy ra khi và chỉ khi x = 0 hoặc y = 0 .
+ 1 + a + 1 + b ≥ 1 + 1 + a + b , ( ab ≥ 0; a, b ≥ −1; a + b ≥ −1) .
5) Kỹ thuật đánh giá phân thức
1 1
Sử dụng đánh giá cơ bản: A > B > 0 ⇒ < .
A B
Ví dụ 1. Chứng minh rằng với mọi a,b,c dương ta có

n
a b c

.v
1< + + <2.
a+b b+c a+c

om
Lời giải
1 1 a a
Ta có : a + b < a + b + c ⇒ > ⇒ > (1)
a+b a+b+c a+b a+b+c

Tương tự ta có :
b
>
b
(2) ,
.c
c
>
c
(3)
b+c a+b+c a+c a+b+c
ok
Cộng vế theo vế các bất đẳng thức (1), (2), (3), ta được :
a b c
+ + > 1 (*)
bo

a+b b+c a+c


a a+c
Ta có : a < a + b ⇒ < (4)
et

a+b a+b+c
b a+b c c+b
Tương tự : < (5) , < (6)
vi

b+c a+b+c c+a a+b+c


Cộng vế theo vế các bất đẳng thức (4), (5), (6), ta được :
ng

a b c
+ + < 2 (**)
a+b b+c a+c
a

a b c
Từ (*) và (**) , ta được : 1 < + + < 2 (đpcm)
kh

a+b b+c a+c


Ví dụ 2. Cho a,b,c là các số thực không âm thoả mãn điều kiện
a b c
+ + = 3.
1 + bc 1 + ca 1 + ab
a b c 3
Chứng minh rằng + + ≥ .
1 + a + bc 1 + b + ca 1 + c + ab 4
Lời giải
a b c
Đặt=x =
;y =
;z ⇒ x + y += z 3.
1 + bc 1 + ca 1 + ab

14
Cty TNHH MTV DVVH Khang Việt

x y z 3
Ta cần chứng minh + + ≥ .
1+ x 1+ y 1+ z 4
x x
Chú ý ≥ .
1+ x 1+ x + y + z
y y

1+ y 1+ x + y + z
z z

1+ z 1+ x + y + z
Cộng theo vế ba bất đẳng thức trên ta có điều phải chứng minh.

n
Đẳng thức xảy ra khi và chỉ khi có một số bằng 3 và hai số bằng 0.

.v
Ví dụ 3. Cho a,b,c là các số thực không âm có tổng bằng 1. Chứng minh
1 + a2 1 + b2 1 + c2

om
7
+ + ≤ .
1+ b
1+ c 2
1+ a 2 2 2
Lời giải
Ta thấy dấu bằng đạt tại khi một số bằng 1 và hai số bằng 0.
.c
Vậy giả sử a = max {a, b, c}. Khi đó ta mạnh dạn đánh giá 1 + b 2 ≥ 1;1 + c 2 ≥ 1 .
ok
1 + a2
Ta có 1 + b 2 ≥ 1 ⇒ ≤ 1 + a2 ; .
bo

1 + b2
1 + b2
1 + c2 ≥ 1 ⇒ ≤ 1 + b2
et

1+ c 2

1 + c2 1
vi

Suy ra P ≤ 2 + a 2 + b 2 + ≤ 2 + a 2 + b2 + c2 + .
1+ a 2
1 + a2
ng

1 1
≤ 2 + a2 + (b + c ) + =2 + a 2 + (1 − a ) +
2 2
1+ a 2
1 + a2
a

1 7
Ta chỉ cần chứng minh 2 + a 2 + (1 − a ) +
2
≤ .
kh

1+ a 2 2
(
⇔ ( a − 1) 4a + 3a − 1 ≤ 0
3
)
Bất đẳng thức cuối luôn đúng nên ta có đpcm. Đẳng thức xảy ra khi và chỉ khi
a= 1, b= c= 0 hoặc các hoán vị.
1 + ak 1 + bk 1 + ck 7
Chú ý. Bằng cách tương tự ta chứng minh được + + ≤ .
1+ b k
1+ c k
1+ a k 2
Với k là số nguyên dương.

15
Khám phá tư duy Kỹ thuật giải bất ĐT Bài toán Max – Min – Đặng Thành Nam

Ví dụ 4. Cho x,y,z là các số thực thoả mãn điều kiện x, y ≥ −1; x + y + z =3 .


x2 y2 −1
=
Tìm giá trị lớn nhất của biểu thức P + .
x 2 + y 2 + 4 ( xy + 1) z2 − 4z + 5
Lời giải
Trước hết đánh gia hai mẫu số ở hai phân thức bằng cách thay z = 3 − x − y .
Ta chứng minh x 2 + y 2 + 4 ( xy + 1) ≥ z 2 − 4 z + 5 .
⇔ x 2 + y 2 + 4 xy − ( 3 − x − y ) + 4 ( 3 − x − y ) − 1 ≥ 0
2

⇔ 2 ( x + y ) + 2 xy + 2 ≥ 0 ⇔ ( x + 1)( y + 1) ≥ 0

n
Bất đẳng thức đúng.

.v
Vậy ta có P ≤
x2 + y 2 − 1 (
=
x + y ) − 2 xy − 1
2
.

om
z2 − 4z + 5 z2 − 4z + 5
Chú ý. xy ≥ −1 − x − y = z − 4; x + y = 3 − z .

Khi đó P≤
( 3 − z ) − 2 ( z − 4 ) − 1 z 2 − 8 z + 16
2
=
.c =−
( 2 z − 3)
2
+5≤5.
z2 − 4z + 5 z2 − 4z + 5 z2 − 4z + 5
ok
 3
 3  x+ y=
z = 2 
2 
x=
5
−1, y =, z =
3

bo

  5 2 2.
Dấu bằng đạt tại  x + y + z =3 ⇔  xy =− ⇔ 
 xy= z − 4  2  x =, y =
5 3
−1, z =
  3 

et

2 2
 z = 2

vi

6) Kỹ thuật đánh giá bất đẳng thức có chứa dấu giá trị tuyệt đối
Sử dụng hai bất đẳng thức quen thuộc: x + y ≥ x + y ; x − y ≤ x − y .
ng

Chú ý. Tư duy đầu tiên là khử dấu giá trị tuyệt đối muốn vậy ta xét trường hợp.
Ví dụ 1. Chứng minh với mọi số thực a,b,c ta có
a

a + b + c + a+b+c ≥ a+b + b+c + c+a .


kh

Lời giải
Trong ba số a,b,c có ít nhất hai số cùng dấu không mất tính tổng quát giả sử là a
và b khi đó a + b = a + b .
Vậy ta chỉ cần chứng minh c + a + b + c ≥ b + c + c + a

⇔ c 2 + ( a + b + c ) + 2 c ( c + a + b ) ≥ ( a + c ) + ( b + c ) + 2 ( a + c )( b + c )
2 2 2

⇔ ab + c ( c + a + b ) ≥ ( a + c )( b + c )= c ( c + a + b ) + ab
Bất đẳng thức cuối luôn đúng (đpcm).

16
Cty TNHH MTV DVVH Khang Việt

Ví dụ 2. Cho x,y,z là các số thực đôi một không đồng thời bằng 0. Chứng minh

−1 ≤
(x 2
− y2 )( y 2
− z2 )( z 2
− x2 ) ≤1.
(x 2
+ y2 )( y 2
+ z2 )( z 2
+ x2 )
Lời giải

( ) ≤ (x )
2 2
Ta có x 2 − y 2 ≤ x 2 + y 2 ⇔ x 2 − y 2 2
+ y2 ⇔ 4x2 y 2 ≥ 0 .

Từ đó suy ra
(x 2
− y2 )( y 2
− z2 )( z 2
) ≤1
− x2

(x 2
+ y2 )( y 2
+ z2 )( z 2
+x )2

n
(x 2
− y2 )( y 2
− z2 )( z − x ) ≤ 1
2 2

.v
⇔ −1 ≤
(x 2
+ y2 )( y 2
+ z2 )( z + x )
2 2

om
Bất đẳng thức được chứng minh.
Ví dụ 3. Cho a,b,c là các số thực không âm chứng minh
3 3 abc + a − b + b − c + c − a ≥ a + b + c.
.c
Lời giải
ok
Không mất tính tổng quát giả sử a ≥ b ≥ c. Khi đó bất đẳng thức tương đương với:
3 3 abc + (a − b) + (b − c) + (c − a) ≥ a + b + c
bo

⇔ a − b − 3c + 3 3 abc ≥ 0 ⇔ (a − b) + 3 3 c ( 3
)
ab − c 2 ≥ 0
3
et

Bất đẳng thức cuối luôn đúng và ta có đpcm.


Bài tập tương tự
vi

Cho a,b,c là các số thực dương thoả mãn điều kiện


ng

3 3 abc + a − b + b − c + c − a =
1.
1
Chứng minh rằng a bc + b ca + c ab ≤ .
a

3
kh

Ví dụ 4. Cho x,y,z là các số thực chứng minh


x − y + y − z + z − x ≥ 2 x 2 + y 2 + z 2 − xy − yz − zx .
Lời giải
Không mất tính tổng quát giả sử x ≥ y ≥ z. Bất đẳng thức trở thành
1 1 1
( x − y) + ( y − z) + ( x − z) ≥ 2 ( x − y )2 + ( y − z )2 + ( z − x )2
2 2 2
⇔ 2 ( x − z ) ≥ 2 ( x − y ) + ( y − z ) + ( z − x ) 
2 2 2
 

17
Khám phá tư duy Kỹ thuật giải bất ĐT Bài toán Max – Min – Đặng Thành Nam

⇔ 4 ( x − z ) ≥ 2 ( x − y ) + ( y − z ) + ( z − x ) 
2 2 2 2
 
⇔ ( x − z) ≥ ( x − y) + ( y − z)
2 2 2

⇔ [ ( x − y ) + ( y − z ) ] ≥ ( x − y ) + ( y − z ) ⇔ 2 ( x − y )( y − z ) ≥ 0
2 2 2

Bất đẳng thức cuối luôn đúng ta có đpcm.


Ví dụ 5. Cho n số thực x1 , x2 ,..., xn (với n ≥ 3 ). Chứng minh
x1 + x2 + ... + x n x1 − x2 + x2 − x3 + ... + xn −1 − xn + xn − x1
max { x1 , x2 ,..., xn } ≥ + .
n 2n
Lời giải

n
Chú ý. Với hai số thực x,y bất kỳ ta luôn có

.v
x+ y+ x− y
min { x, y} ≤ x, y ≤ max { x, y} và max { x, y} = .

om
2
x+ y+ x− y
Sử dụng max { x, y} = ta được:
2 .c
x1 + x2 + ... + x n x1 − x2 + x2 − x3 + ... + xn −1 − xn + xn − x1
+
ok
n 2n
x1 + x2 + x1 − x2 x2 + x3 + x2 − x3 x + x + xn − x1
+ + ... + n 1
bo

2n 2n 2n
max { x1 , x2 } + max { x2 , x3 } + ... + max { xn −1 , xn } + max { xn , x1}
≤ max { x1 , x2 ,..., xn }
n
et

Bài toán được chứng minh. Dấu bằng đạt tại chẳng hạn x1= x2= ...= x n .
vi

7) Kỹ thuật đặt ẩn phụ


Với bất đẳng thức đối xứng hai biến ta có thể đặt u =a + b; v =ab .
ng

Với phân thức ta có để đặt các mẫu số là các biến mới.


Ví dụ 1. Chứng minh rằng với mọi a,b dương, ta có
a

( )
a 2b 2 a 2 + b 2 − 2 ≥ ( a + b )( ab − 1) .
kh

Lời giải
Đặt a + = = v , v > 0. Khi đó bất đẳng thức tương đương với:
b 2u , ab 2

a 2b 2 (a 2 + b 2 − 2) ≥ (a + b)(ab − 1)
⇔ a 2b 2 ( a + b ) − 2ab − 2  ≥ ( a + b )( ab − 1)
2
 
⇔ v 4 (4u 2 − 2v 2 − 2) ≥ 2u (v 2 − 1)
⇔ 2v 4u 2 − (v 2 − 1)u − v 4 (v 2 + 1) ≥ 0

18
Cty TNHH MTV DVVH Khang Việt

v 2 − 1 + (v 2 − 1) 2 + 8v8 (v 2 + 1)
Điều này chứng tỏ u ≥ .
4v 4
a+b
Mặt khác u = ≥ ab = v do đó ta chỉ cần chứng minh:
2
v 2 − 1 + (v 2 − 1) 2 + 8v8 (v 2 + 1)
v≥ 4
⇔ (v − 1) 2 (v + 1)(v 2 + v + 1) ≥ 0
4v
Bất đẳng thức cuối đúng. Đẳng thức xảy ra khi và chỉ khi a= b= 1 .
Ví dụ 2. Cho a,b,c là các số thực dương chứng minh rằng
x2 − z 2 y 2 − x2 z 2 − y 2

n
+ + ≥ 0.
y+z z+x x+ y

.v
Lời giải

om
Đặt a = z x khi đó vế trái của bất đẳng thức là
x + y, b =y + z , c =+
( a − b) c + (b − c ) a + (c − a ) b = ab bc ca
+ + −a −b−c
b c a c a b .c
2 2 2
1  ab bc  1  bc ca  1  ca ab 
=  −  +  −  +  −  ≥0
c 
ok
2 c a  2 a b  2 b
Bất đẳng thức được chứng minh. Đẳng thức xảy ra khi và chỉ khi x= y= z.
bo

Ví dụ 3. Cho a,b,c là các số thực dương thỏa mãn điều kiện a + b + c =3.
4 4 4 a b c
Chứng minh rằng + + ≥ + + .
et

( a + b) (b + c ) (c + a ) b + c c + a a + b
3 3 3
vi

Lời giải
a + b =x a= 3 − y
ng

 
Đặt b + c =y do a, b, c > 0 và a + b + c =3 nên x, y, z > 0 và b= 3 − z .
c + a = c= 3 − x
 z 
a

4 4 4 3− y 3− x 3− z
Khi đó bất đẳng thức trở thành: + + ≥ + +
kh

3 3 3
.
x y z y x z

 4 3− x   4 3− y   4 3− z 
⇔ 3 − + − + − ≥0
x x   y 3 y   z 3 z 


( x + 1)( x − 2 )
2
+
( y + 1)( y − 2 )
2
+
( z + 1)( z − 2 )
2
≥0.
x3 y3 z3
Bất đẳng thức cuối luôn đúng, từ đó ta có đpcm.

19
Khám phá tư duy Kỹ thuật giải bất ĐT Bài toán Max – Min – Đặng Thành Nam

8) Kỹ thuật sử dụng phép thế


Từ bài toán có điều kiện từ hai biến trở lên ta rút một biến theo các biến còn lại
rồi thay vào bất đẳng thức cần chứng minh.
+ Dạng này toán nếu có cần kết hợp đánh giá một số là max hoặc một số là min.
Ví dụ 1. Cho a,b,c là các số thực không âm thoả mãn điều kiện ab + bc + ca = 1.
5abc
Chứng minh rằng a + b + c + ≥2.
3
Lời giải
Không mất tính tổng quát giả sử c ≥ b ≥ a .
1− ab 1 − ab 1 − ab
Thay c = ta phải chứng minh a + b + + 5ab. ≥2

n
a+b a+b 3( a + b )

.v
⇔ ab ( 2 − 5ab ) + 3 ( a + b − 1) ≥ 0
2

om
1
Bất đẳng thức hiển nhiên đúng vì ab ≤ .
3

9) Kỹ thuật đánh giá theo cặp


.c
Áp dụng với dạng tích bất đẳng thức dạng tích.
ok
Ví dụ 1. Cho a,b,c là các số thực thuộc khoảng ( 0;1) . Chứng minh rằng

( a − a )(b − b )( c − c ) ≥ ( a − bc )( b − ca )( c − ab ) .
bo

2 2 2

Lời giải
et

a − bc > 0
Không mất tính tổng quát giả sử a ≥ b ≥ c. Khi đó do a, b, c ∈ ( 0;1) ⇒  .
b − ca > 0
vi

Nếu c − ab < 0 bất đẳng thức luôn đúng.


ng

Nếu c − ab ≥ 0 khi đó ta chứng minh bc (1− a ) ≥ ( b − ac )( c − ab ) .


bc (1 − a ) ≥ ( b − ac )( c − ab ) ⇔ bc (1 − a )2 ≥ ( b − ac )( c − ab ) .
a

Thật vậy

( )
kh

⇔ bc a 2 − 2a + 1 ≥ bc − ab 2 − ac 2 + a 2bc ⇔ a ( b − c ) ≥ 0 (luôn đúng).


2

 ac (1 − b ) ≥ ( a − bc )( c − ab )

Tương tự ta có:  .
 ab (1 − c ) ≥ ( a − bc )( b − ca )
Nhân theo vế 3 bất đẳng thức trên ta có điều phải chứng minh.
Đẳng thức xảy ra khi và chỉ khi a= b= c .
Ví dụ 2. Cho a,b,c là các số thực dương thỏa mãn điều kiện a + b + c =
1. Chứng
minh rằng 8a 2b 2 c 2 ≥ ( a − bc )( b − ca )( c − ab ) .

20
Cty TNHH MTV DVVH Khang Việt

Lời giải
a − bc > 0
Không mất tính tổng quát giả sử a ≥ b ≥ c khi đó do a, b, c ∈ ( 0;1) ⇒  .
b − ca > 0
Nếu c − ab < 0 bất đẳng thức luôn đúng.
Nếu c − ab ≥ 0 khi đó ta chứng minh 2ab ≥ ( a − bc )( b − ca ) .
Thật vậy 2ab ≥ ( a − bc )( b − ca ) ⇔ 4a 2b2 ≥ ( a − bc )( b − ca ) .
( )
⇔ 4a 2b 2 ≥ ab − a 2 c − b 2 c + abc 2 ⇔ 4a 2b 2 + c a 2 + b 2 − ab c 2 + 1 ≥ 0 ( )
2
( )
⇔ 4a 2b 2 + c ( a − b ) + ab 2c − c 2 − 1 ≥ 0 ⇔ 4a 2b 2 + c ( a − b ) − ab ( c − 1) ≥ 0
2 2

n
.v
⇔ ab  4ab − ( a + b )  + c ( a − b ) ≥ 0 ⇔ ( c − ab )( a − b ) ≥ 0 (luôn đúng).
2 2 2
 

om
Tương tự ta có 2bc ≥ ( b − ca )( c − ab ) ;2ca ≥ ( c − ab )( a − bc ) .
Nhân theo vế 3 bất đẳng thức trên ta có ngay điều phải chứng minh. Đẳng thức
xảy ra khi và chỉ khi a= b= c=
1
3
.
.c
ok
10). Kỹ thuật sử dụng tính thuần nhất
Đưa bất đẳng thức về dạng đồng bậc sẽ dễ xử lý hơn(xem thêm chương 3).
bo

Ví dụ 1. Cho a,b,c là các số thực thoả mãn điều kiện a 2 + b 2 + c 2 =


3.
Chứng minh rằng a3 ( b + c ) + b3 ( c + a ) + c3 ( a + b ) ≤ 6 .
et

Lời giải
Bất đẳng thức đã cho tương đương với:
vi

(
2 2 2 2
)
2
a 3 ( b + c ) + b3 ( c + a ) + c 3 ( a + b ) ≤ a +b +c
ng

3
( ) ( ) ( ) (
⇔ 2 a 4 + b 4 + c 4 + 4 a 2b 2 + b 2 c 2 + c 2 a 2 ≥ 3ab a 2 + b 2 + 3bc b 2 + c 2 + 3ca c 2 + a 2 ) ( )
a

Bất đẳng thức trên là tổng của ba bất đẳng thức có dạng:
kh

( )
a 4 + b 4 + 4a 2b 2 − 3ab a 2 + b 2 = ( a − b ) + ab ( a − b )
4 2

(
= ( a − b ) a 2 − ab + b 2 ≥ 0
2
)
Bất đẳng thức được chứng minh. Đẳng thức xảy ra khi và chỉ khi a= b= c= 1.
11) Biến đổi hàm lượng giác
Ví dụ 1. Chứng minh với mọi số thực x ta có cos(sin x) > sin(cos x) .
Lời giải
π 
Bất đẳng thức đã cho tương đương với: sin  − sin x  − sin(cos x) > 0
2 
21
Khám phá tư duy Kỹ thuật giải bất ĐT Bài toán Max – Min – Đặng Thành Nam

π π
− sin x + cos x − sin x − cos x
⇔ 2cos 2 .sin 2 >0.
2 2
Bất đẳng thức cuối luôn đúng do
 π  π
sin x − cos
= x 2 sin  x −  ≤ 2; sin x + cos
= x 2 sin  x +  ≤ 2 .
 4  4
π π π
− 2 − sin x − cos x + 2
π
Vì vậy 0 < 2 < 2 ≤ 2 < ;
2 2 2 2
π π π
− 2 − sin x − cos x + 2

n
π
0< 2 ≤ 2 ≤ 2 <

.v
2 2 2 2
Bất đẳng thức được chứng minh.

om
B. BÀI TOÁN CHỌN LỌC
Bài 1. Cho x, y, z là các số thực dương thỏa mãn điều kiện x ≤ y ≤ z . .c
1 1 1 1 1
Chứng minh rằng y  +  + ( x + z ) ≤  +  ( x + z ) .
ok
x z y x z
Lời giải
bo

BĐT tương đương với:


( x + z) 2

y( x + z)
+
x+z
xz xz y
et

⇔ y ( x + z ) ≥ y 2 + zx ⇔ y 2 − y ( z + x ) + zx ≤ 0 ⇔ ( y − x )( y − z ) ≤ 0
vi

Bất đẳng thức cuối đúng vì 0 < x ≤ y ≤ z .


Ta có điều phải chứng minh. Đẳng thức xảy ra khi và chỉ khi x= y= z .
ng

Bài 2. Cho x,y,z là các số thực dương thoả mãn điều kiện x ≥ y ≥ z. Chứng minh
x( x 2 + y 2 ) y ( z 2 + x 2 ) z ( y 2 + z 2 )
a

+ + ≥ x2 + y 2 + z 2 .
x+ y z+x y+z
kh

Lời giải
Bất đẳng thức đã cho tương đương với:
xy ( y − x)  1 1   1 1 
 −  + yz ( y − z )  − ≥0
x+ y  x+z x+ y  y+z x+z
xy ( x − y )( y − z ) yz ( x − y )( y − z )
⇔ + ≥0
( x + y )( x + z ) ( x + z )( y + z )
Ta có điều phải chứng minh.

22
Cty TNHH MTV DVVH Khang Việt

Bài 3. Cho x, y, z là các số thực thuộc đoạn [ 0;1] . Tìm giá trị lớn nhất của biểu

( ) (
thức P= 2 x3 + y 3 + z 3 − x 2 y + y 2 z + z 2 x . )
Lời giải
Ta có x, y, z ∈ [ 0;1] ⇒ x ≤ x ≤ x; y 3 ≤ y 2 ≤ y; z 3 ≤ z 2 ≤ z.
3 2

( )
Từ đó suy ra 2 x3 + y 3 + z 3 ≤ x 2 + x + y 2 + y + z 2 + z.

(
P ≤ x + y + z + x2 + y 2 + z 2 − x2 y + y 2 z + z 2 x . )
(
Ta chứng minh x + y + z + x 2 + y 2 + z 2 − x 2 y + y 2 z + z 2 x ≤ 3 . )

n
⇔ x 2 (1 − y ) + y 2 (1 − z ) + z 2 (1 − x ) + x + y + z − 3 ≤ 0

.v
( ) ( ) (
⇔ x 2 − 1 (1 − y ) + y 2 − 1 (1 − z ) + z 2 − 1 (1 − x ) ≤ 0 . )

om
Bất đẳng thức cuối đúng do
( x − 1) (1 − y ) ≤ 0; ( y − 1) (1 − z ) ≤ 0; ( z − 1) (1 − x ) ≤ 0 .
2 2 2
.c
Vậy giá trị lớn nhất của P bằng 3 xảy ra khi x= y= z= 1 .
ok
Bài 4. Chứng minh rằng với mọi số thực không âm a,b,c ta có
(
a 4 + b 4 + c 4 + abc ( a + b + c ) ≥ 2 a 2b 2 + b 2 c 2 + c 2 a 2 . )
bo

Lời giải
Không mất tính tổng quát giả sử a ≥ b ≥ c. Khi đó c 2 ( c − a )( c − b ) ≥ 0 và
et

a 2 ( a − b )( a − c ) + b 2 ( b − c )( b − a ) = ( a − b )  a 2 ( a − c ) − b 2 ( b − c ) 
 
vi

≥ ( a − b ) a 2 ( b − c ) − b2 ( b − c )
 
ng

(
=( a − b )( b − c ) a 2 − b 2 ≥ 0 )
a

Bất đẳng thức được chứng minh. Đẳng thức xảy ra khi và chỉ khi a= b= c .
kh

Nhận xét. Đây là một trường hợp riêng của bất đẳng thức Schur. Với a,b,c là các
số thực không âm và k > 0 ta luôn có
a k ( a − b )( a − c ) + b k ( b − c )( b − a ) + c k ( c − a )( c − b ) ≥ 0 .
Bài 5. Cho a, b, c ≥ 0 thỏa mãn điều kiện a + b + c =
1.

Tìm giá trị lớn nhất của biểu thức P = a+


( b − c)
2
+ b+
( c − a)
2
+ c+
( a − b)
2
.
4 4 4
Lời giải
Chuyển mỗi biểu thức trong căn về cùng bậc hai ta có :

23
Khám phá tư duy Kỹ thuật giải bất ĐT Bài toán Max – Min – Đặng Thành Nam

a+
( b − c)
2
= a (a + b + c) +
( b − c)
2

4 4

= a + a (b + c ) +
( b + c ) − 4bc
2
=
 b+c 
2
b+c .
2

a +  − bc ≤  a +
2

4  2   2 

Suy ra a+
( b − c)
2
≤a+
b+c
4 2

Tương tự ta có : b+
( c − a )2 ≤b+
c+a
;
4 2

n
( a − b)
2
a+b

.v
c+ ≤c+
4 2

om
Cộng theo vế 3 bất đẳng thức trên ta được :

P= a+
( b − c )2 + b+
( c − a )2 + c+
( a − b )2
≤ 2 ( a + b + c )= 2.
4 4 4 .c
Vậy giá trị lớn nhất của P bằng 2 đạt tại a= b= 0, c= 1 hoặc các hoán vị.
ok
Nhận xét. Ta có thể tổng quát thành bài toán như sau :
Cho a,b,c,k là các số thực không âm thỏa mãn a + b + c =k . Chứng minh rằng
bo

ka +
( b − c)
+
2
+ kb +
( c − a)
2
≤ 2k . kc +
( a − b)
2

4 4 4
et

Bài 6. Cho a,b,c là các số thực không âm thỏa mãn điều kiện a + b + c =
1.
a + ( b − c ) + b + ( c − a ) + c + ( a − b ) ≥ 3.
2 2 2
vi

Chứng minh rằng


Lời giải
ng

Không mất tính tổng quát giả sử a ≥ b ≥ c khi đó :


Sử dụng bất đẳng thức Mincopsi ta có :
a

a + (b − c ) + b + (c − a ) + c + ( a − b) ≥
2 2 2
kh

( )
2
+ ( a − b ) + ( b − c ) + ( c − a ) 
2
a+ b+ c

( )
2
+ 4(a − c) .
2
= a+ b+ c
Bất đẳng thức được chứng minh nếu ta chứng minh được bất đẳng thức sau đúng.

( )
2
+ 4 ( a − c ) ≥ 3( a + b + c )
2
a+ b+ c

⇔ 4(a − c) ≥ ( ) +( ) +( )
2 2 2 2
a− b b− c c− a .

24
Cty TNHH MTV DVVH Khang Việt

Ta có :

( ) ( ) ( ) + ( b − c ) + 2 ( a − b )( )
2 2 2
 a− b + b− c  = a− b b− c
 
≥( a − b) +( b − c)
2 2

⇒( c − a) ≥( a − b) +( b − c)
2 2 2

( ) +( c ) + ( c − a ) ≤ 2( c − a ) .
2 2 2 2
Suy ra a− b b−
Mặt khác :

( ) ( ) 2 ( ) 

n
2 2 2
4(a − c) − 2
2
c− a = 2 a− c a+ c − 1

.v
( ) 2 ( a + c ) − 1 + 4 ac 
2
= 2 a− c

om
= 2( c )  2a + c + c − 1 + 4 ac 
2
a−

≥ 2( c )  a + b + c + c − 1 + 4 ac 
2 .c
a−

=2 ( c ) c + 4 ac  ≥ 0
ok
2
a−

1
Bài toán được chứng minh. Đằng thức xảy ra khi và chỉ khi a= b= c=
bo

hoặc
3
a= 1, b= c= 0 và các hoán vị.
et

Bài 7. Cho a,b,c là các số thực dương thỏa mãn điều kiện a + b + c =3.
vi

(
Chứng minh rằng 2 a 2b 2 + b 2 c 2 + c 2 a 2 + 3 ≤ 3 a 2 + b 2 + c 2 . ) ( )
ng

Lời giải
giả sử a min {a, b, c} ⇒ a ≤ 1 .
Không mất tính tổng quát =
a

( )
Ta có 2 a 2b 2 + b 2 c 2 + c 2 a 2 + 3 − 3 a 2 + b 2 + c 2 ≤ 0 ( )
kh

( )( )
⇔ 2a 2 − 3 b 2 + c 2 + 2b 2 c 2 + 3 − 3a 2 ≤ 0

⇔ ( 2a − 3) ( b + c ) + 2b c + 2 ( 3 − 2a ) bc + 3 − 3a ≤ 0
2 2 2 2 2 2

⇔=
P ( 2a − 3) ( 3 − a ) + 2b c + 2 (3 − 2a ) bc + 3 − 3a ≤ 0 .
2 2 2 2 2 2

2 2
b + c  3− a 
Ta có bc ≤   =  . Vì a ≤ 1 ⇒ 3 − 2a > 0 do đó
2

 2   2 

25
Khám phá tư duy Kỹ thuật giải bất ĐT Bài toán Max – Min – Đặng Thành Nam

4 2
(
P ≤ 2a − 3 ( 3 − a )
2
) 2 3−a 
+ 2. 
 2 
 + 2 3 − 2a  (
2 3−a 
 2 
 + 3 − 3a)2

.
=
3
8
(
( a − 1)2 3a 2 − 14a − 1 ≤ 0 )
Vì 3a 2 − 14a −
= 1 3a ( a − 1) − 11a − 1 < 0, ∀a ∈ ( 0;1] .
Bài toán được chứng minh. Đẳng thức xảy ra khi và chỉ khi a= b= c= 1 .
3
Bài 8. Cho các số thực a, b, c ∈ [ 0;1] thỏa mãn a + b + c = .
2
Tìm giá trị lớn nhất và giá trị nhỏ nhất của biểu thức=
P cos a 2 + b 2 + c 2 . ( )

n
.v
Lời giải
3 π
Do a, b, c ∈ [ 0;1] nên 0 ≤ a 2 + b 2 + c 2 ≤ a + b + c = < .

om
2 2
Vậy P lớn nhất( nhỏ nhất) khi a 2 + b 2 + c 2 nhỏ nhất (lớn nhất)
- Tìm giá trị nhỏ nhất của a 2 + b 2 + c 2 . .c
1 3 3
Ta có a 2 + b 2 + c 2 ≥ ( a + b + c ) = . Suy ra GTLN của P bằng cos ; xảy
2
ok
3 4 4
1
ra khi a= b= c=
bo

2
- Tìm giá trị lớn nhất của a 2 + b 2 + c 2 .
et

3 1
Giả sử a ≤ b ≤ c ⇒ a + b + c = ≤ 3c ⇒ c ≥ .
2 2
vi

2
3  5
Vậy a + b + c = ( a + b ) − 2ab + c ≤ ( a + b )
2 2
2 2 2 2
+ c =c +  − c ≤
2 2
2 
ng

4
Do ( c − 1)( 2c − 1) ≤ 0
a

5  1
; xảy ra khi ( a, b, c ) =  0,0,  hoặc các hoánvị.
Suy ra GTNN của P bằng cos
kh

4  2
Bài 9. (TSĐH Khối D 2008) Cho x, y là các số thực không âm.

Tìm giá trị lớn nhất, giá trị nhỏ nhất của biểu thức P =
( x − y )(1 − xy ) .
( x + 1)2 ( y + 1)2
Lời giải
Ta có :

=P
( x − y )(1=− xy ) ( x + xy ) − ( y =
2
+ yx ) 2
x ( y + 1) − y ( x + 1)
2
=
2
x

y
.
( x + 1)2 ( y + 1)2 ( x + 1) ( y + 1)
2 2
( x + 1)2 ( y + 1)2 ( x + 1) 2
( y + 1)2
26
Cty TNHH MTV DVVH Khang Việt

x 1 y 1
Với x, y ≥ 0 ta có : 0 ≤ ≤ ;0 ≤ ≤ .
( x + 1) 2 4 ( y + 1) 4
2

1
Suy ra giá trị lớn nhất của P bằng đạt tại =
x 1,=
y 0 và giá trị nhỏ nhất của
4
1
P − đạt tại=
x 0,= y 1.
4
Cách 2 : Ta có đánh giá thông qua trị tuyệt đối như sau :
( x − y )(1 − xy ) ( x + y )(1 + xy ) ( x + y + 1 + xy )2 1
P = ≤ ≤ = .
(1 + x )2 (1 + y )2 (1 + x )2 (1 + y 2 ) 4 (1 + x )2 (1 + y )2 4

n
.v
1 1
Do đó − ≤ P ≤ . Ta có kết quả tương tự.
4 4

om
Bài 10. Cho a, b, c ≥ 0 là các số đôi một khác nhau. Chứng minh rằng
 1 1 1 
( ab + bc + ca )  + + ≥4.
2 
 (a − b) (b − c ) ( ) 
2 2
.c c − a
Lời giải
ok
Giả sử c = min {a, b, c} , khi đó do a, b, c ≥ 0 ta suy ra: ab + bc + ca ≥ ab;
1 1
bo

≥ ;
(b − c ) 2
b2
et

1 1

(a − c) 2
a2
vi

 1 1 1 
Vậy ta chỉ cần chứng minh : ab  + + ≥ 4.
ng

 ( a − b )2 b 2 a 2 
 
(a − b) − 2 ≥ 0 2
a

ab a b ab
⇔ + + −4≥0⇔ +
( a − b )2 b a ( a − b )2 ab
kh

2
 a − b ) 
⇔  ab

( 2
≥0
 ( a − b )2 ab 
 
Bài toán được chứng minh. Xem thêm chương 3.
1 1 2
Bài 11. Cho các số thực thoả mãn điều kiện a, b, c > 0 và + =.
a c b
a+b c+b
Tìm giá trị nhỏ nhất của biểu thức
= P + .
2 a − b 2c − b

27
Khám phá tư duy Kỹ thuật giải bất ĐT Bài toán Max – Min – Đặng Thành Nam

Lời giải
2ac
Ta có b = thay vào biểu thức của P ta được :
a+c
2ac 2ac
a+ c+
a + c a +c = a + 3c c + 3a 3 a c 
P= + + =+
1  +  ≥ 4.
2a −
2ac
2c −
2ac 2a 2c 2 c a 
a+c a+c
(đúng theo AM-GM).
Vậy giá trị nhỏ nhất của P bằng 4 đạt tại a= b= c .
Bài 12. Cho các số thực a, b, c ∈ [1;3] thỏa mãn điều kiện a + b + c =6.

n
Tìm giá trị lớn nhất của biểu thức P = a 2 + b 2 + c 2 .

.v
Lời giải

om
Đặt a =x + 1; b =y + 1; c =z + 1; x, y, z ∈ [ 0;2].

Khi đó P = a 2 + b 2 + c 2 = ( x + 1) + ( y + 1) + ( z + 1)
2 2 2

= x2 + y 2 + z 2 + 2 ( x + y + z ) + 3
.c
ok
= ( x + y + z ) − 2 ( xy + yz + zx ) + 2 ( x + y + z ) + 3
2

−2 ( xy + yz + zx ) + 18
=
bo

Từ x, y, z ∈ [ 0;2] ⇒ ( 2 − x )( 2 − y )( 2 − z ) ≥ 0
⇔ 8 − 4 ( x + y + z ) + 2 ( xy + yz + zx ) − xyz ≥ 0
et

⇒ −2 ( xy + yz + zx ) = −4 − xyz ≤ −4 do xyz ≥ 0
vi

Từ đó suy ra P ≤ −2 ( xy + yz + zx ) + 18 ≤ 14


ng

Đẳng thức xảy ra khi và chỉ khi ( a, b, c ) = (1, 2,3) hoặc các hoán vị
Chú ý. Đặt a =x + 1; b =y + 1; c =z + 1 để chúng ta tận dụng tích xyz ≥ 0
a

Bài 13. Cho các số thực a, b, c ∈ [ 0;1] .


kh

1 1 1
Chứng minh rằng + + ≥ 3abc .
2−a 2−b 2−c
Lời giải
1
Ta có : ( a − 1) ≥ 0 ⇒ a ( 2 − a ) ≤ 1 ⇒
2
≥a.
2−a
1 1
Tương tự ta có : ≥ b; ≥c.
2−b 2−c
Cộng theo vế các bất đẳng thức trên ta được:

28
Cty TNHH MTV DVVH Khang Việt

1 1 1
+ + ≥ a + b + c ≥ 3 3 abc ≥ 3abc do abc ≤ 1 .
2−a 2−b 2−c
Đẳng thức xảy ra khi và chỉ khi a= b= c= 1 .
Bài 14. Cho các số thực a, b, c ∈ [ 0;1] và a + b + c ≠ 0 .
1 1 1 5
Chứng minh rằng + + ≤ .
ab + 1 bc + 1 ca + 1 a + b + c
Lời giải
Do bất đẳng thức đối xứng với ba biến nên không mất tính tổng quát ta giả sử
1≥ a ≥ b ≥ c ≥ 0 .
a + b + c 1 + b + c + (1 − b )(1 − c ) + bc

n
c a b
Khi đó + + ≤ ≤ =
2
ab + 1 bc + 1 ca + 1 bc + 1 bc + 1

.v
a+b b+c c+a  a+b   b+c   c+a 
Mặt khác + + =  − 1 +  − 1 +  − 1 + 3

om
ab + 1 bc + 1 ca + 1  ab + 1   bc + 1   ca + 1 

=−
(1 − a )(1 − b ) − (1 − b )(1 − c ) − (1 − c )(1 − a ) + 3 ≤ 3
ab + 1 bc + 1 ca + 1 .c
Cộng theo vế các bất đẳng thức trên ta suy ra điều phải chứng minh.
ok
Đẳng thức xảy ra khi và chỉ khi a= b= 1, c= 0 hoặc các hoán vị.
bo

Bài tập tương tự


Cho a,b,c là các số thực thuộc đoạn [0;1] và a + b + c > 0.
et

5 ab bc ca
Tìm giá trị nhỏ nhất của biểu thức P= + + + .
a + b + c 1 + ab 1 + bc 1 + ca
vi

Bài 15. Cho các số thực không âm a,b,c. Chứng minh rằng
ng

a 2 + b 2 + c 2 ≥ 4 ( a + b + c )( a − b )( b − c )( c − a ) .
Lời giải
a

Không mất tính tổng quát ta giả sử b nằm giữa a và c , ta xét hai trường hợp
kh

- Nếu a ≥ b ≥ c ⇒ VT ≥ 0 ≥ VP , ta có đpcm.


- Nếu c ≥ b ≥ a , khi đó vế phải
VP= 4 ( a + b + c )( a − b )( b − c )( c − a ) = 4 ( a + b + c )( b − a )( c − b )( c − a )

≤ ( a + b + c )( b − a ) + ( c − b )( c − a ) 
2

Ta chỉ cần chứng minh ( a + b + c )( b − a ) + ( c − b )( c − a ) ≤ a 2 + b 2 + c 2 .


Thật vậy bất đẳng thức này tương đương với : − a ( 2a + 2c − b ) ≤ 0 , đúng và ta
có đpcm.

29
Khám phá tư duy Kỹ thuật giải bất ĐT Bài toán Max – Min – Đặng Thành Nam

C. BÀI TẬP RÈN LUYỆN


Bài 1. Chứng minh rằng với mọi số thực không âm a và b ta có
( a + b ) ( a 2 + b2 ) ≥ 8ab ( a + b ) − 12ab ab .
Bài 2. Cho a,b,c là độ dài ba cạnh một tam giác thoả mãn a ≤ b ≤ c. Chứng minh
( a + b + c )2 ≤ 9bc .
x 2 cos a − 2 x + cos a
Bài 3. Cho a ∈ ( 0; π ) , x ∈ ; y =2 . Chứng minh rằng −1 ≤ y ≤ 1 .
x − 2 x cos a + 1
Bài 4. Cho x,y,z là các số thực dương thoả mãn điều kiện x ≥ y ≥ z .

n
x2 y y 2 z z 2 x
Chứng minh rằng + + ≥ x2 + y 2 + z 2 .

.v
z x y
Bài 5. Cho a, b, c, d, e là các số thực. Chứng minh các bất đẳng thức

om
a 2 + b2 + c 2 + d 2 + e2 ≥ a ( b + c + d + e ) ;
a 2 + b 2 + c 2 + d 2 + 1 ≥ a ( b + c + d + 1) .c
Bài 6. Cho a,b,c là các số thực dương. Chứng minh
ok
a3 b3 a+b+c c3
+ + . ≥
a + ab + b
2
b + bc + c
2 2
c + ca + a
2 23 2

Bài 7. Cho x,y là hai số thực không âm không đồng thời bằng 0. Chứng minh
bo

3 x 2 + 2 xy + 3 y 2
≥ 2 x2 + 2 y 2 .
2( x + y)
et

( x + y ) ( x3 + y 3 )
vi

9
Bài 8. Cho x,y là 2 số thực dương. Chứng minh 1 ≤ ≤ .
(x 2
+y )
2 2 8
ng

a+b a+b
Bài 9. Chứng minh với mọi số thực a và b ta có ≤ .
1+ a + b 1+ a + b
a
kh

Dấu bằng xảy ra khi nào?


1 b
Bài 10. Cho a,b là các số thực và a khác 0. Chứng minh a 2 + b 2 + 2
+ ≥ 3.
a a
Bài 11. Chứng minh rằng với mọi số thực dương a,b,c ta có
a 2 b2 c2
+ + ≥ a +b+c.
b c a
Bài 12. Chứng minh rằng với mọi số thực a,b,c ta có
a 4 + b 4 + c 4 ≥ abc ( a + b + c ) .

30
Cty TNHH MTV DVVH Khang Việt

Bài 13. Cho a,b,c là độ dài ba cạnh một tam giác. Chứng minh
a b c b c a
+ + − − − <1.
b c a a b c
Bài 14. Cho a,b,c là độ dài ba cạnh một tam giác thoả mãn điều kiện a < b < c.
( )
Chứng minh rằng a3 b 2 − c 2 + b3 c 2 − a 2 + c3 a 2 − b 2 < 0 . ( ) ( )
Bài 15. Cho a,b,c là độ dài ba cạnh một tam giác. Chứng minh
a ( b − c ) + b ( c − a ) + c ( a + b ) > a 3 + b3 + c 3 .
2 2 2

Bài 16. Cho a,b,c là độ dài ba cạnh một tam giác. Chứng minh

n
2a 2b 2 + 2b 2 c 2 + 2c 2 a 2 − a 4 − b 4 − c 4 > 0 .
Bài 17. Cho x,y,z là các số thực dương có tích bằng 1 và z = min { x, y, z}.

.v
2
x+2 y+2  z+2 

om
Chứng minh rằng .   ≥1.
2x + 1 2 y + 1 2z + 1 
Bài 18. Cho a,b,c là các số thực không âm thoả mãn điều kiện ab + bc + ca =
1. .c
1 1 1 5
Chứng minh rằng + + ≥ .
a+b b+c c+a 2
ok
Bài 19. Chứng minh với mọi a,b,c là các số thực ta có
a ( a + b) + b (b + c ) + c (c + a ) ≥ 0 .
3 3 3
bo

Bài 20. Cho a,b,c là độ dài 3 cạnh một tam giác. Chứng minh
a b c
et

a) + + <2;
b+c c+a a+b
vi

b) a 2 + b 2 + c 2 < 2 ( ab + bc + ca ) ;
ng

a b c
c) + + < 23 4 .
b +c a +c b +a
3 3 3 3 3 3 3 3 3
a

a 3 + b3 a 2 + b 2
Bài 21. Cho a,b là các số thực dương. Chứng minh 3 ≤ .
kh

2 a+b
Bài 22. Chứng minh rằng với mọi số thực x, y thoả mãn điều kiện x + y ≥ −1, xy ≤ 2.
Ta có x3 + y 3 ≥ −7 .
Bài 23. Cho a, b, c, d là các số thực.
( )(
Chứng minh rằng a 2 + b 2 c 2 + d 2 ≥ ( ac + bd ) . ) 2

Bài 24. Cho a,b là 2 số thực không âm. Chứng minh rằng ta luôn có
 2 3  2 3  1  1
 a + b +  b + a +  ≥  2a +  2b +  .
 4  4  2  2
31
Khám phá tư duy Kỹ thuật giải bất ĐT Bài toán Max – Min – Đặng Thành Nam

Bài 25. Cho x, y, z ∈ [ 0;2] thỏa mãn x + y + z =3.

Tìm giá trị lớn nhất của biểu thức P = x 2 + y 2 + z 2 .


Bài 26. Cho a,b,c là các số thực dương. Chứng minh
3 2
b+c 1b+c
a) 1+   ≤1+   .
 a  2 a 

a3 b3 c3
b) + + ≥1.
a3 + ( b + c ) b3 + ( c + a ) c3 + ( a + b )
3 3 3

a 2a

n
c) ≥ .
b+c a+b+c

.v
a b c
d) + + ≥2.

om
b+c c+a a+b
4
a a3
e) ≥ 4 4 4
. .c
a + 8bc
2
a3 + b3 + c3
ok
a b c
f) + +
≥1.
a 2 + 8bc b 2 + 8ca c 2 + 8ab
bo

Bài 27. Cho a,b,c là các số thực đôi một phân biệt. Chứng minh
a2 b2 c2
+ + ≥2.
et

( b − c )2 ( c − a )2 ( a − b )2
Bài 28. Chứng minh rằng với ba số thực a, b, c ta luôn có
vi

( a + 1)(b + 1)( c + 1) ≥ ( ab + bc + ca − 1) .
2 2 2 2
ng

Bài 29. Cho x,y,z là các số thực dương thỏa mãn x = max { x, y, z} .
a

x +1 y +1 z +1 x y z
Chứng minh + + ≤ + + .
y +1 z +1 x +1 y z x
kh

Bài 30. Cho x,y là hai số thực dương thoả mãn điều kiện x + 2 y =
1.
1 1 25
Chứng minh + ≥ .
x y 1 + 48 xy 2
1 1
Bài 31. Cho a,b là hai số thực khác 0 thoả mãn điều kiện ab ≥ + +3.
a b
3
 1 1 
Chứng minh rằng ab ≥  3 + 3  .
 a b

32
Cty TNHH MTV DVVH Khang Việt

Bài 32. Cho a,b,c,d là các số thực thuộc đoạn [ 0;2] . Chứng minh
a + b + c + d ≤ 1 + ab + 1 + bc + 1 + ca + 1 + ad .
Bài 33. Cho a, b, c là các số thực thuộc đoạn [ 0;2] và a + b + c =3.
a 2 + b2 + c2
Tìm giá trị lớn nhất và nhỏ nhất của biểu thức P = .
ab + bc + ca
Bài 34. Cho a, b, c, d là các số thực thỏa mãn điều kiện a 2 + b 2 + c 2 + d 2 =
1.
Tìm giá trị lớn nhất của biểu thức
P a 3 ( b + c + d ) + b3 ( c + d + a ) + c 3 ( d + a + b ) + d 3 ( a + b + c ) .
=

n
Bài 35. Cho a, b, c là các số thực thuộc đoạn [1;3] thỏa mãn điều kiện a 2 + b 2 + c 2 =
14 .

.v
 b  c
Chứng minh 1 −  2 +  ≥ −8 .

om
 a  a
Bài 36. Cho a,b,c là các số thực không âm. Chứng minh rằng
a 2 + b 2 + c 2 + ab 2 + bc 2 + ca 2 + 9 ≥ 5 ( a + b + c ) . .c
Bài 37. Cho a, b, c là các số thực dương. Chứng minh rằng
( ) ( )
ok
9 a3 + b3 + c3 + abc ≥ 8 ( a + b + c ) a 2 + b 2 + c 2 .

Bài 38. Cho a, b > 0 thỏa mãn điều kiện a 2 + b 2 =


bo

1.
2
1 1  a b
Chứng minh rằng + ≥ 2 2 +  −  .
a 
et

a b  b
5
vi

Bài 39. Cho a, b, c > 0 thỏa mãn điều kiện a 2 + b 2 + c 2 =.


3
ng

1 1 1 1
Chứng minh rằng + − < .
a b c abc
a

Bài 40. Cho a, b là hai số thực thỏa mãn a , b < 1, ta luôn có


kh

1 1 2
+ ≥ .
1− a 2
1− b 2 1 − ab
1 1 ab 3
Bài 41. Với mọi số thực a, b ≥ 1 ta luôn có + + ≤ .
a + 1 b + 1 ab + 1 2
Bài 42. Cho a, b, c ∈ [ 0;1] . Chứng minh rằng a (1 − b ) + b (1 − c ) + c (1 − a ) ≤ 1.
Bài 43. Cho a, b, c > 0 thỏa mãn a ≤ b ≤ c .
1 1 1
Chứng minh rằng ( a − b + c )  − +  ≥ 1 .
a b c

33
Khám phá tư duy Kỹ thuật giải bất ĐT Bài toán Max – Min – Đặng Thành Nam

Bài 44. Cho các số thực a, b, c thỏa mãn điều kiện a + b + c =


1.
Tìm giá trị nhỏ nhất của biểu thức
P= a 2 + ab + b 2 + b 2 + bc + c 2 + c 2 + ca + a 2 .

Bài 45. Cho a,b,c,d thuộc đoạn [1;2] . Chứng minh rằng
(a 2
)(
+ b2 c2 + d 2 ) ≤ 25 .
( ac + bd ) 2 12
Bài 46. Cho x,y,z là các số thực. Chứng minh
 3 ( x − y )2 3 ( y − z )2 3 ( z − x )2 
x 2 + y 2 + z 2 − xy − yz − zx ≥ max  , , .
4 4 4
 

n
Bài 47. Cho x,y,z là các số thực không âm. Chứng minh

.v
3
{
x3 + y 3 + z 3 − 3 xyz ≥ ( x + y + z ) .max ( x − y ) , ( y − z ) + ( z − x ) .
2 2 2
}

om
4
Bài 48. Cho a,b,c là các số thực không âm. Chứng minh
a+b+c 3
3
− abc ≤ max
2
{(
a− b ; b− c ; c− a
2
) ( .c 2
) ( )}
ok
D. HƯỚNG DẪN GIẢI – ĐÁP SỐ

( ) 
( ) 
2 2
( a + b )
bo

Bài 1. Bất đẳng thức tương đương với: a− b a+ b − 6ab  ≥ 0 .


( ) ( )
2
Chú ý ( a + b ) a+ b − 6ab = ( a − b )2 + 2 ab a + b − ab ≥ 0.
et

Bất đẳng thức được chứng minh. Đẳng thức xảy ra khi và chỉ khi a = b .
vi

Bài 2. Ta có a ≤ b ⇒ ( a + b + c ) ≤ ( 2b + c ) .
2 2
ng

Vậy ta chứng minh ( 2b + c ) ≤ 9bc ⇔ 4b 2 − 5bc + c 2 ≤ 0 ⇔ ( b − c )( 4b − c ) ≤ 0 .


2

Bất đẳng thức cuối luôn đúng do b − c ≤ 0; 4b − c ≥ 3b + a − c = ( a + b − c ) + 2b > 0 .


a

Bất đẳng thức được chứng minh. Đẳng thức xảy ra khi và chỉ khi a= b= c
kh

Bài tập tương tự

( )
2
Cho a,b,c là độ dài ba cạnh một tam giác ta có 4a 2b 2 > a 2 + b 2 − c 2

=
Bài 3. Ta có 1 − y 2
( x − 1)2 sin 2 a ≥ 0 ⇒ −1 ≤ y ≤ 1 .
(x )
2
2
− 2 x cos a + 1

x2 ( y − z ) y2 ( z − x) z2 ( x − y )
Bài 4. Bất đẳng thức tương đương với + + ≥0.
z x y

34
Cty TNHH MTV DVVH Khang Việt

 x2 ( y − z ) x2 ( y − z )
y ≥ z ⇒ ≥
 z y
Theo giả thiết ta có  .
 y2 ( z − x) y2 ( z − x)
x ≥ y ≥ z ⇒ ≥
 x y
Suy ra
x2 ( y − z ) y2 ( z − x) z2 ( x − y ) x2 ( y − z ) + y 2 ( z − x ) + z 2 ( x − y )
+ + ≥
z x y y

=
( x − y )( x − z )( y − z ) ≥ 0

n
y
Ta có điều phải chứng minh. Đẳng thức xảy ra khi và chỉ khi x= y= z .

.v
Bài 5. Bất đẳng thức tương đương với

om
4a 2 + 4b 2 + 4c 2 + 4d 2 + 4e2 ≥ 4a ( b + c + d + e )

( ) ( ) (
⇔ a 2 − 4ab + 4b 2 + a 2 − 4ac + 4c 2 + a 2 − 4ad + 4d 2 + a 2 − 4ae + 4e2 ≥ 0
.c ) ( )
⇔ ( a − 2b ) + ( a − 2c ) + ( a − 2d ) + ( a − 2e ) ≥ 0.
2 2 2 2
ok
Bất đẳng thức luôn đúng. Dẳng thức xảy ra khi và chỉ khi =
a 2=
b 2=
c 2=
d 2e .
Bất đẳng thứ hai là trường hợp riêng khi e = 1
bo

x3 2x − y
Bài 6. Ta chứng minh ≥ với mọi số thực dương x và y.
x + xy + y
2 2 3
et

Thật vậy bất đẳng thức tương đương với: ( x + y )( x − y ) ≥ 0 (luôn đúng).
2
vi

Áp dụng bất đẳng thức trên ta có


a3 2a − b
ng

≥ ;
a + ab + b
2 2 3
b3 2b − c
a

≥ ;
b + bc + c
2 2 3
kh

c3 2c − a

.
c + ca + a
2 32

Cộng theo vế ba bất đẳng thức trên ta có điều phải chứng minh. Đẳng thức xảy
ra khi và chỉ khi a= b= c .
Bài tập tương tự
Cho n là số nguyên dương lớn hơn hoặc bằng 2 và các số thực x1 , x2 ,..., xn có
tích bằng 1. Chứng minh

35
Khám phá tư duy Kỹ thuật giải bất ĐT Bài toán Max – Min – Đặng Thành Nam

n xi9 + x9j n ( n − 1)
∑ 6
+ xi3 x3j + x 6j

3
.
1≤i < j ≤ n xi

Bài 7. Bất đẳng thức đã cho tương đương với :


2
 3 x 2 + 2 xy + 3 y 2 
 ≥ 2 x + 2 y
2 2

 2 ( x + y ) 
( ) ( )( x + y)
2
≥ 0 ⇔ ( x − y) ≥ 0
2 4
⇔ 3 x 2 + 2 xy + 3 y 2 − 8 x2 + y 2
Bất đẳng thức được chứng minh. Đẳng thức xảy ra khi và chỉ khi x = y .

n
xy ( x − y )
2
Bài 8. Bất đẳng thức vế trái tương đương với: ≥ 0.

.v
( )
2
x2 + y 2

om
Dấu bằng xảy ra khi và chỉ khi x = y .

(x )
2
2
− 4 xy + y 2
Bất đẳng thức vế phải tương đương với: .c ≥0.
(
8 x +y 2 2 2
)
ok
Đẳng thức xảy ra khi và chỉ khi =
x (2 ± 3) y .
Bài 9. Biến đổi tương đương bất đẳng thức đã cho về dạng luôn đúng
bo

a + b ≥ a + b . Đẳng thức xảy ra khi và chỉ khi ab ≥ 0 .


Chú ý. Thực chất bất đẳng thức xuất phát từ tính đồng biến trên khoảng
et

x
( −1; +∞ ) của hàm số y = .
vi

x +1
Bài tập tương tự
ng

2014 + a + b 2014 + a + b
Chứng minh với mọi số thực a và b ta có ≤ .
a + b + 2015 a + b + 2015
a

2
 1 
kh

3
Bài 10. Bất đẳng thức tương đương với  b +  +a + 2 − 3≥0
2
 2a  4a

( )
2
 1 
2 2a 2 − 3
⇔ b +  + ≥0
 2a  4a 2
Bất đẳng thức được chứng minh. Đẳng thức xảy ra khi và chỉ khi
1 3
b= − ,a = ±4 .
2a 4
Bài 11. Bất đẳng thức chính là phần rút gọn của bất đẳng thức sau

36
Cty TNHH MTV DVVH Khang Việt

( a − b )2 + ( b − c )2 + ( c − a )2 ≥ 0 .
b c a
Đẳng thức xảy ra khi và chỉ khi a= b= c .
Bài 12. Bất đẳng thức đã cho chính là phần rút gọn của bất đẳng thức sau

(a ) + (b ) + (c ) + (a ) + (b ) + (c )
2 2 2 2 2 2
2
− b2 2
− c2 2
− a2 2
− bc 2
− ca 2
− ab ≥0.
Đẳng thức xảy ra khi và chỉ khi a= b= c .
Bài 13. Chú ý đẳng thức
a b c b c a a 2 − b2 b2 − c2 c2 − a 2
+ + − − =
− + +

n
b c a a b c ab bc ca
( ) (
c a 2 − b2 + a b2 − c2 + c a 2 − b2 ) ( ).

.v
=
abc

om
=
( a − c )( a − b )( c − b )
abc

Ta có
( a − c )( a − b )( c − b ) <
abc
=
1.
.c
abc abc
ok
Bất đẳng thức được chứng minh.
Bài 14. Chú ý nếu coi vế trái là một đa thức bậc ba của a thì ta có 2 hai nghiệm
bo

= a b= , a c. Vì vậy ta phân tích được vế trái dưới dạng


( a − b )( a − c )( b − c )( ab + bc + ca ) .
et

Rõ ràng với a < b < c ⇒ ( a − b )( a − c )( b − c )( ab + bc + ca ) < 0 .


vi

Bất đẳng thức được chứng minh.


Bài 15. Bất đẳng thức đã cho tương đương với
ng

a ( b − c ) − a 2  + b ( c − a ) − b 2  + c ( a + b ) − c 2  > 0
2 2 2
     
⇔ ( a + b − c )( b + c − a )( c + a − b ) > 0
a
kh

Bất đẳng thức cuối luôn đúng và ta có đpcm.


Bài 16. Gọi P là biểu thức vế trái ta có đẳng thức sau
P= ( a + b + c )( a + b − c )( b + c − a )( c + a − b ) > 0 .
Bất đẳng thức được chứng minh.
Bài 17. Quy đồng rút gọn bất đẳng thức tương đương với :
1 + 2 x + 2 y ≥ 2 xz 2 + 2 z 2 + z
⇔ 1 − z + 2 (1 − z ) ( x + y) ≥ 0
2

37
Khám phá tư duy Kỹ thuật giải bất ĐT Bài toán Max – Min – Đặng Thành Nam

Bất đẳng thức cuối đúng do vậy ta có đpcm. Đẳng thức xảy ra khi và chỉ khi
x= y= z= 1 .
5abc
Bài 18. Sử dụng bất đẳng thức a + b + c + ≥ 2 và quy đồng rút gọn bất đẳng
3
thức cần chứng minh
2 ( a + b )( a + c ) + 2 ( b + c )( b + a ) + 2 ( c + a )( c + b ) ≥ 5 ( a + b )( b + c )( c + a )
⇔ 2 ( a + b + c ) + 5abc + 2 ≥ 5 ( a + b + c )
2

Sử dụng 5abc ≥ 6 − 3 ( a + b + c ) ta được

2 ( a + b + c ) + 5abc + 2 − 5 ( a + b + c ) ≥ 2 ( a + b + c ) + 8 − 8 ( a + b + c )
2 2

n
= 2 ( a + b + c − 2) ≥ 0

.v
2

Bất đẳng thức được chứng minh. Đẳng thức xảy ra khi và chỉ khi a= b= 1, c= 0

om
hoặc các hoán vị.
 2a = x + y − z

Bài 19. Đặt x = a + b, y = b + c, z = c + a ⇒ 2b = y + z − x .
 2c = z + x − y
.c

ok
Bất đẳng thức trở thành
x3 ( x + y − z ) + y 3 ( y + z − x ) + z 3 ( z + x − y ) ≥ 0
bo

.
⇔ x 4 + y 4 + z 4 + x3 y + y 3 z + z 3 x − xy 3 − yz 3 − zx3 ≥ 0
et

Bài 20. a) Ta có
a 2a 2a
= <
vi

b+c b+c+b+c a+b+c


b 2b 2b
ng

= < .
c+a c+a+c+a a+b+c
c 2c 2c
= <
a

a+b a+b+a+b a+b+c


kh

Cộng theo vế 3 bất đẳng thức trên ta có điều phải chứng minh.
b) Ta có
0 < a < b + c ⇒ a2 < a (b + c )
0 < b < c + a ⇒ b2 < b ( c + a ) .
0 < c < a + b ⇒ c2 < c ( a + b )
Cộng theo vế ba bất đẳng thức trên ta có điều phải chứng minh.
c) Ta có

38
Cty TNHH MTV DVVH Khang Việt

3 1
a3 + b3 =( a + b ) − 3ab ( a + b ) ≥ ( a + b ) − ( a + b )2 .( a + b ) = ( a + b )3
3 3
4 4
.
c c
⇒ ≤ 3 4.
3
a 3 + b3 a+b
Tương tự rồi cộng theo vế 3 bất đẳng thức trên ta có điều phải chứng minh.
Bài 21. Bất đẳng thức đã cho tương đương với:
2
a 3 + b3  a 2 + b 2 
2
≤
 a + b 
4
(
⇔ ( a − b ) a 2 + ab + b 2 ≥ 0 . )
 
Bài 22. Bất đẳng thức đã cho tương đương với

n
x3 + y 3 + 1 − 3 xy + 3 ( xy + 2 ) ≥ 0

.v
( )
⇔ ( x + y + 1) x 2 + y 2 + 1 − x − y − xy + 3 ( xy + 2 ) ≥ 0 .

om

1
2
(
( x + y + 1) ( x − y )2 + ( x − 1)2 + ( y − 1)2 + 3 ( xy + 2 ) ≥ 0)
Bất đẳng thức cuối luôn đúng theo giả thiết ta có đpcm. .c
( )( )
Bài 23. Ta có a 2 + b 2 c 2 + d 2 = ( ac + bd ) + ( ad − bc ) ≥ ( ac + bd ) .
2 2 2
ok
Đẳng thức xảy ra khi và chỉ khi ad = bc .
2 2
bo

 1  1 1 1
Bài 24. Ta có :  a −  ≥ 0;  b −  ≥ 0 ⇒ a 2 + ≥ a; b 2 + ≥ b .
 2  2 4 4
et

Suy ra :
2
 2 3  2 3  1  1  1
 a + b +  b + a +  ≥  a + b +  a + b +  =  a + b + 
vi

 4  4  2  2  2
.
ng

2
 1   1   1  1   1  1
=  a +  +  b +   ≥ 4  a +  b +  =  2a +  2b + 
 4  4   4  4  2  2
a

1
Bất đẳng thức được chứng minh. Đẳng thức xảy ra khi và chỉ khi a= b= .
kh

2
Bài 25. Theo giả thiết ta có :
x ( x − 2) + y ( y − 2) + z ( z − 2) ≤ 0 ⇔ x2 + y 2 + z 2 ≤ 2 ( x + y + z ) =
6.
b+c
Bài 26. a) Đặt
= x > 0. Ta cần chứng minh
a
1 2
( ) ( )
2
1 + x3 ≤ 1 + x ⇔ x2 + 2 ≥ 4 x3 + 1
2
⇔ x 4 − 4 x3 + 4 x 2 ≥ 0 ⇔ x 2 ( x − 2 ) ≥ 0
2

39
Khám phá tư duy Kỹ thuật giải bất ĐT Bài toán Max – Min – Đặng Thành Nam

Bất đẳng thức cuối đúng suy ra đpcm.


Đẳng thức xảy ra khi và chỉ khi b + c =2a .
b)Áp dụng câu a) ta có
a3 a2
≥ ;
a3 + ( b + c ) a 2 + b2 + c2
3

b3 b2
≥ ;
b3 + ( c + a ) a 2 + b2 + c2
3

c3 c2

n
.
c3 + ( a + b ) a 2 + b2 + c2
3

.v
Cộng theo vế ba bất đẳng thức trên ta có điều phải chứng minh. Đẳng thức xảy
ra khi và chỉ khi a= b= c .

om
a) Bất đẳng thức tương đương với:
a ( a + b + c ) ≥ 4a 2 ( b + c )
2

⇔ a ( a + b + c ) − 4 a ( b + c )  ≥ 0 ⇔ a ( a − b − c ) ≥ 0
2 2
.
.c
 
ok
Bất đẳng thức luôn đúng. Đẳng thức xảy ra khi và chỉ khi a = 0 hoặc a= b + c .
b) Áp dụng câu c) ta có
bo

a 2a b 2b c 2c
≥ ; ≥ ; ≥ .
b+c a+b+c c+a a+b+c a+b a+b+c
et

Cộng theo vế ba bất đẳng thức trên ta có đpcm. Với a,b,c thì đẳng thức không
xảy ra.
vi

c) Bất đẳng thức tương đương với:

(a ) ( )( )
ng

2
4/3
+ b 4/3 + c 4/3 − a8/3 = b 4/3 + c 4/3 2a 4/3 + b 4/3 + c 4/3

≥ 2(bc) 2/3 .4a 2/3 (bc)1/3


a

( ) ( )
2
⇒ a 4/3 + b 4/3 + c 4/3 ≥ a8/3 + 8bc.a 2/3
= a 2/3 a 2 + 8bc
kh

a a 4/3
⇒ ≥
a 2 + 8bc a + b + c
4/3 4/3 4/3

Bất đẳng thức cuối đúng theo AM – GM.


d) Áp dụng chứng minh ở câu e) xây dựng ba bất đẳng thức cùng dạng rồi cộng lại
ta có điều phải chứng minh.
Bài 27. Chú ý hằng đẳng thức
bc ca ab
+ + =
1.
( a − b )( a − c ) ( b − c )( b − a ) ( c − a )( c − b )
40
Cty TNHH MTV DVVH Khang Việt

2
a2 b2 c2  a b c 
Khi đó + + =  + +  +2≥2.
( b − c )2 ( c − a )2 (a − b)  b − c c − a a − b 
2

Bài 28. Chú ý đẳng thức :


( a + 1)(b + 1)( c + 1) =
2 2 2
( a + b + c − ab − bc − ca )2 + ( ab + bc + ca − 1)2 .
Bài 29. Bất đẳng thức tương đương với:
x + y +1 y + z +1
( x − z )( x − y ) + ( y − z)
2
≥ 0 (luôn đúng) do
xy ( x + 1)( y + 1) yz ( y + 1)( z + 1)
x − z ≥ 0, x − y ≥ 0 .

n
Bất đẳng thức được chứng minh đẳng thức đạt tại x= y= z .

.v
t +a t +b t +c
Nhận xét. Nếu để tinh ý ta có thể khảo sát hàm f (t ) = + + . Lúc này
t +b t +c t +a

om
vế trái bất đẳng thức thay số 1 bởi một số dương bất kỳ bất đẳng thức vẫn đúng.
Bài 30. Thay x = 1 − 2 y vào bất đẳng thức cần chứng minh đưa về chứng minh.

( )
1 1
+ ≥
25
1 − 2 y y 1 + 48 (1 − 2 y ) y 2
2 .c
⇔ 12 y 2 − 7 y + 1 ≥ 0 (luôn đúng).
ok
1 1 1
Bài toán được chứng minh. Đẳng thức xảy ra tại=x =,y hoặc x= y= .
2 4 3
bo

1 1
Bài 31. Đặt x =
3
ab , y =
− 3 ,z =
− 3 ⇒ xyz =
1.
a b
et

Theo giả thiết ta có x3 + y 3 + z 3 ≥ 3 =3 xyz

( )
vi

⇔ ( x + y + z ) x 2 + y 2 + z 2 − xy − yz − zx ≥ 0 .
ng

1 1 1
Do x 2 + y 2 + z 2 − xy − yz − zx= ( x − y )2 + ( y − z )2 + ( z − x )2 ≥ 0 .
2 2 2
a

3
 1 1 
Do đó x + y + z ≥ 0 ⇔ x ≥ − y − z ⇔ ab ≥  3 + 3  .
kh

 a b
Bất đẳng thức được chứng minh.
Bài 32. Xuất phát từ a − b ≤ 2, ∀a, b ∈ [ 0;2] ta được:

( a − b )2 ≤ 4 ⇔ a 2 + b2 ≤ 4 + 2ab .
⇒ ( a + b ) ≤ 4 + 2ab + 2ab= 4 (1 + ab ) ⇒ a + b ≤ 2 1 + ab .
2

Tương tự ta có: b + c ≤ 2 1 + bc ; c + d ≤ 2 1 + cd ; d + a ≤ 2 1 + da .
Cộng theo vế 4 bất đẳng thức trên ta có ngay điều phải chứng minh.

41
Khám phá tư duy Kỹ thuật giải bất ĐT Bài toán Max – Min – Đặng Thành Nam

=
Bài 33. Ta có: P
( a + b + c )2 − 2 ( ab=
+ bc + ca )
9
−2.
ab + bc + ca ab + bc + ca
1
Sử dụng bất đẳng thức cơ bản ta có: ab + bc + ca ≤ ( a + b + c ) =
2
3 suy ra
3
9
P≥ −2= 1.
3
Vậy giá trị nhỏ nhất của P bằng 1 đạt tại a= b= c= 1 .
Để tìm giá trị lớn nhất của P ta tìm giá trị nhỏ nhất của ab + bc + ca .
Vì a, b, c ∈ [ 0;2] nên

n
( a − 2 )( b − 2 )( c − 2 ) ≤ 0 ⇔ abc − 2 ( ab + bc + ca ) + 4 ( a + b + c ) − 8 ≤ 0 .

.v
abc + 4 ( a + b + c ) − 8 4 + abc 9 5
⇔ ab + bc + ca ≥ = ≥2⇒ P≤ = −2 .

om
2 2 2 2
5
Vậy giá trị lớn nhất của P bằng đạt tại=
a 2,=
b 1,=
c 0 và các hoán vị.
2
Bài 34. Viết lại biểu thức P dưới dạng:
.c
( ) ( ) ( ) ( ) (
P= ab a 2 + b 2 + bc b 2 + c 2 + cd c 2 + d 2 + da d 2 + a 2 + ac a 2 + c 2 + bd b 2 + d 2 . ) ( )
ok
Để ý:
bo

x4 + y 4 + 6 x2 y 2 − ( x − y )
4
( x − y )4 = x 4 + y 4 + 6 x 2 y 2 − 4 xy ( x 2 + y 2 ) ⇒ xy ( x 2 + y 2 ) = .
4
et

Áp dụng vào bài toán suy ra

( )
2
3 a 2 + b2 + c2 + d 2 ( a − b )4 + ( b − c )4 + ( c − d )4 + ( d − a )4 + ( a − c )4 + ( b − d )4
vi

P −
4 4
ng


(
3 a +b +c +d
2 2
=
3
2
)
2 2
a

4 4
3 1
kh

Vậy giá trị lớn nhất của P bằng đạt tại a= b= c= d= .


4 2
Tổng quát. Cho n số thực không âm x1 , x2 ,..., xn thỏa mãn mãn x1 + x2 + ... + xn =
1.
 n  n
Tìm giá trị lớn nhất của biểu thức P = xi2  ∑ x 2j  .
 j =1  ∑
1≤i ≠ j ≤ n  
Bài 35. Đặt vế trái bất đẳng thức là P. Khi đó

42
Cty TNHH MTV DVVH Khang Việt

=P
ac − 2ab − bc
= +2
(a 2
)
+ b 2 + c 2 + 2ac − 2ab − 2bc − 2ab − 14
+2
a2 2a 2

=
( a − b + c )2 − b − 7
+2≥− − 2 +2
b 7
2 a a 2 a a
2a
b 7
Mặt khác a, b, c ∈ [1;3] nên − ≥ −3, − 2 ≥ −7 ⇒ P ≥ −3 − 7 + 2 = −8 .
a a
Bất đẳng thức được chứng minh. Đẳng thức xảy ra khi và chỉ khi a = 1, b = 3,
c = 2.
Bài 36. Nhận xét. Dự đoán dấu bằng của bất đẳng thức đạt tại a= b= c= 1 nên ta

n
đặt ẩn phụ a = x + 1, b = y + 1, c = z + 1, ( x, y, z ≥ −1) đưa về chứng minh bất đẳng thức:

.v
( x + 1)2 + ( y + 1)2 + ( z + 1)2 + ( x + 1)( y + 1)2 + ( y + 1)( z + 1)2 + ( z + 1)( x + 1)2 + 9

om
≥ 5 ( x + y + z + 3) .
Khai triển và rút gọn ta được:
( )
2 x 2 + y 2 + z 2 + 2 ( xy + yz + zx ) + xy 2 + yz 2 + zx 2 ≥ 0 .
.c
ok
⇔ ( x + y + z ) + x 2 ( z + 1) + y 2 ( x + 1) + z 2 ( y + 1) ≥ 0 (luôn đúng).
2

Bài toán được chứng minh.


bo

Đẳng thức xảy ra khi và chỉ khi x = y = z = 0 ⇔ a = b = c = 1 .


Bài 37. Bất đẳng thức đã cho tương đương với:
( )
et

5 a3 + b3 + c3 + 9abc ≥ 4  ab ( a + b ) + bc ( b + c ) + ca ( c + a )  .
vi

Theo bất đẳng thức AM-GM ta có: a3 + b3 + c3 ≥ 3abc .


( ) (
Suy ra 5 a3 + b3 + c3 + 9abc ≥ 4 a3 + b3 + c3 + 3abc . )
ng

Ta đi chứng minh: a3 + b3 + c3 + 3abc ≥ ab ( a + b ) + bc ( c + a ) + ca ( a + b ) .


a

⇔ a ( a − b )( a − c ) + b ( b − c )( b − a ) + c ( c − a )( c − b ) ≥ 0 .
kh

Không mất tính tổng quát ta giả sử a ≥ b ≥ c khi đó: c ( c − a )( c − b ) ≥ 0 và


a ( a − b )( a − c ) + b ( b − c )( b − a ) = ( a − b )  a ( a − c ) − b ( b − c ) 

= ( a − b ) a 2 − b2 + c ( b − a ) = ( a − b ) ( a + b − c ) ≥ 0
2
 
Suy ra a ( a − b )( a − c ) + b ( b − c )( b − a ) + c ( c − a )( c − b ) ≥ 0 .
Bất đẳng thực được chứng minh. Đẳng thức xảy ra khi và chỉ khi a= b= c .
Bài 38. Bất đẳng thức cần chứng minh tương đương với

43
Khám phá tư duy Kỹ thuật giải bất ĐT Bài toán Max – Min – Đặng Thành Nam

1 1 a b
+ ≥ 2 2 + + − 2 ⇔ a + b + 2ab ≥ 2 2ab + 1
a b b a
 ( a + b )2 − 1 
⇔ a + b + (a + b) −1 ≥ 2 2 
2
 +1
 2 

( )
⇔ 1 − 2 t 2 + t + 2 − 2 ≥ 0 (*) ; với t = a + b ∈ 1; 2  (
(Vì ( a + b ) = a 2 + b 2 + 2ab > a 2 + b 2 =1 ⇒ a + b > 1
2

( )
Và ( a + b ) ≤ 2 a 2 + b 2 = 2 ⇒ a + b ≤ 2
2

n
( (
Suy ra t ∈ 1; 2  ). Bất đẳng thức (*) luôn đúng với t ∈ 1; 2  .

.v
Bài 39. Do a, b, c > 0 nên bất đẳng thức tương đương với : bc + ca − ab < 1

om
Ta có : ( a + b − c ) ≥ 0 ⇒ bc + ca − ab ≤
2
2
(
1 2 5
)
a + b 2 + c 2 = < 1 (luôn đúng).
6
Bài toán được chứng minh. .c
1 1 2
Bài 40. Ta có: + ≥ .
(1 − a )(1 − b )
ok
1− a 2
1 − b2 2 2

Ta chứng minh
bo

(1 − a )(1 − b ) ≤ 1 − ab ⇔ 1 − a
2 2 2
− b 2 + a 2b 2 ≤ 1 − 2ab + a 2b 2 ⇔ ( a − b ) ≥ 0
2
et

(luôn đúng).
Bài 41. Quy đồng rút gọn đưa về bất đẳng thức: ( ab − 1)( a − 1)( b − 1) ≥ 0 (luôn
vi

đúng do a, b ≥ 1 ).
ng

Bài 42. Bất đẳng thức cần chứng minh tương đương với :
a + b + c − ( ab + bc + ca ) ≤ 1
a

Xuất phát từ giả thiết ta có :


(1 − a )(1 − b )(1 − c ) ≥ 0 ⇔ 1 − ( a + b + c ) + ( ab + bc + ca ) − abc ≥ 0
kh

⇒ ( a + b + c ) − ( ab + bc + ca ) ≤ 1 − abc ≤ 1
Đẳng thức xảy ra khi và chỉ khi a= b= 0, c= 1 hoặc các hoán vị.
Bài 43. Bất đẳng thức cần chứng minh tương đương với
1 1 1 1 1 1 1 1
− + ≥ ⇔ + ≥ +
a b c a −b+c a c b a −b+c
a+c a+c 1 1
⇔ ≥ ⇔ ≥
ac b(a − b + c) ac b ( a − b + c )

44
Cty TNHH MTV DVVH Khang Việt

⇔ b ( a − b + c ) ≥ ac ⇔ a ( b − c ) − b ( b − c ) ≥ 0 ⇔ ( a − b )( b − c ) ≥ 0
Bất đẳng thức cuối luôn đúng do a ≤ b ≤ c . Ta có đpcm.
3 1 3
Bài 44. Ta có a 2 + ab + b 2 = ( a + b ) + ( a − b ) ≥ ( a + b ) .
2 2 2
4 4 4
3 3
⇒ a 2 + ab + b 2 ≥ a+b ≥ (a + b) .
2 2
3 3
Tương tự ta có : b 2 + bc + c 2 ≥ b+c ≥ (b + c ) ;
2 2
3 3
c 2 + ac + a 2 ≥ c+a ≥ (c + a) .

n
2 2

.v
Cộng theo vế ba bất đẳng thức trên ta được : P ≥ 3 ( a + b + c ) =3 .

om
1
Đẳng thức xảy ra khi và chỉ khi a= b= c= .
3
3( x − y )
2  3 ( x − y )2 3 ( y − z )2 3 ( z − x )2 
.c
Bài 46. Giả sử = max  , , .
4  4 4 4 
ok
3( x − y )
2
Ta chứng minh x + y + z − xy − yz − zx ≥
2 2 2
.
4
bo

⇔ 4 x 2 + 4 y 2 + 4 z 2 − 4 xy − 4 yz − 4 zx ≥ 3x 2 + 3 y 2 − 6 xy .

⇔ x 2 + y 2 + 4 z 2 + 2 xy − 4 yz − 4 zx ≥ 0 ⇔ ( x + y − 2 z ) ≥ 0 (luôn đúng).
et

Bài toán được chứng minh.


vi

(
Bài 47. Ta có: x3 + y 3 + z 3 − 3 xyz = ( x + y + z ) x 2 + y 2 + z 2 − xy − yz − zx . )
ng

Bài toán đưa về chứng minh


 3 ( x − y )2 3 ( y − z )2 3 ( z − x )2 
a

x 2 + y 2 + z 2 − xy − yz − zx ≥ max  , , .
 4 4 4 
kh

Đây chính là kết quả bài toán trên và ta có điều phải chứng minh.

CHỦ ĐỀ 2: KỸ THUẬT CHỨNG MINH PHẢN CHỨNG


A. NỘI DUNG PHƯƠNG PHÁP
Giả sử cần chứng minh bất đẳng thức nào đó đúng, ta giả sử bất đẳng thức đó
là sai và kết hợp với điều kiện giả thiết chỉ ra điều vô lý. Điều vô lý có thể là trái
với giả thiết hoặc trái với một điều đúng. Từ đó suy ra bất đẳng thức cần chứng
minh là đúng.

45
Khám phá tư duy Kỹ thuật giải bất ĐT Bài toán Max – Min – Đặng Thành Nam

B. BÀI TẬP MẪU


Bài 1. Chứng minh rằng ít nhất một trong các bất đẳng thức sau là đúng
a 2 + b 2 ≥ 2bc; b 2 + c 2 ≥ 2ca; c 2 + a 2 ≥ 2ab .
Lời giải
a 2 + b 2 < 2bc

Giả sử tất cả các bất đẳng thức trên đều sai khi đó b 2 + c 2 < 2ca .
 2
c + a < 2ab
2

( )
Cộng theo vế 3 bất đẳng thức trên ta được 2 a 2 + b 2 + c 2 < 2 ( ab + bc + ca )

n
( ) ( ) (
⇔ a 2 − 2ab + b 2 + b 2 − 2bc + c 2 + c 2 − 2ca + a 2 < 0 )

.v
⇔ ( a − b ) + ( b − c ) + ( c − a ) < 0 (mâu thuẫn).
2 2 2

om
Vì vậy điều phản chứng là sai nên khẳng định đề bài đúng (đpcm).
Bài 2. Cho a,b,c là các số thực thuộc khoảng ( 0;1) . Chứng minh ít nhất một trong
.c
1 1 1
các bất đẳng thức sau là đúng a (1 − b ) ≤ ; b (1 − c ) ≤ ; c (1 − a ) ≤ .
ok
4 4 4
Lời giải

bo

1
a (1 − b ) > 4

 1
Giả sử tất cả các bất đẳng thức trên đều sai khi đó b (1 − c ) > .
et

 4
 1
c (1 − a ) > 4
vi


ng

1
Nhân theo vế ba bất đẳng thức trên ta được a (1 − a ) b (1 − b ) c (1 − c ) > (1).
64
a

2
 a +1− a  1
Chú ý. 0 < a (1 − a ) ≤   =.
kh

 2  4
2
 b +1− b  1
0 < b (1 − b ) ≤   =
 2  4
2
 c +1− c  1
0 < c (1 − c ) ≤   =
 2  4
Nhân theo vế ba bất đẳng thức trên ta được
1
a (1 − a ) b (1 − b ) c (1 − c ) ≤ , dẫn tới mâu thuẫn với (1).
64

46
Cty TNHH MTV DVVH Khang Việt

Vậy điều phản chứng là sai do đó ít nhất một trong các bất đẳng thức đã cho là
đúng (đpcm).
Bài tập tương tự
Cho a,b,c là các số thực thuộc khoảng ( 0;1) . Chứng minh ít nhất một trong các
bất đẳng thức sau là sai
1 1 1
a (1 − b ) > ; b (1 − c ) > ; c (1 − a ) > .
4 4 4
Bài 3. (HOMC 2007) Cho p = abcd là một số nguyên tố có bốn chữ số.
Chứng minh rằng phương trình ax3 + bx 2 + cx + d =0 không có nghiệm hữu tỷ.

n
Lời giải

.v
Giả sử phương trình có nghiệm hữu tỷ thì nghiệm này phải âm giả sử nghiệm đó
p
là x0 =− , p , q ∈ * , ( p , q ) =
1. Khi đó

om
q
p3 p2 p a q
−a +b −c + d = 0 ⇔ −ap3 + bp 2 a − cpq 2 + dq3 = 0 ⇒  .
d  p
q 3
q 2 q .c
Do đó p,q là các số tự nhiên có một chữ số và vì p,q là nghiệm của phương trình
ok
nên f ( x) = ax3 + bx 2 + cx + d = ( qx + p ) ( ex 2 + fx + g ) , e, f , g ∈ * .
(10q + p )(100e + 10 f + g )= qp.efg trái với giả thiết
bo

Ta có abcd = f (10)=
abcd là một số nguyên tố.
et

Vậy điều phản chứng là sai ta có điều phải chứng minh.


Bài tập tương tự
vi

(HOMC 2007) Cho p = abc là một số nguyên tố có ba chữ số.


ng

Chứng minh rằng phương trình ax 2 + bx + c =0 không có nghiệm hữu tỷ.


Bài 4. Giả sử f1 ( x) =x 2 + a1 x + b1; f 2 ( x) =x 2 + a2 x + b2 là hai tam thức bậc hai với
a

hệ số nguyên có nghiệm chung là a. Chứng minh rằng nếu a không là số nguyên thì
kh

tam thức bậc hai sau luôn có nghiệm thực f ( x) = x 2 + ( a1 + a2 ) x + b1 + b2 .


Lời giải
p
Ta chứng minh a không thể là số hữu tỷ thật vậy giả sử a = , p, q ∈ , ( p, q ) =1 .
q
Do a là nghiệm của
p2 p
f1 ( x) ⇒ a 2 + a1a + b1 =0 ⇔ 2
+ a1 + b1 =0 ⇔ p 2 + a1 pq + b1q 2 =0 .
q q

47
Khám phá tư duy Kỹ thuật giải bất ĐT Bài toán Max – Min – Đặng Thành Nam

Suy ra p 2  q ⇒ p q ⇒ ( p, q ) = q =1 ⇒ a ∈  trái với giả thiết a không là số nguyên.


Vì vậy a không là số hữu tỷ.
Do a là nghiệm chung của
a + a1a + b1 =
2
0
f1 ( x), f 2 ( x) ⇒  ⇒ ( a1 − a2 ) a =b2 − b1 (1) .
a + a2 a + b2 =
2
0
Do a không là số hữu tỷ nên (1) ⇔ a= 1 a2 ; b= 1 b2 .

Khi đó f1 ( x) =x 2 + a1 x + b1; f ( x) =x 2 + 2a1 x + 2b1 .


Do f1 ( x) có nghiệm nên a12 − 4b1 ≥ 0 khi đó f(x) có

n
1 2
(1
)

.v
∆ =' a12 − 2b1= a1 − 4b1 + a12 ≥ 0 .
2 2
Điều đó chứng đó f(x) có nghiệm ta có đpcm.

om
1 1 1
Bài 5. Cho a,b,c là các số thực dương thoả mãn điều kiện + + =3.
a b c
Chứng minh rằng a+b + b+c + c+a ≥3 2 .
.c
Lời giải
ok
a = x 2 + z 2 − y 2
a+b b+c c+a 
bo

Đặt x = ,y= ,z = ⇒ b = x 2 + y 2 − z 2 .
2 2 2 
c = z + y − x
2 2 2
et

Ta cần chứng minh x + y + z ≥ 3 với điều kiện


vi

1 1 1
+ + =
3.
x +y −z
2 2 2
y +z −x
2 2 2
z + x2 − y 2
2
ng

Thật vậy giả sử x + y + z < 0. Khi đó sử dụng bất đẳng thức AM – GM cho ba
số dương ta có
a

1 1 1 3
+ 2 2 2+ 2 2 2≥ .
kh

x +y −z y +z −x z +x −y ( )( )( )
2 2 2
3 x2 + y 2 − z 2 y 2 + z 2 − x2 z 2 + x2 − y 2

Mặt khác
6
( x2 + y 2 − z 2 )( y 2 + z 2 − x2 )(  x+ y+z
z 2 + x2 − y 2 ≤ x2 y 2 z 2 ≤ 
 3
)
 < 1.

1 1 1
Suy ra 2 + 2 + 2 > 3 mâu thuẫn với giả thiết.
x +y −z
2 2
y +z −x
2 2
z + x2 − y 2
Vậy điều phản chứng là sai và ta có điều phải chứng minh.

48
Cty TNHH MTV DVVH Khang Việt

Ta cùng xét bài toán quen thuộc sau trích từ đề thi IMO 2001
Bài 6.( IMO 2001) Cho a,b,c là các số thực dương. Chứng minh rằng:
a b c
+ + ≥1.
a 2 + 8bc b 2 + 8ac c 2 + 8ba
Lời giải
a b c
= Đặt x = ;y = ;z ,( x; y; z ∈ (0;1))
a 2 + 8bc b 2 + 8ac c 2 + 8ba
a2 x2 b2 y2 c2 z2
Để ý =
rằng: = ; = ; .
8bc 1 − x 2 8ac 1 − y 2 8ba 1 − z 2

n
1 x2 y2 z2

.v
Suy ra =( )( )( ).
512 1 − x 2 1 − y 2 1 − z 2

om
Ta cần chứng minh x + y + z ≥ 1 với x, y, z ∈ (0;1) và
(1 − x 2 )(1 − y 2 )(1 − z 2 ) =
512( xyz )2 (1).
Giả sử ngược lại x + y + z < 1 .
.c
Theo bất đẳng thức AM – GM ta có:
ok
(1 − x 2 )(1 − y 2 )(1 − z 2 ) > [( x + y + z )2 − x 2 ][( x + y + z )2 − y 2 ][( x + y + z )2 − z 2 ]
= ( x + x + y + z )( y + z )( x + y + z + y )( z + x)( z + z + x + y )( x + y ) .
bo

1 1 1 1 1 1
≥ 4( x 2 yz ) 4 .2( yz ) 2 .4( y 2 zx) 4 .2( xz ) 2 .4( z 2 xy ) 4 .2( xy ) 2 =
512( xyz ) 2
et

Điều này mẫu thuẫn với (1).


Vậy điều phản chứng là sai và ta có điều phải chứng minh.
vi

Bài tập tương tự


Cho a,b,c là các số thực dương có tích bằng 1. Chứng minh
ng

1 1 1
+ + ≤ 1.
5a + 1 5b + 1 5c + 1
a

Bài 7. Cho a,b,c là các số thực dương thỏa điều kiện a + b + c + abc =4.
kh

Chứng minh rằng: a + b + c ≥ ab + bc + ca .


Lời giải
Giả sử ngược lại có a + b + c < ab + bc + ac .
Sử dụng bất đẳng thức Schur bậc 3 ta có(Xem chương 4).
9abc
≥ 4(ab + bc + ac) − (a + b + c) 2
a+b+c
> (a + b + c)[4 − (a + b + =
c)] abc(a + b + c)
⇒ a +b+c <3.
Khi đó abc < 1 ⇒ 4= abc + a + b + c < 4 . Mâu thuẫn.

49
Khám phá tư duy Kỹ thuật giải bất ĐT Bài toán Max – Min – Đặng Thành Nam

Suy ra bất đẳng thức đầu đúng.


Từ bất đẳng thức này ta chứng minh được một bất đẳng thức khó sau
Cho a,b,c là các số thực dương và k số thực thoả mãn điều kiện
3
 a  b  c   1
4k 2 + 2k − 1 ≥ 0 ta luôn có  k +  k +  k +  ≥ k +  .
 b + c  c + a  a+b  2

C. BÀI TẬP RÈN LUYỆN


Bài 1. Chứng minh rằng nếu a + b =2cd thì ít nhất một trong các bất đẳng thức
sau là đúng c 2 ≥ a; d 2 ≥ b .

n
Bài 2. Chứng minh rằng ít nhất một trong ba bất đẳng thức sau là đúng

.v
a 2 + b2 ≥
( b + c )2 ; b 2 + c 2 ≥ ( c + a )2 ; c 2 + a 2 ≥ ( a + b )2 .
2 2 2

om
Bài 3. Cho a,b,c là các số thực thuộc khoảng ( 0;2 ) . Chứng minh rằng ít nhất một
trong các bất đẳng thức sau là sai a ( 2 − b ) > 1; b ( 2 − c ) > 1; c ( 2 − a ) > 1 .
.c
Bài 4. Cho a,b,c là các số thực thoả mãn a + b + c > 0; ab + bc + ca > 0; abc > 0 .
Chứng minh rằng a > 0, b > 0, c > 0 .
ok
Bài 5. Chứng minh rằng nếu a1a2 ≥ 2 ( b1 + b2 ) thì ít nhất một trong hai phương
bo

trình sau có nghiệm x 2 + a1 x + b1= 0; x 2 + a2 x + b2= 0 .


Bài 6. Cho abc ≠ 0. Chứng minh ít nhất một trong ba phương trình sau có nghiệm
et

ax 2 + 2bx +=
c 0; bx 2 + 2cx +=
a 0; cx 2 + 2ax +=
b 0.
Bài 7. Cho f ( x) = x 2 + ax + b . Chứng minh rằng với mọi giá trị của a và b thì ít
vi

nhất một trong ba số


ng

1
f (0) , f (1) , f (−1) lớn hơn hoặc bằng .
2
a

Bài 8. Cho a,b,c là các số thực thoả mãn điều kiện


kh

1. a + b + c =−2; a 2 + b 2 + c 2 =2 .
 4 
2. Chứng minh rằng cả ba số a,b,c đều thuộc đoạn  − ;0  .
 3 
Bài 9. Cho a,b,c là các số thực thoả mãn điều kiện a 2 + b 2 + ab + bc + ca < 0 .
Chứng minh rằng a 2 + b 2 < c 2 .
Bài 10. Cho a,b,c là các số thực đôi một khác nhau chứng minh rằng ít nhất một
trong ba bất đẳng thức sau là sai
9ab ≥ ( a + b + c ) ;9bc ≥ ( a + b + c ) ;9ca ≥ ( a + b + c ) .
2 2 2

50
Cty TNHH MTV DVVH Khang Việt

Bài 11. Cho a,b,c,d là các số thực dương chứng minh ít nhất một trong các bất
đẳng thức sau là sai a + b < c + d ; ( a + b )( c + d ) < ab + cd ; ( a + b ) cd < ( c + d ) ab .
Bài 12. Cho a,b,c là các số thực dương có tích bằng 1. Chứng minh rằng:
1 1 1
+ + ≥1
1 + 8a 1 + 8b 1 + 8c
Bài 13. Cho a,b,c,d là các số thực chứng minh. Chứng minh rằng:
1
min{a − b 2 ; b − c 2 ; c − d 2 ; d − a 2 } ≤
4
Bài 14. (USAMO 2000 ) Cho a,b,c là các số thực dương.

n
Chứng minh rằng:
a+b+c 3
{ }

.v
− abc ≤ max ( a − b ) 2 ;( b − c ) 2 ;( c − a ) 2
3

om
Bài 15. Cho a,b,c là các số thực dương và k số thực thoả mãn điều kiện
3
 a  b  c   1
4k + 2k − 1 ≥ 0 ta luôn có  k +
2
 k +  k +  ≥ k +  .
 b + c  c + a 
.c a+b  2

D. HƯỚNG DẪN GIẢI – ĐÁP SỐ


ok
Bài 1. Giả sử cả hai bất đẳng thức đều sai khi đó
c 2 < a;
bo

d2 < b
Cộng theo vế hai bất đẳng thức trên ta được
et

c 2 + d 2 < a + b= 2cd ⇔ ( c − d ) < 0 vô lý.


2
vi

Vậy điều phản chứng là sai ta có điều phải chứng minh.


ng

Bài tập tương tự


Cho a,b,c,d là các số thực thoả mãn điều kiện ac ≥ 2 ( b + d ) . Chứng minh rằng
a

ít nhất một trong các bất đẳng thức sau là sai a 2 < 4b; c 2 < 4d .
kh

Bài 2. Giả sử tất cả các bất đẳng thức trên đều sai khi đó

a +b
2 2
<
( b + c)
2
;
2

b +c
2 2
<
( c + a)
2
;
2

c2 + a2 <
( a + b )2 .
2
Cộng theo vế ba bất đẳng thức trên ta được

51
Khám phá tư duy Kỹ thuật giải bất ĐT Bài toán Max – Min – Đặng Thành Nam

( a + b) + (b + c ) + (c + a )
2 2 2

(
2 a +b +c
2 2 2
) <
2
(
⇔ 2 a +b +c
2 2 2
) < 2 ( ab + bc + ca ) (vô lý).

⇔ ( a − b) + (b − c ) + (c − a ) < 0
2 2 2

Điều phản chứng là sai do đó ta có điều phải chứng minh.


Bài 3. Giả sử tất cả các bất đẳng thức trên là đúng khi đó
a ( 2 − b ) > 1; b ( 2 − c ) > 1; c ( 2 − a ) > 1 .
Nhân theo vế ba bất đẳng thức trên ta được a ( 2 − a ) b ( 2 − b ) c ( 2 − c ) > 1 (1).

n
Chú ý.

.v
0 < a ( 2 − a ) =− ( a − 1) + 1 ≤ 1
2

om
0 < b ( 2 − b ) =− ( b − 1) + 1 ≤ 1 .
2

0 < c ( 2 − c ) =− ( c − 1) + 1 ≤ 1
2

.c
Nhân theo vế ba bất đẳng thức trên ta được a ( 2 − a ) b ( 2 − b ) c ( 2 − c ) ≤ 1 mẫu
thuẫn với (1).
ok
Vậy điều phản chứng là sai(đpcm).
Bài 4. Giả sử tồn tại một số nhỏ hơn hoặc bằng 0 giả sử là a khi đó do abc > 0 nên
bo

a < 0; bc < 0 .
Khi đó ab + bc + ca= a ( b + c ) + bc > 0 ⇒ a ( b + c ) > −bc > 0 ⇒ b + c < 0 .
et

Suy ra a + b + c < 0 mâu thuẫn với giả thiết. Vậy điều phản chứng là sai(đpcm).
vi

a 2 − 4b1 < 0


Bài 5. Giả sử cả hai phương trình đều vô nghiệm khi đó  1 .
a2 − 4b2 < 0
2
ng

Cộng theo vế hai bất đẳng thức trên ta được 4 ( b1 + b2 ) > a12 + a22 .
a

Mặt khác a12 + a22 ≥ 2a1a2 ≥ 4 ( b1 + b2 ) .


kh

Suy ra 4 ( b1 + b2 ) > 4 ( b1 + b2 ) vô lý.


Vậy điều phản chứng là sai ta có điều phải chứng minh.
Bài 6. Giả sử cả ba phương trình đều vô nghiệm khi đó
∆ '1 = b 2 − ac < 0

∆ '2 = c − ab < 0 .
2


∆ '3 = a − bc < 0
2

Cộng theo vế ba bất đẳng thức trên ta được a 2 + b 2 + c 2 − ab − bc − ca < 0

52
Cty TNHH MTV DVVH Khang Việt

1 1 1
⇔ ( a − b )2 + ( b − c )2 + ( c − a )2 < 0 (vô lý).
2 2 2
Vậy điều phản chứng là sai ta có đpcm.
Bài 7. Giả sử không có số nào trong ba số f (0) , f (1) , f (−1) lớn hơn hoặc bằng
 1  1 1
 f (0) = b < 2  − < b < (1)
2 2
 
1  1  3 1
khi đó  f (1) = a + b + 1 < ⇔  − < a + b < − (2) .
2  2  2 2
 1  3 1
 f (−1) = − a + b + 1 < 2 − 2 < − a + b < − 2 (3)
 

n
Cộng theo vế của (2) và (3) ta được

.v
1
−3 < 2b < −1 ⇒ b < − mâu thuẫn với (1).

om
2
Vậy điều phản chứng là sai ta có đpcm.
 4 
Bài 8. Giả sử tồn tại một số không thuộc đoạn  − ;0  ta có thể giả sử là a khi đó
.c
 3 
b + c =−2 − a b + c =−2 − a b + c =−2 − a
ok
 2 ⇔  ⇔  .
( b + c ) − 2bc =− ( a + 1)2
2
b + c =2 − a bc =
2 2
2 a2
bo

4
Chú ý. ( b + c ) ≥ 4bc ⇒ ( −2 − a ) ≥ 4 ( a + 1) ⇔ 3a 2 + 4a ≤ 0 ⇔ − ≤ a ≤ 0 mâu thuẫn.
2 2 2
3
Vậy điều phản chứng là sai ta có điều phải chứng minh.
et

Bài 9. Giả sử ngược lại a 2 + b 2 ≥ c 2 khi đó


( )
vi

2 a 2 + b 2 + ab + bc + ca ≥ a 2 + b 2 + c 2 + 2 ( ab + bc + ca ) = ( a + b + c ) ≥ 0
2
ng

(mẫu thuẫn với giả thiết).


Vậy điều phản chứng là sai ta có đpcm.
Bài 10. Giả sử tất cả các bất đẳng thức trên là đúng cộng theo vế ba bất đẳng thức
a

ta được 9 ( ab + bc + ca ) ≥ 3 ( a + b + c )
2
kh

⇔ ( a + b + c ) − 3 ( ab + bc + ca ) ≤ 0
2

1 1 1
⇔ ( a − b )2 + ( b − c )2 + ( c − a )2 ≤ 0 ⇔ a = b = c
2 2 2
Điều này mâu thuẫn với giả thiết ba số a,b,c đôi một phân biệt.
Vậy điều phản chứng là sai và bài toán được chứng minh.
Bài 11. Giả sử tất cả các bất đẳng thức là đúng khi đó

53
Khám phá tư duy Kỹ thuật giải bất ĐT Bài toán Max – Min – Đặng Thành Nam

a + b < c + d (1)
( a + b )( c + d ) < ab + cd (2) .
( a + b ) cd < ( c + d ) ab (3)
Nhân theo vế của (1) và (2) ta được
( a + b )2 ( c + d ) < ( c + d )( ab + cd ) ⇔ ( a + b )2 < ab + cd
.
⇒ cd > a 2 + ab + b 2 = ( a − b ) + 3ab ≥ 3ab ⇒ cd > 3ab (4)
2

Nhân theo vế của (2) và (3) ta có


( a + b )2 cd ( c + d ) < ( c + d ) ab ( ab + cd )

n
⇒ ( a + b ) cd < ( ab + cd ) ab ⇒ ab ( ab + cd ) > 4abcd .
2

.v
⇒ ab + cd > 4cd ⇒ cd > 3ab (5)

om
Từ (4) và (5) ta có điều mâu thuẫn.
Vậy phản chứng là sai và ta có đpcm.
1 1 1
=
Bài 12. Đặt x = ;y = ;z .c
.
1 + 8a 1 + 8b 1 + 8c
ok
1 − x2 1 − y2 1 − z2
Dễ thấy 0 < x, y, z <=
1 và a = ;b = ; c .
8x 8 y2 8z 2
bo

Do abc=1 nên ta có 83 x 2 y 2 z 2 =
(1 − x 2 )(1 − y 2 )(1 − z 2 ) .
Theo đề bài ta cần chứng minh x + y + z ≥ 1 .
et

Giả sử ngược lại x + y + z < 1 .


Ta có:
vi

1 − x 2 > ( x + y + z )2 − x 2 =( z + y )[( x + y ) + ( x + z )] ≥ 2( y + z ) ( x + y )( x + z > 0 .


ng

Thiết lập tương tự với hai biểu thức còn lại ta có


1 − y 2 > 2( x + z ) ( y + z )( y + x) > 0
a

.
1 − z 2 > 2( y + x) ( x + z )( z + y ) > 0
kh

Nhân lại theo vế ba bất đẳng thức trên ta có:


83 x 2 y 2 z 2 =(1 − x 2 )(1 − y 2 )(1 − z 2 ) > 8( x + z )2 ( y + x)2 ( z + y )2
.
⇒ 8 xyz > ( x + y )( z + x)( y + z )
Điều này mâu thuẫn do theo bất đẳng thức AM – GM ta có
( x + y )( z + x)( y + z ) ≥ 8 xyz .
Do đó ta có điều phải chứng minh.
1
Bài 13. Giả sử min{a − b 2 ; b − c 2 ; c − d 2 ; d − a 2 } > .
4

54
Cty TNHH MTV DVVH Khang Việt

Suy ra a + b + c + d − (a 2 + b 2 + c 2 + d 2 ) > 1 .
Mặt khác:
(a + b + c + d )2
a 2 + b 2 + c 2 + d 2 − (a + b + c + d ) > − (a + b + c + d ) + 1 − 1
4 .
a+b+c+d
= ( − 1) − 1 > −1
2
2
Suy ra a + b + c + d − (a 2 + b 2 + c 2 + d 2 ) < 1 nên điều giả sử là sai .
Suy ra điều cần chứng minh.
Bài 14. Giả sử ngược lại

n
a+b+c 3
{
− abc > max ( a − b ) 2 ;( b − c ) 2 ;( c − a ) 2 }

.v
3
Suy ra a + b + c − 3 3 abc > 2(a + b + c − ab − bc − ca ) .

om
⇔ 2 ab + 2 bc + 2 ca > a + b + c + 3 3 abc
Đổi biến lại x3 + y 3 + z 3 + 3 xyz < 2 ( xy )3 + 2 ( yz )3 + 2 ( zx)3 .
.c
Nhưng theo bất đẳng thức Schur ta có:
ok
x3 + y 3 + z 3 + 3 xyz ≥ xy ( x + y ) + yz ( y + z ) + zx( z + x)
≥ 2 ( xy )3 + 2 ( yz )3 + 2 ( zx)3
bo

Suy ra mâu thuẫn, vậy giả sử sai, suy ra điều cần chứng minh.
et

2a 2b 2c
Bài 15. Đặt x= , y= ,=
z ⇒ xy + yz + zx + xyz= 4 .
b+c c+a a+b
vi

Ta có xyz ≤ 1; x + y + z ≥ xy + yz + zx .
ng

Bất đẳng thức trở thành: ( 2k + x )( 2k + y )( 2k + z ) ≥ ( 2k + 1)


3

⇔ 4k 2 ( x + y + z ) + 2k ( xy + yz + zx ) + xyz ≥ 12k 2 + 6k + 1 .
a

Ta có 4k 2 ( x + y + z ) + 2k ( xy + yz + zx ) + xyz
kh

≥ 4k 2 ( xy + yz + zx ) + 2k ( xy + yz + zx ) + xyz
= ( 4k 2
+ 2k ) ( xy + yz + zx ) + xyz= ( 4k 2
)
+ 2k ( 4 − xyz ) + xyz

= 16k 2 + 8k − xyz ( 4k + 2k − 1)
2

≥ 16k 2 + 8k − 4k 2 − 2k + 1= 12k 2 + 6k + 1
Bất đẳng thức được chứng minh.
Đẳng thức xảy ra khi và chỉ khi a= b= c

55
Khám phá tư duy Kỹ thuật giải bất ĐT Bài toán Max – Min – Đặng Thành Nam

CHỦ ĐỀ 3: KỸ THUẬT QUY NẠP TOÁN HỌC

A. NỘI DUNG PHƯƠNG PHÁP


Giả sử ta cần chứng minh bất đẳng thức A(n) ≥ 0 với A(n) là một biểu thức có
chứa số nguyên dương n với n ≥ n0 .
Ta thực hiện chứng minh bằng quy nạp như sau
 Chứng minh bất đẳng thức đúng với giá trị đầu tiên của n là n0 .
 Giả sử bất đẳng thức đúng với n= k > n0 tức A(k ) ≥ 0 .
 Sau đó chứng minh A(k + 1) ≥ 0 dựa vào A(k ) ≥ 0 .

n
 Kết luận bất đẳng thức đúng.

.v
B. BÀI TẬP MẪU
Ví dụ 1. Cho n là số nguyên dương n ≥ 5. Chứng minh rằng 2n > n 2 .

om
Lời giải
+ Với n = 5;25 = 32 > 52 = 25 . Vậy bất đẳng thức đúng với n = 5 .
+ Giả sử bất đẳng thức đúng với n= k > 5 tức 2k > k 2 .
.c
Khi đó
ok
2k +1 = 2.2k > 2k 2 = ( k + 1)2 + k 2 − 2k − 1 = ( k + 1)2 + k ( k − 5) + 3k − 1 > ( k + 1)2 .
Vậy bất đẳng thức đúng với n= k + 1 .
bo

Theo nguyên lý quy nạp ta có điều phải chứng minh.


Bài tập tương tự
et

n −1
Cho n là số nguyên dương lớn hơn 1. Chứng minh n n > ( n + 1) .
vi

Bài 2. Cho a,b là các số thực không âm và n là số nguyên dương. Chứng minh rằng
ng

n
a n + bn  a + b 
≥  .
2  2 
a

Lời giải
+ Với n = 1 bất đẳng thức trở thành đẳng thức.
kh

k
a k + bk  a + b 
+ Giả sử bất đẳng thức đúng với n= k > 1 tức ≥  .
2  2 
k +1
a k +1 + b k +1  a + b 
+ Ta cần chứng minh ≥  .
2  2 
Chú ý
k +1 k
a+b  a + b  a + b  a + b a k + bk
=
     ≤ . .
 2   2  2  2 2

56
Cty TNHH MTV DVVH Khang Việt

a k +1 + b k +1 a + b a k + b k
Vậy ta chứng minh ≥ . ⇔ a k +1 + b k +1 ≥ ab k + ba k
2 2 2
( )
⇔ a a k − bk + bk ( b − a ) ≥ 0 ⇔ ( a − b ) a k − bk ≥ 0 ( )
Bất đẳng thức cuối luôn đúng.
Vậy theo nguyên lý quy nạp ta có điều phải chứng. Đẳng thức xảy ra khi và chỉ
khi a = b .
Tổng quát với n số thực không âm và m là một số nguyên dương ta có
m
a1m + a2m + ... + anm  a1 + a2 + ... + an 
≥  .
n  n 

n
Bài 3. Cho số nguyên dương M > 3. Giả sử x1 , x2 ,..., x2014 là các số nguyên dương

.v
sao cho x1.x2 .....x2014 = M . Tìm giá trị lớn nhất của biểu thức:

om
S = x13 + x23 + ... + x2014
3
.
Lời giải
Ta chứng minh với n là số nguyên dương x1 , x2 ,..., xn thoả mãn điều kiện
.c
x1 x2 ...xn = M thì x13 + x23 + ... + xn3 ≤ ( x1 x2 ...xn ) + n − 1 .
3
ok
Và bài toán chính là trường hợp riêng khi n bằng 2014.
+ Với n bằng 1 bất đẳng thức chính là hằng đẳng thức.
bo

( )( )
+ Với n bằng 2 ta có x13 − 1 x23 − 1 ≥ 0 ⇒ x13 + x23 ≤ 1 + x13 x23 .
Vậy bất đẳng thức đúng với n bằng 1,2.
et

+ Giả sử bất đẳng thức đúng với n= k > 2 tức


vi

x13 + x23 + ... + xk3 ≤ ( x1 x2 ...xk ) + k − 1 (1).


3
ng

Ta cần chứng minh x13 + x23 + ... + xk3+1 ≤ ( x1 x2 ...xk +1 ) + k .


3

Do (1) nên ta chỉ cần chứng minh xk3+1 + ( x1 x2 ...xk ) ≤ 1 + ( x1 x2 ...xk xk +1 )


3 3
a

(
⇔ xk3+1 − 1) (( x x ...x ) − 1) ≥ 0
3
kh

1 2 k

Bất đẳng thức cuối đúng.


Theo nguyên lý quy nạp ta có điều phải chứng minh.
Vậy giá trị lớn nhất của S bằng M 3 + 2013 đạt tại một số bằng M và 2013 số
bằng 1.
Bài 4. Cho n là số nguyên dương xk ≥ 1, k =
1, n. Chứng minh rằng
1 1 1 n
+ + ... + ≥ .
1 + x1 1 + x2 1 + xn 1 + x1 x2 ...xn
n

57
Khám phá tư duy Kỹ thuật giải bất ĐT Bài toán Max – Min – Đặng Thành Nam

Lời giải
+ Với n = 1 bất đẳng thức trở thành đẳng thức.
+ Giả sử bất đẳng thức đúng với n= k > 1, tức
1 1 1 k
+ + ... + ≥ (1).
1 + x1 1 + x2 1 + xk 1 + x1 x2 ...xk
k

+ Ta cần chứng minh


1 1 1 1 k +1
+ + ... + + ≥ .
1 + x1 1 + x2 1 + xk 1 + xk +1 1 + k +1 x1 x2 ...xk +1
Do (1) nên ta chỉ cần chứng minh

n
k 1 k +1
+ ≥ (2).

.v
1 + k x1 x2 ...xk 1 + xk +1 1 + k +1 x1 x2 ...xk +1
k −1

om
1 k
Chú ý + ≥
1 + xk +1 1 + k +1 x1 x2 ...xk +1 k −1 2 k
( )
k
1+ x1 x2 ...xk k +1 xkk++11
k
+
k

.c 2k
k −1 2k 1 + k x1 x2 ...xk 1 + k +1 x1 x2 ...xk +1
ok
( x1 x2 ...xk ) k +1 xkk++11
k
1+
Cộng theo vế hai bất đẳng thức trên ta có (2). Tức bất đẳng thức đúng với
bo

n= k + 1 .
Vậy theo nguyên lý quy nạp ta có điều phải chứng minh. Đẳng thức xảy ra khi
et

và chỉ khi x1= x2= ...= xn .


Bài 5. (VMO 2011) Chứng minh rằng với mọi số thực dương x và số nguyên
vi

dương n ta có
(
x n x n +1 + 1 ) ≤  x +1 2 n +1
ng

  .
xn + 1  2 
Đẳng thức xảy ra khi nào?
a

Lời giải
( )
kh

 x +1
3 x x2 + 1
+ Với n = 1 bất đẳng thức trở thành   ≥
 2  x +1

( )
⇔ ( x + 1) − 8 x x 2 + 1 ≥ 0 ⇔ ( x − 1) ≥ 0 .
4 4

Vậy bất đẳng thức đúng với n = 1 .

+ Giả sử bất đẳng thức đúng với n = k tức


(
x k x k +1 + 1 ) ≤  x +1 2 k +1

  (1).
xk + 1  2 
Ta cần chứng minh

58
Cty TNHH MTV DVVH Khang Việt

(
x k +1 x k + 2 + 1 ) ≤  x +1 2 k +3

  .
x k +1 + 1  2 
Do (1) nên ta chỉ cần chứng minh

 x +1 x x +1 x
2 k
(
k +1

k +1 k + 2
x +1 ) ( )
  +
 2  x +1
k
x +1
k 1

( ) ( ) ( )
2
⇔ ( x + 1) x k x k +1 + 1 − 4 x k + 1 x k +1 x k + 2 + 1 ≥ 0 .
2

(x )
2
⇔ ( x − 1) k +1
2
−1 ≥ 0

n
Bất đẳng thức luôn đúng. Vậy theo nguyên lý quy nạp ta có điều phải chứng minh.

.v
Đẳng thức xảy ra khi và chỉ khi x = 1 .
Để kết thức tôi trình bày lời giải chứng minh bất đẳng thức AM – GM bằng

om
quy nạp.
Bài 6. (BĐT AM – GM ) Cho a 1 , a 2 , …, a n là các số thực không âm.
Chứng minh rằng ta luôn có a1 + a2 + ... + an ≥ n n a1a2 ...an .c
Lời giải
ok
Cơ sở quy nạp với n = 1, 2 được kiểm tra dễ dàng.
Giả sử bất đẳng thức đã được chứng minh cho n số.
bo

Xét n+1 số không âm a 1 , a 2 , …, a n+1 . Đặt a 1 a 2 …a n+1 = An+1. Nếu tất cả các số
bằng nhau thì bất đẳng thức đúng.
Trong trường hợp ngược lại, phải tồn tại hai số a i , a j sao cho a i < A < a j .
et

Không mất tính tổng quát, có thể giả sử a n < A < a n+1 .
Khi đó ta có (a n – A)(a n+1 – A) < 0, suy ra a n + a n+1 > a n a n+1 /A + A.
vi

Từ đó ta có a 1 + a 2 + …+ a n + a n+1 > a 1 + … + a n-1 + a n a n+1 /A + A (1)


ng

Bây giờ áp dụng bất đẳng thức Cauchy cho n số a 1 + … + a n-1 + a n a n+1 /A ta
an an +1
được a1 + a2 + ... + an −1 + an ≥ n n a1a2 ...an −1 =
nA .
a

A
Kết hợp với (1) ta có điều phải chứng minh.
kh

C. BÀI TẬP RÈN LUYỆN


Bài 1. Cho a,b,c là độ dài ba cạnh một tam giác vuông có c là cạnh huyền. Chứng
minh a 2 n + b 2 n ≤ c 2 n với mọi số nguyên dương n.

Bài 2. Cho n số thực không âm x1 , x2 ,..., xn thoả mãn điều kiện


2 ( x1 + x2 + ... + xn ) ≤ 1 .

59
Khám phá tư duy Kỹ thuật giải bất ĐT Bài toán Max – Min – Đặng Thành Nam

1
Chứng minh rằng (1 − x1 )(1 − x2 ) ... (1 − xn ) ≥
.
2
Bài 3. Chứng minh rằng với n số nguyên dương phân biệt ta có
a13 + a23 + ... + an3 ≥ ( a1 + a2 + ... + an ) .
2

Bài 4. Cho x 1 , x 2 , …, x n là các số thực thuộc [0, 1]. Chứng minh rằng
x 1 (1-x 2 ) + x 2 (1-x 3 ) + … + x n (1-x 1 ) ≤ [n/2]
Bài 5. Cho n ≥ 2 và x 1 , x 2 , …, x n là n số nguyên phân biệt.
Chứng minh rằng (x 1 -x 2 )2 + (x 2 -x 3 )2 + … + (x n – x 1 )2 ≥ 4n – 6
Bài 6. Chứng minh rằng với x 1 ≥ x 2 ≥ … ≥ x n ≥ 0 ta có bất đẳng thức

n
n n
xi
∑ xi2 ≤ ∑

.v
.
=i 1 =i 1 i

om
Bài 7. Chứng minh rằng nếu a 1 , a 2 , …, a n là các số nguyên dương phân biệt thì ta
2
n  n 3
có bất đẳng thức 7

(ai + ai ) ≥ 2  ai 
5
∑ .
=i 1 = i 1  .c
ok
Bài 8. (Bất đẳng thức Mc-Lauflin) Với mọi số thực a 1 , a 2 , …, a 2n và b 1 , b 2 , …, b 2n
2 2
2n2n  n   2n 
ta có bất đẳng ∑ ∑ ∑
ak2 bk2 −  (a2 k b2 k −1 − a2 k −1b2 k )  ≥  ∑ ak bk  .
bo

thức
=k 1= k 1 = k 1  = k 1 
et

D. HƯỚNG DẪN GIẢI – ĐÁP SỐ


Bài 1. Với n = 1 bất đẳng thức trở thành đẳng thức.
vi

Giả sử bất đẳng thức đúng với n= k > 1 tức a 2 k + b 2 k ≤ c 2 k (1).


ng

Ta cần chứng minh a ( ) + b ( ) ≤ c ( ) .


2 k +1 2 k +1 2 k +1

(
Chú ý do có (1) nên ta chỉ cần chứng minh c 2 a 2 k + b 2 k ≥ a ( ) + b ( )
2 k +1 2 k +1
)
a

( )( )
⇔ a 2 + b 2 a 2 k + b 2 k ≥ a ( ) + b ( ) ⇔ a 2b 2 k + b 2 a 2 k ≥ 0 .
kh

2 k +1 2 k +1

Bất đẳng thức. Vậy theo nguyên lý quy nạp ta có điều phải chứng minh.

CHỦ ĐỀ 4: KỸ THUẬT MIỀN GIÁ TRỊ

A. GIỚI THIỆU
Phương pháp này khá cơ bản và thường được sử dụng trong các bài toán tìm
cực trị với các bất đẳng thức có dạng: a ≤ f ( x) ≤ b với x ∈ D .

60
Cty TNHH MTV DVVH Khang Việt

Nguyên tắc chung là đưa về tìm điều kiện để phương trình m = f ( x) có nghiệm
trên D.
Trong trường hợp có nhiều biến ta cần đưa về phương trình với một biến hoặc
đưa về hệ phương trình nếu có.

B. NỘI DUNG PHƯƠNG PHÁP


1) Tìm miền giá trị bằng cách xét điều kiện có nghiệm của phương trình
bậc hai
Một phương trình bậc hai có dạng: Ax 2 + Bx + C =
0 với A ≠ 0 .
Điều kiện để phương trình có nghiệm là: ∆
= B 2 − 4 AC ≥ 0 .

n
Vì vậy cần tìm giá trị lớn nhất và giá trị nhỏ nhất của một biểu thức

.v
m = f ( x, y , z ) .

om
Ta biến đổi tương đương m để đưa về một phương trình bậc của x hoặc y hoặc
của z.
Khi đó sử dụng điều kiện có nghiệm ta tìm được Max và Min của m.
.c
 x − y + z =
Ví dụ 1. Cho x,y,z là các số thực thoả mãn điều kiện  2
3
.
 x + y + z =
2 2
5
ok
x+ y−2
Tìm giá trị lớn nhất và nhỏ nhất của biểu thức P = .
z+2
bo

Lời giải
Trong biểu thức của P có z khác so với x và y do vậy ta tìm cách rút x + y theo
et

z và đưa về phương trình bậc hai đối với z. Việc tìm Max và Min của P ta chặn
bằng điều kiện có nghiệm của phương trình bậc hai đối với z.
vi

Ta có ( z + 2 ) P = x + y − 2 ⇒ ( z + 2 ) P + 2  = ( x + y )2 .
2
ng

Chú ý

 x − y = 3 − z  x − y =3 − z
a


2 ⇒ ( x + y) =
2
 2 ⇔ 1 2 1 −3z 2 + 6 z + 1 .
 2 ( ) ( ) 2
+ + − =−
kh

 x + y =5 − z
2 2
x y x y 5 z
2
Vì vậy
( z + 2 ) P + 2  =−3 z 2 + 6 z + 1
2

⇔ ( z + 2 ) P 2 + 4 ( z + 2 ) P + 3z 2 − 6 z + 3 =
2
0 .

( ) ( )
⇔ P 2 + 3 z 2 + 4 P 2 + 4 P − 6 z + 4 P 2 + 8P + 3 =0 (1)
Ta có (1) là phương trình bậc hai đối với z điều kiện có nghiệm là

61
Khám phá tư duy Kỹ thuật giải bất ĐT Bài toán Max – Min – Đặng Thành Nam

( 2P ) − (P )( )
2
=
∆' 2
+ 2P − 3 2
+ 3 4 P 2 + 8P + 3 ≥ 0
36
⇔ 23P 2 + 36 P ≤ 0 ⇔ −
≤P≤0
23
+ Với=
x 2,=
y 0,=
z 1 ta có P bằng 0. Vậy giá trị lớn nhất của P bằng 0.
20 66 7
+ Với x = ,y= − , z =thì P bằng -36/23. Vậy giá trị nhỏ nhất của P bằng
31 31 31
36
− .
23
Bài tập tương tự

n
 x − y + z =3

.v
Cho x,y,z là các số thực thoả mãn điều kiện  2 .
 x + y + z =
2 2
5

om
x + y −1
Tìm giá trị lớn nhất và nhỏ nhất của biểu thức P = .
z+2
3− 2 3 3+ 2 3 .c
Đáp số: ≤P≤ .
3 3
2) Kỹ thuật sử dụng điều kiện có nghiệm của phương trình lượng giác
ok
Phương trình dạng: A sin x + B cos x =
C
bo

có nghiệm khi và chỉ khi A2 + B 2 ≥ C 2 .


2 cos x + 2sin x + 3
Ví dụ 1. Chứng minh rằng với mọi số thực x ta có ≤ ≤2.
11 2cos x − sin x + 4
et

Lời giải
vi

cos x + 2sin x + 3
Đặt y= ⇒ y ( 2cos x − sin x + 4 )= cos x + 2sin x + 3 .
2cos x − sin x + 4
ng

⇔ ( 2 y − 1) cos x − ( y + 2 ) sin x =3 − 4 y (1) .

Điều kiện để (1) có nghiệm là ( 2 y − 1)2 + ( y + 2 )2 ≥ ( 3 − 4 y )2


a
kh

2
⇔ 11 y 2 − 24 y + 4 ≤ 0 ⇔ ≤ y≤2
11
Bài toán được chứng minh.
3) Kỹ thuật điều kiện có nghiệm của phương trình - hệ phương trình
Thay vì tìm trực tiếp Max và Min của biểu thức ta đưa về tìm điều kiện có
nghiệm của hệ phương trình.
Ví dụ 1. Cho x,y là hai số thực thoả mãn điều kiện

(x )
2
2
− y 2 + 1 + 4 x2 y 2 − x2 − y 2 =
0.

62
Cty TNHH MTV DVVH Khang Việt

3− 5 3+ 5
Chứng minh rằng ≤ x2 + y 2 ≤ .
2 2
Lời giải
Đặt m
= x 2 + y 2 . Ta cần tìm Max và Min của m.
Thay vì đi chứng minh trực tiếp ta tìm điều kiện để hệ phương trình sau có
m= x2 + y 2

nghiệm 
( )
2
 x − y + 1 + 4 x y − x − y =
2 2 2 2 2 2
0

m
= x2 + y 2
⇔

n
 x + y + x − 3 y + 2 x y + 1 =
4 4 2 2 2 2
0

.v
m = x2 + y 2

om
⇔
(
 x + y
2
)
2 2
( )
− 3 x2 + y 2 + 4 x2 + 1 = 0

 2 m 2 − 3m + 1
= −
⇔
m
= x2 + y 2
⇔


x
4
.c
ok
m − 3=m + 1 + 4x 0  2 m + m + 1
2 2 2

 y = 4
Điều kiện để hệ phương trình có nghiệm là
bo

 m 2 − 3m + 1
− ≥0
3− 5 3+ 5

et

4
 2 ⇔ ≤m≤ .
m + m +1 2 2
 ≥0
vi

4
3− 5 3+ 5
ng

Vì vậy ≤ x2 + y 2 ≤ .
2 2
Bài tập tương tự
a

Cho x,y là các số thực thoả mãn điều kiện x, y ≥ 1;4 x 2 y 2 − x 2 − y 2= 18 xy − 16 .


kh

Tìm giá trị lớn nhất và giá trị nhỏ nhất của biểu thức
2 x+ y
P= .
x2 + x + y 2 + y + 5 + x + y
Đáp số: Pmin =16 ( )
17 − 4 ; Pmax = 64 ( 37 − 6 . )
Ví dụ 2. Cho x,y là các số thực thay đổi thoả mãn điều kiện
x + y −=
2 2x + 1 + 2 y + 1 .

=
Tìm giá trị lớn nhất của biểu thức P ( x − y )2 .
63
Khám phá tư duy Kỹ thuật giải bất ĐT Bài toán Max – Min – Đặng Thành Nam

Lời giải
 u2 −1
u  x =
= 2x + 1 
, ( u, v ≥ 0 ) ⇒ 
Đặt  2 .
= v 2y +1  v −1
2

 y = 2

u 2 + v2 − 2
Điều kiện bài toán trở thành − 2 = u + v ⇔ u 2 + v2 − 6 = 2 (u + v )
2
⇔ ( u + v ) − 2 ( u + v ) − 2uv − 6 =
2
0.

( )
2

n
 u 2 − 1 v2 − 1 
2
u 2 − v2
Khi đó P = −  = .

.v
 2 2  4

 

om
( u − v )2 ( u + v )2 ( u + v ) − 4uv  ( u + v )
2 2

= =
4 4
( u + v )2 − 2 ( u + v ) − 2uv − 6 =
.c 0 (1)

Vậy ta có hệ phương trình  ( u + v )2 − 4uv  ( u + v )2 .
ok
 P=   (2)
 4
bo

1
Từ (1) ta có 2uv = ( u + v ) − 2 ( u + v ) − 6 ≤ ( u + v )
2 2
2
⇒ ( u + v ) − 4 ( u + v ) − 12 ≤ 0 ⇔ −2 ≤ u + v ≤ 6 .
et

Chú ý u + = 2( x + y) + 2 + 2 ( 2 x + 1)( 2 y + 1) ≥ 2( x + y) + 2 ≥ 6 .
vi

Vì vậy ta có t = u + v ∈  6;6  .
ng

{(u + v ) 2

2
}
− 2 ( u + v ) − 2 ( u + v ) − 6  ( u + v )

2
a

Khi đó P=
4
kh

(
 − ( u + v )2 + 4 ( u + v ) + 12  ( u + v )2 t 2 −t 2 + 4t + 12
=  )
4 4

Xét hàm số f (t ) =
(
t 2 −t 2 + 4t + 12 ) liên tục trên đoạn  
4  6;6  ta có

( )  
f '(t )= t −t 2 + 3t + 6 ; f '(t )= 0 ←→ t=
t∈ 6;6  3 + 33
2
.

64
Cty TNHH MTV DVVH Khang Việt

Ta có f ( 6) =
 3 + 33  3 69 + 11 33
9 + 6 6; f 
( )
 = ; f (6) =
0.
 2  8

Vì vậy P= f=
 3 + 33  3 69 + 11 33
f  =
( )
max max  .
 2  8
Dấu bằng xảy ra khi và chỉ khi
 3 + 33  7 + 33
=u + v = x + y
 2  2
 ⇔
 x + y − 2 3 + 33  2= 3 + 33

n
=  x +1 + 2y +1
2  2

.v
 14 + 2 33 − 414 + 66 33 14 + 2 33 + 414 + 66 33
x = ,y

om
⇔
8 8

 x 14 + 2 33 + 414 + 66 33 14 + 2 33 − 414 + 66 33
= ,y
 8 8 .c
(
3 69 + 11 33 ).
ok
Vậy giá trị lớn nhất của P bằng
8
Ví dụ 3. (TSĐH Khối A 2006) Cho x,y là 2 số thực và x ≠ 0, y ≠ 0 thỏa mãn điều
bo

kiện xy ( x + y ) = x 2 + y 2 − xy .
1 1
Tìm giá trị lớn nhất của biểu thức = +
et

A 3
.
x y3
vi

Lời giải
x + y = u
ng

Nhận thấy x, y đối xứng nên đặt:  .


 x. y = v
u2
a

Giả thiết bài toán trở thành: u.v = u 2 − 3v ⇔ v = (do u ≠ −3 )


u+3
kh

2
1 1 x3 + y 3 u (u 2 − 3v) u +3
u2
Ta có + = = = 2 = 
x3 y3 ( xy )3 v3 v  u 

4u 2 4 u −1 u ≥ 1
Vì u 2 ≥ 4v ⇒ u 2 ≥ ⇔ ≤1⇔ ≥0⇔ .
u+3 u+3 u+3 u ≤ −3
u+3
Chú ý để tìm Max của A ta chỉ cần xét với > 0 nên ta chỉ cần chứng minh
u
u+3
Max của A bằng 16 vậy : ≤ 4 với u ≥ 1 hoặc u ≤ −3 .
u
65
Khám phá tư duy Kỹ thuật giải bất ĐT Bài toán Max – Min – Đặng Thành Nam

u+3 −3
Xét hàm số f(u) = ⇒ f’(u) = 2 < 0
u u
Trên mỗi khoảng ( −∞; −3 ) và [1; +∞) do đó f(u) ≤ f(1); ∀u ≥ 1 .
Còn 0 < f(-3) < f(u) <1, ∀ u > -3.
Vậy giá trị lớn nhất của A bằng 16.
C. BÀI TẬP RÈN LUYỆN
1 2sin x + cos x + 1
Bài 1. Chứng minh rằng với mọi số thực x ta có − ≤ ≤ 2.
2 sin x − 2cos x + 3
Bài 2. Chứng minh rằng với mọi số thực x và y ta có

n
2
 cos3 x + y sin 3 x + 1  3y2 + 2 + 2 3y2 + 1
  ≤ .

.v
 cos3 x + 2  9
Bài 3. Chứng minh rằng với mọi số thực x,y thoả mãn điều kiện

om
( )
2
5 x2 + y 2 x2 + y 2
x − y=
2 2
+ 3. − .
4 2 2
.c
5− 5 5+ 5
Chứng minh rằng ≤ x2 + y 2 ≤ .
ok
2 2
Bài 4. Cho x,y là các số thực thoả mãn điều kiện x 2 + xy + y 2 ≤ 3 .
bo

Chứng minh rằng −4 3 − 3 ≤ x 2 − xy − 3 y 2 ≤ 4 3 − 3 .


et

 x + y + z =3
Bài 5. Cho x,y,z là các số thực thoả mãn điều kiện  2 .
 x + y + z =
2 2
vi

5
x− y+3
ng

Tìm giá trị lớn nhất, giá trị nhỏ nhất của biểu thức P = .
z+2
Bài 6. Cho x,y là hai số thực thay đổi thoả mãn điều kiện 2x + 3 + y + 3 =4.
a

Tìm giá trị lớn nhất và giá trị nhỏ nhất của biểu thức P = x+2 + y+9 .
kh

D. HƯỚNG DẪN GIẢI – ĐÁP SỐ


2sin x + cos x + 1
Bài 1. Đặt y = ⇒ ( 2 − y ) sin x + ( 2 y + 1) cos x = 3 y − 1 .
sin x − 2cos x + 3
Điều kiện để phương trình có nghiệm là: ( 2 − y ) + ( 2 y + 1) ≥ ( 3 y − 1)
2 2 2

1
⇔ 4 y2 − 6 y − 4 ≤ 0 ⇔ − ≤ y ≤ 2 .
2
Bài toán được chứng minh.

66
Cty TNHH MTV DVVH Khang Việt

cos3 x + y sin 3 x + 1
Bài 2. Đặt m = ⇒ ( m − 1) cos3 x − y sin 3 x =−
1 2m .
cos3 x + 2
Điều kiện để phương trình có nghiệm là ( m − 1) + y 2 ≥ (1 − 2m )
2 2

1 − 3y2 + 1 1 + 3y2 + 1
⇔ 3m 2 − 2m − y 2 ≤ 0 ⇔ ≤m≤ .
3 3
2
1 + 3y2 + 1  1 + 3y2 + 1  3y2 + 2 + 2 3y2 + 1
Suy ra m ≤ ⇒ m2 ≤   = .
3  3  9
 
Bài toán được chứng minh.

n
 2 m 2 − 5m + 5

.v
 x = −
 4
Bài 3. Tìm được:  .

om
 2 m −m+5
2

 y = 4
Điều kiện để hệ phương trình có nghiệm là: .c
 m 2 − 5m + 5
− ≥0
ok
 4 5− 5 5+ 5
 2 ⇔ m 2 − 5m + 5 ≤ 0 ⇔ ≤m≤ .
m − m + 5 2 2
 ≥0
bo

4
Bài toán được chứng minh.
Bài 4. Nếu y = 0 bất đẳng thức luôn đúng.
et

x 2 − xy − 3 y 2 t 2 − t − 3
y ≠ 0 đặt m
Xét với = =
vi

.
x 2 + xy + y 2 t 2 + t + 1

( )
ng

Suy ra m t 2 + t + 1 = t 2 − t − 3 ⇔ ( m − 1) t 2 + ( m + 1) t + m + 3 = 0 .

Điều kiện để phương trình có nghiệm là ∆= ( m + 1)2 − 4 ( m − 1)( m + 3) ≥ 0


a

−3 − 4 3 −3 + 4 3
kh

⇔ 3m 2 − 6m − 13 ≤ 0 ⇔ ≤m≤ .
3 3
Chú ý do 0 < x 2 + xy + y 2 ≤ 3 ⇒ −4 3 − 3 ≤ x 2 − xy − 3 y 2 ≤ 4 3 − 3 .
Bài toán được chứng minh.
 x + y = 5
Bài 5. Chú ý z ≠ −2 vì ngược lại ta có  2 vô nghiệm.
 x + y =
2
1
Khi đó ( z + 2 ) P = x − y + 3 ⇔ ( z + 2 ) P − 3 = x − y .
Từ hệ điều kiện ta có

67
Khám phá tư duy Kỹ thuật giải bất ĐT Bài toán Max – Min – Đặng Thành Nam

 x + y = 3 − z  x + y =3 − z

2 ⇒ ( x − y) =
2
 2 ⇔ 1 2 1 −3z 2 + 6 z + 1 .
 x + y =5 − z
2 2
( + ) + ( − ) 2
=−
 2
x y x y 5 z
2
Khi đó ( z + 2 ) P − 3 =−3 z 2 + 6 z + 1
2

⇔ ( z + 2 ) P 2 − 6 ( z + 2 ) P + 3z 2 − 6 z + 8 =
2
0

( ) ( )
⇔ P 2 + 3 z 2 + 4 P 2 − 6 P − 6 z + 4 P 2 − 12 P + 8 =0
Điều kiện phương trình có nghiệm là

( 4P ) − (P )( )
2
2
− 3P − 3 2
+ 3 4 P 2 − 12 P + 8 ≥ 0

n
.v
27 − 8 6 27 + 8 6
⇔ 23P 2 − 54 P + 15 ≤ 0 ⇔ ≤P≤
23 23

om
27 − 8 6 27 + 8 6
=
Vì vậy Pmin = ; Pmax .
23 23

Bài 6. Thực hiện tương tự bài tập mẫu: Pmax =


2 3 10
+ 22; Pmin = .
.c
2 2
ok
CHỦ ĐỀ 5: KỸ THUẬT SỬ DỤNG NGUYÊN LÝ DIRICLE
bo
et

BÀI TẬP MẪU


Bài 1. Cho x,y,z là các số thực dương thoả mãn điều kiện x + y + z + 1 =4 xyz .
vi

Chứng minh rằng xy + yz + zx ≥ x + y + z .


ng

Lời giải
Theo nguyên lý Dirichlet tồn tại 2 trong 3 số ( x − 1),( y − 1),( z − 1) cùng dấu,
không mất tính tổng quát giả sử ( x − 1)( y − 1) ≥ 0 ⇔ xy ≥ x + y − 1 .
a

⇒ xy + yz + zx ≥ x + y − 1 + yz + zx .
kh

Vậy ta cần chứng minh x + y − 1 + yz + zx ≥ x + y + z ⇔ z ( x + y − 1) ≥ 1 .


x + y +1
Theo giả thiết ta có z = .
4 xy − 1
Vậy ta chứng minh
x + y +1
. ( x + y − 1) ≥ 1 ⇔ ( x + y ) − 1 ≥ 4 xy − 1 ⇔ ( x − y ) ≥ 0 .
2 2
4 xy − 1
Bất đẳng thức được chứng minh. Đẳng thức xảy ra khi và chỉ khi x= y= z= 1 .

68
Cty TNHH MTV DVVH Khang Việt

Bài tập tương tự


Cho a,b,c là các số thực dương thoả mãn điều kiện a + b + c + abc =4.
Chứng minh rằng a + b + c ≥ ab + bc + ca .

Bài 2. Cho a,b,c là các số thực không âm thỏa mãn điều kiện a + b + c =3.
( )( )(
Chứng minh rằng 1 − a + a 2 1 − b + b 2 1 − c + c 2 ≥ 1 . )
Lời giải
Trong ba số 1 − a,1 − b và 1 − c luôn tồn tại hai số cùng dấu không mất tính tổng
quát giả sử hai số đó là 1 − b và 1 − c suy ra (1 − b )(1 − c ) ≥ 0 .

n
( )(
Khi đó 1 − b + b 2 1 − c + c 2 = ) (b 2
)( )
− b c2 − c + b2 + c2 − b − c + 1

.v
= bc ( b − 1)( c − 1) + b 2 + c 2 − b − c + 1

om
1
≥ b2 + c2 − b − c + 1 ≥ ( b + c )2 − ( b + c ) + 1
2
1 .c a 2 − 4a + 5
( 3 − a )2 − ( 3 − a ) +=1
=
2 2
ok
Vì vậy bất đẳng thức được chứng minh nếu ta chứng minh được

(1 − a + a ). a − 4a + 5
( )
2
≥ 1 ⇔ ( a − 1) a 2 − 3a + 3 ≥ 0 (luôn đúng).
2 2
bo

2
Đẳng thức xảy ra khi và chỉ khi a= b= c= 1 .
Nhận xét. Ngoài ra bài toán có thể giải bằng kỹ thuật dồn biến xem chương 4.
et

Bài 3. Cho x,y,z là các số thực không âm thoả mãn điều kiện
vi

x 2 + y 2 + z 2 + 2 xyz =
1.
ng

Tìm giá trị lớn nhất của biểu thức P = xy + yz + zx − 2 xyz .


Lời giải
Trong 3 số (2 x − 1),(2 y − 1),(2 z − 1) luôn tồn tại hai số cùng dấu, không mất tính
a
kh

z
tổng quát giả sử ( 2 x − 1)( 2 y − 1) ≥ 0 ⇒ 2 ( x + y ) − 4 xy ≤ 1 ⇒ z ( x + y ) − 2 xyz ≤ .
2
1− z
Chú ý 1 − z 2= 2 xyz + x 2 + y 2 ≥ 2 xy + 2 xyz= 2 xy ( z + 1) ⇒ xy ≤ .
2
1− z z 1
Vì vậy P ≤ + =.
2 2 2
1 1 1
Với x= y= z= thì P bằng . Vậy giá trị lớn nhất của P bằng .
2 2 2

69
Khám phá tư duy Kỹ thuật giải bất ĐT Bài toán Max – Min – Đặng Thành Nam

Bài tập tương tự


Cho a,b,c là các số thực dương thoả mãn điều kiện a 2 + b 2 + c 2 + abc =4.
Chứng minh rằng ab + bc + ca − abc ≤ 2 .
Bài 4. Cho x,y,z là các số thực dương có tích bằng 1.
1 1 1 2
Chứng minh rằng + + + ≥1.
( x + 1) ( y + 1) ( z + 1) ( x + 1)( y + 1)( z + 1)
2 2 2

Lời giải
Trong ba số ( x − 1),( y − 1),( z − 1) luôn có hai số cùng dấu, không mất tính tổng
quát giả sử ( x − 1)( y − 1) ≥ 0 ⇒ xy + 1 ≥ x + y ⇒ 2 ( xy + 1) ≥ ( x + 1)( y + 1) .

n
1 1 1
Sử dụng bất đẳng thức + ≥

.v
.
( x + 1) 2
( y + 1) 2 1 + xy

om
Chứng minh xem chương 3.
1 1 1 z 1 z
Suy ra VT ≥ + + = + + =
1.
1 + xy ( z + 1) 2
(1 + xy )(1 + z ) 1 + z ( z + 1) (1 + z )2
2
.c
Bất đẳng thức được chứng minh. Đẳng thức xảy ra khi và chỉ khi x= y= z= 1 .
ok
Bài 5. Cho a,b,c là các số thực không âm. Chứng minh
bo

1 
abc + 2 +

( a − 1)2 + ( b − 1)2 + ( c − 1)2  ≥ a + b + c .
2
et

Lời giải
Luôn tồn tại hai trong ba số (a − 1),(b − 1),(c − 1) cùng dấu, không mất tính tổng
vi

quát giả sử ( a − 1)( b − 1) ≥ 0 ⇒ ab ≥ a + b − 1 .


ng

Khi đó ta chỉ cần chứng minh


1 
c ( a + b − 1) + 2 +

( a − 1)2 + ( b − 1)2 + ( c − 1)2  ≥ a + b + c
a

2
⇔ ( a − 1) + ( b − 1) + ( c − 1) ≥ 2 ( a + b − 2 )( c − 1)
2 2 2
kh

Sử dụng bất đẳng thức C –S và bất đẳng thức AM – GM ta có

( a + b − 2 )2 + ( a + b − 2 )2 .
( a − 1) + ( b − 1) + ( c − 1) ≥
2 2 2
( c − 1) ≥ 2
2
( c − 1)2
2 2
= 2 ( a + b − 2 )( c − 1) ≥ 2 ( a + b − 2 )( c − 1)

Bài 6. Cho a,b,c là các số thực không âm chứng minh


a 2 + b 2 + c 2 + 2abc + 1 ≥ 2 ( ab + bc + ca ) .

70
Cty TNHH MTV DVVH Khang Việt

Lời giải
Bài toán đã được trình bày trong chủ đề Kỹ thuật sử dụng tam thức bậc hai.
Dưới đây là lời giải tiếp cận theo nguyên lý Dircihlet.
Luôn tồn tại 2 trong 3 số ( a − 1) , ( b − 1) , ( c − 1) cùng dấu, không mất tính tổng
quát ta giả sử ( a − 1)( b − 1) ≥ 0 ⇒ 2c ( a − 1)( b − 1) ≥ 0 ⇔ 2abc ≥ 2 ( ac + bc − c ) .
Vậy ta chỉ cần chứng minh
a 2 + b 2 + c 2 + 1 + 2 ( ac + bc − c ) ≥ 2 ( ab + bc + ca )
⇔ ( a − b ) + ( c − 1) ≥ 0
2 2

Bất đẳng thức được chứng minh. Đẳng thức xảy ra khi và chỉ khi a= b= c= 1 .

n
Bài toán tương tự

.v
Cho a,b,c là các số thực không âm chứng minh

om
a 2 + b 2 + c 2 + a 2b 2 c 2 + 2 ≥ 2 ( ab + bc + ca ) .
Bài 7. Cho a,b,c là các số thực không âm thoả mãn điều kiện a 2 + b 2 + c 2 + abc =
4.
Chứng minh rằng abc ≤ ab + bc + ca ≤ abc + 2 . .c
Lời giải
ok
Bất đẳng thức vế trái đơn giản bởi trong 3 số có ít nhất một số không vượt quá 1
giả sử là c khi đó ab + bc + ca − abc= ab (1 − c ) + c ( a + b ) ≥ 0 .
bo

Đẳng thức xảy ra chẳng hạn tại a= 2, b= c= 0 .


Ta chứng minh bất đẳng thức vế phải:
Trong ba số (a − 1),(b − 1),(c − 1) luôn có hai số cùng dấu, không mất tính tổng
et

quát giả sử ( a − 1)( b − 1) ≥ 0 ⇒ ab ≥ a + b − 1 ⇒ abc ≥ c ( a + b − 1) .


vi

Vậy ta chứng minh c ( a + b − 1) + 2 ≥ ab + bc + ca ⇔ ab ≤ 2 − c .


ng

Chú ý 4 = (a 2 + b 2 ) + c 2 + abc ≥ 2ab + c 2 + abc ⇒ ab ≤ 2 − c .


Bất đẳng thức được chứng minh.
a

Bài 8. Cho a,b,c là các số thực dương có tích bằng 1.


kh

1 1 1 1
Chứng minh rằng + + + ≥1.
(1 + a ) (1 + b ) (1 + c ) a + b + c + 1
2 2 2

Lời giải
Theo nguyên lý Dirichlet thì 2 trong 3 số ( a − 1) , ( b − 1) , ( c − 1) cùng dấu, không
c +1
mất tính tổng quát ta giả sử ( a − 1)( b − 1) ≥ 0 ⇒ 1 + ab
= ≥ a +b.
c
1 1 1 c
Chú ý. + ≥ = .
(1 + a ) 2
(1 + b ) 2 1 + ab c + 1

71
Khám phá tư duy Kỹ thuật giải bất ĐT Bài toán Max – Min – Đặng Thành Nam

c 1 1
Gọi P là biểu thức vế trái ta có P ≥ + + =
1.
c + 1 ( c + 1) 2 c +1
+ c +1
c
Bất đẳng thức được chứng minh. Đẳng thức xảy ra khi và chỉ khi a= b= c= 1 .

A. BÀI TẬP RÈN LUYỆN


Bài 1. Cho a,b,c là các số thực thoả mãn điều kiện
1 1 1 1
+ 2 + 2 = .
a + 8 4b + 8 9c + 8 3
2

n
Tìm giá trị lớn nhất và giá trị nhỏ nhất của biểu thức P =a + 2b + 3c .

.v
B. HƯỚNG DẪN GIẢI – ĐÁP SỐ

om
1 1 11
Bài 1. Để đơn giản đặt=
x a=
, y 2b=
, z 3c ta có + + = .
x +8
2
y +8
2
z +8 3
2

Ta cần Max và Min của P = x + y + z .


.c
Theo giả thiết kết hợp sử dụng bất đẳng thức AM – GM ta có
ok
1 1 1 1 1
= − 2 + − 2
x +8
2 6 y +8 6 z +8
bo

y2 + 2 z2 + 2 1 y2 + 2 z2 + 2
= + ≥ .
(
6 y2 + 8 ) 6( z 2
+8 ) 3 y2 + 8 z2 + 8
et

Tương tự ta có
vi

1 1 x2 + 2 z 2 + 2 1 1 y 2 + 2 x2 + 2
≥ ≥
ng

. 2 ; 2 .
y2 + 8 3 x +8 z +8 z +8 3
2
y 2 + 8 x2 + 8
Nhân theo vế ba bất đẳng thức trên ta được:
a

(x 2
)( )( )
+ 2 y 2 + 2 z 2 + 2 ≤ 27 (1).
kh

Mặt khác ( x + 2 )( y + 2 )( z )
+ 2 ≥ 3 ( x + y + z ) (2).
2 2 2 2

Thật vậy bất đẳng thức đã cho tương đương với:


( )
x 2 y 2 z 2 + 2 x 2 y 2 + y 2 z 2 + z 2 x 2 + 8 ≥ 6 ( xy + yz + zx ) .

Trong ba số ( x 2 − 1),( y 2 − 1),( z 2 − 1) luôn có hai số cùng dấu, không mất tính
tổng quát giả sử
( x − 1)( y − 1) ≥ 0 ⇒ x
2 2 2 2
(
y ≥ x2 + y 2 − 1 ⇒ x2 y 2 z 2 ≥ z 2 x2 + y 2 − 1 . )
72
Cty TNHH MTV DVVH Khang Việt

Do đó ta chỉ cần chứng minh


( ) ( )
z 2 x 2 + y 2 − 1 + x 2 + y 2 + z 2 + 2 x 2 y 2 + y 2 z 2 + z 2 x 2 + 8 ≥ 6 ( xy + yz + zx )

⇔ ( x − y ) + 3 ( yz − 1) + 2 ( xy − 1) + 3 ( xz − 1) ≥ 0
2 2 2 2

Bất đẳng thức cuối đúng suy ra điều phải chứng minh.
Kết hợp (1) và (2) ta có
( x + y + z )2 ≤ 9 ⇔ −3 ≤ x + y + z ≤ 3 .
1 1
Vậy giá trị nhỏ nhất của P bằng -3 đạt tại a =
−1, b =− ,c =
− .
2 3

n
1 1
Giá trị lớn nhất của P bằng 3 đạt tại =
a 1,=b =
,c .

.v
2 3

om
CHỦ ĐỀ 6: KỸ THUẬT SỬ DỤNG TAM THỨC BẬC HAI

Các tính chất về nghiệm và dấu của tam thức bậc hai có một ứng dụng hết sức
.c
sâu rộng trong giải toán. Dưới đây tôi trình bày một ứng dụng của tam thức bậc hại
trong chứng minh bất đẳng thức. Bất đẳng thức tiếp cận bằng phương pháp này có
ok
thể nói là rất tự nhiên.
A. NỘI DUNG PHƯƠNG PHÁP
bo

Xét tam thức bậc hai f ( x) = ax 2 + bx + c .


b
Đính lý 1. Nếu a > 0 thì f(x) đạt giá trị nhỏ nhất tại x0 = −
et

và giá trị nhỏ nhất


2a
b 2 − 4ac
vi

f ( x0 ) = − .
4a
ng

b
Định lý 2. Nếu a < 0 thì f(x) đạt giá trị lớn nhất tại x0 = − và giá trị lớn nhất
2a
a

b 2 − 4ac
f ( x0 ) = − .
kh

4a
Định lý 3. Nếu ∆= b 2 − 4ac ≥ 0 thì f(x) luôn có nghiệm.
a > 0
Định lý 4. Nếu  thì f ( x) ≥ 0 với mọi x.
∆= b − 4ac ≤ 0
2

a < 0
Định lý 5. Nếu  thì f ( x) ≤ 0 với mọi x.
∆= b − 4ac ≤ 0
2

73
Khám phá tư duy Kỹ thuật giải bất ĐT Bài toán Max – Min – Đặng Thành Nam

B. BÀI TẬP MẪU


Bài 1. Chứng minh rằng với x,y,z là các số thực có tổng bằng 1 ta có
( 3x + 4 y + 5 z )2 ≥ 44 ( xy + yz + zx ) .
Lời giải
Thay z =1 − x − y bất đẳng thức trở thành:

( 3x + 4 y + 5 − 5 x − 5 y )2 ≥ 44 xy + 44 ( x + y )(1 − x − y )
⇔ 48 x 2 + 16 x ( 3 y − 4 ) + 45 y 2 − 54 y + 25 ≥ 0

Vế trái là tam thức bậc hai của x với hệ số của x 2 dương và có

n
∆ 'x =
2
(
64 ( 3 y − 4 ) − 48 45 y 2 − 54 y + 25 = )
−176 ( 3 y − 1) ≤ 0 .
2

.v
Vậy bất đẳng thức được chứng minh. Đẳng thức xảy ra khi và chỉ khi

om
1 1 1
= x = ,y = ,z .
2 3 6
Cách 2: Bất đẳng thức đã cho tương đương với: .c
9 x 2 + 16 y 2 + 25 z 2 ≥ 20 xy + 4 yz + 14 zx .
ok
Sử dụng bất đẳng thức AM – GM ta có
5
( 5
)
4 x 2 + 9 y 2 ≥ 2. 4 x 2 .9 y 2 ≥ 20 xy
bo

3 3
7
12
(
4 x 2 + 36 z 2 ≥ 2.
7
12
)
4 x 2 .36 z 2 ≥ 14 xz
et

y 2 + 4 z 2 ≥ 2 y 2 .4 z 2 ≥ 4 yz
vi

Cộng theo vế ba bất đẳng thức trên ta có đpcm. Như vậy không cần giả thiết bài
toán ba số có tổng bằng 1(Xem thêm chủ đề kỹ thuật tham số hoá – Chương 2).
ng

Bài 2. Chứng minh rằng với mọi số thực a và b ta luôn có


( )( ) (
3 1 − a + a 2 1 − b + b 2 ≥ 2 1 − ab + a 2b 2 . )
a
kh

Lời giải
Viết lại bất đẳng thức dưới dạng:
(a 2
) ( )
− 3a + 3 b 2 − 3a 2 − 5a + 3 b + 3a 2 − 3a + 1 ≥ 0 .

Vế trái là tam thức bậc hai của b có a 2 − 3a + 3 > 0, ∀a ∈  và

( ) ( )( ) ( )
2 2
∆b =3a 2 − 5a + 3 − 4 a 2 − 3a + 3 3a 2 − 3a + 1 =
− a 2 − 3a + 1 ≤ 0, ∀a ∈ 
Do đó vế trái luôn không âm. Bài toán được chứng minh.

74
Cty TNHH MTV DVVH Khang Việt

 a 2 − 3a + 1 =0
 3± 5
Đẳng thức xảy ra khi và chỉ khi  3a − 5a + 3 ⇔ a = b =
2
.
b =
( )
 2
 2 a 2 − 3a + 3

Bài tập tương tự


Chứng minh rằng với mọi số thực a,b,c,d ta có
3(a 2 − ab + b 2 )(c 2 − cd + d 2 ) ≥ 2(c 2 a 2 − abcd + b 2 d 2 ) .
Ta cùng xét một số bài toán cùng dạng sau đây
Bài 2.1. Cho a,b,c là các số thực không âm. Chứng minh rằng
( )( )( )

n
3 1 − a + a 2 1 − b + b 2 1 − c + c 2 ≥ 1 + abc + a 2b 2 c 2 .

.v
Lời giải
( )( )

om
Ta có 2 1 − a + a 2 1 − b + b 2 =1 + a 2b 2 + ( a − b ) + (1 − a ) (1 − b ) .
2 2 2

Suy ra 2 (1 − a + a )(1 − b + b ) ≥ 1 + a b
2 2 2 2
.
Do ta chỉ cần chứng minh bất đẳng thức sau
.c
( )( ) ( )
ok
3 1 + a 2b 2 1 − c + c 2 ≥ 2 1 + abc + a 2b 2 c 2 .

( ) (
⇔ 3 + a 2b 2 c 2 − 3 + 2ab + 3a 2b 2 c + 1 + 3a 2b 2 ≥ 0 . )
bo

Coi vế trái là tam thức bậc hai của c có hệ số của c 2 dương và

( ) ( )( )
et

2
− 4 3 + a 2b 2 1 + 3a 2b 2 = −3 (1 − ab ) ≤ 0 .
4
∆ = 3 + 2ab + 3a 2b 2
Do đó vế trái luôn không âm.
vi

Bài toán được chứng minh đẳng thức xảy ra khi và chỉ khi a= b= c= 1 .
ng

Bài 2.2. Cho x và y là hai số thực không cùng dương ta luôn có


4 2
( )(
x − x + 1 y 2 − y + 1 ≥ x 2 y 2 − xy + 1 . )
a

3
kh

Lời giải
Theo giả thiết trong hai số luôn có một số không dương không mất tính tổng
quát ta giả sử y ≤ 0 khi đó viết lại bất đẳng thức dưới dạng:

( y − 2 )2 x 2 − ( 4 y 2 − 7 y + 4 ) x + (1 − 2 y )2 ≥ 0 .
Coi vế trái là tam thức bậc hai của x ta có

(4y ) ( )
2
− 4 ( y − 2 ) (1 − 2 =
y)
2 2
=
∆ 2
− 7y + 4 y 24 y 2 − 51 y + 24 ≤ 0, ∀y ≤ 0 .

75
Khám phá tư duy Kỹ thuật giải bất ĐT Bài toán Max – Min – Đặng Thành Nam

1
Bài toán được chứng minh đẳng thức xảy ra khi và chỉ khi=x =, y 0 hoặc
2
1
=
x 0,=
y .
2
Mở rộng bài toán cho 3 biến ta có kết quả
Cho x,y,z là các số thực không cùng dương ta luôn có
16 2
9
( )( )( )
x − x + 1 y 2 − y + 1 z 2 − z + 1 ≥ 1 − xyz + x 2 y 2 z 2 .

Bài 3. Cho a,b,c là các số thực không âm thỏa mãn điều kiện a 2 + b 2 + c 2 =
2.
Chứng minh rằng: 1 + 2abc ≥ ab + bc + ac .

n
Lời giải

.v
2
Giả sử c max {a, b, c} ⇒ c ≥
= .

om
3
Khi đó đặt S =+
a b, P =ab theo giả thiết bài toán ta có
S 2 + c2 − 2
.c
S 2 − 2 P + c 2 =2 ⇒ P = .
2
ok
S 2 + c2 − 2 S 2 + c2 − 2
Ta cần chứng minh 1 + 2c. ≥ + c.S
2 2
⇔ ( 2c − 1) S 2 − 2cS + 2c3 − c 2 − 4c + 4 ≥ 0
bo

Vế trái là tam thức bậc hai với hệ số dương do vậy ta chỉ cần chứng minh
( )
et

∆ 'S = c 2 − ( 2c − 1) 2c3 − c 2 − 4c + 4 ≤ 0

⇔ 4 ( c − 1) ( c )
vi

2 2
+ c −1 ≥ 0
ng

2
Bất đẳng thức cuối luôn đúng do c ≥ .
3
Bất đẳng thức được chứng minh. Đẳng thức xảy ra khi và chỉ khi a= b= 1, c= 0
a
kh

hoặc các hoán vị.


Bài tập tương tự
Cho a, b, c là các số thực dương thoả mãn điều kiện
(a + 1)(b + 1)(c + 1) =1 + 4abc .
Chứng minh rằng: a + b + c ≤ 1 + abc .
Bài 4. Cho a,b,c là các số thực không âm thay đổi thỏa mãn điều kiện
a+b+c = 1.
Tìm giá trị lớn nhất của biểu thức P = a ( b − c ) + b ( c − a ) + c ( a − b ) .
4 4 4

76
Cty TNHH MTV DVVH Khang Việt

Lời giải
Không mất tính tổng quát giả sử a ≥ b ≥ c, khi đó
P = a ( b − c ) + b ( c − a ) + c ( a − b ) ≤ a ( b + c ) + ba 4 + ca 4
4 4 4 4

= a ( b + c ) + a 4 ( b + c ) = a ( b + c ) ( b + c ) + a 3 
4 3
 
= a ( b + c ) ( a + b + c ) − 3a ( b + c )( a + b + c ) 
3
.
 
=a ( b + c ) (1 − 3a ( b + c ) ) =−3  a ( b + c )  + a ( b + c )
2

2
 1 1 1
=−3  a ( b + c ) −  + ≤

n
 6  12 12

.v
1 3+ 6 3− 6
Vậy giá trị lớn nhất của P bằng đạt
= tại a = ,b = , c 0 hoặc

om
12 6 6
các hoán vị.
Chú ý. Ta có bất đẳng thức mạnh hơn như sau
a (b + c ) + b (c + a ) + c ( a + b) ≤
4 4
.c
4 1
( a + b + c )4
12
ok
Bài 5. Cho các số thực thay đổi x,y,z thõa mãn điều kiện
16
x2 + y 2 + z 2 + xy =
bo

3.
25
Tìm giá trị nhỏ nhất của biểu thức
et

3 2 5
P= ( x + y 2 ) + z 2 + xy − 10( xy + yz + zx) .
5 6
vi

Lời giải
Nhận xét. Trước hết ta chưa quan tâm đến điều kiện của x,y,z mà để ý đến tính
ng

chất đối xứng của x và y trong điều kiện cũng như biểu thức của P nên ta sử dụng
( x + y )2
a

đánh giá: x 2 + y 2 ≥ .
2
kh

Sử dụng bất đẳng thức AM-GM ta được:


3 5
P ≥ ( x + y ) 2 + z 2 + xy − 10( xy + yz + zx)
10 6
3 5
≥2 ( x + y )2 . z 2 + xy − 10 ( xy + yz + zx )
10 6
= ( x + y ) z + xy − 10( xy + yz + zx) ≥ xy + yz + zx − 10( xy + yz + zx )

Một đánh giá dễ nhận ra là đưa về tam thức bậc 2:

77
Khám phá tư duy Kỹ thuật giải bất ĐT Bài toán Max – Min – Đặng Thành Nam

2
 5 5 5
xy + yz + zx − 10 ( xy + yz + zx =
)  xy + yz + zx −  − ≥ − .
 2  2 2
Đẳng thức xảy ra khi và chỉ khi
 5
 xy + yz + zx = 2  5 5 2
 x = − ,y=
− ,z =−3
 x = y 17
⇔
34 34
 16 
.
 x2 + y 2 + z 2 + xy =
3= x =
5
,y =
5
,z 3
2
 25  34 34 17
z ( x + y) ≥ 0

n
5 5 5 2

.v
Vậy giá trị nhỏ nhất của P bằng − đạt tại − ,y=
− ,z =
−3 hoặc
2 34 34 17

om
5 5 2
=x = ,y = ,z 3 .
34 34 17
Bài 6. Cho x,y,z là các số thực thỏa mãn điều kiện x 2 + y 2 + z 2 =
.c 3.
Tìm giá trị lớn nhất và giá trị nhỏ nhất của biểu thức P = xy + yz + 2 yz .
ok
Lời giải
Ta tìm giá trị nhỏ nhất của P
bo

Xuất phát từ bất đẳng thức cơ bản:


x2 + y 2 + z 2 3
( x + y + z )2 ≥ 0 ⇔ xy + yz + zx ≥ − =− .
et

2 2
3 3 x2 + z 2 3 x2 + y 2 + z 2
Suy ra P = ( xy + yz + zx ) + zx ≥ − + zx ≥ − − ≥− − = −3 .
vi

2 2 2 2 2
Đẳng thức xảy ra khi và chỉ khi
ng


x + y + z = 0  3 3
 2 x = − ,y= 0, z =
a

2 2
=
x + y + z
2 2
3 ⇔ .

kh

 3 3
 xz = x +z
2 2
x +y +z
2 2 2
3 x = , y = 0, z = −
− = =
−  2 2
 2 2 2
3 3
Vậy giá trị nhỏ nhất của P bằng −3 đạt tại x =
− , y=
0, z = hoặc
2 2
3 3
x= , y = 0, z = − .
2 2
Cách 2: Viết lại biểu thức P dưới dạng
P + 3 = ( x + z ) + y ( x + z ) + y 2 ⇔ ( x + z ) + y ( x + z ) + y 2 − P − 3 = 0 (1) .
2 2

78
Cty TNHH MTV DVVH Khang Việt

Coi (1) là phương trình bậc 2 với ẩn là ( x + z ) ta có điều kiện có nghiệm của

phương trình là ∆ ( x + z=
) ( )
y 2 − 4 y 2 − P − 3 ≥ 0 ⇔ P ≥ −3 +
3y2
4
≥ −3 .

y = 0  3 3
 x = − ,y= 0, z =
 y 2 2
Đẳng thức xảy ra khi và chỉ khi  x + z =− =0 ⇔  .
 2  3 3
x = , y = 0, z = −
 x + y + z =
2 2 2
3  2 2
Ta có kết quả tương tự cách trên.
Tìm giá trị lớn nhất của P

n
(
Ta tìm số thực k > 0 nhỏ nhất sao cho P = xy + yz + 2 zx ≤ k x 2 + y 2 + z 2 đúng )

.v
với mọi x,y,z và x 2 + y 2 + z 2 . Khi đó giá trị lớn nhất của P bằng 3k.

om
Viết lại bất đẳng thức trên dưới dạng: ky 2 − y ( x + z ) + kx 2 + kz 2 − 2 xz ≥ 0 .

Đây là một tam thức bậc hai với hệ số của y 2 dương nên ta chỉ cần tìm k sao
.c
cho ∆ y ≤ 0 .
ok
Tức
( x + z )2 − 4k ( kx 2 + kz 2 − 2 xz ) ≤ 0 ⇔ (1 − 4k 2 )( x 2 + z 2 ) + 2 xz (1 + 4k ) ≤ 0, ∀x, z .
bo

Đây là một bất đẳng thức đối xứng với x và z nên ta chọn x= z= 1 .
 1− 3
et

k ≤
Suy ra 1 − 4k 2 + 1 + 4k ≤ 0 ⇔ 2k 2 − 2k − 1 ≥ 0 ⇔  2 .
 1+ 3
vi

k ≥
 2
ng

Vậy=
k
1+ 3
⇒P≤
3 1+ 3
.
( )
2 2
a

3+ 3 3− 3
kh

Đẳng thức xảy ra khi và chỉ khi x= z= , y= .


4 2
Nhận xét. Ngoài lời giải bằng tam thức bậc hai như trên ta có thể sử dụng kỹ thuật
tham số hóa hoặc sử dụng C –S.
1 
Bài 7. Cho a,b,c là các số thực thuộc đoạn  ;3 . Chứng minh
3 
a b c 7
+ + ≥ .
a+b b+c c+a 5

79
Khám phá tư duy Kỹ thuật giải bất ĐT Bài toán Max – Min – Đặng Thành Nam

Lời giải
Không mất tính tổng quát giả sử a = max {a, b, c} .
Bất đẳng thức đã cho tương đương với
( 3a − 2b ) c 2 − ( 2a 2 − ab − 3b2 ) c + 3a 2b − 2ab2 ≥ 0 .
Vì 3a − 2b > 0 ta chỉ cần chứng minh

( 2a ) ( )
2
2
− ab − 3b 2 − 4 ( 3a − 2b ) 3a 2b − 2ab 2 ≤ 0

(
⇔ ( a − b )( a − 9b ) 4a 2 + b 2 ≤ 0 )

n
1
Bất đẳng thức cuối đúng do a − b ≥ 0; a − 9b ≤ 3 − 9. =0.

.v
3
1
Bất đẳng thức được chứng minh. Đẳng thức xảy ra khi=
a 3,=
b =
, c 1.

om
3
Nhận xét. Bằng cách tương tự ta chứng minh được
a b c 8
+ + ≤ . .c
a+b b+c c+a 5
Ngoài ra có thể đưa về xét tính đơn điệu của hàm số (Xem chương 3).
ok
Bài tập tương tự
(TSĐH Khối A 2011) Cho các số x, y, z ∈ [1;4] thỏa mãn điều kiện
bo

x ≥ y, x ≥ z .
et

x y z
Tìm giá trị nhỏ nhất của biểu thức P= + + .
2x + 3y y + z z + x
vi

34
HD: Chứng minh Pmin = .
ng

33
Bài 8. Cho a,b,c là các số thực không âm.
Chứng minh rằng a 2 + b 2 + c 2 + 2abc + 1 ≥ 2 ( ab + bc + ca ) .
a
kh

Lời giải
Viết lại bất đẳng thức dưới dạng: a 2 + 2 ( bc − b − c ) a + ( b − c ) + 1 ≥ 0 .
2

Coi vế trái bất đẳng thức trên là tam thức bậc hai của a ta được:
∆ 'a = ( bc − b − c ) − ( b − c ) − 1 = bc ( b − 2 )( c − 2 ) − 1 .
2 2

TH1: Nếu bc − b − c ≥ 0 ta có ngay điều phải chứng minh.


TH2: Nếu bc − b − c ≤ 0 ⇔ ( b − 1)( c − 1) ≤ 1 ta xét hai khả năng sau:
Khả năng 1. Nếu ( b − 2 )( c − 2 ) ≤ 0 ⇒ ∆ 'a ≤ −1 ta có ngay điều phải chứng minh.

80
Cty TNHH MTV DVVH Khang Việt

 b, c ≥ 2
Khả năng 2. Nếu ( b − 2 )( c − 2 ) ≥ 0 ⇔  vì ( b − 1)( c − 1) ≤ 1 nên b, c ≤ 2
 b, c ≤ 2
khi đó sử dụng bất đẳng thức AM-GM ta được:
2 2
b+ 2−b c+ 2−c
∆ 'a= b ( 2 − b ) .c ( 2 − c ) − 1 ≤   .  − 1= 0, ta có điều phải
 2   2 
chứng minh.
Chú ý. Xem thêm lời giải bằng Nguyên lý Dirichlet trong chủ đề tương ứng.
Bài tập tương tự
Cho a,b,c là các số thực dương thoả mãn điều kiện a 2 + b 2 + c 2 − 2abc − 1 ≥ 0 .

n
Chứng minh rằng a − b + b − c + c − a ≥ a 2 + b 2 + c 2 − 2abc − 1 .

.v
Bài 9. Cho x,y,z là các số thực thoả mãn điều kiện x + y + z = xy + yz + zx .

om
x y 1z
Chứng minh rằng + + ≥− .
x +1
2
y +12
z +12 2
Lời giải .c
Theo giả thiết ta có: z ( x + y − 1) = x + y − xy .
ok
x + y = 1
+ Nếu x + y =1 ⇒  , vô nghiệm.
 xy = 1
bo

x + y − xy
+ Vậy x + y ≠ 1 và z = .
x + y −1
et

x + y − xy
x y x + y −1 1
Vậy ta cần chứng minh + + ≥−
vi

x +1
y + 1  x + y − xy 
2 2 2 2
  +1
ng

 x + y −1 
2x 2y 2 ( x + y − xy )( x + y − 1)
⇔ 2 + 2 + ≥ −1
a

x + 1 y + 1 ( x + y − xy )2 + ( x + y − 1)2
kh

( x + 1)2 ( y + 1)2 ( xy − 1)2


⇔ + ≥
x2 + 1 y2 + 1 ( x + y − xy )2 + ( x + y − 1)2
Chú ý theo bất đẳng thức C–S ta có
( x + 1)2 + ( y=+ 1)
2
( x + 1)2 (1 − y )2 + ( y + 1)2 (1 − x )2
x2 + 1 y 2 + 1 ( x 2 + 1) (1 − y )2 ( y 2 + 1) (1 − x )2

( x + 1)(1 − y ) + ( y + 1)(1 − x )  4 ( xy − 1)


2 2
≥ =
( ) (
x 2 + 1 (1 − y ) + y 2 + 1 (1 − x )
2
) 2
( )
x 2 + 1 (1 − y ) + y 2 + 1 (1 − x )
2 2
( )
81
Khám phá tư duy Kỹ thuật giải bất ĐT Bài toán Max – Min – Đặng Thành Nam

Vậy ta chỉ cần chứng minh


(
4 ( x + y − xy ) + 4 ( x + y − 1) ≥ x 2 + 1 (1 − y ) + y 2 + 1 (1 − x )
2 2
) 2
( ) 2

( ) (
⇔ y 2 − 3 y + 3 x2 − 3 y 2 − 8 y + 3 x + 3 y 2 − 3 y + 1 =
0 )
Vế trái bất đẳng thức là tam thức bậc hai của x với hệ số của x2 dương và có

( ) ( )( ) ( )
2 2
∆x =3y2 − 8 y + 3 − 4 y2 − 3y + 3 3y2 − 3y + 1 =
−3 y 2 − 1 ≤ 0 .
Điều đó chứng tỏ bất đẳng thức đúng. Đẳng thức xảy ra khi và chỉ khi
x= y= −1, z =
1 hoặc các hoán vị.
Bài 10. Cho a,b,c là các số thực thỏa mãn điều kiện a + b + c =2 và

n
a3 + b3 + c3 − 3abc =
2.

.v
=
Tìm giá trị lớn nhất của biểu thức P max {a, b, c} − min {a, b, c} .

om
Lời giải
Không mất tính tổng quát giả sử a = max {a, b, c} , c = min {a, b, c} ⇒ P = a − c .

( a + b + c ) ( a 2 + b2 + c 2 − ab − bc − ca ) .
Ta có: 2 = a3 + b3 + c3 − 3abc =
.c
ok
1
⇒ 2= ( a + b + c ) ( a − b )2 + ( b − c )2 + ( c − a )2 
2
⇔ ( a − b) + (b − c ) + (c − a ) =
2 2 2
bo

Suy ra: P 2 + ( a − b ) + ( b − c ) =
2 2
2.
et

⇔ P 2 + ( a − c ) + ( c − b )  + ( b − c ) =2
2 2
vi

⇔ P 2 + ( a − c ) + 2 ( b − c ) − 2 ( a − c )( b − c ) =
2 2
2.
ng

⇔ P2 − P (b − c ) + (b − c ) − 1 =
2
0
Coi đẳng thức trên là phương trình bậc 2 với ẩn là (b – c). Để phương trình này
a

4
có nghiệm ta phải có ∆b −c = P 2 − 4 P 2 − 1 ≥ 0 ⇔ P 2 ≤ ⇒ P ≤ (2
. )
kh

3 3
 2  2+ 3
a − c = 3 a =
  3
 2 1 
Đẳng thức xảy ra khi và chỉ khi ( b − c ) − (b − c ) + =
2 2
0 ⇔ b = .
 3 3  3
a + b + c = 0  2− 3
 c =
a ≥ b ≥ c  3

82
Cty TNHH MTV DVVH Khang Việt

2 2+ 3 2 2− 3
Vậy giá trị lớn nhất của P bằng đạt tại=
a =,b = ,c hoặc
3 3 3 3
các hoán vị.
Cách 2: Không mất tính tổng quát giả sử a = max {a, b, c} , c = min {a, b, c} ⇒ P = a − c .
Ta có
a 2 + b 2 + c 2 − ab − bc − ca =1 ⇔ ( a + b + c ) − 3 ( ab + bc + ca ) =1
2

⇔ ab + bc + ca =
1
Suy ra: P 2 =( a − c ) =( a + c ) − 4ac =( 2 − b ) − 4 1 − b ( a + c ) 
2 2 2

n
2
 2 4 4
=( 2 − b ) − 4 + 4b ( 2 − b ) =−3b 2 + 4b =−3  b −  + ≤
2

.v
 3 3 3
 2+ 3

om
 2
= b 3= a
 3

 2  2
Đẳng thức xảy ra khi và chỉ khi a −=c ⇔ =
b .c
 3  3
a + b + c = 2  2− 3
c =
ok

  3
Cách 3: Không mất tính tổng quát giả sử c = min {a, b, c} .
bo

Đặt a =x + c, b =y + c, ( x, y ≥ 0 ) .
et

Ta có: a 2 + b 2 + c 2 − ab − bc − ca =
1 suy ra
⇔ ( x + c ) + ( y + c ) + c 2 − c ( x + c ) − c ( y + c ) − ( x + c )( y + c ) =
2 2
1.
vi

 2
y  3y2  2
ng

 x −  + =
1 x ≤
 2 
1 
4 3
⇔ x 2 − xy + y 2 =⇒ ⇒ .
2 2
2
 x − y  + 3 x = 
1 y ≤
a

 2 
 4  3
kh

2
Suy ra P max { x, y} ≤
= .
3

C. BÀI TẬP RÈN LUYỆN


Bài 1. Cho a,b,c là độ dài 3 cạnh một tam giác và x,y,z là các số thực thay
đổi thỏa mãn ax + by + cz =0 . Chứng minh xy + yz + zx ≤ 0 .
 x + y + z =
1
Bài 2. Cho x,y,z là các số thực thoả mãn điều kiện  2 .
 x + 2 y + 3 z =
2 2
4

83
Khám phá tư duy Kỹ thuật giải bất ĐT Bài toán Max – Min – Đặng Thành Nam

Tìm giá trị lớn nhất của x.


Bài 3. Cho x,y là hai số thực thoả mãn điều kiện x 2 + xy + y 2 − 6 ( x + y ) + 11 =
0.
Tìm giá trị lớn nhất, giá trị nhỏ nhất của biểu thức P= y + 2 x .
Bài 4. Cho x,y,z là các số thực không âm có tổng bằng 1. Tìm giá trị lớn nhất của
biểu thức P =9 xy + 10 yz + 11zx .
Bài 5. Cho hai số thực dương x,y thoả mãn điều kiện x 2 y = 1 . Chứng minh rằng

x x2 + y 2 + x2 ≥ 2 .

n
Bài 6. Cho n số thực a1 , a2 ,..., an thuộc đoạn [0;1]. Chứng minh

.v
(1 + a1 + a2 + ... + an )2 ≥ 4 ( a12 + a22 + ... + an2 ) .

om
Bài 7. Tìm tất cả các số nguyên dương n sao cho với mọi số thực x1 , x2 ,.., xn ta có
x12 + x22 + ... + xn2 ≥ ( x1 + x2 + ... + xn −1 ) xn .
.c
Bài 8. Chứng minh với mọi số thực x,y,z và a,b,c là độ dài ba cạnh một tam giác ta
có a ( x − y )( x − z ) + b ( y − z )( y − z ) + c ( z − x )( z − y ) ≥ 0 .
ok
Bài 9. (Vasile-Cirtoaje) Chứng minh với mọi số thực a,b,c ta có

(a ) ( )
2
2
+ b2 + c2 ≥ 3 a3b + b3c + c3 a .
bo

Bài 10. Cho a,b,c là các số thực dương có tổng bằng 3. Chứng minh
et

9
a + ab + 2abc ≤ .
2
vi

Bài 11. Cho a,b,c là các số thực không âm. Chứng minh rằng
a 2 + b 2 + c 2 + 2abc + 3 ≥ (1 + a )(1 + b )(1 + c ) .
ng

Bài 12. Cho a,b,c là các số thực không âm. Chứng minh
( )
a

2 a 2 + b 2 + c 2 + abc + 8 ≥ 5 ( a + b + c ) .
kh

Bài 13. Cho a,b,c là các số thực không âm. Chứng minh
(a 2
)( )( )
+ 2 b 2 + 2 c 2 + 2 ≥ 9 ( ab + bc + ca ) .

Bài 14. Cho tam giác có ba góc A,B,C chứng minh với mọi số thực x ta có
1 2
1+ x ≥ cos A + x ( cos B + cos C ) .
2
Bài 15. Chứng minh rằng với mọi số thực x và y ta có
( )
x 2 1 + sin 2 y + 2 x ( sin y + cos y ) + 1 + cos 2 y > 0 .

84
Cty TNHH MTV DVVH Khang Việt

Bài 16. Chứng minh rằng với mọi số thực x và y ta luôn có


( x + y )2 − xy + 1 ≥ 3 ( x + y) .
Bài 17. Chứng minh rằng với mọi số thực x,y,z và ba góc A,B,C của một
tam giác ta có x 2 + y 2 + z 2 ≥ 2 xy cos C + 2 yz cos A + 2 zx cos B .
Bài 18. Cho x,y,z là các số thực không âm thoả mãn điều kiện
xyz + x + y + z =4.
Chứng minh rằng x + y + z ≥ xy + yz + zx .
0 < a ≤ b ≤ c
Bài 19. Cho  . Chứng minh

n
 x, y , z > 0

.v
 x y z  (a + c)
2
ac ( xa + yb + zc )  + +  ≤ ( x + y + z )2 .

om
a b c 4ac
a 2 + b 2 =1
Bài 20. Cho a,b,c,d là các số thực thỏa mãn điều kiện  .
.c c + d = 3
9+6 2
Chứng minh rằng ac + bd + cd ≤
ok
.
4
Bài 21. (TSĐH Khối A 2011) Cho các số x, y, z ∈ [1;4] thỏa mãn điều kiện
bo

x y z
x ≥ y, x ≥ z . Tìm giá trị nhỏ nhất của biểu thức P= + + .
2x + 3y y + z z + x
et

Bài 22. Cho a,b,c,d là các số thực thỏa mãn b > c > d . Chứng minh
( a + b + c + d )2 > 8 ( ac + bd ) .
vi

Bài 23. Cho a,b,c,d,p,q là các số thực thỏa mãn điều kiện
ng

p 2 + q 2 − a 2 − b2 − c2 − d 2 > 0 .

( )( )
a

Chứng minh p 2 − a 2 − b 2 q 2 − c 2 − d 2 ≤ ( pq − ac − bd ) .
2
kh

Bài 24. Cho a,b,c là độ dài ba cạnh một tam giác và p,q,r là ba số thực thay
đổi thỏa mãn điều kiện p + q + r =0 . Chứng minh a 2 qr + b 2 rp + c 2 pq ≤ 0 .

( )( )
Bài 25. Chứng minh rằng với mọi số thực a,b ta có a 2 + 2 b 2 + 2 ≥ 3 ( ab + a + b ) .
D. HƯỚNG DẪN GIẢI – ĐÁP SỐ
ax + by
Bài 1. Từ giả thiết ta có: z = − . Viết lại bất đẳng thức cần chứng minh dưới
c
ax + by
dạng: xy − ( x + y ) ≤ 0 ⇔ ax 2 − y ( a + b − c ) x + by 2 ≥ 0 .
c
85
Khám phá tư duy Kỹ thuật giải bất ĐT Bài toán Max – Min – Đặng Thành Nam

Coi vế trái bất đẳng thức trên là tam thức bậc hai của x ta được:
y 2 ( a + b − c ) − 4aby= y 2 ( a + b − c ) − 4ab 
2 2
∆= 2
x  
= y 2 ( a − b ) + c 2 − 2c ( a + b )  ≤ y 2 c 2 + c 2 − 2c ( a + b=
) 2cy 2 ( c − a − b ) ≤ 0
2
  
Ta có điều phải chứng minh.
Bài 2. Thay z =1 − x − y vào phương trình thứ hai ta được
x 2 + 2 y 2 + 3 (1 − x − y ) − 4 =
2
0
⇔ 5 y 2 + 6 ( x − 1) y + 4 x 2 − 6 x − 1 =0

(
' y 9 ( x − 1) − 5 4 x 2 − 6 x − 1 ≥ 0 )

n
⇒ ∆=
2 .

.v
6 − 190 6 + 190
⇔ 11x 2 − 12 x − 14 ≤ 0 ⇔ ≤x≤

om
11 11
6 + 190 15 − 3 190 10 − 2 190
=
Khi x =⇒y = ,z .
11 55 55
6 + 190
Vậy giá trị lớn nhất của x bằng .
.c
11
ok
Bài 3. Rút y= P − 2 x thay vào điều kiện bài toán ta được:
x 2 + x ( P − 2 x ) + ( P − 2 x ) − 6 ( x + P − 2 x ) + 11 =
2
bo

0
⇔ 3 x 2 − ( 3P − 6 ) x + P 2 − 6 P + 5 =0
.
( 3P − 6 )2 − 12 ( P 2 − 6 P + 5) ≥ 0
et

⇒ ∆=
x
vi

⇔ P 2 − 12 P + 8 ≤ 0 ⇔ 6 − 2 7 ≤ P ≤ 6 + 2 7
Vậy Pmin =
6 − 2 7; Pmin =
6+2 7 .
ng

Bài 4. Thay z =1 − x − y vào biểu thức của P và rút gọn ta được


11x 2 + (12 y − 11) x + 10 y 2 − 10 y + P =
a

(12 y − 11)2 − 44 (10 y 2 − 10 y + P ) ≥ 0


kh

⇒=
∆x .
2
74  2 22 121  74  11  195 195
⇒P≤− y − y− =− y−  + ≤
11  37 196  11  37  148 148
195  25 11 27 
Vậy Pmax = ⇔ ( x; y; z ) =
 ; ; .
148  74 37 74 
1
Bài 5. Gọi P là biểu thức vế trái ta có y = suy ra
x2

86
Cty TNHH MTV DVVH Khang Việt

1 1
P = x2 + x4 + 2
⇔ P − x2 = x4 +
x x2
1
⇒ P 2 − 2 Px + x 4 = x 4 + 2
⇔ 2 Px 4 − P 2 x 2 + 1= 0
x .
⇒∆ =
2 P − 8P ≥ 0 ⇔ P P3 − 8 ≥ 0 ⇒ P ≥ 2
x
4
( )
1
=
Khi x = , y 2 thì P bằng 2. Vậy Min của P bằng 2.
2
Bài 6. Xét tam thức bậc hai
(
f ( x) = x 2 − (1 + a1 + a2 + ... + an ) x + a12 + a22 + ... + an2 . )

n
.v
Ta có
f (0) = a12 + a22 + ... + an2 ≥ 0; f (1) = a1 ( a1 − 1) + a2 ( a2 − 1) + ... + an ( an − 1) ≤ 0 .

om
Suy ra f (0). f (1) ≤ 0 ⇒ ∃x0 ∈ [ 0;1] sao cho f ( x0 ) = 0 tức f(x) có nghiệm.

Vì vậy ∆ ≥ 0 ⇔ (1 + a1 + a2 + ... + an ) ≥ 4 a12 + a22 + ... + an2 .


2
( .c )
 x1= x2= ...= xn −1
ok
Bài 7. Bất đẳng thức đúng với mọi số thực nên đúng với  .
 xn = 2
Khi đó bất đẳng thức trở thành ( n − 1) + 22 ≥ 2 ( n − 1) ⇔ n ≤ 5 ⇒ 1 ≤ n ≤ 5 .
bo

Ta chứng minh với n=1,2,3,4,5 là các giá trị cần tìm.


Xét tam thức bậc hai f ( xn ) = xn2 − ( x1 + x2 + ... + xn −1 ) xn + x12 + x22 + ... + xn2−1 .
et

Ta có ∆ xn = ( x1 + x2 + ... + xn−1 )2 − 4 ( x12 + x22 + ... + xn2−1 ) .


vi

Do n ≤ 5 và theo bất đẳng thức C–S ta có


ng

( ) (
4 x12 + x22 + ... + xn2−1 ≥ ( n − 1) x12 + x22 + ... + xn2−1 )
a

≥ ( x1 + x2 + ... + xn −1 )
2
kh

⇒ ∆ xn ≤ 0 ⇒ f ( xn ) ≥ 0

Điều đó chứng tỏ bất đẳng thức luôn đúng với n = 1,5 . Vậy n=1,2,3,4,5 là tất cả
các giá trị cần tìm.
Bài 8. Vế trái viết lại thành tam thức bậc hai của x ta được
(
ax 2 − ( ay + az + by − bz + cz − cy ) x + ayz + by 2 − byz + cz 2 − cyz . )
Ta có =
∆ ( ay + az + by − bz + cz − cy )2 − 4a ( ayz + by 2 − byz + cz 2 − cyz ) .
= (a 2
+ b 2 + c 2 − 2ab − 2bc − 2ca ( y − z ) ) 2

87
Khám phá tư duy Kỹ thuật giải bất ĐT Bài toán Max – Min – Đặng Thành Nam

Chú ý a 2 + b 2 + c 2 < 2 ( ab + bc + ca ) ⇒ ∆ ≤ 0 ⇒ .
Bài 9. Đặt b =a + x; c =a + y. Bất đẳng thức được viết lại dưới dạng:

(x 2
) ( )
− xy + y 2 a 2 − x3 − 5 x 2 y + 4 xy 2 + y 3 a + x 4 − 3x3 y + 2 x 2 y 2 + y 4 ≥ 0 .
Vế trái là tam thức bậc hai của a có hệ số của a2 không âm.
Chú ý x 2 − xy + y 2 = 0 ⇔ x = y = 0 ⇒ bất đẳng thức luôn đúng.
Với x 2 − xy + y 2 > 0 khi đó

( ) ( )( x )
2
∆ a = x3 − 5 x 2 y + 4 xy 2 + y 3 − 4 x 2 − xy + y 2 4
− 3x3 y + 2 x 2 y 2 + y 4

n
( )
2
=
−3 x3 − x 2 y − 2 xy 2 + y 3 ≤0

.v
Điều đó chứng tỏ bất đẳng thức luôn đúng(đpcm).

om
Bài 10. Thay b = 3 − a − c và đưa về chứng minh

( 2c + 1) a 2 + ( 2c 2 − 5c − 4 ) a +
9
f (a) = ≥ 0.
.c 2

( 2c )
2
Chú ý ∆=
a
2
− 5c − 4 − 18 ( 2c + 1=
) ( 2c − 1)2 c ( c − 4 ) − 2 ≤ 0, ∀c ∈ [0;3] .
ok
Điều đó chứng tỏ bất đẳng thức luôn đúng.
3 1
bo

Đẳng thức xảy ra khi và chỉ khi =


a ,=
b 1,=
c .
2 2
Bài 11. Bất đẳng thức tương đương với:
et

a 2 + b 2 + c 2 + 2abc + 3 ≥ 1 + a + b + c + ab + bc + ca + abc .
⇔ a 2 + a ( bc − b − c − 1) + b 2 + c 2 − bc − b − c + 2 ≥ 0 .
vi

⇔ a 2 + a ( bc − b − c − 1) + ( b − c ) + bc − b − c + 2 ≥ 0 .
ng

Coi vế trái bất đẳng thức trên là tam thức bậc hai của a ta được:
a

∆ a= ( bc − b − c − 1)2 − 4 ( b − c )2 + bc − b − c + 2


kh

= b 2 c 2 − 2b 2 c − 2bc 2 − 3b 2 − 3c 2 + 4bc + 6b + 6c − 7 .

( ) (
= b 2 c 2 − 2c − 3 − 2 c 2 − 2c − 3 b − 3c 2 + 6c − 7 )
Lại coi đây là tam thức bậc hai của b ta được:

( ) − ( c − 2c − 3)( −3c + 6c − 7 )
2
∆ 'b = c 2 − 2c − 3 2 2

= ( c − 2c − 3)( c − 2c − 3 + 3c − 6c + 7 )
2 2 2
.

4 ( c − 2c − 3) ( c − 1) = 4 ( c − 1) ( c + 1)( c − 3)
2 2
= 2

88
Cty TNHH MTV DVVH Khang Việt

TH1: Nếu bc − b − c − 1 ≥ 0 ta có ngay điều phải chứng minh.


TH2: Nếu bc − b − c − 1 ≤ 0 ⇔ ( b − 1)( c − 1) ≤ 2 ta xét hai khả năng

Khả năng 1. Nếu c ≤ 3 ⇒ c 2 − 2c − 3 ≤ 0, ∆ 'b ≤ 0 ⇒ ∆ a ≤ 0 ta có ngay điều phải


chứng minh.
2
Khả năng 2. Nếu c ≥ 3 ⇒ b − 1 ≤ ≤ 1 ⇔ b ≤ 2 lúc này ta lại coi ∆ a là tam
c −1
thức bậc hai của c và thực hiện tương tự trên ta có ngay ∆ 'c ≤ 0 ⇒ ∆ a ≤ 0 ta có
ngay điều phải chứng minh.
Bài 12. Viết lại bất đẳng thức dưới dạng:

n
2a 2 + a ( bc − 5 ) + 2b 2 + 2c 2 − 5b − 5c + 8 ≥ 0 .

.v
Coi vế trái của bất đẳng thức là tam thức bậc hai của a ta được:
( bc − 5)2 − 8 ( 2b2 + 2c 2 − 5b − 5c + 8)

om
∆ a=

= b 2 c 2 − 16b 2 − 16c 2 − 10bc + 40b + 40c − 39 .

( )
.c
= b 2 c 2 − 16 − 10b ( c − 4 ) − 16c 2 + 40c − 39
ok
Ta có: 2b 2 + 2c 2 − 5b − 5c + 8 > 0, ∀b, c (xét tam thức bậc hai với b hoặc c).
TH1: Nếu bc ≥ 5 ta có ngay điều phải chứng minh.
bo

TH2: Nếu bc ≤ 5 lại coi ∆ a là tam thức bậc hai của b ta được:

∆ '=
b ( )(
25 ( c − 4 ) − c 2 − 16 −16c 2 + 40c − 39
2
)
et

.
( )
= 8 2c 4 − 5c3 − 24c 2 + 55c − 28 = 8 ( c − 1) ( c − 4 )( 2c + 7 )
2
vi

Khả năng 1. Nếu c ≤ 4 ⇒ c 2 − 16 ≤ 0, ∆ 'b ≤ 0 ⇒ ∆ a ≤ 0 ta có ngay điều phải


ng

chứng minh.
5 5
Khả năng 2. Nếu c ≥ 4 ⇒ b ≤ ≤ < 4 lúc này coi ∆ a là tam thức bậc hai của c
a

c 4
và thực hiện tương tự trên ta có ∆ 'c ≤ 0 ⇒ ∆ a ≤ 0 ta có điều phải chứng minh.
kh

Vậy bất đẳng thức được chứng minh. Đẳng thức xảy ra khi a= b= c= 1 .
( )( )( )
Bài 13. Ta cần chứng minh: a 2 + 2 b 2 + 2 c 2 + 2 − 9 ( ab + bc + ca ) ≥ 0 .

( ) ( ) (
⇔ 4 a 2 + b 2 + c 2 + 2 a 2b 2 + 1 + 2 b 2 c 2 + 1 + )
2 ( c a + 1) + a b c
2 2 2 2 2
+ 1 − 9 ( ab + bc + ca ) ≥ 0 .
Sử dụng bất đẳng thức AM-GM ta được:
( ) ( ) ( )
2 a 2b 2 + 1 + 2 b 2 c 2 + 1 + 2 c 2 a 2 + 1 + a 2b 2 c 2 + 2 ≥ 4ab + 4bc + 4ca + 2abc + 1 .

89
Khám phá tư duy Kỹ thuật giải bất ĐT Bài toán Max – Min – Đặng Thành Nam

Ta cần đi chứng minh.


( )
4 a 2 + b 2 + c 2 + 4ab + 4bc + 4ca + 2abc + 1 − 9 ( ab + bc + ca ) ≥ 0 .

⇔ 3( a )
+ b 2 + c 2 − ab − bc − ca +  a 2 + b 2 + c 2 + 2abc + 1 − 2 ( ab + bc + ca )  ≥ 0
2
 
(luôn đúng) do
a 2 + b 2 + c 2 − ab − bc − ca ≥ 0 và a 2 + b 2 + c 2 + 2abc + 1 − 2 ( ab + bc + ca ) ≥ 0 .
Đẳng thức xảy ra khi và chỉ khi a= b= c= 1 .
Bài tập tương tự

n
Cho a,b,c là các số thực không âm. Chứng minh
( )

.v
abc + 3 a 2 + b 2 + c 2 + 11 ≥ 7 ( a + b + c ) .

om
Bài 14. Viết lại bất đẳng thức dưới dạng
x 2 − 2 x ( cos B + cos C ) + 2 (1 − cos A ) ≥ 0 .
A 2 B −C 
' x ( cos B + cos C ) − 2 (1 − cos =
Ta có ∆= A ) 4sin 2
2
.c  cos
2 2
− 1 ≤ 0 .

ok
Do đó bất đẳng thức được chứng minh.
Bài 15. Chú ý
bo

∆ 'x = ( )( )
( sin y + cos y ) − 1 + sin 2 y 1 + cos2 y =− (1 − sin y cos y ) < 0 .
2 2

( )
2
et

Bài 16. Chú ý ∆ x =− 3 y − 1 ≤ 0 ⇒ dpcm .


vi

− ( y sin C − z cos B ) ≤ 0 .
2
Bài 17. Chú ý ∆ 'x =
ng

x y z
Đẳng thức xảy ra khi và chỉ khi = = .
sin A sin B sin C
=
Bài 18. Không mất tính tổng quát giả sử z min { x, y, z} ⇒ z ≤ 1. Khi đó
a

4 − yz
kh

x= .
y + z + yz
4 − yz 4 − yz
Ta cần chứng minh + y + z ≥ ( y + z ). + yz
y + z + yz y + z + yz

( ) (
⇔ 1 + z − z 2 y2 + z2 + z − 4 y + ( z − 2) ≥ 0 ) 2

(z ) ( ) ( z − 2) =
2
z ( z − 1) ( 5 z − 8 ) ≤ 0
2 2
∆ y= 2
+ z−4 − 4 1 + z − z2

90
Cty TNHH MTV DVVH Khang Việt

Bài toán được chứng minh. Đẳng thức xảy ra khi và chỉ khi x= y= z= 1
hoặc x= y= 2, z= 0 hoặc các hoán vị.(xem thêm kỹ thuật phản chứng).
Bài 19. Đặt f ( x) = x 2 − (a + c) x + ac = 0 có 2 nghiệm a,c
Mà: a ≤ b ≤ c ⇒ f (b) ≤ 0 ⇔ b 2 − (a + c)b + ac ≤ 0
ac y
⇔b+ ≤ a + c ⇔ yb + ac ≤ ( a + c ) y
b b
 x y z
⇒  xa + ac  + ( yb + ac ) + ( zc + ac ) ≤ ( a + c ) x + ( a + c ) y + (a + c) z
 a b c
x y z

n
⇒ xa + yb + zc + ac  + +  ≤ ( a + c )( x + y + z )
a b c

.v
Theo bất đẳng thức AM – GM ta có

om
( xa + yb + zc ) ac 
x y z
⇒2 + +  ≤ ( a + c )( x + y + z )
a b c
x y z .c
⇔ 4 ( xa + yb + zc ) ac  + +  ≤ ( a + c ) ( x + y + z )
2 2

a b c
ok
 x y z  (a + c)
2
⇔ ( xa + yb + zc ) ac  + +  ≤ ( x + y + z )2 (đpcm)
a b c 4ac
bo

Bài 20. Thay d = 3 − c vào biểu thức vế trái được tam thức bậc hai của c
f (c) =ac + b(3 − c) + c(3 − c) =−c 2 − c(b − a − 3) + 3b
et

 3 + a − b  12b + ( a − b + 3)
2
≤ f =
vi

 2  4
Ta chỉ cần chứng minh
ng

12b + ( a − b + 3)
2
9+6 2
≤ ⇔ 6 ( a + b ) − 2ab ≤ 6 2 − 1
a

4 4
⇔ 6 ( a + b ) − ( a + b ) − 1 ≤ 6 2 − 1
2
kh

.
 
(
⇔ a+b− 2 a+b+6 2 ≥0 )( )
Bất đẳng thức cuối đúng do a + b ≤ 2 a 2 + b 2 =2 .( )
Bất đẳng thức được chứng minh. Đẳng thức xảy ra khi và chỉ khi
1 3
a= b= , c= d= .
2 2

91
Khám phá tư duy Kỹ thuật giải bất ĐT Bài toán Max – Min – Đặng Thành Nam

Bài 21. Ta chứng minh


x y z 34
P= + + ≥
2 x + 3 y y + z z + x 33

( )
⇔ ( 31x − 3 y ) z 2 + 96 y 2 − 35 x 2 − 5 xy z + 31x 2 y − 3 xy 2 ≥ 0

Vế trái là tam thức bậc hai của z với hệ số của z2 dương nên ta chỉ cần
chứng minh

(96 y ) − 4 (31x − 3 y ) (31x y − 3xy ) ≤ 0


2
2
− 35 x 2 − 5 xy 2 2

⇔ ( x − y )( x − 4 y ) (1125 x + 2631xy + 2304 y ) ≤ 0


2 2

n
Bất đẳng thức cuối đúng do x − y ≥ 0; x − 4 y ≤ 0 .

.v
Nhận xét. Với cách làm này ta chỉ cần giả thiết bài toán là x ≥ y .

om
Bài 22. Viết lại bất đẳng thức dưới dạng:
a 2 + 2 ( b + d − 3c ) a + ( b + c + d ) − 8bd > 0 .
2

.c
Vế trái là tam thức bậc hai của a đạt giá trị nhỏ nhất tại 3c − b − d giá trị nhỏ
nhất này bằng
ok
Pmin = ( 3c − b − d )2 − 2 ( 3c − b − d )2 + ( b + c + d )2 − 8bd
= ( b + c + d ) − ( 3c − b − d ) − 8bd
2 2
bo

= 8 ( b + d − c ) c − 8bd = −8 c 2 − ( b + d ) c + bd  = −8 ( c − b )( c − d ) > 0
 
et

Bất đẳng thức được chứng minh.


Vì p 2 + q 2 − a 2 − b 2 − c 2 − d 2 = (p 2
) ( )
− a 2 − b 2 + q 2 − c 2 − d 2 > 0 nên
vi

Bài 23.

tồn tại ít nhất một trong hai số p 2 − a 2 − b 2 , q 2 − c 2 − d 2 dương. Không mất


ng

tính tổng quát ta giả sử p 2 − a 2 − b 2 > 0 .


a

Xét tam thức bậc hai


(p ) (
− a 2 − b 2 x 2 − 2 ( pq − ac − bd ) x + q 2 − c 2 − d 2 . )
kh

f ( x) = 2

− ( aq − pc ) − ( bq − pd ) ≤ 0 nên f ( x) luôn có nghiệm.


2 2
Ta có: p 2 . f (q ) =

(
Suy ra ∆ ' x ≥ 0 ⇔ ( pq − ac − bd ) ≥ p 2 − a 2 − b 2 q 2 − c 2 − d 2 .
2
)( )
Ta có điều phải chứng minh.
Tổng quát với n số thực thoả mãn điều kiện a12 − a22 − ... − an2 > 0 ta có

(a
2
1 )( )
− a22 − ... − an2 b12 − b22 − ... − bn2 ≤ ( a1b1 − a2b2 − ... − an bn ) .
2

92
Cty TNHH MTV DVVH Khang Việt

Bài 24. Thay r =− p − q vào bất đẳng thức cần đưa về chứng minh:

( )
a 2 q ( − p − q ) + b 2 ( − p − q ) p + c 2 pq ≤ 0 ⇔ −a 2 q 2 + p c 2 − b 2 − a 2 q − p 2b 2 ≤ 0 .
Coi vế trái bất đẳng thức trên là tam thức bậc hai của q ta được:

( ) ( )( )
2
∆=
q p 2 c2 − b2 − a 2 − 4 p 2 a 2b=
2
p 2 c 2 − b 2 − a 2 − 2ab c 2 − b 2 − a 2 + 2ab

(
= p2 c2 − ( a + b )
2
)(c 2
− (a − b)
2
)≤0
Do c > a − b , c < a + b . Vậy ta có điều phải chứng minh.
Bài 25. Bất đẳng thức đã cho tương đương với:

n
( )
a 2b 2 + 2 a 2 + b 2 + 4 − 3ab − 3 ( a + b ) ≥ 0

.v
( )
⇔ a 2 b 2 + 2 − 3a ( b + 1) + 2b 2 − 3b + 4 ≥ 0

om
Ta chỉ cần chứng minh rằng

(
∆ a= 9 ( b + 1) − 4 b 2 + 2 2b 2 − 3b + 4
2
)(
.c )
= (
− ( b − 1) 8b 2 + 4b + 23 ≤ 0
2
)
ok
Bất đẳng thức cuối đúng.
Bất đẳng thức được chứng minh. Đẳng thức xảy ra khi và chỉ khi a= b= 1 .
bo

CHỦ ĐỀ 7: KỸ THUẬT ĐÁNH GIÁ BẤT ĐẲNG THỨC TÍCH PHÂN


et

Dưới đây chỉ trình bày một số bài toán bất đẳng thức tích phân mà phương pháp
vi

giải chỉ thông qua các bất đẳng thức đại số rồi lấy tổng tích phân sẽ dẫn đến kết
ng

quả của bài toán.


Bất đẳng thức tích phân là bài toán khá hay để tìm hiểu thêm bạn đọc tìm đọc
các Chuyên đề khác về bất đẳng thức tích phân trên Diễn đàn học tập trực tuyến
a

Mathlinks.vn
kh

A. NỘI DUNG PHƯƠNG PHÁP


Bài toán: Cho f , g , h : [ a, b ] →  là các hàm khả tích trên [ a, b ] đồng thời thỏa
mãn điều kiện g ( x) ≤ f ( x) ≤ h( x) với mọi x ∈ [ a, b ] ta có
b b b

∫ g ( x)dx ≤ ∫ f ( x)dx ≤ ∫ h( x)dx .


a a a
Các bất đẳng thức phụ hay được sử dụng
x3 x3 x5
 sin x < x và x − < sin x < x − + , ∀x > 0 .
3! 3! 5!

93
Khám phá tư duy Kỹ thuật giải bất ĐT Bài toán Max – Min – Đặng Thành Nam

x2 x2 x2 x4
 cos x > 1 − và 1 − < cos x < 1 − + , ∀x > 0 .
2 2 2 4!
x
 < ln (1 + x ) < x .
x +1
Yêu cầu. Các bất đẳng thức đại số như Cô si, Cauchy – Schwarz,… cần nắm vững.
Như vậy dạng bất đẳng thức cơ bản của tích phân chính là quy bài toán về đánh
giá bất đẳng thức trong dấu tích phân.

B. BÀI TẬP MẪU


b

∫(x )

n
2 2 2
Bài 1. [IMC 2011] Chứng minh rằng 2
+ 1 e − x dx ≥ e− a − e −b với mọi số

.v
a
thực a < b .

om
Lời giải
Với mọi số thực x ta luôn có x + 1 ≥ 2 x suy ra
2

b b .c
∫( )
2 2 2 2
x 2 + 1 e − x dx ≥ ∫ 2 xe − x dx =
e − a − e −b .
ok
a a
Bài toán được chứng minh hoàn toàn.
1
bo

x sin x
Bài 2. Chứng minh rằng ∫ 1 + x sin x dx < 1 − ln 2 .
0
et

Lời giải
1
dx
Viết lại bất đẳng thức cần chứng minh dưới dạng: ∫ 1 + x sin x > ln 2 .
vi

0
ng

π
1 1
dx dx
Với mọi x > 0 ⇒ sin x < x ⇒ ∫ >∫ = >ln2.
0
1 + x sin x 0 1 + x 2 4
a

Bài toán được chứng minh hoàn toàn.


kh

 sin x
 x , x ∈ ( 0;1]
Bài 3. Cho hàm số f ( x) được xác định bởi f ( x) =  .
 lim sin x
,x = 0
 x→0+ x
1
17 1703
Chứng minh rằng < ∫ f ( x)dx < .
18 0 1800

94
Cty TNHH MTV DVVH Khang Việt

Lời giải
Rõ ràng với giả thiết hàm số f ( x) đã cho thì f ( x) liên tục trên [ 0;1] .
x3 x3 x5
Tiếp đến sử dụng bất đẳng thức phụ x − < sin x < x − + , ∀x > 0 , ta
6 6 120
x 2 sin x x2 x4
được 1 − < <1− + .
6 x 6 120
Chuyển qua tích phân hai vế của bất đẳng thức trên ta được
1
 x2  1
sin x
1
 x2 x4  17
1
sin x 1703
∫  6  ∫ x
1 − dx < dx < ∫  6 120 
1 − + dx ⇔
18
< ∫ x
dx <
1800
.
   

n
0 0 0 0
Bài toán được chứng minh hoàn toàn.

.v
π
4
1 1

om
Bài 4. Chứng minh rằng < ∫ tan n xdx < với mọi số nguyên dương
2 ( n + 1) 0 2 ( n − 1)
n≥2.

π
Lời giải
.c
ok
4
π
Đặt an = ∫ tan n xdx với mọi 0 ≤ x ≤ ⇒ 0 ≤ tan x ≤ 1 ⇒ tan n x ≤ tan n −1 x
0
4
bo

⇒ an ≤ an −1 tức dãy {an } giảm.


Suy ra an + 2 + an < 2an < an + an − 2 .
et

π
π
( )
4
1 1
vi

Mặt khác ta lại có an + 2=


+ an ∫ tan x 1 + tan = tan n=
n 2
x dx x 4
n +1 n +1
0 0
ng

π
π
( )
4
n−2 1 −2 1
Và an=
− 2 + an ∫ tan x 1 + tan= tan n=
2
a

x dx x 4 .
n −1 n −1
0 0
kh

1 1
Từ đó suy ra < an < .
2 ( n + 1) 2 ( n − 1)
π
4
1 1
Vậy < ∫ tan n xdx < , ∀n ≥ 2 .
2 ( n + 1) 0 2 ( n − 1)
π
4
1
Từ chứng minh trên suy ra lim n ∫ tan n xdx = .
n →∞ 2
0

95
Khám phá tư duy Kỹ thuật giải bất ĐT Bài toán Max – Min – Đặng Thành Nam

4
Bài 5. [VMC 1994] Xét tích phân I n = ∫x 4 − xdx, n ∈ * .
n

0
a) Tính I n .
1

b) Chứng minh rằng I n < 22 n +3 ( 2ne ) 2 .
Lời giải
Ta có
π π π 
4 2  2 2 
I= ∫x
n
= 22 n + 4 ∫ cos 2 n +1t sin 2 tdt= 22 n + 4  ∫ cos 2 n +1tdt − ∫ cos 2 n +3tdt 
4 − xdx

n
n
0 0  0 0 

.v
 
 π
=
với x 4cos 2 t , t ∈ 0,  .

om
 2
π
2
∫ cos
=
Xét tích phân J m
0
m
tdt , m > 1 .
.c
ok
Tích phân từng phần ta được:
π π π
bo

2 2 2
∫ cos td ( sin=
t) ∫ ( m − 1) sin
m −1
=
Jm ∫ cos =
m
tdt 2
tcos m − 2tdt
0 0 0
et

π  π
2 2 
( m − 1)  ∫ cosm−2tdt − ∫ cosmtdt 
=
vi

 0 0 
 
ng

π
m −1 2
⇒ J m = J m − 2 . Ta=
có J1 ∫=
cos tdt 1 suy ra
a

m 0
kh

2 n + 4  2n 2n − 2 2 2n + 2 2n 2 
I n 22 n + 4 ( J 2 n +1 − J 2 n=
= +3 ) 2  . ... .1 − . ... .1
 2n + 1 2n − 1 3 2n + 3 2n + 1 3 
2n ( 2n − 2 ) ...2  2n + 2  2n ( 2n − 2 ) ...2
22 n + 4 = 1 −  2
2n+4
.
( 2n + 1)( 2n − 1) ...3  2n + 3  ( 2n + 3)( 2n + 1)( 2n − 1) ...3
1

Chứng minh I n < 22 n +3 ( 2ne ) 2 .

96
Cty TNHH MTV DVVH Khang Việt

1
2 n +3 1
Đặt x = 4t. =
Khi đó I n 2 ∫t 1 − tdt ta chứng minh t n 1 − t <
n
hay
0 2ne
1
t 2 n (1 − t ) <
.
2ne
Sử dụng bất đẳng thức AM – GM ta được:
2 n +1
 t t t 
+ + ... + + 1 − t 
=
2n 
2n
t  
t 2 n (1 − t ) ( 2n )   (1 − t ) ≤ ( 2n )  2n 2n
2n 2=n ( 2n )2 n .

 
2 n  2 n + 1  ( 2n + 1)2n+1
 

n
Như vậy ta chứng minh

.v
( 2n )2n < 1 ⇔ e <  2n + 1 2n+1 ⇔ 1 < 2n + 1 ln 1 + 1  .
  ( )  
( 2n + 1)2n+1 2ne  2n   2n 

om
x
Dễ dàng chứng minh được ln (1 + x ) > , ∀x > 0 .
x +1 .c
Từ đó ta có điều phải chứng minh.

ok
sin nx 2 1 1 1 
Bài 6. Chứng minh rằng ∫ x
dx >  +
π 1+ n 2 + n
+ ... +  .
2n 
π
bo

Lời giải

sin nx 2 nπ
sin t 2 n −1 ( k +1)π sin t
Đặt t = nx. Khi
= đó I n ∫ = dx =∫ t dt ∑ ∫ dt .
et

π
x nπ k =n kπ
t
( k +1)π
vi

2 n −1
2 n −1
1 2
∑ ( k + 1) π ∫ sin t dt =
> ∑ ( k + 1) π .
ng

=k n= kπ k n

Bài toán được chứng minh hoàn toàn.


a

1
6n 2 + 2n + 1 2n + 1
kh

Bài 7. Chứng minh rằng 2


< ∫ n 1 + xdx < với mọi n ∈ , n ≥ 2 .
6n 0
2n
Lời giải
Ta sẽ chứng minh
x n ( n − 1) 2 n x
1+ − 2
x ≤ 1 + x ≤ 1 + với mọi x ≥ 0, n ∈ * (1.1)
n 2n n
Khi đó lấy tích phân hai vế của (1.1) trên [ 0,1] ta có điều phải chứng minh.

97
Khám phá tư duy Kỹ thuật giải bất ĐT Bài toán Max – Min – Đặng Thành Nam


1
x  x2  1 1 2n + 1
Thật vậy ta có ∫ 1 +  dx = x +  =+ 1 = và
0
n  2n  0 2n 2n
 x 2 ( n − 1) x  1 6n + 3n − ( n − 1) 6n 2 + 2n + 1
1 3 2
 x n −1 2 
∫ 1 + n − 2n2 x  dx =
x+
 2n
− 
6n 2  0
=
6n 2
=
6n 2
.
0 
Việc chứng minh bất đẳng thức (1.1) chỉ việc khảo sát hàm số.
Bình luận: Vì sao lời giải nghĩ đến việc chứng minh bất đẳng thức (1.1)?.
Đây là một kinh nghiệm khi giải toán, đó là dựa vào khai triển Taylor của hàm
số f ( x=
) n
1 + x tại x = 0 ta được

n
x n ( n − 1) 2 n ( n − 1)( n − 2 ) 3
n
1 + x =1 + − x + x + ...

.v
n 2n 2 6n3

(
1 ix
e + e −ix ) (
1 ix
e − e −ix )

om
Cũng tư tưởng này với cos
= x và =
sin x và khai
2 2i
triển Taylor cho hàm e x và ta có các bất đẳng thức sau đây.

x−
x3 x3 x5
< sin x < x − + và 1 −
x2
< cos x < 1 −
.c
x2 x4
+ với mọi x > 0 .
6 6 120 2 2 24
ok
Lúc này chỉ việc lấy tích phân hai vế trên [ a, b ] , ( 0 < a < b ) ta có các bất đẳng
thức hết sức thú vị.
bo


Bài 8. Chứng minh rằng ∫ sin x 2 dx > 0 .
et

0
Lời giải
vi

x3
Trước tiên ta chứng minh: sin x ≥ x − , ∀x ≥ 0
ng

6
x3
Thật vậy xét hàm số f ( x=
) sin x − x + ta có
6
a

x2
kh

f '( x) =cos x − 1 + ⇒ f ''( x) =− sin x + x ⇒ f '''( x) =1 − cos x ≥ 0 .


2
Suy ra f ''( x) ≥ f ''(0) =0 ⇒ f '( x) ≥ f '(0) = 0 ⇒ f ( x) ≥ f (0) = 0.
x3
Tức là sin x ≥ x − , ∀x ≥ 0 .
6
Suy ra
2π 2π
 2 x6   x3 x7  2π 2π 2π 4π 3 2π 2π 2π  2π 
∫ sin x dx ≥ ∫  x −  dx = −  = − = 1 −  > 0 .
2

0 0  6  3 42  0 3 21 3  7 

98
Cty TNHH MTV DVVH Khang Việt

Bài 9. Chứng minh rằng với mỗi n nguyên dương ta luôn có


1
xn + 1 2
∫ xn + xn−1 + ... + x + 1 dx ≥ n + 1 .
0
Lời giải
( )( )
Với mọi k ∈ [ 0, n ] và x ∈ [ 0,1] ta có 1 − x n 1 − x n − k ≥ 0 ⇒ x k + x n − k ≤ x n + 1 .
Cho k = 0,1, 2,..., n và cộng theo vế của n + 1 bất đẳng trên theo vế, suy ra

( )
2 x n + x n −1 + ... + x + 1 ≤ ( n + 1) x n + 1 ⇒( ) x +x
n
xn + 1
n −1
+ ... + x + 1

2
n +1
.

n
1 1
x +1
n
2 2
∫ xn + xn−1 + ... + x + 1 dx ≥ ∫ n + 1 dx =

.v
Suy ra .
0
n +1
0

om
Bài toán được chứng minh hoàn toàn.

8  sin n + 2 x cos n + 2 x 
=
Bài 10. Tìm giá trị nhỏ nhất của biểu thức I n ∫
.c  n
+
sin n x 
dx .
π  cos x
4
ok
Lời giải
 π 3π 
Với mọi x ∈  ,  ⇒ cos x > 0,sin x > 0 khi đó ta có
bo

4 8 
n
(
2( n +1)
x + cos ( ) x
2 n +1
)
sin n + 2 x cos n + 2 x 2 sin
( )
et

≥ 2n sin ( ) x + cos ( ) x .
2 n +1 2 n +1
= n
+ n n
cos x sin x sin 2 x
vi

Mặt khác hàm f (n) = a n +1 là hàm lồi nên ta có

( ) ( )
ng

f sin 2 x + f cos 2 x = sin ( ) x + cos ( ) x


2 n +1 2 n +1

n +1
 cos 2 x + sin 2 x   cos 2 x + sin 2 x 
a

1
≥2f   = 2  =
 2   2 2n
   
kh

Suy ra
3π 3π

( )  sin n + 2 x cos n + 2 x  π
8 8
2n sin ( ) x + cos ( ) x =
2 n +1 2 n +1
≥ 1 ⇒ In ∫  n
+ n 

=
dx ≥ ∫ dx
8
.
π  cos x sin x  π
4 4
Bài 11. Với mọi số thực ai , i = 1, 2,..., n . Chứng minh rằng
ai a j
∑ ≥0.
1≤i ≤ n ,1≤ j ≤ n i + j +1

99
Khám phá tư duy Kỹ thuật giải bất ĐT Bài toán Max – Min – Đặng Thành Nam

Lời giải
2
n  n 
Xét đa thức f ( x) = ∑ ai x ta = ∑ ai xi 
có f ( x) = i 2
∑ ai a j xi + j .
i =1 = i 1  1≤i ≤ n,1≤ j ≤ n

Đến đây lấy tích phân hai vế trên [ 0,1] ta có điều phải chứng minh.

C. BÀI TẬP RÈN LUYỆN


Bài 1. Chứng minh rằng
π
0
3x 2 sin 2 x 3π
a) 2 3 < ∫ x −
3
− 6 x + 5dx < 34 . d) ∫e dx > .

n
2 2
−2 0

.v
π
π π 2
1
dx
b) ≤∫ ≤ . 1 2 sin x ln π − ln 2
∫ dx ≤

om
6 4− x − x
2 3 8 e) .
0 2cos1 1 x π −2
1
3 −x
1 dx 2
π e sin x π
c) ≤∫ ≤ , ∀n ∈ * .
2 0 1 − x 2n 6
f) .c ∫ x +12
dx ≤
12e
.
1
Bài 2. Chứng minh rằng với mọi x > 0 ta luôn có
ok
x
e x − 1 < ∫ e 2t + e −t dt < ( e − 1)  e
x x 1
− .
bo

0
2
π
5 2 sin x
et

3
Bài 3. Chứng minh rằng <∫ dx < .
6 π x 2
vi

6
e
( ln x )2013 dx >
ng

1
Bài 4. Chứng minh rằng ∫ x 2 2014.2015.2016
.
1
a

π
2
( cos x )sin x
kh

Bài 5. Chứng minh rằng ∫ ( cos x )sin x + ( sin x )cos x dx < 1 .


0

Bài 6. Cho 0 < a ≤ b chứng minh rằng


b
x2 + x + 1 1 2b + 1
a) ∫ ( x + 2 )5 dx ≤ ln
54 2a + 1
.
a
b
x4 + x2 + 1 1 ( 2b − 1)( 2a + 1) .
b) ∫ dx ≤ ln
2916 ( 2a − 1)( 2b + 1)
(x )
5
a
2
+ 3x + 2

100
Cty TNHH MTV DVVH Khang Việt

1
e−2 n e −1
Bài 7. Chứng minh rằng 1 + ≤ ∫ e x dx ≤ 1 + với n ∈ * .
n 0
n
1
17
Bài 8. Chứng minh rằng ∫ 1 + x 2 ln 2 ( x + 1)dx ≥ .
0
16
π
4
π
Bài 9. Với mọi số nguyên dương n . Chứng minh rằng ∫ tan xdx >
n
.
0
2nπ + 4
π
π π

n
2
1
Bài 10. Chứng minh rằng < ∫ dx < với mọi k ∈ ( 0,1) .

.v
2 0 1 − k sin x
2 2
2 1− k2
π

om
4
2
 sin nx  n 2π 2
Bài 11. Chứng minh rằng ∫ x   dx < với n ≥ 1 .
0 
sin x  8
Bài 12. Với mọi số nguyên dương n ≥ 2 . .c
1
n+2 n e+n
< ∫ e x dx <
ok
Chứng minh rằng .
n +1 0 n +1
1

∫(x − x )
bo

2 k 2
Bài 13. Với k ≥ 0 chứng minh rằng dx ≤ .
0 4 k
k + 1 + 2k + 1
et

1
In >
,n ≥ 2 .
2ln 2n
vi

Bài 14. Cho các số thực dương a1 , a2 ,..., aN .


ng

N N N
a a
Chứng minh rằng ∑ ∑ m +mn n+ 1 ≤ π ∑ ak2 .
= 0=
m n 0 =
k 1
a
kh

101
Khám phá tư duy Kỹ thuật giải bất ĐT Bài toán Max – Min – Đặng Thành Nam

Chương 2:
BẤT ĐẲNG THỨC VÀ PHƯƠNG PHÁP TIẾP CẬN
CHỦ ĐỀ 1:
CÁC KỸ THUẬT SỬ DỤNG BẤT ĐẲNG THỨC AM – GM CƠ BẢN

A. NỘI DUNG PHƯƠNG PHÁP


I. Bất đẳng thức Cô si (AM-GM) cho 2 số không âm
a+b

n
≥ ab .
2

.v
Đẳng thức xảy ra khi và chỉ khi a = b.
Các dạng tương đương của bất đẳng thức trên

om
a+b
ab ≤ 
2
(a + b) . 2

 và a + b ≥
2 2
 2  .c 2
Ví dụ 1. Chứng minh rằng với mọi số thực dương a và b ta có
ok
1 1 4
+ ≥ .
a b a+b
Lời giải. Sử dụng bất đẳng thức AM-GM cho 2 số ta có
bo

( a + b ) 
1 1 2
+  ≥ 2 ab . =
4.
 a b  ab
et

1 1 4
Do đó + ≥ .
vi

a b a+b
Đẳng thức xảy ra khi và chỉ khi a = b .
ng

II. Bất đẳng thức Cô si (AM-GM) cho 3 số không âm


a+b+c 3
≥ abc .
a

3
kh

Đẳng thức xảy ra khi và chỉ khi a= b= c .


Các dạng tương đương của bất đẳng thức trên
3
a+b+c
abc ≤   .
 3 
Ví dụ 2. Chứng minh rằng với mọi số thực dương a,b,c ta có
1 1 1 9
+ + ≥ .
a b c a+b+c
Lời giải. Sử dụng bất đẳng thức AM-GM cho 3 số dương ta có

102
Cty TNHH MTV DVVH Khang Việt

( a + b + c ) 
1 1 1 3 3
+ +  ≥ 3 abc . 3 =9.
a b c abc
1 1 1 9
Do đó + + ≥ .
a b c a+b+c
Đẳng thức xảy ra khi và chỉ khi a= b= c .
III. Bất đẳng thức Cô si (AM-GM) cho n số không âm
a1 + a2 + ... + an n
≥ a1a2 ...an .
n
Đẳng thức xảy ra khi và chỉ khi a1= a2= ...= an .

n
1. Các bất đẳng thức hay sử dụng
Trước tiên nhắc lại một số bất đẳng thức hay được sử dụng

.v
1
a 2 + b 2 ≥ ( a + b ) ; ( a + b ) ≥ 4ab
2 2

om
2
1
a 2 + b 2 + c 2 ≥ ab + bc + ca; a 2 + b 2 + c 2 ≥ ( a + b + c )
2

.c 3
( a + b + c )2 ≥ 3 ( ab + bc + ca ) ; ( ab + bc + ca )2 ≥ 3abc ( a + b + c )
ok
8
( a + b )( b + c )( c + a ) ≥ ( a + b + c )( ab + bc + ca ) (1)
9
bo

4
a 2b + b 2 c + c 2 a + abc ≤ ( a + b + c )3 (2)
27
Chứng minh (1) và (2) xem bên dưới
et

2. Kỹ thuật đánh giá trung bình cộng qua trung bình nhân
Giả thiết bài toán thường cho tích của các số
vi

Sử dụng bất đẳng thức AM – GM cho n số dạng


ng

a1 + a2 + ... + an n
≥ a1a2 ...an .
n
a

Ví dụ 1. Chứng minh với mọi a,b,c là các số thực ta có


(a )( )( )
kh

2
+ b 2 b 2 + c 2 c 2 + a 2 ≥ 8a 2 b 2 c 2 .
Lời giải
Ta có
a 2 + b 2 ≥ 2 a 2b 2 =
2 ab

b2 + c 2 ≥ 2 b2c 2 =
2 bc

c2 + a2 ≥ 2 c2a2 =
2 ca
Nhân theo vế 3 bất đẳng thức trên ta có điều phải chứng minh.

103
Khám phá tư duy Kỹ thuật giải bất ĐT Bài toán Max – Min – Đặng Thành Nam

Ví dụ 2. (TSĐH Khối D 2005) Cho x,y,z là các số thực dương thoả mãn điều kiện
1 + x3 + y 3 1 + y3 + z3 1 + z 3 + x3
xyz = 1. Chứng minh rằng + + ≥3 3.
xy yz zx
Lời giải
Sử dụng bất đẳng thức AM – GM cho 3 số dương ta được

1 + x3 + y 3 3 3 x3 y 3 .1 3
≥ =
xy xy xy

1 + y3 + z3 3 3 y 3 z 3 .1 3

n
≥ =
yz yz yz

.v
1 + z 3 + x3
3
3 z 3 x3 .1 3
≥ =

om
zx zx zx
Cộng theo vế 3 bất đẳng thức trên ta được
1 + x3 + y 3
+
1 + y3 + z3
+
1 + z 3 + x3  1
≥ 3

.c
+
1
+
1 
.
xy yz zx  xy yz zx 
ok
Mặt khác AM – GM cho 3 số dương ta được
bo

1 1 1 1 1 1
+ + ≥ 33 . . =
3.
xy yz zx xy yz zx
et

Từ đó suy ra điều phải chứng minh. Đẳng thức xảy ra khi và chỉ khi x= y= z= 1 .
Ví dụ 3. Cho a,b,c là các số thực dương có tổng bằng 3.
vi

Chứng minh rằng a ( b + c ) + b ( c + a ) + c ( a + b ) ≥ 3 2abc .


ng

Lời giải
Sử dụng bất đẳng thức AM – GM ta có
a

a ( b + c ) + b ( c + a ) + c ( a + b ) ≥ 36 abc ( a + b )( b + c )( c + a ) .
kh

3
a+b+c
Mặt khác abc ≤   = 1 .
 3 
( a + b )( b + c )( c + a ) ≥ 8abc
Suy ra

a ( b + c ) + b ( c + a ) + c ( a + b ) ≥ 3 8a 2b 2 c 2 ≥ 3 8a 3b3c3 =
6 6
3 2abc .
Bất đẳng thức được chứng minh. Đẳng thức xảy ra khi và chỉ khi a= b= c= 1 .

104
Cty TNHH MTV DVVH Khang Việt

Ví dụ 4. Cho a,b,c là các số thực dương thoả mãn điều a > b; a + b + c =4.
c3
Tìm giá trị nhỏ nhất của biểu thức P = 4a + 3b + .
(a − b)b
Lời giải
Chú ý điều kiện a + b + c= 4, a > b, c3 ta sẽ tách P thành tổng của các số dương
để sử dụng AM – GM sao cho mất đi (a-b) và xuất hiện tổng (a+b+c) vế phải.
Sử dụng bất đẳng thức AM – GM cho ba số dương ta có
c3 c3
P = 4a + 3b + = (a − b) + b + + 3( a + b )
(a − b)b (a − b)b

n
c3

.v
≥ 33 ( a − b ) .b. + 3 ( a + b )= 3 ( a + b + c )= 12
(a − b)b

om
 c3
a − b = b =
Đẳng thức xảy ra khi và chỉ khi  ( a − b ) b ⇔ a = 2; b = c =1
 .c
a + b + c = 4
Vậy giá trị nhỏ nhất của P bằng 12.
ok
Bài tập tương tự
108
bo

Cho a > b > c > 0 chứng minh a + ≥7 .


( a − b )3 ( b − c )2 c
Ví dụ 5. (IMO 2012) Cho a2 , a3 ,..., an là các số thực dương thoả mãn điều kiện
et

a2 a3 ...an = 1 . Chứng minh rằng (1 + a2 ) (1 + a3 ) ... (1 + an ) > n n .


2 3 n
vi

Lời giải
Sử dụng bất đẳng thức AM –GM ta có
ng

k
 
 1 
a

1 1 1
( ak + 1)k = ak + + + ... +  ≥ k .ak .
k
, k =2, n .
 k −
1 k − 1
 k −
 
1 ( k − 1)k −1
kh

 ( k −1) times 
1
Đẳng thức xảy ra khi và chỉ khi
= ak = , k 2, n .
k −1
1
Khi đó a2 a=
3 ...an = 1, vô lý. Vậy dấu bằng không xảy ra khi đó
(n − 1)!
Nhân theo vế (n – 1) bất đẳng thức trên ta được:
22 33 nn
(1 + a2 )2 (1 + a3 )3 ...(1 + an )n > . ... .a a ...an =
n −1 2 3
nn .
1 2 (n − 1)
1 2

105
Khám phá tư duy Kỹ thuật giải bất ĐT Bài toán Max – Min – Đặng Thành Nam

Bất đẳng thức được chứng minh.


Ví dụ 6. Cho x,y,z là các số thực chứng minh
1 ( x + y )( y + z )( z + x )(1 − xy )(1 − yz )(1 − zx ) 1
− ≤ ≤ .
( )( )( )
8 2 2 2 8
1 + x2 1 + y 2 1 + z 2
Lời giải
Sử dụng đẳng thức và bất đẳng thức AM – GM cho hai số ta có
(1 + x )(1 + y ) =( x + y ) + ( xy − 1) ≥ 2 ( x + y )( xy − 1)
2 2 2 2

(1 + y )(1 + z ) =( y + z ) + ( yz − 1) ≥ 2 ( y + z )( zyz − 1)
2 2 2 2

n
(1 + z )(1 + x ) =( z + x ) + ( zx − 1) ≥ 2 ( z + x )( zx − 1)
2 2 2 2

.v
Nhân theo vế ba bất đẳng thức trên ta được

om
( ) (1 + y ) (1 + z )
2 2 2 2 2
8 ( x + y )( y + z )( z + x )(1 − xy )(1 − yz )(1 − zx ) ≤ 1 + x 2

1 ( x + y )( y + z )( z + x )(1 − xy )(1 − yz )(1 − zx ) 1


⇔− ≤ .c ≤
8 1+ x (
2 2
)(
1+ y 2 2
)(
1+ z 2 2
) 8
ok
Bất đẳng thức được chứng minh.
Bài tập tương tự
bo

1 ( x + y )(1 − xy ) 1
Cho x,y là hai số thực. Chứng minh − ≤ ≤ .
(
2 1 + x2 1 + y 2 2 )( )
et

Ví dụ 7. Cho x,y,z là các số thực dương thỏa mãn điều kiện x ≥ max{ y, z}.
vi

x y z
Tìm giá trị nhỏ nhất của biểu thức P = + 2 1 + + 33 1 + .
ng

y z x
Lời giải
Sử dụng bất đẳng thức AM – GM ta có
a
kh

x y z x y z
P≥ +2 2 + 33 2 = + 2 2 4 + 33 2 6
y z x y z x
2x z  2x
=
2 y
y
 + 4 4 + 6 6  + 1 −
z x 
 −3
2  y
( 2−32 ) 6 xz
Sử dụng bất đẳng thức AM – GM ta có
4 6
x y z x4 y  6 z 
+ 4 4 + 6 6 ≥ 1111     =
11 .
y z x y  z   x 
Chú ý giả thiết bài toán ta có

106
Cty TNHH MTV DVVH Khang Việt

 2x
x
y
z
≥ 1; ≤ 1 ⇒ 1 −
x
 ≥ 1 −
2 y 2
2
; −3 ( 2−32 ) 6 xz ≥ −3( 2−32 . )

2  2
Từ đó suy ra P ≥ 11.
2 
+ 1 − ( )
 − 3 2 − 2 =1 + 2 2 + 3 2 .
2 
3 3

Dấu bằng xảy ra khi và chỉ khi x= y= z .


Vậy giá trị nhỏ nhất của P bằng 1 + 2 2 + 3 3 2 .
Bài tập tương tự
Cho x,y,z là các số thực dương thỏa mãn điều kiện x ≥ max{ y, z} .

n
x y z
Tìm giá trị nhỏ nhất của biểu thức P = + 2 1 + + 4 4 1 + .

.v
y z x
3. Đánh giá trung bình nhân qua trung bình cộng

om
a1 + a2 + ... + an
Sử dụng bất đẳng thức AM – GM theo chiều n a1a2 ...an ≤ .
n
Ta có 2 hướng xử lý .c
+ Đánh giá trực tiếp.
ok
+ Nhân thêm hằng số mục đích lược bỏ biến hoặc hằng số không thích hợp.
Nhận dạng bất đẳng thức có dạng: m A1 + m A2 + ... + m An ≤ B .
bo

Khi đó đáp dụng bất đẳng thức AM – GM theo chiều trung bình nhân qua trung
bình cộng cho mỗi căn thức vế trái.
et

Ví dụ 1. Chứng minh rằng với mọi a, b ≥ 1 ta có a b − 1 + b a − 1 ≤ ab .


Lời giải
vi

Nhận xét. Vế phải không chứa hằng số do vậy sử dụng AM – GM để triệt tiêu các
số –1 trong 2 căn thức do đó nhân thêm vào mỗi căn với 1.
ng

Sử dụng bất đẳng thức AM – GM ta có


b − 1 + 1 ab
1 a ( b − 1) .1 ≤ a.
a

a b −= =
2 2
kh

a − 1 + 1 ab
1 b ( a − 1) .1 ≤ b.
b a −= =
2 2
Cộng theo vế 2 bất đẳng thức trên ta có điều phải chứng minh.
Đẳng thức xảy ra khi và chỉ khi a= b= 2 .
Bài tập tương tự

( )
1) Chứng minh rằng x 12 − y + y 12 − x 2 ≤ 12 với x, y là các số thực để các
căn thức có nghĩa.
2) Cho x,y là hai số thực thoả mãn điều kiện x 1 − y 2 + y 1 − x 2 =
1.

107
Khám phá tư duy Kỹ thuật giải bất ĐT Bài toán Max – Min – Đặng Thành Nam

Chứng minh rằng : x 2 + y 2 =


1.
0 ≤ x ≤ 3
Ví dụ 2. Cho  . Tìm giá trị lớn nhất của biểu thức
0 ≤ y ≤ 4
( 3 − x )( 4 − y )( 2 x + 3 y ) .
P=
Lời giải
Nhận xét. Ta cần triệt tiêu đi x và y vì vậy nhân thêm vào (3 − x) và (4 − y ) lần
lượt thành 2 ( 3 − x ) ,3 ( 4 − y ) khi đó sử dụng AM – GM cho 3 số dương.
Sử dụng bất đẳng thức AM – GM cho 3 số không âm ta được

n
1
P =  2 ( 3 − x )  . 3 ( 4 − y )  . ( 2 x + 3 y )

.v
6
1  2 (3 − x ) + 3( 4 − y ) + ( 2 x + 3 y ) 
3

om
≤   =
36
6 3 
Đẳng thức xảy ra khi và chỉ khi=
x 0,=
y 2 . Vậy giá trị lớn nhất của P bằng 36.
Bài tập tương tự
.c
5
Cho 0 ≤ x ≤ ,0 ≤ y ≤ 3 . Tìm giá trị lớn nhất của biểu thức
ok
2
P =( 5 − 2 x )( 3 − y )( 2 x + 3 y − 5 ) .
bo

Ví dụ 3. Cho a,b,c là các số thực không âm thỏa mãn điều kiện a + b + c =


1.
Tìm giá trị lớn nhất của biểu thức P = ( a + 2b + 3c )( 6a + 3b + 2c ) .
et

Lời giải
Với các hệ số lệch nhau trước tiên giảm biến bằng phép thế c =1 − a − b. Khi đó
vi

đưa về P =( 3 − 2a − b )( 2 + 4a + b ) .
ng

Ta thấy nếu áp dụng trực tiếp AM – GM cho hai số triệt tiêu được b tuy nhiên
còn dư 2a không đánh giá được vì vậy nghĩ đến triệt tiêu a bằng cách nhân thêm
a

2 vào (3 − 2a − b) .
kh

Ta có: P = ( a + 2b + 3 (1 − a − b ) ) ( 6a + 3b + 2 (1 − a − b ) )
 b
=( 3 − 2a − b )( 2 + 4a + b ) =2 ( 3 − 2a − b ) 1 + 2a + 
 2
2 2
 b  b
 3 − 2a − b + 1 + 2a + 2  4− 2 
≤ 2  =
2.   =
8
 2   2 
   
1
Vậy giá trị lớn nhất của P bằng 8 đạt tại a= c= , b= 0 .
2
108
Cty TNHH MTV DVVH Khang Việt

Ví dụ 4. Cho a,b,c là các số thực không âm có tổng bằng 3.


Tìm giá trị lớn nhất của biểu thức P =(5a + b)(b 2 + 4ac) .
Lời giải
b b b
Ta có: 5a + b ≤ 5(a + ); b 2 + 4ac =b 2 + 2(ab + bc) + 4ac =4(a + )(c + ) .
2 2 2
Sử dụng bất đẳng thức AM – GM ta có:
b 2a + 2b + 2c 3
P ≤ 10(a + ) 2 (2c + b) ≤ 10( ) = 80 .
2 3
Dấu bằng xẩy ra khi =b 0;= a 2= c 2.

n
Ví dụ 5. Chứng minh rằng với mọi số thực dương a và b ta có

( ) ( )

.v
2a ( a + b ) + b 2 a 2 + b 2 ≤ 3 a 2 + b 2 .
3

om
Lời giải
Trước hết chuyển bất đẳng thức về dạng một biến bằng cách chia hai vế bất
đẳng thức cho b 2 và đưa về chứng minh bất đẳng thức: .c
(
2 x ( x + 1) + 2 x 2 + 1 ≤ 3 x 2 + 1 .
3
) ( )
ok
Chú ý dấu bằng đạt tại x = 1 và bậc vế phải là bậc hai do vậy ta cần tách
2 x ( x + 1) = 2 x ( x + 1) . ( x + 1) để sử dụng bất đẳng thức AM – GM thuận tiện
3 2
bo

cho việc so sánh với vế phải.


Sử dụng bất đẳng thức AM – GM ta có
et

2 x ( x + 1) + ( x + 1)
2
2 x ( x + 1)= 2 x ( x + 1) . ( x + 1) ≤
3 2
vi

( )
2 + x +1 2
ng

2 x2 + 1 ≤
2
Cộng theo vế hai bất đẳng thức trên ta được
a

( x + 1)2 + 2 x ( x + 1) + x 2 + 3 .
(
2 x ( x + 1) + 2 x 2 + 1 ≤ )
kh

3
2
Vậy ta chỉ cần chứng minh
( x + 1) + 2 x ( x + 1) + x 2 + 3
2

( )
3 x +1
2

2
⇔ 6 ( x + 1) ≥ 4 x + 4 x + 4 ⇔⇔ 2 ( x − 1) ≥ 0
2 2 2

Bất đẳng thức cuối đúng và ta có đpcm.


Đẳng thức xảy ra khi và chỉ khi a = b .

109
Khám phá tư duy Kỹ thuật giải bất ĐT Bài toán Max – Min – Đặng Thành Nam

Ví dụ 6. Cho a,b,c là các số thực dương chứng minh


( )
81abc a 2 + b 2 + c 2 ≤ ( a + b + c ) .
5

Lời giải
Sử dụng bất đẳng thức 3abc ( a + b + c ) ≤ ( ab + bc + ca ) .
2

( )
Ta chỉ cần chứng minh 27 a 2 + b 2 + c 2 ( ab + bc + ca ) ≤ ( a + b + c ) .
2 6

Thật vậy sử dụng bất đẳng thức AM – GM cho ba số dương ta được


3
 a 2 + b 2 + c 2 + 2 ( ab + bc + ca ) 
( 2 2 2
)
a + b + c ( ab + bc + ca ) ≤ 

2
=

1
( a + b + c )6 .

n
 3  27

.v
Bất đẳng thức được chứng minh. Đẳng thức xảy ra khi và chỉ khi a= b= c .
Bài tập tương tự

om
Cho a,b,c là các số thực dương có tích bằng 1. Chứng minh
a + b + c 5 a 2 + b2 + c2
≥ . .c
3 3
Ví dụ 7. Cho a,b,c là các số thực dương. Chứng minh rằng
ok
a+b 3 a+b a+b+c
a + 3 ab.
+ abc ≤ 33 a. . .
2 2 3
bo

Lời giải
Bất đẳng thức đã cho tương đương với
et

6a 2 3b 6bc
3 +3 +3 ≤3.
( a + b )( a + b + c ) a + b + c ( a + b )( a + b + c )
vi

Sử dụng bất đẳng thức AM – GM cho ba số dương ta có


ng

6a 2 2a 3a 1 2a 3a 
3 = 3 1. . ≤ 1 + + 
( a + b )( a + b + c ) a + b a + b + c 3 a + b a + b + c 
a

1 3b 
kh

3b 3b
=
3 3 1.1. ≤ 1 + 1 + 
a+b+c a + b + c 3 a+b+c
6bc 2b 3c 1 2b 3c 
3 = 3 1. . ≤ 1 + + 
( a + b )( a + b + c ) a + b a + b + c 3 a + b a + b + c 
Cộng theo vế ba bất đẳng thức trên ta có đpcm. Đẳng thức xảy ra khi và chỉ khi
a= b= c .
Bài tập tương tự
1) Cho a,b,c là các số thực dương. Chứng minh

110
Cty TNHH MTV DVVH Khang Việt

a + 3 ab + 3 abc 3 a + b a + b + c
≤ a. . .
3 2 3
2) Cho a,b,c,d là các số thực dương. Chứng minh
3a bc 2b3 d 25
2 + 33 + 44 ≤ .
a+b+c ( a + b )( a + b + c + d ) 81( a + b ) ( a + b + c + d ) 6
3

Ví dụ 7. Chứng minh rằng với mọi x,y,z không âm ta có


x3 + y 3 ( x + y ) + ( y + z ) + ( z + x )
3 3 3
y3 + z3 z 3 + x3
yz 3 + zx 3 + xy 3 ≤ .
2 2 2 8

n
Lời giải
Sử dụng bất đẳng thức AM – GM ta có

.v
=
y3 + z3 y+z
(
. yz. yz. yz. y 2 − yz + z 2 )

om
yz 3 3
2 2
4
y + z  3 yz + y 2 − yz + z 2 
3
 y+z
≤3 .  =
 
2  
4   2 
.c
ok
3 3
z 3 + x3  z + x  3 x + y ≤ x+ y .
3 3
Tương tự ta có zx 3 ≤  ; xy  
2  2  2  2 
bo

Cộng theo vế ba bất đẳng thức trên ta có đpcm. Đẳng thức xảy ra khi và chỉ khi
x= y= z .
et

Ví dụ 8. Cho a,b,c là các số thực không âm chứng minh rằng


( a + b )2 ( b + c )2 ( c + a )2 ≥ 4 ( a 2 + bc )( b2 + ca )( c 2 + ab ) .
vi

Lời giải
ng

Không mất tính tổng quát giả sử a ≥ b ≥ c. Khi đó a 2 + bc ≤ a 2 + ac ≤ ( a + c ) .


2

(
Vậy ta chứng minh ( a + b ) ( b + c ) ≥ 4 b 2 + ca c 2 + ab .
2 2
)( )
a

Thật vậy sử dụng bất đẳng thức AM – GM ta có


kh

( )( ) ( )
2
4 b 2 + ca c 2 + ab ≤ b 2 + ca + c 2 + ab

= (b )
2
2
+ ab + bc + ca + c 2 − bc

( ( b + a )( b + c ) + c ( c − b ) )
2
=

≤ (b + a ) (b + c )
2 2

Bất đẳng thức được chứng minh. Đẳng thức xảy ra khi và chỉ khi =
a b=
,c 0
hoặc các hoán vị.

111
Khám phá tư duy Kỹ thuật giải bất ĐT Bài toán Max – Min – Đặng Thành Nam

Bài tập tương tự


Cho a,b,c là các số thực không âm thoả mãn điều kiện
( a + b )( b + c )( c + a ) =
2
Tìm giá trị lớn nhất của biểu thức P = ( )( )(
a 2 + bc b 2 + ca c 2 + ab . )
Ví dụ 9. Cho a, b, c là các số thuộc đoạn [0, 1].
Tìm giá trị lớn nhất của biểu thức A = ( a − b )( b − c )( c − a )( a + b + c ) .
Lời giải
Giả sử a = max {a, b, c} .
Nếu a ≥ b ≥ c ⇒ A ≤ 0 .

n
Nếu a ≥ c ≥ b .

.v
2(c − b) ⋅ (1 + 3)(a − c) ⋅ ( −1 + 3)(a + b + c)
Khi đó A ≤ .
4

om
Sử dụng bất đẳng thức AM-GM cho ba số dương ta có :
2(c − b) ⋅ (1 + 3)(a − c) ⋅ (−1 + 3)(a + b + c)

( ) ( )
.c 3
 2 ( c − b ) + 1 + 3 ( a − c ) + −1 + 3 ( a + b + c ) 
≤  .
ok
 3 
 
3 3
 −3b + 2 3a + 3b   2 3  8 3
bo

=   =
≤  
 3   3  9
et

2 3 1
Suy ra Pmax = tại (a, b, c) = (1,0, ) .
9 3
vi

Chú ý. Ta có thể mở rộng đoạn [0;1] và giải bài toán bằng tính đơn điệu của hàm
số(xem chương 3).
ng

4. Kỹ thuật ghép cặp


Xem chi tiết trong chủ đề sau
Ví dụ 1. Cho a,b,c là các số thực dương có tổng bằng 1. Chứng minh
a

a + bc b + ca c + ab
kh

+ + ≥2.
b+c c+a a+b
Lời giải
Chú ý điều kiện tổng bằng 1 ta có
a + bc b + ca c + ab
+ + =
b+c c+a a+b
a ( a + b + c ) + bc b ( a + b + c ) + ca c ( c + a + b ) + ab
= + +
b+c c+a a+b

=
( a + b )( a + c ) + ( b + c )( b + a ) + ( c + a )( c + b )
b+c c+a a+b

112
Cty TNHH MTV DVVH Khang Việt

Sử dụng bất đẳng thức AM – GM cho 2 số dương ta có


( a + b )( a + c ) + ( b + c )( b + a ) ≥ 2 a + b
( )
b+c c+a
( b + c )( b + a ) + ( c + a )( c + b ) ≥ 2 b + c
( )
c+a a+b
( a + b )( a + c ) + ( c + a )( c + b ) ≥ 2 c + a
( )
b+c a+b
Cộng theo vế 3 bất đẳng thức trên ta có điều phải chứng minh. Đẳng thức xảy ra
1
khi và chỉ khi a= b= c= .

n
3
Ví dụ 2. Cho a,b,c là các số thực dương. Chứng minh

.v
1 + a2 1 + b2 1 + c2
+ + ≥3.

om
b+c c+a a+b
Lời giải
Sử dụng bất đẳng thức AM – GM ta có .c
1 + a2 1 + b2 1 + c2 1 + a 2 1 + b2 1 + c2( )( )( )
ok
+ + ≥3 6 .
b+c c+a a+b ( a + b )( b + c )( c + a )
Ta chỉ cần chứng minh
bo

(1 + a )(1 + b )(1 + c ) ≥ 1
2 2 2

( a + b )( b + c )( c + a )
et

⇔ (1 + a 2 )(1 + b 2 )(1 + c 2 ) ≥ ( a + b )( b + c )( c + a )
vi

( )(
Tới đây ta đánh giá ghép cặp 1 + a 2 1 + b 2 ≥ ( a + b ) . ) 2
ng

Chú ý sử dụng bất đẳng thức C–S ta có:


(1 + a )(1 + b ) = (1 + a )(b + 1) ≥ ( a + b ) ;
a

2 2 2 2 2
kh

(1 + b )(1 + c ) = (1 + b )( c + 1) ≥ ( b + c ) ;
2 2 2 2 2

(1 + c )(1 + a ) = (1 + c )( a + 1) ≥ ( a + c ) .
2 2 2 2 2

Nhân theo vế ba bất đẳng thức trên ta có đpcm.


Đẳng thức xảy ra khi và chỉ khi a= b= c= 1 .

( )( )
Chú ý. Ta có đẳng thức sau: 1 + x 2 1 + y 2 =( x + y ) + ( xy − 1) .
2 2

Từ đó cũng có ngay bất đẳng thức trên tương tự như dùng C–S.
Bằng cách tương tự ta chứng minh được với mọi k dương ta có

113
Khám phá tư duy Kỹ thuật giải bất ĐT Bài toán Max – Min – Đặng Thành Nam

k + a2 k + b2 k + c2
+ + ≥ 34 k .
b+c c+a a+b
5. Kỹ thuật đánh giá mẫu số
Các bất đẳng thức hay được sử dụng để đánh giá mẫu số
x3 + y 3 ≥ xy ( x + y ) ; x5 + y 5 ≥ x 2 y 2 ( x + y ) .
Chú ý dạng toán này thường cho các điều kiện ràng buộc ta cần tinh tế phân tích
mẫu số thành nhân tử dựa vào điều kiện.
Chú ý nếu mẫu số có chứa căn hoặc tổng các biến ta có thể đặt mỗi mẫu số là
một ẩn khi đó đưa về sử dụng bất đẳng thức AM – GM đơn giản hơn.

n
Ví dụ 1. Cho a,b,c là các số thực dương.

.v
1 1 1 a+b+c
Chứng minh rằng 2 + 2 + 2 ≤ .
a + bc b + ac c + ab 2abc

om
Lời giải
Áp dụng bất đẳng thức AM – GM ta có
2 1.c1 1 1 
a 2 + bc ≥ 2a bc ⇒ ≤ ≤  + .
a + +bc
2
a bc 2  ab ac 
ok
2 1 11 1 
Tương tự ta có: ≤
≤  + 
b + ac b ac 2  bc ab 
2
bo

2 1 1 1 1 
≤ ≤  + 
c + ab c ab 2  ac bc 
2
et

Cộng theo vế ba bất đẳng thức trên ta được


2 2 2 a+b+c
+ 2 + 2 ≤
vi

.
a + bc b + ac c + ab
2 2abc
Bất đẳng thức được chứng minh. Đẳng thức xảy ra khi và chỉ khi a= b= c .
ng

Ví dụ 2. Cho a,b,c là các số thực dương. Chứng minh


1 1 1 1
a

+ 3 3 + 3 ≤ .
a + b + abc b + c + abc c + a + abc abc
3 3 3
kh

Lời giải
Ta có
( )
a3 + b3 = ( a + b ) a 2 − ab + b 2 = ab ( a + b ) + ( a + b )( a − b ) ≥ ab ( a + b )
2

1 1 1 c
⇒ ≤ = =
a + b + abc
3 3 ab ( a + b ) + abc ab ( a + b + c ) abc ( a + b + c )

114
Cty TNHH MTV DVVH Khang Việt

1 a
Tương tự ta có ≤
b + c + abc
3 3 abc ( a + b + c )
1 b

c + a + abc
3 3 abc ( a + b + c )
Cộng theo vế 3 bất đẳng thức trên ta có điều phải chứng minh. Đẳng thức xảy ra
khi và chỉ khi a= b= c .
Bài tập tương tự
1) Cho x,y,z là các số thực dương có tích bằng 1. Chứng minh
1 1 1
+ + ≤1.
x + y +1 y + z +1 z + x +1

n
.v
HD: Đặt
= x a=
3
, y b=
3
, z c3 đưa về bất đẳng thức trên.
2) Cho x,y,z là các số thực dương có tích bằng 1. Chứng minh

om
xy yz zx
+ 5 + 5 ≤1.
x + y + xy y + z + yz z + x5 + zx
5 5 5

Ví dụ 3. Cho a,b,c là các số thực dương thoả mãn điều kiện a 2 + b 2 + c 2 =


1.
.c
ok
a b c 3 3
Chứng minh rằng + + ≥ .
b +c
2
c +a 2
a +b 2 2 2 2 2
bo

Lời giải
Chú ý điều kiện ta có thể viết bất đẳng thức dưới dạng
a b c 3 3 2
( )
et

+ + ≥ a + b2 + c2 .
b +c2 2
c +a2 2
a +b
2 2 2
vi

a 3 3 2
Điều này làm ta suy nghĩ đánh giá ≥ a .
b +c 2 2 2
ng

a a a2 2a 2
Thật vậy ta có = = = .
b2 + c2 1 − a 2 a 1 − a 2 ( ) 2a 1 − a2
( 2
)(1 − a )
2
a
kh

2a 2 3 3a 2
≥ =
3 2
 2a 2 + 1 − a 2 + 1 − a 2 
 
 3 
3 3b 2
b c 3 3c 2
Tương tự ta có ;≥ ≥
c2 + a2 2 a 2 + b2 2
Cộng theo vế 3 bất đẳng thức trên ta có điều phải chứng minh.
1
Đẳng thức xảy ra khi và chỉ khi a= b= c= .
3

115
Khám phá tư duy Kỹ thuật giải bất ĐT Bài toán Max – Min – Đặng Thành Nam

Bài tập tương tự


Cho a,b,c là các số thực dương. Chứng minh
a b c 3 3
+ + ≥ .
b +c2 2
c +a
2 2
2 a 2 + b2 + c2 a +b2 2

Ví dụ 4. Cho a,b,c là các số thực dương có tổng bằng 1.


ab bc ca 1
Chứng minh rằng + + ≤ .
c + ab a + bc b + ca 2
Lời giải
Theo giả thiết ta có

n
ab ab ab 1  ab ab 
= = ≤  + ;

.v
c + ab c ( a + b + c ) + ab ( c + a )( c + b ) 2  c + a c + b 
1  bc bc 

om
bc bc
= ≤  + ; .
a + bc ( a + b )( a + c ) 2  c + a a + b 
ca ca 1  ca ca 
= ≤  + .c .
b + ca ( b + c )( b + a ) 2  a + b b + c 
ok
Cộng theo vế 3 bất đẳng thức trên ta được
ab bc ca a+b+c 1
+ + ≤ = .
bo

c + ab a + bc b + ca 2 2
1
Bất đẳng thức được chứng minh. Đẳng thức xảy ra khi và chỉ khi a= b= c= .
et

3
Bài tập tương tự
vi

Cho a,b,c là các số thực dương có tổng bằng 1. Chứng minh


1 1 1 1
ng

+ + ≤ .
a + bc b + ca c + ab 6abc
Ví dụ 5. Cho x,y,z là các số thực dương thoả mãn điều kiện xyz = 1 .
a

1 1 1 1
+ + ≤
kh

Chứng minh rằng .


x + 2y + 3
2 2
y + 2z + 3
2 2
z + 2x + 3
2 2 2
Lời giải
Sử dụng bất đẳng thức AM – GM ta có
x 2 + y 2 ≥ 2 xy;
y 2 + 1 ≥ 2 y.
Cộng lại theo vế ta được: x 2 + 2 y 2 + 3 ≥ 2 ( x + xy + 1) .
1 1 1
Suy ra ≤ . .
x + 2 y + 3 2 x + xy + 1
2 2

116
Cty TNHH MTV DVVH Khang Việt

1 1 1
Tương tự ta có ≤ .
y + 2 z + 3 2 y + yz + 1
2 2

1 1 1
≤ .
z + 2 x + 3 2 z + zx + 1
2 2

Cộng theo vế 3 bất đẳng thức trên và gọi P là biểu thức vế trái của bất đẳng thức
1 1 1 1 
ta được P ≤  + + .
2  x + xy + 1 y + yz + 1 z + zx + 1 
1 1 1
Vậy ta cần chứng minh + + ≤1.
x + xy + 1 y + yz + 1 z + zx + 1

n
Tuy nhiên đây là một đẳng thức luôn đúng, thật vậy

.v
1 1 1
+ +
x + xy + 1 y + yz + 1 z + zx + 1

om
1 1 1
= + +
x + xy + 1 y + y. 1 + 1 1 + 1 .x + 1

1 x
xy xy xy
xy
.c
= + + =1
ok
x + xy + 1 xy + 1 + x 1 + x + xy
Bất đẳng thức được chứng minh. Đẳng thức xảy ra khi và chỉ khi x= y= z= 1 .
bo

Nhận xét. Bạn đọc lưu ý đẳng thức quan trọng với x,y,z có tích bằng 1 ta có
1 1 1
+ + = 1.
et

x + xy + 1 y + yz + 1 z + zx + 1
Bài tập tương tự
vi

1) Cho x,y,z là các số thực dương thoả mãn điều kiện xyz = 8 .
ng

1 1 1 1
Chứng minh rằng + + ≤ .
2x + y + 6 2 y + z + 6 2z + x + 6 4
a

2) Cho x,y,z là các số thực dương có tích bằng 1. Chứng minh


kh

1 1 1 1
+ 5 + 5 ≤ .
x − x + 3 xy + 6 y − y + 3 yz + 6 z − z + 3 zx + 6 3
5 2 2 2

1 1 1
Ví dụ 6. Cho a,b,c là các số thực dương thoả mãn điều kiện + + =1.
a b c
a2 b2 c2 a+b+c
Chứng minh rằng + + ≥ .
a + bc b + ca c + ab 4
Lời giải
Chú ý điều kiện được viết lại dưới dạng: abc = ab + bc + ca .

117
Khám phá tư duy Kỹ thuật giải bất ĐT Bài toán Max – Min – Đặng Thành Nam

a2 a3 a3 a3
Khi đó= = = .
a + bc a 2 + abc a 2 + ab + bc + ca ( a + b )( a + c )
Sử dụng bất đẳng thức AM – GM ta có
a3 a + b a + c 3a a3 4a − b − c
+ + ≥ ⇒ ≥ .
( a + b )( a + c ) 8 8 4 ( a + b )( a + c ) 8
b2 4b − a − c c 2 4c − a − b
Tương tự ta có ≥ ; ≥ .
b + ca 8 c + ab 8
Cộng theo vế ba bất đẳng thức trên ta có đpcm. Đẳng thức xảy ra khi và chỉ khi
a= b= c= 3 .

n
Ví dụ 7. Cho a,b,c là các số thực dương.

.v
Tìm giá trị nhỏ nhất của biểu thức
a + 3c 4b 8c

om
P= + − .
a + 2b + c a + b + 2c a + b + 3c
Lời giải
 x =a + 2b + c a =− x + 5 y − 3 z .c
 
Đặt  y = a + b + 2c ⇒ b = x − 2 y + z .
ok
 z = a + b + 3c c = z − y
 
−x + 2 y 4( x − 2 y + z ) 8( z − y )
bo

Khi đó P = + −
x y z
2 y 4x 4z 8 y
= + + + − 17
et

x y y z
 y 2x   z 2y 
vi

= 2 +  + 4 +  − 17
x y  y z 
ng

Sử dụng bất đẳng thức AM – GM ta có


y 2x z 2y
P≥4 +8 . − 17= 12 2 − 17 .
a

.
x y y z
kh

 y 2x
x y=
=a 5 2 − 7 x

( )
 y = 2 x
Đẳng thức xảy ra khi và chỉ khi 

⇔

⇒ b = 3 − 2 2 x . ( )
 z = 2y = z =2 y 2x 
 y z =

c 2− 2 x ( )
Vậy giá trị nhỏ nhất của P bằng 12 2 − 17 .
Bài tập tương tự
Cho x,y là các số thực dương.
Tìm giá trị nhỏ nhất của biểu thức

118
Cty TNHH MTV DVVH Khang Việt

x+3 4y 8
P= + − .
x + 2y +1 x + y + 2 x + y + 3
6. Kỹ thuật nghịch đảo
Một số trường áp dụng trực tiếp AM – GM làm bất đẳng thức đổi chiều vì vậy
ta khéo léo chia phân thức cho tử thức áp dụng AM – GM. Kỹ thuật này hay được
áp dụng với bất đẳng thức Bernoulli(xem chủ đề sau).
Ví dụ 1. Cho a,b,c là các số thực dương chứng minh
a3 b3 c3
+ + ≥1.
a3 + ( b + c ) b3 + ( c + a ) c3 + ( a + b )
3 3 3

n
Lời giải

.v
Sử dụng bất đẳng thức AM –GM ta có

( x + 1) ( x 2 − x + 1) ≤
x2 − x + 1 + x + 1 x2 + 2

om
1 + x3= = .
2 2
Đẳng thức xảy ra khi và chỉ khi x = 0 hoặc x = 2 .

Ta có =
a3 1

.c 2
a3 + ( b + c )
3 2
b+c
3
b+c
ok
1+     +2
 a   a 
bo

2a 2 2a 2 a2
= ≥ =
2a 2 + ( b + c )
2
2a 2 + 2 b 2 + c 2 (
a 2 + b2 + c2 )
et

b3 b2
Tương tự ta có ≥
b3 + ( c + a )
vi

3
a 2 + b2 + c2
ng

c3 c2

c3 + ( a + c ) a 2 + b2 + c2
3
a

Cộng theo vế 3 bất đẳng thức trên ta có điều phải chứng minh.
kh

Đẳng thức xảy ra khi và chỉ khi a= b= c.


Bài tập tương tự
Cho a,b,c là các số thực dương thoả mãn điều kiện abc = 8.
a2 b2 c2 4
Chứng minh + + ≥ .
(1 + a )(1 + b ) (1 + b )(1 + c ) (1 + c )(1 + a )
3 3 3 3 3 3 3

Ví dụ 2. Cho a,b,c là các số thực dương chứng minh rằng


a b c
+ + >2.
b+c c+a a+b

119
Khám phá tư duy Kỹ thuật giải bất ĐT Bài toán Max – Min – Đặng Thành Nam

Lời giải
Sử dụng bất đẳng thức AM – GM ta có
a 2a 2a
= ≥ ;
b + c 2 a (b + c ) a + b + c

b 2b 2b
= ≥ ;
c + a 2 b (c + a) a + b + c

c 2c 2c
= ≥ .
a + b 2 c (a + b) a + b + c
Cộng theo vế 3 bất đẳng thức trên ta được

n
.v
a b c  a b c 
+ + ≥ 2 + + =2.
b+c c+a a+b a+b+c a+b+c a+b+c

om
Đẳng thức xảy ra khi và chỉ khi a =+
b c, b =+
c a, c =+
a b
⇒ a + b + c= 2 ( a + b + c ) vô lý. Vậy đẳng thức không xảy ra ta có điều phải
chứng minh.
Bài tập tương tự
.c
ok
1) Cho a,b,c là các số thực dương. Chứng minh
2 2 2
 2a   2b   2c 
3
  +3  +3  ≥ 3.
bo

b+c c+a a+b


2) Cho a,b,c là các số thực không âm thoả mãn điều kiện a 2 + b 2 + c 2 =
2.
et

1 + bc 1 + ca c
Chứng minh rằng + + ≥2.
(b + c ) 2
(c + a) 2 a+b
vi

3) Cho x,y,z là các số thực không âm và tổng đôi một khác 0. Chứng minh rằng
ng

x y z
+ + ≥ 2.
y +z
2 2
z +x
2 2
x + y2
2
a

4) Cho a,b,c là các số thực không âm thoả mãn điều kiện ab + bc + ca > 0. Chứng minh
kh

a b c
+ +2 ≥ 2.
b+c 2c + a a+b+c
5) Cho x,y là các số thực không âm. Chứng minh rằng
x y 1
+ + ≥ 2.
y +1 x +1 x+ y
Ví dụ 3. Cho a,b,c là các số thực dương có tổng bằng 3. Chứng minh
a 2 + b2 + c b2 + c2 + a c2 + a2 + b
+ + ≥ 3.
a + b + c2 b + c + a2 c + a + b2

120
Cty TNHH MTV DVVH Khang Việt

Lời giải
Sử dụng bất đẳng thức AM – GM cho hai số dương ta có

=
a 2 + b2 + c a 2 + b2 + c

(
2 a 2 + b2 + c )
a + b + c2 ( a + b + c )( a + b
2 2 2
+c ) a + b + c + a 2 + b2 + c2

b2 + c2 + a 2 (b + c + a )
2 2
c2 + a2 + b (
2 c2 + a2 + b )
≥ ; ≥
b+c+a a+b+c+a +b +c
2
c+a+b 2
a + b + c + a + b + c2
2 2 2 2 2

Cộng theo vế ba bất đẳng thức trên và đưa về chứng minh


( )
4 a 2 + b2 + c2 + 2 ( a + b + c )

n
≥ 3 ⇔ a 2 + b2 + c2 ≥ 3 .
a+b+c+a +b +c 2 2 2

.v
Bất đẳng thức cuối đúng theo AM – GM.
Bài toán được chứng minh. Đẳng thức xảy ra khi và chỉ khi a= b= c= 1 .

om
Bài tập tương tự
Cho a,b,c,d là các số thực dương. Chứng minh
a
+
b
+
c
+
d
≥2.
.c
b +c +d
2 2 2
c +d +a
2 2 2
d +a +b
2 2 2
a + b2 + c2
2
ok
Ví dụ 4. Cho a,b,c là các số thực không âm thoả mãn điều kiện .
Tìm giá trị nhỏ nhất của biểu thức
bo

a (b + c ) b (c + a) c (a + b)
P= + + .
a 2 + bc b 2 + ca c 2 + ab
et

Lời giải
vi

Chú ý

( ) (
a ( b + c ) = a b 2 + c 2 + 2bc ≥ a b 2 + c 2
2
)
ng

( a + bc ) ( b + c=) b ( a + c ) + c ( a + b )
2 2 2 2 2
a

Do vậy nếu đặt x =a ( b + c ) ; y =b ( c + a ) ; z =c ( a


2 2 2 2 2
)
+ b 2 ta có
kh

a (b + c ) x b (c + a) y c (a + b) z
≥ ; ≥ ; ≥ .
a + bc
2 y + z b 2 + ca z + x c 2 + ab x+ y
Như vậy sau khi cộng theo vế ba bất đẳng thức trên đưa về bài toán đã trình bày.
x y z
+ + ≥ 2.
y+z z+x x+ y
Ta có giá trị nhỏ nhất của P bằng 2 đạt tại=
a b=
, c 0 hoặc các hoán vị.
Cách 2: Nếu có một số bằng 0, không mất tính tổng quát giả sử là c, khi đó

121
Khám phá tư duy Kỹ thuật giải bất ĐT Bài toán Max – Min – Đặng Thành Nam

( )
2
a b a− b
P= + 2 ≥ 2. + =
ab b a
Dấu bằng xảy ra khi và chỉ khi=
a b= ,c 0 .
Nếu cả ba số đều dương khi đó sử dụng bất đẳng thức AM – GM ta có
a (b + c ) a (b + c ) a (b + c ) 2a ( b + c )
= ≥ = .
a 2 + bc (a 2
)
+ bc a ( b + c ) a 2 + bc + a ( b + c ) ( a + b )( a + c )
2
b (c + a) 2b ( c + a ) c (a + b) 2c ( a + b )
Tương tự ta có ≥ ; ≥ .

n
b + ca
2
( b + a )( b + c ) c + ab
2
( c + a )( c + b )

.v
Cộng theo vế ba bất đẳng thức trên ta được:
a (b + c ) b (c + a) c (a + b) 2a ( b + c ) 2b ( c + a ) 2c ( a + b )

om
+ + ≥ + +
a + bc
2
b + ca
2
c + ab
2
( a + b )( a + c ) ( b + a )( b + c ) ( c + a )( c + b )
2a ( b + c ) + 2b ( c + a ) + 2c ( a + b )
2 2 2
= .c
( a + b )( b + c )( c + a )
2 ( a + b )( a + c )( b + c ) + 8abc
ok
=
( a + b )( b + c )( c + a )
bo

8abc
=
2+ >2
( a + b )( b + c )( c + a )
et

So sánh hai trường hợp ta có giá trị nhỏ nhất của P bằng 2 đạt tại
=
a b= , c 0 hoặc các hoán vị.
vi

7. Kỹ thuật sử dụng bất đẳng thức AM – GM dạng luỹ thừa


ng

m
a1m + a2m + ... + anm  a1 + a2 + ... + an 
≥  .
n  n 
a

Trong đó ak , k = 1, n là các số không âm và m,n là các số nguyên dương.


kh

Đẳng thức xảy ra khi và chỉ khi a1= a2= ...= an .


Chú ý. Xem chứng minh trong chủ đề Bất đẳng thức Bernoulli.
Các bất đẳng thức dạng này hay được sử dụng
1 1
a 3 + b3 ≥ ( a + b ) ; a 4 + b 4 ≥ ( a + b ) .
3 4
4 8
Ví dụ 1. Cho a,b,c là các số thực dương. Chứng minh
a3 b3 c3 3
+ + ≥ .
( b + 3c ) 3
( c + 3a ) 3
( a + 3b ) 3 64

122
Cty TNHH MTV DVVH Khang Việt

Lời giải
Sử dụng bất đẳng thức AM – GM dạng luỹ thừa ta có
3
 a b c 
a3
b 3
c  b + 3c + c + 3a + a + 3b 
3
+ + ≥ 3  .
( b + 3c )3 ( c + 3a )3 ( a + 3b )3  3 
 
Chú ý sử dụng bất đẳng thức C–S ta có
a
+
b
+
c

(a + b + c) 2

b + 3c c + 3a a + 3b a ( b + 3c ) + b ( c + 3a ) + c ( a + 3b )
.

n
=
(a + b + c) 2

3 ( ab + bc + ca )
=
3

.v
4 ( ab + bc + ca ) 4 ( ab + bc + ca ) 4

om
a3 b3 c3 3
Suy ra + + ≥ .
( b + 3c )3
( c + 3a )3
( a + 3b )
3 64
Đẳng thức xảy ra khi và chỉ khi a= b= c . .c
Bài tập tương tự
Cho a,b,c là các số thực dương. Chứng minh
ok
3 3 3
 a   b   c  1
  +  +  ≥ .
bo

 a + 2b   b + 2c   c + 2a  9
Ví dụ 2. Cho a,b,c là các số thực dương. Chứng minh
et

1 1
a 4 + b4 + c4 ≥ ( a + b + c ) .
4
8 64
vi

Lời giải
Sử dụng bất đẳng thức AM – GM dạng luỹ thừa ta có
ng

4
 c c
4 1 4 1 4 1 4 a+b+ 2 + 2  1
a + b + c = a + b + c + c ≥ 4
4 4 4
 = ( a + b + c )4 .
a

8 16 16  4  64
kh

 
c
Bất đẳng thức được chứng minh. Đẳng thức xảy ra khi và chỉ khi a= b= .
2
Ngoài ra bất đẳng thức có dạng đẳng cấp ta có thể chuẩn hoá đưa về xét hàm
số(xem chương 3).
Ví dụ 3. Cho a,b,c là độ dài ba cạnh của một tam giác. Chứng minh
3
a +b−c + 3 b+c−a + 3 c+ a −b ≤ 3 a + 3 b + 3 c .
Lời giải
Sử dụng bất đẳng thức AM – GM dạng luỹ thừa ta có

123
Khám phá tư duy Kỹ thuật giải bất ĐT Bài toán Max – Min – Đặng Thành Nam

3
a + b − c + 3 b + c − a ≤ 3 4(a + b − c + b + c − a) =23 b
3
b + c − a + 3 c + a − b ≤ 3 4 (b + c − a + c + a − b) =23 c
3
c + a − b + 3 a + b − c ≤ 3 4(c + a − b + a + b − c) =23 a
Cộng theo vế ba bất đẳng thức trên ta có điều phải chứng minh.
Đẳng thức xảy ra khi và chỉ khi a= b= c.
Bài tập tương tự
Cho a,b,c là độ dài ba cạnh một tam giác. Chứng minh
a +b−c + b+c−a + c+ a −b ≤ a + b + c .

n
Ví dụ 4. Cho x,y,z là các số thực dương. Chứng minh

.v
x+ y 4 x+ y 4 x+ y  x y z 
4 + + ≥ 2  4 +4 +4 .
z z z  y+z z+x x + y 

om
Lời giải
Sử dụng bất đẳng thức: 8( A4 + B 4 ) ≥ ( A + B ) 4 ta có:
.c
4 4
x x 8x
4 ≤ = .
ok
y+z (4 y + z )
4 4 4 y+4z
4
8
bo

4 
x 4
x 1 1 
Mặt khác: ≤  + .
4 y +4 z 4  4 y 4 z 
et

Xây dựng các BĐT còn lại rồi cộng vế theo vế, ta được:
2  4x+4z
vi

4
VT ≤ . ∑  .
2  4 y

ng

4
x+4z 8( z + x)
Lại có: ≤4 .
4 y y
a

z+x 4 x+ y 4 y+z
kh

Suy ra: VT ≤ 4 + + =
VP .
y z x
Bất đẳng thức được chứng minh. Đẳng thức xảy ra khi và chỉ khi x= y= z .

8. Kỹ thuật biến đổi tương đương kết hợp AM – GM


Đôi khi đánh giá bất đẳng thức trực tiếp bằng AM – GM không hiệu quả khi đó
cần kết hợp biến đổi giữa điều kiện bài toán và bất đẳng thức cần chứng minh. Đưa
về các bất đẳng thức đơn giản hơn để áp dụng AM – GM.

124
Cty TNHH MTV DVVH Khang Việt

Ví dụ 1. Cho a,b,c là các số thực dương thoả mãn điều kiện


1 1 1
+ + =2.
a +1 b +1 c +1
1 1 1
Chứng minh rằng + + ≥1.
4a + 1 4b + 1 4c + 1
Lời giải
Bất đẳng thức đã cho tương đương với
∑ ( 4a + 1)( 4b + 1) ≥ ( 4a + 1)( 4b + 1)( 4c + 1) ⇔ 2 ( a + b + c ) + 1 ≥ 32abc .
Chú ý điều kiện bài toán trở thành
∑ ( a + 1)( b + 1)= 2 ( a + 1)( b + 1)( c + 1) ⇔ ab + bc + ca + 2abc=

n
1.

.v
Sử dụng bất đẳng thức AM – GM ta có
a + b + c ≥ 3 ( ab + bc + ca=
) 3 (1 − 2abc ) .

om
Vậy ta chỉ cần chứng minh 2 3 (1 − 2abc ) + 1 ≥ 32abc

 1 .c
 abc ≤ 32

ok
1
⇔ 2 3 (1 − 2abc ) ≥ 32abc − 1 ⇔  abc ≥ 1 ⇔ abc ≤ .
 8

32
 12 (1 − 2abc ) ≥ ( 32abc − 1)
bo

Bất đẳng thức cuối đúng bởi sử dụng AM – GM cho 3 số dương ta có


et

1= 2abc + ab + bc + ca ≥ 2abc + 3 a 2b 2 c 2 ; t=
3 3
abc
vi

1 1
⇒ 2t 3 + 3t 2 − 1 ≤ 0 ⇔ ( 2t − 1)( t + 1) ≤ 0 ⇔ t ≤ ⇔ abc ≤
2
2 8
ng

1
Bất đẳng thức được chứng minh. Đẳng thức xảy ra khi và chỉ khi a= b= c= .
2
a

Ví dụ 2. Cho a,b,c là các số thực dương thoả mãn điều kiện


kh

1 1 1
+ + = 2.
a +1 b +1 c +1
1 1 1
Chứng minh rằng + + ≥ 4 ( a + b + c ) .
a b c
Lời giải
Cách 1: Thực hiện tương tự bài toán trên.
x y z
Cách 2: Chú ý tồn tại các số dương x,y,z sao= cho a = ,b = ,c .
y+z z+x x+ y
Bài toán đưa về chứng minh

125
Khám phá tư duy Kỹ thuật giải bất ĐT Bài toán Max – Min – Đặng Thành Nam

y+z z+x x+ y  x y z 
+ + ≥ 4 + + .
x y z  y+z z+x x+ y
Bất đẳng thức trên là tổng của ba bất đẳng thức sau
 1 1  4y 1 1 4z 1 1 4x
y +  ≥ ;z +  ≥ ; x +  ≥ .
 x z z+x  y x x+ y  z y y+z
1
Bất đẳng thức được chứng minh. Đẳng thức xảy ra khi và chỉ khi a= b= c= .
2
Ví dụ 3. Cho a,b,c là các số thực dương có tích bằng 1. Chứng minh
1 1 1 2
+ + ≥ +1.

n
a +1 b + c c +1 a + b + c +1

.v
Lời giải
Bất đẳng thức đã cho tương đương với:

om
2 ( a + b + c ) + ab + bc + ca + 3 3+ a +b + c

a + b + c + ab + bc + ca + 2 a + b + c +1
⇔ ( a + b + c + 1)  2 ( a + b + c ) + ab + bc + ca + 3
.c
≥ ( a + b + c + 3)( a + b + c + ab + bc + ca + 2 )
ok
⇔ a 2 + b2 + c2 ≥ 3
Bất đẳng thức cuối luôn đúng bởi theo AM – GM ta có
bo

a 2 + b 2 + c 2 ≥ 3 a 2b 2 c 2 =
3
3.
Bất đẳng thức được chứng minh. Đẳng thức xảy ra khi và chỉ khi a= b= c= 1 .
et
vi

9. Kỹ thuật sắp thứ tự


Áp dụng với các bất đẳng thức hoán vị ta có thể giả sử a = min {a, b, c} hoặc
ng

a = max {a, b, c} hoặc a là số nằm giữa b và c.


Dưới đây là một bất đẳng thức được sử dụng khá nhiều tôi xin trình bày
a

4
a 2b + b 2 c + c 2 a + abc ≤ ( a + b + c ) .
kh

3
27
Chứng minh.
Không mất tính tổng quát giả sử b là số nằm giữa a và c ta có:
( b − a )( b − c ) ≤ 0 ⇔ b2 + ac ≤ ab + bc ⇒ b2c + c 2 a ≤ abc + bc 2
( )
⇒ a 2b + b 2 c + c 2 a + abc ≤ a 2b + bc 2 + 2abc = b a 2 + c 2 + 2ac = b ( a + c )
2

126
Cty TNHH MTV DVVH Khang Việt

3
 a+c a+c 
a+c a+c b+ 2 + 2  4
Khi đó VT ≤ b ( a + c=
) 4.b. ( a + b + c )3 .
2
. ≤ 4 =

2 2  3  27
 
Bất đẳng thức được chứng minh. Đẳng thức xảy ra khi và chỉ khí a= b= c .

Ví dụ 1. Cho a,b,c là các số thực dương có tổng bằng 3. Chứng minh


( ab 2
)
+ bc 2 + ca 2 ( ab + bc + ca ) ≤ 9 .
Lời giải

n
Không mất tính tổng quát giả sử b là số nằm giữa a và c

.v
Ta có: a 2b + b 2 c + c 2 a = b(c + a ) 2 − abc − c(a − b)(b − c)
≤ b(c + a ) 2 − abc= b(c 2 + ac + a 2 )

om
Áp dụng bất đẳng thức AM – GM ta được:
( ab 2
)
+ bc 2 + ca 2 ( ab + bc + ca ) ≤ b(c 2 + ac + a 2 )(ab + bc + ca )
.c 2
 (c 2 + ac + a 2 ) + (ab + bc + ca ) 
ok
≤ b 
 2 
b(c + a )2 (a + b + c)2 9b(a + c)2
bo

= = .
4 4
3
 a+c a+c
et

a+c a+c b + 2 + 2 
=
9b. . ≤ 9  =
9
2 2 3
vi

 
 
ng

Ta có điều phải chứng minh.


Ví dụ 2. Cho a,b,c là các số thực không âm thỏa mãn điều kiện a + b + c =3.
a

Tìm giá trị lớn nhất của biểu thức =


P a b3 + 1 + b c 3 + 1 + c a 3 + 1 .
kh

Lời giải
Sử dụng bất đẳng thức AM-GM ta có:

( b + 1) ( b2 − b + 1) ≤ a.
b + 1 + b2 − b + 1 ab 2
a b3 + 1 = a = a+ .
2 2
bc 2 ca 2
Tương tự: b c3 + 1 ≤ b + ; c a3 + 1 ≤ c + .
2 2
Cộng theo vế 3 bất đẳng thức trên ta được:
ab 2 bc 2 ca 2 ab 2 + bc 2 + ca 2
P≤a+b+c+ + + = 3+ .
2 2 2 2
127
Khám phá tư duy Kỹ thuật giải bất ĐT Bài toán Max – Min – Đặng Thành Nam

Không mất tính tổng quát giả sử b là số nằm giữa a và c ta có:


( b − a )( b − c ) ≤ 0 ⇔ b2 + ac ≤ ab + bc ⇒ ab2 + a 2c ≤ a 2b + abc
⇒ ab 2 + bc 2 + ca 2 ≤ a 2b + abc + bc 2 = b ( a 2 + ac + c 2 ) ≤ b ( a + c )
2

Sử dụng bất đẳng thức AM-GM ta có:


3
 a+c a+c 
a+c a+c b+ 2 + 2 
b ( a + c ) 4.b.
2
= . ≤ 4 =  4.
2 2  3 
 
Đẳng thức xảy ra khi và chỉ khi =
a 0,=
b 1,=
c 2 hoặc các hoán vị.

n
b(a + c)
2
ab 2 + bc 2 + ca 2

.v
Suy ra P ≤ 3 + ≤ 3+ ≤ 5.
2 2

om
Vậy giá trị lớn nhất của P bằng 5 đạt tại =a 0,= b 1,= c 2 hoặc các hoán vị.
Ví dụ 3. Cho a,b,c là các số thực không âm thỏa mãn điều kiện a + b + c =5.
Tìm giá trị lớn nhất của biểu thức P = a 4b + b 4 c + c 4 a .
.c
Lời giải
ok
Không mất tính tổng quát giả sử a = max {a, b, c} , khi đó

a 4 c a 3c 2  ac c 2 
bo

P = a 4b + b 4 c + c 4 a ≤ a 4b + a3bc + + = a3  ab + bc + + 
2 2  2 2 

 c a a a a+c  c
= a3 ( a + c )  b + =  . b + 
et

256. . . . 
 2 4 4 4  4  2
vi

5 5
 a a+c c  3c 
 3. 4 + 4 + b + 2  a+b+ 4  a+b+c
5
≤ 256   = 256   ≤ 256   = 256
ng

 5   5   5 
   
a

Đẳng thức xảy ra khi và chỉ khi =a 4,=b 1,= c 0 hoặc các hoán vị.
kh

Vậy giá trị lớn nhất của P bằng 256 đạt tại =
a 4,=
b 1,=c 0 hoặc các hoán vị.
Nhận xét. Tổng quát với a, b, c không âm thỏa mãn a + b + c =k ta luôn có
n n k n +1
a nb + bn c + cn a ≤ .
( n + 1)n+1
Chứng minh.
Không mất tính tổng quát giả sử a = max {a, b, c} khi đó

128
Cty TNHH MTV DVVH Khang Việt

a n c a n −1c 2
P = a n b + b n c + c n a ≤ a n b + a n −1bc + +
2 2
 ac c 2   c
= a n −1  ab + bc +  a (a + c) b + 
n −1
+ =
 2 2   2

Sử dụng bất đẳng thức AM-GM ta có
a a a a+c 
( a + c )  b +  nn . . ... .
c c
a n −=
1
. b + 
 2 n
 n  n n  2
n −1
n +1 n +1
 a a+c c  1 1 
( n − 1) . +  a + b +  2 + n c 

n
 +b+ 
≤ nn  n n 2 nn 
=   

.v
 n + 1   n +1 
   
 

om
n +1
a+b+c n n k n +1
≤ nn   =
 n +1  ( n + 1)n+1
.c
kn k
Đẳng thức xảy ra khi và chỉ=
khi a = ,b = , c 0 hoặc các hoán vị.
n +1 n +1
ok
Ví dụ 4. Cho a,b,c là các số thực không âm thỏa mãn điều kiện a + b + c =3.
Tìm giá trị lớn nhất của biểu thức
bo

P= (a 2
)( )(
− ab + b 2 b 2 − bc + c 2 c 2 − ca + a 2 . )
et

Lời giải
Không mất tính tổng quát giả sử a ≥ b ≥ c khi đó
vi

b 2 − bc + c 2 = b 2 + c ( c − b ) ≤ b 2
ng

c 2 − ca + a 2 = c ( c − a ) + a 2 ≤ a 2
Suy ra:
a

(
P ≤ a 2b 2 a 2=
− ab + b 2 ) 4 3ab 3ab 2
. . (
a − ab + b 2 )
kh

9 2 2
3
 3ab 3ab 
+ + a 2 − ab + b 2 
4 2 4 4
≤  2 =  ( a + b )6 ≤ ( a + b +=
c ) 12
6
9 3  9.27 9.27
 
Đẳng thức xảy ra khi và chỉ khi = a 2,=
b 1,= c 0.
Vậy giá trị lớn nhất của P bằng 12 đạt tại =
a 2,= b 1,= c 0 hoặc các hoán vị.

129
Khám phá tư duy Kỹ thuật giải bất ĐT Bài toán Max – Min – Đặng Thành Nam

10. Kỹ thuật cô si ngược dấu


Việc áp dụng bất đẳng thức AM – GM cho mẫu số đôi khi làm ngược chiều bất
đẳng thức do vậy trước khi áp dụng ta biến đổi phân thức để có thể áp dụng trực
tiếp bất đẳng thức AM – GM.
Ví dụ 1. Cho a,b,c là các số thực dương có tổng bằng 3. Chứng minh
a b c 3
+ + ≥ .
1+ b 1+ c
2 2
1+ a 2 2
Lời giải
Ta có
( )
a 1 + b 2 − ab 2 ab 2 ab 2

n
a ab
= =a− ≥a− =a−
1 + b2 1 + b2 1 + b2 2b 2

.v
b bc 2bc c ca 2 ca
=b− ; ≥b− = c − ≥c−

om
1+ c 2
1+ c 2 2 1+ a 2
1+ a 2 2
Cộng theo vế 3 bất đẳng thức trên ta được
a b c ab + bc + ca ab + bc + ca 3
+ + ≥a+b+c− = 3− ≥ .
1+ b 1+ c
2 2
1+ a 2 2
.c2 2
ok
1
Vì ab + bc + ca ≤ ( a + b + c )2 =3.
3
Bất đẳng thức được chứng minh. Đẳng thức xảy ra khi và chỉ khi a= b= c= 1 .
bo

Ví dụ 2. Cho x, y, z là các số thực dương. Chứng minh


yz zx xy
et

+ + .
x + 2 yz y + 2 zx z + 2 xy
vi

Lời giải
Sử dụng bất đẳng thức AM – GM ta có
ng

yz 1 x  1 x 
= 1 −  ≤ 1 − 

x + 2 yz 2  x + 2 yz  2  x + y + z 
a

1  1 
kh

zx y y
= 1 −  ≤ 1 − 
y + 2 zx 2  y + 2 zx  2  x + y + z 
xy 1 z  1 z 
= 1 −  ≤ 1 − 

z + 2 xy 2  z + 2 xy  2  x + y + z 
Cộng theo vế 3 bất đẳng thức trên ta có điều phải chứng minh.
Đẳng thức xảy ra khi và chỉ khi x= y= z.
Ví dụ 3. Cho a,b,c là các số thực dương thoả mãn điều kiện a + b + c =3.
Chứng minh rằng

130
Cty TNHH MTV DVVH Khang Việt

a2 b2 c2
+ + ≥ 1.
a + 2b 2
b + 2c 2 c + 2 a 2
Lời giải
Sử dụng bất đẳng thức AM – GM ta có

a2
=
( )
a a + 2b 2 − 2ab 2
=a−
2ab 2
≥a−
2ab 2
=a−
2 2
( ab ) 3
a + 2b 2 a + 2b 2 a + 2b 2 3
3 ab 4 3
b2 2 2
c2 2 2
(≥b−
bc ) 3; ≥ c − ( ca ) 3
b + 2c 2 3 c + 2a 2 3
Cộng theo vế 3 bất đẳng thức trên ta chỉ cần chứng minh

n
2 2 2

.v
(ab) 3 + (bc) 3 + (ca ) 3 ≤ 3 .
Bất đẳng thức trên đúng theo AM – GM vì

om
2
a + ab + b ≥ 3(ab) 3
2 .c
b + bc + c ≥ 3(bc) 3
.
ok
2
c + ca + a ≥ 3(ca ) 3
1
( a + ab + b ) + ( b + bc + c ) + ( c + ca + a ) ≤ 2 ( a + b + c ) + ( a + b + c )2 =
bo

9
3
Bất đẳng thức được chứng minh. Đẳng thức xảy ra khi và chỉ khi a= b= c= 1 .
et

Ví dụ 4. Cho a,b,c là các số thực không âm có tổng bằng 3. Tìm giá trị nhỏ nhất
của biểu thức
vi

a b c
P= 3 + 3 + 3 .
b + 16 c + 16 a + 16
ng

Lời giải
Ta có:
a

ab3 bc3 ca3  ab3 bc3 ca3 


16 P = a − 3 +b− 3 +c− 3 = 3− 3 + 3 + 3
kh

 b + 16 c + 16 a + 16 
.
b + 16 c + 16 a + 16  
ab3 bc3 ca3
Để tìm giá trị lớn nhất của P ta tìm giá trị nhỏ nhất của + + .
b3 + 16 c3 + 16 a3 + 16
Sử dụng bất đẳng thức AM-GM ta được:
ab3 ab3 ab3 ab 2
= ≤ = .
b3 + 16 b3 + 8 + 8 12b 12
bc3 bc 2 ca3 ca 2
Tương tự: ≤ ; 3 ≤ .
c3 + 16 12 a + 16 12

131
Khám phá tư duy Kỹ thuật giải bất ĐT Bài toán Max – Min – Đặng Thành Nam

Cộng lại theo vế 3 bất đẳng thức trên ta được:


ab3 bc3 ca3 ab 2 + bc 2 + ca 2
+ + ≤ .
b3 + 16 c3 + 16 a3 + 16 12
Bài toán đưa về tìm giá trị lớn nhất của ab 2 + bc 2 + ca 2 đây là một bài toán
quen thuộc.
Không mất tính tổng quát giả sử b là số nằm giữa a và c ta có:
( b − a )( b − c ) ≤ 0 ⇔ b2 + ac ≤ ab + bc ⇒ ab2 + ca 2 ≤ a 2b + abc
⇒ ab 2 + bc 2 + ca 2 ≤ a 2b + abc + bc 2 = b a 2 + ac + c 2 ( )

n
3
 a+c a+c 
b+ 2 + 2 

.v
a+c a+c
b ( a + c ) 4.b.
2
≤= . ≤ 4 =  4
2 2  3 

om
 
Đẳng thức xảy ra khi và chỉ khi =
a 0,=
b 1,=
c 2 và các hoán vị.

Suy ra 16 P ≥ 1 ⇔ P ≥
1
16
.
.c
ok
1
Vậy giá trị nhỏ nhất của P bằng đạt tại =
a 0,=
b 1,= c 2 hoặc các hoán vị.
16
bo

Ví dụ 5. Cho a,b,c là các số thực dương có tổng bằng 3.


1 1 1
Chứng minh rằng + + ≥1.
et

2+a b 2
2 + b c 2 + c2a
2

Lời giải
vi

4 2
2
=1−
1 a 2b 1
≥ 1 − a 3 b 3 ≥ 1 − a 2 + ab + ab . ( )
ng

Ta có:
2+a b
2
2+a b
2 3 9

Tương tự ta có:
2 1 2
≥1−
b + bc + ab ;( 2 1
)
≥ 1 − c 2 + ca + bc . ( )
a

2+b c 29 2+c a
2 9
kh

Cộng theo vế ba bất đẳng thức trên ta được:


 1 1 1  (a + b + c) 2
2 + +  ≥ 3 − =2
 2 + a 2b 2 + b 2 c 2 + c 2 a  9
1 1 1
⇔ + + ≥1
2 + a b 2 + b c 2 + c2a
2 2

Bất đẳng thức được chứng minh. Đẳng thức xảy ra khi và chỉ khi a= b= c= 1 .

132
Cty TNHH MTV DVVH Khang Việt

8
11. Sử dụng bất đẳng thức ( a + b )( b + c )( c + a ) ≥ ( a + b + c )( ab + bc + ca ) .
9
Chứng minh.
Đây là một bất đẳng thức được sử dụng khá thường xuyên với bất đẳng thức đối
xứng 3 biến dựa trên đẳng thức
( a + b )( b + c )( c + a ) = ( a + b + c )( ab + bc + ca ) − abc .
Ta có ( a + b + c )( ab + bc + ca ) − abc = ( a + b )( b + c )( c + a ) .
 a + b  b + c  c + a  ( a + b )( b + c )( c + a )
Mặt khác: abc = ab . bc . ca ≤    = .
 2  2  2  8

n
Suy ra:
( a + b )( b + c )( c + a ) = ( a + b + c )( ab + bc + ca ) − abc

.v
≥ ( a + b + c )( ab + bc + ca ) −
( a + b )( b + c )( c + a ) .

om
8
8
Do đó: ( a + b )( b + c )( c + a ) ≥ ( a + b + c )( ab + bc + ca ) .
.c
9
Bất đẳng thức được chứng minh. Đẳng thức xảy ra khi và chỉ khi a= b= c .
ok
Ví dụ 1. Cho a,b,c là các số thực dương thoả mãn điều kiện ( a + b )( b + c )( c + a ) =
1.
3
Chứng minh rằng ab + bc + ca ≤ .
bo

4
Lời giải
Sử dụng bất đẳng thức quen thuộc:
et

8
( a + b )( b + c )( c + a ) ≥ ( a + b + c )( ab + bc + ca ) .
vi

9
Theo giả thiết bài toán kết hợp với bất đẳng thức AM – GM ta có:
ng

8
( a + b )( b + c )( c + a ) ≥ ( a + b + c )( ab + bc + ca )
9
a

8
≥ 3 ( ab + bc + ca ) . ( ab + bc + ca ) .
kh

9
3
⇒ ab + bc + ca ≤
4
1
Bất đẳng thức được chứng minh. Đẳng thức xảy ra khi và chỉ khi a= b= c= .
2
1 1 1
Ví dụ 2. Cho a,b,c là các số thực dương thoả mãn điều kiện a + b + c ≥ + + .
a b c
1 1 1 3
Chứng minh rằng + + ≤ .
( 2a + b + c )2 ( 2b + c + a )2 ( 2c + a + b )2 16
133
Khám phá tư duy Kỹ thuật giải bất ĐT Bài toán Max – Min – Đặng Thành Nam

Lời giải
Sử dụng bất đẳng thức AM – GM ta có
( 2a + b + c )2 =( a + b ) + ( a + c ) ≥ 4 ( a + b )( a + c )
2

1 1 1
⇒ ≤ .
( 2a + b + c ) 2 4 ( a + b )( a + c )
Tương tự cho 2 phân thức còn lại đưa về chứng minh
1 1 1 3
+ + ≤
( a + b )( a + c ) ( b + c )( b + a ) ( c + a )( c + b ) 4
2(a + b + c)

n
3
⇔ ≤
( a + b )( b + c )( c + a ) 4

.v
8
⇔ ( a + b )( b + c )( c + a ) ≥
(a + b + c)

om
3
8
Chú ý ta có ( a + b )( b + c )( c + a ) ≥ ( a + b + c )( ab + bc + ca )
9 .c
Vậy ta chỉ cần chứng minh ab + bc + ca ≥ 3 .
Theo giả thiết ta có abc ( a + b + c ) ≥ ab + bc + ca .
ok
Sử dụng bất đẳng thức AM – GM ta có
( ab + bc + ca )2 ≥ 3abc ( a + b + c ) ≥ 3 ( ab + bc + ca ) .
bo

⇔ ab + bc + ca ≥ 3
Bất đẳng thức được chứng minh. Đẳng thức xảy ra khi và chỉ khi a= b= c= 1 .
et

Ví dụ 3. Cho a,b,c là các số thực dương thoả mãn điều kiện ab + bc + ca ≤ 1 .


vi

 1 1 1 
Chứng minh rằng a + b + c + 3 ≥ 8abc  2 + 2 + 2 .
ng

 a +1 b +1 c +1
Lời giải
Chú ý điều kiện bài toán ab + bc + ca ≤ 1 ta có đánh giá
a

1 1 1 2(a + b + c)
∑ a 2 + 1 ≤ ∑ a 2 + ab + bc + ca =
∑ ( a + b )( a + c ) =
kh

.
( a + b )( b + c )( c + a )
8
Sử dụng bất đẳng thức ( a + b )( b + c )( c + a ) ≥ ( a + b + c )( ab + bc + ca ) .
9
1 2(a + b + c) 9
Ta có ∑ a2 + 1 ≤ 8 =
( + + )
.
( a + b + c )( ab + bc + ca ) 4 ab bc ca
9
Vậy ta chứng minh

134
Cty TNHH MTV DVVH Khang Việt

9
a + b + c + 3 ≥ 8abc.
4 ( ab + bc + ca )
⇔ ( a + b + c )( ab + bc + ca ) + 3 ( ab + bc + ca ) ≥ 18abc
3
 ab + bc + ca  1
Chú ý abc ≤   = .
 3  3 3
Do đó bất đẳng thức cuối là tổng của 2 bất đẳng thức.
( a + b + c )( ab + bc + ca ) ≥ 9abc
3 3abc 3 3abc
3 ( ab + bc + ca ) ≥ 3 3. a 2b= ≥ = 9abc .
2 23

n
c 3
abc 1
3

.v
3 3

om
1
Bất đẳng thức được chứng minh. Đẳng thức xảy ra khi và chỉ khi a= b= c= .
3
12. Kỹ thuật chuẩn hoá
Bất đẳng thức có dạng thuần nhất bậc k có dạng
.c
P ( x1 , x2 ,..., xn ) ≥ 0 trong đó P (tx1 , tx2 ,..., txn ) = t k P ( x1 , x2 ,..., xn ) .
ok
Khi đó ta có thể thực hiện
+ Chuẩn hoá cho x m + y m + z m = const (chuyển bài toán không có điều kiện về có
bo

điều kiện).
+ Hoặc nhân(chia) với đại lượng chứa biến để đưa bất đẳng thức về dạng thuần
et

nhất giúp chứng minh đơn giản hơn.


Ví dụ 1. Cho a,b,c là các số thực dương. Chứng minh
vi

( a + b )2 ( b + c )2 ( c + a )2 4
ng

3 ≥ (a + b + c) .
abc 3
Lời giải
a

Bất đẳng thức có dạng thuần nhất bậc 1 ta chuẩn hoá a + b + c =3.
kh

Ta cần chứng minh 3 ( a + b )2 ( b + c )2 ( c + a )2 ≥4 .


abc
⇔ ( a + b )( b + c )( c + a ) ≥ 8 abc
Chú ý
( a + b )( b + c )( c + a ) = ( 3 − a )( 3 − b )( 3 − c ) = 3 ( ab + bc + ca ) − abc
ab + bc + ca ≥ 3abc ( a + b + c ) = 3 abc

Vậy ta chứng minh 9 abc − abc ≥ 8 abc ⇔ abc ≤ 1 .

135
Khám phá tư duy Kỹ thuật giải bất ĐT Bài toán Max – Min – Đặng Thành Nam

3
a+b+c
Bất đẳng thức cuối đúng bởi vì abc ≤   =1.
 3 
Bất đẳng thức được chứng minh. Đẳng thức xảy ra khi và chỉ khi a= b= c .
Bài tập tương tự
Cho a,b,c là các số thực không âm. Chứng minh

3
( a + b )( b + c )( c + a ) ≥
ab + bc + ca
.
8 3
Ví dụ 2. Cho a,b,c là các số thực dương có tổng bằng 1. Chứng minh
(
a 2 + b 2 + c 2 ≥ 3 a 2b + b 2 c + c 2 a . )

n
.v
Lời giải
Ta chuyển bất đẳng thức về dạng thuần nhất bậc ba bằng cách thay
(a )

om
a 2 + b2 + c2 = 2
+ b2 + c2 ( a + b + c ) .
Ta cần chứng minh
(a ) (
+ b 2 + c 2 ( a + b + c ) ≥ 3 a 2b + b 2 c + c 2 a )
2 .c
( ) ( ) ( ) ( )
ok
⇔ a3 + ab 2 + b3 + bc 2 + c3 + ca 2 ≥ 2 a 2b + b 2 c + c 2 a
Bất đẳng thức này là tổng của 3 bất đẳng thức
bo

a3 + ab 2 ≥ 2a 2b; b3 + bc 2 ≥ 2b 2 c; c3 + ca 2 ≥ 2c 2 a .
1
Bất đẳng thức được chứng minh. Đẳng thức xảy ra khi và chỉ khi a= b= c=
et

.
3
Ví dụ 3. Cho a,b,c là các số thực dương có tổng bằng 1. Chứng minh
vi

a 2 + b 2 + c 2 + 2 3abc ≤ 1 .
ng

Lời giải
Chú ý ta đưa bất đẳng thức về dạng thuần nhất bậc 2 bằng cách thay
a

3abc 2 3abc ( a + b + c )
2=
kh

1= ( a + b + c )2
Bất đẳng thức tương đương với
a 2 + b 2 + c 2 + 2 3abc ( a + b + c ) ≤ ( a + b + c )
2

⇔ ab + bc + ca ≥ 3abc ( a + b + c )

⇔ ( ab + bc + ca ) ≥ 3abc ( a + b + c )
2

136
Cty TNHH MTV DVVH Khang Việt

Bất đẳng thức cuối luôn đúng. Bất đẳng thức được chứng minh. Đẳng thức xảy
1
ra khi và chỉ khi a= b= c= .
3
Ví dụ 4. Cho a,b,c là các số thực dương. Chứng minh

a b c
+ + ≥
(
3 a 2 + b2 + c2
.
)
b c a 3
abc
Lời giải
Chuẩn hoá abc = 1 ta cần chứng minh
a b c
(
+ + ≥ 3 a 2 + b2 + c2 )

n
b c a

.v
2
a b c
(
⇔  + +  ≥ 3 a 2 + b2 + c2
b c a
)

om

a2
b 2
+ 2
a b2
+
c c 2
+ 2
b c2 c
(
+ 2 + 2 ≥ 3 a 2 + b2 + c2
a a b
)
Bất đẳng thức cuối là tổng của 3 bất đẳng thức
.c
ok
a2 a a a4
+ + ≥ 33 2 2 =
3a 2
b2 c c b c
bo

b2 b b b4
+ + ≥ 33 2 2 =
3b 2
c2 a a a c
et

c2 c c c4
2
+ + ≥ 3 3
2 2
=3c 2
a b b a b
vi

Bất đẳng thức được chứng minh. Đẳng thức xảy ra khi và chỉ khi a= b= c .
ng

Ví dụ 5. Cho a,b,c là các số thực dương chứng minh


a+b b+c c+a 6(a + b + c)
+ + ≥ .
a

c a b 3
abc
kh

Lời giải
Bất đẳng thức có dạng thuần nhất bậc 0 chuẩn hoá abc = 1 .
a+b b+c c+a
Ta cần chứng minh + + ≥ 6(a + b + c) .
c a b
Sử dụng bất đẳng thức Mincopski ta có
a+b b+c c+a
+ + =
c a b

137
Khám phá tư duy Kỹ thuật giải bất ĐT Bài toán Max – Min – Đặng Thành Nam

2 2 2 2 2 2
 a  c  b  a  c  b
=   +   +   +   +   +  
 b  b  c  c  a  a
2 2
 a b c  c a b
≥  + +  +  + + 
 b c a  b c a 
Sử dụng bất đẳng thức trình bày trong ví dụ 4 ta có
a b c 3( a + b + c )
+ + ≥ = 3( a + b + c )
b c a 3
b. c. a

n
c a b 3( c + a + b )
+ + ≥ = 3( a + b + c )

.v
b c a 3
b. c. a
Suy ra điều phải chứng minh. Đẳng thức xảy ra khi và chỉ khi a= b= c .

om
13. Kỹ thuật đặt ẩn phụ
Đôi khi đặt ẩn phụ đưa về biến mới giúp đánh giá AM – GM đơn giản hơn.
Ví dụ 1. Cho a, b, c là các số thực dương.
.c
ok
b(a − c) c(3b + a ) 2c(a − b)
Tìm giá trị nhỏ nhất của biểu thức P = + + .
c(a + b) a (b + c) b( a + c )
bo

Lời giải
Đặt x =ab > 0 ; y =bc > 0 ; z =ca > 0, khi đó
et

x − y 3y + z 2z − 2 y
P= + + .
z+ y x+z x+ y
vi

Đặt m = z + y > 0 ; n = x + z > 0 ; p = x + y > 0 .


ng

Suy ra 2 x =n + p − m ; 2y =m + p − n ; 2z =−p + n+ m.
 n 2m   p 2n 
Khi đó P =  + + + − 4.
a

m n   n p 
kh

Sử dụng bất đẳng thức AM-GM ta có


 n 2m   p 2n  n 2m p 2n
P=  + + + −4≥2 . +2 . − 4= 4 2 − 4 .
m n   n p  m n n p

( )
Với ( a, b, c ) = 1,1 + 2 2, −5 + 4 2 ⇒ P =−4 + 4 2 .

Vì vậy Pmin =−4 + 4 2 .


Ví dụ 2. Cho x,y,z là các số thực lớn hơn 1 và thoả mãn điều kiện
xy + yz + zx + xyz =
20 .

138
Cty TNHH MTV DVVH Khang Việt

3
Chứng minh rằng ≥ ( x − 1)( y − 1)( z − 1) .
x+ y + z −3
Lời giải
Đặt a =x − 1, b =y − 1, c =z − 1; a, b, c > 0 ta cần chứng minh abc ( a + b + c ) ≤ 3 .
Chú ý giả thiết bài toán trở thành:
( a + 1)( b + 1) + ( b + 1)( c + 1) + ( c + 1)( a + 1) + ( a + 1)( b + 1)( c + 1) =
20
⇔ ab + bc + ca + 2 ( a + b + c ) + 3 + abc + a + b + c + ab + bc + ca + abc =20
⇔ 3 ( a + b + c ) + 2 ( ab + bc + ca ) + 2abc =
17

Sử dụng bất đẳng thức AM – GM ta có 2 ( ab + bc + ca ) ≥ 2 3abc ( a + b + c )

n
.v
2 a +b+c  4 abc ( a + b + c )
 + abc  ≥
3 3  3 3

om
7(a + b + c) 7 4 7
= ( a + b + c )3 .( a + b + c ) ≥ 4 27abc.( a + b + c )
3 3 3
Cộng theo vế ba bất đẳng thức trên suy ra .c
4 abc ( a + b + c ) 7 4
2 3abc ( a + b + c ) + 27 abc. ( a + b + c ) ≤ 17
ok
+
3 3 3
Vế trái là hàm đồng biến với abc(a + b + c) từ đó suy ra abc ( a + b + c ) ≤ 3 .
bo

Bất đẳng thức được chứng minh. Đẳng thức xảy ra khi và chỉ khi x= y= z= 2 .
et

14. Kỹ thuật lựa chọn điểm rơi


Đánh giá bất đẳng thức điều kiện tiên quyết là đảm bảo dấu bằng. Do vậy với
vi

bất đẳng thức đối xứng các biến thường bằng nhau đây là cơ sở cho việc ta áp dụng
bất đẳng thức AM – GM cho các số dương hợp lý.
ng

Trong trường hợp khó đoán biến dấu bằng xảy ra tại đây ta có thể sử dụng
tham số hoá hoặc giải phương trình đạo hàm riêng.
a

Chi tiết hơn về về phương pháp này xem chủ đề tham số hoá.
Ví dụ 1. Cho a,b,c là các số thực dương có tổng bằng 1. Tìm giá trị lớn nhất của
kh

biểu thức P = 3 a + b + 3 b + c + 3 c + a .
Lời giải
Phân tích tìm lời giải:
 2
a + b = 3

a+b+c 1  2
Biểu thức đối xứng 3 biến dấu bằng đạt tại a = b = c = = ⇒ b + c = .
3 3  3
 2
c + a =
 3
139
Khám phá tư duy Kỹ thuật giải bất ĐT Bài toán Max – Min – Đặng Thành Nam

2
Do đó lựa chọn các số ghép cặp với từng tổng ( a + b ) , ( b + c ) , ( c + a ) .
3
Lời giải chi tiết:
Sử dụng bất đẳng thức AM – GM cho 3 số dương ta có
2 2
a+b+ +
9 2 2 9
3
a=+ b 3 .3 ( a + b ) . . ≤ 3 . 3 3;
4 3 3 4 3
2 2
b+c+ +
9 2 2 9
3
b=+ c 3 .3 ( b + c ) . . ≤ 3 . 3 3;
4 3 3 4 3

n
2 2
c+a+ +

.v
9 2 2 9
3
+ a 3 .3 ( c + a ) . . ≤ 3 .
c= 3 3.
4 3 3 4 3

om
Cộng theo vế 3 bất đẳng thức trên ta được 3
a + b + 3 b + c + 3 c + a ≤ 3 18 .
1
Vậy giá trị lớn nhất của P bằng 3 18 dấu bằng đạt tại a= b= c=
.c .
3
Bài tập tương tự
ok
Cho a,b,c là các số thực dương có tổng bằng 1. Tìm giá trị lớn nhất của biểu thức
P= 3
2a + 3b + 3 2b + 3c + 3 2c + 3a .
bo

Ví dụ 2. Cho a,b,c là các số thực dương thoả mãn điều kiện a 2 + b 2 + c 2 =


1.
1
Tìm giá trị nhỏ nhất của biểu thức P = a + b + c + .
et

abc
Lời giải
vi

Phân tích tìm lời giải:


ng

a 2 + b2 + c2 1 1
Dấu bằng đạt tại a =
b=c= = ⇒ =
3 3.
3 3 abc
a

1  1  8
Vậy tách ghép cặp như sau: a + b + c + = a +b + c + + .
kh

abc  9abc  9abc


Lời giải chi tiết:
Sử dụng bất đẳng thức AM – GM ta có
 1  8 1 8
P = a +b + c + + ≥ 4 4 a.b.c. + = 4 3.
 9abc  9abc 9abc  a 2 + b2 + c2 
3

9 
 3 
 
1
Vậy giá trị nhỏ nhất của P bằng 4 3 đạt tại a= b= c= .
3

140
Cty TNHH MTV DVVH Khang Việt

3
Ví dụ 3. Cho a,b,c là các số thực dương thoả mãn điều kiện a + b + c ≤ .
2
1 1 1
Tìm giá trị nhỏ nhất của biểu thức P = a 2 + b 2 + c 2 + + + .
a b c
Lời giải
Phân tích tìm lời giải:
 2 1
a= b=
2
c=
2
1  4
Dấu bằng đạt tại a= b= c= ⇒ .
2 1 1 1
= = = 2
 a b c

n
1 1 1

.v
Vậy cần tách ghép như sau: a 2 + b 2 + c 2 + + +
a b c

om
 1 1   1 1   1 1  3 1 1 1
=  a2 + +  +  b2 + +  +  c2 + +  +  + +  .
 8a 8a   8b 8b   8c 8c  4  a b c 
Lời giải chi tiết: .c
Sử dụng bất đẳng thức AM – GM cho 3 số dương ta có
 1 1   1 1   1 1  3 1 1 1
ok
P=  a 2 + +  +  b2 + +  +  c2 + +  +  + + 
 8a 8a   8b 8b   8c 8c  4  a b c 
bo

1 1 1 1 1 1 9
≥ 33 a 2 . . + 33 b2 . . + 33 c 2 . . + 3 .
8a 8a 8b 8b 8c 8c 4 abc
9 9 27
≥ + ≥
et

4 4. a + b + c 4
3
vi

27 1
Vậy giá trị nhỏ nhất của P bằng đạt tại a= b= c= .
ng

4 2
Cách 2: Ta có
a

1 9
P ≥ (a + b + c) +
2
a+b+c
kh

3
( 2 ( a + b + c ) − 3) ( 2 ( a + b + c )2 + 3 ( a + b + c ) − 36 ) 27 27
.
+ ≥
12 ( a + b + c ) 4 4

Ví dụ 4. Cho a,b,c là các số thực dương thoả mãn điều kiện a 2 + b 2 + c 2 =


12 .
Chứng minh rằng a b 2 + c 2 + b c 2 + a 2 + c a 2 + b 2 ≤ 24 .
3 3 3

Lời giải
Dấu bằng đạt tại a= b= c= 2 . Khi đó 4=
a 2a=
2
b 2 + c 2 ta sử dụng bất đẳng
thức AM – GM như sau:

141
Khám phá tư duy Kỹ thuật giải bất ĐT Bài toán Max – Min – Đặng Thành Nam

=
a b +c
3 2 2
3
(
4a.2a 2 . b 2 + c 2 ) ≤ 4a + 2a 2
+ b2 + c2
2 6

=
b c +a
3 2 2
3 2
(
4b.2b . c 2 + a 2 ) ≤ 4b + 2b 2
+ c2 + a2
.
2 6

=
c a +b
3 2 2
3 2
(
4c.2c . a 2 + b 2 ) ≤ 4c + 2c 2
+ a 2 + b2
2 6
Cộng theo vế 3 bất đẳng thức trên chú ý a + b + c ≤ 3 a 2 + b 2 + c 2 = (
6 ta có )

n
điều phải chứng minh.

.v
Đẳng thức xảy ra khi và chỉ khi a= b= c= 2 .
a + b + c = 9

om
Ví dụ 5. Cho a,b,c là các số thực dương thoả mãn điều kiện  .
a ≥ 5, a + b ≥ 8
Tìm giá trị lớn nhất của biểu thức P = abc .
Lời giải
.c
Từ điều kiện ta dự đoán dấu bằng đạt tại = a 5,=
b 3,=
c 1. Vì vậy viết lại P và sử
ok
dụng AM – GM cho ba số dương từ trung bình nhân sang trung bình cộng.
Ta có a ≥ 5 ⇒ b + c ≤ 4; a + b ≥ 8 ⇒ c ≤ 1 .
bo

Sử dụng bất đẳng thức AM – GM cho ba số dương ta được:


3
1 1  3a + 5b + 15c 
= . ( 3a.5b.15c ) ≤  
et

P
15 15  3 
1  3 ( a + b + c ) + 2 ( b + c ) + 10c 
3
vi

3
1  3.9 + 2.4 + 10.1 
=   ≤  =  15
15  3  15  3 
ng

Với =
a 5,= b 3,= c 1 thì P bằng 15. Vậy giá trị lớn nhất của P bằng 15.
Ví dụ 6. Cho a,b,c là các số thực dương có tổng bằng 1. Chứng minh
a

1 1
+ ≥ 30 .
kh

a +b +c
2 2 2 abc
Lời giải
Sử dụng bất đẳng thức AM - GM ta có
 1 1 1  7 1 7
=
VT  2 + + + ≥ 33 + .
a +b +c
2 2 9abc 9abc  9abc a 2 + b 2 + c 2 .9abc.9abc 9abc ( )
4
 a 2 + b 2 + c 2 + 3 ( ab + bc + ca ) 
( 2 2 2
)
Ta có a + b + c .3ab.3bc.3ca ≤  
 4 

142
Cty TNHH MTV DVVH Khang Việt

4
 1
+ ab + bc + ca   1 + 3 
4
(a + b + c) 2
1
 =  ≤  .
 4   4  81
 
3
a+b+c 1
Chú ý abc ≤   = .
 3  27
Từ đó suy ra đpcm.
1 3
Cách 2: Ta có: 3(ab + bc + ca ) ≤ (a + b + c)2 ⇒ ≥
ab + bc + ca (a + b + c) 2

n
1 9
Mặt khác: (ab + bc + ca )(a + b + c) ≥ 9abc ⇒ ≥

.v
abc ab + bc + ca
Do đó:

om
1 1 1 7
P ≥ + + +
a +b +c2 2 ab + bc + ca ab + bc + ca ab + bc + ca
2

9 21
≥ + =
30 .c
(a + b + c) 2
(a + b + c)2
Ta có đpcm.
ok

B. BÀI TOÁN CHỌN LỌC


bo

Bài 1. Cho a,b,c là các số thực dương thỏa mãn điều kiện b ≥ a > c > 0 .
c (a − b) 1
et

Chứng minh rằng ab + + ≥ 3.


ab c (a − c)
vi

Lời giải
Nhận xét. Ta chứng minh bất đẳng thức quen thuộc sau đây:
ng

ab ≥ c ( a − c ) + b ( b − c ) .

c (a − c) c (b − c )
a

Thật vậy bất đẳng thức tương đương với: + ≤1.


kh

ab ab
Sử dụng bất đẳng thức AM-GM cho 2 số dương ta có
c (a − c) c (b − c )
1c a−c c b−c
+ ≤  + + + =1.
ab ab 2b a a b 
Ta có điều phải chứng minh.
Giờ ta áp dụng vào bài

143
Khám phá tư duy Kỹ thuật giải bất ĐT Bài toán Max – Min – Đặng Thành Nam

c (a − b) 1 c ( a − c ) − c (b − c ) 1
ab + + =ab + +
ab c (a − c) ab c ( − c)
a
c ( a − c ) − c (b − c ) 1
≥ c ( a − c ) + c (b − c ) + +
c ( a − c ) + c (b − c ) c (a − c)
1
= c ( a − c ) + c (b − c ) + c ( a − c ) − c (b − c ) +
c (a − c)
1 1
= 2 c (c − a) + = c (a − c) + c (a − c) +
c (a − c) c (a − c)

n
1
≥ 33 c ( a − c ). c ( a − c ). =
3
c (a − c)

.v
a= b= 2

om
Đẳng thức xảy ra khi và chỉ khi  .
c = 1
Bài 2. Cho a,b,c là các số thực dương thỏa mãn điều kiện a + b + c = 3.
a+b b+c c+a .c
Chứng minh rằng + + ≥ 3.
c + ab a + bc b + ac
ok
Lời giải
Sử dụng bất đẳng thức AM-GM ta có
a+b b+c c+a a+b b+c c+a
bo

+ + ≥ 36 . . .
c + ab a + bc b + ac c + ab a + bc b + ca
Ta chỉ cần chứng minh ( a + b )( b + c )( c + a ) ≥ ( a + bc )( b + ca )( c + ab ) .
et

Sử dụng bất đẳng thức AM-GM ta có


vi

 a + bc + b + ca 
2
( a + b ) ( c + 1) . 2 2
( a + bc )( b + ca ) ≤   =
 
ng

2 4
Tương tự ta có:
( b + c )2 ( a + 1)2 ; ( a + c )2 ( b + 1)2 .
a

( b + ca )( c + ab ) ≤ ( c + ab )( a + bc ) ≤
4 4
kh

Nhân theo vế 3 bất đẳng thức trên ta được

( a + bc )( b + ca )( c + ab )  ≤ ( a + b )( b + c )( c + a ) 
2 2
.
( a + 1) ( b + 1) ( c + 1)
2 2 2
.
64

Để kết thúc bài toán ta chỉ cần chứng minh


( a + 1) ( b + 1) ( c + 1)
2 2 2
≤ 1 (luôn đúng).
64
3
 a +1+ b +1+ c +1
Vì ( a + 1)( b + 1)( c + 1) ≤   =
8.
 3 
Đẳng thức xảy ra khi và chỉ khi a= b= c= 1 .
144
Cty TNHH MTV DVVH Khang Việt

Bài tập tương tự


Cho a,b,c là các số thực dương có tổng bằng 1. Chứng minh
a+b b+c c+a 3
+ + ≥3 .
c + ab a + bc b + ac 2
Bài 3. Cho a,b,c là các số thực không âm thoả mãn điều kiện ab + bc + ca > 0 .
1 + a2 1 + b2 1 + c2
Chứng minh rằng + + ≥3.
b+c c+a a+b
Lời giải
Sử dụng bất đẳng thức AM – GM cho 3 số dương ta được

n
1 + a2 1 + b2 1 + c2 (
1 + a 2 1 + b2 1 + c2)( )( )

.v
+ + ≥3 6 .
b+c c+a a+b ( a + b )( b + c )( c + a )

om
Vậy ta chứng minh
(1 + a )(1 + b )(1 + c ) ≥ 1
2 2 2

( a + b )( b + c )( c + a )
( )( )( )
⇔ 1 + a 2 1 + b 2 1 + c 2 ≥ ( a + b )( b + c )( c + a ) .
.c
ok
( )( )
Sử dụng bất đẳng thức C –S ta có 1 + a 2 1 + b 2 ≥ ( a + b ) ;
2

(1 + b )(1 + c ) ≥ ( b + c ) ;
2 2 2
bo

(1 + c )(1 + a ) ≥ ( a + c ) .
2 2 2
et

Nhân theo vế 3 bất đẳng thức trên ta có điều phải chứng minh.
Bất đẳng thức được chứng minh. Đẳng thức xảy ra khi và chỉ khi a= b= c .
vi

Bài 4. Cho a,b,c là các số thực không âm thoả mãn điều kiện ab + bc + ca > 0 .
ng

a b c
Chứng minh rằng + + ≥2.
b+c c+a a+b
a

Lời giải
kh

Sử dụng bất đẳng thức AM –GM ta có ( a + b + c ) ≥ 4a ( b + c )


2

a 4a 2 a 2a
⇒ ≥ ⇒ ≥ .
b + c (a + b + c) 2 b+c a+b+c

b 2b
Tương tự ta có ≥
c+a a+b+c
c 2c

a+b a+b+c
Cộng theo vế 3 bất đẳng thức trên ta được

145
Khám phá tư duy Kỹ thuật giải bất ĐT Bài toán Max – Min – Đặng Thành Nam

a b c  a b c 
+ + ≥ 2 + + =2.
b+c c+a a+b  a + b + c a + b + c a + b + c 
Bất đẳng thức được chứng minh. Đẳng thức xảy ra khi và chỉ khi có 1 số bằng 0
và 2 số còn lại bằng nhau.
Nhận xét. Ngoài lời giải trên ta có thể chứng minh bất đẳng này bằng biến đổi
tương đương, bất đẳng thức Holder hoặc Bernuoli.
Bài 5. Cho a,b,c là các số thực dương. Chứng minh
2 2 2
 2a   2b   2c 
  +3
3
 +3  ≥ 3.
b+c c+a a+b

n
Lời giải

.v
Sử dụng bất đẳng thức AM – GM cho 3 số dương ta có
2
 2a 

om
a a 3a
3
  = ≥ =
b+c b+c a+b+c
b+c 3
b+c b+c a+ +
a. .
2 2 2 2
3 .c
2
 2b  b b 3b
 = ≥ =
ok
3

c+a 3
c+a c+a b+ c+a + c+a a+b+c
b. .
2 2 2 2
bo

3
2
 2c  c c 3c
3
  = ≥ =
a+b a+b c+ a+b + a+b a+b+c
et

a+b 3
c. .
2 2 2 2
vi

3
Cộng theo vế 3 bất đẳng thức trên ta có điều phải chứng minh. Đẳng thức xảy ra
ng

khi và chỉ khi a= b= c .


Bài 6. Chứng minh rằng với mọi a,b,c dương ta có
a

 a b  b c  c a 
 +  +  +  ≥1.
kh

 b + c c + a  c + a a + b  a + b b + c 
Lời giải
Sử dụng bất đẳng thức AM –GM cho 2 số dương ta có
a b a b
+ = +1+ +1− 2
b+c c+a b+c c+a
 1 1 
= (a + b + c) + −2
b+c c+a

146
Cty TNHH MTV DVVH Khang Việt

2(a + b + c) ( a + b) + ( a + c ) + (b + c ) − 2
=
≥ −2
( b + c )( c + a ) ( b + c )( c + a )
a+b+2 ( a + c )( b + c ) a+b
≥ −2=
( a + c )( b + c ) ( a + c )( b + c )
b c b+c
Tương tự ta có + ≥ .
c+a a+b ( c + a )( a + b )
c a c+a
+ ≥
a+b b+c ( a + b )( b + c )

n
Nhân theo vế 3 bất đẳng thức trên ta có điều phải chứng minh.

.v
Đẳng thức xảy ra khi và chỉ khi a= b= c .
Bài 7. Cho a,b,c,x,y,z là các số thực dương thoả mãn điều kiện

om
( a + b + c )( x + y + z )= (a 2
+ b2 + c2 )( x 2
)
+ y2 + z2 = 4 .

1 .c
Chứng minh rằng abcxyz < .
36
ok
Lời giải
Sử dụng bất đẳng thức AM – GM ta có
4 ( ab + bc + ca )( xy + yz + zx )
bo


2
(  )
= ( a + b + c ) − a 2 + b 2 + c 2  ( x + y + z ) − x 2 + y 2 + z 2 
2
 ( )
et


2
( ) (
= 20 − ( a + b + c ) x 2 + y 2 + z 2 + a 2 + b 2 + c 2 ( x + y + z ) 
2
 )
vi

≤ 20 − 2 ( a + b + c )( c + y + z ) (a 2
+ b2 + c2 )( x 2
)
+ y2 + z2 =4
ng

Do đó ( ab + bc + ca )( xy + yz + zx ) ≤ 1 .

Mặt khác ( ab + bc + ca ) ≥ 3abc ( a + b + c ) .


a

2
kh

( xy + yz + zx )2 ≥ 3xyz ( x + y + z )
Nhận theo vế 2 bất đẳng thức trên ta được
1
9abcxyz ( a + b + c )( x + y + z ) ≤ 1 ⇔ abcxyz ≤
.
36
Đẳng thức xảy ra khi và chỉ khi a= b= c; x= y= z khi đó trái với giả thiết
( a + b + c )( x + y + z )= (a 2
+ b2 + c2 )( x 2
+ y2 + z2 = 4 . )
1
Vậy abcxyz < .
36

147
Khám phá tư duy Kỹ thuật giải bất ĐT Bài toán Max – Min – Đặng Thành Nam

1 + a2 1 + b2 1 + c2
Bài 8. Cho a, b, c > −1. Chứng minh + + ≥ 2.
1 + b + c2 1 + c + a2 1 + a + b2
Lời giải
1 + b2
Sử dụng bất đẳng thức AM – GM ta có 1 + b + c 2 > 0,1 + b + c 2 ≤ 1 + + c2 .
2

Suy ra
1 + a2

(
2 1 + a2 ) .
1+ b + c 2
1 + b2 + 2 1 + c2 ( )
1 + b2 (
2 1 + b2 )

n
Tương tự ta có ≥
1+ c + a 2
1 + c + 2 1 + a2
2
( )

.v
1 + c2 (
2 1 + c2 )

om

1+ a + b 2
1 + a 2 + 2 1 + b2 ( )
Cộng theo vế 3 bất đẳng thức trên và đặt x =a 2 + 1, y =b 2 + 1, z =c 2 + 1 ta được
.c
1 + a2 1 + b2 1 + c2 2x 2y 2z
+ + ≥ + +
ok
.
1+ b + c 1+ c + a
2
1+ a + b 2 2 y + 2z z + 2x x + 2 y
Sử dụng bất đẳng thức C – S ta có
bo

x
+
y
+
z

(x + y + z) 2

y + 2z z + 2x x + 2 y x ( y + 2z ) + y ( z + 2x) + z ( x + 2 y )
et

=
( x + y + z )2 ≥ 3 ( xy + yz + zx ) = 1
vi

3 ( xy + yz + zx ) 3 ( xy + yz + zx )
Bất đẳng thức được chứng minh. Đẳng thức xảy ra khi và chỉ khi a= b= c= 1 .
ng

Bài 9. Cho a,b,c là các số thực không âm thỏa mãn điều kiện ab + 2bc + 3ca =
6.
Tìm giá trị nhỏ nhất của biểu thức P = ( a + b )( b + c )( c + a ) + 4a + b + c .
a
kh

Lời giải
Sử dụng bất đẳng thức AM-GM ta có
P≥2 ( a + b )( b + c )( c + a ) .( 4a + b + c ) .
Mặt khác sử dụng bất đẳng thức C-S ta có
( a + b )( b + c )( c + a ) .( 4a + b + c=)  a ( b + c )2 + b ( c − a )2 + c ( a + b )2  . ( 4a + b + c )
 
≥  2a ( b + c ) + b ( c − a ) + c ( a + b ) 
2

= ( ab + 2bc + 3ca ) = 36
2

148
Cty TNHH MTV DVVH Khang Việt

Đẳng thức xảy ra khi và chỉ khi =


a 1,=b 0,= c 2.
Vậy giá trị nhỏ nhất của P bằng 12 đạt tại =
a 1,=
b 0,=
c 2.
Bài 10. Cho các số thực a,b,c thuộc đoạn [ 0;1] .

thức P a5b5c5 ( 3 ( ab + bc + ca ) − 8abc ) .


Tìm giá trị lớn nhất và nhỏ nhất của biểu=
Lời giải
Sử dụng bất đẳng thức AM-GM ta có:
3 ( ab + bc + ca ) ≥ 3.3 a= 9 a 2b 2 c 2 ≥ 9abc ≥ 8abc .
3 2 2 2 3
b c
Suy ra P ≥ 0 . Vậy giá trị nhỏ nhất của P bằng 0 xảy ra khi có ít nhất một số

n
bằng 0.
Sử dụng bất đẳng thức AM-GM ta được:

.v
P abc.abc.abc.abc.abc 3 ( ab + bc + ca ) − 8abc 

om
 3 ( ab + bc + ca ) − 8abc + 5abc  ( ab + bc + ca − abc ) .
6 6
≤  =
 6  26
Mặt khác:
.c
ab + bc + ca − abc = a ( b + c − bc ) + bc = a b + c (1 − b )  + bc
ok
.
≤ b + c − bc + bc = b + c ≤ 2
Do đó P ≤ 1 . Vậy giá trị lớn nhất của P bằng 1 xảy ra khi a= b= c= 1 .
bo

Bài 11. Cho a,b,c là các số thực dương có tích bằng 1. Chứng minh
7
( a + b )( b + c )( c + a ) ≥ ( a + b + c ) + 1 .
et

3
vi

Lời giải
8
Sử dụng bổ đề. ( a + b )( b + c )( c + a ) ≥ ( a + b + c )( ab + bc + ca ) , ∀a, b,c ≥ 0
ng

9
ta được:
a

8 8
( a + b )( b + c )( c + a ) ≥ ( a + b + c )( ab + bc + ca ) ≥ ( a + b + c ) .33 a 2b2c 2
kh

9 9
8 7 1
= ( a + b + c= ) (a + b + c) + (a + b + c)
3 3 3
7 7
≥ ( a + b + c ) + 3 abc= (a + b + c) +1
3 3
Bất đẳng thức được chứng minh. Đẳng thức xảy ra khi và chỉ khi a= b= c= 1 .
Nhận xét. Ngoài ra ta có bất đẳng thức mạnh hơn như sau. Với giả thiết cùng điều
kiện bài toán ta có ( a + b )( b + c )( c + a ) ≥ 4 ( a + b + c − 1) .

149
Khám phá tư duy Kỹ thuật giải bất ĐT Bài toán Max – Min – Đặng Thành Nam

Chứng minh.
Ta có:
( a + b )( b + c )( c + a ) = ( a + b + c )( ab + bc + ca ) − abc = ( a + b + c )( ab + bc + ca ) − 1
Mặt khác:
( ab + bc + ca )2 ≥ 3abc ( a + b + c )= 3 ( a + b + c ) ⇒ ab + bc + ca ≥ 3( a + b + c ) .
Vậy bài toán được chứng minh nếu bất đẳng thức sau đúng:
( a + b + c ) 3 ( a + b + c ) − 1 ≥ 4 ( a + b + c − 1) .
Thật vậy, đặt t = ( )
a + b + c , t ≥ 3 . Xét hàm số f (t ) = 3t 3 − 4t 2 + 3 với

n
t≥ 3.
)

.v
'(t ) 3 3t 2 − 8t > 0, ∀t ≥ 3 nên f (t ) đồng biến trên  3; +∞ hay
Ta có f=

om
f (t ) ≥ f ( 3) =
0.
Do đó ( a + b + c ) 3 ( a + b + c ) − 1 ≥ 4 ( a + b + c − 1) .
Bất đẳng thức được chứng minh. Ngoài ra bài toán trên có thể thực hiện bằng
.c
phương pháp dồn biến(xem chương 4).
ok
Bài 15. Cho x,y,z là các số thực dương thỏa mãn điều kiện
( x + y )( y + z )( z + x ) =
8.
bo

2 1 1 1
Tìm giá trị nhỏ nhất của biểu thức P = + + + .
3 xyz x + 2 y y + 2 z z + 2 x
et

Lời giải
Sử dụng bất đẳng thức AM-GM ta có:
vi

1 1 1 9 3
+ + ≥ = .
x + 2 y y + 2z z + 2x x + 2 y + y + 2z + z + 2x x + y + z
ng

Mặt khác:
8 9
( x + y )( y + z )( z + x ) ≥ ( x + y + z )( xy + yz + zx ) ⇒ x + y + z ≤
a

.
9 xy + yz + zx
kh

Suy ra
2 xy + yz + zx 2 1 1
P≥ + ≥ + 3 x2 y 2 z 2 = + + 3 x2 y 2 z 2 ≥ 3 .
3 xyz 3 3 xyz 3 xyz 3 xyz
Vậy giá trị nhỏ nhất của P bằng 3 đạt tại x= y= z= 1 .
Bài 16. Cho a,b,c là các số thực không âm thỏa mãn điều kiện a + b + c =3.
Chứng minh rằng a + b + c ≥ ab + bc + ca .
Lời giải
( )
Ta có: 2 ( ab + bc + ca ) =( a + b + c ) − a 2 + b 2 + c 2 =9 − a 2 + b 2 + c 2 .
2
( )
150
Cty TNHH MTV DVVH Khang Việt

Vậy ta chứng minh bất đẳng thức: a 2 + b 2 + c 2 + 2 ( a + b + c ≥9.)


Theo bất đẳng thức AM-GM ta có:
a 2 + a + a ≥ 3a, b 2 + b + b ≥ 3b, c 2 + c + c ≥ 3c .
Cộng theo vế ba bất đẳng thức trên ta được:
a 2 + b2 + c2 + 2 a + 2 b + 2 c ≥ 3( a + b + c ) =9.
Bất đẳng thức được chứng minh. Đẳng thức xảy ra khi và chỉ khi a= b= c .
Bài 17. Cho x,y,z,t là các số thực không âm thỏa mãn điều kiện x + y + z + t =4.
Tìm giá trị lớn nhất và giá trị nhỏ nhất của biểu thức

n
P= x 3 + yz + y 3 + zt + z 3 + tx + t 3 + xy .

.v
Lời giải
Do x,y,z,t không âm nên

om
P≥ x 3+ y 3+ z 3+t 3 = (x + y + z + t) 3 = 4 3.
Vậy giá trị nhỏ nhất của P bằng 4 3 đạt tại x = 4, y = z = t = 0 hoặc các hoán vị.
.c
Để tìm giá trị lớn nhất của P ta sử dụng bất đẳng thức AM-GM cho hai số dương:
=
1
(
2 x 3 x + xyz + 2 y 3 y + yzt + 2 z 3z + ztx + 2 t 3t + txy )
ok
P
2
1
≤ ( 4 x + 3 x + xyz + 4 y + 3 y + yzt + 4 z + 3z + ztx + 4t + 3t + txy )
bo

4
7 1 1
= ( x + y + z + t ) + ( xyz + yzt + ztx + txy ) =7 + ( xyz + yzt + ztx + txy )
4 4 4
et

Mặt khác:
vi

2 2
 x+ y  z+t 
xyz + yzt + ztx + txy= xy ( z + t ) + zt ( x + y ) ≤   (z + t) +   ( x + y)
 2   2 
ng

2
x+ y+ z+t 
( x + y )( z + t ) ≤ 
1
=( x + y )( z + t )( x + y + z + t ) =  =
4
4  2 
a

Vậy giá trị lớn nhất của P bằng 8 đạt tại x = y = z = t = 1 .


kh

Bài 18. Cho các số thực a,b,c thay đổi thỏa mãn điều kiện a 2 + b 2 + c 2 =
1.
Tìm giá trị lớn nhất, giá trị nhỏ nhất của biểu thức
P = a3 + b3 + c3 − 3abc .
Lời giải
Ta có: P = ( a + b + c ) ( a 2 + b2 + c 2 − ab − bc − ca ) = ( a + b + c )(1 − ab − bc − ca ) .
Suy ra: P 2 = ( a + b + c )2 (1 − ab − bc − ca )2
=(1 + 2ab + 2bc + 2ca )(1 − ab − bc − ca )(1 − ab − bc − ca )

151
Khám phá tư duy Kỹ thuật giải bất ĐT Bài toán Max – Min – Đặng Thành Nam

Do ab + bc + ca ≤ a 2 + b 2 + c 2 =
1 nên các số trong tích của P 2 không âm ta sử
dụng bất đẳng thức AM-GM ta được:
 1 + 2ab + 2bc + 2ca + 2 (1 − ab − bc − ca ) 
3
P2 ≤   = 1.
 3 
Vậy giá trị nhỏ nhất của P bằng −1 đạt tại a = −1, b = c=
0 hoặc các hoán vị.
Giá trị lớn nhất của P bằng 1 đạt tại a= 1, b= c= 0 hoặc các hoán vị.
Bài 19. Cho a,b,c là các số thực không âm thỏa mãn điều kiện a + b + c = 1.
Tìm giá trị lớn nhất của biểu thức P = ( )(
a 3 + b3 b3 + c 3 c 3 + a 3 . )( )

n
Lời giải
Không mất tính tổng quát ta giả sử c = min {a, b, c} .

.v
Khi đó :

om
( )( )( ) ( )
P ≤ a3 + b3 b3 + b 2 c a3 + a 2 c = ( a + b ) a 2 − ab + b 2 ( a + c )( b + c ) a 2b 2

( )
= ( a + b )( b + c )( c + a ) a 2 − ab + b 2 a 2b 2 .c
Mặt khác:
ok
( a + b )( b + c )( c + a ) = ( a + b + c )( ab + bc + ca ) − abc ≤ ( a + b + c )( ab + bc + ca )
Suy ra:
( )
bo

P ≤ ( a + b + c )( ab + bc + ca ) a 2 − ab + b 2 a 2b 2
4
 ab + bc + ca + a 2 − ab + b 2 + ab + ab 
et

≤ ( a + b + c ) . 
 4
 
vi

( a + b + c ) ( a + b )2 + c ( a + b ) ( a + b )4 ( a + b + c )5 ( a + b + c )9
1
= = ≤ =
ng

256 256 256 256


1 1
Vậy giá trị lớn nhất của P bằng đạt tại a= b= , c= 0 hoặc các hoán vị.
a

256 2
kh

1
Bài 20. Cho a,b,c,d,e,f là các số thực không âm có tổng bằng 1 và ace + bdf ≥ .
108
1
Chứng minh rằng abc + bcd + cde + def + efa + fba ≤ .
36
Lời giải
Sử dụng bất đẳng thức AM-GM cho ba số dương ta được:
abc + bcd + cde + def + efa + fba + ace + bdf =( a + d )( b + e )( c + f )
3
a+d +b+e+c+ f  1
≤  =
 3  27

152
Cty TNHH MTV DVVH Khang Việt

1 1 1 1
Suy ra abc + bcd + cde + def + efa + fba ≤ − ( ace + bdf ) ≤ − =.
27 27 108 36
Bất đẳng thức được chứng.
1
Đẳng thức xảy ra khi và chỉ khi a= b= c= d= e= f = .
6
Bài 21. Cho các số thực không âm a,b,c thỏa mãn điều kiện ab + bc + ca > 0 và
a + 2b + 3c = 4.
1 1
Tìm giá trị nhỏ nhất của=
biểu thức P + .
ab + bc + ca ab + bc + c 2

n
Lời giải
Nhận xét. Để P nhỏ nhất từ a + 2b + 3c =

.v
4 ta sẽ suy ra a lớn nhất và c nhỏ nhất.
1 1 2
Dự đoán với c =0 ⇒ P = + = , a + 2b =4 . P lúc này đạt giá trị

om
ab ab ab
nhỏ nhất tại=a 2,= b 1.
Ta có: .c
1 1 2
P= + ≥
ab + bc + ca
( )
ok
ab + bc + c 2 4 ( ab + bc + ca ) ab + bc + c 2

2 2 2 2 4 2 4 2
≥ = ≥ ≥ =
bo

2
2ab + 2bc + ca + c 2 ( a + c )( 2b + c ) a + 2b + 2c a + 2b + 3c
et

Vậy giá trị nhỏ nhất của P bằng 2 xảy ra khi


c = 0
2b + c = a + c a = 2
vi

 
 ⇔ b = 1.
ng

ab + bc + ca = ab + bc + c
2
c = 0
a + 2b + 3c = 
 4
a

Bài 22. Cho a,b,c là các số thực thỏa mãn 0 < a ≤ b ≤ c và


kh

a 2 − 1 b2 − 1 c2 − 1
+ + =
0.
a b c
Tìm giá trị nhỏ nhất của biểu thức P =a + b 2014 + c 2015 .
Lời giải
a 2 − 1 b2 − 1 c2 − 1 a2 − 1
Từ 0 < a ≤ b ≤ c suy ra 0 = + + ≥ 3.
a b c a
c2 − 1
⇒ a ≤ 1 và 0 ≤ 3. ⇒ c ≥ 1.
c

153
Khám phá tư duy Kỹ thuật giải bất ĐT Bài toán Max – Min – Đặng Thành Nam

Ta có:
b2 − 1 c2 − 1 1 − a 2 1 1  1 
+ = ≥ 0 ⇒ b + c − − ≥ 0 ⇔ ( b + c ) 1 −  ≥ 0 ⇔ bc ≥ 1 .
b c a b c  bc 
Mặt khác:
c2 − 1 1 − a 2 1 − b2  1   1 
= + = ( a + b )  − 1 ≥ 2 ab  − 1
c a b  ab   ab 
 1  1
= 2 − ab  ≥ − ab
 ab  ab
1 1  1  ab  1

n
Suy ra c − ≥ − ab ⇔  c −  1 +  ≥ 0 ⇔ c ≥ ⇔ abc 2 ≥ 1 .
c ab  ab   c  ab

.v
Khi đó sử dụng bất đẳng thức AM-GM ta được:

om
33 abc 2 ( bc )
2013
P=
a + b 2014 + c 2015 ≥ 3 ab 2014 c 2015 = ≥ 3.
3

Vậy giá trị nhỏ nhất của P bằng 3 đạt tại a= b= c= 1 .


Bài 23. Cho a,b,c,d là các số thực dương.
.c
b(a + c) c (b + d ) d (c + a) a (d + b)
ok
Chứng minh rằng + + + ≥4.
c (a + b) d (b + c ) a (c + d ) b(d + a)
Lời giải
bo

Đặt P là vế trái của bất đẳng thức.


Viết lại P dưới dạng:
et

 b d   c a 
P = (a + c)  +  + (b + d )  + 
 c ( a + b ) a ( c + d )   d ( b + c ) b ( d + a ) 
vi

 a+c b+d 
= ( abc + bcd + cda + dab ) 
ng

+ 
 ac ( a + b )( c + d ) bd ( b + c )( a + d ) 
 
a

1 1 1 1
 + + 
1 1 1 1 a c b d
=  + + +  + 
kh

.
 a b c d    1 + 1  1 + 1   1 + 1  1 + 1  
  a b   
 c d   b c  a d  
 

 1 1 1 1 
 4 +  4 +  
1 1 1 1 a c b d 
≥  + + + .  + = 4
 a b c d 
 1 1 1 1 2
 1 1 1 1
2
 + + +   + + +  
  a b c d   a b c d  
Bất đẳng thức được chứng minh. Đẳng thức xảy ra khi và chỉ khi a= b= c= d .

154
Cty TNHH MTV DVVH Khang Việt

Bài 24. Cho a,b,c là các số thực không âm. Chứng minh
a 3 + b3 + c 3 3
≥ abc + ( a − b )( b − c )( c − a ) .
3 4
Lời giải
Bất đẳng thức tương đương với:
a3 + b3 + c3 − 3abc 3
≥ ( a − b )( b − c )( c − a ) .
3 4
( a + b + c ) ( a − b )2 + ( b − c )2 + ( c − a )2  3
⇔ ≥ ( a − b )( b − c )( c − a ) .
6 4

n
Sử dụng bất đẳng thức AM-GM cho vế trái ta được:

.v
( a + b + c ) ( a − b )2 + ( b − c )2 + ( c − a )2  ( a + b + c ) 3 ( a − b )2 ( b − c )2 ( c − a )2

om
6 2
Vậy ta chứng minh

( a + b + c ) 3 ( a − b )2 ( b − c )2 ( c − a )2 3
( a − b )( b − c )( c − a ) .
2

4
.c
ok
⇔ 2 ( a + b + c ) ≥ 33 ( a − b )( b − c )( c − a ) .
Không mất tính tổng quát giả sử a ≥ b ≥ c ta cần chứng minh
bo

2 ( a + b + c ) ≥ 3 3 ( a − b )( b − c )( a − c ) .
Sủ dụng bất đẳng thức AM-GM cho vế phải ta được:
et

3 3 ( a − b )( b − c )( a − c ) ≤ a − b + b − c + a − c = 2a − 2c ≤ 2 ( a + b + c ) .
vi

Bất đẳng thức được chứng minh. Đẳng thức xảy ra khi và chỉ khi a= b= c .
Một số kết quả tương tự:
ng

Bài 1. Cho a,b,c là các số thực không âm. Chứng minh


a 3 + b3 + c 3 1
≥ abc + 3 ( a − b )( b − c )( c − a ) .
a

3 2
kh

Bài 2. Cho a,b,c là các số thực không âm thỏa mãn 2 min {a, b, c} ≥ max {a, b, c} .
a 3 + b3 + c 3 3
Chứng minh ≥ abc + 3 ( a − b )( b − c )( c − a ) .
3 2
Bài 25. Cho a,b,c là các số thực không âm thỏa mãn điều kiện a + b + c = 1.
Tìm giá trị nhỏ nhất của biểu thức
1 1 1
P= + + .
a2 +
(b − c ) 2
b2 +
(c − a) 2
c2 +
(a − b) 2

4 4 4

155
Khám phá tư duy Kỹ thuật giải bất ĐT Bài toán Max – Min – Đặng Thành Nam

Lời giải
Không mất tính tổng quát giả sử c = max {a, b, c} .

Ta có: a2 +
( b − c )2 c
≤ a + , b2 +
(c − a) ≤ b + c . 2

4 2 4 2

c2 +
( a − b )2  c  c
≤ c 2 + 2a 2 + 2b 2 ≤ 2  a +  + 2  b +  .
2 2

4  2  2
c c
Đặt x =a+ ,y= b + , ( x, y > 0 ) và x + y =
1.
2 2

n
1 1 1 5
Ta chứng minh được: + + ≥ .
( )

.v
x y 2 x +y
2 2 x + y

om
Do đó P ≥ 5 .
Vậy giá trị nhỏ nhất của P bằng 5 đạt tại a= b= 0, c= 1 hoặc các hoán vị.

C. BÀI TẬP RÈN LUYỆN


.c
ok
BÀI TẬP TRUNG BÌNH
Bài 1. Cho x,y là hai số thực dương. Tìm giá trị nhỏ nhất của biểu thức
x 3 + y 3 + 7 xy ( x + y )
bo

P= .
xy x 2 + y 2
Bài 2. Cho x,y là hai số thực dương.
et

 3  2 3  1  1
Chứng minh rằng  x 2 + y +  y + x +  ≥  2 x +  2 y +  .
vi

 4  4  2  2
ng

Bài 3. Cho a,b là các số thực dương. Chứng minh


(1 + a ) + (1 + b ) ≥ 128ab ( a + b ) .
8 8 2
a
kh

Bài 4. Chứng minh rằng với mọi số thực x,y,z ta có


x2 + y 2 + z 2 ≥ 2 x ( y + z ) .
Bài 5. Cho a,b,c là các số thực thỏa mãn điều kiện ( a + c )( a + b + c ) ≤ 0 .

Chứng minh rằng ( b − c ) ≥ 4a ( a + b + c ) .


2

Bài 6. Cho a,b,c là các số thực không âm thỏa mãn điều kiện
ab + bc + ca + abc =4.
(
Chứng minh rằng 3 a 2 + b 2 + c 2 + abc ≥ 10 . )
156
Cty TNHH MTV DVVH Khang Việt

Bài 7. Cho a,b là các số thực thoả mãn điều kiện −2 < a < 2, −3 < b < 3; ab =−1 .
4 9 12
Chứng minh rằng + ≥ .
4−a 9−b
2 2
5
Bài 8. Cho x,y là hai số thực dương có tổng bằng 2. Chứng minh
1 1 2 2
+ + 2 ≥ 2+ .
x y x + xy + y 2
3
a b c
Bài 9. Cho a,b,c là các số thực dương thỏa mãn điều kiện + + =
2.
a +1 b +1 c +1
Tìm giá trị nhỏ nhất của biểu thức P = ab + bc + ca .

n
Bài 10. Cho a,b,c là các số thực dương thỏa mãn điều kiện

.v
1 1 1
+ + =
1.

om
a +1 b +1 c +1
Chứng minh rằng abc ≥ 8 .

Bài 11. Chứng minh rằng x ( 25 − y 2 ) + y 25 − x ≤ 25 .


.c
x (108 − y 2 ) + y 108 − x ≤ 108 .
ok
Bài 12. Chứng minh rằng
Bài 13. Cho x,y là hai số thực thoả mãn điều kiện
bo

x 2014 − y 2 + y 2014 − x 2 =
2014 .
Chứng minh rằng : x 2 + y 2 =
2014 .
et

4
Bài 14. Cho hai số thực x > y > 0. Chứng minh x + ≥3.
vi

( x − y )( y + 1)
2
ng

Bài 15. Cho a,b,c là các số thực dương thoả mãn điều kiện a + b + c ≤ 3 .
1 2014 2015
Chứng minh rằng + ≥ .
a + b + c ab + bc + ca
a

2 2 2
3
kh

Bài 16. Cho a,b,c là các số thực dương thoả mãn điều kiện a 3 + b3 + c 3 ≤ 3 .
1 8 4
Chứng minh rằng 3 + ≥ .
a + b + c ( a + b )( b + c )( c + a ) 3
3 3

1 2
Bài 17. Cho a,b là hai số thực dương thoả mãn điều kiện + =
2.
a +1 b +1
1
Chứng minh rằng ab 2 ≤ .
8

157
Khám phá tư duy Kỹ thuật giải bất ĐT Bài toán Max – Min – Đặng Thành Nam

1 1 1
Bài 18. Cho n số thực dương thoả mãn điều kiện + + ... + =
1.
1 + a1 1 + a2 1 + an
Chứng minh rằng a1a2 ...an ≥ ( n − 1) .
n

Bài 19. Cho a,b,c là các số thực dương. Chứng minh


a+b+c a 2 + b 2 + c 2 1  ab bc ca 
≤ ≤  + + .
3 3 3 c a b 
Bài 20. Cho a,b,c là các số thực dương thỏa mãn điều kiện a 2 + b 2 + c 2 =
1.
1
Chứng minh a + b + c + ≥4 3.

n
abc

.v
Bài 21. Cho a,b,c là độ dài ba cạnh một tam giác có chu vi bằng 1.
Tìm giá trị nhỏ nhất của biểu thức

om
(a + b − c) (b + c − a ) (c + a − b)
3 3 3

P= + + .
3c 3a
3b
Bài 22. Cho a,b,c là các số thực không âm thỏa mãn a + b + c =3.
.c
Chứng minh rằng a b + b c + c a + abc ≤ 4 .
2 2 2
ok
Bài 23. Cho a,b,c là các số thực không âm thỏa mãn a + b + c =3.
Chứng minh rằng a b + b c + c a + 2abc + 4 ≥ 3 ( ab + bc + ca ) .
2 2 2
bo

Bài 24. Cho a,b,c là các số thực không âm. Chứng minh rằng

( a b + b c + c a ) ( ab + bc + ca ) ≤ 271 ( a + b + c ) 5
et

2 2 2
.

Bài 25. Cho a,b,c là các số thực dương. Tìm giá trị lớn nhất của biểu thức
vi

(
abc a + b + c + a 2 + b 2 + c 2 ).
ng

P=
(a 2
+b +c
2 2
) ( ab + bc + ca )
a

Bài 26. Cho a,b,c là các số thực thỏa mãn điều kiện abc=1 .
b+c c+a a+b
kh

Chứng minh rằng: + + ≥ a + b + c +3.


a b c
3
Bài 27. Cho a,b,c là các số thực dương thoả mãn điều kiện a 2 + b 2 + c 2 ≤ .
4
Tìm giá trị nhỏ nhất của biểu thức
1 1 1
P = ( a + b )( b + c )( c + a ) + + + .
a 2 b2 c2
Bài 28. Cho a,b,c là các số nguyên dương có tổng bằng 100.
Tìm giá trị lớn nhất của tích ba số đó.

158
Cty TNHH MTV DVVH Khang Việt

Bài 29. Cho a,b,c là các số nguyên dương có tổng bằng 2014.
Tìm giá trị lớn nhất của biểu thức P = abc .
Bài 30. Cho x,y,z là các số thực dương. Chứng minh rằng
1 1 1 27
+ 2 + 2 ≥ .
x + xy y + yz z + zx 2 ( x + y + z )2
2

Bài 31. Cho a,b,c là các số thực dương thoả mãn điều kiện ab + bc + ca =
3.
Chứng minh rằng a 6 + b 6 + 1 + b 6 + c 6 + 1 + c 6 + a 6 + 1 ≥ 3 3 .
Bài 32. Cho a,b,c là các số thực dương chứng minh
a 2 + 2b 2 + b 2 + 2c 2 + c 2 + 2a 2 ≥ 3 ( a + b + c ) .

n
.v
Bài 33. Cho n số thực dương có tổng bằng 1. Chứng minh
1  1   1 

om
 − 1 − 1 ...  − 1 ≥ ( n − 1) .
n

 a1  a2   an 
Bài 34. Cho a,b là các số thực dương thoả mãn điều kiện a + b ≤ 1 .
.c
1 1
=
Tìm giá trị nhỏ nhất của biểu thức P + .
1 + a + b 2ab
2 2
ok
Chú ý. Có thể đưa về hàm số(xem chương sau).
Bài tập tương tự
bo

Cho a,b là hai số thực dương thoả mãn điều kiện a + b ≤ 2 .


1 2
= +
et

Tìm giá trị nhỏ nhất của biểu thức P .


1 + a + b ab 2 2

Bài 35. Cho a,b,c là các số thực dương thoả mãn điều kiện a + 2b + 3c ≥ 20 .
vi

3 9 4
Tìm giá trị nhỏ nhất của biểu thức P = a + b + c + + + .
ng

a 2b c
Bài 36. Cho x,y,z là các số thực dương. Chứng minh
a

2 xy 2 yz 3 xz 5
+ + ≥ .
kh

( x + z )( z + y ) ( x + y )( x + z ) ( y + z )( y + x) 3
Bài 37. Cho a,b,c là các số thực thoả mãn a, b, c ≥ 1 .
a ( b 2 + 3) b ( c 2 + 3) c ( a 2 + 3)
Chứng minh rằng + + ≥ 3.
3c 2 + 1 3a 2 + 1 3b 2 + 1
Bài 38. Cho x, y, z là các số thực dương có tích bằng 1.
Tìm giá trị nhỏ nhất của biểu thức

159
Khám phá tư duy Kỹ thuật giải bất ĐT Bài toán Max – Min – Đặng Thành Nam

x4 y4 z4
P= + + .
(1 + y )(1 + z ) (1 + z )(1 + x ) (1 + x )(1 + y )
Bài 38. Sử dụng bất đẳng thức AM – GM cho bốn số dương ta có:

 x4 1+ y 1+ z 1
 (1 + y )(1 + z ) + 8 + 8 + 4 ≥ x

 y4 1+ z 1+ x 1
 + + + ≥y
 (1 + z )(1 + x ) 8 8 4

z4 1+ x 1+ y 1

n
 + + + ≥z
 (1 + x )(1 + y )

.v
 8 8 4
Cộng lại theo vế ba bất đẳng thức trên ta được:

om
1 1 1
Bài 39. Cho a,b,c là các số thực dương thoả mãn điều kiện a + b + c ≥ + + .
a b c
Chứng minh rằng a + b + c ≥
3
+
2
a + b + c abc
.
.c
ok
Bài 40. Cho x,y là các số thực dương. Chứng minh
6
x4 y 4 3
≤  .
bo

( x + y ) ( x + 1) ( y + 1) 8
2 4 4

Bài 41. Cho x,y,z là các số thực dương thoả mãn điều kiện z ( z − x − y ) = x + y + 1 .
et

x4 y 4
Tìm giá trị lớn nhất của biểu thức M =
vi

.
( x + yz )( y + zx)( z + xy )3
ng

Bài 42. Cho x,y,z là các số thực thoả mãn điều kiện x 2 + y 2 + z 2 =
3.
Chứng minh rằng x + y + z − xyz ≤ 4 .
a

Bài 43. Cho a,b,c là các số thực dương có tổng bằng 3. Chứng minh
kh

a +1 b +1 c +1
+ + ≥ 3.
ab + 1 bc + 1 ca + 1
Bài 44. Cho a,b,c là các số thực dương có tổng bằng 3. Chứng minh
a ( a + c − 2b ) b ( b + a − 2c ) c ( c + b − 2a )
+ + ≥ 0.
ab + 1 bc + 1 ca + 1
Bài 45. Cho x,y,z là các số thực lớn hơn 1. Chứng minh
x4 y4 z4
+ + ≥ 48 .
( y − 1) ( z − 1) ( x − 1)
2 2 2

160
Cty TNHH MTV DVVH Khang Việt

3
Bài 46. Cho a,b,c là các số thực dương thoả mãn điều kiện a + b + c = abc .
4
a b c 3
Chứng minh rằng + + ≥ .
a+2 b+2 c+2 2
Bài 47. Cho a,b là hai số thực dương thoả mãn điều kiện a + b ≥ 4 .
3a 2 + 4 2 + b3
=
Tìm giá trị nhỏ nhất của biểu thức P + 2 .
4a b
Bài 48. Cho a,b,c là các số thực dương tìm giá trị lớn nhất của biểu thức

n
.v
Bài 49. Cho các số thực dương a, b, c thỏa mãn điều kiện

om
.

Chứng minh rằng .


.c
Bài 50. Cho x,y là hai số thực dương. Chứng minh rằng
ok
.
bo

Bài 51. Cho . Tìm giá trị lớn nhất của biểu thức
et

.
vi

3
Bài 52. Cho a,b,c là các số thực dương có tổng bằng . Chứng minh
ng

4
.
Bài 53. (TSĐH Khối B 2007) Cho x,y,z là các số thực dương. Tìm giá trị nhỏ nhất
a
kh

của biểu thức .

Bài 54. Cho a,b,c là các số thực không âm thoả mãn điều kiện .

Chứng minh rằng .

Bài 55. Cho a,b,c là các số thực không âm có tổng bằng 3. Chứng minh

161
Khám phá tư duy Kỹ thuật giải bất ĐT Bài toán Max – Min – Đặng Thành Nam

Bài 56. Cho a,b,c là các số thực dương có tổng bằng 3. Chứng minh

Bài 57. Cho a,b,c là các số thực dương thoả mãn điều kiện .

Chứng minh rằng .

BÀI TẬP RÈN LUYỆN NÂNG CAO


Bài 58. Cho x,y,z là các số thực dương thỏa mãn x > y và xy + ( x + y ) z + z 2 =
1.

n
1 1 1
Tìm giá trị nhỏ nhất của biểu thức P = + +

.v
.
4( x − y) (x + z) ( y + z)
2 2 2

om
Bài 59. Cho a,b,c là các số thực dương thỏa mãn a > b và ( a + c )( b + c ) =
1.
1 6 12
Tìm giá trị nhỏ nhất của biểu thức P = + + .
(a − b) (b + c ) (a + c)
.c 2 2 2

Bài 60. Cho a,b,c,x,y,z là các số thực dương thoả mãn điều kiện
ok
.
bo

Chứng minh rằng .

Bài 61. Cho x,y,z là các số thực dương. Chứng minh rằng
et

.
vi
ng

Bài 62. Cho a,b,c là các số thực không âm thoả mãn điều kiện .
Chứng minh rằng .
a

Bài 63. Cho a,b,c là các số thực không âm thoả mãn điều kiện .
kh

Chứng minh rằng .

Bài 64. Cho số nguyên dương n và các số thực dương .


Chứng minh rằng
.
Bài 65. Cho a,b,c là các số thực dương. Chứng minh

162
Cty TNHH MTV DVVH Khang Việt

Bài 66. Cho a,b,c là các số thực không âm thỏa mãn điều kiện ab + bc + ca > 0 .
Tìm giá trị nhỏ nhất của biểu thức
 1 1 
P = ( a + 2b + 2014c )  + .
 a ( b + c ) + 2c 2
 b ( a + 4c )

Bài 67. Cho a,b,c là các số thực đôi một phân biệt thỏa mãn ab + bc + ca > 0 và
a+b+c = 1.
Tìm giá trị nhỏ nhất của biểu thức
2 2 2 5
P= + + + .
a −b b−c c−a

n
ab + bc + ca

.v
Bài 68. Cho x,y,z là các số thực không âm thoả mãn điều kiện
.

om
Chứng minh rằng .
.c
Bài 69. Cho a,b,c là các số thực dương thoả mãn điều kiện
.
ok
Chứng minh rằng .
bo

Bài 70. Cho a,b,c là độ dài 3 cạnh một tam giác thoả mãn điều kiện
et

.
vi

Chứng minh rằng .


ng

Bài 71. Cho a,b,c là các số thực không âm. Chứng minh

.
a

Bài 72. Cho x,y,z là các số thực dương thoả mãn điều kiện
kh

Chứng minh rằng .

Bài 73. Cho a,b,c là độ dài 3 cạnh một tam giác có chu vi bằng 3.

Chứng minh rằng .

Bài 74. Cho x,y,z là các số thực không âm thoả mãn điều kiện
.

163
Khám phá tư duy Kỹ thuật giải bất ĐT Bài toán Max – Min – Đặng Thành Nam

Tìm giá trị nhỏ nhất của biểu thức

Bài 75. Cho a,b,c là các số thực không âm thoả mãn điều kiện .
Chứng minh rằng .
Bài 76. Cho a,b,c là các số thực không âm thoả mãn điều kiện .
Chứng minh rằng .
Bài 79. Cho a,b,c là các số thực dương có tổng bằng 3.

n
Chứng minh rằng .

.v
Bài 80. Cho a,b,c là các số thực dương thoả mãn điều kiện

om
.

Chứng minh rằng .


.c
ok
Bài 81. Cho a,b,c là các số thực dương thoả mãn điều kiện

.
bo

Tìm giá trị nhỏ nhất của biểu thức .


et

Bài 82. Cho a,b,c là các số thực không âm có tổng bằng 2 chứng minh
vi

.
ng

Bài 83. Cho x,y,z là các số thực không âm thoả mãn điều kiện .
a

Tìm giá trị nhỏ nhất của biểu thức


kh

Bài 84.(Việt Nam TST 2010) Cho a,b,c là các số thực dương thoả mãn điều kiện

Chứng minh rằng

164
Cty TNHH MTV DVVH Khang Việt

Bài 85. Cho a,b,c là các số thực dương có tổng bằng 1. Chứng minh

Bài 86. Cho a,b,c là các số thực dương có tổng bằng 1. Chứng minh

Bài 87. Cho a,b,c là các số thực dương có tổng bằng 3. Chứng minh

n
Bài 88. Cho a,b,c là các số thực dương có tổng bằng 1. Chứng minh

.v
om
.

Bài 89. Cho a,b,c là các số thực không âm thoả mãn điều kiện .

Chứng minh rằng


.c .
ok
Bài 90. Cho a,b,c là các số thực không âm có tổng bằng 3. Chứng minh
.
bo

Bài 91. Cho a,b,c là các số thực dương. Chứng minh rằng

.
et

Bài 92. Cho a,b,c là độ dài ba cạnh một tam giác có chu vi bằng 3. Chứng minh
vi

.
ng

Bài 93. Cho a,b,c là các số thực dương. Chứng minh


a

.
kh

Bài 94. Cho a,b,c là các số thực không âm thoả mãn điều kiện .
Chứng minh rằng

Bài 95. Cho a,b,c là các số thực thoả mãn điều kiện
.

165
Khám phá tư duy Kỹ thuật giải bất ĐT Bài toán Max – Min – Đặng Thành Nam

Chứng minh rằng .

Bài 96. Cho a,b,c là các số thực dương thoả mãn điều kiện .

Chứng minh rằng .

Bài 97. Cho x,y,z là các số thực dương có tích bằng 1. Chứng minh

n
.

.v
Bài 98. Cho a,b,c là các số thực dương. Chứng minh

om
.
.c
Bài 99. Cho x,y,z là các số thực dương có tích bằng 1. Chứng minh
ok
.

Bài 100. Cho a,b,c là các số thực dương có tổng bằng 3. Chứng minh
bo
et
vi

D. HƯỚNG DẪN GIẢI – ĐÁP SỐ


Bài 1. Sử dụng bất đẳng thức AM-GM ta được:
ng

x 3 + y 3 + 7 xy ( x + y ) =
( x + y ) + 4 xy ( x + y ) ≥ 4 xy ( x + y ) .
3 2

xy ( x + y )
a

2
xy 2 xy + x 2 + y 2
. 2 xy ( x 2 + y 2 ) ≤
xy
xy=
x +y2 2
. = .
kh

2 2 2 2 2
Do đó P ≥ 8 2 . Đẳng thức xảy ra khi và chỉ khi x = y .
=
Nhận xét. Ngoài ra ta có thể đặt x t. y, ( t > 0 ) đưa về khảo sát hàm một biến với t.
Bài 2. Ta có:
2
 1 1  2 3  1 3 1
 x −  = x − x + =  x + y + − x + y +  ⇒ x + y + ≥ x + y +
2 2

 2 4  4  2 4 2

166
Cty TNHH MTV DVVH Khang Việt

3 1
Tượng tự y 2 + x + ≥ x+ y+ .
4 2
Suy ra:
2 2
 2 3  2 3  1  1 1
 x + y +  y + x +  ≥  x + y +  =  x + + y + 
 4  4  2  4 4
 1  1  1  1
≥ 4  x +  y +  =  2 x +  2 y + 
 4  4  2  2
1
Bất đẳng thức được chứng minh. Đẳng thức xảy ra khi và chỉ khi x= y= .
2

n
Bài 3. Sử dụng bất đẳng thức AM – GM cho hai số dương ta có

.v
(1 + a ) + (1 + b ) ≥ 2 (1 + a ) (1 + b )
8 8 4 4

om
4
= 2 ( ab + 1) + ( a + b )  ≥ 2  2 ( ab + 1)( a + b ) 
4
.

= 32 ( ab + 1) ( a + b ) ≥ 128ab ( a + b )
2 2 2

Bất đẳng thức được chứng minh. Đẳng thức xảy ra khi và chỉ khi
.c .
ok
Bài 4. Sử dụng bất đẳng thức AM – GM ta có
bo

Bài 5. Ta có: ( b − c ) = ( a + b + c − a − 2c ) ≥ 4 ( a + b + c )( −a − 2c ) .
2 2
et

Vậy bất đẳng thức được chứng minh nếu ta chứng minh được:
vi

( a + b + c )( −a − 2c ) ≥ a ( a + b + c ) ⇔ ( a + c )( a + b + c ) ≤ 0 (luôn đúng theo


ng

giả thiết).
Bài 6. Ta có:
a

a+b+c 
3

( a + b + c ) + 
1
4 = ab + bc + ca + abc ≤  ⇒ a +b+c ≥ 3.
2
kh

3  3 
Khi đó
3 ( a 2 + b 2 + c 2 ) + abc= 3 ( a 2 + b 2 + c 2 ) + 4 − ( ab + bc + ca )
1
≥ (a + b + c) + 4 −
(a + b + c)
2 2

3
2 2
= ( a + b + c ) + 4 ≥ .32 + 4= 10
2

3 3
Bất đẳng thức được chứng minh đẳng thức xảy ra khi và chỉ khi a= b= c= 1 .

167
Khám phá tư duy Kỹ thuật giải bất ĐT Bài toán Max – Min – Đặng Thành Nam

Bài 7. Sử dụng bất đẳng thức AM – GM ta có

Bất đẳng thức được chứng minh. Đẳng thức xảy ra khi và chỉ khi

n
.v
hoặc .

Bài 8. Sử dụng bất đẳng thức AM – GM ta có

om
.c
ok
bo
et
vi
a ng
kh

Cộng theo vế hai bất đẳng thức trên ta có đpcm.


Bài 9. Theo giả thiết ta có
a b c 1 1 2
=
1− +1− = + ≥ .
a +1 b +1 c +1 b +1 c +1 ( b + 1)( c + 1)
b 2
Tương tự ta có: ≥ .
b +1 ( a + 1)( c + 1)
Nhân theo vế hai bất đẳng thức trên ta được:

168
Cty TNHH MTV DVVH Khang Việt

ab 4 4 ( a + 1)( b + 1)
≥ ⇔ ab ≥ .
( a + 1)( b + 1) ( c + 1) ( a + 1)( b + 1) c +1

4 ( b + 1)( c + 1) 4 ( c + 1)( a + 1)
Tương tự ta có: bc ≥ ; ca ≥ .
a +1 b +1
Cộng lại theo vế 3 bất đẳng thức trên và sử dụng AM-GM ta được:
P = ab + bc + ca ≥ 12 .
Đẳng thức xảy ra khi và chỉ khi a= b= c= 2 .
a 1 1 1
=
1− = +

n
Bài 10. Theo giả thiết ta có .
a +1 a +1 b +1 c +1

.v
a 2
Sử dụng bất đẳng thức AM-GM ta có: ≥ .
a +1 ( b + 1)( c + 1)

om
b 2 c 2
Tương tự ta có: ≥ ; ≥ .
b +1 ( a + 1)( c + 1) c +1
.c ( a + 1)( b + 1)
Nhân theo vế 3 bất đẳng thức trên ta có ngay điều phải chứng minh.
ok
Đẳng thức xảy ra khi và chỉ khi a= b= c= 2 .
Nhận xét. Với cách làm tương tự trên ta xử lý được bài toán tổng quát cho n
bo

số dương.
1 1 1
Bài toán. Với x1 , x2 ,..., xn > 0 thỏa mãn + + ... + =
1 ta luôn có
1 + x1 1 + x2 1 + xn
et

x1 x2 ...xn ≥ ( n − 1) .
n
vi

Bài 11. Sử dụng bất đẳng thức AM – GM cho hai số ta có


a ng
kh

Cộng lại theo vế hai bất đẳng thức trên ta có đpcm. Đẳng thức xảy ra khi và chỉ
khi .
Bài 14. Sử dụng bất đẳng thức AM – GM ta có

169
Khám phá tư duy Kỹ thuật giải bất ĐT Bài toán Max – Min – Đặng Thành Nam

Bất đẳng thức được chứng minh.

Đẳng thức xảy ra khi và chỉ khi .

Bài 15. Dấu bằng đạt tại .

Sử dụng bất đẳng thức AM – GM ta có


1 2014  1 2  2012
+ =  2 + +
a + b + c ab + bc + ca  a + b + c ab + bc + ca  ab + bc + ca
2 2 2 2 2

n
3 2012
≥ +
( ) ( ab + bc + ca ) 13 ( a + b + c )2

.v
3 a 2 + b2 + c2
2

om
9 2012 2015
≥ + ≥
( a + b + c ) 1 ( a + b + c )2
2
3
3.c
Bất đẳng thức được chứng minh. Đẳng thức xảy ra khi và chỉ khi .
Bài 17. Theo giả thiết ta có
ok
bo
et

Nhân theo vế hai bất đẳng thức trên ta có đpcm. Đẳng thức xảy ra khi và chỉ khi
vi

.
ng

Bài 18. Ta có .
a
kh

Nhân theo vế n bất đẳng thức trên ta có điều phải chứng minh.
Đẳng thức xảy ra khi và chỉ khi .
Bài 19. Bất đẳng thức vế trái là hiển nhiên theo C – S.
Bất đẳng thức vế phải tương đương với

170
Cty TNHH MTV DVVH Khang Việt

Bất đẳng thức được chứng minh. Đẳng thức xảy ra khi và chỉ khi .
Bài 20. Sử dụng bất đẳng thức AM-GM cho ba số dương ta được:
1 1

n
1 = a 2 + b 2 + c 2 ≥ 3 3 a 2b 2 c 2 ⇒ abc ≤ ⇒ ≥3 3
3 3 abc

.v
Sử dụng bất đẳng thức AM-GM cho bốn số dương ta được:

om
1 1 4
a+b+c+ ≥ 4 4 abc. = .
9abc 9abc 3
 1 
Suy ra a + b + c +
1
abc 
= a +b+c + +
9abc  9abc
8 .c

4 8
+ .3 3 = 4 3 .
3 9
ok
1
Bất đẳng thức được chứng minh. Đẳng thức xảy ra khi và chỉ khi a= b= c= .
3
bo

Bài 21. Sử dụng bất đẳng thức AM-GM cho ba số dương ta được:
(a + b − c) (a + b − c) ≥ a + b + 4 c − 1 .
3 3
c 1
et

+ + ≥ a+b−c ⇒
3c 3 3 3c c 3
vi

(b + c − a ) (c + a − b) ≥ a + c + 4 b −1 .
3 3
4
Tương tự: ≥ b + c + a − 1;
ng

3a 3 3b 3
Cộng theo vế 3 bất đẳng thức trên kết hợp với điều kiện a + b + c = 3 ta có ngay
P ≥ 1 .Đẳng thức xảy ra khi và chỉ khi a= b= c= 1 .
a

Vậy giá trị nhỏ nhất của P bằng 1 đạt tại a= b= c= 1 .


kh

Bài 22. Không mất tính tổng quát giả sử b là số nằm giữa a và c.
Ta có:
( b − a )( b − c ) ≤ 0 ⇔ b2 + ac ≤ ab + bc ⇒ b2c + c 2 a ≤ abc + bc 2
⇒ a 2b + b 2 c + c 2 a + abc ≤ a 2b + bc 2 + 2abc = b ( a + c )
2

a+c a+c 
3

a+c a+c b+ 2 + 2  4
= 4.b. ≤ 4 = ( a + b + c=
) 4
3
. 
2 2  3  27
 
171
Khám phá tư duy Kỹ thuật giải bất ĐT Bài toán Max – Min – Đặng Thành Nam

Bất đẳng thức được chứng minh. Đẳng thức xảy ra khi và chỉ khi a= b= c= 1 .
Bài 23. Ta có
3 ( ab + bc + ca ) = ( a + b + c )( ab + bc + ca ) = ( a 2b + b 2 c + c 2 a ) + ( ab 2 + bc 2 + ca 2 ) + 3abc .
Do vậy ta chỉ cần chứng minh ab 2 + bc 2 + ca 2 + abc ≤ 4 .
Đây là một bài toán quen thuộc đã được chứng minh ở bài toán trên.
Bài toán được chứng minh đẳng thức xảy ra khi và chỉ khi a= b= c= 1 .
Bài 24. Không mất tính tổng quát giả sử b là số nằm giữa a và c.
Ta có:
( b − a )( b − c ) ≤ 0 ⇔ b2 + ac ≤ ab + bc ⇒ b2c + c 2 a ≤ abc + bc 2

n
.
⇒ a 2b + b 2 c + c 2 a ≤ a 2b + bc 2 + abc = b ( a 2 + ac + c 2 )

.v
Sử dụng bất đẳng thức AM-GM ta có:

om
2
 a 2 + ac + c 2 + ab + bc + ca 
b ( a + ac + c ) ( ab + bc + ca ) ≤ b 
2 2

 2 
b ( a + c ) + b ( a + c ) 

2

2
b (a + c) (a + b + c)
2 2
a+c a+c
.c
= = b. .( a + b + c )
2
.
ok
4 4 2 2
a+c a+c 
3

b+ +
bo

2  1
≤ (a + b + c)  2 2 = (a + b + c)
5

 3  27
 
et

Bất đẳng thức được chứng minh. Đẳng thức xảy ra khi và chỉ khi a= b= c .
vi

Bài 25. Sử dụng bất đẳng thức C-S ta có:


3 ( a 2 + b2 + c2 ) ≥ ( a + b + c ) ⇒ a + b + c ≤ 3 ( a 2 + b2 + c2 ) .
ng

Suy ra
a

abc  3 ( a 2 + b 2 + c 2 ) + a 2 + b 2 + c 2  3 + 1 abc ( )
  =
kh

P≤ .
( a 2 + b2 + c 2 ) ( ab + bc + ca ) ( ab + bc + ca ) a 2 + b 2 + c 2
Mặt khác: ( ab + bc + ca ) a 2 + b 2 + c 2 ≥ 3 3 a 2b 2 c 2 . 3 3 a 2b 2 c 2 =
3 3abc .
3 +1 3 + 3
Suy ra P ≤ = . Đẳng thức xảy ra khi và chỉ khi a= b= c .
3 3 9
3+ 3
Vậy giá trị lớn nhất của P bằng đạt tại a= b= c .
9

172
Cty TNHH MTV DVVH Khang Việt

Bài 26. Sử dụng AM –GM cho 2 số dương ta được

Xây dựng tương tự cho 2 bất đẳng thức còn lại rồi cộng lại theo vế 3 bất đẳng
thức ta được

n
.

.v
Bài 27. Theo giả thiết và bất đẳng thức AM – GM ta có:

om
.

Suy ra .
.c
ok
Sử dụng bất đẳng thức AM - GM ta có
bo

.
et

Sử dụng bất đẳng thức Cauchy - Schwarz ta có


vi

.
ng

Suy ra .
a

Đẳng thức xảy ra khi và chỉ khi


kh

. Vậy giá trị nhỏ nhất của P bằng 13.

Bài 28. Nếu không có điều kiện ràng buộc a,b,c nguyên dương ta có ngay

Đẳng thức xảy ra tại .

Vì vậy ta dự đoán dấu bằng đạt tại


Không mất tính tổng quát giả sử ta có

173
Khám phá tư duy Kỹ thuật giải bất ĐT Bài toán Max – Min – Đặng Thành Nam

Đẳng thức xảy ra khi và chỉ khi hoặc các hoán vị.

Bài 29. Giả sử .

Sử dụng bất đẳng thức AM – GM ta có

n
3
 a b c 
+ +

.v
  672 ( a + b + c ) − a 
3
a b c  672 671 671
. . ≤  =   ≤1

om
672 671 671  3   671.672.3 
 
⇒ P= abc ≤ 6712.672
Bài 30. Sử dụng bất đẳng thức AM-GM cho ba số dương ta được:
.c
1 1 1 3
+ 2 + 2 ≥
ok
x + xy y + yz z + zx (x + xy )( y 2 + yz )( z 2 + zx )
2 2
3
bo

3 3
= =
( )( y + z )( z + x )
3 xyz x + y 3
( xz + yz )( xy + zx )( yz + xy )
et

3 27
≥ ≥
2 ( xy + yz + zx ) 2 ( x + y + z )2
vi

3
ng

Bất đẳng thức được chứng minh. Đẳng thức xảy ra khi và chỉ khi x= y= z .
Bài 31. Sử dụng bất đẳng thức AM – GM cho 3 số dương ta được
a

.
kh

Tương tự cho hai căn thức còn lại rồi cộng lại theo vế ta có đpcm.
Đẳng thức xảy ra khi và chỉ khi .
Bài 32. Sử dụng bất đẳng thức AM – GM dạng luỹ thừa ta được

Tương tự ta có

174
Cty TNHH MTV DVVH Khang Việt

Cộng lại theo vế ba bất đẳng thức trên ta có điều phải chứng minh. Đẳng thức
xảy ra khi và chỉ khi .
Bài tập tương tự
Cho a,b,c là các số thực dương. Chứng minh
.
Bài 33. Sử dụng bất đẳng thức AM – GM cho (n-1) số dương ta được

n
.v
.

om
Nhân theo vế n bất đẳng thức trên ta có đpcm. Đẳng thức xảy ra khi và chỉ khi

.
.c
ok
Bài 34. Dấu bằng đạt tại .
bo

Ta viết lại P và sử dụng bất đẳng thức AM – GM cho hai số


et
vi
a ng

8
Vậy giá trị nhỏ nhất của P bằng đạt tại .
kh

3
Bài 35. Viết lại P và sử dụng bất đẳng thức AM – GM ta có

Dấu bằng đạt tại .


Bài 36. Bất đẳng thức đã cho tương đương với:

175
Khám phá tư duy Kỹ thuật giải bất ĐT Bài toán Max – Min – Đặng Thành Nam

Đến đây chỉ cần áp dụng AM-GM ta có: .

n
Cộng lại theo vế ba bất đẳng thức trên ta có đpcm.

.v
Bài 37. Sử dụng bất đẳng thức AM – GM cho ba số dương ta được

om
.

Chú ý
.c bởi vì
ok
.
bo

Bất đẳng thức được chứng minh. Đẳng thức xảy ra khi và chỉ khi .
et

Bài tập tương tự


vi

Cho x, y, z là các số thực dương có tích bằng 1.


ng

Tìm giá trị nhỏ nhất của biểu thức

.
a
kh

Bài 39. Theo giả thiết ta có

176
Cty TNHH MTV DVVH Khang Việt

Do đó .

Cộng lại theo vế hai bất đẳng thức trên ta có điều phải chứng minh.
Đẳng thức xảy ra khi và chỉ khi .
Bài 40. Sử dụng bất đẳng thức AM – GM cho bốn số dương ta được

Tương tự ta có .

n
Do đó

.v
.

om
Bất đẳng thức được chứng minh. Đẳng thức xảy ra khi và chỉ khi .
.c
Bài 42. Bất đẳng thức đã cho là tổng của hai bất đẳng thức
ok
.
bo

Bất đẳng thức được chứng minh.


et

Đẳng thức xảy ra chẳng hạn tại .


ta sử dụng bất đẳng thức C –S (xem
vi

Chú ý. Nếu cần tìm Max của


chủ đề bất đẳng thức C –S ).
ng

Bài 43. Sử dụng bất đẳng thức AM – GM cho ba số dương ta được


a

.
kh

Ta chỉ cần chứng minh

Bất đẳng thức cuối đúng vì .

Bất đẳng thức được chứng minh. Đẳng thức xảy ra khi và chỉ khi .

177
Khám phá tư duy Kỹ thuật giải bất ĐT Bài toán Max – Min – Đặng Thành Nam

Bài 45. Dự đoán dấu bằng đạt tại .

Vậy ta sử dụng bất đẳng thức AM – GM như sau

n
.v
Cộng theo vế ba bất đẳng thức trên ta có đpcm.
Bài 46. Bất đẳng thức đã cho tương đương với:

om
.

Bất đẳng thức đúng vì .


.c
Bất đẳng thức được chứng minh. Đẳng thức xảy ra khi và chỉ khi .
Bài 47. Áp dụng bất đẳng thức AM – GM ta có
ok
bo
et
vi
a ng

Vì vậy khi .
kh

Bài 48. Đặt .

Chú ý .

178
Cty TNHH MTV DVVH Khang Việt

Vậy đạt tại .

Chú ý. Ta có thể chứng minh bằng bất đẳng thức C –S.


Bài 49. Theo giả thiết ta có: .
Chú ý

n
Tương tự cho hai biểu thức còn lại suy ra

.v
3 1  ( b + c )( c + a ) ( c + a )( a + b ) ( a + b )( b + c ) 
P≥ + ⋅ + + 
2 2  ( a + b )2 (b + c )
2
( c + a ) 
2

om
3 3 ( b + c )( c + a ) ( c + a )( a + b ) ( a + b )( b + c )
≥ + 3 . . =
3
(a + b) (b + c ) (c + a)
2 2 .c 2
2 2
Bất đẳng thức được chứng minh.
ok
Bài 50. Sử dụng bất đẳng thức AM – GM ta có
x x x+ y 1 x x+ y 
= ≤  + ;
bo

.
x + 3y x + y x + 3y 2  x + y x + 3y 
y 1 2y 11 2y 
et

= . ≤  + ;
x + 3y 2 x + 3y 2  2 x + 3y 
vi

Cộng theo vế hai bất đẳng thức trên ta được .


ng

Tương tự ta có .
a
kh

Cộng theo vế hai bất đẳng thức trên có đcpm.


Đẳng thức xảy ra khi và chỉ khi .
x−2 y −3 z−4
Bài 51. Ta có P = + + .
x y z
2 ( x − 2) 3 ( y − 3) 4 ( z − 4)
= + +
x 2 y 3 2z
2+ x − 2 3+ y −3 4+ z − 4 1 1 1
≤ + + = + +
2 2x 2 3y 4z 2 2 2 3 4
179
Khám phá tư duy Kỹ thuật giải bất ĐT Bài toán Max – Min – Đặng Thành Nam

Đẳng thức xảy ra khi và chỉ khi .


Bài 52. Sử dụng bất đẳng thức AM – GM ta có

Cộng theo vế 3 bất đẳng thức trên ta có điều phải chứng minh. Đẳng thức xảy ra

n
khi và chỉ khi .

.v
om
Bài 53. Ta có .

Dự đoán dấu bằng đạt tại khi ba biến bằng nhau, khi đó
.c .
ok
Tức dấu bằng đạt tại .
Vậy ta sử dụng bất đẳng thức AM – GM như sau
bo
et
vi
a ng

Đẳng thức xảy ra khi và chỉ khi .


kh

Bài 54. Sử dụng bất đẳng thức AM – GM ta có

b2 b
Tương tự ta có ≤
3b + ( c + a )
2 2 2(a + b + c)

c2 c

3c + ( a + b )
2 2 2(a + b + c)

180
Cty TNHH MTV DVVH Khang Việt

Cộng lại theo vế 3 bất đẳng thức trên ta có điều phải chứng minh.
Đẳng thức xảy ra khi và chỉ khi=a b= , c 0 hoặc các hoán vị.
Bài 55. Sử dụng bất đẳng thức AM – GM ta có
2
 1
b+c+ 
 2 = a ( 7 − 2a ) .
2
 1
ab + 2abc
= 2ab  c +  ≤ 2a  
 2  2  8
 
a ( 7 − 2a )
2
9
⇔ ( 4 − a )( 2a − 3) ≥ 0 .
2
Ta chỉ cần chứng minh a + ≤
8 2

n
Bất đẳng thức được chứng minh. Đẳng thức xảy ra khi và chỉ khi

.v
3 1
=
a ,=
b 1,=
c .
2 2

om
Bài 56. Chú ý ( a + b + c ) ≥ 3 ( ab + bc + ca ) ⇒ ab + bc + ca ≤ 3 .
2

ab ab ab 1  ab ab 
Ta có ≤ = ≤  + .
( c + a )( c + b ) 
c +3
2
c + ab + bc + ca
2
.c 4 c + a c +b
Tương tự cho 2 căn thức còn lại cộng theo vế 3 bất đẳng thức ta có điều phải
ok
chứng minh.
Đẳng thức xảy ra khi và chỉ khi a= b= c= 1 .
bo

Bài 57. Sử dụng bất đẳng thức AM – GM ta có

=
ab 1 ( a + b)
2

1  a2
+
b2 
et

.  
(
c2 + 1 4 c2 + a 2 + c2 + b2 ) (
4  c 2 + a 2 c 2 + b 2 )
vi

bc 1  b2 c 2  ca 1  c2 a2 
≤  +  ; ≤  + 
a 2 + 1 4  a 2 + b 2 a 2 + c 2  b 2 + 1 4  b 2 + c 2 b 2 + a 2 
ng

Cộng theo vế 3 bất đẳng thức trên ta có điều phải chứng minh. Đẳng thức xảy ra
a

1
khi và chỉ khi a= b= c= .
kh

3
Bài 58. Ta có:
1 1 x2 + y 2 + 2 z ( x + y ) + 2 z 2
= x 2 + y 2 + 2 (1 − xy ) = ( x − y ) + 2
2
+ =
( x + z) ( y + z) ( xy + xz + yz + z )
2 2 2 2

1 1
+ ( x − y) + 2 ≥ 2 . ( x − y=
) +2 3.
2 2
=
Suy ra P
4( x − y) 4( x − y)
2 2

1 1
Đẳng thức xảy ra khi và chỉ khi = ( x − y )2 ⇔ x − y = do x > y .
4( x − y)
2
2

181
Khám phá tư duy Kỹ thuật giải bất ĐT Bài toán Max – Min – Đặng Thành Nam

Khi đó z là nghiệm dương của phương trình: z 2 + z ( x + y ) + xy − 1 =0.


3
−x− y
−x − y + ( x + y ) 2
− 4 ( xy − 1) − x − y + ( x − y ) 2
+ 4 2
⇔z = = .
2 2 2
Vậy giá trị nhỏ nhất của P bằng 3 đạt tại
3
−x− y
1 2  3 
x −=y =,z , x + y < .
2 2  2
Nhận xét. Đây là một bài toán khá đơn giản dựa vào biến đổi chú ý điều kiện
( z + x )( z + y ) = 1 ngoài ra ta có thể đặt a = x + z , b = z + y ⇒ ab = 1 và khi đó

n
.v
1 1 1
=P + + .
4(a − b)
2 2
a b2

om
Đây là dạng toán quen thuộc được đề cập đến trong chủ đề tính đẳng cấp hoặc
sử dụng hàm số ta có 2 cách xử lý như sau:

Cách=1: P
ab
4( a − b)
2
ab ab
+ 2=
a
+ 2
b a
1 a b
+ + .
b b a
.c
4 − 2 + 
ok
b a
a b 1
Đặt t =+ , ( t > 2 ) đưa về khảo sát hàm số
= f (t ) + t hoặc đánh giá
bo

b a 4 (t − 2)
dựa vào bất đẳng thức AM-GM như sau:
et

1 1 1
=P = +t + (t − 2) + 2 ≥ 2 . ( t − 2=
) +2 3.
4 (t − 2) 4 (t − 2) 4 (t − 2)
vi

1 1 1
Cách 2: Thế b = đưa về khảo sát hàm số = + + a 2 ta tìm
ng

f (a) 2 2
a  1 a
4 a − 
 a
a

được Min của f(a) bằng 3 và có kết quả tương tự hai cách đề cập trên.
kh

Bài 59. Nhận xét. Hình thức bài toán này tương tự bài toán trước đó tuy nhiên hai
nhân tử sau không có dạng đối xứng ta cần khéo léo khai thác giả thiết tích
( b + c )( a + c ) =
1.

6 ( a + c ) + 2 ( b + c ) 
2 2

=   = 6 a + c 2 + 2 b + c 2 .
6 12
Ta có: +

( ) ( ) 
( b + c )2 ( a + c )2 ( a + c )( b + c ) 
2

Theo giả thiết ta có:


( a − b )2 =
( a + c ) − ( b + c )  ( a c ) + (b + c ) − 2 .
2 2 2
=+

182
Cty TNHH MTV DVVH Khang Việt

Đặt x = a + c, y = b + c ⇒ xy = 1
1 6 12 1 12 x2 6 x 4 + 12
=
và P + += + 2 +=
6 x2 + .
x2 + y 2 − 2
( x − 1)
2 2 2
y x 1
x2 + 2 − 2 x 2 x2
x
t 6t 2 + 12
Đến đây ta có thể khảo sát hàm số=
f (t ) + =
với t x 2 , ( t > 1) do
( t − 1)2 t

a>b⇒t = a+c> ( a + b )( a + c ) = 1 .
Hoặc biến đổi tương đương như sau:
 t  ( t − 2 )2 ( 6t 2 − 8t + 3)

n
6t 2 + 12
=P  + − 20  +=
20 + 20 ≥ 20 (với t > 1 ).
 ( t − 1)2 t ( t − 1)

.v
t  2

om
1
Vậy giá trị nhỏ nhất của P bằng 20 đạt tại t = 2 ⇔ a + c = 2, b + c = .
2
Bài 60. Vế trái bất đẳng thức được viết lại dưới dạng
bc ca ab zx xy yz
+ + + + +
.c
y z x a b c
ok
 bc   ca   ab   zx   xy   yz 
=  + c +  + a +  + b +  + z +  + x +  + y − 3
 y   z   x   a   b   c 
bo

c a b z x y a z b x  c y
= + + + + + −3=  + + + + + ≥3
y z x a b c  z a x b  y c
et

Bài 61. Bất đẳng thức đã cho tương đương với:


z( x + y)
vi

1
∑ y (x − y + z ) ≥ 0
ng

.
x−z x−z x+ y
⇔∑ ≥ 0 ⇔ ∑( + 1) ≥ 3 ⇔ ∑ ≥3
y+z y+z y+z
a

Đúng theo AM – GM.


kh

1
Bài 62. Ta có a3 + b3 ≥ ( a + b ) .
3
4
Do đó sử dụng bất đẳng thức AM – GM ta có
1 c3 
( )
3
3 c 9
3 a 3 + b3 + c 3 ≥ 3  ( a + b ) + +  ≥ 3 c 2 ( a + b ) .
4 
2 2 2 2

Vậy ta chỉ cần chứng minh
9  9 
c 2 ( a + b ) ≥ 2c ( ab + bc + ca ) ⇔  3 − 2  c 2 ( a + b ) ≥ 2abc .
2 2 
3
2 2

183
Khám phá tư duy Kỹ thuật giải bất ĐT Bài toán Max – Min – Đặng Thành Nam

9
Bất đẳng thức cuối luôn đúng do 3
− 2 > 1; c ≥ ab .
2 2
Bất đẳng thức được chứng minh.
Bài 63. Sử dụng bất đẳng thức AM – GM ta có
3
a+b+b+c+c+a
( a + b )( b + c )( c + a ) ≤ 
8
=
 ( a + b + c )3 .
 3  27
Do vậy ta chỉ cần chứng minh
9abc 12 ( ab + bc + ca )
3+ ≥
(a + b + c) 3
( a + b + c )2

n
⇔ ( a + b + c ) + 3abc ≥ 4 ( a + b + c )( ab + bc + ca )

.v
3

⇔ a3 + b3 + c3 + 3abc ≥ ab ( a + b ) + bc ( b + c ) + ca ( c + a )

om
Bất đẳng thức cuối đúng theo BĐT Schur bậc 3.
Bất đẳng thức được chứng minh. Đẳng thức xảy ra khi và chỉ khi a= b= c .
.c
Bài 64. Ta đặt
ok
x1
a1 =
1 + x1
bo

x2
a2 =
(1 + x1 )(1 + x1 + x2 )
... .
et

xn
an =
(1 + x1 + x2 +  + xn −1 )(1 + x1 + x2 +  + xn )
vi

1
ng

an +1 =
1 + x1 + x2 +  + xn
Khi đó ta có a1 + a2 +  + an + an +1 =
1.
a

1
kh

Ta cần chứng minh: a1a2  an +1 ≤ .


(n + 1) n +1
Bất đẳng thức cuối đúng theo AM – GM nên có đpcm.
Bài 65. Sử dụng bất đẳng thức AM – GM kết hợp vế trái là hàm đồng biến với tích
ab ta có

184
Cty TNHH MTV DVVH Khang Việt

abc c.ab
=
(
( a + b )( b + c )( c + a ) ( a + b ) c + c ( a + b ) + ab
2
)
2
a+b
c 
≤  2  .
 2 a+b 
2
( a + b ) c + c ( a + b ) 
+  
  2  
c (a + b)
=
( a + b + 2c )2

n
c (a + b) ( a + b )( a + b + 2c ) .

.v
Vậy ta chứng minh
( a + b + 2c ) 2
( 3a + 3b + 2c )2

om
⇔ c ( 3a + 3b + 2c ) ≤ ( a + b + 2c )
2 3

Sử dụng bất đẳng thức AM – GM ta có


1
c ( 3a + 3b + 2=
c) .8c. ( 3a + 3b + 2c )( 3a + 3b + 2c )
2
8
.c
ok
3
1  8c + 3a + 3b + 2c + 3a + 3b + 2c 
 = ( a + b + 2c )
3
≤ 
8 3 
bo

Bất đẳng thức được chứng minh. Đẳng thức xảy ra khi và chỉ khi a= b= c .
Bài 66. Ta có:
et

1 1 2 2 2
+ ≥ ≥
a ( b + c ) + 2c 2 b ( a + 4c ) 4
( ab + ac + 2c ) ( ab + 4bc )
2
2ab + ac + 4bc + 2c 2
vi

2 2 4 2 4 2
ng

= ≥ ≥
( a + 2c )( 2b + c ) a + 2b + 3c a + 2b + 2014c
a

4 2
Suy ra P ≥ ( a + 2b + 2014c ) . =
4 2.
a + 2b + 2014c
kh

Vậy giá trị nhỏ nhất của P bằng 4 2 xảy ra khi = c 0,= a 2b > 0 .
Bài 67. Không mất tính tổng quát ta giả sử a > b > c ta có:
2 2 2 5
P= + + + .
a −b b−c a −c ab + bc + ca
1 1 4
Sử dụng bất đẳng thức: + ≥ , ∀x, y > 0 ta được:
x y x+ y

185
Khám phá tư duy Kỹ thuật giải bất ĐT Bài toán Max – Min – Đặng Thành Nam

4 2 5  2 1 
P ≥ 2. + + = 5 + 
a−b+b−c a−c ab + bc + ca a−c ab + bc + ca 
5.2 2 20
≥ =
( ) (
4 a − c 2 ab + bc + ca
) 4 ( a − c )2 ( 4ab + 4bc + 4ca )
20 20 2 20 2
≥ = =
( a − c )2 + 4ab + 4bc + 4ca ( a + c )2 + 4b ( a + c ) ( a + c )( a + c + 4b )
2
20 2 20 6 40 6
= = ≥ = 10 6

n
(1 − b )(1 + 3b ) ( 3 − 3b )(1 + 3b ) 3 − 3b + 1 + 3b

.v
 2+ 6
a =

om
a − b = b − c 6
 
 2 1  1
Đẳng thức xảy ra khi và chỉ khi
=  = ⇔ b .
 a − c ab + bc + ca  3
3 − 3b =+
1 3b 
.c
c =
2− 6
 6
ok
1 1 1 1 1
Vậy giá trị nhỏ nhất của P bằng 10 6 đạt tại a = + ,b = ,c = − hoặc
3 6 3 3 6
bo

các hoán vị.


Nhận xét. Ta có thể xử lý bài toán trên bằng phương pháp hàm số xem chương 3.
et

Bài 68. Không mất tính tổng quát giả sử z = min { x, y, z} ta có


x 2 + y 2 + z 2= 2 ( xy + yz + zx )
vi

⇔ ( x + y ) + z 2 − 2 z ( x + y ) = 4 xy ⇔ ( x + y − z ) = 4 xy
2 2
ng

Sử dụng bất đẳng thức AM – GM ta có


x+ y−z x+ y−z
a

+ + 2z z( x + y − z)
2
x+ y+z 2 2
= ≥3 =
kh

3 2 xyz .
3 3 2
 x + y − z = 4z  x + y = 5z = x 4=
z, y z
Đẳng thức xảy ra khi và chỉ khi  ⇔ ⇔  .
 x + y − z =2 xy  xy = 4 z 2 =
 x z=
, y 4z
xy + yz + zx 1
Cách 2: Theo giả thiết ta có = .
( x + y + z) 2 4
xyz 1
Ta cần chứng minh ≤ .
( x + y + z)3 54

186
Cty TNHH MTV DVVH Khang Việt

a + b + c = 1
x y z 
Đặt a
= = ,b = ,c ⇒ 1.
x+ y+z x+ y+z x+ y+z ab + bc + ca =
4
1
Ta cần chứng minh=
P abc ≤ (xem chủ đề kỹ thuật sử dụng tính đẳng cấp).
54
Bài 69. Theo giả thiết kết hợp sử dụng bất đẳng thức AM – GM ta có
 2(a + b + c) 
3
8 = ( a + b )( b + c )( c + a ) ≤  
 3 
⇒a+b+c≥3

n
Bất đẳng thức tương đương với:

.v
( a + b + c )27 ≥ 326 ( a3 + b3 + c3 )

om
⇔ (a + b + c) ≥ 326 ( a + b + c ) − 3 ( a + b )( b + c )( c + a ) 
27 3
 
⇔ (a + b + c) ≥ 326 ( a + b + c ) − 24 
27 3
  .c x9
Đặt x = ( a + b + c )3 ⇒ x ≥ 27 ta cần chứng minh ≥ 326 .
ok
x − 24
x9
Xét hàm số f ( x) = với x ≥ 27 ta có
bo

x − 24
8 x8 ( x − 27 )
= ≥ 0, ∀x ≥ 27 .
et

f '( x)
( x − 24 )2
vi

Do đó f(x) là hàm đồng biến trên [ 27;+∞ ) . Vì vậy f ( x) ≥ f (27) =


326 .
Bất đẳng thức được chứng minh. Đẳng thức xảy ra khi và chỉ khi a= b= c= 1 .
ng

Bài 70. Đặt x = b + c − a, y = c + a − b, z = a + b − c ⇒ xyz = 1 và


y+z z+x x+ y
a

=a = ,b = ,c .
2 2 2
kh

Bất đẳng thức trở thành


x + y + z 5 x 2 + y 2 + z 2 + xy + yz + zx

3 6
 x + y + z  ( x + y + z ) − ( xy + yz + zx )
5 2
⇔  ≥
 3  6
 xy + yz + zx ≥ 3 3 x 2 y 2 z 2 =
3
Chú ý điều kiện xyz = 1 ⇒  .
 x + y + z ≥ 3 3 xyz = 3

187
Khám phá tư duy Kỹ thuật giải bất ĐT Bài toán Max – Min – Đặng Thành Nam

Suy ra ( x + y + z ) − xy − yz − zx ≤ ( x + y + z ) − 3 .
2 2

 x + y + z  ( x + y + z) − 3
5 2
Vậy ta chứng minh   ≥ .
 3  6
x+ y+z
Đặt t
= ≥ 1 ta cần chứng minh
3
3t 2 1   1  1 
t5 ≥ − ⇔ ( t − 1) t 4 + t  t 2 −  +  t 2 −   ≥ 0 .
2 2   2  2 
Bất đẳng thức được chứng minh. Đẳng thức xảy ra khi và chỉ khi a= b= c= 1 .
Bài 71. Giả sử a = min {a, b, c} .

n
.v
Đặt b =a + u, c =a + v, ( u , v ≥ 0 ) bất đẳng thức trở thành

om
3 3
(3 − )(u 2 − uv + v 2 )a + u 3 − 3 uv 2 + v3 ≥ 0 .
3
4 4
Bất đẳng thức cuối luôn đúng do

u 3 + v3 = u 3 +
v3 v3
+ ≥ 33
u 3v 6 3
= 3 uv 2 .
.c
ok
2 2 4 4

Bài 72. Đặt


= a 1= / y; c 1 / z bất đẳng thức trở thành:
/ x; b 1=
bo

a2 + 3 b2 + 3 c2 + 3 3
+ + ≥ .
et

ac + 3b ba + 3c cb + 3a 2
Sử dụng bất đẳng thức C –S ta có
vi

=
a2 + 3 4 a2 + 3
=
( ) 4 a2 + 3 (≥
)
a+3
ng

ac + 3b 2  ac + b ( a + b + c )  2 ( b + a )( b + c ) 2 ( b + a )( b + c )

b2 + 3 b+3 c2 + 3 c+3
a

≥ ; ≥
ba + 3c 2 ( c + a )( c + b ) cb + 3a 2 ( a + b )( a + c )
kh

Vậy ta chỉ cần chứng minh


a+3 b+3 c+3 3
+ + ≥
2 ( b + a )( b + c ) 2 ( c + a )( c + b ) 2 ( a + b )( a + c ) 2
⇔ ( a + 3)( a + c ) + ( b + 3)( a + b ) + ( c + 3)( b + c ) ≥ 3 ( a + b )( b + c )( c + a )
⇔ a 2 + b 2 + c 2 + ab + bc + ca + 18 ≥ 3 ( a + b )( b + c )( c + a )
Theo bất đẳng thức AM – GM ta có

188
Cty TNHH MTV DVVH Khang Việt

a 2 + b 2 + c 2 + ab + bc + ca + 18 = ( a + b + c ) − ( ab + bc + ca ) + 18
2

1
≥ (a + b + c) − ( a + b + c )2 + 18 =24
2
3
3
a+b+b+c+c+a
3 ( a + b )( b + c )( c + a ) ≤ 3   =
24
 3 
Do đó bất đẳng thức cuối đúng và ta có đpcm. Đẳng thức xảy ra khi và chỉ khi
x= y= z= 1 .
Bài 73. Đặt x = a + b − c, y= b + c − a, z= c+ a −b .

n
 x2 + y 2 + z 2 = a + b + c = 3

.v

⇒  x 2 y 2 + y 2 z 2 + z 2 x=
2
∑ ( a + b − c )( b + c − a )

( )

om
= ∑ b 2 − ( a − c ) = 4 ( ab + bc + ca ) − 9
2


1 1 1 36
Ta cần chứng minh + + ≥ .c
x y z 9 + x y + y 2 z 2 + z 2 x2
2 2

( )
⇔ x 2 y 2 + y 2 z 2 + z 2 x 2 + 9 ( xy + yz + zx ) ≥ 36 xyz .
ok
Sử dụng bất đẳng thức AM – GM ta có
bo

x 2 y 2 + y 2 z 2 + z 2 x 2 + 9 ≥ 12 3 xyz

xy + yz + zx ≥ 3 3 x 2 y 2 z 2
et

Nhân theo vế 2 bất đẳng thức trên ta có ngay điều phải chứng minh.
Đẳng thức xảy ra khi và chỉ khi a= b= c= 1 .
vi

Bài 74. Đặt a = x + y, b = y + z , c = z + x ⇒ a, b, c ≥ 0; ab + bc + ca > 0 .


ng

a b c
Khi đó P = + + .
b+c c+a a+b
a

a 2a b 2b c 2c
≥ ≥ ≥
kh

Tìm giá trị nhỏ nhất: ; ; .


b+c a+b+c c+a a+b+c a+b a+b+c
Cộng theo vế ba bất đẳng thức trên ta được P ≥ 2 .
Dấu bằng đạt tại x > 0; y ==z 0 hoặc các hoán vị.
Bài 75. Trước hết ta chứng minh

(2a 2 + bc)(2b 2 + ca )(2c 2 + ab) ≤


( a + 2b )2 ( b + 2c )2 ( c + 2a )2 .
8
3 2
Thật vậy ta có (2a + bc)(2b 2 + ca )(2c 2 + ab) − 9abc  =
2 

189
Khám phá tư duy Kỹ thuật giải bất ĐT Bài toán Max – Min – Đặng Thành Nam

( ) ( )
(
= ( a + 2b ) 2c 2 + ab + ( b + 2c ) 2a 2 + bc + ( c + 2a ) 2b 2 + ca )
≥ 33 ( a + 2b ) ( 2c + ab ) ( b + 2c ) ( 2a + bc ) ( c + 2a ) ( 2b
2 2 2
+ ca )

Mặt khác (2a 2 + bc)(2b 2 + ca )(2c 2 + ab) − 9abc ≤ (2a 2 + bc)(2b 2 + ca )(2c 2 + ab) .
Suy ra
3 ( a + 2b ) ( 2c 2 + ab ) ( b + 2c ) ( 2a 2 + bc ) ( c + 2a ) ( 2b2 + ca ) ≤
(2a 2 + bc)(2b 2 + ca )(2c 2 + ab)

2

n
⇒ (2a 2 + bc)(2b 2 + ca )(2c 2 + ab) ≤
( a + 2b )2 ( b + 2c )2 ( c + 2a )2

.v
8
Sử dụng bất đẳng thức AM – GM ta có

om
1
( a + 2b )( b + 2c )( c + 2a ) = .( a + 2b )( 4b + 8c )( c + 2a )
4
1  3 ( a + 2b + 3c ) 
≤   =
16
.c 3

4 
ok
3
Từ đó suy ra điều phải chứng minh. Đẳng thức xảy ra khi và chỉ khi
=
a 2,=b 1,=
c 0.
bo

giả sử c min {a, b, c} ⇒ c ≤ 1 .


Bài 76. Không mất tính tổng quát =
Sử dụng bất đẳng thức AM – GM ta có
et

a 2 + b2 3 − c 2 − c 2 + 2c + 3
c + ab ≤ c + =c+ = ;
vi

2 2 3
( a + b + bc + ca )2 ( a + b )2 ( c + 1)2
ng

( a + bc )( b + ca ) ≤ =
4 4
( c + 1)( a
2 2
+b 2
)=
(3 − c )( c + 1)
2 2
a


kh

2 2
2 ( 2 − a )( 2 − b ) =8 − 4 ( a + b ) + 2ab
=4 − a 2 − b 2 + ( a + b − 2 ) ≥ 4 − a 2 − b 2 =c 2 + 1
2

Do vậy ta chỉ cần chứng minh

( )( ) (
8 ( 2 − c ) ≥ 3 − c 2 3 + 2c − c 2 ⇔ 7 − c 2 ( c − 1) ≥ 0 . ) 2

Bất đẳng thức cuối đúng và ta có đpcm. Đẳng thức xảy ra khi và chỉ khi
a= b= c= 1 .
Bài 79. Sử dụng bất đẳng thức C – S và AM – GM ta có

190
Cty TNHH MTV DVVH Khang Việt

(a 2
)
+ b + c (1 + b + c ) ≥ ( a + b + c ) =9
2

a2 a b + c + 1 a 3 ( b + c + 1) a ( b + c + 1 + 3)
⇒ ≤ = ≤
a +b+c
2 3 3 3 6 3
b2 b ( a + c + 1 + 3) c2 c ( b + a + 1 + 3)
Tương tự ta có ≤ ; ≤ .
b +c+a 2
6 3 c +a+b 6 3 2

Gọi P là biểu thức vế trái và cộng theo vế 3 bất đẳng thức trên và chú ý
1
ab + bc + ca ≤ ( a + b + c ) =
2
3.
3

n
4 ( a + b + c ) + 2 ( ab + bc + ca )
4.3 + 2.3
Ta có P ≤ = 3. ≤

.v
6 3 6 3
Bất đẳng thức được chứng minh. Đẳng thức xảy ra khi và chỉ khi a= b= c= 1 .

om
Bài 80. Theo giả thiết ta có ( ab + bc + ca ) ≥ 3abc ( a + b + c )= 3 ( ab + bc + ca )
2

⇒ ab + bc + ca ≥ 3; ab ≤ ac ≤ bc ⇒ bc ≥ 1
.c
Do đó
( a − 1)2 + ( c − b )( bc − 1) ≥ 0
ok
a bc
1 1 1
⇔ a+c+
+ −2−b− ≥0
bo

a c b
 1  1
⇔ a + c +a +b + c −  − 2−b − ≥ 0
 b b
et

1 1
⇔ a + c −1− ≥ 0 ⇔ b ≥
a + c −1
vi

b
Bất đẳng thức được chứng minh. Đẳng thức xảy ra khi và chỉ khi a= b= c= 1 .
ng

=
Bài 82. Không mất tính tổng quát giả sử c min {a, b, c} ⇒ c 2 − bc ≤ 0; c 2 − ac ≤ 0 .
1 1
a

Suy ra ≥ .
(b 2
− bc + c 2
)( c 2
− ca + a 2
) ab
kh

1 1
+ =
(a 2
− ab + b 2
)(b 2
− bc + c 2
) (c 2
− ca + a 2
)( a 2
− ab + b 2
)
1 1 1  1 1 1
=  + ≥  + 
 
a 2 − ab + b 2  b 2 − bc + c 2 c 2 − ca + a 2  a 2 − ab + b 2  a b 
Sử dụng bất đẳng thức AM – GM ta có

191
Khám phá tư duy Kỹ thuật giải bất ĐT Bài toán Max – Min – Đặng Thành Nam

1 ab + bc + ca 2 ab + bc + ca

(a ) ( ab + bc + ca ) a + c ( a + b ) + b2
2
a − ab + b
2 2 2
− ab + b 2

2 ab + bc + ca 2 ab + bc + ca ab
= = ≥
( a + b )( a + b + c ) − 2ab 2 ( a + b − ab ) a + b − ab
Gọi P là biểu thức vế trái ta có
ab  1 1  1 1 1 ab
P≥  + + = + +
a + b − ab  a b  ab ab ab a + b − ab
1 1 ab 3 3
≥ 33 = ≥ ≥3

n
. .
ab ab a + b − ab 3 ab ( a + b − ab ) a+b
2

.v
3
 
 2 

om
Đẳng thức xảy ra khi và chỉ khi a= b= 1, c= 0 và các hoán vị.
Bài 83. Không mất tính tổng quát giả sử x ≥ y ≥ z khi đó ta có
xy + yz + zx x + z xy + yz + zx y + z .c
(i) ≥ ; ≥ .
y + yz + z
2 2 y + z x 2 + zx + z 2 x + z
ok
xy + yz + zx ( x + z )( y + z )
(ii) ≥ .
x + xy + y
2 2
( x + z ) + ( x + z )( y + z ) + ( y + z )2
2
bo

Các bất đẳng thức (i) và (ii) xem chứng minh trong chương 1.
Áp dụng vào bài toán ta có
et

P≥
x+z y+z ( x + z )( y + z )
+ + .
y+z x+z ( x + z ) + ( x + z )( y + z ) + ( y + z )2
2
vi

x+z y+z
ng

1
Đặt t = + ⇒ t ≥ 2; P ≥ f (t ) = t + .
y+z x+z t2 −1
a

1
Xét hàm số f (t ) = t + trên nửa khoảng [ 2;+∞ ) ta có
kh

t2 −1

(t − 1) − t > t − t
2 3
3
t
f '(t ) = 1 − = > 0, ∀t ≥ 2 .
(t ) (t − 1) (t − 1)
3 3 3
2
−1 2 2

1
Vì vậy f(t) đồng biến trên nửa khoảng [ 2;+∞ ) suy ra f (t ) ≥ f (2) =
2+ .
3
1 1
Với x= y= 1, z= 0 thì P bằng 2 + . Vậy giá trị nhỏ nhất của P bằng 2 + .
3 3

192
Cty TNHH MTV DVVH Khang Việt

Bài 84. Sử dụng bất đẳng thức AM – GM ta có


3
 a+c 
(a + b + ) a+c
3 27
2(a + c) =  a + b + +  ≥ ( a + b )( a + c )
 2 2  2
1 2 1
⇒ ≤ .
27 ( a + b )( a + c )
(a + b + )
3
2(a + c)
Xây dựng tương tự cho hai biểu thức còn lại và đưa về chứng minh
1 1 1
+ + ≤ 12
( a + b )( a + c ) ( b + c )( b + a ) ( c + a )( c + b )

n
2(a + b + c)
⇔ ≤ 12 ⇔ 6 ( a + b )( b + c )( c + a ) ≥ a + b + c

.v
( a + b )( b + c )( c + a )
Chú ý sử dụng bất đẳng thức quen thuộc

om
9 ( a + b )( b + c )( c + a ) ≥ 8 ( a + b + c )( ab + bc + ca ) .
3
Bài toán đưa về chứng minh ab + bc + ca ≥ ⇔ 16 ( ab + bc + ca ) ≥ 3 .
.c
16
Bất đẳng thức cuối đúng theo giả thiết vì
ok
16 3
ab + bc + ca ≤ 16abc ( a + b + c ) ≤ ( ab + bc + ca ) ⇒ ab + bc + ca ≥ .
2
3 16
bo

1
Bài toán được chứng minh. Đẳng thức xảy ra khi và chỉ khi a= b= c= .
4
et

a − bc 2bc 2bc 2bc


Bài 85. Ta có =
1− = 1− =
1− .
a + bc a + bc a ( a + b + c ) + bc ( a + b )( a + c )
vi

Bài toán đưa về chứng minh


ng

bc ca ab 3
+ + ≥
( a + b )( a + c ) ( b + c )( b + a ) ( c + a )( c + b ) 4
a

⇔ ab + bc + ca ≥ 9abc ⇔ ( a + b + c )( ab + bc + ca ) ≥ 9abc
kh

Bất đẳng thức cuối đúng theo AM – GM.


1
Bài toán được chứng minh đẳng thức xảy ra khi và chỉ khi a= b= c= .
3
Bài 86. Chú ý điều kiện bài toán ta có
1 1 1
= =
ab + 2c + 2c ab + 2c + 2c ( a + b + c )
2 2
( 2c + a )( 2c + b )
ab ab ab
= ≥ =
( 2bc + ab )( 2ac + ab )  2bc + 2ca + 2ab 2 ( ab + bc + ca )2
 
 2 

193
Khám phá tư duy Kỹ thuật giải bất ĐT Bài toán Max – Min – Đặng Thành Nam

1 bc 1 ca
Tương tự ta có ≥ ; ≥ .
bc + 2a + 2a
2
( ab + bc + ca ) 2
ca + 2b + 2b
2
( ab + bc + ca )2
Cộng lại theo vế ba bất đẳng thức trên ta có đpcm. Đẳng thức xảy ra khi và chỉ
1
khi a= b= c=
3
Bài 87. Sử dụng bất đẳng thức AM – GM ta có
1 1 1 1 1 1 a+b+c
2
+ 2+ 2≥ + + =
a b c ab bc ca abc
9 27
= ≥
abc ( a + b + c ) ( ab + bc + ca )2

n
.v
Vậy ta chứng minh
27
( )
≥ a 2 + b 2 + c 2 ⇔ a 2 + b 2 + c 2 ( ab + bc + ca ) ≤ 27 .

om
2

( ab + bc + ca ) 2

Bất đẳng thức cuối đúng theo AM – GM vì


.c
 a 2 + b 2 + c 2 + 2 ( ab + bc + ca ) 
3

( 2 2 2
)
a + b + c ( ab + bc + ca ) ≤ 
2
 =27 .
ok
 3 
Bất đẳng thức được chứng minh. Đẳng thức xảy ra khi và chỉ khi a= b= c= 1 .
bo

1− c 1− a 1− b 9
Bài 88. Viết lại bất đẳng thức dưới dạng + + ≥
ab + c bc + a ca + b 2
et

a+b b+c a+c 9


⇔ + + ≥ .
( c + a )( c + b ) ( a + b )( a + c ) ( b + a )( b + c ) 2
vi

Sử dụng bất đẳng thức AM – GM cho vế trái đưa về chứng minh


ng

8
( a + b )( b + c )( c + a ) ≤ .
27
Bất đẳng thức cuối đúng theo AM – GM.
a

Bài 89. Thay 1 bởi điều kiện vào bất đẳng thức đã cho và đưa về chứng minh
kh

4ab 4bc 4ca


+ + +a+b+c≥4.
a+b b+c c+a
Sử dụng bất đẳng thức C –S ta có
ab
+
bc
+
ca

( ab + bc + ca ) ≥
2
1
.
a + b b + c c + a ab ( a + b ) + bc ( b + c ) + ca ( c + a ) a + b + c
4
Ta chỉ cần chứng minh + a +b+c ≥ 4.
a+b+c

194
Cty TNHH MTV DVVH Khang Việt

Bất đẳng thức cuối đúng theo AM – GM. Đẳng thức xảy ra khi và chỉ khi
a= b= 1, c= 0 hoặc các hoán vị.
9 − a 2 − b2 − c2
Bài 90. Sử dụng ab + bc + ca = đưa về chứng minh bất đẳng thức
2
a 2 + b2 + c2 + 2 ( a + b + c ≥9. )
Bất đẳng thức trên là tổng của các bất đẳng thức
a 2 + a + a ≥ 3a
b 2 + b + b ≥ 3c

n
c 2 + c + c ≥ 3c

.v
Bất đẳng thức được chứng minh. Đẳng thức xảy ra khi và chỉ khi a= b= c= 1 .
Bài 91. Bất đẳng thức đã cho tương đương với

om
ab + bc + ca ab + bc + ca ab + bc + ca 9
+ + ≥
(a + b)b (b + c ) c (c + a) a 2
a b c a + b b + c c + a 15 .c
⇔ + + + + + ≥
a+b b+c c+a a b c 2
ok
Bất đẳng thức cuối là tổng của hai bất đẳng thức
a b c 1a+b b+c c+a
+ + +  + + ≥3
a +b b+c c+ a 4 a
bo

b c 
3 a +b b+c c+ a  9
 + + ≥
4 a c  2
et

b
Bài 92. Đặt x = b + c − a; y = c + a − b; z = a + b − c ⇒ x2 + y 2 + z 2 = 3 .
vi

Bài toán đưa về chứng minh


ng

1 1 1 36
+ + ≥
x y z 9 + x y + y 2 z 2 + z 2 x2
2 2

( )
a

⇔ x 2 y 2 + y 2 z 2 + z 2 x 2 + 9 ( xy + yz + zx ) ≥ 36 xyz
kh

Bất đẳng thức trên là tích của hai bất đẳng thức
xy + yz + zx ≥ 3 3 x 2 y 2 z 2 ; x 2 y 2 + y 2 z 2 + z 2 x 2 + 9 ≥ 12 6 xyz .
Bất đẳng thức được chứng minh. Đẳng thức xảy ra khi và chỉ khi a= b= c= 1 .
Bài 93. Chú ý đẳng thức:
(a 2
) ( ) (
+ bc ( b + c )= b c 2 + a 2 + c a 2 + b 2 ; )
(b 2
+ ca ) ( c + a )= a ( b 2
+ c ) + c(a
2 2
+ b );
2

(c2
+ ab ) ( a + b =) b(a 2
+ c ) + a (b
2 2
+c )
2

195
Khám phá tư duy Kỹ thuật giải bất ĐT Bài toán Max – Min – Đặng Thành Nam

Bài toán đưa về chứng minh với ba số dương x,y,z ta có


x+ y y+z z+x
+ + ≥3 2.
z x y
Bất đẳng thức này đúng theo AM – GM
x+ y
+
y+z
+
z+x
≥ 36
( x + y )( y + z )( z + x ) ≥ 36 2 xy .2 yz .2 zx =
3 2.
z x y xyz xyz
Bài 94. Tương tự bài toán trên bất đẳng thức đưa về chứng minh với ba số không
x y z
âm x,y,z ta có + + ≥ 2.
y+z z+x x+ y

n
Bất đẳng thức cuối đúng(xem chứng minh trong bài tập mẫu).

.v
Đẳng thức xảy ra khi và chỉ khi=
a b=
, c 0 hoặc các hoán vị.
Bài 95. Ta có c 2 + 2ab = a 2 + b 2 + 2c 2 − 2bc − 2ca = ( a − c ) + ( b − c ) .

om
2 2

Do đó
a −b
=
( a − b )2 .
( c − a )2 + ( c − b )2
c 2 + 2ab
.c
ok
x y z
Bài toán đưa về chứng minh + + ≥ 2.
y+z z+x x+ y

( a b ) , y =−
( b c ) , z =−
( c a ) xem bài tập mẫu.
bo

2 2 2
Với x =−
Bài 96. Bất đẳng thức tương đương với 3abc + ab + bc + ca ≥ 3 + a + b + c .
et

a b c (a + b + c)
2
Bất đẳng thức luôn đúng do a + b + c ≥ + + ≥
b c a ab + bc + ca
vi

⇒ ab + bc + ca ≥ a + b + c
ng

a b c 1  2a b  1  2b c  1  2c a  a + b + c
a+b+c≥ + +=  + +  + +  + ≥ 3 .
b c a 3 b c  3 c a  3 a b  abc
⇒ abc ≥ 1
a

Bất đẳng thức được chứng minh. Đẳng thức xảy ra khi và chỉ khi a= b= c= 1 .
kh

Bài 97. Sử dụng bất đẳng thức AM – GM ta có

=
x3 + 1
=
(
x x 2 + yz ) (
x x 2 + yz ) xy + yz + zx

x4 + y + z x 4 + xyz ( y + z ) (x 2
( y + z ) + 1) ( x3 + yz ( y + z ) )


(
2 x x 2 + yz ) xy + yz + zx2 x xy + yz + zx
=
x ( y + z ) + 1 + x + yz ( y + z )
2 3 x+ y+z
Tương tự cho 2 căn thức còn lại rồi cộng lại theo vế ta có điều phải chứng minh.
Bất đẳng thức được chứng minh. Đẳng thức xảy ra khi và chỉ khi x= y= z= 1 .
196
Cty TNHH MTV DVVH Khang Việt

Bài 98. Sử dụng bất đẳng thức AM – GM ta có


2
 2a  1 3 3a
3
  = ≥
b+c b+c
= .
b+c 3
b+c b+c + + 1 a+b+c
. .1
2a 2a 2a 2a
2 2
 2b  3b  2c  3c
Tương tự ta có 3
 ≥ ;3   ≥ .
c+a a+b+c a+b a+b+c
Cộng theo vế ba bất đẳng thức trên ta có đpcm. Đẳng thức xảy ra khi và chỉ khi
a= b= c .
Chú ý. Ngoài ra ta có thể chứng minh bằng bất đẳng thức Bernoulli(xem chủ đề sau).

n
Bài 99. Chú ý x5 + y 5 ≥ x 2 y 2 ( x + y )

.v
xy xy 1 z
⇒ ≤ = = .
( x + y ) + xy xy ( x + y ) + 1 x + y + z

om
x + y + xy
5 5 2 2
x y
yz x zx y
Tương tư ta có ≤ ; 5 ≤ .
y + z + yz
5 5 x + y + z z + x + zx x + y + z
5
.c
Cộng theo vế ba bất đẳng thức trên ta có đpcm. Đẳng thức xảy ra khi và chỉ khi
x= y= z= 1 .
ok
Bài 100. Sử dụng bất đẳng thức AM – GM ta có
1 ab ab ab
bo

=1− ≥1− =1−


1 + ab 1 + ab 2 ab 2
1 bc bc bc
et

=1− ≥1− =1−


1 + bc 1 + bc 2 bc 2
vi

1 ca ca ca
=1− ≥1− =1−
1 + ca 1 + ca 2 ca 2
ng

Cộng theo vế ba bất đẳng thức trên ta được

( )
2
ab + bc + ca 15 a+ b+ c
a

1 1 1
+ + ≥ 3− = − .
1 + ab 1 + bc 1 + ca
kh

2 4 4

( )
2
15 a+ b+ c 9
Vậy ta chỉ cần chứng minh − ≥ .
4 4 2 ( a+ b+ c )
Đặt t= a + b + c, ( )
3 < t ≤ 3 đưa về chứng minh

15 t 2
− ≥
4 4
9
2t
(
⇔ t 3 − 15t + 18 ≤ 0 ⇔ ( t − 3) t 2 + 3t − 6 ≤ 0 . )
Bất đẳng thức cuối đúng và ta có đpcm. Đẳng thức xảy ra khi và chỉ khi
a= b= c= 1.

197
Khám phá tư duy Kỹ thuật giải bất ĐT Bài toán Max – Min – Đặng Thành Nam

KỸ THUẬT GHÉP CẶP TRONG


CHỦ ĐỀ 2:
CHỨNG MINH BẤT ĐẲNG THỨC AM – GM
A. NỘI DUNG PHƯƠNG PHÁP
1. Ghép cặp trực tiếp 2 hoặc 3 hạng tử ở một vế của bất đẳng thức
Ta cần chứng minh bất đẳng thức
A+ B +C ≥ X +Y + Z .
Nếu X ≤ AB , Y ≤ BC , Z ≤ CA .
Ta thực hiện ghép cặp sử dụng bất đẳng thức AM- GM cho 2 số dương như sau:

n
A + B ≥ 2 AB ≥ 2 X

.v
B + C ≥ 2 BC ≥ 2Y
C + A ≥ 2 CA ≥ 2Z

om
Cộng theo vế 3 bất đẳng thức trên ta có ngay điều phải chứng minh.
2. Ghép cặp kết hợp 2 giữa hoặc 3 nhân tử ở cả 2 vế của bất đẳng thức
.c
Ta cần chứng minh bất đẳng thức A + B + C ≥ X + Y + Z .
Ta có thể thực hiện đánh giá như sau
ok
A + X ≥ 2 AX ≥ 2Y
B + Y ≥ 2 BY ≥ 2 Z
bo

C + Z ≥ 2 CZ ≥ 2 X
Cộng lại theo vế 3 bất đẳng thức trên ta có ngay điều phải chứng minh.
et

3. Biến đổi điều kiện hoặc bất đẳng thức trước khi ghép cặp
Áp dụng với các bất đẳng thức có điều kiện chưa tương đồng với bất đẳng thức
vi

cần chứng minh. Thông thường ta sử dụng phép bình phương 2 vế, quy đồng 2
vế bất đẳng thức(xem bài tập mẫu).
ng

B. BÀI TẬP MẪU


a

Bài 1. Cho x,y,z là các số thực dương thoả mãn điều kiện 2 xy + xz =
1.
kh

3 yz 4 zx 5 xy
Chứng minh rằng + + ≥4.
x y z
Lời giải
Gọi P là biểu thức vế trái của bất đẳng thức.
Ta viết lại P dưới dạng
 yz zx   yz xy   xy zx 
P =  +  + 2  +  + 3 +  .
 x y  x z   z y
Sử dụng bất đẳng thức AM – GM ta có

198
Cty TNHH MTV DVVH Khang Việt

yz zx yz zx
+ ≥2 . = 2 z;
x y x y
yz xy yz xy
+ ≥2 . = 2 y;
x z x z
xy zx xy zx
+ ≥2 . = 2 x.
z y z y
Suy ra P ≥ 2 z + 4 y + 6 x = 2 ( x + z ) + 4 ( x + y ) ≥ 4 zx + 8 xy = 4 .
 x= y= z 1
Đẳng thức xảy ra khi và chỉ khi  ⇔ x=y=z= .

n
2 xy + zx =1 3

.v
Nhận xét. Để có phân tích trên ta sử dụng đồng nhất thức như sau:
3 yz 4 zx 5 xy  yz zx   yz xy   xy zx 

om
+ + = a +  + b +  + c + 
x y z  x y  x z   z y
a=+b 3 = a 1
   yz zx   yz xy   xy zx 
.c
⇔ b + c = 5 ⇔ b = 2 ⇒ P =  +  + 2  +  + 3  + 
c= c 3  x y  x z   z y
 +a 4 = 
ok
Bài 2. Cho x,y,z là các số thực dương thoả mãn điều kiện x 2 + y 2 + z 2 =
3.
bo

xy yz zx
Chứng minh rằng + + ≥ 3.
z x y
et

Lời giải
Phân tích tìm lời giải:
vi

xy yz
Nếu sử dụng ghép cặp chẳng hạn + ≥ 2 y nhưng điều kiện cho dưới dạng
z x
ng

bậc 2 do vậy ta không thể ghép cặp sử dụng trực tiếp được và cần tạo ra các
nhân tử có chứa x 2 , y 2 , z 2 như vậy lựa chọn phép bình phương 2 vế của bất
a

đẳng thức.
kh

Lời giải chi tiết:


Bình phương 2 vế bất đẳng thức cần chứng minh được viết dưới dạng
x2 y 2
z2
+
y2 z2
x2
+
z 2 x2
y2
( )
+ 2 x2 + y 2 + z 2 ≥ 9

x2 y 2 y2 z2 z 2 x2
⇔ + + ≥3
z2 x2 y2
Sử dụng bất đẳng thức AM –GM cho 2 số dương ta được

199
Khám phá tư duy Kỹ thuật giải bất ĐT Bài toán Max – Min – Đặng Thành Nam

x2 y 2 y2 z2 x2 y 2 y 2 z 2
+ ≥2 . 2 = 2 y2 .
z2 x2 z2 x
y2 z2 z 2 x2 x2 y 2 z 2 x2
Tương tự ta có + ≥ 2z2 ; + ≥ 2 x2 .
x2 y2 z2 y2
Cộng theo vế 3 bất đẳng thức trên ta có ngay điều phải chứng minh. Đẳng thức
xảy ra khi và chỉ khi x= y= z= 1 .
Bài 3. Chứng minh rẳng với mọi a,b,c dương ta luôn có
1 1 1 9
+ + ≥ .
b ( a + b ) c ( c + b ) a ( a + c ) 2 ( ab + bc + ca )

n
Lời giải

.v
Bất đẳng thức cần chứng minh tương đương với
ab + bc + ca ab + bc + ca ab + bc + ca 9

om
+ + ≥
b (a + b) c (c + b) a (a + c) 2
c b a a b c 9
⇔ + + + + + ≥ .c
b a c a+b b+c c+a 2
b+c a+b c+a a b c 15
⇔ + + + + + ≥
ok
b a c a+b b+c c+a 2
Sử dụng bất đẳng thức AM – GM cho 2 số dương ta có
bo

b+c b b+c b
+ ≥2 . = 1.
4b b+c 4b b + c
et

a+b a c+a c
Tương tự ta có: + ≥ 1; + ≥ 1.
4a a+b 4c c+a
vi

Cộng theo vế 3 bất đẳng thức trên ta được


ng

1a+b b+c c+a a b c


 + + + + + ≥ 3.
4 a b c  a+b b+c c+a
Mặt khác
a

3 a +b b+c c+ a  3 b c a 3 b c a 9
kh

 + + = 3 + + +  ≥  3 + 33 . . =
 .
4 a b c  4 a b c 4 a b c 2
Cộng theo vế 2 bất đẳng thức trên ta có ngay điều phải chứng minh. Đẳng thức
xảy ra khi và chỉ khi a= b= c .
Bài 4. Chứng minh rằng với mọi số thực dương x,y,z ta có
 x  y  z 2( x + y + z )
1 + 1 + 1 +  ≥ 2 + .
 y  z  x 3 xyz

Lời giải
Khai triển vế trái viết lại bất đẳng thức đã cho dưới dạng

200
Cty TNHH MTV DVVH Khang Việt

x y z x+ y+z x2 3 y 2 3 z 2
+ + ≥ = 3 + + .
y z x 3 xyz yz zx xy

x x y x2
Sử dụng bất đẳng thức cho 3 số dương ta có + + ≥ 33 .
y y z yz

y y z y2 z z x z2
Tương tự ta có: + + ≥ 33 ; + + ≥ 33 .
z z x zx x x y xy
Cộng theo vế 3 bất đẳng thức trên ta có điều phải chứng minh. Đẳng thức xảy ra
khi và chỉ khi x= y= z .

n
Bài 5. Chứng minh rằng với mọi số thực dương x,y,z ta có

.v
x3 y3 y3  x2 y 2 z 2 
z3 z3 x3
+ + ≥ 2 +
 + +
+ . +
y 3 x3 z 3 y 3 x3 z 3 

om
 yz zx xy 
Lời giải
Sử dụng bất đẳng thức AM –GM cho 3 số dương ta có
x3
+
x3
+ 1 ≥ 33
x3 x3
. .1 =3
x2
.c
ok
y3 z3 y3 z3 yz

y3 y3 y3 y3 y2
+ + 1 ≥ 33 =
bo

. .1 3
z3 x3 z 3 x3 zx
z3 z3 z3 z3 3z 2
+ + 1 ≥ 33 =
et

. .1
x3 y3 x3 y 3 xy
Cộng theo vế 3 bất đẳng thức trên ta có điều phải chứng minh. Đẳng thức xảy ra
vi

khi và chỉ khi x= y= z .


ng

Bài 6. Cho x,y,z là các số thực không âm.


Chứng minh rằng ( x + y − z )( y + z − x )( z + x − y ) ≤ xyz .
a

Lời giải
kh

Do tổng 3 thừa số vế trái bằng x + y + z ≥ 0 nên có các khả năng sau


+ TH1: Nếu có 1 thừa số âm và 2 thừa số dương lúc này bất đẳng thức luôn đúng.
+ TH2: Nếu có 2 thừa số âm và 1 thừa số dương(điều này vô lý) vì tổng của 2 thừa
số chẳng hạn ( x + y − z ) + ( y + z − x ) = 2 y ≥ 0 .
+ TH3: Nếu cả 3 thừa số đều không âm lúc đó ta sử dụng bất đẳng thức cho 2 số
dương ta được

201
Khám phá tư duy Kỹ thuật giải bất ĐT Bài toán Max – Min – Đặng Thành Nam

2
x+ y−z+ y+z−x
( x + y − z )( y + z − x ) ≤   =y2
 2 
2
y+z−x+z+x− y
( y + z − x )( z + x − y ) ≤   =z2
 2 
2
z+x− y+x+ y−z
( z + x − y )( x + y − z ) ≤   =x2
 2 
Nhân theo vế 3 bất đẳng thức trên ta được
( x + y − z )( y + z − x )( z + x − y )  ≤ x 2 y 2 z 2
2
.

n
⇔ ( x + y − z )( y + z − x )( z + x − y ) ≤ xyz

.v
Bất đẳng thức được chứng minh. Đẳng thức xảy ra khi và chỉ khi x= y= z .
Nhận xét. Vì chưa biết tính âm dương của mỗi thừa số vế trái nên chưa thể áp

om
dụng trực tiếp bất đẳng thức AM – GM và ta cần phân chia các trường hợp.
+ Đây là một dạng của bất đẳng thức Schur bậc ba có rất nhiều ứng dụng trong
việc giải quyết một lớp bài toán(xem chủ đề sau). .c
Chẳng hạn với x,y,z có tổng bằng 3 ta có bất đẳng thức
4 ( xy + yz + zx ) − 3 xyz ≤ 9 .
ok
Bài tập tương tự
Cho x,y,z là các số thực dương. Chứng minh
bo

 x  y  z  1
1 −  1 −  1 − ≤ .
 y + z  z + x  x + y  8
et

Bài 7. Cho a,b,c là các số thực dương thoả mãn điều kiện a 2 + b 2 + c 2 =
3.
vi

a3 b3 c3 3
Chứng minh rằng + + ≥ .
ng

b +3
2
c +3 a +3
2 2 2
Lời giải
Sử dụng bất đẳng thức AM – GM cho 3 số dương ta có
a
kh

a3 a3 b2 + 3 3 2
+ + ≥ a
b2 + 3 b2 + 3 8 2
b3 b3 c2 + 3 3 2
+ + ≥ b
c2 + 3 c2 + 3 8 2
c3 c3 a2 + 3 3 2
+ + ≥ c
a2 + 3 a2 + 3 8 2

Cộng theo vế 3 bất đẳng thức trên và kết hợp với điều kiện a 2 + b 2 + c 2 =
3 ta
có ngay điều phải chứng minh. Đẳng thức xảy ra khi và chỉ khi a= b= c= 1 .

202
Cty TNHH MTV DVVH Khang Việt

Bài 8. Cho a,b,c là các số thực dương thoả mãn điều kiện a + b + c =3.

Chứng minh rằng


(1 + a )2 (1 + b )2 + (1 + b )2 (1 + c )2 + (1 + c )2 (1 + a )2 ≥ 24 .
1 + c2 1 + a2 1 + b2
Lời giải
Gọi P là biểu thức vế trái của bất đẳng thức.
Sử dụng bất đẳng thức AM – GM cho 2 số dương ta được:
(1 + a )2 (1 + b )2 = (1 + ab + a + b )2 ≥ 4 (1 + ab )( a + b ) = ( ) ( )
4a 1 + b 2 + 4b 1 + a 2 .

Suy ra
(1 + a )2 (1 + b )2 ≥ 4a.1 + b2 + 4b. 1 + a 2

n
1 + c2 1 + c2 a + c2

.v
Tương tự ta có:
(1 + b )2 (1 + c )2 1 + c2 1 + b2
≥ 4b. + 4c.
1 + a2 1 + a2 1 + a2

om
(1 + c )2 (1 + a )2 1 + c2 1 + a2
≥ 4a. + 4c.
1 + b2 1 + b2 1 + b2
Cộng theo vế 3 bất đẳng thức trên ta có
.c
 1 + b2 1 + c2   1 + a2 1 + c2   1 + a 2 1 + b2 
ok
P ≥ 4a  +  + 4b  +  + 4 c  + 
 1 + c2 1 + b2   1 + c2 1 + a2  1 + a 2 
 1+ b
2
   
bo

1 + b2 1 + c2 1 + a2 1 + c2 1 + a 2 1 + b2
≥ 8a . + 8b . + 8c .
1 + c2 1 + b2 1 + c2 1 + a2 1 + b2 1 + a 2
= 8 ( a + b + c )= 24
et

Bất đẳng thức được chứng minh. Đẳng thức xảy ra khi và chỉ khi a= b= c= 1 .
vi

a b c
Bài 9. Cho a,b,c là các số thực dương thỏa mãn điều kiện + + = 2.
ng

a +1 b +1 c +1
Tìm giá trị nhỏ nhất của biểu thức P = ab + bc + ca .
a

Lời giải
Theo giả thiết kết hợp sử dụng bất đẳng thức AM –GM cho 2 số dương ta có
kh

a b c 1 1 2
=
1− +1− = + ≥ .
a +1 b +1 c +1 b +1 c +1 ( b + 1)( c + 1)
b 2
Tương tự ta có: ≥
b +1 ( a + 1)( c + 1)
c 2

c +1 ( a + 1)( b + 1)

203
Khám phá tư duy Kỹ thuật giải bất ĐT Bài toán Max – Min – Đặng Thành Nam

Lần lượt nhân theo vế hai bất đẳng thức ta được:


ab 4 4 ( a + 1)( b + 1)
≥ ⇔ ab ≥ .
( a + 1)( b + 1) ( c + 1) ( a + 1)( b + 1) c +1

4 ( b + 1)( c + 1) 4 ( c + 1)( a + 1)
Tương tự ta có: bc ≥ ; ca ≥
.
a +1 b +1
Cộng lại theo vế 3 bất đẳng thức trên và sử dụng AM-GM ta được:
 ( a + 1)( b + 1) ( b + 1)( c + 1) ( a + 1)( c + 1) 
P ≥ 4 + + 
 c +1 a +1 b +1 

n
( a + 1)( b + 1) ( b + 1)( c + 1) ( a + 1)( c + 1)

.v
≥ 12 3 . . =12
c +1 a +1 b +1

om
Đẳng thức xảy ra khi và chỉ khi a= b= c= 2 .
Bài 10. Cho x,y,z là các số thực dương thoả mãn điều kiện xy + yz + zx =
1.
x2 y 2 z 2
≥ ( x + y + z) .
Chứng minh rằng 3 − 3 +
y
+
z
+
x
.c
2
ok
Lời giải
Trước hết ta chứng minh bất đẳng thức sau
( )
bo

x2 y 2 z 2 ( x + y + z ) x + y + z
2 2 2
+ + ≥ .
y z x xy + yz + zx
et

Thật vậy, bất đẳng thức tương đương với:


 x2 y 2 z 2 
vi

( xy + yz + zx)  + +  ≥ ( x + y + z )( x 2 + y 2 + z 2 ) .
 y z x 

ng

x3 z y 3 x z 3 y
⇔ + + ≥ xz 2 + zy 2 + yx 2 .
y z x
a

x3 z y 3 x
kh

Sử dụng bất đẳng thức AM-GM ta có: + ≥ 2 x2 y .


y z
y3 x z3 y
+ ≥ 2 y2 z .
z x
x3 z z 3 y
+ ≥ 2z2 x .
y x
Cộng theo vế 3 bất đẳng thức trên ta có điều phải chứng minh.
Gọi P là biểu thức vế trái bất đẳng thức ban đầu ta có
P ≥ 3 − 3 + ( x + y + z )( x 2 + y 2 + z 2 ) .

204
Cty TNHH MTV DVVH Khang Việt

Vậy ta cần chứng minh


3 − 3 + ( x + y + z )( x 2 + y 2 + z 2 ) ≥ ( x + y + z ) 2
⇔ ( x + y + z )( x 2 + y 2 + z 2 ) ≥ x 2 + y 2 + z 2 + 3 − 1
⇔ ( x 2 + y 2 + z 2 )( x + y + z − 1) ≥ 3 − 1
⇔ ( x + y + z ) − 2 ( xy + yz + zx )  ( x + y + z − 1) ≥ 3 − 1
2
 
⇔ ( x + y + z ) − 2  ( x + y + z − 1) ≥ 3 − 1
2
 
Bất đẳng thức luôn đúng do x + y + z ≥ 3( xy + yz + zx) =3 .

n
Bất đẳng thức được chứng minh. Đẳng thức xảy ra khi và chỉ khi

.v
1
x= y= z= .
3

om
Bài tập tương tự
Cho x,y,z là các số thực dương có tổng bằng 1.
x2 y 2 z 2
( )
Chứng minh rằng
y
+
z
+
x
≥ 3 x2 + y 2 + z 2 .
.c
ok
C. BÀI TẬP RÈN LUYỆN
bo

Bài 1. Chứng minh rằng với mọi số thực x ta có


x x x
 12   15   20 
  +  +  ≥3 +4 +5 .
x x x
et

     
5 4 3
Bài 2. Chứng minh rằng với mọi số dương x,y,z ta có
vi

x3 y 3 z 3
+ + ≥ x2 + y 2 + z 2 .
ng

y z x
Bài 3. Chứng minh rằng với mọi x,y,z là các số thực dương ta có
a

x5 y5 z5
+ + ≥ x3 + y 3 + z 3 .
kh

2 2 2
y z x
Bài 4. Chứng minh rằng với mọi số thực dương x,y,z ta có
x3 y 3 z 3
+ + ≥ x+ y+z.
yz zx xy
Bài 5. Cho x,y,z là các số thực dương thoả mãn điều kiện x + y + z =xyz .
x y z
Chứng minh rằng 3
+ 3
+ ≥1.
y z x3

205
Khám phá tư duy Kỹ thuật giải bất ĐT Bài toán Max – Min – Đặng Thành Nam

Bài 6. Cho a,b,c là độ dài 3 cạnh một tam giác. Chứng minh rằng
a b c
+ + ≥ 3.
a+b−c b+c−a c+a−b
Bài 7. Cho a,b,c là các số thực dương thoả mãn điều kiện a 2 + b 2 + c 2 =
3.
a3 b3 c3 3
Chứng minh rằng + + ≥ .
b +1
2
c +1
2
a +1
2 2
Bài 8. Cho x,y,z là các số thực dương thoả mãn điều kiện 2 xz + yz + 3 xy =
1.
7 yz 8 zx 9 xy
Chứng minh rằng + + ≥4.

n
x y z
Bài 9. Cho x,y,z là các số thực dương thoả mãn điều kiện xyz = 1 .

.v
 1  1  1 

om
Chứng minh rằng  x + − 1 y + − 1 z + − 1 ≤ 1 .
 y  z  x 
1 1 1
Bài 10. Cho a,b,c là các số thực dương thỏa mãn + + =
1.
a +1 b +1 c +1
.c
Chứng minh rằng abc ≥ 8 .
ok
Bài 11. Cho x,y,z là các số thực dương. Chứng minh
 x  y  z  1
1 −  1 −  1 − ≤ .
bo

 y + z  z + x  x + y  8
et

D. HƯỚNG DẪN GIẢI – ĐÁP SỐ


Bài 1. Sử dụng bất đẳng thức AM – GM cho 2 số dương ta có
vi

x x x x
 12   15   12   15 
  +  ≥2   .  = 2.3x
ng

 5  4  5  4
x x x x
 15   20   15   20 
  +   ≥ 2   .  = 2.5 x
a

 4  3   4  3 
kh

x x x x
 20   12   20   12 
  +  ≥2   .  = 2.4 x
 3   5  3   5
Cộng lại theo vế 3 bất đẳng thức trên ta có điều phải chứng minh. Đẳng thức
xảy ra khi và chỉ khi x = 0 .
Bài 2. Sử dụng bất đẳng thức AM – GM cho 3 số dương ta được
x3 x3 x3 x3
+ + y 2 ≥ 33 . . y 2 =
3x 2
y y y y

206
Cty TNHH MTV DVVH Khang Việt

y3 y3 y3 y3 2
+ + z 2 ≥ 33 . .z =3y2
z z z z
z3 z3 z3 z3
+ + x 2 ≥ 3 3 . .x 2 = 3z 2
x x x x
Cộng theo vế 3 bất đẳng thức trên ta có điều phải chứng minh. Đẳng thức xảy ra
khi và chỉ khi x= y= z .
Cách 2: Sử dụng bất đẳng thức AM – GM cho 2 số dương ta có
x3 x3 y3 z3
+ xy ≥ 2 =
.xy 2 x 2 ; + yz ≥ 2 y 2 ; + xz ≥ 2 z 2 .
y y z x

n
Cộng theo vế 3 bất đẳng thức trên kết hợp với x 2 + y 2 + z 2 ≥ xy + yz + zx ta có

.v
điều phải chứng minh.
Bài 3. Sử dụng bất đẳng thức AM – GM cho 2 số dương ta có

om
x5 x5
+ xy 2 ≥ 2 .xy 2 =
2 x3
y2 y2 .c
y5 y5
+ yz 2 ≥ 2 . yz 2 =
2 y3
ok
z2 z2
z5 z5
+ zx 2 ≥ 2
.zx 2 =
2z3
bo

x2 x2
Cộng theo vế 3 bất đẳng thức trên ta có
et

x5
y2
+
y5
z2
+
z5
x2
( )
+ xy 2 + yz 2 + zx 2 ≥ 2 x3 + y 3 + z 3 .
vi

Vậy ta chỉ cần chứng minh x3 + y 3 + z 3 ≥ xy 2 + yz 2 + zx 2 .


ng

Bất đẳng thức này hiển nhiên đúng theo AM – GM cho 3 số dương
Vì x3 + y 3 + y 3 ≥ 3 xy 2 ; y 3 + z 3 + z 3 ≥ 3 yz 2 ; z 3 + x3 + x3 ≥ 3 zx 2 .
a

Cộng lại theo vế 3 bất đẳng thức trên ta có điều phải chứng minh. Đẳng thức
kh

xảy ra khi và chỉ khi x= y= z .


Bài 4. Sử dụng bất đẳng thức AM – GM cho 3 số dương ta có
x3 x3
+ y + z ≥ 33 . y.z =3x
yz yz

y3 y3
+ z + x ≥ 33 . z. x =3y
zx zx
z3 z3
+ x + y ≥ 33 .x. y =3z
xy xy

207
Khám phá tư duy Kỹ thuật giải bất ĐT Bài toán Max – Min – Đặng Thành Nam

Cộng theo vế 3 bất đẳng thức trên ta có điều phải chứng minh. Đẳng thức xảy ra
khi và chỉ khi x= y= z .
1 1 1
Bài 5. Theo giả thiết ta có + + =1.
xy yz zx
Vậy bất đẳng thức cần chứng minh được viết lại dưới dạng:
x y z 1 1 1
3
+ 3+ 3≥ + +
y z x xy yz zx
x 4 z 3 + y 4 x3 + z 4 y 3 x+ y+z
⇔ 3 3 3

x y z xyz

n
⇔ x 4 z 3 + y 4 x3 + z 4 y 3 ≥ ( x + y + z ) x 2 y 2 z 2

.v
 x2 z y 2 x z 2 y 
⇔ x2 y 2 z 2  2 + 2 + 2  ≥ ( x + y + z ) x2 y 2 z 2
 y x 

om
 z
x2 z y2 x z2 y
⇔ + + ≥ x+ y+z
y2 z2 x2 .c
Sử dụng bất đẳng thức AM – GM cho 3 số dương ta có
ok
x2 z y2 x x2 z y 2 x
+ + z ≥ 33 . .z =3x
y2 z2 y2 z2
bo

y2 x z2 y y2 x z2 y
+ + x ≥ 33 . .x =3y
z2 x2 z 2 x2
et

z2 y x2 z z 2 y x2 z
+ + y ≥ 33 .y =
vi

. 3z
x2 y2 x2 y 2
ng

Cộng theo vế 3 bất đẳng thức trên ta có điều phải chứng minh. Đẳng thức xảy ra
1
khi và chỉ khi x= y= z= .
a

3
Bài 6. Sử dụng bất đẳng thức AM – GM cho 3 số dương ta được
kh

a b c a b c
+ + ≥ 33 . . .
a+b−c b+c−a c+a−b a+b−c b+c−a c+a−b
Chú ý. ( a + b − c )( b + c − a )( c + a − b ) ≤ abc .
Ta có điều phải chứng minh. Đẳng thức xảy ra khi và chỉ khi a= b= c .
Bài 7. Sử dụng bất đẳng thức AM – GM cho 3 số dương ta có
a3 a3 b2 + 1 3 2
+ + ≥ a ;
b2 + 1 b2 + 1 2 2 2

208
Cty TNHH MTV DVVH Khang Việt

b3 b3 c2 + 1 3 2
+ + ≥ b ;
c2 + 1 c2 + 1 2 2 2
c3 c3 a2 + 1 3
+ + ≥ c2 .
a +1
2
a +1
2 2 2 2
Cộng theo vế 3 bất đẳng thức trên và kết hợp với điều kiện a 2 + b 2 + c 2 =
3 ta
có ngay điều phải chứng minh. Đẳng thức xảy ra khi và chỉ khi a= b= c= 1 .
Bài 8. Gọi P là biểu thức vế trái của bất đẳng thức.
 yz zx   yz xy   xy zx 
Ta viết lại P dưới dạng P = 3  +  + 4  +  + 5  +  .
 x y  x z   z y

n
Sử dụng bất đẳng thức AM – GM ta có

.v
yz zx yz zx
+ ≥2 . = 2z

om
x y x y
yz xy yz xy
+ ≥2 . = 2y
x z x z .c
xy zx xy zx
+ ≥2 . = 2x
ok
z y z y
Suy ra P ≥ 6 z + 8 y + 10 x = 4 ( x + z ) + 2 ( z + y ) + 6 ( x + y )
bo

≥ 8 xz + 4 yz + 12 xy =
4
 x= y= z 1
Đẳng thức xảy ra khi và chỉ khi  ⇔ x=y=z= .
et

2 xz + yz + 3 xy = 1 6
Bài 9. Vì x,y,z là các số thực dương thoả mãn xyz = 1 nên tồn tại các số thực dương
vi

a b c
a,b,c thoả mãn= x = ,y = khi đó bất đẳng thức được viết lại dưới dạng:
ng

,z
b c a
( a + b − c )( b + c − a )( c + a − b ) ≤ abc .
a

Đây là bất đẳng thức đã được chứng minh. Đẳng thức xảy ra khi và chỉ khi
x= y= z= 1 .
kh

Cách 2: Theo giả thiết ta có


1  1 1  1
x + − 1= x 1 + − = x 1 + z − 
y  xy x   x
 1  1   1  1 
⇒  x + − 1 z + − 1= x 1 + z −  z + − 1
 y  x   x  x 
 1  
2
= x  z 2 −  − 1  ≤ xz 2 .
  x  

209
Khám phá tư duy Kỹ thuật giải bất ĐT Bài toán Max – Min – Đặng Thành Nam

 1  1 
Tương tự ta có:  y + − 1  x − + 1 ≤ yx 2 .
 z  y 
 1  1 
 z + − 1 y + − 1 ≤ zy
2
 x  z 
Nhân theo vế 3 bất đẳng thức trên kết hợp điều kiện xyz = 1 ta có điều phải
chứng minh. Đẳng thức xảy ra khi và chỉ khi x= y= z= 1 .
a 1 1 1
Bài 10. Theo giả thiết ta có =
1− = + .
a +1 a +1 b +1 c +1
Sử dụng bất đẳng thức AM-GM cho 2 số dương ta được

n
a 2
≥ .

.v
a +1 ( b + 1)( c + 1)

om
b 2
Tương tự ta có: ≥ .
b +1 ( a + 1)( c + 1)
c 2
≥ .c
c +1 ( a + 1)( b + 1)
ok
Nhân theo vế 3 bất đẳng thức trên ta có ngay điều phải chứng minh.
Đẳng thức xảy ra khi và chỉ khi a= b= c= 2 .
Nhận xét. Với cách làm tương tự trên ta xử lý được bài toán tổng quát cho n số dương.
bo

1 1 1
Bài toán. Với x1 , x2 ,..., xn > 0 thỏa mãn + + ... + =
1 ta luôn có
1 + x1 1 + x2 1 + xn
et

x1 x2 ...xn ≥ ( n − 1) .
n
vi

Bài 11. Bất đẳng thức được viết dưới dạng tương đương:
ng

8 ( x + y − z )( y + z − x )( z + x − y ) ≤ ( x + y )( y + z )( z + x ) .
Chú ý ta có ( x + y − z )( y + z − x )( z + x − y ) ≤ xyz .
a

8 xyz ≤ ( x + y )( y + z )( z + x )
kh

Từ đó suy ra đpcm. Đẳng thức xảy ra khi và chỉ khi x= y= z .

210
Cty TNHH MTV DVVH Khang Việt

CHỦ ĐỀ 3: KỸ THUẬT SỬ DỤNG BẤT ĐẲNG THỨC AM – GM


DẠNG CỘNG MẪU SỐ
1 1 1 n2
+ + ... + ≥ .
A1 A2 An A1 + A2 + ... + An

A. NỘI DUNG PHƯƠNG PHÁP


Xuất phát từ bất đẳng thức
1 1 1  n
( a1 + a2 + ... + an )  + + ... +  ≥ n n a1a2 ...an . n =
n2 .
 a1 a2 an  a1a2 ...an

n
1 1 1 n2
Do đó + + ... + ≥

.v
.
a1 a2 an a1 + a2 + ... + an
Đẳng thức xảy ra khi và chỉ khi a1= a2= ...= an .

om
Hai bất đẳng thức dạng trên hay được sử dụng nhất là
1 1 4
+ ≥ .c
a b a+b
1 1 1 9
+ + ≥
ok
a b c a+b+c
Dưới đây tôi sẽ trình bày một lớp bài toán dạng trên.
bo

B. BÀI TẬP MẪU


Bài 1. Cho a,b,c là các số thực dương thoả mãn điều kiện a + b + c = 3 .
et

1 1 1 1 1 1
Chứng minh rằng + + ≥ + + .
a + 3b b + 3c c + 3a a + 3 b + 3 c + 3
vi

Lời giải
ng

1 1 4
Sử dụng bất đẳng thức + ≥ ta có
x y x+ y
a

1 1 4 2
+ ≥ =.
a + 3b c + 3 a + 3b + c + 3 b + 3
kh

1 1 4 2
Tương tự ta có + ≥ =
b + 3c a + 3 a + b + 3c + 3 c + 3
1 1 4 2
+ ≥ =
c + 3a b + 3 3a + b + c + 3 a + 3
Cộng theo vế 3 bất đẳng thức trên ta có điều phải chứng minh. Đẳng thức xảy ra
khi và chỉ khi a= b= c= 1 .
Bài 2. Cho a,b,c là độ dài ba cạnh một tam giác thỏa mãn điều kiện 2c + b =abc .
Tìm giá trị nhỏ nhất của biểu thức

211
Khám phá tư duy Kỹ thuật giải bất ĐT Bài toán Max – Min – Đặng Thành Nam

3 4 5
P= + + .
b+c−a a +c−b a +b−c
Lời giải
2 1
Từ điều kiện ta có: + = 1.
ab ac
Sử dụng bất đẳng thức AM-GM ta được:
2
12 1 1 32 1 1  3 2 1 3 2 1
=
1  + =  .  + ≤  + +  ⇒ + + ≥2 3.
a  b c  3 a  b c  12  a b c  a b c
Ta có:

n
 1 1   1 1   1 1 
P = 2 + + +  + 3 + .
b+c−a a +b−c b+c−a a +c−b  a +c−b a +b−c

.v
1 1 4
Sử dụng bất đẳng thức + ≥ , x, y > 0 ta có:

om
x y x+ y
8 4 12  3 2 1
P≥ + + = 2 + +  ≥ 4 3 .
2b 2c 2a a b c .c
Đẳng thức xảy ra khi và chỉ khi a= b= c= 3.
ok
Vậy giá trị nhỏ nhất của P bằng 4 3 đạt tại a= b= c= 3 .
Nhận xét. Điểm quan trọng nhất của bài toán là dự đoán dấu bằng tại a= b= c .
bo

 3 2 1
Ngoài ra ta có thể đánh giá P như sau: P ≥ 2  + +  .
a b c
et

2 1 3 
Theo giả thiết ta có: + =a suy ra P ≥ 2  + a  ≥ 4 3 . Đẳng thức xảy ra
 
vi

b c a
khi và chỉ khi a= b= c= 3 .
ng

Bài tập tương tự


Cho a,b,c là độ dài ba cạnh một tam giác thoả mãn điều kiện
a

2ab + 3ac + 4bc =9abc .


kh

5 6 7
Tìm giá trị nhỏ nhất của biểu thức P = + + .
b+c−a a +c−b a +b−c
Bài 3. Cho a,b,c là các số thực dương thoả mãn điều kiện a 2 + b 2 + c 2 =3.
1 1 1 4 4 4
Chứng minh rằng + + ≥ 2 + 2 + 2 .
a+b b+c c+a a +7 b +7 c +7
Lời giải
1 1 4 1 1 4
Sử dụng bất đẳng thức + ≥ ta có + ≥ .
x y x+ y a + b b + c a + 2b + c

212
Cty TNHH MTV DVVH Khang Việt

Theo AM – GM ta có a 2 + 1 ≥ 2a

(
2 b 2 + 1 ≥ 4b)
c 2 + 1 ≥ 2c
Cộng theo vế 3 bất đẳng thức trên ta được
2 ( a + 2b + c ) ≤ a 2 + 2b 2 + c 2 + 4 = (a 2
)
+ b2 + c2 + 4 + b2 = b2 + 7 .

1 1 8
Suy ra + ≥ 2 .
a+b b+c b +7
1 1 8
Tương tự ta có: + ≥ 2

n
b+c c+a c +7

.v
1 1 8
+ ≥ 2
c+a a+b a +7

om
Cộng theo vế 3 bất đẳng thức trên ta có điều phải chứng minh. Đẳng thức xảy ra
khi và chỉ khi a= b= c= 1 .
Bài 4. Cho a,b,c là các số thực dương chứng minh rằng .c
ab bc 11 1 1 ca
+ +
≤  + + .
ok
a +b +a c+b c
3 3 2 2
b +c +b a +c a c +a +c b+a b 4 a b c
3 3 2 2 3 3 2 2

Lời giải
bo

Ta có a3 + b3 ≥ ab ( a + b ) .
ab ab
⇒ ≤
et

a +b +a c+b c
3 3 2 2
(
ab ( a + b ) + c a 2 + b 2 )
vi

 
ab  1 1 
≤ +
4  ab ( a + b ) c a 2 + b 2  ( )
ng

 
1 1 1 ab 1 1  1 1  1
= . + . 2 . ≤  + + c
a

4 a + b 4 a + b 2 c 16  a b  8
kh

bc 1 1 1 1
Tương tự ta có ≤  + + .
b +c +b a+c a
3 3 2 2 16  b c  8a
ca 1 1 1 1
≤  + +
c +a +c b+a b
3 3 2 2 16  c a  8b
Cộng theo vế ba bất đẳng thức trên ta có đpcm. Đẳng thức xảy ra khi và chỉ khi
a= b= c .
Bài 5. Cho a,b,c là các số thực dương có tích bằng 1. Chứng minh
1 1 1 3
+ + ≤ .
ab + a + 2 bc + b + 2 ca + c + 2 4
213
Khám phá tư duy Kỹ thuật giải bất ĐT Bài toán Max – Min – Đặng Thành Nam

Lời giải
1 1 1 1 1  1 c 1 
Ta có = ≤  + =  + 
ab + a + 2 (ab + 1) + (a + 1) 4  ab + 1 a + 1  4  c + 1 a + 1 
1 1 1 1 1  1 a 1 
= ≤  + =  + 
bc + b + 2 (bc + 1) + (b + 1) 4  bc + 1 b + 1  4  a + 1 b + 1 
1 1 1 1 1  1 b 1 
= ≤  + =  + 
ca + c + 2 (ca + 1) + (c + 1) 4  ca + 1 c + 1  4  b + 1 c + 1 
Cộng theo vế ba bất đẳng thức trên ta có đcpm.
Đẳng thức xảy ra khi và chỉ khi a= b= c= 1 .

n
C. BÀI TẬP RÈN LUYỆN

.v
Bài 1. Chứng minh rẳng với mọi x,y,z dương ta có

om
1 1 1 xy + yz + xz
+ + ≤ .
2x + y + z 2 y + z + x 2z + x + y 4 xyz
Bài 2. Cho x,y,z là các số thực dương thoả mãn điều kiện xy + yz + zx + xyz =
.c 4.
1 1 1 1
Chứng minh rằng + + ≤
2x + y + 6 2 y + z + 6 2z + x + 6 3
ok
Bài 3. Cho a,b,c là các số thực dương. Chứng minh
6(a + b + c)
bo

a 2b 3c
+ + ≤ .
a +1 b + 2 c + 3 a + b + c + 6
Bài 4. Cho a,b,c là các số thực dương. Chứng minh
et

1 1 1 4 4 4
+ + ≥ + + .
a + 2b b + 2c c + 2a 3a + 4b + 5c 3b + 4c + 5a 3c + 4a + 5b
vi

Bài 5. Cho x,y,z là các số thực dương thỏa mãn điều kiện x + y 2 + z 3 =
13 .
ng

1 4 12
Tìm giá trị nhỏ nhất của biểu thức P= + + .
1+ x y z
a

Bài 6. Chứng minh rằng với mọi số thực dương tùy ý a,b,c,d ta luôn có
kh

a −d d −b b−c c−a
+ + + ≥0.
b+d c+b c+a d +a
Bài 7. Cho a,b,c là độ dài ba cạnh một tam giác thoả mãn điều kiện
2ab + 3ac + 4bc = 9abc .
 5 6 7 
Tìm giá trị nhỏ nhất của biểu thức P= 2  + + .
b+c−a a +c−b a +b−c
Bài 8. Cho a,b,c là các số thực dương có tích bằng 1. Chứng minh
a
+
b
+
c
ca + 1 ab + 1 bc + 1 2
1
(
≤ a 2 + b2 + c2 .)
214
Cty TNHH MTV DVVH Khang Việt

1 1 1
Bài 9. Cho a,b,c là các số thực dương thoả mãn điều kiện + + = 1.
a b c
1 8 1
Chứng minh rằng + ≤ .
( a − 1)( b − 1)( c − 1) ( a + 1)( b + 1)( c + 1) 4

HƯỚNG DẪN GIẢI – ĐÁP SỐ


1 1 1 1 16
Bài 1. Sử dụng bất đẳng thức + + + ≥ ta được:
a1 a2 a3 a4 a1 + a2 + a3 + a4
1 1 2 1 1
≤  + + 

n
2 x + y + z 16  x y z 

.v
1 1 2 1 1
≤  + + 
2 y + z + x 16  y z x 

om
1 1 2 1 1
≤  + + 
2 z + x + y 16  z x y 
.c
Cộng theo vế 3 bất đẳng thức trên ta có điều phải chứng minh. Đẳng thức xảy ra
khi và chỉ khi x= y= z .
ok
Bài 2. Gọi P là biểu thức vế trái của bất đẳng thức đã cho.
1 1 1 9
bo

Sử dụng bất đẳng thức + + ≥ ta được:


a1 a2 an a1 + a2 + a3
1 1 1 2 1 
et

= ≤  +
( x + 2 ) + ( x + 2 ) + ( y + 2 ) 9  x + 2 y + 2 
.
2x + y + 6
vi

Tương tự ta có
1 2 1 
ng

1
≤  + 
2y + z + 6 9  y + 2 z + 2 
1 1 2 1 
a

≤  + 
2z + x + 6 9  z + 2 x + 2 
kh

Cộng theo vế 3 bất đẳng thức trên ta được


1 1 1 1 
P≤  + + .
3 x + 2 y + 2 z + 2 
Chú ý điều kiện bài toán được viết lại dưới dạng:
1 1 1
+ + =1.
x+2 y+2 z+2
Từ đó suy ra điều phải chứng minh. Đẳng thức xảy ra khi và chỉ khi x= y= z= 1 .

215
Khám phá tư duy Kỹ thuật giải bất ĐT Bài toán Max – Min – Đặng Thành Nam

Bài 3. Bất đẳng thức được viết lại dưới dạng


a 2b 3c 6(a + b + c)
−1+ −2+ −3≤ −6
a +1 b+2 c+3 a+b+c+6
1 4 9 36
⇔ + + ≥
a +1 b + 2 c + 3 a + b + c + 6
Bất đẳng thức cuối luôn đúng. Đẳng thức xảy ra khi và chỉ khi
b+2 c+3
a+=1 = .
2 3
Bài 4.
Ta có

n
(1 + 1 + 1 + 1)2

.v
16 1 2 1
≤ + + .
3a + 4b + 5c ( a + 2b ) (
+ b + 2 c ) (
+ b + 2 c ) (
+ c + 2 a ) a + 2b b + 2 c c + 2a

om
Xây dựng tương tự 2 bất đẳng thức như trên rồi cộng lại ta có điều phải chứng minh.
Bài 5. Điểm rơi của bài toán là=
x 1,= y 2,= z 2.
Sử dụng bất đẳng thức AM-GM ta có .c
33 =x + y 2 + z 3 + 20 =x + y 2 + 4 + z 3 + 8 + 8 ≥ x + 4 y + 12 z .
ok
Sử dụng bất đẳng thức C-S dạng phân thức ta có:
1 (1 + 4.1 + 12.1)
2
1 1 17 2 17 2 289 17
bo

P= + 4. + 12. ≥ = ≥ = = .
1+ x y z 1 + x + 4 y + 12.z x + 4 y + 12 z + 1 34 34 2
Đẳng thức xảy ra khi và chỉ khi =
x 1,=
y 2,=
z 2.
et

17
Vậy giá trị nhỏ nhất của P bằng đạt tại x= 1, y= z= 2 .
vi

2
Bài 6. Cộng 4 vào 2 vế bất đẳng thức tương đương với:
ng

a+b c+d b+a c+d


+ + + ≥ 4.
b+d c+b c+a d +a
a

 1 1   1 1 
⇔ (c + d )  +  + ( a + b)  + ≥ 4.
kh

b+c a+d  b+d c+a


1 1 4
Chú ý + ≥ ;
b+c a+d a+b+c+d
1 1 4
+ ≥
b+d c+a a+b+c+d
Từ đó suy ra điều phải chứng minh. Đẳng thức xảy ra khi và chỉ khi a= b= c= d .
4 3 2
Bài 7. Theo giả thiết ta có: 2ab + 3ac + 4bc
= 9abc ⇔ + + = 9 .
a b c
Sử dụng bất đẳng thức AM – GM dạng cộng mẫu, ta có

216
Cty TNHH MTV DVVH Khang Việt

 1 1   1 1   1 1 
P= 2.  +  + 4.  +  + 3.  + 
b+c−a a +c−b  a +c−b a +b−c a+b−c b+c−a
.
4 4 4  4 3 2
⇒ P ≥ 2. + 4. + 3. ⇒ P ≥ 2  + +  = 18
2c 2a 2b a b c
Dấu bằng xảy ra khi và chỉ khi tam giác ABC đều a= b= c= 1 .
x y z
Bài 8. Tồn tại các số thực dương x,y,z sao cho=a = ,b = ,c .
y z x
x2 y2 z2 2x 2y 2z
Bất đẳng thức trở thành + + ≥ + + .
y 2
z 2
x 2 y+z z+x x+ y

n
Chú ý sử dụng bất đẳng thức AM – GM ta có

.v
x2 y2 z2 x y z
+ + ≥ + +

om
2 2 2 y z x
y z x
x2 y2 z2 x y z
2
+ 2
+ 2
≥ + +
y z x z x y
.c
Cộng theo vế hai bất đẳng thức trên và chú ý
ok
x y z x y z 1 1 1 1 1 1
+ + + + + = x +  + y +  + z + 
y z x z x y  y z x z x y
bo

4x 4y 4z
≥ + +
y+z z+x x+ y
et

Bất đẳng thức được chứng minh. Đẳng thức xảy ra khi và chỉ khi a= b= c= 1 .
Bài 9. Giả thiết ta có ab + bc + ca = abc .
vi

Suy ra
ng

1 8 1 8
+ = + .
( a − 1)( b − 1)( c − 1) ( a + 1)( b + 1)( c + 1) a + b + c − 1 2abc + a + b + c + 1
a

9
Chú ý a + b + c ≥ =9;
1 1 1
kh

+ +
a b c
abc = ab + bc + ca ≥ 3abc ( a + b + c ) ≥ 3 3abc ⇒ abc ≥ 27
Từ đó ta có ngay điều phải chứng minh. Đẳng thức xảy ra khi và chỉ khi
a= b= c= 3 .

217
Khám phá tư duy Kỹ thuật giải bất ĐT Bài toán Max – Min – Đặng Thành Nam

CHỦ ĐỀ 4: KỸ THUẬT SỬ DỤNG BẤT ĐẲNG THỨC


CAUCHY – SCHWARZ

A. NỘI DUNG PHƯƠNG PHÁP


Cho hai dãy số thực ( a1 , a2 ,..., an ) ; ( b1 , b2 ,..., bn ) khi đó ta có

( a1b1 + a2b2 + ... + anbn )2 ≤ ( a12 + a22 + ... + an2 )( b12 + b22 + ... + bn2 ) .
Đẳng thức xảy ra khi và chỉ khi= =
ai kbi , i 1, n, k ∈  .
Chứng minh.

n
Xét tam thức bậc hai

.v
f ( x)= (a 2
1 ) (
+ a22 + ... + an2 x 2 − 2 ( a1b1 + a2b2 + ... + an bn ) x + b12 + b22 + ... + bn2 ).

om
= ( a1 x − b1 )2 + ( a2 x − b2 )2 + ... + ( an x − bn )2 ≥ 0
Theo định lý về dấu của tam thức bậc hai ta có
( )(
∆ ' ≤ 0 ⇔ ( a1b1 + a2b2 + ... + an bn ) ≤ a12 + a22 + ... + an2 b12 + b22 + ... + bn2 .
2 .c )
Hệ quả(Bất đẳng thức Mincopski) Với 2 dãy số thực a1 , a2 ,.., an và b1 , b2 ,..., bn ta có
ok
a12 + b12 + a22 + b22 + ... + an2 + bn2 ≥ ( a1 + a2 + ... + an )2 + ( b1 + b2 + ... + bn )2 .
bo

Đẳng thức xảy ra khi và chỉ khi a1 , a2 ,.., an và b1 , b2 ,..., bn là 2 bộ số tỷ lệ.


et

B. BÀI TẬP MẪU


47
vi

Bài 1. Cho x,y,z là các số thực dương thoả mãn điều kiện x + y + z = .
12
ng

235
Chứng minh rằng 3 x 2 + 4 y 2 + 5 z 2 ≥ .
12
a

Lời giải
Phân tích tìm lời giải:
kh

Ta cần tìm các số a,b,c dương sao cho

( 3x )( ) ( )
2
2
+ 4 y 2 + 5 z 2 a 2 + b 2 + c 2 ≥ a 3 x + 2by + c 5 z .
Với vế phải bất đẳng thức là một hằng số vì vậy chỉ cần chọn a,b,c sao cho
a 3 x + 2by + c 5 z = x + y + z .
 1 1 1 
Ta có ngay ( a; b; c ) =  ; ; .
 3 2 5

218
Cty TNHH MTV DVVH Khang Việt

Lời giải chi tiết:


2
( )
1 1 1 2  47 
3x 2 + 4 y 2 + 5 z 2  + +  ≥ ( x + y + z ) =
3 4 5
 
 12  .
235
⇒ 3x 2 + 4 y 2 + 5 z 2 ≥
12
5 5 
Đẳng thức xảy ra khi và chỉ khi ( x; y; z ) =  ; ;1 .
3 4 
Bài 2. Cho x,y,a,b,c là các số thực thoả mãn điều kiện
( x + a )2 + ( y + b )2 + ( x + y )2 =
c2 .

n
Chứng minh rằng ( a + b ) ≤ 3c 2 .
2

.v
Lời giải

om
Phân tích tìm lời giải:

(
Ta cần chứng minh ( a + b ) ≤ 3 ( x + a ) + ( y + b ) + ( x + y )
2 2

.c 2 2
).
Chú ý a + b = ( x + a) + ( y + b) + (−x − y ) .
ok
Nên dựa vào bất đẳng thức trên ta sử dụng bất đẳng thức C –S cho hai bộ số
(1;1;1) ; ( x + a; y + b; − x − y ) .
bo

Lời giải chi tiết:


Sử dụng bất đẳng thức C –S ta có
( a + b )2 =
( x + a ) + ( y + b ) + ( − x − y )  ≤ 3 ( x + a ) + ( y + b ) + ( x + y )  =
et

2 2 2 2
3c 2 .
 
Bất đẳng thức được chứng minh.
vi

Bài 3. Cho x,y,z là các số thực thoả mãn x ≥ 5, y ≥ 6, z ≥ 7; x 2 + y 2 + z 2 ≥ 125 .


ng

Tìm giá trị nhỏ nhất của biểu thức P = x + y + z .


Lời giải
a

x + y + z= x + y + z 2 + 2 yz + 2 zx + 2 xy
2 2
kh

≥ 125 + 2 xy + 2( x + y ) z
.
≥ 125 + 2 xy + 2( x + y ) 125 − x − y 2 2

≥ 125 + 60 + 22 64 =
19
Cơ sở nhỏ nhất ta ép về 2 biến có chặn dưới nhỏ nhất là x và y khi đó tích xy
nhỏ nhất trong các tích.

219
Khám phá tư duy Kỹ thuật giải bất ĐT Bài toán Max – Min – Đặng Thành Nam

 x2 + y 2 =9

Bài 3. Cho x,y,z,t là các số thực thoả mãn điều kiện  z 2 + t 2 =16 .
 xt + yz = 12

Chứng minh rằng x + z ≤ 5 .
Lời giải
Chú ý hệ điều kiện ta có đẳng thức sau: ( xt + yz ) = x 2 + y 2
2
( )( z 2
)
+ t 2 = 144 .
Do đó=x kt=
, y kz .

( )

n
9
Vì vậy x 2 + y =
2
k 2 t2 + z2 ⇒ k=
2

.v
16

y kx + y ( kx + y )
2 k 2 + 1 x2 + y 2 ( )( )

om
⇒S = x+z= x+ = ⇒S =
2
≤ = 25 .
k k k2 k2
Bất đẳng thức được chứng minh.
Bài tập tương tự .c
a 2 + b2 + c 2 =9
ok

Cho a,b,c,x,y,z là các số thực thoả mãn điều kiện  x + y + z =
2 2 2
16 .
ax + by + cz =12
bo


Chứng minh rằng a + y + z ≤ 41 .
et

Bài 4. Cho x,y,z là các số thực không âm. Chứng minh


( x + y + 2 z )2
vi

x3 y + y 3 z + z 3 x + x3 z + y 3 x + z 3 y ≤ .
2
ng

Lời giải
Sử dụng bất đẳng thức C –S ta có:

( )
a

x3 y + y 3 z + z 3 x + x3 z + y 3 x + z 3 y ≤ 2 x3 y + y 3 z + z 3 x + x3 z + y 3 x + z 3 y
kh

= 2∑ xy ( x 2 + y 2 )

≤ 2( xy + yz + zx)( x 2 + y 2 + z 2 )
x 2 + y 2 + z 2 + 2( xy + yz + zx)

2
( x + y + z )2 ( x + y + 2 z )2
= ≤
2 2
Đẳng thức xảy ra khi và chỉ khi=
x y=
,z 0.

220
Cty TNHH MTV DVVH Khang Việt

Bài 5. Cho x1 , x2 , y1 , y2 , z1 , z2 là các số thực thoả mãn điều kiện


x1 , x2 > 0, x1 y1 > z12 , x2 y2 > z22 .
1 1 8
Chứng minh rằng + ≥ .
x1 y1 − z12 x2 y2 − z22 ( x1 + x2 )( y1 + y2 ) − ( z1 + z2 )2
Lời giải
Bất đẳng thức tương đương với:
 1 1 
 + ( x + x )( y + y ) − ( z + z )2  ≥ 8 .
2 
  1 2 1 2 1 2 
 x1 y1 − z1 x2 y2 − z2 
2

n
Sử dụng bất đẳng thức C –S ta có

.v
2
 z z   z2 z2 
( z1 +=
z2 )  x1 . 1 + x2 . 2  ≤ ( x1 + x2 )  1 + 2  .

2
x 
x2 

om
 x1  1 x2 
Kết hợp sử dụng bất đẳng thức AM – GM ta có:
 z12 z22 
( x1 + x2 )( y1 + y2 ) − ( z1 + z2 )2 ≥ ( x1 + x2 )( y1 + y2 ) − ( x1 + x2 ) 
.c  x1
+ 
x2 
ok
 z2 z2 
= ( x1 + x2 )  y1 − 1 + y2 − 2 
 x1 x2 

bo

 x y − z2 x y − z2 
= ( x1 + x2 )  1 1 1 + 2 2 2 
 x1 x2 
et

x1 y1 − z12 x2 y2 − z22
≥ 2 x1 x2 .2 .
vi

x1 x2

( x y − z )( x y )
ng

=4 1 1
2
1 2 2 − z22

1 1 2
a

Mặt khác + ≥ .
x1 y1 − z12 x2 y2 − z22 ( x y − )( x y
z12 − z22 )
kh

1 1 2 2

Nhân theo vế hai bất đẳng thức trên ta có ngay điều phải chứng minh.
Đẳng thức xảy ra khi và chỉ khi
= x1 x= 2 ; y1 y=
2 ; z1 z2 .

Tổng quát với xk > 0, xk yk > zk2 , k =


1, n ta có
n
1 n3
∑x − zk2
≥ 2
k =1 k yk  n  n   n 
 ∑ k 
x  ∑ y k  −
   ∑ z k 
=  k 1=  k 1 =  k 1 

221
Khám phá tư duy Kỹ thuật giải bất ĐT Bài toán Max – Min – Đặng Thành Nam

Bài 3. Cho a,b,c,x,y,z là các số thực dương thoả mãn điều kiện ax + by + cz =
xyz .
Chứng minh rằng x + y + z > a + b + b + c + c + a .
Lời giải
a b c
Theo giả thiết ta có + + =
1.
yz zx xy
Đặt a= myz , b= nzx, c= pxy ⇒ m + n + p= 1 .
Bất đẳng thức trở thành
x + y + z > z ( my + nx ) + x ( nz + py ) + y ( mz + px ) .
Sử dụng bất đẳng thức C –S cho vế phải ta có

n
z ( my + nx ) + x ( nz + py ) + y ( mz + px )

.v
≤ ( x + y + z )( my + mz + nx + nz + px + py )

om
.
< ( x + y + z )( m + n + p )( x + y + z ) = x + y + z
Bất đẳng thức được chứng minh. .c
Nhận xét. Ta có thể tìm được giá trị nhỏ nhất của tổng x + y + z bằng
ok
M ( M + a )( M + b )( M + c ) .
Chính là nghiệm dương duy nhất của phương trình
bo

2 1 1 1
= + + .
M a+M b+M c+M
et

Bài 3. Cho x,y là 2 số thực dương thoả mãn điều kiện x 2 + y 3 ≥ x3 + y 4 .


vi

Chứng minh rằng x3 + y 3 ≤ 2 .


Lời giải
ng

Phân tích tìm lời giải:


Bằng cách sử dụng C –S làm xuất hiện các bậc x 2 , y 4 để tận dụng điều kiện bài toán.
a

Sử dụng bất đẳng thức C – S, ta có


kh

(x ) (x ) ≤ (x )( x )
2 2
3
+=
y3 x .x x + y 2 . y 3
+ y4 3
+ y2
2.
 x 2 + y 3 + x3 + y 2 
(
≤ x +y
2 3
)( x 3
+y 2
) ≤
 2

 
x 2 + y 3 + x3 + y 2
Suy ra x3 + y 3 ≤ ⇔ x3 + y 3 ≤ x 2 + y 2 .
2
Vậy ta chỉ cần chứng minh x 2 + y 2 ≤ 2 .

222
Cty TNHH MTV DVVH Khang Việt

Thật vậy sử dụng bất đẳng thức C –S ta có

(x ) (x ) ≤ ( x3 + y 3 ) ( x + y ) ≤ ( x 2 + y 2 ) ( x + y )
2 2
2
+=
y2 x. x + y y. y

(
⇒ x2 + y 2 ≤ x + y ≤ 2 x2 + y 2 ⇒ x2 + y 2 ≤ 2 )
Bất đẳng thức được chứng minh. Đẳng thức xảy ra khi và chỉ khi x= y= 1 .
Bài 5. Cho x,y,z là các số thực dương thỏa mãn điều kiện 4 xy + 2 yz − zx =
25

x2 + 4 y 2 2 2
Tìm giá trị nhỏ nhất của biểu thức=
P + z + 4 xy .
z + 4 xy
2 5

n
Lời giải

.v
Áp dụng bất đẳng thức C –S ta có
=
25 4 xy . 4 xy + z (2 y − x)

om
≤ (4 xy + z 2 )[4 xy + (2 y − x)2 ]

=( z 2 + 4 xy )( x 2 + 4 y 2 ).c
25
⇒ x2 + 4 y 2 ≥
ok
z 2 + 4 xy
25 2 2
Vì vậy P ≥ + z + 4 xy .
bo

z + 4 xy
2 5
Áp dụng BĐT AM-GM ta có:
et

25 2 2 25 1 2 1 2
vi

+ z + 4 xy= 2 + z + 4 xy + z + 4 xy
z + 4 xy
2 5 z + 4 xy 5 5
ng

25
1 2 1 2
≥ 33 z + 4 xy .
. z + 4 xy =
3
z + 4 xy 5
2 5
a

Với=
x 3,=
y 2,=
z 1 thì P bằng 3.
kh

Vậy giá trị nhỏ nhất của P bằng 3.


Bài 4. Cho x,y,z là các số thực thoả mãn điều kiện x + y + =
z 0; x 2 + y 2 + z=
2
6.
Tìm giá trị lớn nhất của biểu thức P =( x − y )( y − z )( z − x ) .
Lời giải
Không mất tính tổng quát giả sử x ≥ y ≥ z .
( x − y )( y − z )( x − z ) .
Khi đó P =
Sử dụng bất đẳng thức C – S ta có

223
Khám phá tư duy Kỹ thuật giải bất ĐT Bài toán Max – Min – Đặng Thành Nam

( ) (
x − z ≤ 2 x2 + z 2 ≤ 2 x2 + y 2 + z 2 =
2 3. )
Sử dụng bất đẳng thức AM – GM ta có
2
 x− y+ y−z 1
P≤  ( x − z )= ( x − z )3 ≤ 6 3 .
 2  4
Đẳng thức xảy ra khi và chỉ khi x = 3, y = 0, z = − 3 hoặc các hoán vị.

Bài 5. Cho x,y,z là các số thực thoả mãn điều kiện x 2 + y 2 + z 2 =


2.
Chứng minh rằng x + y + z ≤ 2 + xyz .

n
Lời giải

.v
Phân tích tìm lời giải:
Bất đẳng thức viết lại dưới dạng x (1 − yz ) + y + z ≤ 2 .

om
Ta đánh giá thông qua tổng x 2 + y 2 + z 2 vì vậy áp dụng C – S cho 2 bộ số
( x; y; z ) và (1 − yz;1;1) hoặc ( x; y + z ) ; (1 − yz;1) . .c
Ta lựa chọn 2 bộ số thứ 2 vì khi đó ta có một biểu thức tích của yz .
ok
Lời giải chi tiết:
Sử dụng bất đẳng thức C – S ta có
bo

x (1 − yz ) + y + z ≤ (x 2
+ ( y + z)
2
) ((1 − yz ) 2
+ 12 )
( )
et

= ( 2 + 2 yz ) 2 + y 2 z 2 − 2 yz
vi

Vậy ta chỉ cần chứng minh ( 2 + 2 yz ) ( 2 + y 2 z 2 − 2 yz ) ≤ 2


ng

( )
⇔ (1 + yz ) 2 + y 2 z 2 − 2 yz ≤ 2 ⇔ y 3 z 3 ≤ y 2 z 2 .
Bất đẳng thức cuối luôn đúng vì
a

2 = x 2 + y 2 + z 2 ≥ y 2 + z 2 ≥ 2 yz ⇒ yz ≤ 1 ⇒ y 3 z 3 ≤ y 2 z 2 .
kh

Bất đẳng thức được chứng minh đẳng thức xảy ra chẳng hạn 1 số bằng 0 và 2 số
bằng 1.
Bài tập tương tự
Cho x,y,z là các số thực không âm thoả mãn điều kiện x 2 + y 2 + z 2 =
1.
9
Chứng minh rằng x + y + z ≤ 2 + xyz .
4

224
Cty TNHH MTV DVVH Khang Việt

Bài 6. Cho x,y,z là các số thực thoả mãn điều kiện x 2 + y 2 + z 2 =


1 . Chứng minh
3 +1
rằng xy + yz + 2 zx ≤ .
2
Lời giải
Chú ý vai trò đối xứng của x và z vì vậy sử dụng bất đẳng thức C –S ta có

(
y ( x + z ) ≤ 2 y 2 x2 + y 2=
z2 ) (
2 y2 1 − y2 ).
2 zx ≤ x 2 + z 2 =−
1 y2
Cộng theo vế hai bất đẳng thức đưa về chứng minh

n
( )
3 +1
2 y2 1 − y2 + 1 − y2 ≤
.

.v
2
Đến đây có thể đưa về khảo sát hàm số hoặc sử dụng trực tiếp C –S như sau

om
(
2 y 2 1 − y 2 += )
1 − y2
1
2
1
4 y2 − 4 y4 + 1 − 2 y2 +
2
1
2
( )
.
2 2  1 1+ 3
( )
 1 1  2
≤  +  4y − 4y + 1− 2y
 2 4 
4
.c
+ =
 2 2
ok
1 1 1 1
Đẳng thức xảy ra khi và chỉ khi x =
z=± 1− ,y=
± + .
2 3 2 2 3
bo

Bài 5. Chứng minh rằng nếu phương trình x 4 + bx3 + cx 2 + bx + 1 =0 có nghiệm thì
b2 + ( c − 2 ) > 3 .
2
et

Lời giải
vi

Nếu x0 là nghiệm của phương trình rõ ràng x0 ≠ 0 viết lại phương trình dưới dạng
ng

2
1  1   1   1 
x02 + 2 + b  x0 +  + c =⇔
0  x0 +  + b  x0 +  + c − 2 =
0.
x0  x0   x0   x0 
a

1
Đặt X =
x0 + , X ≥ 2 phương trình trở thành
kh

x0

X 2 + bX + c − 2 = 0 ⇒ ( bX + c − 2 ) = X 4 .
2

Sử dụng bất đẳng thức C –S ta có


( bX + c − 2 )2 ≤ b2 + ( c − 2 )2  ( X 2 + 1) .
X4
1
Suy ra b 2 + ( c − 2 ) ≥
2
= X 2 −1+ 2 > X 2 −1 ≥ 3 .
X +1 X +1
2

Bất đẳng thức được chứng minh.

225
Khám phá tư duy Kỹ thuật giải bất ĐT Bài toán Max – Min – Đặng Thành Nam

Bài tập tương tự


Chơ phương trình x 4 + ax3 + bx 2 + 2ax + 4 =0.
16
Tìm nghiệm của phương trình biết a 2 + b 2 = .
9
1 1 1
Bài 3. Cho x,y,z là các số thực thoả mãn điều kiện x, y, z ≥ 1 và + + =2.
x y z
Chứng minh rằng x + y + z ≥ x −1 + y −1 + z −1 .
Lời giải
Viết lại điều kiện giả thiết dưới dạng

n
1 1 1
1 = 1− +1− +1−

.v
x y z
x −1 y −1 z −1

om
⇔ + + =
1
x y x
Sử dụng bất đẳng thức C –S ta có

+
.c
 x −1 y −1 z −1 
x + y + z = ( x + y + z) + 
 x y z 
ok
2
 x −1 y −1 z −1 
≥  x . + y. + z. 
z  .
bo

 x y

( )
2
= x −1 + y −1 + z −1
et

⇒ x + y + z ≥ x −1 + y −1 + z −1
vi

3
Bất đẳng thức được chứng minh. Đẳng thức xảy ra khi và chỉ khi x= y= z= .
ng

2
Bài 4. Cho x,y,z là các số thực thuộc đoạn [ −1;1] và thoả mãn điều kiện
x + y + z + xyz =0.
a

x +1 + y +1 + z +1 ≤ 3 .
kh

Chứng minh rằng


Lời giải
+ Nếu x + y + z ≤ 0 ta có ngay điều phải chứng minh vì
x +1 + y +1 + z +1 ≤ (1 + 1 + 1)( x + 1 + y + 1 + z + 1)
= 3( x + y + z ) + 9 ≤ 3
+ Ta xét với x + y + z > 0 ⇒ xyz =− ( x + y + z ) < 0 do đó phải tồn tại ít nhất 1 số
âm và 1 số dương.

226
Cty TNHH MTV DVVH Khang Việt

 xy > 0
Không mất tính tổng quát giả sử z < 0 ⇒  ⇒ x, y > 0 .
x + y > −z > 0
Khi đó vai trò của x và y như nhau nên ta áp dụng C – S cho 2 căn thức đầu tiên
ta được
x + 1 + y + 1 + z + 1 ≤ 2 ( x + 1 + y + 1) + z + 1
= 2( x + y) + 4 + z +1
Bất đẳng thức được chứng minh nếu ta chứng minh được
2( x + y) + 4 + z +1 ≤ 3

n
⇔ 2( x + y) + 4 − 2 ≤ 1 − z +1

.v
2( x + y) z
⇔ ≤−
2( x + y) + 4 + 2

om
1+ z +1

Chú ý x + y =− z ( xy + 1) ;1 + z =−
x+ y
+1 =
(1 − x )(1 − y ) .
xy + 1 1 + xy
.c
Vậy ta đi chứng minh
ok
−2 z ( xy + 1) −z

2( x + y) + 4 + 2 (1 − x )(1 − y )
1+
bo

1 + xy
x+ y
⇔ xy + (1 − x )(1 − y )(1 + xy ) ≤ 1 + 1+
et

2
Bất đẳng thức luôn đúng vì
vi

xy + (1 − x )(1 − y )(1 + =
xy ) x . xy 2 + (1 − x ). (1 − y )(1 + xy )
ng

≤ ( x + 1 − x ) ( xy 2 + (1 − y )(1 + xy ) )
a

x+ y
= 1 + xy − y = 1 + y ( x − 1) ≤ 1 ≤ 1 + 1 +
kh

2
Bất đẳng thức được chứng minh. Đẳng thức xảy ra tại x= y= z= 0 .
Bài 6. Cho a,b,c là các số thực dương. Chứng minh rằng

( )( )( 5
)
a 2 + 1 b 2 + 1 c 2 + 1 ≥ ( a + b + c + 1) .
16
2

Lời giải
1
Dự đoán dấu bằng đạt tại a= b= c thay vào bất đẳng thức ta tìm được a= b= c= .
2
Sử dụng bất đẳng thức C –S ta có

227
Khám phá tư duy Kỹ thuật giải bất ĐT Bài toán Max – Min – Đặng Thành Nam

2
5
16
( a + b + c +=
1)
2 5
 a.1 + 1.
16 
b + c +1
1
 ≤
 16
5 2
 (
a + 1 1 + ( b + c + 1)  .
2
 )
(
Vậy ta chỉ cần chứng minh b 2 + 1 c 2 + 1 ≥ )( ) 5 
16 
1 + ( b + c + 1) 
2

⇔ 16b 2 c 2 + 11b 2 + 11c 2 + 6 ≥ 10bc + 10b + 10c .
Bất đẳng thức trên là tổng của các bất đẳng thức sau
16b 2 c 2 + 1 ≥ 8bc
b 2 + c 2 ≥ 2bc
 1

n
10  b 2 +  ≥ 10b
 4

.v
 1
10  c 2 +  ≥ 10c

om
 4
1
Bất đẳng thức được chứng minh. Đẳng thức xảy ra khi và chỉ khi a= b= c= .
.c 2
Cách 2: Chú ý. Với mọi số thực a1 , a2 ,..., an > −1 và cùng dấu ta luôn có
ok
(1 + a1 )(1 + a2 ) ...(1 + an ) ≥ 1 + a1 + a2 + ... + an .
1 1 1
Trong 3 số a 2 − , b 2 − , c 2 − có ít nhất 2 số cùng dấu không mất tính tổng
bo

4 4 4
1 1
quát giả sử a 2 − ; b 2 − cùng dấu ta có:
et

4 4

( a + 1)(b + 1)=
2 2  2 1  5   2 1  5 
 a − 4  + 4  .  b − 4  + 4 
vi

     
ng

25  4  1   4  1 
= 1 +  a 2 −   . 1 +  b 2 −  
16  5  4   5  4 
a

25  4  2 1  2 4  2 1  
≥ 1 +  a −  a +  b − 
16  5 
kh

4 5 4 
Bất đẳng thức được chứng minh nếu ta chứng minh được:
25  4  2 1  4  2 1   2

16  5  4 5 4 
5
16
(
1 +  a −  +  b −   c + 1 ≥ ( a + b + c + 1) .
2
)
 4 1 4 1 
( )
5 1 +  a 2 −  +  b 2 −   c 2 + 1 ≥ ( a + b + c + 1) .
 5 4 5 4 
2

( )( )
⇔ 4a 2 + 4b 2 + 3 c 2 + 1 ≥ ( a + b + c + 1) .
2

228
Cty TNHH MTV DVVH Khang Việt

Theo C-S ta có

( 4a 2
)(
+ 4b 2 + 3 c 2 +=
1 ) ( 4a 2 1 1
)
1 1
+ 4b 2 + 1 + 1 + 1  + + c 2 + + 
4 4 4 4
2
 1 1 1 1
≥  2a. + 2b. + 1.c + 1. + 1.  = ( a + b + c + 1)2
 2 2 2 2
1
Bất đẳng thức được chứng minh. Đẳng thức xảy ra khi và chỉ khi a= b= c= .
2
Bài tập tương tự
Cho a,b,c là các số thực dương chứng minh

(a )( )( )

n
+ 3 b 2 + 3 c 2 + 3 ≥ 4 ( a + b + c + 1) .
2 2

.v
Bài 10. Cho a,b,c là các số thực dương thoả mãn điều kiện a + b + c + 1 =4abc .

om
1 1 1 3
Chứng minh rằng 4 + 4 + 4 ≤ .
a +b+c b +c+a c +a+b a+b+c
Lời giải .c
Sử dụng bất đẳng thức C – S ta có

(a )( ) ( )
2
ok
4
+ b + c 1 + b3 + c 3 ≥ a 2 + b 2 + c 2

1 1 + b3 + c 3
⇒ ≤
bo

a4 + b + c (a 2
+ b2 + c2 )
1 + c3 + a 3 1 + a 4 + b4
et

1 1
Tương tự ta có ≤ ; ≤ .
b4 + c + a
( ) c4 + a + b
( )
2 2
a 2 + b2 + c2 a 2 + b2 + c2
vi

Cộng theo vế ba bất đẳng thức trên ta được


ng

P≤
(
3 + 2 a 3 + b3 + c 3 ).
(a 2
+ b2 + c 2 2
)
a
kh

Ta chỉ cần chứng minh


(
3 + 2 a 3 + b3 + c 3 )≤ 3

(a 2
+ b2 + c )
2 2 a+b+c

( ) (
⇔ a 4 + b 4 + c 4 + 6 a 2b 2 + b 2 c 2 + c 2 a 2 ≥ 2 a3b + b3a + b3c + c3b + c3a + a3c + 3 ( a + b + c ) )
( ) ( )
2
⇔ a 2 + b 2 + c 2 − ab − bc − ca + 3 a 2b 2 + b 2 c 2 + c 2 a 2 − a − b − c ≥ 0
.

229
Khám phá tư duy Kỹ thuật giải bất ĐT Bài toán Max – Min – Đặng Thành Nam

Chú ý

( )
a 2b 2 + b 2 c 2 + c 2 a 2 ≥ 3a 2b 2 c 2 a 2 + b 2 + c 2 ≥ abc ( a + b + c ) ≥ a + b + c

bởi vì 4abc = 1 + a + b + c ≥ 1 + 3 3 abc ⇒ abc ≥ 1 .


Bất đẳng thức được chứng minh. Đẳng thức xảy ra khi và chỉ khi a= b= c= 1 .
Bài tập tương tự
Cho a, b, c là các số thực dương thoả mãn điều kiện
1 1 1
+ + ≥1.

n
a + b +1 b + c +1 c + a +1

.v
Chứng minh rằng a + b + c ≥ ab + bc + ca .
Bài 11. Cho a, b, c là các số thực dương. Chứng minh

om
( a + b + c ) 
1 1 1
+ +  ≥1+ 3 5 +
a b c
(a 3
)  1 1 1
+ b3 + c 3  3 + 3 + 3  .
a b c 
Lời giải
.c
ok
3
( a + b + c )  + + 
31
1 1
Ta có
a b c
bo

 a3 + b3 + c3 + 3 ( a + b )( b + c )( c + a )   1 + 1 + 1 + (
 3 a + b )( b + c )( c + a ) 
= 
  a 3 b3 c 3 a 2b 2 c 2
 
et

1  3 ( a + b )( b + c )( c + a )
≥ (a 3
) 1 1
+ b3 + c 3  3 + 3 + 3  +
vi

a b c  abc
ng

Chú ý
( a + b )( b + c )( c + a ) = ( a + b + c )( ab + bc + ca ) − 1 =
( a + b + c ) 
1 1 1
+ +  −1
a b c
a

abc abc
Do đó
kh

1 1 1 1 1 1
( a + b + c ) 
1 1 1
3( a + b + c )  + +  − 3 ≥ 3( a + b + c )  + +  − 3 + +  +6
a b c a b c a b c
1 1 1
Vì ( a + b + c )  + +  ≥ 9 .
a b c

230
Cty TNHH MTV DVVH Khang Việt

Từ đó suy ra
3
 

 ( a + b + c )  + +  − 1 ≥ 5 +
1 1 1
a b c 
(a 3  1 1
) 1
+ b3 + c 3  3 + 3 + 3 
a c 
 b

⇒ ( a + b + c ) 
1 1 1
+ +  ≥1+ 3 5 +
a b c
(a 3  1 1
) 1
+ b3 + c 3  3 + 3 + 3 
a b c 
Bất đẳng thức được chứng minh. Đẳng thức xảy ra khi và chỉ khi a= b= c .

Bài tập tương tự

n
Cho a,b,c là các số thực dương chứng minh P = (a 2  1 1 1 
)
+ b2 + c2  2 + 2 + 2  .
a c 

.v
b
C. BÀI TẬP RÈN LUYỆN

om
Bài 1. Cho a,b là các số thực chứng minh (1 + a )(1 + b ) ≥
1
2
( a + b + ab − 1) .
2 2

Bài 2. Cho x,y,z là các số thực thoả mãn điều kiện 1 ≤ x ≤ y ≤ z ≤ 4 .


.c 2 2 2
2  y  z  4 
Tìm giá trị nhỏ nhất của biểu thức P =( x − 1) +  − 1 +  − 1 +  − 1 .
ok
x  y  z 
Bài 2. Sử dụng bất đẳng thức C –S ta có
bo

2
1 y z 4 
P ≥  x − 1 + − 1 + − 1 + − 1 .
4 x y z 
et

Chú ý sử dụng bất đẳng thức AM – GM ta có


vi

y z 4 y z 4 1
( ) ( )
2 2
x+ + + ≥ 4 4 x. . . = 4 2 ⇒ P ≥ 4 2 − 4 = 4 2 −1 .
x y z x y z 4
ng

y z 4
Đẳng thức xảy ra khi và chỉ khi x = = = ⇔x= 2, y = 2, z = 2 2 .
x y z
a

Bài 2. Chứng minh rằng với mọi x,y,z là các số thực dương ta có
kh

x y z 9
+ + ≥ .
( y + z ) ( z + x) ( x + y) 4( x + y + z )
2 2 2

Bài 3. Cho x,y,z là các số thực không âm.


Chứng minh rằng x2 + 1 + y 2 + 1 + z 2 + 1 ≥ 6 ( x + y + z ) .
Bài 4. Chứng minh rằng với mọi x,y,z là các số thực dương ta có
x3 y3 z3 x2 y 2 z 2
+ + ≥ + + .
y2 z2 x2 y z x

231
Khám phá tư duy Kỹ thuật giải bất ĐT Bài toán Max – Min – Đặng Thành Nam

Bài 5. Cho các số thực a,b,c thay đổi thỏa mãn điều kiện a 2 + b 2 + c 2 =
1.
Tìm giá trị lớn nhất, giá trị nhỏ nhất của biểu thức P = a3 + b3 + c3 − abc .
Bài 6. Cho a,b,c là các số thực thoả mãn điều kiện a 2 + b 2 + c 2 =
2.
Chứng minh rằng a3 + b3 + c3 − abc ≤ 2 2 .

Bài 7. Cho a,b,c là các số thực thoả mãn điều kiện a 2 + b 2 + c 2 =


9.
Chứng minh rằng 2 ( a + b + c ) − abc ≤ 10 .

Bài 8. Cho a,b,c là các số thực thoả mãn điều kiện x 2 + y 2 + z 2 = k , ( k ≥ 0 ) .

n
Chứng minh rằng 18k ( x + y + z ) − xyz ≤ 270k k .

.v
Bài 9. Cho x,y,z là các số thực không âm thoả mãn điều kiện x 2 + y 2 + z 2 =
1.

om
9
Chứng minh rằng x + y + z ≤ 2 + xyz .
4
Bài 10. Chứng minh rằng với mọi a,b,c là các số thực dương ta có .c
b+c c+a a+b 4(a + b + c)
+ + ≥ .
ok
a b c ( a + b )( b + c )( c + a )
Bài 11. Chứng minh với mọi a,b,c là các số thực dương ta có
bo

a 2 + b2 b2 + c2 c2 + a2
+ + ≥1.
( )
2 a 2 + b2 + c ( a + b ) ( )
2 b2 + c2 + a ( b + c ) ( )
2 c2 + a2 + b ( c + a )
et

2a 2b 2c
Bài 12. Cho a,b,c là các số thực dương. Chứng minh + + ≤3
vi

a+b b+c c+a


Bài 13. Cho a,b,c là các số thực dương. Chứng minh
ng

a2 b2 c2 3
+ + ≤ .
a

b + (c + a) c + (a + b) a + (b + c )
2 2 2 2 2 2
5
kh

Bài 14. Cho a,b,c là các số thực dương chứng minh

(a 2
)( )( )
+ 3 b 2 + 3 c 2 + 3 ≥ 4 ( a + b + c + 1) .
2

Bài 15. Cho a,b,c là các số thực dương thỏa mãn điều kiện
1 1 1
+ + ≥1.
a + b +1 b + c +1 c + a +1
Chứng minh rằng a + b + c ≥ ab + bc + ca .
Bài 16. Cho a,b,c là các số thực dương có tích bằng 1. Chứng minh

232
Cty TNHH MTV DVVH Khang Việt

a5 − a 2 b5 − b 2 c5 − c 2
+ + ≤0.
a 5 + b 2 + c 2 b5 + c 2 + a 2 c 5 + a 2 + b 2
Bài 17. Chứng minh rằng với mọi số thực a,b,c ta có
2(1 + abc) + 2(1 + a 2 )(1 + b 2 )(1 + c 2 ) ≥ (1 + a )(1 + b)(1 + c) .
Bài 18. Cho x,y,z,t là các số thực không âm thoả mãn điều kiện
| x − y | + | y − z | + | z − t | + | t − x |=
4.
Chứng minh rằng x 2 + y 2 + z 2 + t 2 ≥ 2 .
Bài 19. Cho a,b,c là các số thực dương thoả mãn điều kiện a 2 + b 2 + c 2 =
3.

n
.v
Chứng minh rằng
5a 2 5b 2 5c 2
+ + ≥3.

om
5a 2 + 4bc + 2 bc 5b 2 + 4ca + 2 ca 5c 2 + 4ab + 2 ab
Bài 20. Cho a, b, c không âm thoả mãn điều kiện a + b + c = 1.
Tìm giá trị nhỏ nhất của biểu thức P = a + b + b + c + c + a .
Bài 21. Cho a,b,c là độ dài 3 cạnh một tam giác.
.c
ok
Chứng minh rằng a +b−c + b+c−a + c+ a −b ≤ a + b + c .
Bài 22. Cho a,b,c là các số thực không âm thoả mãn điều kiện a 2 + b 2 + c 2 =
8.
bo

Chứng minh rằng 4 ( a + b + c ) − abc ≤ 16 .


Bài 23. Cho x,y,z là các số thực thoả mãn điều kiện x 2 + y 2 + z 2 =
1.
et

Tìm giá trị lớn nhất và giá trị nhỏ nhất của biểu thức
vi

P = x3 + y 3 + z 3 − 3 xyz .
Bài 24. Chứng minh rằng với mọi a,b,c là độ dài 3 cạnh một tam giác ta có
ng

a 2b ( a − b ) + b 2 c ( b − c ) + c 2 a ( c − a ) ≥ 0 .
Bài 25. Cho x,y,z là các số thực dương chứng minh
a

1 + yz + zx 1 + xy + zx 1 + xy + yz
kh

+ + ≥1.
(1 + x + y )2 (1 + y + z )2 (1 + z + x )2
Bài 26. Cho a,b,c là các số thực dương thoả mãn điều kiện

( a + b − c )  + −  =4 .
1 1 1
a b c

(  1
a
)
b
1 1 
Chứng minh rằng a 4 + b 4 + c 4  4 + 4 + 4  ≥ 2304 .
c 
Bài 27. Cho a,b,c là các số thực dương thoả mãn điều kiện

233
Khám phá tư duy Kỹ thuật giải bất ĐT Bài toán Max – Min – Đặng Thành Nam

( a + b − c ) 
1 1 1
+ − =4.
a b c

Tìm giá trị nhỏ nhất của biểu thức P = (a 3


)  1 1 1
+ b3 + c 3  3 + 3 + 3  .
a b c 
D. HƯỚNG DẪN GIẢI – ĐÁP SỐ
Bài 1. Ta có

(1 + a )(1 + b ) =( a + b ) + ( ab − 1) ≥ 12 ( a + b + ab − 1)
2 2 2 2 2

⇒ (1 + a )(1 + b ) ≥
1
2
( a + b + ab − 1)
2

n
2

.v
Điều phải chứng minh.
Bài 2. Sử dụng bất đẳng thức C –S ta có

om
 x y z   x y z 
2
( x + y + z) + +  ≥  + +  .
 ( y + z )2 ( z + x )2 ( x + y )2   y + z z + x x + y 
x
+
y
+
z  1
( x + y + z )
= +
1
+
1 
.c
Mặt khác −3
y+z z+x x+ y  y+z z+x x+ y
ok
1  1 1 1 
= ( x + y ) + ( y + z ) + ( z + x )   + + −3
bo

2  y+z z+x x+ y
1 3
≥ (1 + 1 + 1)2 − 3 =
et

2 2
Ta có điều phải chứng minh. Đẳng thức xảy ra khi và chỉ khi x= y= z .
vi

Bài 3. Sử dụng hệ quả bất đẳng thức C –S (Mincopski) ta có


ng

x2 + 1 + y 2 + 1 + z 2 + 1 ≥ ( x + y + z )2 + (1 + 1 + 1)2 .
Vậy ta cần chứng minh ( x + y + z ) + 9 ≥ 6 ( x + y + z ) ⇔ ( x + y + z − 3) ≥ 0
2 2
a

Bất đẳng thức luôn đúng. Đẳng thức xảy ra khi và chỉ khi x= y= z= 1 .
kh

Bài 4. Sử dụng bất đẳng thức C – S ta có


2
 x3 y 3 z 3   x2 y 2 z 2 
 2 + 2 + 2  ( x + y + z ) ≥  + + 
y z x   y z x 
 x2 y 2 z 2 
+ ( y + z + x) ≥ ( x + y + z )
2
 + 
 y z x 
Nhân theo vế 2 bất đẳng thức trên ta có điều phải chứng minh. Đẳng thức xảy ra
khi và chỉ khi x= y= z .

234
Cty TNHH MTV DVVH Khang Việt

x k +1 y k +1 z k +1 xk yk zk
Tổng quát ta có + + ≥ + + ,k ≥ 1.
yk zk xk y k −1 z k −1 x k −1
Bài 5. Sử dụng bất đẳng thức C-S ta được:

( ) ( )
2 2
P 2 = a3 + b3 + c c 2 − ab  = a.a 2 + b.b 2 + c c 2 − ab 
   

( 
) 
( )
2
≤ a 2 + b 2 + c 2  a 4 + b 4 + c 2 − ab 
 
= a 4 + b 4 + c 4 − 2abc 2 + a 2b 2
≤ a 4 + b 4 + c 4 + a 2b 2 + c 2 a 2 + b 2 ( )

n
( ) (a )
2

.v
≤ a 4 + b 4 + c 4 + 2 a 2b 2 + b 2 c 2 + c 2 a 2 = 2
+ b2 + c2 = 1
Với a= 1, b= c= 0 thì P bằng 1; với a = −1, b = c=

om
0 thì P bằng -1.
Vậy giá trị lớn nhất của P bằng 1 đạt tại a= 1, b= c= 0 hoặc các hoán vị.
Giá trị nhỏ nhất của P bằng −1 đạt tại a = −1, b = c= 0 hoặc các hoán vị.
Bài 6. Gọi P là biểu thức vế trái.
.c
Sử dụng bất đẳng thức C-S ta có
ok
( ) ( )
2 2
P 2 = a3 + b3 + c c 2 − ab  = a.a 2 + b.b 2 + c c 2 − ab 
   
bo

( 
) ( )
2
≤ a 2 + b 2 + c 2  a 4 + b 4 + c 2 − ab 
 
( )
et

= 2 a 4 + b 4 + c 4 − 2abc 2 + a 2b 2
.
≤ 2  a 4 + b 4 + c 4 + a 2b 2 + c 2 a 2 + b 2  ( )
vi

 
( )
ng

≤ 2 a + b + c + 2 a b + b c + c a2 
4 4 4 2 2 2 2 2
 

( )
2
=2 a 2 + b 2 + c 2 =8 ⇒ P ≤ 2 2
a
kh

Bất đẳng thức được chứng minh. Đẳng thức xảy ra khi a =
± 2, b =
c=
0 hoặc
các hoán vị.
Bài tập tương tự
Cho x,y,z là các số thực không âm thoả mãn điều kiện x 2 + y 2 + z 2 =
3.
Chứng minh rằng x3 + y 3 + z 3 − xyz ≤ 3 3 .
Bài 7. Sử dụng bất đẳng thức C – S ta có

235
Khám phá tư duy Kỹ thuật giải bất ĐT Bài toán Max – Min – Đặng Thành Nam

a ( 2 − bc ) + 2 ( b + c ) ≤  a 2 + ( b + c )  ( 2 − bc ) + 22 
2 2
  
= ( 2bc + 9 ) ( b2c 2 − 4bc + 8)
Vậy ta cần chứng minh
( 2bc + 9 ) ( b2c 2 − 4bc + 8) ≤ 100 ⇔ ( bc + 2 )2 ( 2bc − 7 ) ≤ 0 .
Bất đẳng thức cuối đúng nếu ta giả sử a 2 = max a 2 , b 2 , c 2 . { }
Thật vậy ta có 2bc ≤ b 2 + c 2 = 9 − a 2 ≤ 9 − 3 = 6 ⇒ 2bc − 7 ≤ −1 .
Bất đẳng thức được chứng minh. Đẳng thức xảy ra chẳng hạn tại a =

n
−1, b =
c=2.

.v
1
giả sử x max { x, y, z} ⇒ x ≥
Bài 9. Không mất tính tổng quả= .
3

om
Sử dụng bất đẳng thức C –S ta có
 9  2 
 9 
x 1 − yz  + y + z ≤
 4 
(x 2
+ ( y + z)
.c 2
) 1 − yz  + 1
 4  
ok
( 2 yz + 1)  
81 2 2 9
= y z − yz + 2 
 16 2 
bo

 81 9 
Ta cần chứng minh ( 2 yz + 1)  y 2 z 2 − yz + 2  ≤ 2
 16 2 
( )
et

⇔ yz 162 y 2 z 2 − 63 yz − 8 ≤ 0

y 2 + z 2 1 − x2 1
vi

Bất đẳng thức luôn đúng vì yz ≤ = ≤ .


2 2 3
ng

Bất đẳng thức được chứng minh. Đẳng thức xảy ra khi và chỉ khi
1
x= y= , z= 0 hoặc các hoán vị.
a

2
kh

Bài 10. Bất đẳng thức cần chứng minh tương đương với
b+c
∑ a ( a + b )( a + c ) ≥ 4 ( a + b + c ) .
cyc

Sử dụng bất đẳng thức C – S ta có


( a + b )( a + c ) ≥ a + bc

( b + c )( b + a ) ≥ b + ca

( c + a )( c + b ) ≥ c + ab

236
Cty TNHH MTV DVVH Khang Việt

Áp dụng và ta cần chứng minh


b+c c+a a+b
bc + ca + ab ≥ 2 ( a + b + c ) .
a b c
Không mất tính tổng quát ta giả sử
bc ca ab
a ≥ b ≥ c ⇒ b + c ≤ c + a ≤ a + b; ≤ ≤ .
a b c
Sử dụng bất đẳng thức Chebyshev ta có
b+c c+a a+b 1  bc ca ab 
bc + ca + ab ≥ ( b + c + c + a + a + b )  + + 
a b c 3  a b c 

n
bc ca ab
.≥ 2 ( a + b + c ) .3
. = 2(a + b + c)

.v
a b c
Bất đẳng thức được chứng minh. Đẳng thức xảy ra khi và chỉ khi a= b= c .

om
Bài 11. Sử dụng bất đẳng thức C – S ta có
a 2 + b2 1 1 a+b
= ≥ = .
( ) + c (a + b) 2 + ( ) 2+ ( ) 2(a + b + c)
2 a +b2 2 c a + b c a + b
.c
a 2 + b2 1
( a + b )2
ok
2
b2 + c2 b+c
Tương tự ta có ≥ .
( )
bo

2 b2 + c2 + a ( b + c ) 2(a + b + c)

c2 + a2 c+a
et


( )
2 c2 + a2 + b ( c + a ) 2(a + b + c)
vi

Cộng theo vế 3 bất đẳng thức trên ta có điều phải chứng minh.
Đẳng thức xảy ra khi và chỉ khi a= b= c .
ng

Bài 12. Gọi P là biểu thức vế trái và sử dụng bất đẳng thức C –S ta có
2a ( a + c ) 2b ( b + a ) 2c ( c + b )
a

P= + +
( a + b )( a + c ) ( b + c )( b + a ) ( c + a )( c + b )
kh

 2a 
≤  ∑ ( a + c )   ∑ 
 ( a + b )( a + c ) 

8 ( a + b + c )( ab + bc + ca )
= .
( a + b )( b + c )( c + a )
8 ( a + b + c )( ab + bc + ca )
Ta chỉ cần chứng minh ≤9
( a + b )( b + c )( c + a )

237
Khám phá tư duy Kỹ thuật giải bất ĐT Bài toán Max – Min – Đặng Thành Nam

8
⇔ ( a + b )( b + c )( c + a ) ≥
( a + b + c )( ab + bc + ca ) ⇔ ( a + b )( b + c )( c + a ) ≥ 8abc
9
Bất đẳng thức cuối luôn đúng theo AM – GM.
Bài 13. Sử dụng bất đẳng thức C –S ta có
a2 a
(1 + 4 ) b2 + ( c + a )2  ≥ b + 2 ( c + a )
2
⇒ ≤ .
b + (c + a)
2 2
5 ( b + 2 a + 2c )
Tương tự cho 2 căn thức còn lại bài toán đưa về chứng minh
a b c 3
+ + ≤
b + 2a + 2c c + 2b + 2a a + 2b + 2c 5
.

n
b + 2c c + 2a a + 2b 9
⇔ + + ≥

.v
b + 2a + 2c c + 2b + 2a a + 2b + 2c 5
Sử dụng bất đẳng thức C –S ta có

om
b + 2c c + 2a a + 2b
+ + ≥
b + 2a + 2c c + 2b + 2a a + 2b + 2c
( ∑ ( b + 2c ) )
.
2
9(a + b + c)
2 .c
9
≥ = =
∑ ( b + 2c )( b + 2a + 2c ) 5 ( a + b + c )2 5
ok
Bất đẳng thức được chứng minh. Đẳng thức xảy ra khi và chỉ khi a= b= c .
Bài 14. Dự đoán đẳng thức đạt tại khi 3 số bằng nhau thay vào bất đẳng thức ta có
bo

a= b= c= 1 .
Ta sử dụng dụng bất đẳng thức C –S
et

2  ( b + c + 1)2 
4 ( a + b + c + 1=

)2 4  a.1 + 3.
b + c +1
 ≤4 a +3
2
( ) 1 +

.

vi

 3  
3

ng

 ( b + c + 1)2 
Vậy ta chứng minh 4 1 +
 3 
(
 ≤ b2 + 3 c2 + 3 )( ) .
 
a

⇔ 3b 2 c 2 + 5b 2 + 5c 2 + 11 ≥ 8bc + 8b + 8c
kh

Bất đẳng thức cuối là tổng của 3 bất đẳng thức sau
( )
3 b 2 c 2 + 1 ≥ 6bc

b 2 + c 2 ≥ 2bc .

( )
4 b 2 + c 2 + 2 ≥ 4 ( 2b + 2c ) = 8b + 8c
Bất đẳng thức được chứng minh. Đẳng thức xảy ra khi và chỉ khi a= b= c= 1 .
Cách 2: Xem bài tập mẫu.
Bài 15. Sử dụng bất đẳng thức C –S ta có

238
Cty TNHH MTV DVVH Khang Việt

( a + b + 1) ( a + b + c 2 ) ≥ ( a + b + c )2 ⇒
1 a + b + c2
≤ .
a + b + 1 ( a + b + c )2

1 b + c + a2 1 c + a + b2
Tương tự ta có ≤ ; ≤ .
b + c + 1 ( a + b + c )2 c + a + 1 ( a + b + c )2
Cộng theo vế ba bất đẳng thức trên và kết hợp với điều kiện ta được:
a 2 + b2 + c2 + 2 ( a + b + c )
≥ 1 ⇔ a + b + c ≥ ab + bc + ca .
( a + b + c )2
Bất đẳng thức được chứng minh. Đẳng thức xảy ra khi và chỉ khi a= b= c= 1 .

n
Bài 16. Bất đẳng thức tương đương với:

.v
1 1 1 3
+ 5 + 5 ≤ 2 .
a +b +c
5 2 2
b +c +a
2 2
c +a +b
2 2
a + b2 + c2

om
Sử dụng bất đẳng thức C –S ta có
.c 1
+ b2 + c2
( )1 
( ) 1
2
a5 + b2 + c 2  + b2 + c 2  ≥ a 2 + b2 + c 2 ⇒ ≤ a .
( )
ok
a  a5 + b2 + c 2 a 2 + b2 + c2
2

Tương tự ta có
bo

1 2 1
+ c + a2 + a 2 + b2
1 b 1 c
≤ ; ≤ .
et

b5 + c 2 + a 2
( ) c5 + a 2 + b 2
( )
2 2
a 2 + b2 + c2 a 2 + b2 + c2
vi

Cộng theo vế ba bất đẳng thức trên và đưa về chứng minh

( ) ( )
ng

1 1 1
+ + + 2 a 2 + b2 + c2 ≤ 3 a 2 + b2 + c2
a b c .
⇔ a 2 + b 2 + c 2 ≥ ab + bc + ca
a

Bất đẳng thức cuối đúng vậy ta có đpcm. Đẳng thức xảy ra khi và chỉ khi
kh

a= b= c= 1 .
Bài 17. Bất đẳng thức tương đương với:
2(1 + a 2 )(1 + b 2 )(1 + c 2 ) ≥ a + b + c + ab + bc + ca − abc − 1 .

( )(
Chú ý 2(1 + a 2 )(1 + b 2 )(1 + c 2 ) = (a + b) 2 + (1 − ab) 2 (1 + c) 2 + (c − 1) 2 . )
Sử dụng bất đẳng thức C –S ta có

239
Khám phá tư duy Kỹ thuật giải bất ĐT Bài toán Max – Min – Đặng Thành Nam

2(1 + a 2 )(1 + b 2 )(1 + c 2 )= [(a + b)2 + (1 − ab)2 ][(1 + c)2 + (c − 1)2 ]


≥ ( a + b )(1 + c ) + ( c − 1)(1 − ab )
= a + b + c + ab + bc + ca − abc − 1
Bất đẳng thức được chứng minh.
Bài 18. Sử dụng bất đẳng thức C –S ta có
x2 + y 2 y 2 + z 2 z 2 + t 2 t 2 + x2
x2 + y 2 + z 2 =
+ t2 + + +
2 2 2 2
( x − y ) 2 ( y − z ) 2 ( z − t ) 2 (t − x) 2
≥ + + +
2 2 2 2

n
(| x − y | + | y − z | + | z − t | + | t − x |)2

.v
≥ =
2
8

om
Bất đẳng thức được chứng minh.
Dấu bằng xảy ra chẳng hạn tại x = z = 1, y = t = 0 .
.c
Bài 19. Sử dụng bất đẳng thức C –S ta có
ok
5a 2 + 4bc
5a 2 + 4bc
= + 2 bc 3. + 2. 2bc
bo

3
 5a 2 + 4bc 
≤ ( 3 + 2 )  + 2bc=

5
(5a 2 + 10bc )
et

 3  3

( )) =
vi

( a +b +c
2 2 2
5
≤ 5a 2 + 5 b 2 + c 2 5
3 3
ng

5a 2 3
⇒ ≥ a2
5a 2 + 4bc + 2 bc a + b2 + c2 2
a

Tương tự rồi cộng lại theo vế ta có đpcm.


kh

Bất đẳng thức được chứng minh. Đẳng thức xảy ra khi và chỉ khi a= b= c .
Bài 20. Ta có P 2= 2 ( a + b + c ) + 2∑ ( a + b )( a + c ) .
Chú ý sử dụng bất đẳng thức C –S ta có
∑ ( a + b )( a + c ) ≥ ∑ ( a + )
bc = a + b + c + ab + bc + ca .

Do đó P 2 ≥ 4 ( a + b + c ) + 2 ( )
ab + bc + ca ≥ 4 ⇒ P ≥ 2 .
Dấu bằng đạt tại a= 1, b= c= 0 hoặc các hoán vị.
Vậy giá trị nhỏ nhất của P bằng 2.

240
Cty TNHH MTV DVVH Khang Việt

Bài 21. Sử dụng bất đẳng thức C – S ta có


a + b − c + b + c − a ≤ 2(a + b − c + b + c − a) =2 b
b + c − a + c + a − b ≤ 2 (b + c − a + c + a − b) =2 c
c + a − b + a + b − c ≤ 2(c + a − b + a + b − c) ≤ 2 a
Cộng theo vế 3 bất đẳng thức trên ta có điều phải chứng minh.
Đẳng thức xảy ra khi và chỉ khi a= b= c .
8
giả sử a max {a, b, c} ⇒ a ≥
Bài 22. Không mất tính tổng quát = .
3

n
Sử dụng bất đẳng thức C –S ta có

.v
a ( 4 − bc ) + 4 ( b + c ) ≤ (a 2
+ (b + c )
2
) (( 4 − bc ) 2
+ 42 )

om
= (8 + 2bc ) ( b2c 2 − 8bc + 32 )

( )
.c
Ta chứng minh ( 8 + 2bc ) b 2 c 2 − 8bc + 32 ≤ 162 ⇔ 2 ( bc − 4 ) b 2 c 2 ≤ 0 .
ok
8
8−
b2 + c2 8 − a 2 3=8
Bất đẳng thức luôn đúng vì bc ≤ = ≤ < 4.
bo

2 2 2 3
Bất đẳng thức được chứng minh.
Đẳng thức xảy ra khi và chỉ khi a= b= 2, c= 0 hoặc các hoán vị.
et

Bài 23. Ta có xy + yz + zx ≤ x 2 + y 2 + z 2 =
1.
vi

(
Khi đó P = ( x + y + z ) x 2 + y 2 + z 2 − xy − yz − zx )
ng

= ( x + y + z )(1 − xy − yz − zx )
⇒ P 2 =(1 + 2 ( xy + yz + zx ) ) (1 − xy − yz − zx )
a

2
kh

= 2 ( xy + yz + zx ) − 3 ( xy + yz + zx ) + 1
3 2

= ( xy + yz + zx )2 ( 2 ( xy + yz + zx ) − 3) + 1 ≤ 1
Vậy giá trị lớn nhất của P bằng 1 đạt tại x= 1, y= z= 0 hoặc các hoán vị.
Giá trị nhỏ nhất của P bằng -1 đạt tại x =
−1, y =
z=0 hoặc các hoán vị.
Bài 24. Đặt a =+
y z , b =+ x y, x, y, z > 0 bất đẳng thức trở thành
z x, c =+

241
Khám phá tư duy Kỹ thuật giải bất ĐT Bài toán Max – Min – Đặng Thành Nam

x3 z + y 3 x + z 3 y ≥ xyz ( x + y + z )
x2 y 2 z 2
⇔ + + ≥ x+ y+z
y z x
 x2 y 2 z 2 
+ ( y + z + x) ≥ ( x + y + z ) .
2
⇔ +
 y z x 
 
Luôn đúng theo C – S.
Bất đẳng thức được chứng minh. Đẳng thức xảy ra khi và chỉ khi a= b= c .
Bài 25. Sử dụng bất đẳng thức C –S ta có

n
 x+ y
1 + z ( x + y )  1 +  ≥ (1 + x + y )
2

 z 

.v
1 + yz + zx 1 z
⇒ ≤ =

om
(1 + x + y ) 1 +
2 x + y x+ y+z
z
Tương tự cho hai biểu thức còn lại rồi cộng lại theo vế ta có đpcm. Đẳng thức
.c
xảy ra khi và chỉ khi x= y= z= 1 .
Bài 26. Theo giả thiết kết hợp sử dụng bất đẳng thức AM – GM ta có
ok
  1 1 
( a + b − c )  
( a + b ) 
1 1 1 1
+ − c= +  + 1 − c ( a + b ) + c  +  
a b  a b   a b 
bo

1 1
( a + b ) 
1 1
≤ (a + b) +  + 1 − 2 + 
a b a b
et

2 .
 
( a + b )  +  − 1
1 1
vi

= 
  a b  
ng

( a + b ) 
1 1 a b a b
Suy ra +  −1 ≥ 2 ⇔ + +2 ≥3⇔ + ≥7.
a b b a b a
a

Khi đó sử dụng bất đẳng thức C –S ta có


kh

2
1   1  
( 4  1
4
a b
14
)
a +b +c  4 + 4 + 4 ≥ 
c  
( 4  1
a
4
)
a + b  4 + 4  + 1
b  
2 2
 a 2 b2   a b  2 
( )
2
=  2 + 2 + 1 =  +  − 1 ≥ 7 2 − 1 = 2304
b   b a 
 a  
Bất đẳng thức được chứng minh. Đẳng thức xảy ra khi và chỉ khi ab = c 2 và
a b
+ = 7.
b a

242
Cty TNHH MTV DVVH Khang Việt

CHỦ ĐỀ 5: KỸ THUẬT SỬ DỤNG BẤT ĐẲNG THỨC


CAUCHY – SCHWARZ DẠNG PHÂN THỨC

A. NỘI DUNG PHƯƠNG PHÁP

B. BÀI TẬP MẪU


Để mở đầu ta xét bài toán quen thuộc sau đây
Bài 1. Chứng minh rằng với mọi số thực dương a,b,c ta có
a b c 3
+ + ≥ .

n
b+c c+a a+b 2

.v
Lời giải
Đây là bất đẳng thức Nebits nổi tiếng có rất nhiều lời giải dành cho nó. Dưới

om
đây tôi đề cập 2 phương pháp tiếp cận cũng chính là kỹ thuật chính chúng ta cần
áp dụng trong chủ đề này.
Cách 1: Cộng thêm vào mỗi hạng tử vế trái với 1 ta được
a
+
b c
+ = 
 a   b
+ 1 + 
  c
+ 1 + 
.c 
+ 1 − 3
b+c c+a a+b b+c  c+a  a+b 
ok
 1 1 1 
= (a + b + c) + + −3
b+c c+a c+a
bo

1  1 1 1 
= ( a + b ) + ( b + c ) + ( c + a )   + + −3
2 a+b b+c c+a
et

2
1 1 1 1  3
≥ a + b. + b + c. + c + a.  −3=
2 
vi

a+b b+c c+a 2


ng

Bất đẳng thức được chứng minh. Đẳng thức xảy ra khi và chỉ khi a= b= c= 1 .
Cách 2: Nhân thêm vào mỗi hạng tử vế trái tương ứng với a,b,c để xuất đại lượng
a

bình phương ta được:


kh

a b c a2 b2 c2
+ + = + +
b + c c + a a + b a (b + c ) b (c + a ) c (c + a )


( a + b + c )2 =
( a + b + c )2 .
a ( b + c ) + b ( c + a ) + c ( c + a ) 2 ( ab + bc + ca )
3 ( ab + bc + ca ) 3
≥ =
2 ( ab + bc + ca ) 2
a b c
Đẳng thức xảy ra khi và chỉ khi = = ⇔ a =b =c .
a (b + c ) b (c + a ) c (c + a )

243
Khám phá tư duy Kỹ thuật giải bất ĐT Bài toán Max – Min – Đặng Thành Nam

Nhận xét. Như vậy 2 kỹ thuật ta cần áp dụng đó là:


+ Cộng thêm vào mỗi phân thức 1 hệ số thích hợp để tất cả các phân thức có dung
tử thức.
+ Nhân thêm vào mỗi phân thức một biến thích hợp nhằm tạo bình phương trình
trên tử thức ở mỗi phân thức.
+ Khi có thêm điều kiện ràng buộc giữa các biến ta cần khéo léo vận dụng C –S
đưa về biến chung.
a 3
Bài 2. Cho a,b,c là các số thực dương thoả mãn điều kiện ≥ .
b+c 2
a b c
Chứng minh rằng + + ≥2.

n
b+c c+a a+b

.v
Lời giải
Sử dụng bất đẳng thức C –S ta có

om
b
+
c

(b + c ) =
2
(b + c ) 2

c + a a + b b ( c + a ) + c ( a + b ) a ( b + c ) + 2bc

( b + c )2
≥ =
1
.c
1 a 1
( b + c )2
a (b + c ) + +
ok
2 b+c 2
Gọi P là biểu thức vế trái ta có
bo

a 1 x ( 2 x − 3)  a 
P≥ + = + 2=
≥ 2,  x .
b+c a
+
1 2 x + 1  b + c 
b+c 2
et

Bất đẳng thức được chứng minh. Đẳng thức xảy ra khi và chỉ khi =
a 3=
b 3c .
vi

Bài 3. Cho x,y,z là các số thực thoả mãn điều kiện x + y + z =0.
x ( x + 2) y ( y + 2) z ( z + 2)
ng

Chứng minh rằng + + ≥0.


2x + 1
2
2y +12
2z2 + 1
a

Lời giải
Giả sử z = max { x , y , z } ta có bất đẳng thức đã cho tương đương với:
kh

x ( x + 2) 1 y ( y + 2) 1 z ( z + 2)
+ + + + −1 ≥ 0
2 x2 + 1 2 2 y2 + 1 2 2z2 + 1


( 2 x + 1)
2
+
( 2 y + 1)
2

2 ( z − 1)
2

2 x2 + 1 2 y2 + 1 2z2 + 1
Sử dụng bất đẳng thức C –S cho vế trái ta có
( 2 x + 1)2 + ( 2 y + 1)2 ≥ ( 2 x + 2 y + 2 )2 =
2 ( z − 1)
.
2

2 x2 + 1 2 y2 + 1 2 x2 + 2 y 2 + 2 x2 + y 2 + 1

244
Cty TNHH MTV DVVH Khang Việt

Vậy ta chỉ cần chứng minh

Bất đẳng thức cuối đúng ta có đpcm.


1
Đẳng thức xảy ra khi và chỉ khi x =
y=− ,z =
1 hoặc các hoán vị.
2
Bài 4. Cho các số thực x, y, z ≥ 1 và thỏa mãn 3 ( x + y + z ) = x 2 + y 2 + z 2 + 2 xy .
x2 x
=
Tìm giá trị nhỏ nhất của biểu thức P + 2 .
( x + y) + x z + x
2

n
Lời giải
 1 

.v
1 4x
Ta có x 2 ≥ x suy ra: P ≥ x  + ≥ .
 ( x + y )2 + x z + x  ( x + y ) + z 2 + 2 x
2 2

om

Theo giả thiết ta có:

( x + y) + z 2 = 3 ( x + y ) + z  ≤ 3 2 ( x + y ) + z 2  ⇒ ( x + y ) + z 2 ≤ 18 .
2 2 2

.c 
2 ( 2 x + 18 ) − 36
ok
4x 18 18 1
Suy ra P ≥ = = 2− ≥ 2− = .
18 + 2 x 2 x + 18 x+9 1+ 9 5
bo

1
Vậy giá trị nhỏ nhất của P bằng đạt tại= x 1,= y 2,=
z 3.
5
Bài 5. Cho a,b,c là các số thực dương thỏa mãn điều kiện a 2 + b 2 + c 2 =
3.
et

a2 b2 c2
Tìm giá trị nhỏ nhất của biểu thức P = + + .
vi

b + 2c c + 2a a + 2b
ng

Lời giải
Phân tích tìm lời giải:
Mỗi phân thức có dạng bình phương trên tử thức tuy nhiên ta cần sử dụng triệt
a

để điều kiện giả thiết .


kh

Ta xử lý như sau:

245
Khám phá tư duy Kỹ thuật giải bất ĐT Bài toán Max – Min – Đặng Thành Nam

Theo giả thiết ta có:

Sử dụng bất đẳng thức AM – GM cho 3 số dương ta được

Cộng theo vế 3 bất đẳng thức trên ta được

n
.

.v
Suy ra .

om
Mặt khác .

Suy ra . .c
Đẳng thức xảy ra khi và chỉ khi .
ok
Bài 6. Cho a,b,c là các số thực dương. Chứng minh rằng
bo

Lời giải
et

Viết lại bất đẳng thức dưới dạng


vi

.
ng

Gọi P là biểu thức vế trái ta có


a
kh

Bất đẳng thức được chứng minh. Đẳng thức xảy ra khi và chỉ khi .
Bài 7. Cho x,y,z là các số thực dương thoả mãn điều kiện .

246
Cty TNHH MTV DVVH Khang Việt

Chứng minh rằng .

Lời giải
Theo giả thiết ta có .
Sử dụng bất đẳng thức C-S ta có

Tương tự ta có

n
.v
om
Cộng theo vế 3 bất đẳng thức trên đưa về chứng minh
.c .
ok
Đặt đưa về bất đẳng thức
bo

.
et

Bất đẳng thức này đã được chứng minh. Đẳng thức xảy ra khi và chỉ khi
vi

Cách 2: Sử dụng bất đẳng thức C-S ta có


a ng
kh

Tương tự ta có

247
Khám phá tư duy Kỹ thuật giải bất ĐT Bài toán Max – Min – Đặng Thành Nam

Cộng theo vế 3 bất đẳng thức trên ta có điều phải chứng minh.
Nhận xét. Như vậy linh hoạt sử dụng điều kiện cũng như sử dụng C-S hợp lý ta có
lời giải ngắn gọn cho bài toán như cách 2.
Bài 8. Cho x,y,z là các số thực dương có tổng bằng 1.

Chứng minh rằng .

Lời giải
Bất đẳng thức có dạng mạnh hơn tôi đã đề cập trong chương 1 bất đẳng thức
AM – GM

n
.

.v
Dưới đây ta chứng minh bằng C –S cho bài toán trên.

om
Ta có .

Chú ý
.c
ok
bo
et

Từ đó suy ra điều phải chứng minh.


vi

Đẳng thức xảy ra khi và chỉ khi .


ng

Bài 9. Cho x,y,z là các số thực dương thoả mãn điều kiện .
a

Chứng minh rằng .


kh

Lời giải
Sử dụng bất đẳng thức C – S ta có

Vậy ta chứng minh .

Đặt .

248
Cty TNHH MTV DVVH Khang Việt

Bất đẳng thức trở thành (luôn đúng).

Bất đẳng thức được chứng minh. Đẳng thức xảy ra khi và chỉ khi .
Bài 10. Cho a,b,c là các số thực dương thoả mãn điều kiện .

Chứng minh rằng

Lời giải

Theo giả thiết ta có .

n
.v
Suy ra .

om
Tương tự ta có

.c
ok
Cộng theo vế 3 bất đẳng thức trên ta cần chứng minh bất đẳng thức
bo

Gọi P là biểu thức vế trái viết lại P và sử dụng bất đẳng thức C – S dạng phân
et

thức ta có
vi
a ng
kh

Bất đẳng thức được chứng minh. Đẳng thức xảy ra khi và chỉ khi .
Bài 11. Cho a,b,c là các số thực dương có tích bằng 1 chứng minh

Lời giải

Theo giả thiết tồn tại các số dương x, y, z thoả mãn .

249
Khám phá tư duy Kỹ thuật giải bất ĐT Bài toán Max – Min – Đặng Thành Nam

Bất đẳng thức trở thành

Sử dụng bất đẳng thức C –S ta có

n
Vậy ta chỉ cần chứng minh

.v
om
.c
Bất đẳng thức đúng ta có đpcm. Đẳng thức xảy ra khi và chỉ khi .
Bài 12. Cho a,b,c là các số thực dương có tổng bằng 3 chứng minh
ok

.
bo

Lời giải
Sử dụng bất đẳng thức AM – GM ta có
et
vi

.
ng

Tương tự ta có
a

.
kh

Ta chỉ cần chứng minh

Sử dụng bất đẳng thức C –S cho vế trái ta được .

Vậy ta chứng minh

250
Cty TNHH MTV DVVH Khang Việt

Bất đẳng thức cuối đúng. Đẳng thức xảy ra khi và chỉ khi .
Bài 13. Cho a,b,c là các số thực dương thoả mãn điều kiện .

Chứng minh rằng .

Lời giải

n
Do a,b,c dương và nên tồn tại các số dương thoả mãn

.v
.

om
Bất đẳng thức được viết lại dưới dạng
.c .
ok
Gọi P là biểu thức vế trái của bất đẳng thức trên viết lại P và sử dụng bất đẳng
thức C – S ta có
bo
et
vi

Đặt
ng

.
Sử dụng bất đẳng thức C – S ta có
a
kh

Mặt khác sử dụng bất đẳng thức quen thuộc ta có

Do đó .

251
Khám phá tư duy Kỹ thuật giải bất ĐT Bài toán Max – Min – Đặng Thành Nam

Bất đẳng thức được chứng minh. Đẳng thức xảy ra khi và chỉ khi .
Bài 14. Cho a,b,c là các số thực dương thoả mãn điều kiện .

Chứng minh rằng .

Lời giải
Gọi P là biểu thức vế trái viết lại P và sử dụng bất đẳng thức C – S ta có

n
.v
om
.c
Vậy ta cần chứng minh .
ok
Thật vậy giả sử ngược lại .
Sử dụng bất đẳng thức Schur bậc 3
bo

Ta có
et
vi
a ng
kh

Vô lý. Vậy .
Bất đẳng thức được chứng minh. Đẳng thức xảy ra khi và chỉ khi .
Bài 15. Cho a,b,c là các số thực không âm thoả mãn điều kiện .

Chứng minh rằng .

Lời giải
Gọi P là biểu thức vế trái sử dụng bất đẳng thức C- S ta có

252
Cty TNHH MTV DVVH Khang Việt

a3 b3 c3
P= + +
a 2 ( b2 + c2 ) b2 ( c2 + a 2 ) c2 ( a 2 + b2 )

( )
2
a 3 + b3 + c 3

2 ( a 2b 2 + b 2 c 2 + c 2 a 2 )
Vậy ta chỉ cần chứng minh

n
.v
om
Sử dụng bất đẳng thức C – S ta có
.

Vậy ta chỉ cần chứng minh


.c
ok
bo

Đặt
et
vi

Ta cần chứng minh .


ng

Không mất tính tổng quát giả sử ta có


a
kh

Chú ý khảo sát hàm số trên đoạn ta có

điều phải chứng minh.


Bất đẳng thức được chứng minh. Đẳng thức xảy ra khi và chỉ khi
hoặc các hoán vị.

253
Khám phá tư duy Kỹ thuật giải bất ĐT Bài toán Max – Min – Đặng Thành Nam

Bài 16. Cho a,b,c là các số thực không âm thoả mãn điều kiện .
Chứng minh rằng

Lời giải
Gọi P là biểu thức vế trái và sử dụng bất đẳng thức C –S ta có:

n
.v
om
Vậy ta chỉ cần chứng minh
2 ≥ 1 + 3abc + 4 ( a 2b + b 2 c + c 2 a + ab 2 + bc 2 + ca 2 )
.c
⇒ (a + b + c)3 ≥ 3abc + ab ( a + b ) + bc ( c + a ) + ca ( c + a )
ok
Bất đẳng thức cuối chính là bất đẳng thức Schur bậc 3.

Bài toán được chứng minh. Đẳng thức xảy ra khi và chỉ khi
bo

hoặc

và các hoán vị.


et
vi

C. BÀI TẬP RÈN LUYỆN


Bài 1. Cho x,y là hai số thực dương thỏa mãn điều kiện x + y =
ng

1.
x y
Tìm giá trị nhỏ nhất của biểu =
thức P + .
a

y +1
2
x +1
2
kh

Bài 2. Cho x,y là hai số thực dương thay đổi thỏa mãn điều kiện 2 x + 3 y =
5.
Tìm giá trị nhỏ nhất của biểu thức

(1 + x )(1 + y ) − 1
2 2
(1 + x )(1 + y ) − 1
3 3

=P + .
2y 3x 2
Bài 3. Cho . Tìm giá trị nhỏ nhất của biểu thức

254
Cty TNHH MTV DVVH Khang Việt

Bài 4. Cho x, y, z là các số thực dương. Chứng minh rằng


x y z 3
+ + ≥ .
3x + 7 ( y + z ) 3y + 7 ( z + x) 3z + 7 ( x + y ) 17
Bài 5. (TSĐH Khối A 2007) Cho x, y, z là các số thực dương thoả mãn điều kiện
.
Tìm giá trị nhỏ nhất của biểu thức

Bài 6. Chứng minh rằng với mọi a,b,c là các số thực dương ta có

n
.v
.

om
Bài 7. Cho a,b,c là các số thực dương thoả mãn điều kiện .

Chứng minh rằng .c .

Bài 8. Cho x,y,z là các số thực dương thoả mãn điều kiện .
ok
Tìm giá trị nhỏ nhất của biểu thức
bo

Bài 9. Cho x,y,z là các số thực dương thoả mãn điều kiện .
et

Chứng minh rằng


vi

.
ng

Bài 10. Cho x,y,z là các số thực dương thoả mãn điều kiện .
a

Chứng minh rằng


kh

Bài 11. Cho a,b,c là các số thực dương. Chứng minh rằng

Bài 12. Cho a,b,c là các số thực dương. Tìm giá trị nhỏ nhất của biểu thức

255
Khám phá tư duy Kỹ thuật giải bất ĐT Bài toán Max – Min – Đặng Thành Nam

Bài 13. Cho a,b,c là các số thực dương. Tìm giá trị nhỏ nhất của biểu thức

Bài 14. Cho a,b,c là các số thực dương. Tìm giá trị nhỏ nhất của biểu thức

Bài 15. Cho a,b,c là các số thực dương. Chứng minh rằng

n
Bài 16. Cho a,b,c là các số thực dương. Tìm giá trị nhỏ nhất của biểu thức

.v
.

om
Bài 17. Cho a,b,c là các số thực dương. Tìm giá trị nhỏ nhất của biểu thức

.c .

Bài 18. Cho a,b,c,x,y,z là các số thực dương.


ok
Chứng minh rằng
a b c 3( xy + yz + zx)
( y + z) + ( z + x) + ( x + y) ≥
bo

b+c c+a a+b x+ y+z


Bài 19. Cho x,y,z là các số thực dương và k số thực, .
et

Chứng minh rằng .


vi

Bài 20. Chứng minh với a,b,c là các số thực dương thoả mãn điều kiện ta có
ng

.
a

Bài 21. Chứng minh với mọi a,b,c là các số thực dương ta có
kh

Bài 22. Cho a,b,c là các số thực dương có tổng bằng 3.

Chứng minh rằng .

Bài 23. Chứng minh với mọi a,b,c dương ta có

256
Cty TNHH MTV DVVH Khang Việt

Bài 24. Cho a,b,c là các số thực dương thoả mãn điều kiện
.

Chứng minh rằng .

Bài 25. Cho a,b,c là các số thực dương có tích bằng 1.

Chứng minh rằng .

Bài 26. Cho a,b,c là độ dài 3 cạnh một tam giác. Chứng minh

n
.

.v
Bài 27. Cho a,b,c là các số thực dương thỏa mãn điều kiện ab + bc + ca =
1.

om
1 1 1 9
Chứng minh rằng + + ≥ .
a b +c
2 2
b c +a
2 2
c a +b
2 2
2 2 − 3 6abc
Bài 28. Cho a,b,c là các số thực không âm thoả mãn điều kiện
.c.
ok
Chứng minh rằng (a + b + c) 
1 1 1 
+ +  ≥ a + b + c +1
a+b b+c c+a
bo

Bài 29. Cho a,b,c là các số thực dương thoả mãn điều kiện .
et

Chứng minh rằng .

Bài 30. Cho a,b,c là các số thực dương. Chứng minh


vi
ng

Bài 31. Cho a,b,c là các số thực dương có tích bằng 1. Chứng minh
a

.
kh

Bài 32. Cho a,b,c là các số thực dương. Chứng minh

Bài 33. Cho a,b,c là 3 số thực dương.


Chứng minh rằng

257
Khám phá tư duy Kỹ thuật giải bất ĐT Bài toán Max – Min – Đặng Thành Nam

Bài 34. Cho x,y là hai số thực dương có tổng bằng 1.

Chứng minh rằng .

Bài 35. Cho x,y là hai số thực dương. Chứng minh


1 1 4 32 ( x 2 + y 2 )
+ + ≥ .
x2 y 2 x2 + y 2 ( x + y)
4

Bài 36. Cho x,y là các số thực dương thoả mãn điều kiện
.

n
.v
Tìm giá trị nhỏ nhất của biểu thức .

Bài 37. Cho a,b,x,y,z là các số thực dương. Chứng minh

om
.

Bài 38. Cho x,y,z là các số thực dương. Chứng minh


.c
ok
.

Bài 39. Cho a,b,c là các số thực dương thỏa mãn a + b + c =


1.
bo

Tìm giá trị nhỏ nhất của biểu thức


a 3 + 2b3 b3 + 2c 3 c 3 + 2a 3
P= + + .
et

4ab + 3a 2 4bc + 3b 2 4ca + 3c 2


Bài 40. Chứng minh rằng với mọi a, b, c dương ta có
vi

.
ng

A. HƯỚNG DẪN GIẢI – ĐÁP SỐ


a

Bài 1. Sử dụng bất đẳng thức C-S dạng phân thức ta có:
kh

( x + y)
2
x y x2 y2
P= + = + ≥
y2 +1 x2 + 1 x y2 +1 y x2 + 1 x y 2 + 1 + y x2 + 1
1 1

x . xy 2 + x + y . yx 2 + y ( x + y ) ( xy 2 + x + yx 2 + y )
1 1 1 2
= = ≥ =
xy ( x + y ) + x + y 1 + xy  x+ y 5 2

1+  
 2 

258
Cty TNHH MTV DVVH Khang Việt

1
Đẳng thức xảy ra khi và chỉ khi x= y= .
2
2 1
Vậy giá trị nhỏ nhất của P bằng đạt tại x= y= .
5 2
Bài 2. Sử dụng bất đẳng thức C-S ta được:

(1 + x )(1 + y ) ≥ 1 + xy, (1 + x )(1 + y ) ≥ 1 + xy


2 2 3 3
xy .

xy xy xy 1  y
Suy ra P ≥ + =  3 x + 2 y .
2y 3x 2
6 x 

n
Nhận xét. Vế phải của bất đẳng thức bậc 1 và điều kiện bậc 1 nên sử dụng kỹ thuật

.v
y y y
3x + 2 y 3+ 2

om
5 5
x = x x .
đồng bậc: P ≥ . .
6 2x + 3y 6 2 + 3y
x

=
Đặt t
y
, ( t > 0 ) khi đó
.c
ok
x
5 3 + 2t 3 5  3 + 2t 3  5 5 ( t − 1) ( 2t + 1) 5 5
2

P≥ .=  −=1 + . + ≥ .
bo

6 2 + 3t 2 6  2 + 3t 2  6 6 2 + 3t 2 6 6
Đẳng thức xảy ra khi và chỉ khi t =1 ⇔ x = y ⇒ x = y =1 .
et

5
Vậy giá trị nhỏ nhất của P bằng đạt tại x= y= 1 .
vi

6
Bình luận. Dự đoán được điểm rơi x= y= 1 ta có thể đánh giá nhanh bằng bất
ng

đẳng thức AM-GM như sau:

1 y  1 y y  1 y y  2x + 3y 5
a

P ≥  3x + 2 y =   2 x + x + y + y  ≥  2 x + 3 3 xy y =  = .
6 x  6 x x  6 x x 6 6
 
kh

Bài 3. Sử dụng bất đẳng thức C-S ta có

Đẳng thức xảy ra khi và chỉ khi .

Suy ra .

Sử dụng bất đẳng thức C-S ta có

259
Khám phá tư duy Kỹ thuật giải bất ĐT Bài toán Max – Min – Đặng Thành Nam

Suy ra

n
.v
Đẳng thức xảy ra khi và chỉ khi

om
.

Vậy giá trị nhỏ nhất của P bằng 24 đạt tại


.c .
Bài tập tương tự
ok
1) Cho . Tìm giá trị nhỏ nhất của biểu thức
bo

Bài 4. Sử dụng bất đẳng thức C-S dạng phân thức ta có


et

x2 y2 z2
VT = 2 + +
vi

3x + 7 ( y + z ) x 3 y 2 + 7 ( z + x ) y 3z 2 + 7 ( x + y ) z
ng

( x + y + z) ( x + y + z)
2 2

≥ =
3 ( x + y + z ) + 14 ( xy + yz + zx ) 3 ( x + y + z ) + 8 ( xy + yz + zx )
2 2 2 2
a

( x + y + z)
2
3
kh

≥ =
8
3 ( x + y + z ) + ( x + y + z ) 17
2 2

3
Đẳng thức xảy ra khi và chỉ khi x= y= z .
Bài 5. Sử dụng bất đẳng thức AM – GM cho 2 số dương ta được

Suy ra .

260
Cty TNHH MTV DVVH Khang Việt

Tương tự ta có

Cộng theo vế 3 bất đẳng thức trên ta được

n
Đặt .

.v
Ta có
2a 2b 2c

om
P≥ + +
b + 2c c + 2a b + 2b
 a2 b2 c2 
= 2 + + 
 a ( b + 2c ) b ( c + 2a ) c ( a + 2b ) 
.c
2(a + b + c)
ok
2


a ( b + 2c ) + b ( c + 2a ) + c ( a + 2b )
bo

2(a + b + c) 6 ( ab + bc + ca )
2

= ≥ =2
3 ( ab + bc + ca ) 3 ( ab + bc + ca )
et

Vậy giá trị nhỏ nhất của P bằng 1 đạt tại .


Bài 6. Sử dụng bất đẳng thức AM –GM cho 2 số dương ta được
vi
a ng
kh

Tương tự ta có

Cộng theo vế 3 bất đẳng thức trên và gọi P là biểu thức vế trái của bất đẳng thức
ta được
261
Khám phá tư duy Kỹ thuật giải bất ĐT Bài toán Max – Min – Đặng Thành Nam

Bất đẳng thức được chứng minh. Đẳng thức xảy ra khi và chỉ khi .
Bài 8. Sử dụng bất đẳng thức AM – GM cho 2 số dương ta được

n
Tương tự ta có

.v
om
.

Cộng theo vế 3 bất đẳng thức trên ta được


.c
ok
.
bo

Đẳng thức xảy ra khi và chỉ khi . Vậy giá trị nhỏ nhất của P bằng 2.
et

Cách 2: Viết lại P dưới dạng


vi

.
ng

Chú ý sử dụng AM –GM ta có


a
kh

Cộng theo vế 2 bất đẳng thức trên có kết quả tương tự trên.
Bài 9. Gọi P là biểu thức vế trái, với giả thiết suy ra

Sử dụng bất đẳng thức C – S dạng phân thức ta có

262
Cty TNHH MTV DVVH Khang Việt

Ta cần chứng minh

Bất đẳng thức trên là tổng của các bất đẳng thức

n
.v
om
Chứng minh (1).
Sử dụng bất đẳng thức AM – GM cho 3 số dương ta có
.c
ok
bo

Chứng minh (2).


Sử dụng bất đẳng thức AM – GM cho 4 số dương ta được
et
vi

.
ng

Cộng theo vế 3 bất đẳng thức trên ta có điều phải chứng minh.
Chứng minh (3).
a

Bất đẳng thức tương đương với


kh

.
Bài toán được chứng minh. Đẳng thức xảy ra khi và chỉ khi .
Bài 10. Gọi P là biểu thức vế trái của bất đẳng thức.
Viết lại P và sử dụng bất đẳng thức C – S ta có

263
Khám phá tư duy Kỹ thuật giải bất ĐT Bài toán Max – Min – Đặng Thành Nam

Vậy ta chỉ cần chứng minh

n
.

.v
Gọi Q là biểu thức vế trái của bất đẳng thức viết lại Q và sử dụng bất đẳng thức

om
C-S ta có

.c
ok
bo

Chú ý

x 2 y 2 + z 2 + y 2 z 2 + x 2 + z=
2
x2 + y 2 ∑ x2 x2 ( y 2 + z 2 )
et

≤ ( x + y + z )  x ( y
2 2 2 2 2
+ z 2 ) + y 2 ( z 2 + x 2 ) + z 2 ( x 2 + y 2 ) 
vi

= 2 ( x + y + z )( x y
2 2 2 2 2
+ y2 z 2 + z 2 x2 )
ng

(
2 2
x + y 2 + z 2 )( x 2 + y 2 + z 2 )
2

a

3
kh

Suy ra

Bất đẳng thức được chứng minh. Đẳng thức xảy ra khi và chỉ khi .
Bài 11. Viết lại bất đẳng thức dưới dạng

264
Cty TNHH MTV DVVH Khang Việt

Gọi P là biểu thức vế trái của bất đẳng thức ta có

n
.v
Bất đẳng thức được chứng minh. Đẳng thức xảy ra khi và chỉ khi .

om
Bài 12. Ta có:

.c
ok
bo
et
vi
ng

Đẳng thức xảy ra khi và chỉ khi .


a

Bài 13.
Phân tích tìm lời giải:
kh

Tương tự bài toán trên ta cần thêm vào mỗi phân thức các hằng số thích hợp để
có cả 3 phân thức có chung tử thức.
Ta chọn x,y dương sao cho:

265
Khám phá tư duy Kỹ thuật giải bất ĐT Bài toán Max – Min – Đặng Thành Nam

Vậy ta thêm vào

Vậy ta có P

n
.v
om
Bài 14. Ta có:
 3(b + c )   4a + 3c  12 ( b − c ) 
=P  + 2 +  + 1 +  .c + 8 − 11
 2a   3b   2a + 3c 
ok
 1 1 4 
= ( 4a + 3b + 3c )  + +  − 11
 2a 3b 2a + 3c 
bo

 1 1 4 
=  2a + 3b + ( 2a + 3c )   + +  − 11
 2a 3b 2a + 3c 
et

≥ (1 + 1 + 2 ) − 11 =5
2
vi

đẳng thức xảy ra khi và chỉ khi .


ng

Bài 15. Gọi P là biểu thức vế trái của bất đẳng thức.
Ta có
a

b+c   2a + c   4 ( a + b ) 
=
P  + 2 +  + 1 +  + 4 − 7
kh

 a   b   a+c 

( 2a + b + c ) 
1 1 4 
= + + −7
a b a+c
1 1 4 
≥ ( a + b + ( a + c ))  + +  − 7 ≥ (1 + 1 + 2 ) − 7 =9
2

a b a+c

Đẳng thức xảy ra khi và chỉ khi .

Bài 16. Ta có

266
Cty TNHH MTV DVVH Khang Việt

n
Đẳng thức xảy ra khi và chỉ khi .

.v
Bài 17. Ta có

om
 9b + 16c   25 ( 4a + 16c )   64 ( 4a + 9b ) 
=P  + 4 +  + 9.25 +  + 64.16  − 1253
 a   b   .c c 

( 4a + 9b + 16c ) 
1 25 64 
= + +  − 1253 ≥ ( 2 + 3.5 + 4.8 ) − 1253 = 1148
2
ok
a b c 
Đẳng thức xảy ra khi và chỉ khi .
bo

Bài 18. Gọi P là biểu thức vế trái của bất đẳng thức.
Ta viết lại P và sử dụng bất đẳng thức C –S ta có
et
vi
a ng
kh

Vậy ta chứng minh

267
Khám phá tư duy Kỹ thuật giải bất ĐT Bài toán Max – Min – Đặng Thành Nam

∑ ( x + y + z) ( )
2
x 2 + xy + yz + zx ≥ + 3 xy + yz + zx
2
Ta có

= ( x + y + z) + 9 ( xy + yz + zx )
2

Vậy ta chứng minh

n
Bất đẳng thức luôn đúng.

.v
Bất đẳng thức được chứng minh. Đẳng thức xảy ra khi và chỉ khi
.

om
Bài 19. Gọi P là biểu thức vế trái, viết lại P và sử dụng bất đẳng thức C – S ta có

.c
ok
bo
et

Vậy ta chứng minh


( k + 1)( x + y + z ) ≥ 9  x ( x 2 + kyz ) + y ( y 2 + kzx ) + z ( z 2 + kxy ) 
3
vi

⇔ ( k − 8 ) ( x3 + y 3 + z 3 ) + 3 ( k + 1)( x + y )( y + z )( z + x ) ≥ 27 kxyz
ng

Bất đẳng thức cuối luon đúng dựa vào AM – GM.


Bất đẳng thức được chứng minh. Đẳng thức xảy ra khi và chỉ khi .
a

Bài 20. Gọi P là biểu thức vế trái, viết lại P và sử dụng bất đẳng thức C – S ta được
kh

268
Cty TNHH MTV DVVH Khang Việt

Bất đẳng thức được chứng minh. Đẳng thức xảy ra khi và chỉ khi .

Bài 21. Đặt bất đẳng thức trở thành

Sử dụng bất đẳng thức AM – GM ta có

n
.v
om
.c
Cộng theo vế 3 bất đẳng trên kết hợp sử dụng C – S ta có
ok
.
bo

Bất đẳng thức được chứng minh. Đẳng thức xảy ra khi và chỉ khi .
Bài tập tương tự
Cho a,b,c là các số thực dương. Chứng minh rằng
et
vi

Bài 22. Gọi P là biểu thức vế trái của bất đẳng thức và sử dụng AM – GM ta có
a ng
kh

Cộng theo vế 3 bất đẳng thức trên ta được

Vậy ta cần chứng minh


269
Khám phá tư duy Kỹ thuật giải bất ĐT Bài toán Max – Min – Đặng Thành Nam

Sử dụng bất đẳng thức C –S ta có

Vậy ta chứng minh

n
.v
om
.c
ok
bo

Bất đẳng thức được chứng minh. Đẳng thức xảy ra khi và chỉ khi .
Bài tập tương tự
Cho a,b,c là các số thực dương có tổng bằng 3.
et

Chứng minh rằng


vi

.
ng

Bài 23. Gọi P là biểu thức vế trái sử dụng bất đẳng thức C –S ta có

(a + b2 + c2 )
a

2 2

P≥
kh

a b 2 − bc + c 2 + b c 2 − ca + a 2 + c a 2 − ab + b 2
(a + b2 + c2 )
2 2


( a + b + c )  a ( b2 − bc + c 2 ) + b ( c 2 − ca + a 2 ) + c ( a 2 − ab + b2 )
Ta cần chứng minh

(a + b 2 + c 2 ) ≥ ( a + b + c )  a ( b 2 − bc + c 2 ) + b ( c 2 − ca + a 2 ) + c ( a 2 − ab + b 2 ) 
2 2

⇔ a 4 + b 4 + c 4 + abc ( a + b + c ) ≥ ab ( a 2 + b 2 ) + bc ( b 2 + c 2 ) + ca ( c 2 + a 2 )

270
Cty TNHH MTV DVVH Khang Việt

Bất đẳng thức cuối luôn đúng(xem chủ đề chứng minh tương đương).
Bất đẳng thức được chứng minh. Đẳng thức xảy ra khi và chỉ khi .

Bài 24. Đặt .

Bất đẳng thức trở thành .

Gọi P là biểu thức vế trái và sử dụng bất đẳng thức C –S ta có

n
.v
om
Mặt khác sử dụng bất đẳng thức AM – GM ta có

.c
ok
bo
et

Tương tự ta có
vi
a ng

Cộng theo vế 3 bất đẳng thức trên từ đó suy ra


kh

Bất đẳng thức được chứng minh. Đẳng thức xảy ra khi và chỉ khi .
Bài 25. Sử dụng bất đẳng thức C – S ta có

Tương tự ta suy ra

271
Khám phá tư duy Kỹ thuật giải bất ĐT Bài toán Max – Min – Đặng Thành Nam

Bất đẳng thức được chứng minh. Đẳng thức xảy ra khi và chỉ khi .
Bài 26. Gọi P là biểu thức vế trái ta có

n
.v
om
.c
ok

Bất đẳng thức được chứng minh. Đẳng thức xảy ra khi và chỉ khi
bo

.
Bài 27.
Sử dụng bất đẳng thức C-S ta có
et

1 1 1 9
+ + ≥ .
vi

a b +c
2 2
b c +a
2 2
c a +b
2 2
a b + c + b c + a 2 + c a 2 + b2
2 2 2

Tương tự ta có
a ng
kh

1 1 1 9
Vậy + + ≥
a b +c
2 2
b c +a2 2
c a +b
2 2
6 − 12abc ( a + b + c )
Bất đẳng thức được chứng minh nếu ta chứng minh được

272
Cty TNHH MTV DVVH Khang Việt

9 9
≥ ⇔ 2 2 − 3 6abc ≥ 6 − 12abc ( a + b + c ) .
6 − 12abc ( a + b + c ) 2 2 − 3 6abc
1
Luôn đúng do a + b + c ≥ 3 ( ab + bc +=
ca ) 3; abc ≤ .
3 3
1
Đẳng thức xảy ra khi và chỉ khi a= b= c= .
3
Bài 28. Bất đẳng thức tương đương với

n
.v
om
Sử dụng bất đẳng thức C –S ta có

.c
ok
bo
et
vi

Bất đẳng thức được chứng minh đẳng thức xảy ra khi và chỉ khi
ng

hoặc các hoán vị.


Bài 29. Sử dụng bất đẳng thức C –S ta có
a
kh

Vậy ta chỉ cần chứng minh .

Sử dụng bất đẳng thức AM – GM ta có .

273
Khám phá tư duy Kỹ thuật giải bất ĐT Bài toán Max – Min – Đặng Thành Nam

a+b+c = ( a + b + c ) ( a 2 + b2 + c2 )
= ( a 3 + ac 2 ) + ( b3 + ba 2 ) + ( c 3 + cb 2 ) + ab 2 + bc 2 + ca 2
≥ 2 ( a 2 c + b 2 a + c 2b ) + ab 2 + bc 2 + ca 2= 3 ( ab 2 + bc 2 + ca 2 )

Suy ra .

Cộng theo vế 2 bất đẳng thức trên ta có điều phải chứng minh. Đẳng thức xảy ra

khi và chỉ khi .

n
.v
Bài 30. Do tính thuần nhất của bất đẳng thức chuẩn hoá cho .

om
Ta cần chứng minh .

Sử dụng bất đẳng thức C –S và gọi P là biểu thức vế trái ta có


.c
ok
.
bo

Vậy ta chỉ cần chứng minh

.
et

Thật vậy sử dụng bất đẳng thức C –S ta có


vi

a b+c +b c+a +c a+b ≤ ( a + b + c )  a ( b + c ) + b ( c + a ) + c ( a + b )


ng

3
= 3 ( ab + bc + ca ) ≤ a + b=
+c
2
Bất đẳng thức được chứng minh. Đẳng thức xảy ra khi và chỉ khi
a

.
Bài 31. Bất đẳng tương đương với
kh

Gọi P là biểu thức vế trái và sử dụng bất đẳng thức C –S ta có

Vậy ta cần chứng minh

274
Cty TNHH MTV DVVH Khang Việt

Sử dụng bất đẳng thức C –S ta có

Bất đẳng thức được chứng minh. Đẳng thức xảy ra khi và chỉ khi .

Bài 32. Đặt và .

n
.v
Bất đẳng thức trở thành: .

om
Theo bất đẳng thức C –S ta có:
.c .
ok
Vì vậy ta chỉ cần chứng minh
( x + y + z )2 3
bo

≥ 2
k ( x + y + z ) + (k + 1)( xy + yz + zx) k + k + 1
2 2 2 2

⇔ (k − 1) 2 ( x 2 + y 2 + z 2 − xy − yz − zx) ≥ 0
et

⇔ ( k − 1) ( x − y ) + ( y − z ) + ( z − x )  ≥ 0
2 2 2 2
 
vi

Bất đẳng thức cuối đúng ta có đpcm.


ng

Bài 33. Đặt ta có: .


a

Bất đẳng thức đã cho trở thành:


kh

Áp dụng bất đẳng thức C –S ta có:

Bất đẳng thức được chứng minh. Đẳng thức xảy ra khi và chỉ khi .

Bài 34. Ta có .

275
Khám phá tư duy Kỹ thuật giải bất ĐT Bài toán Max – Min – Đặng Thành Nam

Bất đẳng thức được chứng minh. Đẳng thức xảy ra khi và chỉ khi .

Bài 35.
Ta có:

(x + y 2 ) + 4 x2 y 2 ( x + y + 2 xy )
2 2
( x + y)
2 2 2 4
1 1 4
+= + 2 ≥ = .
x + y2 x2 y 2 ( x2 + y 2 ) 2x y ( x + y ) 2x2 y 2 ( x2 + y 2 )
2 2 2 2 2 2
x y
Ta chứng minh
( x + y)
4
32 ( x 2 + y 2 )
⇔ ( x + y ) ≥ 64 x 2 y 2 ( x 2 + y 2 ) .
2

8

(x )

n
+y ( x + y)
2 2 2 2 4
2x y

.v
⇔ ( x + y ) ≥ 8 xy ( x 2 + y 2 ) ⇔ (( x + y 2 ) + 2 xy ) ≥ 8 xy ( x 2 + y 2 ) .
4 2
2

om
⇔ ( x 2 + y 2 ) − 4 xy ( x 2 + y 2 ) + 4 x 2 y 2 ≥ 0 ⇔ ( x − y ) ≥ 0 (luôn đúng).
2 4

Bất đẳng thức được chứng minh. Đẳng thức xảy ra khi và chỉ khi x = y . .c
Nhận xét. Ta có thể biến đổi tương đương bất đẳng thức như sau:
4  32 ( x + y )
ok
2 2
 1 1
( x + y)  2 + 2 + 2 2  ≥
2
.
x +y  ( x + y)
2
x y
bo

x y
Đặt t =+ , ( t ≥ 2 ) và đưa về chứng minh bất đẳng thức:
y x
et

⇔ ( x − 2 ) ( x 2 + 8 x + 4 ) ≥ 0 (luôn đúng).
8 32 x
x2 + 2x + +4≥
2
vi

x x+2
Bài 36. Ta có
ng

.
a

Vậy ta có
kh

Sử dụng bất đẳng thức C –S ta có

Bài 37. Sử dụng bất đẳng thức C –S ta có

276
Cty TNHH MTV DVVH Khang Việt

Bất đẳng thức được chứng minh. Đẳng thức xảy ra khi và chỉ khi .

Bài 38. Gọi P là biểu thức vế trái và đặt .

Suy ra .

n
.v
Chú ý: .

om
Thay t lần lượt bởi a,b,c và sử dụng bất đẳng thức quen thuộc:

.c .

. Đẳng thức xảy ra khi và chỉ khi


ok
Vì vậy .
bo

Bài 39. Sử dụng bất đẳng thức AM-GM ta có:


a 3 + b3 ≥ ab ( a + b ) ⇒ a 3 + 2b3 ≥ ab ( a + b ) + b3 = b ( a 2 + ab + b 2 ) ≥ 3ab 2 .
et

a 3 + 2b3 3ab 2 3b 2
Suy ra ≥ = .
vi

4ab + 3a 2 4ab + 3a 2 3a + 4b
b3 + 2c3 3c 2 c 3 + 2a 3 3a 2
ng

Tương tự ta có: ≥ ; ≥ .
4bc + 3b 2 3b + 4c 4ca + 3c 2 3c + 4a
 a2 b2 c2 
a

Suy ra P ≥ 3  + + .
 4a + 3c 4b + 3a 4c + 3b 
kh

Sử dụng bất đẳng thức C-S ta có:


(a + b + c) = a + b + c = 1 .
2
a2 b2 c2
+ + ≥
4a + 3c 4b + 3a 4c + 3b 7 ( a + b + c ) 7 7
3 1
Vậy P ≥ . Đẳng thức xảy ra khi và chỉ khi a= b= c= .
7 3
3 1
Vậy giá trị nhỏ nhất của P bằng đạt tại a= b= c= .
7 3

277
Khám phá tư duy Kỹ thuật giải bất ĐT Bài toán Max – Min – Đặng Thành Nam

Bài 40. Gọi P là biểu thức vế trái bất đẳng thức ta có

Sử dụng bất đẳng thức C-S dạng phân thức ta có

Bất đẳng thức được chứng minh. Đẳng thức xảy ra khi và chỉ khi .

n
CHỦ ĐỀ 6: KỸ THUẬT THAM SỐ HÓA

.v
A. NỘI DUNG PHƯƠNG PHÁP

om
Khi áp dụng các bất đẳng thức cơ bản như AM-GM, C-S cái khó nhất của bài
toán là dự đoán dấu bằng. Để khắc phục điều này ta lựa chọn kỹ thuật tham số hóa
tức giả sử dấu bằng của bất đẳng thức đạt tại các điểm cho trước và đi tìm các điểm
.c
xảy ra dấu bằng bằng cách giải phương trình và hệ phương trình ta có kết quả của
ok
bài toán.
Ta xét bài toán sau đây
Cho x,y,z là các số thực thoả mãn điều kiện xy + yz + zx = 1.
bo

Tìm giá trị nhỏ nhất của biểu thức P = kx 2 + ly 2 + z 2 với k,l là các số thực
et

không âm.
Ta có 2 cách cân bằng hệ số cho bài toán trên cũng chính là 2 phương pháp cơ
bản ta hay sử dụng để xử lý các bài toán phải sử dụng đến tham số hoá.
vi

B. BÀI TẬP CHỌN LỌC


ng

Bài 1. Cho x,y,z là các số thực thoả mãn điều kiện xy + yz + 3 zx =


1.
Tìm giá trị nhỏ nhất của biểu thức P = x 2 + y 2 + z 2 .
a

Lời giải
kh

Phân tích tìm lời giải:


Với điều kiện đã cho ta thấy vai trò của x và z như nhau vậy giả sử P đạt min tại
x= z= k . y, ( k ∈  ) .
Với a dương sử dụng bất đẳng thức AM-GM ta có
a ( x 2 + z 2 ) ≥ 2azx

 x + k y ≥ 2kxy .
2 2 2

 z 2 + k 2 y 2 ≥ 2kyz


278
Cty TNHH MTV DVVH Khang Việt

Cộng theo vế các bất đẳng thức trên ta được:


( a + 1) ( x 2 + z 2 ) + 2k 2 y 2 ≥ 2ky ( z + x ) + 2azx
2k 2 2 2a  3k 
⇔ x2 + z 2 + y ≥  ( yz + xy ) + 3zx 
a +1 3 ( a + 1)  a 
Ta cần chọn a và k thoả mãn
 2k 2
 a +1 = 1
 a > 0

a =
3 3 + 33 ( )
 3k  
 = 1 ⇔ a = 3k ⇔ 4

n
 a  2  3 + 33
a > 0 2k − 3k − 3 =0 k =

.v
  4

om
2a 15 − 33
Với a và k chọn được từ trên ta có P ≥ =.
3 ( a + 1) 16
Lời giải chi tiết:
.c
Sử dụng bất đẳng thức AM-GM ta có:
ok
(
3 3 + 33 ) (x 2
+ z2 ) ≥
(
3 3 + 33 ) zx
bo

4 2
2
 3 + 33  3 + 33
x 2 +   ≥ yz
et

 4  2
vi

 3 + 33  2 3 + 33
z 2 +   y ≥ xy
ng

 4  2
Cộng theo vế ba bất đẳng thức trên ta được

(
 3 3 + 33  )
a

2
 3 + 33  2
 + 1 ( x + z ) + 2. 
2 2
 y
kh

 4   4 
 
3 + 33 3 + 33
≥ ( xy + yz + 3zx ) =
2 2

hay
10 + 3 33
.P ≥
3 + 33
⇔P≥
2 3 + 33 15 − 33
= .
( )
4 2 10 + 3 33 16
3 + 33
Đẳng thức xảy ra khi và chỉ khi x ==
z y=
± 6 − 33 .
2
279
Khám phá tư duy Kỹ thuật giải bất ĐT Bài toán Max – Min – Đặng Thành Nam

Bài 2. Cho x,y,z là các số thực dương có tổng bằng 3.


Tìm giá trị nhỏ nhất của biểu thức P = x 2 + y 2 + z 3 .
Lời giải
Vai trò của x và y như nhau cho phép ta giả sử P đạt min tại x= y= a, z= b .
Theo giả thiết ta có 2a + b =3.
Sử dụng bất đẳng thức AM-GM ta có
x 2 + a 2 ≥ 2ax
y 2 + a 2 ≥ 2ay
z 3 + b3 + b3 ≥ 2b 2 z

n
Cộng theo vế 3 bất đẳng thức trên ta được

.v
x 2 + y 2 + z 3 + ( 2a 2 + 2b3 ) ≥ 2a ( x + y ) + 3b 2 z .

om
Ta chọn các hệ số a và b dương sao cho 2a = 3b 2 .
 19 − 37
 3−b a =
 2a + b = 3 a = 
.c 12
Vậy ta có hệ phương trình  ⇔  2 ⇔  .
 = 2
3b − b − 3 = − +
0 b =
2 a 3b 1 37
ok
2
  6
2 3
 19 − 37   −1 + 37 
bo

Suy ra P =
≥ 2a + b 2 
2 3
 +   .
 12   6 
et

Bài 3. Cho x,y là hai số thực không âm thoả mãn điều kiện x 2 + y 2 =
5.
= x3 + y 6 .
vi

Tìm giá trị nhỏ nhất của biểu thức P


Lời giải
ng

Giả sử P đạt min tại= , y b theo giả thiết ta có a 2 + b 2 =


x a= 5.
Sử dụng bất đẳng thức AM-GM ta có
a

x3 + x 3 + a 3 ≥ 3ax 2
kh

2 ( y 6 + b 6 + b 6 ) ≥ 2.3b 4 y 2
Cộng theo vế 2 bất đẳng thức trên ta được
2 x3 + 2 y 6 + a 3 + 4b 6 ≥ 3 ( ax 2 + 2by 2 ) .
Ta chọn các số dương a,b thoả mãn a = 2b .
a = 2b a = 2
Khi đó ta có hệ phương trình  ⇔ .
2
5 b = 1
a + b =
2

Do đó P ≥ a 3 + b 6 =
9 . Đẳng thức xảy ra khi và chỉ khi=
x 2,=
y 1.

280
Cty TNHH MTV DVVH Khang Việt

Bài 4. Cho x,y,z là các số thực không âm thoả mãn điều kiện x + y + z =3.
Tìm giá trị nhỏ nhất của biểu thức P =x 4 + 2 y 4 + 3 z 4 .
Lời giải
Giả sử P đạt min tại=
x a=
, y b=
, z c theo giả thiết ta có a + b + c =3.
Sử dụng bất đẳng thức AM-GM ta có
x 4 + a 4 + a 4 + a 4 ≥ 4a 3 x
2 ( y 4 + b 4 + b 4 + b 4 ) ≥ 8b3 y
3 ( z 4 + c 4 + c 4 + c 4 ) ≥ 12c 3 z

n
Cộng theo vế 3 bất đẳng thức trên ta được

.v
x 4 + 2 y 4 + 3 z 4 + 3a 4 + 6b 4 + 9c 4 ≥ 4 ( a 3 x + 2b3 y + 3c3 z ) .

om
Ta chọn các số dương a,b,c sao cho =
a 3 2=
b3 3c3 .

 .c
a = 3
 1 1
ok
 1+ 3 + 3
 2 3
a + b + c =3  3
bo

Vậy ta có hệ phương trình  3 ⇔ b = .


=
a 2= 3
b 3c 3
  1 1 
3
2 1 + 3 + 3 
  2 3
et



3
9
 c =
vi

1 1
 1+ 3 + 3
 2 3
ng

Suy ra P ≥ a 4 + 2b 4 + 3c 4 với a,b,c xác định như trên.


a

Tổng quát. Với các số thực không âm x1 , x2 ,..., xn có tổng bằng n. Và n số thực
dương k1 , k2 ,..., kn . Tìm giá trị nhỏ nhất của biểu thức P= k1 x1m + k2 x2m + ... + kn xnm
kh

với m > 1 là số nguyên dương cho trước.


Bài 5. Cho x,y,z là các số thực dương thoả mãn điều kiện 2 x + 4 y + 3 z 2 =
68 .
Tìm giá trị nhỏ nhất của biểu thức P = x 2 + y 2 + z 3 .
Lời giải
Giả sử P đạt min tại=
x a= , z c theo giả thiết ta có 2a + 4b + 3c 2 =
, y b= 68 .
Sử dụng bất đẳng thức AM-GM ta có

281
Khám phá tư duy Kỹ thuật giải bất ĐT Bài toán Max – Min – Đặng Thành Nam

x 2 + a 2 ≥ 2ax
y 2 + b 2 ≥ 2by
c3 3cz 2
z 3 + z 3 + c3 ≥ 3cz 2 ⇔ z 3 + ≥
2 2
Cộng theo vế 3 bất đẳng thức trên ta được
c3 3cz 2
x2 + y 2 + z 3 + a 2 + b2 +
≥ 2ax + 2by + .
2 2
3c
2a 2b 2

n
Chọn các số dương a,b,c sao cho = = ⇔ b = c = 2a .
2 4 3

.v
2a + 4b + 3c 2 =
68 a = 2
Vậy ta có hệ phương trình  ⇔

om
.
b= c= 2a b= c= 4
Suy ra P ≥ a 2 + b 2 + c 3 = 84 .
Bài 6. Cho x,y,z là các số thực không âm thỏa mãn x + y + z =
.c 3.
Tìm giá trị nhỏ nhất và lớn nhất của biểu thức
ok
P= x2 − x + 1 + y 2 − y + 1 + z 2 − z + 1 .
bo

Lời giải
Tìm giá trị lớn nhất của P
Nhận xét. Tìm một bất đẳng thức dạng x 2 − x + 1 ≤ mx + n .
et

Công việc của ta là đi tìm m và n. Bất đẳng thức đối xứng với 3 biến x,y,z nên ta
vi

không thể quyết định xem biến nào bằng 0 và biến nào bằng 3. Vì vậy cách tốt nhất
là tìm m và n sao cho dấu bằng đạt tại 0 và 1 đều thỏa mãn. Hay cách khác ta cần
ng

tìm m,n sao cho cả 0 và 3 là nghiệm của phương trình: x 2 − x + 1= mx + n .


 7 −1
a

n = 1 m =
Vậy ta có hệ phương trình  ⇔
kh

3
3m + n =7 n = 1

Cuối cùng kiểm tra xem

x2 − x + 1 ≤ 1 +
( )
7 −1 x

(1 + 2 7 ) x ( x − 3) ≤ 0 (luôn đúng
3
x − x +1 +1+
2
( 7 − 1) x
3
với x ∈ [ 0;3] ).

282
Cty TNHH MTV DVVH Khang Việt

Vậy P ≤ 3 +
( )
7 −1 ( x + y + z )
= 3 + 7 −1 = 2 + 7 .
3
Đẳng thức xảy ra khi và chỉ khi 2 biến bằng 0 và một biến bằng 3.
Vậy giá trị lớn nhất của P bằng 2 + 7 đạt tại a= b= 0, c= 3 hoặc các hoán vị.
Tìm giá trị nhỏ nhất của P

3 ( x − 1)  x + 1 
2
x +1
2

Ta có x= − x +1
2
+  ≥ .
4  2  2
y +1 z +1
Tương tự ta có: y 2 − y + 1 ≥ ; z2 − z +1 ≥

n
.
2 2

.v
x+ y+ z +3
Cộng theo vế 3 bất đẳng thức trên ta được: P ≥ =
3.

om
2
Đẳng thức xảy ra khi và chỉ khi x= y= z= 1 .
Vậy giá trị nhỏ nhất của P bằng 3 đạt tại x= y= z= 1 . .c
Bài 7. Cho x,y,z là các số thực thỏa mãn điều kiện xy + yz + zx =
1.
ok
Tìm giá trị nhỏ nhất của biểu thức P =x 2 + 2 y 2 + 5 z 2 .
Lời giải
bo

Ta chứng minh bất đẳng thức:


x 2 + 2 y 2 + 5 z 2 ≥ 2k ( xy + yz + zx ) =
2k với k > 0 là số chọn sau.
et

(
Cộng thêm vào hai vế bất đẳng thức trên với k x 2 + y 2 + z 2 ta được: )
vi

( k + 1) x 2 + ( k + 2 ) y 2 + ( k + 5) z 2 ≥ k ( x + y + z )
2
.
ng

Sử dụng bất đẳng thức C-S ta được:


( x + y + z)
2
x2 y2 z2
( ) (
k + 1 x +
2
+ ) (+ +
2
) = +2
+ ≥
a

k 2 y k 5 z
1 1 1 1 1 1
+ +
kh

k +1 k + 2 k + 5 k +1 k + 2 k + 5
Vậy ta chỉ cần chọn k dương sao cho:

= k ⇔ ( k − 1) ( k 2 + 5k + 5 )= 0 ←
1 k >0
→ k= 1 .
1 1 1
+ +
k +1 k + 2 k + 5
Lời giải.
Ta có:

283
Khám phá tư duy Kỹ thuật giải bất ĐT Bài toán Max – Min – Đặng Thành Nam

x2 y 2 z 2 ( x + y + z )
2

2x + 3y + 6z = + + ≥ = ( x + y + z)
2 2 2 2

1 1 1 1 1 1
+ +
2 3 6 2 3 6
⇒ P = x 2 + 2 y 2 + 5 z 2 ≥ 2 ( xy + yz + zx ) = 2
1 1 1
Đẳng thức xảy ra khi và chỉ khi x : y : =
z : : = 3 : 2 :1 ⇔ =
x 3z, =
y 2z
2 3 6
thay vào điều kiện ta có
1  3 2 1 
3 z 2 + 6 z 2 + 2 z 2 =⇔
1 z =± ⇒ ( x; y; z ) =±
 ;± ;± .
 11 11 

n
11 111

.v
 3 2 1 
Vậy giá trị nhỏ nhất của P bằng 2 đạt tại ( x; y; z ) =
± ;± ;± .
 11 111 11 

om
Bài 8. Cho a,b,c là các số thực dương thỏa mãn điều kiện a 2 + b 2 + c 2 =
1.
Tìm giá trị lớn nhất của biểu thức P = a 2b3c 3 .
Lời giải
.c
Phân tích chọn điểm rơi:
ok
Giả sử P đạt giá trị nhỏ nhất tại=
a x=, b y= ,c z .
Khi đó theo điều kiện ta có: x 2 + y 2 + z 2 =
1.
bo

Sử dụng bất đẳng thức AM-GM ta có:


a 2b 3c 4
et

2a 3b 4c
+ + ≥ 99 2 3 4 .
x y z x y z
vi

Sử dụng bất đẳng thức C-S ta có:


ng

2
 2a 3b 4c  2  4 9 16  4
 + +  ≤ (a + b + c ) 2 + 2 + 2  = 2 + 2 + 2 .
2 2 9 16
 x y z  x y z  x y z
a

Đẳng thức xảy ra khi và chỉ khi


kh

2 3 4 ax by cz x2 y 2 z 2
a :b:c = : : ⇔ = = ⇔ = = .
x y z 2 3 4 2 3 4
 2
x =
 3
 x2 y 2 z 2
= =  3
Vậy ta chọn x, y, z > 0 sao cho  2 3 4 ⇔ y = .
 2 3
x + y + z =
2 2
1 
 2
z = 3

284
Cty TNHH MTV DVVH Khang Việt

Lời giải chi tiết:


Sử dụng bất đẳng thức AM-GM ta có:
2a 3b 4c a 2b3c 4
+ + ≥9 2 3
.
2 3 2 9  2   3 2
4

3 3 3      
 3   3  3
Sử dụng bất đẳng thức C-S ta có:
 2
    2 
2

 
2   3  4 
≤ ( a 2 + b 2 + c 2 ) 
2a 3b 4c
+ +  +  +  = 9.
 2   3   2  

n
2 3 2
    3 
 3   3    
3

.v
3 3
2 3
 2  3   2  4 32 3

om
Suy ra: P =
a 2b3c 4 ≤       = .
 3   3  3 6561

2 3 2
=
Đẳng thức xảy ra khi và chỉ khi a = ,b .c = ,c .
3 3 3
Bài 9. Cho x,y, z là các số thực dương. Tìm giá trị lớn nhất của biểu thức
ok
2 xy + 4 yz + 3 zx
P= .
( x + y + z)
2
bo

Lời giải
Ta có
et

x y y z z x
P = 2. . + 4. . + 3. . .
x+ y+z x+ y+z x+ y+z x+ y+z x+ y+z x+ y+z
vi

x y z
Đặt a =
= ,b = , ( a, b, c > 0 ) ta có a + b + c =1.
ng

,c
x+ y+z x+ y+z x+ y+z
Ta cần tìm giá trị lớn nhất của biểu thức
a

P = 2ab + 4bc + 3ca với a + b + c = 1.


Ta chọn các hằng số m,n,p dương sao cho
kh

= ma ( b + c ) + nb ( c + a ) + pc ( a + b )
2ab + 4bc + 3ca
⇔ ( m + n ) ab + ( n + p ) bc + ( p + m ) ca = 2ab + 4bc + 3ca
 1
m = 2
m + n = 2 
  3
⇔ n + p = 4 ⇔ n =
p+m =  2
 3
 5
p = 2

285
Khám phá tư duy Kỹ thuật giải bất ĐT Bài toán Max – Min – Đặng Thành Nam

Khi đó
1 3 5
=
P a (b + c ) + b (c + a ) + c ( a + b)
2 2 2
1 3 5
= a (1 − a ) + b (1 − b ) + c (1 − c )
2 2 2
9 1  1 
2 2 2
1  1 
= −  a −  + 3  b −  + 5  c −  
8 2  2  2  2  
 2
1 
2
1  1 
2

 − − −  
9 1      
a b c
2  2  2 

n
=− + +
8 2 1 1 

.v
1
 3 5 
 

om
2
 1 1 1
 a− +b− +c− 
≤ −
9 2 2 2 24
= .c
8  1 1 23
2 1 + + 
 3 5
ok
Đẳng thức xảy ra khi và chỉ khi
a + b + c = 1
bo


 1  1  1 1 1 ⇒ x = 3 y −1= 5z − 2 .
 a − 2  :  b − 2  :  c − 2  =
1: :
      3 5
et

Bài tập tương tự


vi

1) Cho x,y,z là các số thực


Bằng cách tương tự ta tổng quát bài toán sau đây
ng

c k , ( k > 0 ) . Tìm giá trị


Cho a,b,c là các số thực không âm thoả mãn a + b +=
lớn nhất của biểu thức
a

P = m.ab + n.bc + p.ca, ( m, n, p > 0 ) .


kh

( )
Bài 10. Chứng minh với mọi 0 ≤ x ≤ 1 ta có x 9 1 + x 2 + 13 1 − x 2 ≤ 16 .

Lời giải
Ta có
9ax 1 + x 2 13bx 1 − x 2 9 ( a x + 1 + x ) 13 ( b x + 1 − x )
2 2 2 2 2 2

VT = + ≤ + .
a b 2a 2b
Ta chọn a và b dương sao cho hệ số của x 2 triệt tiêu và thoả mãn dấu bằng của
bất đẳng thức AM – GM

286
Cty TNHH MTV DVVH Khang Việt

 9 ( a 2 + 1) 13 ( b 2 − 1)
 + =
0  3
 2a 2b a 2 − 1 = b 2 + 1 a=
 2 2  
a x = 1 + x2 ⇔  9 ( a 2 + 1) 13 ( b 2 − 1) ⇔ 2.
b 2 x 2 = 1 − x 2  + =
0 b = 1
  2a 2b  2

Lời giải chi tiết:

(
x 9 1 + x 2 + 13 1 − =
x2 ) 3 x
6. x 1 + x 2 + 26. 1 − x 2
2 2

n
 9x 2
  x2 
≤ 3 + 1 + x 2  + 13  + 1 − x 2  =16

.v
 4   4 

om
 9x2
 4 = 1 + x
2
2
Đẳng thức xảy ra khi và chỉ khi  2 ⇔x=
 x = 1 − x2 .c 5
 4
ok
C. BÀI TẬP RÈN LUYỆN
Bài 1. Cho x,y,z là các số thực không âm thoả mãn điều kiện xy + yz + zx =
1.
bo

Chứng minh rằng 10 x 2 + 10 y 2 + z 2 ≥ 4 .


Bài 2. Cho x,y,z là các số thực dương có tổng bằng 3. Tìm giá trị nhỏ nhất của biểu
et

15 5 3
thức P = + + .
x y3 z5
vi

Bài 3. Cho ba số thực x,y,z thuộc đoạn [ 0;1] và x + y + z =2.


ng

Tìm giá trị lớn nhất và nhỏ nhất của biểu thức
P= x2 − 4 x + 5 + y 2 − 4 y + 5 + z 2 − 4z + 5 .
a

Bài 4. Cho a,b,c là các số thực dương. Chứng minh rằng


kh

10a 2 + 10b 2 + c 2 ≥ 4 ( ab + bc + ca ) .
Bài 5. Cho x,y,z là các số thực không âm thoả mãn điều kiện x 2 + y 2 + z 2 =
1.
Tìm giá trị nhỏ nhất của biểu thức P =x 3 + 2 y 3 + 3 z 3 .
Bài 6. Cho x,y,z là các số thực không âm thỏa mãn x + y + z =
1.
1 3
Tìm giá trị nhỏ nhất của biểu thức P = x 3 + y 3 + z .
2

287
Khám phá tư duy Kỹ thuật giải bất ĐT Bài toán Max – Min – Đặng Thành Nam

D. HƯỚNG DẪN GIẢI – ĐÁP SỐ


Bài 1. Cách 1: Ta cần chứng minh
10 x 2 + 10 y 2 + z 2 ≥ 4 ( xy + yz + zx )
⇔ 12 x 2 + 12 y 2 + 3 z 2 ≥ 2 ( x + y + z )
2

x2 y 2 z 2
⇔ + + ≥ 2( x + y + z)
2

1 1 1
12 12 3
Bất đẳng thức luôn đúng theo C-S, thật vậy

n
x2 y 2 z 2 ( x + y + z )
2

+ + ≥ = 2( x + y + z)
2

.v
1 1 1 1 1 1
+ +
12 12 3 12 12 3

om
 1
 1 1 1  x= y=
x : y : z = : :  3
Đẳng thức xảy ra khi và chỉ khi  12 12 3 ⇔ 
.c .
 xy + yz + zx =  4
1 z=
 3
ok
Cách 2: Sử dụng bất đẳng thức AM-GM ta có
2 x 2 + 2 y 2 ≥ 4 xy
bo

z2
8x2 + ≥ 4 xz
et

2
z2
8 y 2 + ≥ 4 yz
vi

2
Cộng theo vế ba bất đẳng thức trên ta được
ng

10 x 2 + 10 y 2 + z 2 ≥ 4 ( xy + yz + zx ) =
4.
a

x = y  1
4 x = z x= y=
kh

 
 3
Đẳng thức xảy ra khi và chỉ khi  ⇔ .
4 y = z z = 4
1 
 xy + yz + zx = 3
Bài 2. Giả sử P đạt min tại=
x a=
, y b=
, z c theo giả thiết ta có a + b + c =3.
Sử dụng bất đẳng thức AM-GM ta có

288
Cty TNHH MTV DVVH Khang Việt

 1 x  30
15  + 2  ≥ ;
x a  a
 1 y y y  20
5 3 + 4 + 4 + 4  ≥ 3 ;
y b b b  b
1 z z z z z  18
3 5 + 6 + 6 + 6 + 6 + 6  ≥ 5 .
z c c c c c  c
Cộng theo vế 3 bất đẳng thức trên ta được
15 5 3 15 x 15 y 15 z 30 20 18
+ + + + 4 + 6 ≥ + + .

n
x y3 z5 a2 b c a b3 c 5

.v
1 1 1
Ta chọn các số dương a,b,c thoả mãn =
2
=4
.
a b c6

om
a + b + c = 3

Vậy ta có hệ phương trình  1 1 1 ⇔ a = b = c =1 .
 a=
2
b
=4
c6
.c
15 5 3
ok
Suy ra P ≥ + + = 23 .
a b3 c 5
( )
Bài 4. Cộng thêm vào hai vế của bất đẳng thức đại lượng 2 a 2 + b 2 + c 2 ta cần
bo

chứng minh: 12a 2 + 12b 2 + 3c 2 ≥ 2 ( a + b + c ) .


2
et

Sử dụng bất đẳng thức C-S ta có:


a 2 b2 c2 ( a + b + c )
2
vi

12a 2 + 12b 2 + 3c 2 = + + ≥ = 2(a + b + c) .


2

1 1 1 1 1 1
+ +
ng

12 12 3 12 12 3
1 1 1 1
Đẳng thức xảy ra khi và chỉ khi a : b : c = : : ⇔ a = b = c .
a

12 12 3 4
kh

Bài 6. Phân tích tìm điểm rơi


Biểu thức của P đối xứng với x,y nên dấu bằng xảy ra tại x= y= a, z= b . Khi
đó theo điều kiện ta có: 2a + b =
1.
Khi đó áp dụng bất đẳng thức AM-GM ta được:
b 2 ( a 3 + a 3 + x 3 ) ≥ 3a 2b 2 x
b 2 ( a 3 + a 3 + y 3 ) ≥ 3a 2b 2 y
a 2 ( b3 + b3 + z 3 ) ≥ 3a 2b 2 z

289
Khám phá tư duy Kỹ thuật giải bất ĐT Bài toán Max – Min – Đặng Thành Nam

Cộng theo vế 3 bất đẳng thức trên ta được:


b ( x3 + y 3 ) + a 2 z 3 + 4a 3b 2 + 2a 2b3 ≥ 3a 2b 2 ( x + y + z ) =
2
3a 2b 2
⇒ b 2 ( x3 + y 3 ) + a 2 z 3 ≥ 3a 2b 2 − 4a 3b 2 − 2a 2b3 (1)

b2 a 2
Ta chọn a,b không âm sao cho: = ⇔ b 2 = 2a 2 .
1 1
2
 2− 2
b 2 = 2a 2 a =
Ta có hệ phương trình:  ⇔ 2 .
 2a + b =

n
1 
=
b 2 −1

.v
Lời giải.
Thay a,b vào bất đẳng thức (1) ta được:

om
3 − 2 2 3 17 − 12 2 3− 2 2
(3 − 2 2 ) ( x 3
+ y3 ) +
2
z ≥
2 .c 1
⇔ x3 + y 3 + z 3 ≥
2 2
.

2− 2
Đẳng thức xảy ra khi và chỉ khi x= y= , z= 2 −1.
ok
2
Cách 2: Đây là dạng toán quen thuộc sử dụng tính đơn điệu hàm số.
bo

Ta có:
1 3 3 1
P = ( x + y ) − 3 xy ( x + y ) + z ≥ ( x + y ) − ( x + y ) ( x + y ) + z3
3 3 2
et

2 4 2
1 1
⇒ f (z) = (1 − z ) + z 3
3
vi

4 2
ng

1 1
Xét hàm số f (z) = (1 − z ) + z 3 liên tục trên đoạn [ 0;1] ta có:
3

4 2
3 3 z∈[ 0;1]
f '( z ) = − (1 − z ) + z 2 ; f '(z) = 0 ←
a

→ z = 2 −1.
2

4 2
kh

Ta có f’(z) đổi dấu từ âm sang dương khi đi qua=


z 2 − 1 nên f(z) đạt cực tiểu
3− 2 2
tại=
z 2 − 1 hay P ≥ f ( z ) ≥ f ( 2 − 1) = .
2
3− 2 2 2− 2
Vậy giá trị nhỏ nhất của P bằng đạt tại x= y= , z= 2 −1.
2 2

290
Cty TNHH MTV DVVH Khang Việt

CHỦ ĐỀ 7: BẤT ĐẲNG THỨC HOLDER VÀ ỨNG DỤNG

A. NỘI DUNG PHƯƠNG PHÁP


Dưới đây tôi trình bày bất đẳng thức Holder cho 3 dãy số mỗi dãy gồm 3 số dương.
Cho a, b, c, x, y, z, m, n, p là các số thực dương ta có
(a 3
+ b3 + c 3 )( x 3
)( )
+ y 3 + z 3 m3 + n3 + p3 ≥ ( axm + byn + czp ) .
3

Chứng minh. Sử dụng bất đẳng thức AM – GM ta có


a3 x3 m3
+ +
a 3 + b3 + c 3 x 3 + y 3 + z 3 m3 + n 3 + p 3

n
3axm

.v
3
(a 3
+ b3 + c 3 )( x 3
)(
+ y 3 + z 3 m3 + n3 + p 3 )

om
Tương tự ta có
b3 y3 n3
+ +
a 3 + b3 + c 3 x3 + y 3 + z 3 m3 + n 3 + p 3.c
3byn

ok
3
(a 3
+ b3 + c 3 )( x 3
)(
+ y 3 + z 3 m3 + n 3 + p 3 )
bo

c3 z3 p3
+ +
a 3 + b3 + c 3 x3 + y 3 + z 3 m3 + n 3 + p 3
et

3czp

3
(a 3
+ b3 + c 3 )( x 3
)(
+ y 3 + z 3 m3 + n 3 + p 3 )
vi

Cộng lại theo vế 3 bất đẳng thức trên ta có điều phải chứng minh.
ng

Hệ quả 1. Cho a,b,c là các số thực dương ta có

(1 + a )(1 + b )(1 + c ) ≥ (1 + 3 abc )


3
.
a
kh

B. BÀI TẬP MẪU


Bài 1. Chứng minh với mọi a,b,c là các số thực dương ta có
( )( )( )
2 a 2 + 1 b 2 + 1 c 2 + 1 ≥ ( a + 1)( b + 1)( c + 1)( abc + 1) .
Lời giải
Nhận xét. Với a= b= c bất đẳng thức trở thành

( ) ( ) ( )
3
2 a 2 + 1 ≥ a3 + 1 ( a + 1) ⇔ ( a − 1) a 2 + a + 1 ≥ 0 (luôn đúng).
3 4

Vậy ta có

291
Khám phá tư duy Kỹ thuật giải bất ĐT Bài toán Max – Min – Đặng Thành Nam

( ) ( )
3
2 a 2 + 1 ≥ ( a + 1) a3 + 1
3

2 ( b + 1) ≥ ( b + 1) ( b + 1)
2 3 3 3

2 ( c + 1) ≥ ( c + 1) ( c + 1)
2 3 3 3

Nhân theo vế 3 bất đẳng thức trên ta được

( ) (b + 1) ( c + 1) ( )( )( )
3 3 3
≥ ( a + 1) ( b + 1) ( c + 1) a3 + 1 b3 + 1 c3 + 1 .
3 3 3
8 a2 + 1 2 2

(
Vậy ta chỉ cần chứng minh a3 + 1 b3 + 1 c3 + 1 ≥ (1 + abc ) .)( )( ) 3

n
Đây chính là bất đẳng thức Holder. Bất đẳng thức được chứng minh đẳng thức

.v
xảy ra khi và chỉ khi a= b= c= 1 .
Bài 2. Chứng minh với mọi a,b,c là các số thực dương ta có

om
a b c
+ + ≥ 3.
2b 2 + 2c 2 − a 2 2c 2 + 2a 2 − b 2 2a 2 + 2b 2 − c 2

Gọi P là biểu thức vế trái và đặt


Lời giải
.c
ok
( ) (
S= a 2b 2 + 2c 2 − a 2 + b 2c 2 + 2a 2 − b 2 + c 2a 2 + 2b 2 − c 2 . ) ( )
bo

Sử dụng bất đẳng thức Holder ta có P.P.S ≥ ( a + b + c ) ⇒ P


3 2

( a + b + c)
3
.
S
et

Vậy ta chứng minh


( a + b + c )3 ≥ 3
vi

S
( ) + b ( 2c ) ( )
ng

⇔ ( a + b + c ) ≥ 3  a 2b + 2c − a + 2a 2 − b 2 + c 2a 2 + 2b 2 − c 2 
3 2 2 2 2
 
( )
⇔ 4 a3 + b3 + c3 + 6abc ≥ 4  ab ( a + b ) + bc ( b + c ) + ca ( c + a ) 
a
kh

Chú ý a3 + b3 + c3 + 3abc ≥ ab ( a + b ) + bc ( b + c ) + ca ( c + a )

a3 + b3 + c3 ≥ 3abc
Cộng theo vế 2 bất đẳng thức trên ta có điều phải chứng minh. Đẳng thức xảy ra
khi và chỉ khi a= b= c .
Bài 3. Cho a,b,c là độ dài 3 cạnh một tam giác thoả mãn điều kiện
a 2 + b2 + c2 =3.
a b c
Chứng minh rằng + + ≥ 3.
b+c−a c+a−b a+b−c

292
Cty TNHH MTV DVVH Khang Việt

Lời giải
Gọi P là biểu thức vế trái và đặt S= a ( b + c − a ) + b ( c + a − b ) + c ( a + b − c ) .

Sử dụng bất đẳng thức Holder ta có P.P.S ≥ ( a + b + c ) .


3

Vậy ta cần chứng minh ( a + b + c ) ≥ 9 S


3

⇔ ( a + b + c ) ≥ 9  2 ( ab + bc + ca ) − 3 .
3

( )
Đặt x = a + b + c, x ∈  3;3 ta có 2 ( ab + bc + ca ) = x 2 − 3 ta cần chứng minh

( ) ( )
x3 ≥ 9 x 2 − 6 ⇔ ( x − 3) x 2 − 6 x − 18 ≥ 0, ∀x ∈  3;3 .

n
Bất đẳng thức được chứng minh. Đẳng thức xảy ra khi và chỉ khi a= b= c= 1 .

.v
Bài 4. Cho a,b,c là các số thực dương có tích bằng 1.

om
a b c
Chứng minh rằng + + ≥1.
b+c+7 c+a+7 a+b+7
Lời giải .c
Gọi P là biểu thức vế trái và đặt S = a ( b + c + 7 ) + b ( c + a + 7 ) + c ( a + b + 7 ) .
ok
Sử dụng bất đẳng thức Holder ta có P.P.S ≥ ( a + b + c ) .
3

Vậy ta cần chứng minh ( a + b + c ) ≥ S


3
bo

⇔ ( a + b + c ) ≥ 2 ( ab + bc + ca ) + 7 ( a + b + c ) .
3
et

Ta có ab + bc + ca ≤
( a + b + c)
2
.
vi

3
2
Vậy ta chứng minh ( a + b + c ) ≥ ( a + b + c )2 + 7 ( a + b + c )
3
ng

3
⇔ 3 ( a + b + c ) − 2 ( a + b + c ) − 21 ≥ 0
2
a

⇔ ( a + b + c − 3) 3 ( a + b + c ) + 7  ≥ 0
kh

Bất đẳng thức cuối luôn đúng do a + b + c ≥ 3 3 abc =3.


Bất đẳng thức được chứng minh. Đẳng thức xảy ra khi và chỉ khi a= b= c= 1 .
Bài 5. Cho x,y,z là độ dài 3 cạnh một tam giác.
1 1 1 x+ y+z
Chứng minh rằng + + ≥ .
x+ y−z y+z−x z+x− y xyz
Lời giải
S x3 ( y + z − x ) + y 3 ( z + x − y ) + z 3 ( x + y − z ) .
Gọi P là biểu thức vế trái và đặt =

293
Khám phá tư duy Kỹ thuật giải bất ĐT Bài toán Max – Min – Đặng Thành Nam

Sử dụng bất đẳng thức Holder ta có P.P.S ≥ ( x + y + z ) .


3

( x + y + z )3 ≥  x + y + z 
2

Vậy ta cần chứng minh  


S  xyz 

⇔ xyz ( x + y + z ) ≥ x3 ( y + z − x ) + y 3 ( z + x − y ) + z 3 ( x + y − z )

( ) (
⇔ x 4 + y 4 + z 4 + xyz ( x + y + z ) ≥ xy x 2 + y 2 + yz y 2 + z 2 + zx z 2 + x 2 ) ( )
Bất đẳng thức cuối luôn đúng(xem thêm chủ đề biến đổi tương đương).
Bất đẳng thức được chứng minh. Đẳng thức xảy ra khi và chỉ khi x= y= z .

n
Bài 6. Cho a,b,c là các số thực không âm thoả mãn ab + bc + ca > 0 .

.v
a b c
Chứng minh rằng + + ≥2.
b+c c+a a+b

om
Lời giải
Nhận xét. Bất đẳng thức trên đã được chứng minh đơn giản bằng bất đẳng thức
AM – GM dưới đây ta tiếp cận bài toán theo bất đẳng thức Holder. .c
Gọi P là biểu thức vế trái và đặt S= a 2 ( b + c ) + b 2 ( c + a ) + c 2 ( a + b ) .
ok
Sử dụng bất đẳng thức Holder ta có P.P.S ≥ ( a + b + c ) .
3

Vậy ta cần chứng minh ( a + b + c ) ≥ 4 S


3
bo

⇔ a3 + b3 + c3 + 6abc ≥ ab ( a + b ) + bc ( b + c ) + ca ( c + a )
et

Bất đẳng thức cuối luôn đúng do


a3 + b3 + c3 + 3abc ≥ ab ( a + b ) + bc ( b + c ) + ca ( c + a )
vi

3abc ≥ 0
ng

Bất đẳng thức được chứng minh. Đẳng thức xảy ra khi và chỉ khi có 1 số bằng 0
và 2 số còn lại bằng nhau.
a

Bài 7. Cho a,b,c là các số thực dương thoả mãn điều kiện a 2 + b 2 + c 2 =a + b + c .
kh

Chứng minh rằng a 2b 2 + b 2 c 2 + c 2 a 2 ≤ ab + bc + ca .


Lời giải
Ta có

(a )
2
2
+ b2 + c2 = ( a + b + c )2
( )
⇒ 2 ab + bc + ca − a 2b 2 − b 2 c 2 − c 2 a 2 = a 4 + b 4 + c 4 − a 2 − b 2 − c 2

Vậy ta chứng minh a 4 + b 4 + c 4 ≥ a 2 + b 2 + c 2 .


Sử dụng bất đẳng thức Holder ta có

294
Cty TNHH MTV DVVH Khang Việt

( a + b + c )2 ( a 4 + b 4 + c 4 ) ≥ ( a 2 + b 2 + c 2 )
3
⇒ a 4 + b4 + c4 ≥ a 2 + b2 + c2 .
Bất đẳng thức được chứng minh. Đẳng thức xảy ra khi và chỉ khi a= b= c= 1 .
Bài 8. Cho a,b,c là các số thực dương thoả mãn điều kiện a 2 + b 2 + c 2 =
3.
a5 b5 c5
Chứng minh rằng + + ≥ 3.
a3 + 2bc b3 + 2ca c3 + 2ab
Lời giải
( ) (
Gọi P là biểu thức vế trái và đặt S = a a3 + 2bc + b b3 + 2ca + c c3 + 2ab . ) ( )
( )

n
3
Sử dụng bất đẳng thức Holder ta có P 2 S ≥ a 2 + b 2 + c 2 =
27 .

.v
Vậy ta chỉ cần chứng minh
( ) ( ) ( )

om
a a3 + 2bc + b b3 + 2ca + c c3 + 2ab ≤ 9

+ 6abc ≤ ( a + b )
2
⇔ a 4 + b4 + c4 2 2
+ c2

⇔ a 2b 2 + b 2 c 2 + c 2 a 2 ≥ 3abc
.c
Bất đẳng thức cuối đúng bởi vì theo AM – GM ta có
ok
(
a 2b 2 + b 2 c 2 + c 2 a 2 ≥ 3a 2b 2 c 2 a 2 + b 2 + c 2 =
3abc . )
bo

Bất đẳng thức được chứng minh. Đẳng thức xảy ra khi và chỉ khi a= b= c= 1 .
et

C. BÀI TẬP RÈN LUYỆN


Bài 1. Chứng minh với mọi a,b,c là các số thực dương ta có
vi

( ) ≥ (a )
2 2
3 a 3 + b3 + c 3 2
+ b2 + c2 .
ng

Bài 2. Cho a,b,c là các số thực dương có tổng bằng 1.


a b c 3
a

Chứng minh rằng + + ≥ .


b+c c+a a+b 2
kh

Bài 3. Chứng minh với mọi a,b,c là các số thực dương ta có


a b c
+ + ≥1.
a + 8bc
2
b + 8ca
2
c + 8ab
2

Bài 4. Cho a,b,c là các số thực dương có tích bằng 1.


a b c
Chứng minh rằng + + ≥1.
b +c +7
2 2
c +a +7
2 2
a + b2 + 7
2

Bài 5. Cho a,b,c là các số thực không âm có tổng bằng 3.

295
Khám phá tư duy Kỹ thuật giải bất ĐT Bài toán Max – Min – Đặng Thành Nam

a b c
Chứng minh rằng + + ≥ 3.
1 + b + bc 1 + c + ca 1 + a + ab

Bài 6. Cho a,b,c là các số thực dương chứng minh rằng

(a 5
)( )( )
− a 2 + 3 b5 − b 2 + 3 c 5 − c 2 + 3 ≥ ( a + b + c ) .
3

Bài 7. Cho x,y,z là các số thực dương thoả mãn điều kiện x + y + z = xy + yz + zx .

( )
2
Chứng minh rằng ( x + y + z ) x+ y+ z ≥ 27 .
Bài 8. Cho a,b,c là các số thực dương có tổng bằng 3.

n
Chứng minh rằng a + b + c ≥ ab + bc + ca .

.v
Bài 9. Cho x,y,z là các số thực dương.

om
x2 y 2 z 2 x4 + y 4 + z 4
Chứng minh rằng + + ≥ 34 .
y z x 3
Bài 10. Cho a,b,c là các số thực dương. Chứng minh rằng .c
(1 + a )(1 + b )(1 + c ) ≥ (1 + ab )(1 + bc )(1 + ca ) .
3 3 3 2 2 2
ok
Bài 11. Cho a,b,c,x,y,z là các số thực dương. Chứng minh
a 3 b3 c 3 ( a + b + c )
3
bo

+ + ≥ .
x y z 3( x + y + z )
Bài 12. Cho a,b,c là các số thực dương.
et

1 1 1 3
Chứng minh rằng + + ≥ .
(1 + a ) (1 + b ) (1 + c ) 4 (1 + abc )
vi

3 3 3

Bài 13. Cho a,b,c là các số thực dương có tổng bằng 1. Chứng minh
ng

a b c
+3 +3 ≥1.
3
a + 2b b + 2c c + 2a
a

Bài 14. Cho a,b,c là các số thực dương. Chứng minh


kh

( )( )( ) ( )
2
9 a 4 + 1 b 4 + 1 c 4 + 1 ≥ 8 a 2b 2 c 2 + abc + 1 .
Bài 15. Cho a,b,c là các số thực dương. Chứng minh
a+b b+c c+a a+b+c
+ + ≥ 4(a + b + c) .
c a b 3 ( a + b )( b + c )( c + a )
Bài 16. Cho a,b,c là các số thực dương. Chứng minh
a b c 27
+ + ≥ .
(b + c) 3
(a + c) 3
( a + b) 2
8(a + b + c) 2

296
Cty TNHH MTV DVVH Khang Việt

Bài 17. Cho a,b,c là các số thực dương. Chứng minh

( a + b )3 +
( b + c )3 +
( c + a )3 ≥ 1.
8ab ( 4a + 4b + c ) 8bc ( 4b + 4c + a ) 8ca ( 4c + 4a + b )
Bài 18. Cho a,b,c là các số thực không âm. Chứng minh
a b c
+ + ≥ 1.
( b + c ) 2
+ 5c 2
( c + a ) 2
+ 5 a 2
( a + b ) 2
+ 5b 2

Bài 19. Cho a,b,c là các số thực dương thoả mãn điều kiện ab + bc + ca =
1.
 1 1 1
Chứng minh rằng abc  3 6a + + 3 6b + + 3 6c +  ≤ 1 .

n
 c a b

.v
Bài 20. Cho a,b,c là các số thực dương thoả mãn điều kiện

om
a 3 + b3 + c 3 = a 4 + b 4 + c 4 .
a b c
Chứng minh rằng 2 + 3 + 3 ≥1.
a +b +c
3 3
a +b +c 2 3
a + b3 + c 2
.c
Bài 21. Cho a,b,c là các số thực dương thoả mãn điều kiện
ok
1 1 1
a + b + c= 2 + 2 + 2 .
a b c
bo

7 a 2b + 1 + 7b 2 c + 1 + 7c 2 a + 1 ≤ 2(a + b + c) .
3 3 3
Chứng minh rằng
Bài 22. Cho a,b,c là các số thực dương có tổng bằng 1. Chứng minh
et

 a b c  13
4abc  + +  + 1 ≥ ( ab + bc + ca ) .
vi

 ( a + 1) 2
( b + 1) ( c + 1) 
2 2 4
ng

D. HƯỚNG DẪN GIẢI – ĐÁP SỐ


Bài 1. Sử dụng bất đẳng thức Holder ta có
a

( ) ( ) ≥ (a ).
2 2 3
=(1 + 1 + 1) a3 + b3 + c3
kh

3 a 3 + b3 + c 3 2
+ b2 + c2
Bất đẳng thức được chứng minh. Đẳng thức xảy ra khi và chỉ khi a= b= c .
Bài 2. Gọi P là biểu thức vế trái và đặt S = a ( b + c ) + b ( c + a ) + c ( a + b ) .

Sử dụng bất đẳng thức Holder ta có P.P.S ≥ ( a + b + c ) =


3
1.
2 2 3
Mặt khác S = 2 ( ab + bc + ca ) ≤ ( a + b + c )2 = ⇒ P ≥ .
3 3 2
1
Bất đẳng thức được chứng minh. Đẳng thức xảy ra khi và chỉ khi a= b= c= .
3
297
Khám phá tư duy Kỹ thuật giải bất ĐT Bài toán Max – Min – Đặng Thành Nam

Bài 3. Gọi P là biểu thức vế trái và đặt


( ) ( ) (
S = a a 2 + 8bc + b b 2 + 8ca + c c 2 + 8ab . )
Sử dụng bất đẳng thức Holder ta có P.P.S ≥ ( a + b + c )
3

Vậy ta cần chứng minh ( a + b + c ) ≥ S


3

( ) ( ) (
⇔ ( a + b + c ) ≥ a a 2 + 8bc + b b 2 + 8ca + c c 2 + 8ab
3
)
⇔ ( a + b )( b + c )( c + a ) ≥ 8abc
Bất đẳng thức hiển nhiên đúng theo AM – GM. Đẳng thức xảy ra khi và chỉ khi

n
a= b= c .
Bài 4. Gọi P là biểu thức vế trái và đặt

.v
( ) ( ) (
S= a b 2 + c 2 + 7 + b c 2 + a 2 + 7 + c a 2 + b 2 + 7 . )

om
Sử dụng bất đẳng thức Holder ta có P.P.S ≥ ( a + b + c ) .
3

Vậy ta cần chứng minh ( a + b + c ) ≥ S


3 .c
⇔ ( a + b + c ) ≥ 7 ( a + b + c ) + ab ( a + b ) + bc ( b + c ) + ca ( c + a )
3
ok
⇔ ( a + b + c ) ≥ 7 ( a + b + c ) + ( a + b + c )( ab + bc + ca ) − 3abc
3
bo

⇔ ( a + b + c ) ≥ 7 ( a + b + c ) + ( a + b + c )( ab + bc + ca ) − 3
3

Luôn đúng. Bất đẳng thức được chứng minh. Đẳng thức xảy ra khi và chỉ khi
et

a= b= c= 1 .
Bài 5. Gọi P là biểu thức vế trái và đặt
vi

S= a (1 + b + bc ) + b (1 + c + ca ) + c (1 + a + ab ) .
ng

3
 2 2 2
Sử dụng bất đẳng thức Holder ta có P.P.S ≥  a 3 + b 3 + c 3  .
 
a

 
kh

3
 2 2 2
Vậy ta cần chứng minh  a 3 + b 3 + c 3  ≥ 3 ( 3 + ab + bc + ca + 3abc )
 
 
2 2 2 2 2
⇔ ∑ a 2 + 3∑ a 3 b 3  a 3 + b 3  + 6 ( abc ) 3 ≥ 9 + 3 ( ab + bc + ca ) + 9abc .
 
 
2 2 2 2
Sử dụng bất đẳng thức AM – GM ta có ∑ a 3 b 3  a 3 + b 3  ≥ 2 ( ab + bc + ca ) .
 
 

298
Cty TNHH MTV DVVH Khang Việt

2
Vậy ta cần chứng minh ab + bc + ca + 6 ( abc ) 3 ≥ 9abc .
Bất đẳng thức cuối luôn đúng do
3
2
a+b+c 2
ab + bc + ca + 6 ( abc ) ≥ 9 ( abc ) ≥ 9abc vì abc ≤ 
3  =
1. 3
 3 
Bất đẳng thức được chứng minh. Đẳng thức xảy ra khi và chỉ khi
a= b= c= 1 hoặc a= 3, b= c= 0 và các hoán vị.
Bài 6. Chú ý a5 − a 2 + 3 − a3 + 2 = ( ) ( a − 1)2 ( a + 1) ( a 2 + a + 1) ≥ 0 .
Thiết lập tương tự và ta chứng minh

n
(a 3
)( )(
+ 1 + 1 1 + b3 + 1 1 + 1 + c 3 ≥ ( a + b + c ) . ) 3

.v
Đây chính là bất đẳng thức Holder.

om
Bài 7. Sử dụng bất đẳng thức Holder ta có

(x )( )
2
≥ ( x + y + z) .
3
2
+ y2 + z2 x+ y+ z

( x + y + z )4
.c
Vậy ta cần chứng minh ≥ 27
ok
x2 + y 2 + z 2

⇔ ( x + y + z ) ≥ 27 ( x + y + z )( x + y + z − 2 )
4
bo

⇔ ( x + y + z ) + 54 ≥ 27 ( x + y + z )
3

⇔ ( x + y + z − 3) ( x + y + z + 6) ≥ 0
2
et

Bất đẳng thức được chứng minh. Đẳng thức xảy ra khi và chỉ khi x= y= z= 1 .
vi

Bài 8. Sử dụng bất đẳng thức Holder ta có


ng

(a )( )
2
≥ (a + b + c) =
3
2
+ b2 + c2 a+ b+ c 27 .

Vậy ta cần chứng minh 27 ≥ ( ab + bc + ca ) a 2 + b 2 + c 2 .


2
( )
a
kh

Bất đẳng thức cuối đúng theo AM – GM


3
 2 ( ab + bc + ca ) + a 2 + b 2 + c 2 
(
( ab + bc + ca ) a + b + c ≤ 
2 2 2
3
2
)  =

27 .
 
Bất đẳng thức được chứng minh. Đẳng thức xảy ra khi và chỉ khi a= b= c= 1 .
Bài 9. Gọi P là biểu thức vế trái và đặt S = x 2 y 2 + y 2 z 2 + z 2 x 2 .

( )
3
Sử dụng bất đẳng thức Holder ta có P.P.S ≥ x 2 + y 2 + z 2 .
Ta cần chứng minh

299
Khám phá tư duy Kỹ thuật giải bất ĐT Bài toán Max – Min – Đặng Thành Nam

(x )
3
2
+ y2 + z2 x4 + y 4 + z 4
≥9
x2 y 2 + y 2 z 2 + z 2 x2 3

( ) ( ) 3( x )
3
⇔ x2 + y 2 + z 2 ≥ 3 x2 y 2 + y 2 z 2 + z 2 x2 4
+ y4 + z4

Đặt a = x 2 + y 2 + z 2 , b = x 2 y 2 + y 2 z 2 + z 2 x 2 ⇒ x 4 + y 4 + z 4 = a 2 − 2b .

Bất đẳng thức trở thành a3 ≥ 3b 3 a 2 − 2b ( )


( ) (a )
2
⇔ a 6 − 27 a 2b + 54b3 ≥ 0 ⇔ a 2 − 3b 2
+ 6b ≥ 0 .

n
Luôn đúng. Bất đẳng thức được chứng minh. Đẳng thức xảy ra khi và chỉ khi

.v
x= y= z .

om
Bài tập tương tự
Chứng minh x,y,z là các số thực dương thoả mãn x 4 + y 4 + z 4 =
3 ta có
x2 y 2 z 2
+ + ≥3.
.c
ok
y z x
Bài 10. Sử dụng bất đẳng thức Holder ta có

(1 + a )(1 + b )(1 + b ) ≥ (1 + ab )
bo

3 3 3 2 3

(1 + b )(1 + c )(1 + c ) ≥ (1 + bc ) .
3 3 3 2 3
et

(1 + c )(1 + a )(1 + a ) ≥ (1 + ca )
3 3 3 2 3
vi

Nhân theo vế 3 bất đẳng thức trên ta có điều phải chứng minh.
ng

Bài 11. Sử dụng bất đẳng thức Holder ta có


 a3 b3 c 3 
(1 + 1 + 1)( x + y + z )  + +  ≥ (a + b + c)
3
a


 x y z 
kh

a 3 b3 c 3 ( a + b + c )
3
⇒ + + ≥
x y z 3( x + y + z )
Bất đẳng thức được chứng minh.
Bài 12. Bất đẳng thức tương đương với:
1 + abc 1 + abc 1 + abc 3
+ + ≥ .
(1 + a )3 (1 + b )3 (1 + c )3 4
Sử dụng bất đẳng thức Holder ta được:

300
Cty TNHH MTV DVVH Khang Việt

1 + abc
(1 + abc ) 1 +
a  a  bc
1 +  ≥ (1 + a ) ⇒
3
≥ .
 b  c  (1 + a ) ( a + b )( a + c )
3

1 + abc ac 1 + abc ab
Tương tự: ≥ ; ≥ .
(1 + b ) 3
( b + a )( b + c ) (1 + c ) (
3 c + a )( c + b )
Cộng theo vế 3 bất đẳng thức trên và cần chứng minh
bc ac ab 3
+ + ≥ .
( a + b )( a + c ) ( b + a )( b + c ) ( c + a )( c + b ) 4
3
⇔ ab ( a + b ) + bc ( b + c ) + ca ( c + a ) ≥ ( a + b )( b + c )( c + a ) .

n
4
⇔ ab ( a + b ) + bc ( b + c ) + ca ( c + a ) ≥ 6abc .

.v
Bất đẳng thức cuối theo AM-GM.

om
Bài toán được chứng minh đẳng thức xảy ra khi và chỉ khi a= b= c .
Bài 13. Gọi P là biểu thức vế trái và đặt
S =a ( a + 2b ) + b ( b + 2c ) + c ( c + 2a ) =( a + b + c ) =1 .
.c 2

Sử dụng bất đẳng thức Holder ta có P.P.P.S ≥ ( a + b + c ) ⇔ P3 ≥ 1 ⇔ P ≥ 1 .


4
ok
1
Bất đẳng thức được chứng minh đẳng thức xảy ra khi và chỉ khi a= b= c= .
bo

3
Bài 14. Với a= b= c bất đẳng thức trở thành

( ) ( )
3 2
9 a 4 + 1 ≥ 8 a 6 + a3 + 1
et

3 2.
 1   
vi

1
⇔ 9  a 2 + 2  ≥ 8  a 3 + 3 + 1
 a   a 
ng

1
Đặt x = a + , ( x ≥ 2 ) bất đẳng thức trở thành
a
a

( ) ( ) ( x ( x − 8) + 4 ( x − 5) + 6 x ) ≥ 0 .
3 2
⇔ ( x − 2)
2
9 x2 − 2 ≥ 8 x3 − 3 x + 1 3 3 2
kh

Bất đẳng thức luôn đúng với x ≥ 2 .

( ) ( )
3 2
Áp dụng ta có 9 a 4 + 1 ≥ 8 a 6 + a3 + 1

9 ( b + 1) ≥ 8(b + b + 1)
4 3 6 3 2

9 ( c + 1) ≥ 8(c + c + 1)
4 3 6 3 2

Nhân theo vế 3 bất đẳng thức trên ta được

301
Khám phá tư duy Kỹ thuật giải bất ĐT Bài toán Max – Min – Đặng Thành Nam

( ) (b + 1) ( c + 1) ( ) (b ) (c )
3 3 3 2 2 2
93 a 4 + 1 4 4
≥ 83 a 6 + a3 + 1 6
+ b3 + 1 6
+ c3 + 1 .
Mặt khác theo bất đẳng thức Holder ta có

(a )( )( ) ( )
3
6
+ a3 + 1 b6 + b3 + 1 c 6 + c3 + 1 ≥ a 2b 2 c 2 + abc + 1 .
Từ đó ta có điều phải chứng minh.
Đẳng thức xảy ra khi và chỉ khi a= b= c= 1 .
Bài 15. Gọi P là biểu thức vế trái và đặt S = c ( a + b ) + b ( c + a ) + a ( b + c ) .
2 2 2

Sử dụng bất đẳng thức Holder ta có P.P.S ≥ ( a + b + b + c + c + a ) = 8 ( a + b + c ) .


3 3

n
Vậy ta cần chứng minh

.v
8( a + b + c) 16 ( a + b + c )
3 3

a (b + c ) + b (c + a ) + c ( a + b) 3 ( a + b )( b + c )( c + a )

om
2 2 2

⇔ 3 ( a + b )( b + c )( c + a ) ≥ 2  a ( b + c ) + b ( c + a ) + c ( a + b ) 
2 2 2
 .
⇔ ( a + b + c )( ab + bc + ca ) ≥ 9abc
.c
Luôn đúng theo AM – GM.
ok
Bài 16. Gọi P là biểu thức vế trái và đặt S = a + b + c .
Sử dụng bất đẳng thức Holder ta có
bo

3 3
 a b c  3 27
P.S .S ≥  + +  ≥  ⇒ P≥ .
b+c c+a a+b 2 8( a + b + c)
2
et

Bất đẳng thức được chứng minh. Đẳng thức xảy ra khi và chỉ khi a= b= c .
vi

Bài 17. Gọi P là biểu thức vế trái và đặt


=S 8ab ( 4a + 4b + c ) + 8bc ( 4b + 4c + a ) + 8ca ( 4c + 4a + b )
ng

.
= 32 ( a + b + c )( ab + bc + ca ) − 72abc
Sử dụng bất đẳng thức Holder ta có
a

P.P.S ≥ ( a + b + b + c + c + a ) = 8 ( a + b + c ) .
3 3
kh

Vậy ta chứng minh 8 ( a + b + c ) ≥ S


3

⇔ 8 ( a + b + c ) ≥ 32 ( a + b + c )( ab + bc + ca ) − 72abc
3
.
⇔ ( a + b + c ) + 9abc ≥ 4 ( a + b + c )( ab + bc + ca )
3

Đây chính là bất đẳng thức Schur bậc 3. Bất đẳng thức được chứng minh. Đẳng
thức xảy ra khi và chỉ khi a= b= c .
Bài 18. Gọi P là biểu thức vế trái và đặt

302
Cty TNHH MTV DVVH Khang Việt

S = a ( b + c ) + 5c 2  + b ( c + a ) + 5a 2  + c ( a + b ) + 5b 2  .
2 2 2
     
Sử dụng bất đẳng thức Holder ta có P 2 .S ≥ ( a + b + c ) .
3

Vậy ta cần chứng minh


( a + b + c )3 ≥ a ( b + c )2 + 5c 2  + b ( c + a )2 + 5a 2  + c ( a + b )2 + 5b2 
⇔ a ( a − b) + b (b − c ) + c (c − a ) ≥ 0
2 2 2

Bất đẳng thức cuối luôn đúng ta có đpcm. Đẳng thức xảy ra khi và chỉ khi
a= b= c .

n
Bài 19. Sử dụng bất đẳng thức Holder ta có

.v
1 3 1 1 1 1 1
3 6a + + 6b + + 3 6c=
+ 3 ( 6ab + 1) + 3 ( 6bc + 1) + 3 ( 6ca + 1)
c a b a b c

om
1 1 1 3 1
≤ 3 3  + +  ( 6ab + 1 + 6bc + 1 + 6ca +=
1) ≤
a b c 3
abc abc

 ab + bc + ca 
Vì theo AM – GM ta có abc ≤   =
1
.
.c 3

 3 
ok
3 3
1
Bất đẳng thức được chứng minh. Đẳng thức xảy ra khi và chỉ khi a= b= c= .
bo

3
Bài 20. Sử dụng bất đẳng thức C –S ta có
a b c
+ 3 + 3
et

a +b +c
2 3 3
a +b +c
2 3
a + b3 + c 2
vi

a2 b2 c2
= + +
a3 + ab3 + ac3 a3b + b3 + bc3 ca3 + cb3 + c3
ng


( a + b + c )2 .
( ) (
a3 + b3 + c3 + ab a 2 + b 2 + bc b 2 + c 2 + ca c 2 + a 2 ) ( )
a

( a + b + c )2
kh

=
( ) (
a 4 + b 4 + c 4 + ab a 2 + b 2 + bc b 2 + c 2 + ca c 2 + a 2 ) ( )
=
( a + b + c)
2
a+b+c
( a + b + c ) ( a 3 + b3 + c 3 ) a + b3 + c 3
3

Ta chứng minh a + b + c ≥ a3 + b3 + c3 .

( ) ≥ (a )
2 3
⇔ ( a + b + c ) a 4 + b4 + c4 3
+ b3 + c 3
Đúng theo bất đẳng thức Holder.

303
Khám phá tư duy Kỹ thuật giải bất ĐT Bài toán Max – Min – Đặng Thành Nam

Bài 21. ọi P là biểu thức vế trái sử dụng bất đẳng thức Holder ta có
3
 7 a 2b + 1 7b 2 c + 1 7c 2 a + 1 
P = 3
3
.a.a + 3 .b.b + 3 .c.c 
 a 2
b 2
c 2 
 
 7 a 2 b + 1 7b 2 c + 1 7 c 2 a + 1 
 (a + b + c) = 8 ( a + b + c )
3
≤ + + 2
 a2 2 2
 b c 
Bất đẳng thức được chứng minh. Đẳng thức xảy ra khi và chỉ khi a= b= c= 1 .
Bài 22. Sử dụng bất đẳng thức Holder ta có
a
+
b
+ ≥
c ( a + b + c)
3
=
1
.

n
( a + 1) ( b + 1) ( c + 1) ( a ( a + 1) + b ( b + 1) + c ( c + 1) ) ( t + 1)2
2 2 2 2

.v
1 1
Với t = a 2 + b 2 + c 2 ≥ ( a + b + c )2 = .

om
3 3
p2 − t 1 − t
Đặt p = a + b + c = 1; q = ab + bc + ca = = ; r = abc .
2 2
Mặt khác theo bất đẳng thức Schur bậc ba ta có
.c
abc ≥ ( a + b − c )( b + c − a )( c + a − b )
ok
4q − 1 1 − 2t .
⇒ abc ≥ (1 − 2a )(1 − 2b )(1 − 2c ) ⇒ 9r + 1 − 4q ≥ 0 ⇔ r ≥ =
9 9
bo

Vậy ta chỉ cần chứng minh


4.
1 − 2t
.
1 13
(
+ 1 ≥ (1 − t ) ⇔ ( 3t − 1) 39t 2 + 76t + 13 ≥ 0 . )
et

9 ( t + 1) 2 8
vi

1
Bất đẳng thức cuối đúng ta có đpcm. Đẳng thức xảy ra khi và chỉ khi a= b= c= .
3
ng

CHỦ ĐỀ 8: KỸ THUẬT SỬ DỤNG BẤT ĐẲNG THỨC CHEBYSHEV


a

A. NỘI DUNG PHƯƠNG PHÁP


kh

1. Bất đẳng thức Chebyshev với hai dãy đơn điệu cùng chiều
a1 ≥ a2 ≥ ... ≥ an a1 ≤ a2 ≤ ... ≤ an
Nếu có  hoặc  thì ta có
b1 ≥ b2 ≥ ... ≥ bn b1 ≤ b2 ≤ ... ≤ bn
n ( a1b1 + a2b2 + ... + an bn ) ≥ ( a1 + a2 + ... + an )( b1 + b2 + ... + bn ) .
a1= a2= ...= an
Đẳng thức xảy ra khi và chỉ khi  .
b1= b2= ...= bn
Chứng minh. Ta có đẳng thức

304
Cty TNHH MTV DVVH Khang Việt

n
n ( a1b1 + a2b2 + ... + anbn ) − ( a1 + a2 + ... + an )( b1 + b2 + ... +=
bn ) ∑ ( ai − a j )( bi − b j ) ≥ 0 .
1≤i ≠ j

( )( )
Vì ai − a j bi − b j ≥ 0, ∀i, j =
1, n .
2. Bất đẳng thức Chebyshev với hai dãy đơn điệu ngược chiều
a1 ≥ a2 ≥ ... ≥ an a1 ≤ a2 ≤ ... ≤ an
Nếu có  hoặc  thì ta có
b1 ≤ b2 ≤ ... ≤ bn b1 ≥ b2 ≥ ... ≥ bn
n ( a1b1 + a2b2 + ... + an bn ) ≤ ( a1 + a2 + ... + an )( b1 + b2 + ... + bn ) .
a1= a2= ...= an
Đẳng thức xảy ra khi và chỉ khi 

n
.
b1= b2= ...= bn

.v
Chứng minh tương tự cho dãy đơn điệu cùng chiều.
Bất đẳng thức Chebyshev được áp dụng cho các bất đẳng thức đối xứng ba

om
biến và bài toán được giải theo cách này khá đơn giản.

B. BÀI TẬP MẪU .c


Bài 1. Cho a,b,c là các số thực dương thoả mãn điều kiện a 2 + b 2 + c 2 =
1.
ok
a3 b3 c3 1
Chứng minh rằng + + ≥ .
b+c a+c a+b 2
bo

Lời giải
Do a,b,c đối xứng, không mất tính tổng quát giả sử a ≥ b ≥ c
et

 a 2 ≥ b2 ≥ c2

⇒  a b c tức là hai dãy đơn điệu cùng chiều.
vi

 ≥ ≥
b + c a + c a + b
ng

Áp dụng bất đẳng thức Chebyshev ta có :


a b c a 2 + b2 + c2  a b c  1 3 1
a2 . + b2 . + c2 . ≥ . + + ≥ . =
a

b+c a+c a+b 3 b+c a+c a+b 3 2 2


kh

1
Bất đẳng thức được chứng minh. Dấu bằng xảy ra khi a = b = c=
3
Chú ý. Ta có thể chứng minh bằng C –S

( )
2
a3 b3 c3 a 2 + b2 + c2
+ + ≥
b + c a + c a + b a (b + c ) + b (c + a ) + c ( a + b)
1 1 1
= ≥ =
2
(
2 ( ab + bc + ca ) 2 a + b + c
2 2 2 )
305
Khám phá tư duy Kỹ thuật giải bất ĐT Bài toán Max – Min – Đặng Thành Nam

Bài 2. Chứng minh với mọi số thực dương x,y,z ta có


1 1 1 x+ y+z 3
+ + ≤ + .
x + y y + z z + x xy + yz + x 2 ( x + y + z )
Lời giải
Bất đẳng thức đã cho tương đương với:
 1 1 1  3 ( xy + yz + zx )
( xy + yz + zx )  + + ≤ x+ y+z+
 x+ y y+z z+x 2( x + y + z )
xy yz zx 3 ( xy + yz + zx )
⇔ + + ≤
x+ y y+z z+x 2( x + y + z )

n
 

.v
 1 1 1 
⇔ ( x + y ) + ( y + z ) + ( z + x )  .  + +  ≤ 3 ( xy + yz + zx )
1+1 1+1 1+1

om
x y y z z x
 
Không mất tính tổng quát giả sử

x ≥ y ≥ z ⇒ x + y ≥ x + z ≥ y + z;
1
1 1

1 1
1.c ≥
1
1 1
.
+ + +
ok
x y x z z y
Sử dụng bất đẳng thức cho hai dãy đơn điệu cùng chiều trên ta có ngay điều
bo

phải chứng minh.


Bất đẳng thức xảy ra khi và chỉ khi x= y= z .
et

Bài 3. Cho x,y,z là các số thực có tổng bằng 1. Chứng minh


x y z 9
+ 2 + 2 ≤ .
vi

x + 1 y + 1 z + 1 10
2
ng

Lời giải
Trước hết chuyển bài toán về đánh giá với các số dương
x y z x y z
a

+ + ≤ + + .
x +1
2
y +1
2
z +1
2
x +1
2
y +1
2
z +1
2
kh

Nên ta chỉ cần chứng minh bất đẳng thức trong trường hợp cả ba số đều không âm.
Vì bất đẳng thức đối xứng với ba biến nên không mất tính tổng quát giả sử
x ≥ y ≥ z khi đó
x y z
x 2 + 1 ≥ y 2 + 1 ≥ z 2 + 1; ≥ ≥ .
x +1
2
y +1
2
z +1
2

Thật vậy ta chỉ cần chứng minh


x y
≥ 2 ⇔ xy 2 + x ≥ x 2 y + y ⇔ ( x − y )(1 − xy ) ≥ 0 .
x +1 y +1
2

306
Cty TNHH MTV DVVH Khang Việt

2
 x+ y 1 3
Bất đẳng thức đúng do x − y ≥ 0; 1 − xy ≥ 1 −   ≥1− = > 0 .
 2  4 4
Vậy ta có hai dãy đơn điệu cùng chiều
 x z 
(
x 2 + 1; y 2 + 1; z 2 + 1 ;  2 ; 2
y
)
 x +1 y +1 z +1
; 2  .

Áp dụng bất đẳng thức Chebyshev ta có


 x z 
(
x 2 + 1 + y 2 + 1 + z 2 + 1  2 + 2 )
 x +1 y +1 z +1
y
+ 2 

 z 
( ) ( ) ( )

n
x y
≤ 3  x2 + 1 . 2 + y2 + 1 . 2 + z2 + 1 . 2 
 x +1 y +1 z + 1

.v
x y z 3( x + y + z )
⇒ 2 + 2 + 2 ≤ 2

om
x + 1 y + 1 z + 1 x + y2 + z2 + 3
3( x + y + z )9 1
Vậy ta chứng minh ≤
⇔ x2 + y 2 + z 2 ≥ .
x + y + z + 3 10
2 2 2 3 .c
1 1
Bất đẳng thức đúng vì x 2 + y 2 + z 2 ≥ ( x + y + z )2 = .
ok
3 3
1
Bài toán được chứng minh. Đẳng thức xảy ra khi và chỉ khi x= y= z= .
bo

3
1 1 1
Bài 3. Cho a,b,c là các số thực dương thoả mãn điều kiện a + b + c ≥ + + .
et

a b c
Chứng minh rằng 2 ( a + b + c ) ≥ a 2 + 3 + b 2 + 3 + c 2 + 3 .
vi

Lời giải
Bất đẳng thức đã cho tương đương với
ng

( 2a − )(
a 2 + 3 + 2b − b 2 + 3 + 2c − c 2 + 3 ≥ 0 )( )
a

( )
3 a2 − 1 (
3 b2 − 1 ) (
3 c2 − 1 )
kh

⇔ + + ≥0
2a + a 2 + 3 2b + b 2 + 3 2c + c 2 + 3
1 1 1 a 2 − 1 b2 − 1 c2 − 1
Chú ý điều kiện a + b + c ≥ + + ⇔ + + ≥ 0.
a b c a b c
Nên ta biến đổi bất đẳng thức về dạng
a2 − 1 b2 − 1 c2 − 1
a + b + c ≥0.
3 3 3
2 + 1+ 2 2 + 1+ 2 2 + 1+ 2
a b c

307
Khám phá tư duy Kỹ thuật giải bất ĐT Bài toán Max – Min – Đặng Thành Nam

Không mất tính tổng quát giả sử a ≥ b ≥ c , khi đó


1 1 1
≥ ≥
3 3 3
2 + 1+ 2 + 1+ 2 + 1+
a2 b2 c2
a 2 − 1 b2 − 1 c2 − 1
≥ ≥
a b c
Khi đó sử dụng bất đẳng thức Chebyshev ta có
a2 − 1 b2 − 1 c2 − 1
a + b + c

n
3 3 3
2 + 1+ 2 2 + 1+ 2 2 + 1+
c2

.v
a b
 

om
 
1  a −1 b −1 c −1 
2 2 2
1 1 1 ≥0
≥  + +  + +
3  a b c   3 3 3 
 2 + 1+ 2 2 + 1+ 2 2 + 1+ 2 
 a b .c c 
Bất đẳng thức được chứng minh. Đẳng thức xảy ra khi và chỉ khi a= b= c= 1 .
ok
Bài tập tương tự
1 1 1
Cho n số thực dương thoả mãn điều kiện a1 + a2 + ... + an ≥ + + ... + .
bo

a1 a2 an

Chứng minh rằng 2 ( a1 + a2 + ... + an ) ≥ a12 + 3 + a22 + 3 + ... + an2 + 3 .


et

Bài 4. Cho x,y,z là các số thực dương. Chứng minh


6 ( x + y + z)
vi

x+ y y+z z+x
+ + ≥ .
z x y xy + yz + zx
ng

Lời giải
x+ y y+z z+x x+ y x+z y+z
a

Ta có + + = + + .
z x y z ( x + y) y( x + z) x( y + z)
kh

Không mất tính tổng quát giả sử x ≥ y ≥ z ⇒ x + y ≥ x + z ≥ z + y và


x+ y x+z y+z
≥ ≥
z ( x + y) y( x + z) x( y + z)
z ( x + y) ≤ y ( z + x) ≤ x ( y + z )
Khi đó sử dụng bất đẳng thức Chebyshev cho hai dãy đơn điệu ngược chiều nên
ta có

308
Cty TNHH MTV DVVH Khang Việt

 x+ y 

 z ( x + y)
+
x+z
y( x + z)
+
y+z
x ( y + z ) 
(
 z ( x + y) + y ( z + x) + x ( y + z ) )

≥ 3 ( x + y + x + z + y + z )= 6 ( x + y + z )
Kết hợp sử dụng bất đẳng thức C –S ta có
x+ y x+z y+z 6( x + y + z )
+ + ≥
z ( x + y) y( x + z) x( y + z) z ( x + y) + y ( z + x) + x ( y + z )
6( x + y + z ) 6 (x + y + z)
≥ =
6 ( xy + yz + zx ) xy + yz + zx

n
Bất đẳng thức được chứng minh. Đẳng thức xảy ra khi và chỉ khi x= y= z .

.v
Bài tập tương tự

om
Chứng minh rằng với mọi x,y,z dương ta có
8( x + y + z )
2 2
3 x+ y z+x y+z
 +  ≥ + .
2  z y  ( x + y )( y + z )( z + x )
x .c
Bài 5. Cho a,b,c là các số thực không âm chứng minh
ok
3 ( a + b + c ) ≥ a 2 + 8bc + b 2 + 8ca + c 2 + 8ab .
Lời giải
bo

+ Nếu tồn tại một số bằng 0 bất đẳng thức có dạng 2 ( x + y ) ≥ 2 2 xy luôn đúng.
+ Ta xét cả ba số đều dương khi đó viết lại bất đẳng thức dưới dạng
et

(3a − )(
a 2 + 8bc + 3b − b 2 + 8ca + 3c − c 2 + 8ab ≥ 0 )( )
vi

(
8 a 2 − bc ) (
8 b 2 − ca ) (
8 c 2 − ab )
ng

⇔ + + ≥0
3a + a 2 + 8bc 3b + b 2 + 8ca 3c + c 2 + 8ab
(a )
− bc ( b + c ) (b )
− ca ( c + a ) (c )
− ab ( a + b )
a

2 2 2
⇔ + + ≥0
( ) ( ) ( )
kh

3a + a 2 + 8bc ( b + c ) 3b + b 2 + 8ca ( c + a ) 3c + c 2 + 8ab ( a + b )

( ) ( )
Chú ý a 2 − bc ( b + c ) + b 2 − ca ( c + a ) + c 2 − ab ( a + b ) =
0. ( )
Do vậy sử dụng bất đẳng thức Chebyshev ta chỉ cần chứng minh hai dãy sau
đơn điệu ngược chiều
(a 2
) ( )
− bc ( b + c ) , b 2 − ca ( c + a ) , c 2 − ab ( a + b ) ( )
(3a + ) ( )
a 2 + 8bc ( b + c ) , 3b + b 2 + 8ca ( c + a ) , 3c + c 2 + 8ab ( a + b ) ( )
309
Khám phá tư duy Kỹ thuật giải bất ĐT Bài toán Max – Min – Đặng Thành Nam

Không mất tính tổng quát giả sử a ≥ b ≥ c ta có


(a 2
) ( ) (
− bc ( b + c ) − b 2 − ca ( c + a ) =( a − b ) c 2 + ab + bc + ca ≥ 0 )
(b 2
− ca ) ( c + a ) − ( c 2
− ab ) ( a + b ) = ( b − c ) ( a 2
+ ab + bc + ca ) ≥ 0

( ) (
Và 3a + a 2 + 8bc ( b + c ) ≤ 3b + b 2 + 8ca ( c + a ) )
⇔ 3c ( a − b ) + ( b + c ) a 2 + 8bc − ( c + a ) b 2 + 8ca ≤ 0

( b + c )2 ( a 2 + 8bc ) − ( c + a )2 ( b2 + 8ca )
⇔ 3c ( a − b ) + ≤0
( b + c ) a 2 + 8bc + ( c + a ) b2 + 8ca

n
 ( )
8 a 2 + b 2 + c 2 + 6ab + 15c ( a + b ) 

.v

⇔ 3c ( a − b ) 1 − ≤0
 ( b + c ) a 2 + 8bc + ( c + a ) b 2 + 8ca 

om
 
Bất đẳng thức cuối đúng vì
a 2 + 8bc ≤ a + b + c; b 2 + 8ca ≤ a + b + c .c
⇒ ( b + c ) a 2 + 8bc + ( c + a ) b 2 + 8ca ≤ ( a + b + c )( a + b + 2c )
ok
( )
< 8 a 2 + b 2 + c 2 + 6ab + 15c ( a + b )
bo

Bất đẳng thức được chứng minh. Đẳng thức xảy ra khi và chỉ khi a= b= c .
et

C. BÀI TẬP RÈN LUYỆN


Bài 1. Cho a,b,c là các số thực dương thoả mãn điều kiện a + b + c ≤ 3abc .
vi

1 1 1
Chứng minh rằng + + ≤1.
1 + 2bc 1 + 2ca 1 + 2ab
ng

1 1 1
Bài 2. Cho các số thực a, b, c > 1 thoả mãn điều kiện 2 + 2 + 2 =
1.
a −1 b −1 c −1
a

1 1 1
Chứng minh rằng + + ≤1.
kh

a +1 b +1 c +1
Bài 3. Cho x,y,z là các số thực dương có tổng bằng 1. Chứng minh
x y z 27
+ + ≤ .
xyz + x + 1 xyz + y + 1 xyz + z + 1 31
2 2 2

Bài 4. Cho a,b,c là các số thực dương thoả mãn điều kiện ab + bc + ca = 3.
1 1 1 3
Chứng minh rằng + + ≤ .
1 + a ( b + c ) 1 + b ( c + a ) 1 + c ( a + b ) 1 + 2abc
2 2 2

310
Cty TNHH MTV DVVH Khang Việt

Bài 5. Cho x,y,z là các số thực dương. Chứng minh


16 x 16 y 16 z
1+ + 1+ + 1+ ≥9.
y+z z+x x+ y

D. HƯỚNG DẪN GIẢI – ĐÁP ÁN


Bài 1. Bất đẳng thức đã cho tương đương với
a b c
+ + ≤1.
a + 2abc b + 2abc c + 2abc
Không mất tính tổng quát giả sử a ≥ b ≥ c. Khi đó ta có
a + 2abc ≥ b + 2abc ≥ c + 2abc

n
1 1 1 a b c

.v
≥ ≥ ⇒ ≥ ≥
1 + 2bc 1 + 2ca 1 + 2ab a + 2abc b + 2abc c + 2abc

om
Sử dụng bất đẳng thức Chebyshev cho hai dãy đơn điệu cùng chiều

( a + 2abc; b + 2abc; c + 2abc ) ;  


a b c
; ; 
 a + 2abc b + 2abc c + 2abc 
.c
ta có
 
ok
a b c
3 ( a + b + c ) ≥ ( a + b + c + 6abc )  + + 
 a + 2 abc b + 2 abc c + 2 abc 
( + + ) ≤1
bo

a b c 3 a b c
⇒ + + ≤
a + 2abc b + 2abc c + 2abc a + b + c + 6abc
Bởi vì a + b + c ≤ 3abc .
et

Bất đẳng thức được chứng minh. Đẳng thức xảy ra khi và chỉ khi a= b= c= 1 .
Bài tập tương tự
vi

1 1 1
Cho a,b,c là các số thực dương thoả mãn điều kiện + + ≥1.
ng

1 + 2bc 1 + 2ca 1 + 2ab


Chứng minh rằng a + b + c ≥ 3abc .
a

Bài 2. Bất đẳng thức đã cho tương đương với


a−2 b−2 c−2
kh

1 1 1 1 1 1
− + − + − ≥0⇔ + + ≥0.
3 a +1 3 b +1 3 c +1 a +1 b +1 c +1
Chú ý điều kiện được viết lại thành
1 1 1 1 1 1
− + 2 − + 2 − = 0
a −1 3 b −1 3 c −1 3
2

4 − a2 4 − b2 4 − c2
⇔ + =0+
a 2 − 1 b2 − 1 c2 − 1
a−2 a+2 b−2 b+2 c−2 c+2
⇔ . + . + . =
0
a +1 a −1 b +1 b −1 c +1 c −1

311
Khám phá tư duy Kỹ thuật giải bất ĐT Bài toán Max – Min – Đặng Thành Nam

a − 2 b − 2 c − 2
 a + 1 ≥ b + 1 ≥ c + 1
Không mất tính tổng quát a ≥ b ≥ c ⇒  .
 a + 2 b + 2 c + 2
≤ ≤
 a − 1 b − 1 c − 1
Sử dụng bất đẳng thức Chebyshev cho hai dãy đơn điệu ngược chiều trên ta có
ngay đpcm.
Bất đẳng thức được chứng minh. Đẳng thức xảy ra khi và chỉ khi a= b= c= 2 .
Tổng quát cho n số thực a1 , a2 ,..., an > 1 thoả mãn điều kiện
1 1 1
+ + ... + =
1.
a12 −1 a22 −1 an2 −1

n
.v
1 1 1
Chứng minh rằng + + ... + ≤ n +1 −1.
a1 + 1 a1 + 1 an + 1

om
Bài 3. Không mất tính tổng quát giả sử
 xyz + x 2 + 1 ≥ xyz + y 2 + 1 ≥ xyz + z 2 + 1

x≥ y≥z⇒ x y z
.c
 ≥ ≥
 xyz + x + 1 xyz + y + 1 xyz + z + 1
2 2 2
ok
Sử dụng bất đẳng thức Chebyshev cho hai dãy đơn điệu cùng chiều trên ta được
x y z 3( x + y + z )
bo

+ + ≤
xyz + x + 1 xyz + y + 1 xyz + z + 1 3xyz + 3 + x 2 + y 2 + z 2
2 2 2

3
=
et

3 xyz + 3 + x 2 + y 2 + z 2
Do đó ta chỉ cần chứng minh
vi

3 27 4
≤ ⇔ 3 xyz + x 2 + y 2 + z 2 ≥ .
ng

3 xyz + 3 + x + y + z
2 2 2 31 9
Chú ý theo bất đẳng thức Schur bậc ba ta có
a

9 xyz
≥ 4 ( xy + yz + zx ) − ( x + y + z )
2
kh

x+ y+z
4 1
⇒ 3 xyz ≥ ( xy + yz + zx ) −
3 3
4 1
⇒ 3 xyz + x 2 + y 2 + z 2 ≥ ( xy + yz + zx ) − + x 2 + y 2 + z 2
3 3
1
3 ( x + y + z ) − 2 ( xy + yz + zx ) − 1 3 − 2. 3 − 1 4
2
= ≥
3 3 9

312
Cty TNHH MTV DVVH Khang Việt

1
Bất đẳng thức được chứng minh. Đẳng thức xảy ra khi và chỉ khi x= y= z= .
3
Bài 4. Bất đẳng thức đã cho tương đương với
1 1 1 1 1 1
− + − + − ≥0
1 + 2abc 1 + a 2 ( b + c ) 1 + 2abc 1 + b 2 ( c + a ) 1 + 2abc 1 + c 2 ( a + b )
a ( ab + ac − 2bc ) b ( bc + ab − 2ca ) c ( ca + bc − 2ab )
⇔ + + ≥0
1 + a2 (b + c ) 1 + b2 ( c + a ) 1 + c2 ( a + b )
a (1 − bc ) b (1 − ca ) c (1 − ab )
⇔ + + ≥0
1 + a ( 3 − bc ) 1 + b ( 3 − ca ) 1 + c ( 3 − ab )

n
1 − bc 1 − ca 1 − ab
⇔ + + ≥0

.v
bc + abc ( 3 − bc ) ca + abc ( 3 − ca ) ab + abc ( 3 − ab )

om
 1 − bc 1 − ca 1 − ab
 bc + abc ( 3 − bc ) ≥ ca + abc ( 3 − ca ) ≥ ab + abc ( 3 − ab )
Giả sử a ≥ b ≥ c ⇒  .
bc + abc ( 3 − bc ) ≤ ca + abc ( 3 − ca ) ≤ ab + abc ( 3 − ab )
 .c
Sử dụng bất đẳng thức Chebyshev cho hai dãy đơn điệu ngược chiều trên ta có
ok
ngay điều phải chứng minh. Đẳng thức xảy ra khi và chỉ khi a= b= c= 1 .
Bài 5. Bất đẳng thức đã cho tương đương với:
     
bo

16 x 16 y 16 z
 1 + − 3  +  1 + − 3  +  1 + − 3  ≥ 0
 y+z   z+x   x+ y 
et

 2x   2y   2z 
8 − 1 8 − 1 8 − 1
y+z  z+x  x+ y 
⇔  +  +  ≥0
vi

16 x 16 y 16 z
1+ +3 1+ +3 1+ +3
y+z z+x x+ y
ng

Không mất tính tổng quát giả sử


 2x
a

2y 2z
 y + z −1 ≥ z + x −1 ≥ x + y −1
kh


x≥ y≥z⇒ 1 1 1
 ≥ ≥
16 x 16 y 16 z
 1+ +3 1+ +3 1+ +3
 y+z z+x x+ y
Sử dụng bất đẳng thức Chebyshev cho hai dãy đơn điệu cùng chiều trên và đưa
về chứng minh
2x 2y 2z x y z 3
−1+ −1+ −1 ≥ 0 ⇔ + + ≥ .
y+z z+x x+ y y+z z+x x+ y 2

313
Khám phá tư duy Kỹ thuật giải bất ĐT Bài toán Max – Min – Đặng Thành Nam

Bất đẳng thức cuối đúng. Bài toán được chứng minh. Đẳng thức xảy ra khi và
chỉ khi x= y= z .
Bài tập tương tự
Cho x,y,z là các số thực không âm thoả mãn điều kiện xy + yz + zx > 0 .
48 x 48 y 48 z
Chứng minh rằng 1 + + 1+ + 1+ ≥ 15 .
y+z z+x x+ y

CHỦ ĐỀ 9: BẤT ĐẲNG THỨC BERNOULLI VÀ ỨNG DỤNG

n
A. NỘI DUNG PHƯƠNG PHÁP

.v
α
Với mọi số thực a ≥ −1,α ≥ 1 ta luôn có (1 + a ) ≥ 1 + α .a (1).

om
Chứng minh.
+ Với α = 1 bất đẳng thức trở thành đẳng thức.
α
+ Với α > 1 Xét hàm số f (a ) = (1 + a ) − 1 − α .a với a ≥ −1 ta có
.c
α −1
f '(a ) = α ( a + 1) − 1 ; f '(a ) = 0 ⇔ a = 0 .
 
ok
Ta có f’(a) đổi dấu từ âm sang dương khi đi qua a = 0 nên f(a) đạt cực tiểu tại
a=0.
bo

α
Vì vậy f (a ) ≥ f (0) ⇔ (1 + a ) ≥ 1 + α .a .
Bất đẳng thức được chứng minh. Đẳng thức xảy ra khi và chỉ khi a = 0 hoặc
et

α = 1.
Chú ý. Với 0 < α ≤ 1 bất đẳng thức đổi chiều.
vi

Một dạng phát biểu khác của bất đẳng thức Bernoulli hay được sử dụng
ng

a a
x a ≥ xb + 1 − với x ≥ 0, a ≥ b > 0 .
b b
a

Chứng minh.
kh

( ) a
(
x = 1 + xb − 1  b ≥ 1 + xb − 1 =
a
  b
a b
b
) a
x +1− .
b
Áp dụng ta chứng minh được bất đẳng thức AM – GM dạng luỹ thừa
m
a1m + a2m + ... + anm  a1 + a2 + ... + an 
≥  .
n  n 
Chứng minh.
Viết lại bất đẳng thức dưới dạng

314
Cty TNHH MTV DVVH Khang Việt

m m m
 na1   na2   nan 
  +  + ... +   ≥ n.
 a1 + a2 + ... + an   a1 + a2 + ... + an   a1 + a2 + ... + an 
Chú ý
m
 n  n
m  ( n − 1) ak − ∑ ai  ( n − 1) ak − ∑ ai
 nak   1≤i ≠ k  ≥ 1 − m. 1≤i ≠ k
  = 1 + 
, k = 1, n
a +
 1 2 a + ... + an a + a + ... + a a + a + ... + an
 1 2 n
 1 2

 
Lấy tổng tất n bất đẳng thức trên ta có điều phải chứng minh.
Đẳng thức xảy ra khi và chỉ khi a1= a2= ...= an .

n
Xét hai bất đẳng thức AM – GM

.v
a1 + a2 + ... + an n
≥ a1a2 ...an .

om
n
Chứng minh.
a1 + a2 + ... + an
= Đặt An = ; Gn n a1a2 ...an . .c
n
Sử dụng bất đẳng thức Bernoulli ta có
ok
 nAn − ( n − 1) An −1
n n
 An   An   A a
  = 1 + − 1 ≥ n + n  n − 1 = = n
bo

 An −1   An −1   An −1  An −1 An −1 .
⇒ Ann ≥ an Ann−−11
et

Sử dụng liên tiếp bất đẳng thức trên ta được


Ann ≥ an Ann−−11 ≥ an an −1 Ann−−22 ≥ ... ≥ an an −1...a2 . A1 = a1a2 ...an = Gnn ⇒ An ≥ Gn .
vi

Vậy bất đẳng thức được chứng minh.


ng

Ta có một dạng suy rộng của Bernoulli hay được sử dụng như sau
Với n số thực cùng dấu và lớn hơn hoặc bằng -1 ta có
(1 + a1 )(1 + a2 ) ...(1 + an ) ≥ 1 + a1 + a2 + ... + an (1).
a
kh

Chú ý. (1) dễ chứng minh bằng quy nạp toán học.

B. BÀI TẬP MẪU


Bài 1. Cho a, b là các số thực thay đổi thỏa mãn điều kiện 0 < a, b < 1 .
Chứng minh rằng ab + b a > 1 .
Lời giải
Sử dụng bất đẳng thức Bernoulli ta có
b
1  1− a  1 − a a + b − ab a
= 1 +  ≤ 1 + b. = ⇒ ab ≥
a b
 a  a a a + b − ab

315
Khám phá tư duy Kỹ thuật giải bất ĐT Bài toán Max – Min – Đặng Thành Nam

a
1  1− b  1 − b a + b − ab b
= 1 +  ≤ 1 + a. = ⇒ ba ≥ .
b a
 b  b b a + b − ab
a+b a+b
Cộng theo vế hai bất đẳng thức trên ta được ab + b a ≥ > =1.
a + b − ab a + b
Bất đẳng thức được chứng minh.
Chú ý. Bất đẳng thức trên đúng với mọi a,b dương.
Bài 2. Cho a,b là các số thực dương thay đổi thỏa mãn điều kiện a + b =ab .
Chứng minh rằng ab + b a > 6 .
Lời giải

n
1 1 a
Từ giả thiết ta có: + =1 ⇒ a > 1, b > 1 và b = .

.v
a b a −1
a a
 a 

om
Ta cần chứng minh a −1
a +  >6.
 a −1
 a a
a
a a −1 = (1 + a − 1) a −1 ≥ 1 + ( a − 1) . =1+ a
.c
 a −1
Ta có:  .
ok
a a
 a   1  a
 a − 1  = 1 + a − 1  ≥ 1 + a − 1
   
bo

a a
 a  a 1 1
Suy ra: a −1
a +  ≥2+a+ = 4 + ( a − 1) + ≥4+2 ( a − 1) . =6.
 a −1 a −1 a −1 a −1
et

Dấu đẳng thức không xảy ra nên ab + b a > 6 .


Bài 3. Cho hai số thực a,b thoả mãn điều kiện 0 < a, b ≤ 1 .
vi

Chứng minh rằng ab − a + b a −b ≤ 2 .


ng

Lời giải
Sử dụng bất đẳng thức Bernoulli ta có
a

b−a
ab − a = (1 + a − 1) ≤ 1 + ( a − 1)( b − a )
kh

a −b
b a −b = (1 + b − 1) ≤ 1 + ( b − 1)( a − b )
Cộng theo vế hai bất đẳng thức trên ta được
ab − a + b a −b ≤ 2 + ( b − 1)( a − b ) + ( a − 1)( b − a ) = 2 − ( a − b ) ≤ 2 .
2

Bất đẳng thức được chứng minh. Đẳng thức xảy ra khi và chỉ khi a = b .
Bài tập tương tự
Cho a,b,c là các số thực dương có tổng bằng 1. Chứng minh
1
a1− a b1−b c1−c ≤ .
3
316
Cty TNHH MTV DVVH Khang Việt

Bài 4. Chứng minh với mọi số thực a, b, c thuộc đoạn [ 0;1] ta luôn có
3
a 2b + b 2 c + c 2 a ≥ .
4
Lời giải
Sử dụng bất đẳng thức Bernoulli ta có:
y
1  1− x  1 − x x + y − xy x
= 1 +  ≤ 1 + y. = ⇒ xy ≥ .
x y
 x  x x x + y − xy
Áp dụng bất đẳng thức trên suy ra:
2 2 2
 a   b   c 

n
a 2b + b 2 c + c 2 a ≥   +  + 
 a + b − ab   b + c − bc   c + a − ac 

.v
2 2 2
 a   b   c  3
≥  +  +  ≥

om
a+b b+c c+a 4
Bài toán được chứng minh.
n
 1 3 .c
Bài 5. Chứng minh với mọi số nguyên dương n ta có 1 +  ≤ 3 − .
 n n+2
ok
Lời giải
Ta chứng minh bài toán bằng quy nạp kết hợp sử dụng bất đẳng thức Bernoulli.
bo

+ Với n = 1 bất đẳng thức trở thành đẳng thức.


k
 1 3
+ Giả sử bất đẳng thức đúng với n = k tức 1 +  ≤ 3 − .
et

 k k+2
k +1
 1 
vi

3
+ Ta cần chứng minh 1 +  ≤ 3− .
 k +1 ( k + 1) + 2
ng

Chú ý
k +1
 1 
1 +
a

 k + 2  k 2 + 2k 
k
k+2
 k +1 1
= =  2  .
kh

 1
k k + 1  k + 2k + 1  k +1  1 
k
 1 +   1 + 
 k  k 2 + 2k 


k+2
.
1
=
( k + 2) 2

k +1 1+ k ( k + 1)( k + 3)
k + 2k
2

Suy ra
 1 
k +1
 1
k
( k + 2 )2 ≤  3 − 3  ( k + 2 )2 =3 − 3
1 +  ≤ 1 +    .
 k +1  k ( k + 1)( k + 3)  k + 2  ( k + 1)( k + 3) ( k + 1) + 2
317
Khám phá tư duy Kỹ thuật giải bất ĐT Bài toán Max – Min – Đặng Thành Nam

Điều đó chứng tỏ bất đẳng thức đúng với n= k + 1 .


Vậy theo nguyên lý quy nạp ta có điều phải chứng minh.
Đẳng thức xảy ra khi và chỉ khi =
n 1,=n 2.
Bài tập tương tự
n +1 n
 1   1
Chứng minh với mọi số nguyên dương n ta có 1 +  > 1 +  .
 n +1  n
Ta cùng xét các bài toán đã được đề cập.
Bài 6. Cho a,b,c là các số thực không âm thoả mãn điều kiện ab + bc + ca > 0
chứng minh với mọi số nguyên dương n ≥ 2 ta có

n
a b c
+n
n +n ≥2.

.v
b+c c+a a+b
Lời giải

om
+ Nếu tồn tại một số bằng 0 giả sử là khi đó sử dụng bất đẳng thức AM – GM cho
hai số ta có ngay điều phải chứng minh.
+ Xét cả 3 số đều dương sử dụng bất đẳng thức Bernoulli ta có .c
1
a  a n 1 1
ok
=
n =  =
( n − 1)( b + c )  ( n − 1)( b + c ) 
1 1
 ( n − 1)( b + c )  n  ( n − 1)( b + c ) − a  n
  1 +  (1)
bo

 a   a 
1 na
≥ =
1 ( − )( ) + − ( )( a + b + c )

et

n 1 b c a n 1
1+ .
n a
vi

b nb
Tương tự ta có n ≥
( n − 1)( c + a ) ( n − 1)( a + b + c )
ng

c nc
n ≥
( n − 1)( a + b ) ( n − 1)( a + b + c )
a
kh

Gọi P là biểu thức vế trái và cộng theo vế ba bất đẳng thức trên ta được
n n
P≥ n −1 ≥ 2 .
n −1
Bài toán được chứng minh.
Chú ý trong (1) ta dùng phép nghịch đảo trước khi sử dụng Bernoulli bởi với số
mũ nhỏ hơn 1 nếu đánh giá ngay sẽ có chiều bất đẳng thức ngược lại. Vì vậy
cần nghịch đảo để có chiều bất đẳng thức cần chứng minh.

318
Cty TNHH MTV DVVH Khang Việt

Bài 7. Cho a,b,c là các số thực dương. Chứng minh


a b c
3 +3 +3 ≥1.
2b + 25c 2c + 25a 2a + 25b
Lời giải
Áp dụng BĐT Bernoulli ta có
27 a 1 1 81a
3 = ≥ = .
2b + 25c 1 1  2b + 25c  2b + 25c + 54a
 2b + 25c  3 1 +  − 1
  3  27 a 
 27 a 

n
27b 81b 27c 81c
Tương tự ta có 3 ≥ ;3 ≥ .
2c + 25a 2c + 25a + 54b 2a + 25b 2a + 25b + 54c

.v
Cộng theo vế ba bất đẳng thức trên và kết hợp sử dụng bất đẳng thức C – S ta

om
a b c (a + b + c)2
được 3 +3 +3 ≥ 27. ≥1
2b + 25c 2c + 25a 2a + 25b 81(ab + bc + ac)
Bất đẳng thức được chứng minh. Đẳng thức xảy ra khi và chỉ khi a= b= c .
.c
Bài 8. Cho a,b,c là các số thực dương. Chứng minh
ok
7 7 7
 2a   2b   2c 
3
  +3  +3  ≥ 3.
a+b b+c c+a
bo

Lời giải

Sử dụng bất đẳng thức Bernoulli dạng x a ≥ 1 + (


a b
)
x − 1 ta có
et

b
7
7  2a  
7 2
3 =
2a   2a  3
vi

    ≥ 1 +    − 1 ;
a+b a+b 6  b + c  
ng

7
7  2b  
7 2
3 =
2b   2b  3
    ≥ 1 +    − 1; .
b+c b+c 6  b + c 
a


kh

7
7  2c  
7 2
3 =
2c   2c  3
    ≥ 1 +    − 1 .
c+a c+a 6  c + a  
Cộng theo vế ba bất đẳng thức trên và gọi P là biểu thức vế trái ta được
7  2a   2b   2c  
2 2 2
P ≥ 3 +   +  +  − 3 .
6  a + b   b + c   c + a  
Chú ý.

319
Khám phá tư duy Kỹ thuật giải bất ĐT Bài toán Max – Min – Đặng Thành Nam

2 2 2
 a   b   c  1 1 1  b c a
  +  +  = 2+ + , x = ,y= ,z =
a+b b+c c+a (1 + x ) (1 + y ) (1 + z )  a
2 2 b c
1 1 z 1 3
≥ + = + ≥
1 + xy (1 + z ) 2 z + 1 ( z + 1) 2 4
Vì vậy P ≥ 3 . Đẳng thức xảy ra khi và chỉ khi a= b= c .
Chú ý. Câu hỏi đặt ra là tại sao không sử dụng trực tiếp Bernoulli dạng suy rộng
ban đầu bởi như vậy ta có
7
7
3
2a   2a 3 7  2a 
  = 1 + − 1 ≥ 1 +  − 1 .

n
a+b  a+b  3 a +b 

.v
a b c 3
Khi đó đưa về chứng minh + + ≥ .
a+b b+c c+a 2

om
Tuy nhiên bất đẳng thức này không phải đúng với mọi a,b,c dương chẳng hạn
=
a 1,=b 2,=c 3.
Bài tập tương tự .c
1) Cho a,b,c là các số thực không âm thoả mãn điều kiện ab + bc + ca > 0 .
ok
a b c
Chứng minh rằng +3
3 +3 ≥2.
b+c c+a a+b
bo

2) Cho bốn số thực dương a, b, c, d. Chứng minh


7 7 7 7
 2a   2b   2c   2d 
3
  +3  +3  +3  ≥4.
et

a+b b+c c+d  d +a


Bài 9. Cho a,b,c là các số thực dương chứng minh ab + c + bc + a + c a +b ≥ 1 .
vi

Lời giải
ng

Ta cần so sánh các số (b + c),(c + a ),(a + b) với số 1.


+ Nếu tồn tại một trong ba số lớn hơn 1 thì rõ ràng bất đẳng thức luôn đúng.
a

+ Ta chỉ cần xét khi a,b,c cùng thuộc nửa khoảng ( 0;1] , không mất tính tổng quát
kh

ta giả sử a ≥ b ≥ c suy ra min {(a + b),(b + c),(c + a )} =+


b c.
Đến đây ta lại chia trường hợp để xử lý
- Nếu b + c ≥ 1 sử dụng bất đẳng thức Bernoulli ta có
b+c
1 + ( a − 1) 
ab+c = ≥ 1 + ( a − 1)( b + c )
c+a
bc + a =1 + ( b − 1)  ≥ 1 + ( b − 1)( c + a ) .
a +b
c a +b =1 + ( c − 1)  ≥ 1 + ( c − 1)( a + b )
Cộng theo vế ba bất đẳng thức trên đưa về chứng minh

320
Cty TNHH MTV DVVH Khang Việt

1 + ab + bc + ca − a − b − c ≥ 0 ⇔ (1 − a )(1 − b )(1 − c ) + abc ≥ 0 .


Bất đẳng thức đúng.
- Nếu b + c < 1 ta lại chia thành các trường hợp sau đây:
Khả năng 1: b + c ≤ c + a ≤ a + b ≤ 1 sử dụng bất đẳng thức Bernoulli ta có
a a a a
ab+c = ≥ = ≥ .
( ( ))
1 + a − 1
1−( b + c ) 1 + ( a − 1)( 1 − b − c ) a + b + c − a ( b + c ) a + b + c

Tương tự cho 2 bất đẳng thức còn lại rồi cộng lại theo vế ta có điều phải
chứng minh.
Khả năng 2: b + c ≤ c + a ≤ 1 ≤ a + b ta có

n
a b + c ≥ a; b c + a ≥ b ⇒ a b + c + b c + a + c a + b ≥ a + b + c a + b

.v
≥ a + b + 1 + ( c − 1)( a + b=
) c (a + b) + 1 ≥ 1

om
Ta có điều phải chứng minh.
Khả năng 3: b + c ≤ 1 ≤ a + c ≤ a + b ta có
ab+c ≥ a .c
bc + a ≥ 1 + ( b − 1)( c + a )
ok
c a +b ≥ 1 + ( c − 1)( a + b )
Cộng theo vế ba bất đẳng thức trên và đưa về chứng minh
bo

1 + bc + ( ab + bc + bc − a − b − c ) ≥ 0 ⇔ (1 − a )(1 − b )(1 − c ) + abc + bc ≥ 0 .


Bất đẳng thức đúng.
et

Vậy với mọi số thực dương a,b,c ta luôn có ab + c + bc + a + c a +b ≥ 1 .


Bài tập tương tự
vi

Cho a,b,c là các số thực dương chứng minh ( a + b ) + ( b + c ) + ( c + a ) ≥ 2 .


c a b
ng

Bài 10. Cho a,b là các số thực không âm có tổng bằng 2 chứng minh
2
 a −b
a

a +b + 2b
 ≤2.
2a
 2 
kh

Lời giải
Không mất tính tổng quát giả sử a ≥ b ⇒ 0 ≤ a − 1 ≤ 1;0 ≤ b ≤ 1 sử dụng bất đẳng
thức Bernoulli ta có
1 + ( a − 1)  ≤ 1 + ( a − 1) b =
b
ab = −b 2 + b + 1
a −1
b a = b. 1 + ( b − 1)  ≤ b 1 + ( a − 1)( b − 1)  = b 2 ( 2 − b )
Vậy ta chỉ cần chứng minh

321
Khám phá tư duy Kỹ thuật giải bất ĐT Bài toán Max – Min – Đặng Thành Nam

2
( −b )  a −b
2
2
+ b +1 + b 4
(2 − b) 2
+  ≤2
 2 

( )
2
⇔ −b 2 + b + 1 + b 4 ( 2 − b ) + ( b − 1) ≤ 2 ⇔ b3 ( b − 1) ( b − 2 ) ≤ 0
2 2 2

Bất đẳng thức cuối đúng do đó ta có đpcm.


Đẳng thức xảy ra khi và chỉ khi a= b= 1 .
Bài tập tương tự
Cho a,b là các số thực không âm có tổng bằng 2. Chứng minh
4
 a −b
a3b + b3a +   ≤2.

n
 2 

.v
C. BÀI TẬP RÈN LUYỆN

om
Bài 1. Cho n số thực thoả mãn điều kiện 0 < x1 , x2 ,..., xn ≤ 1 . Chứng minh rằng
1 1 1
(1 + x1 ) x2 (1 + x2 ) x3 ...(1 + xn ) x1 ≥ 2n . .c π
Bài 2. Chứng minh rằng với mọi số thực 0 < x < ta có
ok
4

( cos x )
cot x
2
> sin 2 x .
bo

π
Bài 3. Chứng minh với mọi số thực 0 < x < ta có
2
et

( 2 + sin x )2+ tan x > ( 3 + tan x )1+sin x .


vi

Bài 4. Cho a,b là các số thực thuộc khoảng (0;1). Chứng minh
a b
a b
ng

  +  ≥ 2.
b a
a

4
Bài 5. Cho a,b là các số thực dương thoả mãn điều kiện b ≥ , a + b =2. Chứng
3
kh

minh ab b a + 2 ≥ 3ab .
Bài 6. Cho x,y là hai số thực dương. Chứng minh x y + y x > 1 .
Bài 7. Cho a,b,c là các số thực không âm thoả mãn điều kiện ab + bc + ca > 0 .
a b c
Chứng minh rằng 3 +3 +3 ≥2.
b+c c+a a+b

322
Cty TNHH MTV DVVH Khang Việt

D. HƯỚNG DẪN GIẢI – ĐÁP SỐ


1
xk xk
Bài 1. Chú ý (1 + xk ) xk +1 ≥ 1 + ≥2 = , k 1, n=
; xn +1 x1 .
xk +1 xk +1
Nhân theo vế n bất đẳng thức trên ta có điều phải chứng minh.
Đẳng thức xảy ra khi và chỉ khi x1= x2= ...= xn= 1 .
π
Bài 2. Chú ý với 0 < x < ta có cot x > 1 khi đó sử dụng bất đẳng thức
4
Bernoulli ta có

( cos x )
cot x
= (1 − sin x ) (1 + sin x )cot x ≥ (1 − sin x cot x )(1 + sin x cot x )
2 cot x

n
(1 − cos x )(1 + cos x ) =

.v
= 1 − cos 2 x =
sin 2 x
Đẳng thức không xảy ra nên bài toán được chứng minh.

om
Bài 3. Đặt a = 1 + sin x, b = 2 + tan x ⇒ 1 < a < b ta cần chứng minh
b
(1 + a )b > (1 + b )a ⇔ (1 + a ) a .c >1+ b .
Bất đẳng thức cuối đúng theo Bernoulli.
ok
Bài 4. Sử dụng bất đẳng thức Bernoulli ta có:
a a a
b  b−a b−a a 1
  = 1 +  ≤ 1 + a. = b +1− a ⇒   ≥
bo

.
a  a  a b b +1− a
b
b 1
Tương tự ta có:   ≥
et

.
a a +1− b
Cộng theo vế hai bất đẳng thức trên và chú ý
vi

1 1 4
+ ≥ =
ng

2.
b +1− a a +1− b b +1− a + a +1− b
Bất đẳng thức được chứng minh. Đẳng thức xảy ra khi và chỉ khi a = b .
a

Bài 5. Sử dụng bất đẳng thức Bernoulli ta có


1 + b ( a − 1)
kh

1
ab b a =1 + ( a − 1)  .
b
≥ .
 1 
a
1 
1 + a  − 1
 1 + − 1 b 
 b 
1 + b ( a − 1) 1 + b (1 − b )
Vậy ta chứng minh + 2 ≥ 3ab ⇔ b. + 2 ≥ 3b ( 2 − b )
1  2 − b (2 − b)
1 + a  − 1
b 
⇔ ( b − 1)( 3b − 4 ) ≥ 0 đúng.
Bất đẳng thức được chứng minh.

323
Khám phá tư duy Kỹ thuật giải bất ĐT Bài toán Max – Min – Đặng Thành Nam

Bài 6. Ta xét ba trường hợp sau:


+ Nếu x > 1, y > 1 ⇒ x y + y x > x + y > 2 , bất đẳng thức luôn đúng.
+ Nếu x > 1, y < 1 ⇒ x y + y x > 1 + y x > 1 , bất đẳng thức luôn đúng.
+ Nếu x, y ≤ 1, áp dụng bài tập mẫu ta có đpcm.
Bài 7. Nếu một số bằng 0 sử dụng bất đẳng thức AM – GM ta có ngay điều phải
chứng minh.
Ta xét cả ba số đều dương khi đó sử dụng bất đẳng thức Bernoulli ta có

a 1 1 2a 2/3 2a 2/3
= 1 ≥ = ≥

n
3 .
b+c 2
 b + c 3 1  b + c  3 1 ( b + c ) 2/3
+ a 2/3 a 2/3 + b 2/3 + c 2/3

.v
    +
 a  2 a  2

om
b 2b 2/3 c 2c 2/3
Tương tự ta có 3 ≥ 2/3 ; 3 ≥ .
c + a a + b 2/3 + c 2/3 a + b a 2/3 + b 2/3 + c 2/3
Cộng lại theo vế ba bất đẳng thức trên ta có đpcm.
.c
Đẳng thức xảy ra khi và chỉ khi có một số bằng 0 và hai số bằng nhau.
ok
bo
et
vi
a ng
kh

324
Cty TNHH MTV DVVH Khang Việt

Chương 3:
PHƯƠNG PHÁP HÀM SỐ TRONG GIẢI TOÁN
BẤT ĐẲNG THỨC VÀ CỰC TRỊ

CHỦ ĐỀ 1: KỸ THUẬT SỬ DỤNG TÍNH ĐƠN ĐIỆU VỚI BÀI TOÁN


CỰC TRỊ VÀ BẤT ĐẲNG THỨC MỘT BIẾN SỐ
A. TÓM TẮT LÝ THUYẾT
1. Định nghĩa 1. Giả sử K là một khoảng, nửa khoảng, một đoạn.

n
Hàm số y = f ( x) xác định trên D được gọi là:

.v
+ Đồng biến trên K nếu với mọi x1 , x2 ∈ K , x1 < x2 ⇒ f ( x1 ) < f ( x2 ) .

om
+ Nghịch biến trên K nếu với mọi x1 , x2 ∈ K , x1 < x2 ⇒ f ( x1 ) > f ( x2 ) .
2. Điều kiện cần để hàm số đơn điệu
Giả sử hàm số y = f ( x) có đạo hàm trên K .c
+ Nếu hàm số y = f ( x) đồng biến trên K thì f '( x) ≥ 0 với mọi x thuộc K.
ok
+ Nếu hàm số y = f ( x) nghịch biến trên K thì f '( x) ≤ 0 với mọi x thuộc K.
3. Điều kiện đủ để hàm số đơn điệu
Định lý. Giả sử K là một khoảng, nửa khoảng, một đoạn. Hàm số y = f ( x) liên
bo

tục trên K và có đạo hàm tại mọi điểm trong của K(tức các điểm thuộc K nhưng
không phải đầu mút của K). Khi đó
et

+ Nếu f '( x) > 0, ∀x ∈ K thì hàm số y = f ( x) đồng biến trên K.


+ Nếu f '( x) < 0, ∀x ∈ K thì hàm số y = f ( x) nghịch biến trên K.
vi

+ Nếu f '( x) = 0, ∀x ∈ K thì hàm số y = f ( x) không đổi trên K.


ng

Chú ý. Nếu hàm số y = f ( x) liên tục trên đoạn [a;b] và có đạo hàm
f '( x) > 0 ( f '( x) < 0 ) với mọi x thuộc khoảng (a;b) thì hàm số y = f ( x) đồng biến
a

(nghịch biến) trên đoạn [a;b].


kh

4. Định nghĩa 2.
Cho hàm số y = f ( x) xác định trên K.
+ Số M được gọi là giá trị lớn nhất của hàm số y = f ( x) trên K khi và chỉ khi
 f ( x) ≤ M , ∀x ∈ K
 .
∃x0 ∈ K | f ( x0 ) = M
+ Số m được gọi là giá trị nhỏ nhất của hàm số y = f ( x) trên K khi và chỉ khi
 f ( x) ≥ m, ∀x ∈ K
 .
∃x0 ∈ K | f ( x0 ) =m

325
Khám phá tư duy Kỹ thuật giải bất ĐT Bài toán Max – Min – Đặng Thành Nam

B. NỘI DUNG PHƯƠNG PHÁP


1) Kỹ thuật tìm giá trị lớn nhất, giá trị nhỏ nhất của hàm số
Để tìm giá trị lớn nhất, giá trị nhỏ nhất của hàm số y = f ( x) trên D ta tính y’
rồi tìm các điểm tới hạn(các điểm mà tại đó y’ triệt tiêu hoặc không tồn tại) lập
bảng biến thiên. Từ bảng biến thiên suy ra max, min.
Chú ý. Nếu hàm số y = f ( x) luôn tăng hoặc luôn giảm trên đoạn [a;b] thì
ymax =
max { f (a); f (b)} ; ymin min { f (a); f (b)} .
+ Nếu hàm số y = f ( x) liên tục trên đoạn [ a; b ] luôn tồn tại giá trị lớn nhất, giá trị
nhỏ nhất trên đoạn đó và ta tìm max, min đơn giản như sau:

n
Bước 1. Tính đạo hàm y’, tìm các điểm tới hạn của hàm số giả sử là x1 , x2 ,..., xn .

.v
Bước 2. Tính các giá trị f ( x1 ), f ( x2 ),..., f ( xn ) .

om
Bước 3. Kết luận
ymax =
max { f ( x1 ), f ( x2 ),..., f ( xn )}; ymin min { f ( x1 ), f ( x2 ),..., f ( xn )} .
( )
.c 3
Ví dụ 1. Tìm giá trị lớn nhất, giá trị nhỏ nhất của hàm số y =x 6 + 4 1 − x 2 trên

đoạn [ −1;1] .
ok
Lời giải
bo

Xét hàm số y =x 6 + 4 1 − x ( )
2 3
liên tục trên đoạn [ −1;1] .Ta có

( ) 
( )  =0
2 2
et

y' =6 x5 − 24 x 1 − x 2 ; y' =0 ⇔ 6 x  x4 − 4 1 − x2

vi

 x = 0
x = 0 x = 0 
( )
ng

 2
⇔ x =
2
⇔ 4 ⇔ x = 2 1− x 2
±
( ) 
2
 x= 4 1 − x 2 3
  2
 x = −2 1 − x 2( 
)
 x = ±
a

2
kh

 2 4
Suy ra y (0)= 4; y  ± =
 ; y (±1)= 1 .
 3 9
 2 4
Vì vậy ymin =y  ±  = ; ymax =y (0) =4.
 3 9
Ví dụ 2. Tìm giá trị lớn nhất và nhỏ nhất của hàm số

1 + x 2015 + 1 + (1 − x ) với x ∈ [ 0;1] .


2015
y=

326
Cty TNHH MTV DVVH Khang Việt

Lời giải
2015 (1 − x )
2014
2015 x 2014
=
Ta có y ' − ;
2 1 + x 2015 2 1 + (1 − x )
2015

Suy ra

y ' = 0 ⇔ x 2014 1 + (1 − x ) =(1 − x )


2015 2014
1 + x 2015

1 + (1 − x )
2015
1 + x 2015
⇔ = (1)
x 2014 (1 − x )2014

n
1 + t 2015
Xét hàm số f (t ) = trên đoạn [0;1] ta có

.v
t 2014
2013t 2015 + 4028

om
f '(t ) =
− <0.
2t 2015 1 + t 2015
Vì vậy f(t) là hàm nghịch biến trên đoạn [0;1], do đó .c
1
(1) ⇔ f ( x) = f (1 − x) ⇔ x =1 − x ⇔ x = .
ok
2
2015
1 1
Tính các giá trị: y (0) =
y (1) =
1 + 2; y   =
2 1+   .
bo

2 2
2015
1
Vậy giá trị nhỏ nhất của hàm số bằng 2 1 +   ; giá trị lớn nhất bằng 1 + 2 .
et

2
Bài tập tương tự
vi

Cho a,b là hai số thực không âm có tổng bằng 1. Tìm giá trị lớn nhất, giá trị nhỏ
ng

nhất của biểu thức P = 1 + a 2014 + 1 + b 2014 .


Ví dụ 3. Tìm giá trị lớn nhất và giá trị nhỏ nhất của hàm số
a

y= 5 | cos x + sin x | + | 7 cos x + sin x | .


kh

Lời giải
Ta đưa về chung một biểu thức bằng cách sử dụng đẳng thức:
a + b= a 2 + b 2 + 2 ab .
Khi đó

327
Khám phá tư duy Kỹ thuật giải bất ĐT Bài toán Max – Min – Đặng Thành Nam

25 ( sin x + cos x ) + ( 7 cos x + sin x ) + 10 ( sin x + cos x )( 7 cos x + sin x )


2 2
=
y

=50 + 32sin 2 x + 24cos 2 x + 10 sin 2 x + 7 cos 2 x + 4sin 2 x

= 50 + 32sin 2 x + 24cos 2 x + 10 4 + 3cos 2 x + 4sin 2 x


= 8 ( 3cos 2 x + 4sin 2 x ) + 10 4 + 3cos 2 x + 4sin 2 x + 50

= 40sin ( 2 x + α ) + 40 + 50sin ( 2 x + α ) + 50
4 3
cos α
Với= =,sin α .
5 5

n
16
Giờ chỉ cần khảo sát hàm số y =4sin t + 4 + 5sin t ⇒ ymax =
13; ymin =−

.v
.
5
Từ đó suy ra giá trị lớn nhất và nhỏ nhất của hàm số đã cho là

om
=
ymax 6=
5; ymin 3 2 .

2) Kỹ thuật đổi biến


.c
Sử dụng khi hàm một biến có chứa căn phức tạp; dạng lượng giác; mũ hoặc đa
ok
thức có chứa bậc cao.
x − x ( x − 1) + 2
Ví dụ 1. Tìm giá trị lớn nhất và nhỏ nhất của hàm số y =
bo

.
x − x −1 +1
Lời giải
Tập xác định: D= [1; +∞ ) .
et

1
≤ 1, ∀x ≥ 1 ⇒ t ∈ ( 0;1] .
vi

Đặt t= x − x − 1=
x + x −1
ng

t2 +1
Chú ý. t 2 = 2 x − 2 x ( x − 1) − 1 ⇒ x − x ( x − 1) = .
2
a

t2 + 5
Khi đó y = .
kh

2t + 2
t 2 + 2t − 5 3
=
Ta có y ' < 0, ∀t ∈ ( 0;1] . Do đó ymin
= y=(1) . Không tồn tại giá
2 ( t + 1)
2 2
trị lớn nhất.
Ví dụ 2. Chứng minh rằng với mọi số tự nhiên n > 1 ta có
n
n n nn
n 1+ + 1− < 2.
n n

328
Cty TNHH MTV DVVH Khang Việt

Lời giải
n
n
Đặt x= ∈ ( 0;1) , ∀n ∈ N * . BĐT cần chứng minh trở thành
n
n 1 + x + n 1 − x < 2, ∀x ∈ ( 0;1) .

Xét hàm số f ( x) = n 1 + x + n 1 − x liên tục x ∈ [ 0;1) có


 
1 1 1  < 0, ∀x ∈ 0;1
=f '( x) −
n  n (1 + x)n−1 n (1 − x)n−1 
( )
 
Vậy f(x) nghịch biến [0; 1) nên f(x) < f(0) = 2, ∀x ∈ ( 0;1) (đpcm).

n
.v
Ví dụ 3. Cho hàm số f xác định trên tập số thực, lấy giá trị trên R và thỏa mãn điều
kiện f ( cot=
x ) sin 2 x + cos 2 x, ∀x ∈ ( 0; π ) .

om
Hãy tìm giá trị lớn nhất và giá trị nhỏ nhất của hàm số
( ) (
g ( x ) = f sin x . f cos x trên R.
2 2
) .c
Lời giải
Ta có f ( cot=
x ) sin 2 x + cos 2 x, ∀x ∈ ( 0; π )
ok
cot 2 x − 1 cot 2 x + 2cot x − 1
2cot x
( cot x )
⇔ f= = + , ∀x ∈ ( 0; π )
bo

cot 2 x + 1 cot 2 x + 1 cot 2 x + 1


Từ đó với chú ý rằng với mỗi t ∈ R đều tồn tại x ∈ ( 0; π ) sao cho cot x = t ta
et

t 2 + 2t − 1
được
= f (t ) , ∀t ∈ R .
t2 +1
vi

Suy ra g ( x ) f =
= (
sin 2 x . f cos 2 x ) ( ) sin 4 2 x + 32sin 2 2 x − 32
, ∀x ∈ R (1) .
ng

sin 4 2 x − 8sin 2 2 x + 32
1  1
Đặt u = sin 2 2 x ta có u thuộc đoạn 0;  .
a

4  4
kh

Vì vậy từ (1) ta được


min g ( x) = min h(u ) và m ax g ( x) = m ax h(u )
x∈R  1 x∈R  1
0; 4  0; 4 

u 2 + 8u − 2
trong đó h ( u ) = .
u 2 − 2u + 2

Ta có h′ ( u ) =
(
2 −5u 2 + 4u + 6 ).
(u )
2
2
− 2u + 2

329
Khám phá tư duy Kỹ thuật giải bất ĐT Bài toán Max – Min – Đặng Thành Nam

 1
Dễ dàng chứng minh được h′ ( u ) > 0, ∀u ∈ 0;  .
 4
 1
Do đó hàm h(u) đồng biến trên 0;  .
 4
 1 1 1
Vì vậy trên 0;  ta có min h ( u ) = h ( 0 ) = −1 và max =
h ( u ) h=
  .
 4  4  25
1
Vậy min g ( x ) = −1 , đạt được chẳng hạn khi x = 0 và max g ( x ) = , đạt được
25
π
chẳng hạn khi x =

n
.
4

.v
3) Kỹ thuật khảo sát trực tiếp

om
Trong một số bài toán có thể phải đạo hàm nhiều lần liên tiếp thậm chí phải
khảo sát thêm hàm số phụ. Ta thường sử dụng
 f(x) đồng biến trên [a; b] thì f(x) > f(a) với mọi x > a.
.c
 f(x) nghịch biến trên [a; b] thì f(x) > f(b) với mọi x < b.
Chú ý. Lập bảng biến thiên nếu cần thiết hoặc sử dụng tính chất về điểm cực đại,
ok
cực tiểu.
x3
Ví dụ 1. Chứng minh rằng với mọi x > 0 ta có x − < sinx < x .
bo

6
Lời giải
et

3
x
Xét hàm số f ( x) =− x + + sinx trên nửa khoảng [ 0;+∞ ) . Ta có
6
vi

x2
f ′( x)= − 1 + cos x
ng

2
f ′′( x)= x − sinx
a

f ′′′( x) = 1 − cos x
Ta có f ′′′( x) = 1 − cos x ≥ 0, ∀x ∈ [ 0; +∞ ) ⇒ f ′′( x) ≥ f ′′(0) = 0 , nên f’(x) đồng
kh

biến trên [ 0;+∞ ) . Suy ra f ′( x) ≥ f ′(0) =


0 ⇒ f ( x) đồng biến trên [ 0;+∞ ) .
Do đó f ( x) ≥ f (0) = 0, ∀x ≥ 0 và f ( x) > f (0) = 0, ∀x > 0.
x3 x3
Tức là − x + sin x > 0 ⇔ x − < sin x với x > 0 (1)
6 6
Lưu ý f ′′( x) > f ′′(0) =
0 với x > 0 ta có sinx < x (2)
Từ (1) và (2) ta có đpcm.
Ví dụ 2. Chứng minh với mọi x dương và n là số nguyên dương ta có

330
Cty TNHH MTV DVVH Khang Việt

x 2 x3 xn
ex > 1 + x + + ... + .
2! 3! n!
Lời giải
 x 2 x3 xn 
Xét hàm số f n ( x) = e x − 1 + x + + ... +  trên khoảng ( 0;+∞ ) ta có
 2! 3! n ! 

 x 2 x3 x n −1 
f 'n ( x) = e x − 1 + x + + + ... +  .
 2! 3! ( n − 1) !
 
Từ đó suy ra f ( n ) n ( x) = e x − 1 > 0, ∀x > 0 ⇒ f ( n −1) n ( x) > f ( n −1) n (0) = 0 .

n
Sử dụng liên tiếp ta có f 'n ( x) > f 'n (0) = 0 (đpcm).
0 ⇒ f n ( x) > f n (0) =

.v
Chú ý. Số thực dương a để bất đẳng thức sau luôn đúng với mọi số thực x là e.
x2 xn

om
ax ≥ 1+ x +
+ ... + , ∀x ∈ ; n ∈ * .
2 n!
Thật vậy, Bất đẳng thức đã cho tương đương với:

x ln a ≥ ln 1 + x +

x2
+ ... + 
xn 
.c
 2 n ! 
ok
  x2 xn 
 ln 1 + x + + ... + 
bo

  2 n! 
ln a ≥ ,x > 0.
x
⇔
  x2 xn 
et

 ln  1 + x + + ... + 
2 n! 
ln a ≤  ,x < 0
vi

 x
ng

Chú ý
 x2 xn   x2 xn 
ln 1 + x + + ... +  ln 1 + x + + ... + 
a

 2 n!   2 n! 
=
lim =
lim 1.
x →0+ x →0−
kh

x x
ln a ≥ 1
Do đó  ⇔a= e.
ln a ≤ 1
Ngược lại với a là e thì bất đẳng thức đúng.
Ví dụ 3. Cho n là số lẻ lớn hơn 3. Chứng minh rằng với mọi x ≠ 0 ta có
 x2 x n  x 2 x3 xn 

1 + x + + ... + 

1 − x + − + ... + 
< 1.
 2! n !  2! 3! n ! 

331
Khám phá tư duy Kỹ thuật giải bất ĐT Bài toán Max – Min – Đặng Thành Nam

Lời giải
 x 2
x n
u ( x) =1 + x + + ... +
 2! n!
Đặt  .
2 3
 x x xn
v( x) =1 − x + 2! − 3! + ... + n !
=
Ta cần chứng minh f ( x) u ( x).v( x) < 1.
Ta có

u '( x) =1 + x +
x2 + ... + xn −1 = u ( x) − xn
 2! (n − 1)! n!

n

 x2 + x3 + ... + xn −1 = xn

.v
 v '( x ) =−1 + x − − v ( x ) −
 2! 3! (n − 1)! n!

om
Vì vậy
).v( x) ′ u′( x)v( x) + u ( x).v′( x)
f ′( x) u ( x=
=

=

u ( x ) −
xn   xn 
 .v( x) + u ( x)  −v( x) −  =
.cxn
− u ( x) + v( x) 
ok
 n!   n!  n!
2 xn  x2 x4 x n−1 
=
− 1 + + + ... + 
bo

n!  2! 4! (n − 1)!
Vì n là số lẻ lớn hơn 3 nên f ′( x) đổi dấu từ dương sang âm khi đi qua x = 0 .
et

Do đó tại x = 0 thì hàm số đạt cực đại vì vậy f ( x) < f (0) = 1, ∀x ≠ 0. (đpcm).
b+ x b
a+x a
vi

Ví dụ 4. Chứng minh với mọi a, b, x > 0 và a ≠ b ta có   >  .


b+ x b
ng

Lời giải
b+ x
a+x a+x
Xét= x ≥ 0. Khi đó ln f ( x= ( b + x ) ln 
a

hàm số f ( x)   , ) .
b+ x b+ x
kh

a + x ′
]′ (b + x) ln
Suy ra [ ln f ( x)=
 b + x 
f ′( x) a+x b + x  a + x ′ a+ x b−a
⇔ = ln + (b + x) . =  ln +
f ( x) b+x a+x b+x b+x a+x
 a+ x b−a
f ′( x) f ( x)  ln
⇒= += 
 b+ x a+x
b+ x b+ x
a+x  a+ x b−a a+ x
=    ln =+    .g ( x )
b+ x  b+ x a+ x b+ x
332
Cty TNHH MTV DVVH Khang Việt

a+ x b−a
trong đó=
g ( x) ln + . Ta có
b+ x a+ x
b+ x b−a a −b ( a − b) 2
g ′( x) = . + = − < 0.
a + x (b + x) 2 (a + x) 2 (a + x) 2 (b + x)
Do đó g(x) nghịch biến trên ( 0;+∞ ) . Suy ra
 a+ x b−a
g ( x) > lim =g ( x) lim  ln + =  0.
x →∞ x →∞  b+ x a+x
Vậy f ′( x) > 0, ∀x > 0 , nên f(x) đồng biến trên ( 0;+∞ ) .
Suy ra f(x) > f(0) (đpcm).

n
.v
Ví dụ 5. Cho a, b ∈ [ 0;1] . Chứng minh rằng

om
x b a
+ + + (1 − x )(1 − a )(1 − b ) ≤ 1, ∀x ∈ [ 0;1] .
a + b +1 x + a +1 x + b +1
Lời giải .c
x b a
Xét hàm số = f ( x) + + + (1 − x )(1 − a )(1 − b ) liên tục
a + b +1 x + a +1 x + b +1
ok
trên đoạn [0;1] ta có
1 b a
′( x) − (1 − a )(1 − b )
bo

f= − −
a + b + 1 ( x + a + 1) 2
( x + b + 1)2
2b 2a
f ′′ ( x ) ≥ 0, ∀∈ [ 0;1]
et

= +
( x + a + 1)3
( x + b + 1)3
vi

Nên hàm số f’(x) đồng biến trên [0; 1], suy ra phương trình f’(x) = 0 có nhiều
nhất một nghiệm trên (0; 1).
ng

• Nếu phương trình f’(x) = 0 vô nghiệm thì f(x) đơn điệu trên [0; 1], thì
max { f ( 0 ) ; f (1)}.
f ( x ) ≤ max f ( x ) =
a

[0;1]
kh

Ta có
b a a 2b 2 + a + b + 1 ab + a + b + 1
f ( 0=
) + + (1 − a )(1 − b=
) ≤ = 1;
a +1 b +1 ( a + 1)( b + 1) ( a + 1)( b + 1)
1 a b 1 a b
f (1=
) + + ≤ + + = 1.
a + b +1 b + 2 a + 2 a + b +1 b + a +1 a + b +1
Suy ra f ( x ) ≤ max= f ( x ) max { f ( 0 ) ; f (1)} ≤ 1 .
[0;1]
• Nếu phương trình f’(x) = 0 có nghiệm duy nhất x = x0 . Khi đó do f’(x) đồng
biến trên [0; 1], nên
333
Khám phá tư duy Kỹ thuật giải bất ĐT Bài toán Max – Min – Đặng Thành Nam

f ′ ( x ) < 0, ∀x ∈ [ 0; x0 ] và f ′ ( x ) > 0, ∀x ∈ [ x0 ;1] .


Do đó x0 là điểm cực tiểu của hàm số, mà f(x) liên tục trên [0; 1] nên
f ( x ) ≤ max= f ( x ) max { f ( 0 ) ; f (1)} ≤ 1 .
[0;1]
Từ hai trường hợp ta có điều phải chứng minh.
Ví dụ 6. Cho tam giác ABC có A > B > C . Tìm giá trị nhỏ nhất của biểu thức
x − sin A x − sin B
=P + .
x − sin C x − sin C
Lời giải

n
Điều kiện xác định: D = ( −∞;sin C ]  [sin A; +∞ ) .

.v
x − sin A x − sin B
=
Xét hàm số f ( x) + liên tục trên D ta có
x − sin C x − sin C

om
1 x − sin C sin A − sin C 1 x − sin C sin B − sin C
f '( x) . + . > 0, ∀xD .
2 x − sin A ( x − sin C )2 2 x − sin B ( x − sin C )2
.c sin A − sin B
Do đó f(x) đồng biến trên D và có =
lim f ( x) 2;=
f (sin A) <1.
ok
x →−∞ sin A − sin C
sin A − sin B
Vậy giá trị nhỏ nhất của P bằng đạt tại x = sin A .
bo

sin A − sin C
α +n β +n
 1  1
Ví dụ 7. (VMC 2011) Cho α , β ∈  thỏa mãn 1 +  < e < 1 +  với mọi
et

 n  n
n nguyên dương. Tìm min α − β .
vi

Lời giải
ng

1
Lấy logarit tự nhiên suy ra α < − n < β , ∀n ∈ * .
 1
ln 1 + 
 n
a

1
kh

Xét hàm số = f ( x) − x, x ≥ 1 . Khi đó


 1
ln 1 + 
 x
min α=
− β max f ( x) − min f ( x) .
x ≥1 x ≥1
1
Mặt khác =
f '( x) − 1 < 0, ∀x ≥ 1 .
x ( x + 1)

334
Cty TNHH MTV DVVH Khang Việt

 
1  1 
Do đó max f ( x) − min f ( x= ) f (1) − lim f ( x=) − 1 − lim  − x .
x ≥1 x ≥1 x →+∞ ln 2 x →+∞   1 
 ln 1 + x  

 1  1 1
1 − x ln 1 +  − ln 1 +  +
1  x 1  x  x +1
− 1 − lim = − 1 − lim .
ln 2 x →+∞  1 ln 2 x →+∞

1
ln 1 + 
 x x ( x + 1)
1 1

x ( x + 1) ( x + 1)2 x ( x + 1) − x 2

n
1 1 1 3
= − 1 − lim == − 1 − lim = − .
2x + 1

.v
ln 2 x →+∞ ln 2 x →+∞ 2x + 1 ln 2 2
x ( x + 1)
2 2

om
1 3
Vậy min α − β = − .
ln 2 2
Ví dụ 8. [ MR No.4 2009] Cho các số thực .. thỏa mãn điều kiện
.c
a1x + a2x + ... + anx ≥ b1x + b2x + ... + b nx với mọi x ∈  .
ok
Chứng minh rằng f :  → ( 0, +∞ )
x x x
a  a  a 
bo

f (=
x)  1  +  2  + ... +  n  tăng trên ( 0, +∞ ) .
 b1   b2   bn 
et

Lời giải
x x x
a  a  a  a  a  a 
Ta=
có f '( x)  1  ln  1  +  2  ln  2  + ... +  n  ln  n  ,
vi

 b1   b1   b2   b2   bn   bn 
ng

x 2 x 2 x 2
a    a1    a2 
  a2    an    an  
=f ''( x)  1  ln    +   ln    + ... +   ln    ≥ 0 .
 b1    b1    b2 
  b2    bn    bn  
a

Suy ra f '( x) là hàm tăng trên  . Ta chứng minh f '(0) = 0 , thật vậy.
kh

Theo giả thiết ta có


  a x    a x    a x 
b1x   1  − 1 + b2x   2  − 1 + ... + bnx   n  − 1 ≥ 0, ∀x ∈  .
 b   b   b 
 1    2    n  
x x x
 a1   a2   an 
  −1   −1   −1
x  b1  x  b2  x  bn 
Với x > 0 ta có b1 . + b2 . + ... + bn . ≥ 0, ∀x > 0 .
x x x
Cho x → 0+ ta được

335
Khám phá tư duy Kỹ thuật giải bất ĐT Bài toán Max – Min – Đặng Thành Nam

a1 a a
ln + ln 2 + ... + ln n ≥ 0 (0.1)
b1 b2 bn
Với x < 0 ta có
x x x
 a1   a2   an 
  −1   −1   −1
x  b1  x  b2  x  bn 
b1 . + b2 . + ... + bn . ≤ 0, ∀x < 0 .
x x x
Cho x → 0− ta được
a a a
ln 1 + ln 2 + ... + ln n ≤ 0 (0.2)
b1 b2 bn

n
a1 a a

.v
= ln
Từ (0.1) và (0.2) suy ra f '(0) + ln 2 + ... + ln n= 0
b1 b2 bn

om
0 ⇒ điều phải chứng minh.
⇒ f '( x) ≥ f '(0) =
Ví dụ 9. Cho các số thực a1 , a2 ,..., an > 0 và hàm số f : ( 0, +∞ ) → ( 0, +∞ ) xác định

 a x + a2x + ... + anx 


bởi f ( x) =  1  .
1/ x .c
 n
 
ok
Chứng minh rằng f là hàm tăng và tính giới hạn của f ( x) khi x → +∞ .
bo

Lời giải
Xét hàm số g : ( 0, +∞ ) → ( 0, +∞ ) xác định bởi g ( x) = xt là hàm lồi.
et

y
Xét =
t > 1 tức x < y, khi đó ta có
x
vi

y
 a1x + a2x + ... + anx   a1x + a2x + ... + anx x 1
(( ) ( ) ( ))
ng

g =    ≤ g a1 + g a2 + ... + g an
x x x
 n n n
   
1 y
( )
a

=
a + a2y + ... + any
n 1
kh

Bất đẳng thức trên tương đương với


1/ x 1/ y
 a y + a2y + ... + any 
 a x + a2x + ... + anx 
 1 ≤ 1   ⇒ f ( x) ≤ f ( y ) .
n  n 
   
Vậy f là hàm tăng. Ta có điều phải chứng minh.
 π
Ví dụ 10. (IMC 2006) So sánh sin ( tan x ) và tan ( sin x ) với x ∈  0;  .
 2

Lời giải
336
Cty TNHH MTV DVVH Khang Việt

 π
số f ( x) tan ( sin x ) − sin ( tan x ) trên  0;  .
Xét hàm=
 2
cos x cos ( tan x ) cos3 x − cos ( tan x ) cos 2 ( sin x )
Ta có f '( x) = − = .
cos 2 ( sin x ) cos 2 x cos 2 x cos 2 ( sin x )
π
Nếu 0 < x < arctan ⇒ cos ( tan x ) > 0 sử dụng bất đẳng thức cô si ta được
2
3 3
 cos(tan x) + 2cos(sin x)    tan x + 2sin x  
cos ( tan x ) cos 2 ( sin x ) ≤   ≤ cos    < cos x
3
 3    3 

n
 π
do cos x là hàm lõm và tan x + 2sin x > 3 x, ∀x ∈  0;  .

.v
 2
 π

om
Do đó f '( x) > 0 hay f ( x) là hàm tăng trên  0;arctan  .
 2
0 ⇒ tan ( sin x ) > sin ( tan x ) .
Vì vậy f ( x) > f (0) =

Nếu x = arc tan


π
thì
.c
2
ok
  π  π /2 π   π 
tan  sin =
arc tan   tan > tan > 1 > sin  tan  arc tan   .
  2  1+ π 2 / 4 4   2 
bo

 π π
Nếu x ∈  arc tan ;  ⇒ tan ( sin x ) > 1 > sin x ( tan x ) .
 2 2
et

 π
Vậy với mọi x ∈  0;  thì tan ( sin x ) > sin ( tan x ) .
vi

 2
4) Vận dụng kết hợp các bất đẳng thức khác như AM – GM; C –S
ng

Trước khi khảo sát tính đơn điệu của hàm số ta sử dụng các đánh giá cơ bản
thông qua các bất đẳng thức cơ bản như AM – GM; C – S sẽ đưa về chứng minh
a

một bất đẳng thức cơ bản hơn thuận lợi cho việc khảo sát hàm số.
kh

π
Ví dụ 1. Chứng minh rằng nếu 0 ≤ x < ta có 2s inx + 2 t anx ≥ 2 x+1 .
2
Lời giải
Sử dụng bất đẳng thức AM –GM ta có 2s inx + 2 t anx ≥ 2. 2s inx.2 t anx .

Ta chứng minh 2 2s inx.2 t anx ≥ 2 x +1 ⇔ 2s inx + t anx ≥ 22 x ⇔ sinx + t anx ≥ 2 x .


 π
Xét hàm số f ( x) = sinx + tan x − 2 x liên tục trên 0;  , ta có
 2

337
Khám phá tư duy Kỹ thuật giải bất ĐT Bài toán Max – Min – Đặng Thành Nam

1 1  π
f ′( x) =cos x + − 2 > cos 2 x + − 2 ≥ 0, ∀x ∈ 0;  .
2
cos x 2
cos x  2
 π
(vì với x ∈ 0;  thì cos x > cos 2 x và theo BĐT AM-GM ta có
 2
1
cos 2 x + ≥ 2 ).
cos 2 x
 π
Do đó f(x) đồng biến trên 0;  .
 2
 π
0 , hay sinx + t anx ≥ 2x với mọi x ∈ 0; 
Suy ra f ( x) ≥ f (0) = (đpcm).

n
 2

.v
Bài tập tương tự
3x+2

om
 π
Chứng minh rằng 4 sin x
+2 tan x
≥ 2 2 , ∀x ∈ 0;  .
 2

 sinx 
3
 π
.c
Ví dụ 2. Chứng minh rằng   > cos x, ∀x ∈  0;  .
ok
 x   2
Lời giải
Bất đẳng thức tương đương với:
bo

3 3
 sinx  sin x
 > cos x ⇔ 3 > cos x ⇔ sin x.tan x- x > 0 (1) .
2 3

et

 x  x
 π
Xét hàm số f ( x) = sin 2 x.tan x- x3 với x ∈ 0;  .
vi

 2
ng

Ta có f ′ ( x ) = 2sin 2 x + tan 2 x − 3 x 2 .

Áp dụng BĐT 3(a 2 + b 2 + c 2 ) ≥ ( a + b + c ) ta có


2
a

1
f ′ ( x )= sin 2 x + sin 2 x + tan 2 x − 3 x 2 ≥ ( 2sin x + tan x )2 − 3x 2 .
kh

3
 π
Đặt g ( x) = 2sin x + tan x − 3 x, x ∈ 0;  , thì
 2
1 1 1
g ′ ( x=
) 2cos x + 2
−=
3 cos x + cos x + 2
− 3 ≥ 33 cos 2 x. −=
3 0
cos x cos x cos 2 x

 π  π
với mọi x ∈ 0;  , nên g(x) đồng biến trên 0;  .
 2  2

338
Cty TNHH MTV DVVH Khang Việt

 π
Suy ra g ( x ) ≥ g ( 0 ) = 0, ∀x ∈ 0;  .
 2
1  π
Do đó f ′ ( x ) ≥ ( 3 x )2 − 3 x 2 =
0 nên f(x) đồng biến trên 0; 2  . Suy ra
3  
π
) 0, ∀x ∈ 0;  .
f ( x ) > f ( 0=
 2
Nhận xét: Khi trong BĐT có chứa các loại hàm số khác nhau ta thường cô lập
mỗi loại hàm số để dễ xét dấu của đạo hàm, hoặc ta có thể đạo hàm liên tiếp để
khử bớt một loại hàm số.

n
x3

.v
Cách 2: Ta có x − < sin x, suy ra
6

om
3
 sin x   x 
3 2
x 2 x 4 x6 x2 x4
  >  1 −  =−
1 + − > 1 − + ( 2)
 x   6  2 12 216 2 24

Ta chứng minh được cos x < 1 −


x2 x4
+
.c
( 2)
2 24
ok
Từ (1) và (2) ta có đpcm.
5) Kỹ thuật vận dụng định lý Lagrange
bo

Định lý. Nếu hàm số y = f ( x) là hàm liên tục trên đoạn [a; b] và có đạo hàm trên
f (b) − f (a )
khoảng (a; b) thì tồn tại c ∈ (a; b) sao cho f '(c) =
et

.
b−a
vi

1 1 x +1
Ví dụ 1. Chứng minh (1 + ) x < (1 + ) , ∀x ∈ (0; +∞) .
x x +1
ng

Lời giải
1 1 x +1
Ta có: (1 + ) x < (1 + ) ⇔ x[ln( x + 1) - ln x] < ( x + 1)[ln( x + 2) - ln( x + 1)]
a

x x +1
kh

Xét hàm số = f ( x) x[ln( x + 1) - ln x] ta có


x 1
f '( x)= ln( x + 1) − ln x + − 1= ln( x + 1) − ln x − .
x +1 x +1
Áp dụng định lí Lagrange đối với hàm số: y = lnt trên [x; x+1], thì tồn tại
1
c ∈ (x; x+1) sao cho: f '(c)= ln( x + 1) − ln x ⇒ = ln( x + 1) − ln x.
c
1 1 1
Mặt khác 0 < x < c < x + 1 ⇒ > >
x c x +1

339
Khám phá tư duy Kỹ thuật giải bất ĐT Bài toán Max – Min – Đặng Thành Nam

1 1 1
⇒ > ln( x + 1) − ln x > ⇒ ln( x + 1) − ln x − >0
x x +1 x +1
⇒ f '( x) > 0, ∀x ∈ (0;+∞) ⇒ hàm số f ( x) đồng biến trên (0;+∞).
Từ (1) suy ra: f '( x) > 0, ∀x ∈ (0; +∞) ⇒ f ( x) đồng biến trên (0; +∞).
Suy ra: f ( x + 1) > f ( x), ∀x ∈ (0; +∞) ⇒ điều phải chứng minh.
t
 1
Chú ý. Ta có thể chứng minh trực tiếp hàm số y= 1 +  đồng biến trên khoảng
 t
( 0;+∞ ) .
x +1

n
x
 1  1
+ Với mọi x dương ta có 1 +  < e < 1 +  .

.v
 x  x
Ví dụ 2. Cho số nguyên dương n. Chứng minh rằng

om
1
xn . 1 − x < với mọi x∈ ( 0;1) .
2n.e
Lời giải .c
Bất đẳng thức tương đương với
ok
1 1
xn . 1 − x < ⇔ x 2n (1 − x) < (*)
2n.e 2ne
bo

( x) x 2n (1 − x) với x ∈ ( 0;1) , ta có
Xét hàm số f=

f ′=
( x) x 2n−1  2n − (2n + 1) x  .
et

2n
Suy ra f’(x) đổi dấu từ dương sang âm khi đi qua x = .
vi

2n + 1
 2n  (2n) 2 n
ng

Vì vậy =
max f ( x) f=
  .
(0;1)  2n + 1  (2n + 1) 2 n +1
2 n +1
a

(2n) 2 n 1  2n  1
Ta chứng minh 2 n +1
< ⇔   <
 2n + 1 
kh

(2n + 1) 2ne e
2 n +1
 2n + 1 
⇔  > e ⇔ (2n + 1) [ ln(2n + 1) − ln(2n) ] > 1 .
 2n 
1
⇔ ln(2n + 1) − ln(2n) > (2) .
2n + 1
Áp dụng định lý Lagrange cho hàm số f ( x) = ln x trên [2n; 2n+1] suy ra tồn tại
f (2n + 1) − f (2n)
c ∈ [ 2n;2n + 1] thuộc sao cho f ′( x) = .
2n + 1 − 2n

340
Cty TNHH MTV DVVH Khang Việt

1 1
Suy ra ln(2n + 1) − ln(2n) = > (3) .
e 2n + 1
Từ (1), (2) và (3) suy ra đpcm.
C. BÀI TẬP RÈN LUYỆN
Bài 1. Chứng minh các bất đẳng thức sau
x3 x3 x5
a) x − < sin x < x − + với mọi x > 0 .
6 6 120
x2 x2 x4
b) 1 − < cos x < 1 − + với mọi x là số thực dương.
2 2 24
 π

n
1 1 4
c) < +1− với mọi x ∈  0;  .
2 2
π2
 2

.v
sin x x
π +2  π
d) 1 < x.cos 2 x < với mọi x ∈ 0;  .

om
4  4
2
e− x x4
e) 1 − x ≤ ≤1− x + với mọi x thuộc đoạn [0;1].
2 (1 + x )
x +1
.c
ok
Bài 2. Chứng minh rằng
 π 4 4
bo

a) Với mọi x ∈ 0;  ta có sin x ≤ x − 2 x 2 .


 2  π π
2  π
b) 3 x − x3 < với x ∈  0;  .
et

sin 2 x  2
vi

sin 3 x cos3 x  π
Bài 3. Tìm giá trị nhỏ nhất của hàm số =
f ( x) + trên khoảng  0;  .
cos x sin x  4
ng

n
1
Bài 4. Cho số nguyên dương n. Chứng minh rằng ∑ k (2k − 1) < 2ln 2 .
a

k =1
n
kh

1
Bài 5. Chứng minh rằng: 1 + ln n > ∑ > ln(n + 1), ∀n ∈ N * .
i =1 i

Bài 6. Tìm GTNN của hàm số f ( x=


) x2 + x + 1 + x2 − x + 1 .
x 1
2
Bài 7. Chứng minh rằng > , ∀x ∈ ( 0;1) .

x x
1 + −
x x
1
e
Bài 8. Chứng minh rằng với mọi x ta có
17 ≤ cos 2 x + 4cos x + 6 + cos 2 x − 2cos x + 3 ≤ 2 + 11 .

341
Khám phá tư duy Kỹ thuật giải bất ĐT Bài toán Max – Min – Đặng Thành Nam

1+ x + 3
Bài 9. Tìm giá trị lớn nhất, giá trị nhỏ nhất của hàm số y = trên nửa
x −1
khoảng (1;6] .

Bài 10. Tìm giá trị lớn nhất và giá trị nhỏ nhất của hàm số y =( x + 1) 1 − x 2 .
Bài 11. (TSĐH Khối D 2010) Tìm giá trị nhỏ nhất của hàm số
y = − x 2 + 4 x + 21 − − x 2 + 3 x + 10 .
11 7
Bài 12. Tìm giá trị nhỏ nhất của hàm số y =x + + 2 1 + 2 trên khoảng
2x x

n
( 0;+∞ ) .

.v
Bài 13. (TSĐH Khối B 2003) Tìm giá trị lớn nhất, giá trị nhỏ nhất của hàm số

om
y =x + 4 − x 2 .
Bài 14. (TN THPT 2014) Tìm giá trị lớn nhất và nhỏ nhất của hàm số
1 2
=
y x − x − 4 x − x2 . .c
4
ok

Bài 15. (TSĐH Khối D 2003) Tìm giá trị lớn nhất, giá trị nhỏ nhất của hàm số
bo

x +1
y= trên đoạn [ −1;2] .
x2 + 1
et

x2
Bài 16. Tìm giá trị lớn nhất, giá trị nhỏ nhất của hàm số y = e x − 1 − x − trên
vi

2
đoạn [ 0;1] .
ng

ln x
Bài 17. Tìm giá trị nhỏ nhất, giá trị lớn nhất của hàm số y = trên đoạn
x2 + 1
a

1 
 2 ;1 .
kh

 
ln 2 x
Bài 18. (TSĐH Khối B 2004) Tìm giá trị nhỏ nhất của hàm số y = trên đoạn
x
1;e3  .
 
Bài 19. Tìm giá trị lớn nhất và giá trị nhỏ nhất của hàm =
số y sin 5 x + 3 cos x .
x2
Bài 20. Chứng minh rằng e x + cos x ≥ 2 + x − với mọi số thực x.
2

342
Cty TNHH MTV DVVH Khang Việt

( )
Bài 21. Chứng minh rằng 1 + x ln x + x 2 + 1 ≥ x 2 + 1 với mọi số thực x.

( )
Bài 22. Chứng minh rằng ln 1 + 1 + x 2 < ln x +
1
x
với mọi số thực dương x.

Bài 25. Cho hàm số f ( x) =


( x + 1) − 2
2
. Tìm giá trị lớn nhất và giá trị nhỏ nhất
x2 + 1
=
của hàm số y f ( x). f (1 − x) trên đoạn [ −1;1] .
x
sin x + 2cos
Bài 26. Tìm giá trị lớn nhất và giá trị nhỏ nhất của hàm số y = 2 trên

n
x
cos x + 2sin
2

.v
 π
đoạn 0;  .

om
 2
Bài 27. Tìm giá trị lớn nhất và giá trị nhỏ nhất của hàm số
x +1 + 2 3 − x + 2
y= .c
.
2 x +1 + 3 − x +1
ok
Bài 28. Tìm giá trị lớn nhất và giá trị nhỏ nhất của hàm số

(
y = x 2013 + 2015 − x 2 . )
bo

1
1+
sin x cos x
Bài 29. Chứng minh rằng với mọi số thực x ta có 3 ≤ 2 +2 ≤2 . 2
et

Bài 30. Tìm tất cả các số thực a để bất đẳng thức sau luôn đúng với mọi x không
âm ln (1 + x ) ≥ x − ax 2 .
vi

Bài 31. Tìm tất cả các giá trị của k để bất đẳng thức sau đúng với mọi số thực a và
ng

b không âm e a +b ≥ 1 + a + b + k ( a − b ) .
2

Bài 31. Đáp số: k= e − 2 .


a

π π
Bài 32. Chứng minh rằng với mọi x > 2 ta có ( x + 1) cos − x cos > 1.
kh

x +1 x
1  x − x2 x 
Bài 33. Chứng minh rằng với mọi 0 < x < 1 ta có  ln − ln  < −4 .
x  x − x + 1
2 2 1 − x 

Bài 34. Chứng minh rằng với mọi số nguyên dương n ≥ 3 ta có n n +1 ≥ ( n + 1) .


n

Bài 35. Cho tam giác ABC có A ≤ B ≤ C < 900 .


2cos3C − 4cos 2C + 1
Chứng minh ≥2.
cos C

343
Khám phá tư duy Kỹ thuật giải bất ĐT Bài toán Max – Min – Đặng Thành Nam

Bài 36. Cho a,b,c là các số thực thoả mãn điều kiện a ≤ 6, b ≤ −8, c ≤ 3 . Chứng
minh với mọi x ≥ 1 ta có x 4 − ax 2 − bx − c ≥ 0 .
π
n+2
4
1 π 
37. Cho I n ∫ x tan xdx, n ≥ 1 . Chứng minh I n >
Bài= n
  .
0
n+ 2 4 
Bài 38. (Đại học khối A, 2004) Cho tam giác ABC không tù, thỏa mãn điều kiện
3 (1) .
cos 2 A + 2 2 cos B + 2 2 cos C =
Tính các góc của tam giác ABC.

n
D. HƯỚNG DẪN GIẢI – ĐÁP

.v
1
1 − sin 2 2 x
sin 4 x + cos 4 x
2 2 − sin 2 2 x
Bài 3. = =
=

om
Ta có f ( x) .
1
sin x cos x sin 2 x
sin 2 x
2

=
(1 − sin 2 x )( 2 + sin 2 x ) + 1 ≥ 1, ∀x ∈  0; π  .c
 
sin 2 x  2
π
ok
Vậy giá trị nhỏ nhất của f(x) bằng 1 đặt tại x = .
4
bo

 x 2 x3 x 2 n −1 x 2 n 
Bài 4. Xét hàm số f ( x)= ln(1 + x) −  x − + − ... + + 
 2 3 2n − 1 2n 

et

trên [ 0;+∞ ) . Hàm số đồng biến trên [ 0;+∞ ) suy ra f ( x) > f (0) , đpcm.
vi

1
Bài 5. Xét f ( x) =ln x ⇒ f '( x) = và f '( x) nghịch biến trên (0 : +∞) .
ng

x
Tương tự bài toán trên ta có:
a

n
f (n) − f (1) + f '(1) > ∑ f '(i ) > f (n + 1) − f (1), ∀n ∈ N *
kh

i =1
n
1
⇒ 1 + ln n > ∑ > ln(n + 1), ∀n ∈ N *
i =1 i

Bài 6. Xét hàm số f ( x)= x 2 + x + 1 + x 2 − x + 1 trên R. Ta có

344
Cty TNHH MTV DVVH Khang Việt

1 1 1 1
x+ x− x+ −x
f ′( x) = 2 + 2 = 2 + 2
2 2 2 2
 1 3  1 3  1 3 1  3
x+  + x−  + x+  +  − x +
 2 4  2 4  2 4 2  4
 1 1 
= g x +  + g − x
 2   2 
t
với mọi x, trong=
đó g (t ) , t ∈ R.
3
t + 2
4

n
Vì hàm g đồng biến trên R nên f ′( x) = 0 ⇔ x = 0

.v
 1 1 
f ′( x) > 0 ⇔ g  x +  > g  − x  ⇔ x > 0
 2 2 

om
Điều đó chứng tỏ f(x) đạt cực tiểu tại x = 0. Vì vậy min f ( x) = 2 ⇔ x = 0 .
Bài tập tương tự
Tìm tập giá trị của hàm số f ( x)=
.c
x2 + 2 x + 4 − x2 − 2 x + 4 .
ok
Bài 7. HD: Xét hàm số
x 1 x x x
x) x1− x + x1−=
f (= x x1− x + x.x1−=
x (1 + x) x1− x , ∀x ∈ ( 0;1) .
bo

−x − 7 − 2 x + 3
=
Bài 9. Ta có y ' < 0, ∀x ∈ (1;6] .
2 x + 3 ( x − 1)
2
et

Do đó hàm số nghịch biến trên (1;6] .


vi

4
Vì vậy y= y=
(6) , không tồn tại max.
ng

min
5
Bài 10. Tập xác định: D = [ −1;1] .
a

 1
−2 x 2 − x + 1  x=
kh

Ta có y ' = ; y ' = 0 ⇔ 2x + x −1 = 0 ⇔
2
2 .
1 − x2 
 x = −1

1 3 3
Ta có y (−1)= y (1)= 0; y  = .
2 4
1 3 3
Vậy ymax = y   = ; ymin = y (±1) = 0 .
2 4
− x 2 + 4 x + 21 ≥ 0
Bài 11. Điều kiện:  ⇔ −2 ≤ x ≤ 5 .
− x + 3 x + 10 ≥ 0
2

345
Khám phá tư duy Kỹ thuật giải bất ĐT Bài toán Max – Min – Đặng Thành Nam

Xét hàm số y = − x 2 + 4 x + 21 − − x 2 + 3 x + 10 trên đoạn [-2;5] ta có


−2 x + 4 −2 x + 3
y' − ;
2 − x + 4 x + 21
2
2 − x 2 + 3x + 10
y ' =0 ⇔ ( −2 x + 4 ) − x 2 + 3 x + 10 =−
( 2 x + 3) − x 2 + 4 x + 21
( −2 x + 3)( −2 x + 4 ) ≥ 0
 1
⇔ ⇔x=
2 2
(
( −2 x + 4 ) − x + 3 x + 10 =− )
( 2 x + 3) − x + 4 x + 21
2 2
3 ( )
1
Ta có y (=
−2) 3; y =
  2; =
y (5) 4 .

n
3

.v
1
Vậy =
ymin y=
  2.
3

om
( ( x + 3)( 5 − x ) − ( x + 2 )( 7 − x ) )
2
Chú ý. y > 0; y=
2
+2≥2⇒ y ≥ 2.

Khảo sát hàm số cho ta cả kết quả max. .c


Bài 12. Ta có

( 2x )
ok
11 14
2
− 11 x 2 + 7 − 28
y' =
1− − = ;
2 x2 7 2 x2 x2 + 7
3
1+ 2
bo

x
x

(
y ' = 0 ⇔ 2 x 2 − 11 ) x 2 + 7 − 28 = 0 ⇔ x = 3
et

Ta có y’ đổi dấu từ âm sang dương khi đi qua x = 3 nên y đạt cực tiểu tại x = 3 .
vi

15
Vì vậy y=min y=
(3) .
2
ng

Bài tập tương tự


1  1 
a

Tìm giá trị nhỏ nhất của hàm số y = 2 x + + 2 1 + 2  với x > 0 .


x  x 
kh

Bài 13. Tập xác định: D = [ −2;2] .


x
Ta có y ' =1 − ; y ' =0 ⇔ 4 − x 2 =x ⇔ x = 2 .
4− x 2

Suy ra y (−2) =−2; y ( 2) =2 2; y (2) =2.


Vậy ymax =y ( 2) =2 2; ymin =y (−2) =−2 .
Bài 14. Tập xác định: D = [ 0;4] .

346
Cty TNHH MTV DVVH Khang Việt

1 2− x 1 1 
Ta có y ' = x −1− = ( x − 2)  + ; y ' = 0 ⇔ x = 2 .
 
2 4 x − x2 2 4 x − x2 
Suy ra y (0) =
0; y (2) =
−3; y (4) =
0.
Vì vậy y=
max y=
(0) y=
(4) 0; y=
min y=
(2) −3 .
1− x
Bài 15. Ta có y ' = ; y ' = 0 ⇔ x =1.
( x2 + 1 ) x2 + 1

3
Suy ra y (−=
1) 0; y=
(1) =
2; y (2) .
5

n
Vì vậy ymax = y (1) = 2; ymin = y (−1) = 0 .

.v
Bài 16. Ta có

om
y '= e x − 1 − x; y ''= e x − 1 ≥ 0, ∀x ∈ [ 0;1]
5 .
⇒ y ' ≥ y '(0) =0 ⇒ ymax =y (1) =e − ; ymin =y (0) =0
2 .c
x2 xn
Chú ý. e x ≥ 1 + x + + ... + , ∀x ≥ 0, n ∈ N * .
ok
2 n!
Bài 17. Ta có

( )
bo

1 2 1
x + 1 − 2 x ln x x + − 2 x ln x
x x 2 − 2 x ln x 1 
= y' = ≥ > 0, ∀x ∈  ;1 .
( ) ( ) 2 
( )
2 2 2
x2 + 1 x2 + 1 x2 + 1
et

1 4ln 2
vi

Suy ra ymin =
y  =
− ; ymax =
y (1) =
0.
2 5
ng

ln x ( 2 − ln x ) ln x = 0 x =1
Bài 18. Ta có y ' =2 ; y' =
0⇔ ⇔ .
ln x = 2 x = e
2
x
a

4 9
= y (1) 0;=
y (e 2 ) =
; y (e3 )
kh

Suy ra 2
.
e e3
4
= y (e=
Vì vậy ymax 2
) = y=
; ymin (1) 0 .
e2
Bài 19. Vì hàm số đã cho tuần hoàn với chu kỳ T = 2π nên ta chỉ cần tìm giá trị
lớn nhất và giá trị nhỏ nhất trên đoạn [ 0;2π ] .

Ta có =
y ' 5sin 4 x cos x − 3 sin=
x sin x 5sin 3 x cos x − 3 ; ( )
y' = (
0 ⇔ sin x 5sin 3 x cos x − 3 (1) )
Chú ý.
347
Khám phá tư duy Kỹ thuật giải bất ĐT Bài toán Max – Min – Đặng Thành Nam

3
1  3cos 2 x + 3sin 2 x 
( )
1 3
2
sin x cos=
3
x 3cos 2 x.sin 2 x.sin 2 x.sin 2 x ≤   < .
3 3  4  27
Vì vậy (1) ⇔ sin x = 0 ⇔ x = kπ ⇒ x = 0, x = π , x = 2π .
Suy ra y (0) = y (2π ) = 3; y (π ) = − 3 .
Vậy ymax = y (0) = y (2π ) = 3; ymin = y (π ) = − 3 .
x2
Bài 20. Xét hàm số y = e x + cos x + − x − 2 ta có
2
y ' = e x − sin x + x − 1; y '' = e x − cos x + 1 > 0, ∀x .

n
Mặt khác y '(0) = 0 nên y ' > 0, ∀x ∈ ( 0; +∞ ) ; y ' < 0, ∀x ∈ ( −∞;0 ) .

.v
0 (đpcm).
Do đó hàm số đạt cực tiểu tại x = 0 . Vì vậy y ≥ y (0) =

om
(
Bài 21. Xét hàm số y =1 + x ln x + x 2 + 1 − x 2 + 1 ta có ) .c
y' = (
ln x + x 2 + 1 ; y ' =
0 ⇔ x + x2 + 1 =
1⇔ x =)
0.
ok
Ta có y’ đổi dấu từ âm sang dương khi đi qua x = 0 nên đạt cực tiểu tại x = 0 .
Vì vậy y ≥ y (0) = 0 (đpcm).
bo

x 2 (1 − x ) + 8 x (1 − x ) − 2
2
Bài 25. Tính được y = .
x 2 (1 − x ) − 2 x (1 − x ) + 2
2
et

 1 t 2 + 8t − 2
t x (1 − x ) ∈  −2;  , ∀x ∈ [ −1;1] khi đó y = 2
vi

Đặt = .
 4 t − 2t + 2
( ) ; y ' = 0 ⇔ 5t
ng

 1
2 5t 2 − 4t − 6 t∈ −2; 
2 − 34
 4
Ta có y ' = − 2
− 4t − 6 = 0 ←→ t = .
(t )
2 5
2
− 2t + 2
a
kh

 2 − 34  1 1
Tính các giá trị suy ra ymin = y   =4 − 34; ymin =
y  = .
 5   4  25
3x
sin − 1
2  π
=
Bài 26. Ta có y ' ≤ 0, ∀x ∈ 0;  .
 x
2
 2
 cos x + 2sin 
 2
π  2
Vì vậy ymin = y (0) = 2; ymax = y   = 1 + .
2 2

348
Cty TNHH MTV DVVH Khang Việt

Bài 27. Tập xác định: D = [ −1;3] .


(
2 1− t2 ).
( ) +( ) 4t
2 2
Do x +1 3− x = 4 ⇒ ∃t ∈ [ 0;1] | x + 1 = , 3− x=
t2 +1 t2 +1
2t 2 − 4t − 6
Khi đó y = .
t 2 − 8t − 3
4
Từ đó dễ có ymin
= y=(1) = y=
; ymax (0) 2 .
5
Bài 28. Tập xác định: D =  − 2015; 2015  .

n
2015 − 2 x 2
Ta có y ' =
2013 + ; y ' =⇔ 2 x 2 − 2015 =
2013 2015 − x 2 .

.v
0
2015 − x 2

om
2 x − 2015 ≥ 0
2

⇔ 2 ⇔x=± 2014
(
 2 x − 2015
= 20132 2015 − x 2 ) ( )
Suy ra ymax =
y ( 2014) =
2014 2014; ymin =
y (− 2014) =
−2014 2014 .
.c
ok
Bài 29. Đặt và t sin x , t ∈ [ 0;1] khi đó = 1−t 2
sin x cos x
= y 2 +2 = y 2t + 2
t 1−t 2 t 1−t 2
bo

Ta có y ' =2t ln 2 − 2 ln 2; y ' =0 ⇔ 2t = 2


1− t2 1− t2
0 < t < 1
et


⇔  2t 2 1−t 2
 =
vi

(1)
 t 1− t2
ng

2a
Xét hàm số f (a ) = trên khoảng ( 0;1) ta có
a
a

2a ( a ln 2 − 1)
=f '(a ) < 0, ∀a ∈ ( 0;1) . Do đó f(a) nghịch biến trên khoảng ( 0;1) .
kh

a2

Vì vậy (1) ⇔ f (t ) = f ( 1− t ) ⇔ t = 2
1− t2 ⇔ t =
1
2
.

1
 1  1+
Ta có y=
(0) y=
(1) 3; y  =  2
2 .
 2
1
 1  1+
= y=
Suy ra ymin (0) y= = y =
(1) 3; ymax  2
2 (đpcm).
 2

349
Khám phá tư duy Kỹ thuật giải bất ĐT Bài toán Max – Min – Đặng Thành Nam

Bài tập tương tự


1
sin x cos x
Chứng minh rằng với mọi số thực x ta có 8 ≤ 4 + 4.2 ≤ 5.4 5 .
Bài 30. Xét hàm số f ( x)= ax 2 − x + ln (1 + x ) trên [ 0;+∞ ) .
Ta cần tìm a để f ( x) ≥ 0, ∀x ≥ 0 .
x
Ta có =
f '( x) ( 2ax + 2a − 1) .
x +1
+ Nếu a ≤ 0 ⇒ f '( x) ≤ 0 ⇒ f ( x) ≤ f (0) =0 không thoả mãn.
1 1 − 2a
+ Nếu 0 < a < ⇒ f '( x) = 0 ⇔ x = 0 hoặc x = .

n
2 2a

.v
Khi đó f ( x) ≤ f (0) =0 không thoả mãn.
1

om
+ Nếu a ≥ ⇒ 2ax + 2a − 1 ≥ 0 ⇒ f '( x) ≥ 0 ⇒ f ( x) ≥ f (0) =0 thoả mãn.
2
1
Vậy giá trị cần tìm của a là : a ≥ . .c
2
π
Bài 32. HD: Sử dụng định lý Lagrange cho hàm số y = t cos .
ok
t
x
Bài 33. Hướng dẫn sử dụng định lý Lagrange cho hàm số y = ln .
bo

1− x
n +1 n
Bài 34. Lấy logarit tự nhiên hai vế ta cần chứng minh ≥ .
ln ( n + 1) ln n
et

x ln x − 1
Xét hàm số f ( x) = với x ≥ 3 ta có f= > 0, ∀x ≥ 3 .
vi

'( x)
ln x ln 2 x
ng

Từ đó suy ra f (n + 1) > f (n) ⇒ .


 1
Bài 35. Đưa về chứng minh y = 8t 3 − 8t 2 − 8t + 5 ≥ 0, t = cos C ∈  0;  .
a

 2
kh

π n+2
 π 1 π 
Bài 37. Sử dụng tan x ≥ x, ∀x ∈ 0;  ⇒ I n > ∫ 4 x n +1dx =   . (Xem
 4 0 n+ 2 4 
thêm chủ đề Kỹ thuật đánh giá bất đẳng thức Tích phân).
π A 1
Bài 38. Từ giả thiết 0 < A ≤ , suy ra 0 < sin ≤ .
2 2 2
Ta có (1) ⇔ 1 − 2sin 2 A + 2 2 ( cos B + cos C ) =3
A B−C
⇔ − sin 2 A + 2 2 sin .cos =1 ( 2)
2 2

350
Cty TNHH MTV DVVH Khang Việt

A B −C A
Lại có 2 2 sin .cos ≤ 2 2 sin ( 3)
2 2 2
A
Từ (2) và (3) ta có − sin 2 A + 2 2 sin − 1 ≥ 0 .
2
A
(
Đặt t = sin thì bất đẳng thức trở thành: −4t 2 1 − t 2 + 2 2t − 1 ≥ 0
2
) ( 4)
Xét hàm f ( t ) = 4t 4 − 4t 2 + 2 2t − 1 .
1
Ta phải có f ( t ) ≥ 0 với 0 < t ≤ (5).
5

n
1
f ′ ( t ) = 16t 3 − 8t + 2 2; f ′′ ( t ) = 48t 2 − 8 ⇒ f ′′ ( t ) = 0 ⇔ t =

.v
6

om
1 1 1
Suy ra f ′′ ( t ) < 0 ⇔ 0 < t < ; f ′′ ( t ) < 0 ⇔ <t <
6 6 2
 1   1 
Vì vậy f ′ ( t ) ≥ f ′   > 0 nên f(t) đồng biến trên .c  0; .
 6  2
 1 
ok
Do đó f ( t ) ≤ f  =0.
 2
π
bo

1 A 1
Vì vậy dấu bằng xảy ra khi và chỉ khi t = ⇔ sin = ⇔ A=
2 2 2 2
y = x 2 + x − 12, y ≤ 0 ⇔ x 2 + x − 12 ≤ 0 ⇔ −4 ≤ x ≤ 3
et

B−C π
Thay vào ta được cos =1 ⇒ B =C = .
vi

2 4
ng

CHỦ ĐỀ 2: KỸ THUẬT SỬ DỤNG TÍNH ĐƠN ĐIỆU CHO BÀI TOÁN


CỰC TRỊ VÀ BẤT ĐẲNG THỨC HAI BIẾN SỐ
a
kh

A. NỘI DUNG PHƯƠNG PHÁP


Nguyên tắc chung của kỹ thuật này là dùng phép thế(rút biến x theo y hoặc
ngược lại) thay vào biểu thức cần tìm cực trị đưa về khảo sát hàm một biến. Tuy
nhiên nếu tinh tế sử dụng các bất đẳng thức cơ bản(tổng và tích đối xứng với hai
biến) và nhận ra các phép đặt đặc biệt đưa về một biến mới chung giữa hai biến sẽ
giúp ta xử lý bài toán đơn giản hơn.
1. Kỹ thuật thế biến đưa về khảo sát hàm một biến
Rút x theo y(hoặc rút y theo x) thay vào biểu thức cần tìm cực trị(bất đẳng
thức) đưa về khảo sát hàm một biến(bất phương trình).
Như vậy kỹ năng cần có là bài toán khảo sát tính đơn điệu của hàm một biến số.

351
Khám phá tư duy Kỹ thuật giải bất ĐT Bài toán Max – Min – Đặng Thành Nam

Ví dụ 1. Cho x,y là hai số thực thỏa mãn điều kiện y ≤ 0, x 2 + x − y − 12 =0.


Tìm giá trị nhỏ nhất của biểu thức P = xy + x + 2 y + 17 = 0 .
Lời giải
Từ giả thiết ta có: y = x + x − 12, y ≤ 0 ⇔ x 2 + x − 12 ≤ 0 ⇔ −4 ≤ x ≤ 3 .
2

( ) ( )
Khi đó P = x x 2 + x − 12 + x + 2 x 2 + x − 12 + 17 = x3 + 3 x 2 − 9 x − 7 .

Xét hàm số f ( x) = x3 + 3 x 2 − 9 x − 7 liên tục trên đoạn [ −4;3] ta có:


x =1
f '( x) =3 x 2 + 6 x − 9; f '( x) =0 ⇔ 
 x = −3

n
Ta có f (−4) = 13, f (−3) = 20, f (1) = −12, f (3) =20

.v
⇒ max f ( x) =f (−3) =f (3) =20, min f ( x) =f(1) =−12 .
x∈[ −4;3] x∈[ −4;3]

om
Vậy giá trị lớn nhất của P bằng 20 đạt tại ( x=
; y) ( 3;0 ) ; ( −3; −6 ) và giá trị nhỏ
nhất của P bằng −12 đạt tại ( x; y=
) (1; −10 ) .
Ví dụ 2. Cho x,y là hai số thực thỏa mãn điều kiện 2 x − y =2.
.c
ok
x 2 + ( y + 1) + x 2 + ( y − 3) .
2 2
Tìm giá trị nhỏ nhất của biểu thức P =
Lời giải
bo

y 2 x − 2 thay vào biểu thức của P ta được


Theo giả thiết ta có: =

=
P 5 x 2 − 4 x + 1 + 5 x 2 − 20 x + 25 .
et

Xét hàm số f (=
x) 5 x 2 − 4 x + 1 + 5 x 2 − 20 x + 25 liên tục trên  ta có:
vi

5x − 2 5 x − 10
=f '( x) + ;
ng

5x − 4x + 1
2
5 x − 20 x + 25
2

0 ⇔ ( 5 x − 2 ) 5 x 2 − 20 x + 25 =
f '( x) = − ( 5 x − 10 ) 5 x 2 − 4 x + 1
a

( 5 x − 2 )( 5 x − 10 ) ≤ 0

⇔
kh

( ) (
( 5 x − 2 ) 5 x − 20 x + 25 = ( 5 x − 10 ) 5 x − 4 x + 1
2 2 2 2
)
2
 ≤x≤2 2
⇔ 5 ⇔x=
24 x 2 − 16 x = 3
 0
2
Từ đó suy ra P =f ( x) ≥ f   =2 5 .
3
2 2
Đẳng thức xảy ra khi và chỉ khi x = , y = − .
3 3
352
Cty TNHH MTV DVVH Khang Việt

Ví dụ 3. Cho a,b là các số thực dương thoả mãn điều kiện a + b =


1.
Tìm giá trị nhỏ nhất của biểu thức P =3 1 + 2a 2 + 2 40 + 9b 2 .
Lời giải
Theo giả thiết ta có a =1 − b, ( 0 < b < 1) .

Khi đó P = 3 1 + 2 (1 − b ) + 2 40 + 9b 2 .
2

Xét hàm số f (b) = 3 1 + 2 (1 − b ) + 2 40 + 9b 2 với 0 < b < 1 ta có


2

6 ( b − 1) 18b
=f '(b) +

n
2b − 4b + 3
2
9b 2 + 40

.v
f '(b) =0 ⇔ (1 − b ) 9b 2 + 40 =3b 2b 2 − 4b + 3
.
(
⇔ (1 − b ) 9b += ) ( 2b )

om
2
40 9b 2 2 2
− 4b + 3

( 2
⇔ ( b + 2 )( 3b − 2 ) 3b 2 − 10b + 10 = 0 ⇔ b =
3
) .c
1 2
Vậy giá trị nhỏ nhất của P bằng 5 11 đạt tại=
a = ;b .
ok
3 3
Ví dụ 4. Cho a,b là hai số thực thỏa mãn a 2 + b 2 =
1.
bo

3 3
Chứng minh rằng ab + max {a, b} ≤ .
4
et

Lời giải
3 3
Nếu a và b trái dấu khi đó ab + max {a, b} ≤ max {a, b} ≤ 1 < ta có ngay điều
vi

4
phải chứng minh.
ng

Vậy a và b cùng dấu và nếu cả a và b đều âm khi đó:


a 2 + b2 1 3 3
ab + max {a, b} ≤ ab ≤
a

= < , ta có ngay điều phải chứng minh.


2 2 4
kh

Xét với a, b ≥ 0 khi đó không mất tính tổng quát giả sử a = max {a, b} ta cần
3 3
chứng minh: ab + a ≤ .
4
Từ điều kiện ta có: a = 1 − b 2 ⇒ ab + a = f (b) = ( b + 1) 1 − b 2 .

Xét hàm số f (b) =( b + 1) 1 − b2 trên đoạn [0;1] ta có:


f '(b) =
(1 + b )(1 − 2b ) ; f '(b) = 0 ⇔ b = 1 .
1 − b2 2

353
Khám phá tư duy Kỹ thuật giải bất ĐT Bài toán Max – Min – Đặng Thành Nam

1 1 3 3
Ta có f’(b) đổi dấu từ dương sang âm khi đi qua b = nên f   = là giá
2 2 4
3 3
trị cực đại hay f (b) ≤ . Ta có ngay điều phải chứng minh. Đẳng thức xảy
4
3 1
=
ra khi và chỉ khi a = ,b .
2 2
Cách 2: Thực hiện lập luận như trên và sử dụng bất đẳng thức AM-GM ta có:
1  3  1 
ab + a= a ( b + 1)= .  2. .a   2. .b + 1
3  2  2 

n
2
 2 3 5

.v
a + + b2 + 
1  2 3  2 5  1  4 4 =3 3
≤  a +  b +  ≤  
3 4  4

om
3 2  4
 

2. Kỹ thuật xử lý với biểu thức đối xứng hai biến .c


Bước 1: Từ điều kiện đặt t= x + y (hoặc t = xy ) rút t theo xy(hoặc x+y).
ok
Bước 2: Thay vào biểu thức của P ta được một hàm số với t.
Bước 3: Tìm miền giá trị của t dựa vào bất đẳng thức cơ bản ( x + y ) ≥ 4 xy (hoặc
2
bo

điều kiện ràng buộc của biến đề bài cho). Ta tìm được miền giá trị của t là [ a; b ] .
Bước 4: Xét hàm số f(t) trên đoạn [ a; b ] từ đó tìm được giá trị lớn nhất, nhỏ nhất
et

của f(t) và suy ra giá trị lớn nhất, nhỏ nhất của P.
vi

Chú ý. Đôi khi cần đánh giá P thông qua các bất đẳng thức cơ bản trước tiên để
giảm sự cồng kềnh của hàm f(t) giúp bài toán thực hiện đơn giản hơn. Một số bài
ng

toán có thể thực hiện theo đánh giá điều kiện có nghiệm của phương trình bậc 2.
Dấu hiện nhận biết:
- Rút được biến này theo kiến kia và đưa biểu thức về dạng một biến.
a

- Biểu thức có dạng đối xứng giữa tổng và tích.


kh

- Biểu thức có dạng đối xứng giữa các biến.


- Biểu thức là tổng hợp của các hàm(hàm đa thức,căn thức, lượng giác, mũ và
logarit).
Một số đánh giá cơ bản:
a) Với hai số thực không âm x,y ta có:
t2
t = x + y ⇒ xy ≤
4
(
,0 ≤ t ≤ 2 x 2 + y 2 . )
a b 2
b) Với a,b dương ta có + ≥ .
b + ab a + ab 1 + ab

354
Cty TNHH MTV DVVH Khang Việt

Ví dụ 1. Cho x,y là hai số thực không âm thay đổi thỏa mãn điều kiện
( )
4 x 2 + y 2 + xy ≤ 1 + 2 ( x + y ) .

Tìm giá trị lớn nhất của biểu thức P = xy + x + y − x 2 − y 2 .


Lời giải
2
 x+ y 1
 , −x − y ≤ − ( x + y) .
2
Nhận xét. Ta có: xy ≤  2 2
 2  2
Do vậy để tìm giá trị lớn nhất của P ta quy về ẩn t= x + y và việc đầu tiên cần
làm đó là tìm khoảng giá trị của t= x + y .

n
2
 x+ y 1 1
 + x + y − ( x + y) = − ( x + y) + x + y .
2 2
Ta có: P ≤ 

.v
 2  2 4

om
1
Đặt =
t x + y , (t ≥ 0) ⇒ P ≤ − t 4 + t .
4
Xuất phát từ điều kiện bài toán ta có:

 
2
.c
4 ( x + y ) − xy  ≤ 1 + 2 ( x + y ) ⇔ 4 ( x + y ) − 2 ( x + y ) − 1 ≤ 4 xy ≤ ( x + y ) .
2 2
ok
1
Suy ra 3t 2 − 2t − 1 ≤ 0 ⇔ − ≤ t ≤ 1 do t > 0 nên 0 < t ≤ 1 .
3
bo

1
Xét hàm số f (t ) = − t 4 + t với 0 < t ≤ 1 ta được:
4
3 3
f '(t ) =−t 3 + 1 ≥ 0, ∀t ∈ ( 0;1] ⇒ f (t ) ≤ f (1) = ⇒ P ≤ .
et

4 4
vi

3 1
Vậy giá trị lớn nhất của P bằng đạt tại x= y= .
4 2
ng

Ví dụ 2(TSĐH Khối D 2014) Cho hai số thực x,y thoả mãn các điều kiện
1 ≤ x ≤ 2;1 ≤ y ≤ 2 .
a

x + 2y y + 2x 1
Tìm giá trị nhỏ nhất của biểu thức P = + + .
4 ( x + y − 1)
kh

x + 3y + 5
2
y + 3x + 5
2

Lời giải
Do 1 ≤ x ≤ 2 nên ( x − 1)( x − 2 ) ≤ 0 ⇔ x 2 + 2 ≤ 3 x .
Tương tự ta có y 2 + 2 ≤ 3 y .
x + 2y y + 2x 1
Suy ra P ≥ + +
3x + 3 y + 3 3x + 3 y + 3 4 ( x + y − 1)
x+ y 1
= +
x + y + 1 4 ( x + y − 1)

355
Khám phá tư duy Kỹ thuật giải bất ĐT Bài toán Max – Min – Đặng Thành Nam

t 1
Đặt t = x + y, ( 2 ≤ t ≤ 4 ) ta có P ≥ f (t ) = + .
t + 1 4 ( t − 1)
Ví dụ 3. Cho x,y là hai số thực dương.
1 1 1 5
Tìm giá trị nhỏ nhất của biểu thức P = + + + ln(xy) .
x 2
y 2
x +y
2 2 2

Lời giải
Nhận xét. Biểu thức đối xứng với x và y dự đoán dấu bằng xảy ra tại x = y, khi đó
1 1 1 5 5 1 1 1 5
+ + = = nên ta đánh giá 2 + 2 + 2 với .
x 2
y 2
x +y2 2
2x 2 2 xy x y x +y 2 2xy

n
Ta có:

.v
1 1 1 5  1 1 2  1 1 
+ + − =  2 + 2 −  +  2 − 
2 xy 

om
x 2
y 2
x +y2 2 2 xy  x y xy   x + y 2

 
( x − y)
2
( x − y)
2
1 1
= − = ( x − y )  2 2 − 2 
2 2
x y 2 xy x + y
2 2
(  )
.c
x y 2 xy x 2 + y 2 ( ) 

( )
( x − y )2  2 x 2 + y 2 − xy  ( x − y )2 [ 2.2 xy − xy ]
ok
= ≥ ≥0
2 x2 y 2 x2 + y 2 ( )
2 x2 y 2 x2 + y 2 ( )
bo

5 1  5 1  1 
Vậy P ≥  + ln( xy )  =  − ln    .
2  xy  2  xy  xy  
et

1 5
Đặt
= t , ( t > 0 ) khi đó P ≥ f (t ) = ( t − ln t ) .
vi

xy 2
ng

5
Xét hàm số f =
(t ) ( t − ln t ) với t > 0 ta có:
2
5  1
a

f '(t ) = 1 −  ; f '(t ) = 0 ⇔ t = 1 .
2 t 
kh

Ta có f’(t) đổi dấu từ âm sang dương khi đi qua t = 1 nên f(t) đạt cực tiểu tại
5
t = 1 hay P ≥ f (t ) ≥ f (1) = .
2
Đẳng thức xảy ra khi và chỉ khi x= y= 1 .
5
Vậy giá trị nhỏ nhất của P bằng đạt tại x= y= 1 .
2

356
Cty TNHH MTV DVVH Khang Việt

2
Ví dụ 4. Cho x,y là hai số thực dương thỏa mãn điều kiện 3 xy + 3 = x 4 + y 4 + .
xy
16
=
Tìm giá trị lớn nhất của biểu thức P x2 y 2 + .
x + y2 + 2
2

Lời giải
2 2
Xuất phát từ giả thiết ta có: 3 ( xy + 1) = x 4 + y 4 + ≥ 2 x2 y 2 + .
xy xy
⇔ 2 x3 y 3 + 2 ≤ 3 x 2 y 2 + 3 xy ⇔ 2 x3 y 3 − 3 x 2 y 2 − 3 xy + 2 ≤ 0
1
⇔ ( xy + 1)( 2 xy − 1)( xy − 2 ) ≤ 0 ⇔ ≤ xy ≤ 2

n
2

.v
16 8
Khi đó P ≤ x 2 y 2 + = x2 y 2 + .
2 xy + 2 xy + 1

om
1  8
Đặt
= t xy,  ≤ t ≤ 2  khi đó P ≤ f (t ) =
t2 + .
2  t +1 .c
8 1 
Xét hàm số f (t=
) t2 +
t +1
liên tục trên  2 ;2  ta được:
ok
 
8
f '(t ) = 2t − ; f '(t ) = 0 ⇔ t = 1 .
( t + 1)2
bo

 1  67 11 20 20
Ta có= f  = , f (1) = , f (2) suy ra P ≤ f (t ) ≤ f(2) =.
et

 2  12 3 3 3
20
đạt tại x= y= 2 .
vi

Vậy giá trị lớn nhất của P bằng


3
ng

Ví dụ 5. Cho x, y là các số thực dương thỏa mãn điều kiện x + y + xy = 3.


x +1 3 y +1 3
Tìm giá trị nhỏ nhất của biểu thức:=
P 4( ) + 4( ) + x2 + y 2 .
a

y x
kh

Lời giải
Sử dụng bất đẳng thức a3 + b3 ≥ ab ( a + b ) ta có
 x +1 y +1 x +1 y +1
P ≥ 4   + + x + y
2 2

 y   x   y x 
(
16 x 2 + y 2 + x + y )+ x 2 + y 2 ≥ 64 + 2
2 2
x y

357
Khám phá tư duy Kỹ thuật giải bất ĐT Bài toán Max – Min – Đặng Thành Nam

3. Đa thức có dạng đẳng cấp


Dạng toán này đã đề cập đến trong chủ đề kỹ thuật sử dụng tính đẳng cấp đó là
đặt x = t. y hoặc y = t.x đưa về khảo sát hàm một biến với t.
Dạng toán này vài năm trở lại đây thường xuất hiện trong đề thi TSĐH của Bộ và
kỹ thuật khá đơn giản nên các em nắm chắc phương pháp có thể hoàn toàn xử lý được.
Ví dụ 1. Cho x,y là hai số thức khác 0. Tìm giá trị nhỏ nhất của biểu thức
x4 y4 x2 y2 x y
P= 4
+ 4
− 2( 2
+ 2
)+ + .
y x y x y x
Lời giải

n
4 2
 x y  x y x y
Ta có P =  +  − 6  +  + + + 6 .

.v
 y x  y x y x
x y

om
Đặt t =+ , t ≥ 2 khi đó P = t 4 − 6t 2 + t + 6 .
y x
Xét hàm số f (t ) = t 4 − 6t 2 + t + 6 trên ( −∞; −2]  [ 2; +∞ ) , ta có .c
f '(= t ) 12t 2 − 12 > 0, ∀t ∈ ( −∞; −2]  [ 2; +∞ ) .
t ) 4t 3 − 12t + 1; f ''(=
ok
Do đó f '(t ) ≤ f '(−2) = −7, ∀t ∈ ( −∞; −2]; f '(t ) ≥ f '(2) = 9, ∀t ∈ [ 2; +∞ ) .
Vì vậy f(t) nghịch biến trên ( −∞; −2] và đồng biến trên [ 2;+∞ ) .
bo

Suy ra f (t ) ≥ min { f (−2), f (2)} =f (−2) =−4 .


Vậy giá trị nhỏ nhất của P bằng -4 đạt tại x = − y .
et

Ví dụ 2. Cho x,y là hai số thực dương phân biệt thỏa mãn điều kiện x 2 + 2 y =
12 .
vi

4 4 5
Tìm giá trị nhỏ nhất của biểu thức P = + + .
ng

8( x − y )
4 4 2
x y
Lời giải
a

Ta thấy P chưa có dạng đẳng cấp tuy nhiên khéo léo sử dụng điều kiện
kh

Ta có: 12 = x 2 + 2 y = x 2 + y + y ≥ 3 3 x 2 y 2 ⇒ xy ≤ 8 .
Suy ra
1 2 2 4 4  xy 5 1  x2 y 2  5 xy
P≥ x y  4 + 4  + . =  2 + 2  + .
64 x y  8 8( x − y ) 2 16  y x  64 ( x − y )2

1  x y   5
2
1
=   +  − 2 + . x
16  y x   64 − 2 + y

y x

358
Cty TNHH MTV DVVH Khang Việt

x y
Đặt t =+ , ( t > 2 ) , do x,y dương phân biệt.
y x
1 2 5 1
Khi đó P ≥ f (=
t) t − 2 + . .
16   64 t − 2
1 2 5 1
Xét hàm số f (=
t) t − 2 + . trên ( 2;+∞ ) ta có
16   64 t − 2
t 5 5
t ) 0 ⇔ 8t ( t − 2 ) −=
2 t >2
f '(=
t) − ; f '(= 5 0 ← →=
t .
8 64 ( t − 2 ) 2 2
5 5
Tại t = f’(t) đổi dấu từ âm sang dương nên tại t = f(t) đạt cực tiểu hay

n
2 2

.v
 5  27
f (t ) ≥ f   = .
 2  64

om
27
Vậy giá trị nhỏ nhất của P bằng đạt tại=
x 2,=
y 4.
64
Ví dụ 3. Cho x,y là hai số thực khác 0. Chứng minh rằng .c
 x2 y 2 
48 x 2 y 2
+ 1001 2 + 2  ≥ 2014 .
ok
y x 
(x )
2
2
+ y2 
bo

Lời giải
Bất đẳng thức đã cho tương đương với:
 x y  2 
et

2
48 48  x y
+ 1001  +  − 2  ≥ 2014 ⇔ + 1001  +  ≥ 4016 .
 x y
2
 y x    x y
2
 y x
vi

 +   + 
 y x  y x
ng

Chú ý sử dụng bất đẳng thức AM – GM ta có


2 2
48  x y 48  x y
a

2
+ 3 +  ≥ 2 2
.3  +  =24
 x y  y x  x y  y x
kh

 +   + 
 y x  y x
2
 x y
998  +  ≥ 998.22 = 3992
 y x
Cộng theo vế hai bất đẳng thức trên ta có đpcm.
4. Kỹ thuật khảo sát hàm đặc trưng
Nếu bất đẳng thức cần chứng minh có dạng hàm đặc trưng f (a ) > f (b) hoặc
f (a ) < f (b) .

359
Khám phá tư duy Kỹ thuật giải bất ĐT Bài toán Max – Min – Đặng Thành Nam

Khi đó ta chứng minh hàm số f ( x) luôn đồng biến hoặc luôn nghịch biến trên
miền D thuộc điều kiện của bài toán.
Ví dụ 1. (TSĐH Khối D 2007) Cho a ≥ b > 0 . Chứng minh rằng
b a
 a 1   b 1 
2 + a  ≤ 2 + b  .
 2   2 
Lời giải
Bất đẳng thức đã cho tương đương với:
(
ln 1 + 4a ) ≤ ln (1 + 4 ) .
b

(1 + 4 ) ≤ (1 + 4 )
a b b a
⇔ b ln 1 + 4 ( a
) ≤ a ln (1 + 4 ) ⇔
b
a b

n
(
ln 1 + 4 x ) với x > 0 ta có

.v
Xét hàm số f ( x) =
x

om
f '( x)
x x
( ) (
4 ln 4 − 1 + 4 ln 1 + 4 x
x
) < 0, ∀x > 0 .
x2 (1 + 4 ) x
.c
Do đó f(x) là hàm nghịch biến trên ( 0;+∞ ) . Vì vậy với a ≥ b ⇒ f (a ) ≤ f (b) .
ok
Bất đẳng thức được chứng minh. Đẳng thức xảy ra khi và chỉ khi a = b .
Ví dụ 2. Chứng minh rằng với mọi x, y ∈ ( 0;1) , x ≠ y ta có
bo

1  y x 
 ln − ln >4 (1) .
y − x  1− y 1− x 
et

Lời giải
• Nếu y > x thì
vi

y x y x
(1) ⇔ ln − ln > 4 ( y − x ) ⇔ ln − 4 y > ln − 4x .
ng

1− y 1− x 1− y 1− x
• Nếu y < x thì
a

y x y x
(1) ⇔ ln − ln < 4 ( y − x ) ⇔ ln − 4 y < ln − 4x .
kh

1− y 1− x 1− y 1− x
t
f ( t ) ln
Xét hàm số= − 4t với t ∈ ( 0;1) . Ta có
1− t
1 − t  t ′ (2t − 1) 2
f ′(t )
=  = − 4 > 0, ∀t ∈ ( 0;1) .
t 1− t  t (1 − t )
Suy ra f(t) tăng trên (0;1). Suy ra f(y) > f(x) nếu y > x và f(y) < f(x) nếu y < x
1  y x 
⇔  ln − ln  > 4, ∀x, y ∈ ( 0;1) , x ≠ y .
y − x  1− y 1− x 

360
Cty TNHH MTV DVVH Khang Việt

Bất đẳng thức được chứng minh.


t
Chú ý. Theo định lý Lagrange cho hàm số y = ln ta có
1− t
f ( y ) − f ( x) 1 1 4
VT = =f '(c) =+ ≥ 4 (đpcm).
=
y−x c 1− c c +1− c
5. Kỹ thuật xét riêng lẻ từng biến
Nội dung kỹ thuật này tôi sẽ đề cập chi tiếp hơn trong nội dung chủ đề khảo sát
hàm nhiều biến. Loại toán này thường áp dụng với các bài toán cực trị khi các biến
thuộc đoạn giá trị cho trước hoặc có điều kiện ràng buộc giữa các biến với nhau.
Ví dụ 1. Xét hàm số f ( x, y ) =−
(1 x )( 2 − y )( 4 x − 2 y ) trên miền

n
= {( x, y ) | 0 ≤ x ≤ 1,0 ≤ y ≤ 2} .

.v
D
Tìm giá trị nhỏ nhất của hàm f trên miền D.

om
Lời giải
Biến đổi hàm số đã cho thành
f ( x, y ) = 2 (1 − x )( 2 − y )( (2 − y ) − 2(1 − x) ) .c
Đặt u= 1 – x, v = 2 – y, ta chuyển về tìm GTNN của hàm số:
ok
F ( u, v ) = =
−2uv 2 + u 2 v trên miền E {( u, v ) ,0 ≤ u ≤ 2,0 ≤ v ≤ 1} , nghĩa là
(u , v) min  min (−2uv 2 + u 2 v)  .
bo

min F =
E  0≤v≤1
0≤u ≤ 2  
Xét hàm số g ( v ) =
−2uv 2 + u 2 v với 0 ≤ v ≤ 1 , coi u là tham số. Ta có
et

g′(v) =−4uv + u 2 =u (−4v + u ) .


vi

u u 1 u
Ta thấy g’(v) = 0 khi v0 = , mà 0 ≤ ≤ và qua v0 = thì g’(v) đổi dấu từ
ng

4 4 2 4
dương sang âm, suy ra
ax g ( v ) min { g (=
m= 0 ) , g (1)} min 0; u 2 − 2u { } ( do u 2 − 2u ≤ 0 ).
a

0≤v ≤1
kh

Vậy min F (u , v) = −1 khi u = 1, v = 1. Từ đó


min (u 2 − 2u ) =
E 0≤u ≤ 2

min f ( x, y ) = 2 min F (u , v) = −2 khi x = 0, y = 1.


D E
Ví dụ 2. Cho 0 < a, b ≤ 1 . Chứng minh rằng tan a.tan b ≥ tan ab .
Lời giải
Giả sử a ≥ b . Đặt
= f ( x ) tan b.tan x − tan bx với b ≤ x ≤ 1 . Ta có
tan b b
′( x)
f= 2
− .
cos x cos 2bx

361
Khám phá tư duy Kỹ thuật giải bất ĐT Bài toán Max – Min – Đặng Thành Nam

1 1
Do 0 < a, b ≤ 1 nên tan b > b > 0 và suy ra f ′ ( x ) > 0 , nên f

cos x cos 2bx 2

đồng biến trên [b; 1]. Vì vậy với a ≥ b ta có f ( a ) ≥ f ( b ) . Suy ra

tan a.tan b − tan ab ≥ tan 2 b − tan b 2 (1) .


g ( x ) tan 2 x − tan x 2 , có
Đặt =
2 tan x 2x
′( x)
g= − > 0; ∀x ∈ ( 0;1)
cos x cos 2 x
Suy ra g ( b ) > g ( 0 ) =
0 ⇒ tan 2 b − tan b 2 > 0 ( 2)

n
Từ (1) và (2) suy ra đpcm.

.v
π π
Ví dụ 3. Chứng minh rằng với mọi 0 < x < và 0 < y < ta luôn có
4 4

om
4cos x cos y
cos ( x − y ) ≤ .
( cos x + cos y )2
Lời giải
.c
x+ y x− y
ok
Ta có cosx + cos y = 2cos cos .
2 2
Do vai trò x, y như nhau nên ta có thể giả sử x ≥ y .
bo

Khi đó viết lại bất đẳng thức được viết lại dưới dạng
x+ y x− y
cos 2 cos 2 .cos ( x − y ) ≤ cos x.cos y .
et

2 2
Lấy logarit tự nhiên hai vế ta được
vi

x+ y x− y
2ln cos + 2ln cos + ln cos ( x − y ) ≤ ln cos x + ln cos y .
ng

2 2
Đến đây ta xét hàm số
x+ y x− y  π
a

=
f ( x) ln cos x + ln cos y − 2ln cos − 2ln cos − ln cos ( x − y ) , ∀x ≥ y; x, y ∈  0; 
2 2  4
kh

x− y x+ y
− tan x + tan ( x − y ) + 2 tan
Ta có f '( x) = + 2 tan .
2 2
sin y x− y x+ y  π
= + 2 tan + 2 tan > 0, ∀x ≥ y; x, y ∈  0; 
cos x.cos ( x − y ) 2 2  4
Do đó f là hàm tăng. Do đó f ( x) ≥ f ( y ) =
0 , điều này tương đương với
x+ y x− y
2ln cos + 2ln cos + ln cos ( x − y ) ≤ ln cos x + ln cos y .
2 2
Bài toán được chứng minh hoàn toàn. Đẳng thức xảy ra khi và chỉ khi x = y .

362
Cty TNHH MTV DVVH Khang Việt

6. Kỹ thuật sử dụng định lý Lagrange


Ví dụ 1. Cho hai số thực dương a, b thỏa mãn a < b. Chứng minh rằng:
b−a b b−a
< ln <
b a a
Lời giải
1
Xét hàm số f ( x=
) ln x ⇒ f '( x=
) , ∀x ∈ (0; +∞).
x
f (b) − f (a )
Theo định lí Lagrange luôn tồn tại c ∈ (a; b) sao cho f '(c) = .
b−a
1 ln b − ln a a −b b

n
Vì vậy= ⇔ = ln
c b−a c a

.v
1 1 1 b−a b b−a
Mặt khác 0 < a < b < c ⇒ < < ⇒ < ln < (đpcm).

om
b c a b a a
7. Kết hợp sử dụng các bất đẳng thức cơ bản AM – GM ; Cauchy – Schwarz
Để việc khảo sát hàm số thuận tiện và đơn giản trước khi chứng minh bất đẳng
thức hoặc tìm Min, Max ta đánh giá qua các đại lượng trung bình của hai biến số
.c
x+ y x2 + y 2
ok
( , xy , ) hoặc kết hợp sử dụng một số bất đẳng thức phụ quen biết.
2 2
Sau đây là một số đánh giá hay được sử dụng:
bo

1 1
x 2 + y 2 ≥ ( x + y ) ; x3 + y 3 ≥ ( x + y ) .
2 3
+
2 4
et

+ e x ≥ x + 1, ∀x ≥ 0;ln ( x + 1) ≤ x, ∀x ≥ 0 .

( a + x )2 + ( b + y )2 .
vi

+ x2 + y 2 + a 2 + b2 ≥
Ví dụ 1. (TSĐH Khối B 2006) Cho x,y là hai số thực thay đổi.
ng

Tìm giá trị nhỏ nhất của biểu thức P = ( x − 1)2 + y 2 + ( x + 1)2 + y 2 + y−2 .
a

Lời giải
kh

Trên mặt phẳng tọa độ Oxy, xét các điểm M(1-x; y) và N(1+x; y).
Ta có OM + ON ≥ MN, suy ra ( x − 1)2 + y 2 + ( x + 1)2 + y 2 ≥ 4 + 4 y2 .

Đẳng thức xảy ra khi x = 0, ta được P ≥ 2 1 + y 2 + y − 2 .

Xét hàm số f ( y )= 2 1 + y 2 + y − 2 .

*) Với y ≥ 2 thì f ( y )= 2 1 + y 2 + y − 2 là hàm đồng biến.


2y
*) Với y < 2 thì f ( y )= 2 1 + y 2 + 2 − y có =
f ′( y ) −1 .
1 + y2
363
Khám phá tư duy Kỹ thuật giải bất ĐT Bài toán Max – Min – Đặng Thành Nam

1
Khi đó f ′ ( y ) = 0 ⇔ y = .
3
 1 
Lập bảng biến thiên của hàm số f(y) ta được min P =2 + 3 ⇔ ( x; y ) =0;  .
 3
Chú ý. Ta có thể sử dụng bất đẳng thức Mincopski:

( x − 1)2 + y 2 + ( x + 1)2 + y 2 ≥ (1 − x + x + 1)2 + 4 y 2= 2 y2 + 1 .


Ví dụ 2. (TSĐH Khối B 2011) Cho a,b là số thực dương thỏa mãn điều kiện
( )
2 a 2 + b 2 + ab = ( a + b )( ab + 2 ) .

n
 a 3 b3   a 2 b 2 
Tìm giá trị nhỏ nhất của biểu thức P = 4  3 + 3  − 9  2 + 2  .

.v
b a   b a 

om
Lời giải
a b
Nhận xét. P có thể biểu diễn theo t= + nên ta nghĩ đến việc đánh giá
b a .c
a b
t= + dựa vào điều kiện bài toán.
b a
ok
a b
Đặt t= + , khi đó
b a
bo

  a b 3  a b   a b  2 
P = 4  +  − 3  +   − 9  +  − 2  = 4t 3 − 9t 2 − 12t + 18 .
 b a   b a    b a  
et

Xuất phát từ điều kiện ta có:


vi

a b 1 1 1 1  a b
2  +  + 1=  +  ( ab + 2 ) ≥ 2 2ab  + = 2 2  + .
a 
ng

b a a b a b  b
5
Suy ra 2t + 1 ≥ 2 2. t + 2 ⇔ t ≥ .
a

2
kh

5 
Xét hàm số f (t ) = 4t 3 − 9t 2 − 12t + 18 trên  ; +∞  ta có:
 2 
5 5 
f '(=
t ) 12t 2 − 18t − 12 > 0, ∀t ≥ nên f(t) đồng biến trên  2 ; +∞  suy ra
2  
5 23
P=
f (t ) ≥ f   =
− .
2 4
23
Vậy giá trị nhỏ nhất của P bằng − đạt tại=
a 2,=
b 1 hoặc =
a 1,=
b 2.
4

364
Cty TNHH MTV DVVH Khang Việt

Bài tập tương tự


1) Cho x,y là 2 số thực dương thỏa mãn điều kiện
( )
6 x 2 + y 2 + 20 xy = 5 ( x + y )( xy + 3) .
Tìm giá trị nhỏ nhất của biểu thức
 x4 y 4   x3 y 3   x2 y 2 
P = 9  4 + 4  − 16  3 + 3  + 25  2 + 2  .
y x  y x  y x 
  
2) Cho a,b là hai số thực dương thỏa mãn điều kiện

(a ) ( )( )
2
2
+ 2b 2 + 3a 2b 2 =2 a 2 + b 2 a 2 + 2b 2 .

n
Tìm giá trị nhỏ nhất của biểu thức

.v
( a + b )2 + 2a 2 + 5b 2  ( a − b )2 + 2a 2 + 5b 2 
a 3 + b3 8b3
=
P + 3 +  .
( )

om
b3 a ab a + 2b
2 2

Ví dụ 3. Cho a,b là các số thực dương thỏa mãn điều kiện


a+b+2 ( a + 2 )( b + 2 ) =
.c 12 .

a3 b3
ok
48
Tìm giá trị nhỏ nhất của biểu thức P = + + .
b+2 a+2 a+b
Lời giải
bo

2
a+2+b+2
( a + 2 )( b + 2 ) ≤ 
1
Theo giả thiết ta có: (12 − a − b )2 =  .
4  2 
et

Đặt t = a + b, ( 0 < t < 12 ) suy ra (12 − t ) ≤ ( t + 4 ) ⇔ t + 4 ≥ 12 − t ⇔ t ≥ 4 .


2 2
vi

Khi đó P
=
a 3 ( a + 2 ) + b3 ( b + 2 )
= +
48 a 4 + b 4 + 2 a 3 + b3
+
48 ( )
ng

.
( a + 2 )( b + 2 ) a+b ( a + 2 )( b + 2 ) a+b
1 1
Sử dụng bất đẳng thức AM-GM ta có a 4 + b 4 ≥ ( a + b )4 ; a3 + b3 ≥ ( a + b )3 .
a

8 4
kh

1 1
( a + b )4 + 2. ( a + b )3 48
Suy ra P ≥ 8 4 +
( a + 2 )( b + 2 ) a+b

=
( a + b )4 + 4 ( a + b )3 + 48
=
t 4 + 4t 3
+
48

8.
(12 − a − b )
2 a+b 2 (12 − t )
2 t
4

365
Khám phá tư duy Kỹ thuật giải bất ĐT Bài toán Max – Min – Đặng Thành Nam

t 4 + 4t 3 48
=
Nhận xét. Nếu ta xét hàm số f (t ) + này rõ ràng đây là một hàm số
2 (12 − t )
2 t
phức tạp nếu đạo hàm lên không xác định tính dương âm của đạo hàm trên khoảng
[ 4;+∞ ) . Do đó ta cần đơn giản hàm số đó đi bằng cách đánh giá
1 1
2 (12 − t ) ≤ 2 (12 − 4 ) = 128 ⇒
2 2
≥ .
2 (12 − t )
2 128

t 4 + 4t 3 48
Vậy ta có đánh giá P ≥ f (t=
) + .
128 t

n
t 4 + 4t 3 48 t 5 + 3t 4 − 1536
=
Xét hàm số f (t ) + với t ≥ =
4 ta có f '(t ) > 0, ∀t ≥ 4 .

.v
128 t 32t 2
Do đó f(t) đồng biến trên [ 4;+∞ ) suy ra P ≥ f (t ) ≥ f (4) =
16 . Đẳng thức xảy ra

om
khi và chỉ khi a= b= 2 .
Vậy giá trị nhỏ nhất của P bằng 16 đạt tại a= b= 2 .

B. BÀI TOÁN CHỌN LỌC


.c
ok
Bài 1. Cho x,y là hai số thực thỏa mãn điều kiện x + y −=
1 2x − 4 + y + 1 .
Tìm giá trị lớn nhất và giá trị nhỏ nhất của biểu thức
bo

1
P = ( x + y) − 9 − x − y +
2
.
x+ y
et

Lời giải
Nhận xét. P là một biểu thức biểu diễn theo t= x + y nên ta tìm cách đưa về khảo
vi

sát hàm với t= x + y nhưng trước tiên phải tìm miền giá trị của t thỏa mãn điều
ng

kiện bài toán.


Đặt t= x + y xuất phát từ điều kiện và bất đẳng thức C-S ta có:
a

x + y −=
1 2 ( x − 2) + y + 1 ≤ ( 2 + 1)( x − 2 + y + 1=) 3 ( x + y − 1) .
kh

t ≥ 1
⇒ ⇔1≤ t ≤ 4.
( t − 1) ≤ 3 ( t − 1)
2

1
Khi đó P = f (t ) = t 2 − 9 − t + .
t
1 1 1 4t t − 1
Ta có f '(t )= 2t + − = + > 0, ∀t ∈ [1;4] .
2 9 − t 2t t 2 9 − t 2t t
33
Suy ra max f (t ) =
f(4) = − 5; min f (t ) =
f(1) =
2−2 2.
t∈[1;4] 2 t∈[1;4]

366
Cty TNHH MTV DVVH Khang Việt

33
Vậy giá trị lớn nhất của P bằng − 5 đạt tại=
x 4,=
y 0 . Giá trị nhỏ nhất
2
của P bằng 2 − 2 2 đạt tại x = 2, y = −1 .
Bài 2. Cho x,y là hai số thực dương thỏa mãn điều kiện xy + x + y =3.
xy 3x 3y
Tìm giá trị nhỏ nhất của biểu thức P = x 2 + y 2 − − − .
x + y y +1 x +1
Lời giải

Viết lại P =( x + y )
2
− 2 xy −
xy
− 3.
( x + y ) − 2 xy + 3 ( x + y ) .
2

x+ y xy + x + y + 1

n
Đặt t = x + y ⇒ xy = 3 − t . Ta có ( x + y ) ≥ 4 xy ⇒ t 2 ≥ 4 ( 3 − t ) ←
t >0
2
→t ≥ 2 .

.v
12 3
Khi đó P = t 2 − t − − .

om
t 2
12 3
Xét hàm số f (t ) = t 2 − t − − trên [ 2;+∞ ) ta có
t 2 .c
12 t 2 ( 2t − 1) + 12
f '(t) = 2 t − 1 + = > 0, ∀t ≥ 2 .
ok
t2 t2
3
Do đó f(t) là hàm đồng biến trên [ 2;+∞ ) suy ra P =≥
f (t ) f (2) =
− .
bo

2
3
Vậy giá trị nhỏ nhất của P bằng − đạt tại x= y= 1 .
et

2
Bài 3. Cho x,y là hai số thực thỏa mãn điều kiện
vi

(x )
2
2
+ y 2 + 1 + 3x 2 y 2 + =
1 4 x2 + 5 y 2 .
ng

x 2 + 2 y 2 − 3x 2 y 2
Tìm giá trị lớn nhất, giá trị nhỏ nhất của biểu thức P = .
x2 + y 2 + 1
a

Lời giải
kh

Nhận xét. Để ý đến đại lượng x + y 2 + 1 từ điều kiện và biểu thức của P ta nghĩ
2

đến việc biến đổi P theo x 2 + y 2 + 1 dựa vào điều kiện bài toán.

( ) ( )
2
Từ giả thiết ta có: x 2 + y 2 + 1 + 3 x 2 y 2 + 1= 4 x 2 + y 2 + 1 + y 2 − 4

( ) ( )
2
⇔ x 2 + y 2 + 1 − 4 x 2 + y 2 + 1 + 5 = y 2 − 3x 2 y 2 .

(=
x + y + 1) + ( y − 3 x y − 1) ( x ) ( )
2
2 2 2 2 2 2
+ y 2 + 1 − 3 x2 + y 2 + 1 + 4
Do đó P .
x2 + y 2 + 1 x2 + y 2 + 1

367
Khám phá tư duy Kỹ thuật giải bất ĐT Bài toán Max – Min – Đặng Thành Nam

t 2 − 3t + 4
Đặt t = x 2 + y 2 + 1 khi đó P = . Ta cần tìm miền giá trị của t.
t
Xuất phát từ điều kiện ta có:

(x ) ( )
2
2
+ y 2 + 1 + 3 x 2 y 2 + 1= 5 x 2 + y 2 + 1 − x 2 − 5

( ) ( )
2
⇔ x2 + y 2 + 1 − 5 x2 + y 2 + 1 + 6 =
− x 2 − 3x 2 y 2 ≤ 0

⇒ (x 2
)(
+ y 2 + 1 − 2 x 2 + y 2 + 1 − 3 ≤ 0 ⇒ t ∈ [ 2;3] )
t 2 − 3t + 4
Vậy xét hàm số f (t ) = liên tục trên [ 2;3] ta được:

n
t

.v
t2 − 4
f=
'(t) ≥ 0, ∀t ∈ [ 2;3] . Vậy f(t) là hàm đồng biến trên [ 2;3] suy ra
t2

om
4 4
hay 1 ≤ P ≤ .
1 = f (2) ≤ f (t ) ≤ f (3) =
3 3
Vậy giá trị nhỏ nhất của P bằng 1 đạt tại x = 0, y = ±1 và giá trị lớn nhất của P
.c
4
bằng đạt tại x = 0, y = ±2 .
ok
3
Bài 4. Xét phương trình ax3 − x 2 + bx − 1 =0 với a, b là các số thực, a ≠ 0 , a ≠ b
bo

sao cho các nghiệm đều là số thực dương. Tìm giá trị nhỏ nhất của biểu thức:
5a 2 − 3ab + 2
P= .
a2 (b − a )
et

Lời giải
vi

Gọi u, v, s là ba nghiệm thực dương của đa thức ax3 − x 2 + bx − 1 .


ng

1 b 1
Theo định lý Viete ta có u + v +=
s ; uv + vs + su
= =
; uvs (1) .
a a a
a

Từ đó suy ra a > 0, b > 0.


kh

1
Đặt c = . Áp dụng BĐT AM – GM cho ba số dương ta có
a
c = uvs = u + v + s ≥ 3 3 uvs = 3 3 c ⇒ c3 ≥ 27c ⇒ c ≥ 3 3 ( 2) .
1
Mặt khác ( u + v + s ) − 3 ( uv + vs + su=) 2

( u − v )2 + ( v − s )2 + ( s − u )2  ≥ 0 .
2
Do đó c 2 = ( u + v + s )2 ≥ 3 ( uv + vs + su ) = 3bc ( 3)
Từ (1), (2) và (3) ta có

368
Cty TNHH MTV DVVH Khang Việt

( )
b 1
5 − 3 + 2 3 c 5 − 3bc + 2c 2
5a − 3ab + 2 1 a
2
a
=P = .=
a (b − a )
2 a b
−1 bc − 1
a


(
c 5 − c 2 + 2c 2 )=
5c ( c + 5 )
2

( 4)
c2 c2 − 3
−1
3

Xét hàm số f ( c ) =
(
5c c 2 + 5 ) với c ≥ 3 3. Ta được f ( c ) ≥ 12 3 .
c2 − 3

n
Dấu bằng xảy ra khi c = 3 3. Suy ra P ≥ 12 3 .

.v
1
Đẳng thức xảy ra khi u= v= s= 3 , tức=
là a = ,b 3.

om
3 3
Vậy min P = 12 3 .
Bài 5. Cho a,b,c là các số thực không âm thỏa mãn a > b > c và .c
3ab + 5bc + 7ca ≤ 9 .
32 1 1
ok
Tìm giá trị nhỏ nhất của biểu thức P = + + .
( a − b ) ( b − c ) ( c − a )4
4 4
bo

Lời giải
Theo giả thiết ta có a > b > c ≥ 0 ⇒ 9 ≥ 7ca + 5bc + 3ab ≥ 3ab ⇒ ab ≤ 3 .
Khi đó:
et

32 1 1 a 2b 2  32 1 1
P≥ + + ≥  + + 
vi

( a − b ) 4
a 4
b 4 9
(
 a − b ) 4
a 4
b 4

ng

 
   2 
1 32a 2b 2 a 2 b2  1  a b
+  +  − 2
32
= =
+ +
9  a 2 − 2ab + b 2 2 b 2 a 2  9   a b
( ) b a 
a

2
    + − 2  
  b a  
kh

a b 1  32 
Đặt t =+ , ( t > 2 ) . Do a > b, khi đó P=
≥ f (t )  + t 2
− 2 
b a 9  ( t − 2 )2 

1  32 
=
Xét hàm số f (t )  + t 2
− 2  với t > 2 ta có:
9  ( t − 2 )2 

1 64 
 ; f '(t ) = 0 ⇔ 2t ( t − 2 ) = 64 ⇔ t = 4 .
3
f '(t ) = − + 2t
9  (t − 2)3


369
Khám phá tư duy Kỹ thuật giải bất ĐT Bài toán Max – Min – Đặng Thành Nam

Ta có f’(t) đổi dấu từ âm sang dương khi đi qua t = 4 nên tại t = 4 thì f(t) đạt
10
cực tiểu hay P ≥ f (t ) ≥ f (4) = .
9
 =
c = 0 a 6+3 3


Đẳng thức xảy ra khi và chỉ khi: ab = 3

⇔ b = 2 + 3 2 3 − 3 . ( )
a b 
 + = 4 c = 0
b a 
Vậy giá trị nhỏ nhất của P bằng 10/9 đạt tại
(
a = 6 + 3 3 , b = 2 + 3 2 3 − 3 , c =0 . )

n
.v
C. BÀI TẬP RÈN LUYỆN

om
Bài 1. (TSCĐ 2009) Cho a và b là hai số thực thỏa mãn 0 < a < b < 1 . Chứng minh
rằng a 2 ln b − b 2 ln a > ln a − ln b .
.c
Bài 2. Cho x,y là các số thực không âm thỏa mãn điều kiện x 2 − xy + y 2 =
1.
2 xy
ok
Tìm giá trị nhỏ nhất của biểu thức P = x + y − .
( x + 1)( y + 1)
Bài 3. Cho x,y là hai số thực dương thỏa mãn điều kiện xy + x + y =3.
bo

4x 4y
Tìm giá trị nhỏ nhất của biểu thức P= + + 2 xy − 7 − 3 xy .
y +1 x +1
et

Bài 4. Cho x, y > 0 thỏa mãn điều kiện x + 2 y − xy =


0.
vi

x2 y2
Tìm giá trị nhỏ nhất của biểu thức
= P + .
4 + 8y 1+ x
ng

Bài 5. Cho x,y là hai số thực dương có tổng bằng 1. Chứng minh rằng
x y
a

+ ≥ 2.
1− x 1− y
kh

Bài 6. Cho x,y là hai số thực dương thay đổi thỏa mãn 3 x 2 + 8 y 3 =
20 .
4 4 1
Tìm giá trị nhỏ nhất của biểu thức P = + + .
x 2
y 2
( x − y )2
Bài 7. Cho x,y là hai số thực dương phân biệt thỏa mãn xy ≤ 4 .
2 2 3
Tìm giá trị nhỏ nhất của biểu thức P = + + .
x 4
y 4
( x − y )4
Bài 8. Cho x,y là hai số thực dương thỏa mãn xy ≤ 3 .

370
Cty TNHH MTV DVVH Khang Việt

1 1 1
Tìm giá trị nhỏ nhất của biểu thức=P + + .
( x − y ) x y4 4 4

Bài 9. Cho x,y là hai số thực dương thỏa mãn 8 ( x + 2 y ) xy =


5 x + 2 y ( 5 + 16 x ) .
Tìm giá trị lớn nhất và nhỏ nhất của biểu thức

P =x 2 + 4 y 2 +
(1 + 2 xy )2 + 7 xy − 6 .
xy
Bài 10. Cho x,y là 2 số thực dương thỏa mãn điều kiện
( )
6 x 2 + y 2 + 20 xy = 5 ( x + y )( xy + 3) .

n
Tìm giá trị nhỏ nhất của biểu thức

.v
 x4 y 4   x3 y 3   x2 y 2 
P = 9  4 + 4  − 16  3 + 3  + 25  2 + 2  .
y x  y x  y x 

om
  
Bài 11. Cho a,b là hai số thực dương thỏa mãn điều kiện

(a ) ( )( )
2
2
+ 2b 2 + 3a 2b 2 =2 a 2 + b 2 a 2 + 2b 2 .
.c
Tìm giá trị nhỏ nhất của biểu thức
ok
( a + b )2 + 2a 2 + 5b 2  ( a − b )2 + 2a 2 + 5b 2 
a 3 + b3 8b3   .
=
P + 3 +
( )
bo

b3 a ab a + 2b
2 2

Bài 12. Cho x,y là hai số thực thỏa mãn x + y =


1.
et

Tìm giá trị lớn nhất của biểu thức P =x3 + 1 y 3 + 1 . ( )( )


vi

Bài 13. Cho x,y là hai số thực thỏa mãn x 2 + y 2 =x + y .


ng

Tìm giá trị lớn nhất và nhỏ nhất của biểu thức P = x3 + y 3 + x 2 y + xy 2 .
Bài 14. Cho x,y là hai số thực thỏa mãn x3 + y 3 =
2.
a

Tìm giá trị nhỏ nhất của biểu thức =


P x2 + y 2 .
kh

Bài 15. Cho x,y là hai số thực dương thỏa mãn x ≥ 1, y ≥ 1 và 3 ( x + y ) =


4 xy .
 1 1 
Tìm giá trị nhỏ nhất và lớn nhất của biểu thức P = x3 + y 3 + 3  2 + 2  .
x y 

Bài 16. Cho x,y là hai số thực khác 0 và thỏa mãn xy ( x + y ) = x 2 + y 2 − x − y + 2 .


1 1
Tìm giá trị lớn nhất của biểu thức P= + .
x y

371
Khám phá tư duy Kỹ thuật giải bất ĐT Bài toán Max – Min – Đặng Thành Nam

Bài 17. Cho x,y là hai số thực thỏa mãn x 2 + y 2 + xy =


3.
Tìm giá trị lớn nhất và nhỏ nhất của biểu thức P = x3 + y 3 − 3 x − 3 y .
Bài 18. Cho x,y là hai số thực thỏa mãn x 2 + y 2 =
2.
Tìm giá trị lớn nhất và giá trị nhỏ nhất của biểu thức P = 2 x3 + y 3 − 3 xy . ( )
Bài 19. Cho x,y là 2 số thực không âm thỏa mãn x + y =
1.
x 2 y + xy 2
Tìm giá trị lớn nhất của biểu thức P = .
x 2 + y 2 − xy

( )

n
Bài 20. Cho x,y là hai số thực thỏa mãn 2 x 2 + y 2 − xy =
1.

.v
(
Tìm giá trị lớn nhất và nhỏ nhất của biểu thức P = 7 x 4 + y 4 + 4 x 2 y 2 . )

om
Bài 21. Cho a,b là hai số thực dương thỏa mãn a + b + 1 = 3ab .
3a 3b 1 1
Tìm giá trị lớn nhất của biểu thức= P + − 2− 2.
( a + 1) b ( b + 1) a a b
.c
 1 1
Bài 22. Cho các số thực x, y ∈ 0;  và x + y ≤ .
ok
 2  2
x+ y
Tìm giá trị lớn nhất của biểu thức P =
bo

.
1− x + 1− y
Bài 23. Cho a,b là hai số thực dương thỏa mãn a + b =6ab .
et

3a + 1 3b + 1
Tìm giá trị nhỏ nhất của biểu thức =P + 2 + ( 3a + b )( 3b + a ) .
9b + 1 9a + 1
2
vi

Bài 24. Cho x,y là hai số thực thỏa mãn điều kiện x 2 − y 2 + y 2 − x 2 =
2.
ng

Tìm giá trị lớn nhất của biểu thức P = ( x + y ) − 12 ( x − 1)( y − 1) + xy .


3
a

Bài 25. Cho a,b là các số thực không âm thỏa mãn điều kiện
kh

a 2 + b 2 + ab ≤ 1 − a − b .
1 1
=
Tìm giá trị lớn nhất của biểu thức P + + 1 + ab .
1 + a2 1 + b2
1
Bài 26. Cho x,y là hai số thực dương thỏa mãn x 4 + y 4 + = xy + 2 .
xy
2 2 3
Tìm giá trị lớn nhất của biểu thức P = + − .
1+ x 2
1+ y 2 1 + 2 xy

372
Cty TNHH MTV DVVH Khang Việt

Bài 27. Cho x,y là hai số thực thuộc khoảng ( 0;1) và

(x 3
+ y3 ) ( x + y )= xy ( x − 1)( y − 1) .

1 1
+ 5 xy − ( x + y ) .
2
Tìm giá trị lớn nhất của biểu thức=P +
1+ x 2
1+ y 2

Bài 28. (TSĐH Khối D 2009) Cho x,y là hai số thực không âm thỏa mãn điều kiện
x+ y= 1.
Tìm giá trị lớn nhất và nhỏ nhất của biểu thức
P= ( 4x 2
)(
+ 3 y 4 y 2 + 3 x + 25 xy .)

n
Bài 29. (TSĐH Khối D 2012) Cho x,y là hai số thực thỏa mãn điều kiện

.v
( x − 4 )2 + ( y − 4 )2 + 2 xy ≤ 32 .

om
Tìm giá trị nhỏ nhất của biểu thức P = x3 + y 3 + 3 ( xy − 1)( x + y − 2 ) .
Bài 30. (TSĐH Khối B 2009) Cho x,y là hai số thực thỏa mãn điều kiện
( x + y )2 + 4 xy ≥ 2 .
.c
( ) ( )
ok
Tìm giá trị nhỏ nhất của biểu thức P= 3 x 4 + y 4 + x 2 y 2 − 2 x 2 + y 2 + 1 .

Bài 31. Cho x,y là 2 số thực không âm thỏa mãn xy ( x + y ) =


2.
bo

( ) (
Chứng minh rằng 3 x3 + y 3 − 2 x 2 + y 2 + x + y + 4 ≥ 0 . )
Bài 32. Cho x,y là hai số thực dương thỏa mãn
et

( xy + 1) ( 9 ) (
xy − 2 xy = 7 x 2 + y 2 − 2 xy + 2 . )
vi

1 1
Tìm giá trị lớn nhất và nhỏ nhất của biểu thức P = xy + xy + +
ng

.
xy xy
Bài 33. Cho x,y là hai số thực thỏa mãn 2 x 2 + y 2 =xy + 1 .( )
a
kh

x4 + y 4
Tìm giá trị nhỏ nhất và lớn nhất của biểu thức P = .
2 xy + 1
Bài 34. Cho x,y là hai số thực thỏa mãn x 2 − xy + y 2 =
1.
x4 + y 4 + 1
Tìm giá trị lớn nhất và nhỏ nhất của biểu thức P = .
x2 + y 2 + 1
Bài 35. Cho x,y là 2 số thực thỏa mãn x 4 + y 4 =
2 xy .
Tìm giá trị lớn nhất và nhỏ nhất của biểu thức

373
Khám phá tư duy Kỹ thuật giải bất ĐT Bài toán Max – Min – Đặng Thành Nam

( )
P = xy + 3 x 2 y 2 + 2 xy x 2 + y 2 − ( x + y ) .
2

Bài 36. Cho x 2 + y 2 > 0 và x 4 + y 4 += (


2 2 x 2 + y 2 + xy . )
4
Tìm giá trị lớn nhất và nhỏ nhất của biểu thức =
P 2 x 2 y 2 + xy − .
x + y2
2

Bài 37. Cho x,y là các số thực dương. Tìm giá trị nhỏ nhất của biểu thức
x + y + x2 y 2 13xy
=P + .
xy 1 + xy (1 + x + y )

Bài 38. Cho x,y là hai số thực dương thỏa mãn x 4 − x 2 y 2 + y 4 =

n
1.

.v
x +1 y +1 2 xy
Tìm giá trị nhỏ nhất của biểu thức P = + − .
3 x + 1 3 y + 1 xy ( x + y ) + 2

om
Bài 39. Cho x,y là hai số thực dương thỏa mãn xy + x + y =3.
3x 3y xy
Tìm giá trị lớn nhất của biểu thức P= + + .c − x2 − y 2 .
y +1 x +1 x + y
x +1 + y +1 =
ok
Bài 40. Cho x,y là hai số thực thỏa mãn 4.
64
= xy +
Tìm giá trị lớn nhất và nhỏ nhất của biểu thức P .
4− x− y
bo

Bài 41. Cho các số thực x,y thỏa mãn x + y= x −1 + 2 y + 2 .


et

Tìm giá trị lớn nhất và nhỏ nhất của biểu thức
P = x 2 + y 2 + 2 ( x + 1)( y + 1) + 8 4 − x − y .
vi

 x −1 y −1
Bài 42. Cho x,y là hai số thực dương thỏa mãn x + y=
ng

+ 2 3 + .
 y x 

2 x y2 3 
2
Tìm giá trị nhỏ nhất của biểu thức P =( x − y )  4 + 4 −  .
a

y xy 
 x
kh

Bài 43. Cho x > 1, y > 1 . Tìm giá trị nhỏ nhất của biểu thức

=P
x3 + y 3 − x 2 − y 2
( x − 1)( y − 1)
(
+ 2 x 2 + y 2 − 16 xy . )
Bài 44. Cho x,y là hai số thực dương. Tìm giá trị lớn nhất của biểu thức
x4 + y 4 x2 + y 2 5 xy
P= + + .
( x + y) 4
( x + y) 2 x+ y

374
Cty TNHH MTV DVVH Khang Việt

Bài 45. Cho x,y là hai số thực không âm. Tìm giá trị nhỏ nhất của biểu thức
2 4 ( 2 x + 1) ( 2 y + 1)
3 3
P= 3 4 e ( 3x
+e 3y
)− 3
.
E. HƯỚNG DẪN GIẢI – ĐÁP SỐ
Bài 1. Bất đẳng thức tương đương với:

( a + 1) ln b > (b + 1) ln a ⇔ bln+b1 > aln+a1 .


2 2
2 2

ln x
Với 0 < a < b < 1 vậy ta cần chứng minh hàm số f ( x) = đồng biến trên
x2 + 1
khoảng ( 0;1) .

n
( )

.v
1 2
x + 1 − 2 x ln x
x 2 + x − 2 x ln x
Ta có f '( x) x
= = > 0, ∀x ∈ ( 0;1) .
( ) ( )

om
2 2
x +1
2
x x +1
2

Vậy f(x) đồng biến trên khoảng ( 0;1) .

Do đó 0 < a < b < 1 ta có f (b) > f (a ) ⇔


ln b
.c >
ln a
.
b +1
2
a2 + 1
ok
Bài toán được chứng minh.
Bài 2. Theo giả thiết ta có
bo

( x + y ) − 1  x + y 2
2
( x + y) 2
− 3xy = 1 ⇔ xy = ≤ ⇒1≤ x + y ≤ 2 .
 
3  2 
et

Khi đó
( x + y )2 − 1
vi

P=x+ y−
2.
3 =x+ y−
2 3
.
( x + y )2 − 1 .
ng

x + y +1+
( x + y) −1
2 3
( x + y ) + 3( x + y ) + 2
2

3
a

2 3 t2 −1
Đặt t = x + y,1 ≤ t ≤ 2 ta có P = t − . .
kh

3 t 2 + 3t + 2
Ta chứng minh P ≥ 1 , thật vậy ta cần chứng minh.
2 3 t2 −1
t− . ≥1
3 t 2 + 3t + 2
⇔ ( t − 1) 3 t 2 + 3t + 2 − 2 3 ( t + 1)  ≥ 0
 
( t − 1) ( −3t 2 + 3t + 6 )
≥ 0 ⇔ ( t − 1) ( 2 − t ) ≥ 0
2

3 t 2 + 3t + 2 + 2 3 ( t + 1)

375
Khám phá tư duy Kỹ thuật giải bất ĐT Bài toán Max – Min – Đặng Thành Nam

Bất đẳng thức luôn đúng. Đẳng thức xảy ra khi và chỉ khi t = 1 hoặc t = 2 tương
ưng với ( x; y ) = ( 0;1) ; (1;0 ) ; (1;1) .
Vậy giá trị nhỏ nhất của P bằng 1.
Bài 3. Ta có:
4 x ( x + 1) + 4 y ( y + 1)
P + 2 xy − 7 − 3 xy
( x + 1)( y + 1)
4 ( x2 + y 2 ) + 4 ( x + y )
+ 2 xy − 7 − 3 xy
xy + x + y + 1
4 ( x + y ) + 4 ( x + y ) − 8 xy
2

n
+ 2 xy − 7 − 3 xy

.v
4
=( x + y ) + x + y − 7 − 3 xy ≥ 22 + 2 − 7 − 3.1 =4
2

om
2
1 1  x + 2y 
Bài 4. Theo giả thiết ta có : x + 2 y = xy = .x.2 y ≤   ⇒ x + 2y ≥ 8
2 2 2 
Theo bất đẳng thức C-S ta có :
.c
(2y) ≥ ( x + 2y) = ( x + 2y) .
2 2 2
ok
x2 y2 x2
+ = +
4 + 8 y 1 + x 4 + 8 y 4 + 4x 4 + 8 y + 4 + 4x 8 + 4( x + 2 y )
bo

t2
Vậy đặt t= x + 2 y và xét hàm số
= f (t ) ,t ≥ 8 .
8 + 4t
et

4t 2 + 8t
=
Ta có f '(t ) > 0 với t ≥ 8 .
(8 + 4t )2
vi

8
Suy ra min f=
(t ) f=
(8)
ng

t∈[8; +∞ ) 5
8
Từ đó suy ra giá trị nhỏ nhất của P bằng , khi=
x 4;=
y 2.
a

5
kh

Bài 5. Ta có y = 1 – x nên BĐT cần chứng minh là


x 1− x
+ ≥ 2, ∀x ∈ (0; 1) .
1− x x
x 1− x
Xét =
f ( x) + , ∀x ∈ (0; 1). Ta có
1− x x

376
Cty TNHH MTV DVVH Khang Việt

1 2− x x + 1  1  1 + (1 − x) 1+ x 
f ′( x )
=  =
−   − 
2  (1 − x ) 1 − x x x  2  (1 − x ) 1 − x x x 

=
1
2
(
h 1− x − h x 
 ) ( )
1+ t2 1 1
trong đó h ( t=
) = 3+ nghịch biến trên ( 0;+∞ ) , nên
t3 t t
f ′( x ) = 0 ⇔ h ( ) ( x) ⇔
1− x = h
1
1− x = x ⇔ x =

n
2

.v
Và f ′ ( x ) > 0 ⇔ h ( ) ( )
1− x > h
1 
x ⇔ 1 − x < x ⇔ x ∈  ;1
2 

om
Do vậy: f ′ ( x ) < 0 ⇔ x ∈  0; 
 1
 2 .c
1
f ( x)
Vậy min = f=
  2 . Suy ra f ( x ) ≥ 2, ∀x ∈ ( 0;1) .
ok
( 0;1) 2
Chú ý. Ta có thể đánh giá đơn giản bằng C –S .
bo

Bài 6. Ý tưởng là đưa P về dạng hàm số có nhân tử chung đặt là t. Muốn vậy ta tạo
cho P dạng đồng bậc nên thêm vào P một lượng xy.
 1 1   y x
et

xy 1
Ta có: P.xy = 4 xy  + 2 + = 4 +  + .
x
2
y  ( x − y) 2
 x y x
−2+
y
vi

y x
ng

x y 1
Đặt t =+ , ( t > 2 ) đưa về P.xy= 4t + .
y x t −2
Muốn tìm giá trị nhỏ nhất của P vậy trước tiên ta tìm giá trị lớn nhất của xy.
a

1
kh

Khảo sát hàm số f (t=


) 4t + với t > 2 ta được:
t −2
1 5
f '(t ) = 4 − ; f '(t ) = 0 ⇔ t = ⇒ x = 2 y .
(t − 2)
2
2
Vậy khi đó x = 2 y ⇒ 12 y 2 + 8 y 3 = 20 ⇔ y = 1, x = 2 tức P đạt min tại
=
x 2,=
y 1 . Đây là cơ sở cho ta đánh giá lượng lớn nhất của xy.
Xuất phát từ điều kiện ta có:

377
Khám phá tư duy Kỹ thuật giải bất ĐT Bài toán Max – Min – Đặng Thành Nam

20= 3 x 2 + 8 y 3= 3 x 2 + 4 ( y 3 + y 3 + 1) − 4 ≥ 3 x 2 + 12 y 2 − 4 .
⇒ 8 = x 2 + 4 y 2 ≥ 4 xy ⇔ xy ≤ 2 .
1 5 1
Do đó P ≥ f (t ) ≥
f  = 6.
2 2 2
Vậy giá trị nhỏ nhất của P bằng 6 đạt tại=
x 2,=
y 1.
Bài 10. Ta có
 x y x
4
y
2
  x y
3
x y  x y
2

P = 9  +  − 4 +  + 2  − 16  +  − 3 +   + 25  +  − 50
 y x y x   y x y x   y x

n
Bài 11. Từ điều kiện và sử dụng bất đẳng thức AM-GM ta có:

.v
(a + 2b 2 ) + 3a 2b 2 = 2 ( a 2 + b 2 )( a 2 + 2b 2 ) ≥ 4ab ( a 2 + 2b 2 )
2 2

om
2
 a 2b   a 2b  a 2b
⇒  +  + 3 ≥ 4 +  ⇔ + ≥3
b a  b a  b a .c
a 2b 4
Khi đó đặt t= + và ta có P = f (t ) = t 3 + 3t − + 1 .
ok
b a t
4
Xét hàm số f (t ) = t 3 + 3t − + 1 trên [3; +∞ ) ta có
bo

t
4
t ) 3t 2 + 2 + 3 > 0, ∀t ∈ [3; +∞ ) nên f(t) đồng biến trên [3; +∞ ) .
f '(=
et

t
97
Do đó P = f (t ) ≥ f(3) =
vi

.
3
ng

97
Vậy giá trị nhỏ nhất của P bằng đạt tại a= b= 1 .
3
Bài 25. Xuất phát từ điều kiện ta có:
a

1
(a + b)
kh

+ a + b + ab ≤ 1 ⇒ 1 ≥ ab + 2 ab + 4ab ⇒ 0 ≤ ab ≤ < 1.
2

9
1 1 2
Áp dụng bất đẳng thức cơ bản ta có: + ≤ .
1+ a 2
1+ b 2
1 + ab
2
Suy ra P ≤ + 1 + ab .
1 + ab
 10  2 2
Đặt t = 1 + ab , t ∈ 1;  ta có P ≤ + t .
 3  t

378
Cty TNHH MTV DVVH Khang Việt

(a + b)
3

Bài 45. HD: Sử dụng bất đẳng thức quen thuộc: a + b 3 3


≥ .
4
Sử dụng bất đẳng thức AM-GM ta có:
  2 x + 1 + 2 y + 1 
6
 ( 2 x + 1) ( 2 y + 1) ≤ 4  = ( x + y + 1)
3 3 3
4

  2  .
 x+ y
e x + e y ≥ 2 e x .e y = 2e 2
x+ y
2 2
Suy ra P ≥ e + e − ( x + y + 1) ≥ 2e − ( x + y + 1)
x y 3 3
2
.

n
3 3

.v
CHỦ ĐỀ 3: KỸ THUẬT SỬ DỤNG TÍNH ĐƠN ĐIỆU CHO BÀI TOÁN

om
CỰC TRỊ VÀ BẤT ĐẲNG THỨC BA BIẾN SỐ
A. NỘI DUNG PHƯƠNG PHÁP
1) Sử dụng phép thế đưa về bài toán một biến số .c
x + y + z = p
ok

Với điều kiện  xy + yz + zx =q thì luôn biểu diễn được S n = x n + y n + z n theo
 xyz = r

bo

p,q,r.
Do vậy với bài toán giả thiết cho 2 trong 3 điều kiện (p,q,r) ta hoàn toàn có thể
et

đưa S n về một đa thức của một biến.


x + y + z =
vi

0
Ví dụ 1. Cho x,y,z là các số thực thỏa mãn  .
 xy + yz + zx =
−1
ng

Tính P = x 3 + y 3 + z 3 theo x.
a

Lời giải
 y + z =− x
kh

Theo giả thiết ta có:  .


 yz =−1 − x ( y + z ) =−1 − x ( − x ) =x − 1
2

Ta có
( x)3 + 3x ( x 2 − 1) + x3 =3x3 − 3x .
P =x3 + y 3 + z 3 =( y + z ) − 3 yz ( y + z ) + x3 =−
3

Nhận xét. Ta chặn giá trị của biến x như sau

≥ 4 yz ⇔ x 2 ≥ 4 ( x 2 − 1) ⇔ 3 x 2 ≤ 4 ⇔ −
2 2
( y + z) ≤x≤
2

3 3

379
Khám phá tư duy Kỹ thuật giải bất ĐT Bài toán Max – Min – Đặng Thành Nam

Vậy bài toán tìm giá trị lớn nhất và nhỏ nhất của P đưa về khảo sát hàm một
 2 2 
x) 3 x3 − 3 x trên đoạn  −
biến f (= ; .
 3 3
a b c
Ví dụ 2. Cho a,b,c là các số thực dương thoả mãn điều kiện + + = 5.
b c a
b c a
Tìm giá trị lớn nhất và giá trị nhỏ nhất của biểu thức P = + + .
a b c
Lời giải
a b c  xyz = 1
Đặt x= , y= , z= ⇒ .

n
b c a x + y + z =5

.v
1 1 1
Ta cần tìm giá trị lớn nhất nhỏ nhất của biểu thức P = + + .
x y z

om
 1
 yz = 4
⇒ ( y + z ) ≥ 4 yz ⇔ ( 5 − x ) ≥ .
2 2
Ta có  x
 y + z = 5 − x .cx

x ≥ 3 + 2 2
( )
ok
⇔ ( x − 4) x2 − 6 x + 1 ≥ 0 ⇔  .
3 − 2 2 ≤ x ≤ 4
x2 (5 − x ) + 1
bo

1 y+z 1
Khi đó P = + = + x (5 − x ) = .
x yz x x
et

x2 (5 − x ) + 1
Xét hàm số f ( x) = liên tục trên
x
vi

)
D= 3 − 2 2;4   3 + 2 2; +∞ dễ có
ng

min f ( x) =
x∈D
1
f  =
2
17
f (4) =;max f ( x) =
4 x∈D
f 3− 2 2 =
1+ 4 2 . ( )
a

Ví dụ 3. (TSĐH Khối B 2012) Cho các số thực x,y,z thỏa mãn x + y + z =0 và


kh

x2 + y 2 + z 2 =
1 . Tìm giá trị lớn nhất của biểu thức P = x5 + y 5 + z 5 .
Lời giải
Ta có:

 x + y + z =  x + y + z = 0 x + y + z = 0
0 
 2 ⇔ ⇔ 1.
1 ( x + y + z ) − 2 ( xy + yz + zx ) =
1  xy + yz + zx =

2
 x + y + z =
2 2
 2
1 1
Suy ra y + z =− x và x ( y + z ) + yz =− ⇒ yz =− + x2 .
2 2

380
Cty TNHH MTV DVVH Khang Việt

 1  6 6
Mặt khác ( y + z ) ≥ 4 yz ⇒ x 2 ≥ 4  − + x 2  ⇔ −
2
≤x≤ .
 2  3 3
Khi đó :
( )
P = x5 + ( y + z ) − 5 yz y 3 + z 3 − 10 y 2 z 2 ( y + z )
5

= x5 + ( y + z ) − 5 yz ( y + z ) − 3 yz ( y + z )  − 10 y 2 z 2 ( y + z )
5 3
 
2
 1   1    1  10 x3 − 5 x
= x5 + (− x)5 − 5  − + x 2   − x3 − 3  − + x 2  (− x)  − 10  − + x 2  (− x) =
 2   2    2  4

10 x3 − 5 x  6 6

n
Xét hàm số f ( x) = liên tục trên − ;  ta được:

.v
4  3 3 
30 x 2 − 5 6

om
f '( x) = ; f '( x) =0⇔ x=± .
4 6
 6  6 5 6  6  6 5 6
Ta có f  −  =
f   =
− ,f   =
f  −
.c  = .
 3   6  36  3   6  36
ok
5 6
Suy ra=P f ( x) ≤ .
36
bo

5 6 6 6
Vậy giá trị lớn nhất của P bằng đạt tại x = ,y = z = − .
36 3 6
Bình luận. Biểu thức của P là một đối với x nên theo kết quả trên ta tìm được cả
et

giá trị nhỏ nhất của P. Ngoài ra có thể biến đổi P như sau:
(
P = x5 + y 5 + z 5 = x5 + y 3 + z 3 )( y 2
)
+ z2 − y2 z2 ( y + z ) .
vi

Ví dụ 4. Cho a,b,c là các số thực thuộc đoạn [1;4] và a + b + 2c =


ng

8.

Tìm giá trị lớn nhất của biểu thức P = a3 + b3 + 5c3 .


a

Lời giải
Theo giả thiết ta có 2 + c ≤ a + b + 2c =8 ≤ 8 + 2c ⇔ c ∈ [1;3]
kh

Ta có P = ( a + b ) − 3ab ( a + b ) + 5c3 = ( 8 − 2c ) − 3ab ( 8 − 2c ) + 5c3


3 3

−3c3 + 96c 2 − 384c + 512 − 3ab ( 8 − 2c )


=
Với a, b ≥ 1 ⇒ ( a − 1)( b − 1) ≥ 0 ⇔ ab ≥ a + b − 1 = 7 − 2c > 0
Khi đó
P ≤ −3c3 + 96c 2 − 384c + 512 − 3 ( 7 − 2c )( 8 − 2c ) =
−3c3 + 84c 2 − 294c + 344

−3c3 + 84c 2 − 294c + 344 liên tục trên đoạn [1;3] , ta có


Xét hàm số f (c) =

381
Khám phá tư duy Kỹ thuật giải bất ĐT Bài toán Max – Min – Đặng Thành Nam

28 − 7 10
f '(c) =−9c 2 + 168c + 512; f '(c) =0 ⇔ c =c0 = ∈ [1;3] .
3
Ta có f’(c) đổi dấu từ âm sang dương khi đi qua c0 nên f(c) đạt cực tiểu tại c0 .
Do đó P ≤ f (c) ≤ max { f (1); f (3)} = f (3) = 137 .
Đẳng thức xảy ra khi và chỉ khi a= b= 1, c= 3 .
Vậy giá trị lớn nhất của P bằng 137 đạt tại a= b= 1, c= 3 .
2) Đánh giá thông qua các đại lượng trung bình của ba biến số
Với mọi số thực x,y,z ta luôn có
( )
3 x2 + y 2 + z 2 ≥ ( x + y + z ) ;
2

n
.v
( x + y + z )2 ≥ 3 ( xy + yz + zx ) ;
( xy + yz + zx )2 ≥ 3xyz ( x + y + z ) .

om
x+ y+z 3
Với x,y,z là các số thực không âm ta luôn có ≥ xyz ;
.c 3
xyz ≥ ( x + y − z )( y + z − x )( z + x − y )
ok
8
( x + y )( y + z )( z + x ) ≥ ( x + y + z )( xy + yz + zx )
9
Nhận xét. Với các bài toán có x, y, z ∈ [ a; b ] và x + y + z =
bo

s ta thường sử dụng bất


đẳng thức để tìm mối rang buộc giữa các đại lượng đối xứng xy + yz + zx và xyz .
et

Một số đẳng thức đánh chú ý


( a + b + c )3 = a3 + b3 + c3 + 3 ( a + b )( b + c )( c + a )
vi

( a + b )( b + c )( c + a ) = ( a + b + c )( ab + bc + ca ) − abc
ng

2 ( a 2b 2 + b 2 c 2 + a 2 a 2 ) − ( a 4 + b 4 + c 4 ) = ( a + b + c )( a + b − c )( b + c − a )( c + a − b )
Ví dụ 1. (TSĐH Khối B 2011) Cho a,b,c là các số thực không âm thỏa mãn điều
a

kiện a + b + c =1.
kh

Tìm giá trị nhỏ nhất của biểu thức

( )
P 3 a 2b 2 + b 2 c 2 + c 2 a 2 + 3 ( ab + bc + ca ) + 2 a 2 + b 2 + c 2
=
Lời giải
Theo giả thiết ta có:
a 2 + b 2 + c 2 =( a + b + c ) − 2 ( ab + bc + ca ) =1 − 2 ( ab + bc + ca ) .
2

( )
Mặt khác: 3 a 2b 2 + b 2 c 2 + c 2 a 2 ≥ ( ab + bc + ca ) .
2

382
Cty TNHH MTV DVVH Khang Việt

Suy ra P ≥ ( ab + bc + ca ) + 3 ( ab + bc + ca ) + 2 1 − 2 ( ab + bc + ca ) .
2

Đặt t = ab + bc + ca khi đó: P ≥ f (t ) = t 2 + 3t + 2 1 − 2t .

( a + b + c )2 = ⇒ t ∈ 0;  .
1 1 1
Với 0 ≤ ab + bc + ca ≤
3 3  3
 1
Xét hàm số f (t ) = t 2 + 3t + 2 1 − 2t liên tục trên đoạn 0;  ta có:
 3
2 2  1
f '(t ) = 2t + 3 − ; f ''(t ) = 2 − ≤ 0, ∀t ∈ 0;  .
1 − 2t (1 − 2t )
3  3

n
 1  11

.v
Do đó f '(t ) ≥ f '   = − 2 3 > 0 .
3 3

om
 1
Vì vậy f(t) đồng biến trên đoạn 0;  . Suy ra P ≥ f (t ) ≥ f (0) =
2.
 3
Đẳng thức xảy ra khi và chỉ khi a= b= 0, c= 1 hoặc các hoán vị.
.c
Vậy giá trị nhỏ nhất của P bằng 2 đạt tại a= b= 0, c= 1 hoặc các hoán vị.
ok
Ví dụ 2. Cho x,y,z là các số thực thuộc đoạn [0;2] và thoả mãn điều kiện
x+ y+z= 3.
bo

1 1 1
Tìm giá trị lớn nhất của biểu thức P= + + + xy + yz + zx .
x+ y y+z z+x
et

Lời giải
Chú ý bất đẳng thức
vi

8
( x + y )( y + z )( z + x ) ≥ ( x + y + z )( xy + yz + zx ) .
9
ng

Khi đó
( x + y )( x + z ) + ( y + z )( y + x ) + ( z + x )( z + y ) + xy + yz + zx
a

P
( x + y )( y + z )( z + x )
kh

( x + y + z )2 + xy + yz + zx + xy + yz + zx
( x + y )( y + z )( z + x )

( x + y + z ) + xy + yz + zx
2
+ xy + yz + zx
8
( x + y + z )( xy + yz + zx )
9
27 3
= + xy + yz + zx +
8 ( xy + yz + zx ) 8
Chú ý điều kiện x, y, z ∈ [ 0;2] nên

383
Khám phá tư duy Kỹ thuật giải bất ĐT Bài toán Max – Min – Đặng Thành Nam

4 + xyz
( 2 − x )( 2 − y )( 2 − z ) ≥ 0 ⇒ xy + yz + zx ≥ ≥2.
2
1
Khi đó đặt t = xy + yz + zx vì xy + yz + zx ≤ ( x + y + z ) = 3 nên t ∈ [ 2;3] .
2
3
27 3
Do đó P ≤ f (t )= +t + .
8t 8
27 3 27
Xét hàm số f (t )= + t + trên đoạn [2;3] ta có f '(t ) = 1 − 2 > 0, ∀t ≥ 2 .
8t 8 8t
9
Do đó f(t) đồng biến trên đoạn [2;3] suy ra f (t ) ≤ f (3) = .

n
2
Dấu bằng xảy ra khi và chỉ khi x= y= z= 1 .

.v
Vậy giá trị lớn nhất của P bằng 9/2.

om
Nhận xét. Ta có thể đánh giá thông qua bất đẳng thức(xem chương 2).
1 1 1 3 x+ y+z
+ + ≤ + .
x + y y + z z + x 2 ( x + y + z ) xy + yz + zx
.c
Ví dụ 3. Cho x, y, z là các số thực thuộc khoảng (0, 1) thoả mãn điều kiện
ok
(1 − x )(1 − y )(1 − z ) .
xyz =
bo

1 1 1
Tìm giá trị nhỏ nhất của biểu thức P = x + y + z + + + .
x y z
et

Lời giải
Theo giả thiết ta có: x + y + z − xy − yz − zx + 2 xyz − 1 =0.
vi

1
Dự đoán dấu bằng xảy ra khi x = y = z ⇒ x3 = (1 − x ) ⇒ x = y = z =
3
.
ng

2
Do đó ta xét hai khả năng sau:
3 3 1
a

+ Nếu xy + yz + zx < ⇒ > xy + yz + zx ≥ 3 3 x 2 y 2 z 2 ⇒ xyz < .


4 4 8
kh

Khi đó sử dụng bất đẳng thức AM – GM ta có


 1   1   1  3 1 1 1
P = x+ + y+ +z+ +  + + 
 4x   4y   4z  4  x y z 
1 1 1 9
≥ 2 x. + 2 y. + 2 z. +
4x 4y 4 z 4 3 xyz
9 9 15
= 3+ > 3+ =
43 xyz 1 2
4.
2

384
Cty TNHH MTV DVVH Khang Việt

3
+ Nếu xy + yz + zx ≥ khi đó biến đổi P theo xy + yz + zx bằng cách rút
4
xy + yz + zx + 1 − x − y − z
xyz = .
2
Và chú ý x + y + z ≥ 3 ( xy + yz + zx ) .
xy + yz + zx
Ta có P = x + y + z +
xyz
xy + yz + zx
= x + y + z + 2.
xy + yz + zx + 1 − x − y − z

n
xy + yz + zx
≥ 3 ( xy + yz + zx ) + 2.

.v
xy + yz + zx + 1 − 3 ( xy + yz + zx )

om
3
Đặt
= t 3 ( xy + yz + zx ) , ≤ t < 3 ta có
2
t2 .c
3 2t 2
P ≥ t + 2. =t+
t2 t 2 − 3t + 3
ok
+1− t
3
( 2t − 3) ( t 2 − 7t + 15) 3 
bo

15 15
+ ≥ , ∀t ∈  ;3 
(
2 t − 3t + 3
2
) 2 2 2 
et

1
So sánh hai trường hợp ta có Min P bằng 15/2 đạt tại x= y= z= .
2
vi

Nhận xét. Lý do xét hai trường hợp do suy nghĩ làm theo hướng hai đầu tiên tuy
ng

3
nhiên bất đẳng thức cuối chỉ đúng với xy + yz + zx ≥ . Do vậy cần phân chia làm
4
3
a

hai trường hợp, với xy + yz + zx < ta khéo léo kết hợp AM – GM để chỉ ra P lớn
4
kh

hơn 15/2.
Bài tập tương tự
Cho x, y, z là các số thực thuộc khoảng (0, 1) thoả mãn điều kiện
(1 − x )(1 − y )(1 − z ) .
xyz =

Tìm giá trị nhỏ nhất của biểu thức P = x 2 + y 2 + z 2 .


3
ĐS: Pmin = .
4

385
Khám phá tư duy Kỹ thuật giải bất ĐT Bài toán Max – Min – Đặng Thành Nam

Ví dụ 4. Cho a,b,c là các số thực dương. Tìm giá trị nhỏ nhất của biểu thức
a b c 8abc
P= + + −2 + .
b c a ( a + b )( b + c )( c + a )
Lời giải
Sử dụng bất đẳng thức AM-GM ta có:
8abc 8 8.27
= ≥ .
( a + b )( b + c )( c + a )  a  b  c   a b c 
3
 + 1 + 1 + 1  + + + 3 
 b  c  a   b c a 
a b c 216
Đặt t = + + + 3, ( t ≥ 6 ) ta có P ≥ f (t ) = t −5 +

n
.
b c a t3

.v
216
Xét hàm số f (t ) = t −5 + với t ≥ 6 ta có:
t3

om
1 648 t 4 − 1296 t − 5
=
f '(t) −= ≥ 0, ∀t ≥ 6 .
2 t − 5 t4 2t 4 t − 5
.c
2 . Đẳng thức xảy ra khi và chỉ khi a= b= c .
Suy ra P ≥ f (t ) ≥ f (6) =
Vậy giá trị nhỏ nhất của P bằng 2 đặt tại a= b= c .
ok
Ví dụ 5. Cho x,y,z là các số thực không âm thỏa mãn điều kiện x 2 + y 2 + z 2 =
3.
xy + yz + zx
bo

16
=
Tìm giá trị nhỏ nhất của biểu thức P + .
x y +y z +z x
2 2 2 2 2 2 x+ y+z
et

Lời giải

(x )
2
+ y 2 + z 2 − x4 − y 4 − z 4 9 − x4 − y 4 − z 4
2
vi

Ta=có x y + y z + z x
2 2 2 2 2 2
= .
2 2
ng

Sử dụng bất đẳng thức AM-GM ta có :


x 4 + x + x ≥ 3x 2 ; y 4 + y + y ≥ 3 y 2 ; z 4 + z + z ≥ 3z 2
a

Cộng theo vế 3 bất đẳng thức trên ta được :


kh

( )
x4 + y 4 + z 4 ≥ 3 x2 + y 2 + z 2 − 2 ( x + y + z ) = 9 − 2 ( x + y + z ) .

=
xy + yz + zx
( x +=
y + z ) − x2 − y 2 − z 2 ( x + y + z ) − 3
2 2
.
2 2

Do đó P ≥
16
+
( x + y + z) −1 . 2

x + y + z +1 2( x + y + z )
16 t2 −1
Đặt t = x + y + z , t ∈  3;3 . Khi đó P ≥ f (t ) = + .
t +1 2t

386
Cty TNHH MTV DVVH Khang Việt

16 t2 −1
Xét hàm số=
f (t ) + liên tục trên đoạn  3;3 ta có
t +1 2t
1 1 8 1 1 8
f '(t )= 2
+ − ≤ + − < 0 nên f(t) là hàm nghịch biến trên
2t 2
( t + 1)
3 6 2 43

đoạn  3;3 .

28
Do đó P ≥ f (t ) ≥ f (3) = . Đẳng thức xảy ra khi và chỉ khi x= y= z= 1 .
3
Ví dụ 6. Cho a,b,c là các số thực dương thỏa mãn điều kiện ab + bc + ca = 1.

n
Tìm giá trị nhỏ nhất của biểu thức P = a + b + c + abc .

.v
Lời giải
1 − ab
= , c > 0 ⇒ ab < 1 .

om
Theo giả thiết ta có: c
a+b
Thay vào biểu thức của P ta được:
1 − ab 1 − ab 1 − a 2b 2 .c
P = a+b+ + ab. = a+b+ .
a+b a+b a+b
ok
4
a+b
Sử dụng bất đẳng thức AM-GM ta có: a 2b 2 ≤   .
 2 
bo

4
a+b
1−   − ( a + b ) + 16 ( a + b ) + 16
4 2
Suy ra P ≥ a + b +  2 
= .
et

a+b 16 ( a + b )
−t 4 + 16t 2 + 16
vi

a b, ( t > 0 ) khi đó P ≥ f (t ) =
Đặt t =+ .
16t
ng

−t 4 + 16t 2 + 16
Xét hàm số f (t ) = với t > 0 ta có:
16t
a

3t 4 − 16t 2 + 16 (
4 − t 2 3t 2 − 4)( )t = 2
0⇔
kh

f '(t ) =
− = ;f'(t) = .
16t 2
16t 2 t = 2
 3
 2  10 3 10 3
Lập bảng biến thiên suy ra Min f (=t) f  =  ⇒P≥ . Đẳng thức
t >0  3 27 27
1
xảy ra khi và chỉ khi a= b= c= .
3
10 3 1
Vậy giá trị nhỏ nhất của P bằng đạt tại a= b= c= .
27 3

387
Khám phá tư duy Kỹ thuật giải bất ĐT Bài toán Max – Min – Đặng Thành Nam

Ví dụ 7. Cho a,b,c là các số thực thỏa mãn điều kiện a 2 + b 2 + c 2 =


2.
Tìm giá trị lớn nhất và nhỏ nhất của biểu thức P = a3 + b3 + c3 − 3abc .
Lời giải
Ta có:

2 = a 2 + b 2 + c 2 = ( a + b + c ) − 2 ( ab + bc + ca ) ⇒ ab + bc + ca =
2 ( a + b + c )2 − 2 .
2
Khi đó:
P= ( a + b + c ) ( a 2 + b2 + c 2 − ab − bc − ca )

n
= ( a + b + c ) ( a + b + c )2 − 3 ( ab + bc + ca )

.v
 ( a + b + c) − 2  .
2
= ( a + b + c ) ( a + b + c ) − 3.
2
 

om
 2 

−(a + b + c) + 6
2
= ( a + b + c ). .c
2
( )
Đặt t = a + b + c ⇒ t 2 = ( a + b + c ) ≤ 3 a 2 + b 2 + c 2 = 6 ⇔ t ∈  − 6; 6  .
2
ok
1
Khi đó P = f (t ) =− t 3 + 3t .
bo

2
1
Xét hàm số f (t ) = − t 3 + 3t liên tục trên  − 6; 6  ta có
et

2
3
f '(t ) =− t 2 + 3; f '(t ) =0 ⇔ t =± 2 .
vi

2
Bảng biến thiên:
ng

t − 6 − 2 2 6
f’(t) − 0 + 0 −
a

f(t) 0 2 2
kh

−2 2 0
Dựa vào bảng biến thiên suy ra f(t) đạt giá trị lớn nhất bằng 2 2 tại t = 2 và
đạt giá trị nhỏ nhất bằng 2 2 đạt tại t = − 2 .
Vậy giá trị lớn nhất của P bằng 2 2 đạt tại a= 2, b= c= 0 hoặc các hoán vị.
giá trị nhỏ nhất của P bằng −2 2 đạt tại a =
− 2, b =
c=
0 hoặc các hoán vị.

388
Cty TNHH MTV DVVH Khang Việt

3) Bất đẳng thức có hai biến đối xứng


Ghép cặp hai biến đối xứng với nhau và đánh giá bất đẳng thức cơ bản như AM
– GM và Cauchy – Schwarz hoặc một số bất đẳng thức phụ đưa về biến còn lại
và hoàn tất bằng khảo sát hàm số (xem chủ đề sau).
Ví dụ 1. Cho x,y,z là các số thực dương thoả mãn điều kiện x 2 + y 2 + z 2 =
1.
Tìm giá trị lớn nhất của biểu thức P= 6 ( y + z − x ) + 27 xyz .
Lời giải
Sử dụng bất đẳng thức AM – GM và C –S ta có

(
y + z ≤ 2 y 2 + z 2= ) (
2 1 − x2 )

n
y 2 + z 2 1 − x2
yz ≤ =

.v
2 2

( )
Suy ra P ≤ 6  2 1 − x 2 − x  +
27
( )

om
x 1 − x2 .
  2

Xét hàm số f =

(
( x) 6  2 1 − x 2 − x  +
 2
)
27
( )
x 1 − x 2 trên khoảng (0;1) ta có
.c
3 4 2x  4 2x
f '( x) =  −27 x 2 + 5 −  ; f '( x) = 0 ⇔ −27 x 2 + 5 − =0
ok
2  1 − x2

 1 − x2

bo

0 < x < 1
4 2x 
⇔ = 5 − 27 x 2 ⇔ 5 − 27 x 2 > 0 .
et

1 − x2 
( )
2
 32 x =
2
5 − 27 x 2 = 0
1 − x
vi

0 < x < 1
ng


 1
⇔ 5 − 27 x 2 > 0 ⇔x=
 2 3
( )( )( )
a

 3 x − 1 9 x 2
− 1 27 x 2
− 25 =0
kh

1 1
Ta có f’(x) đổi dấu từ dương sang âm khi đi qua nên f(x) đạt cực đại tại .
3 3
 
1 1
Vì vậy P ≤ f ( x) ≤ f   = 10 . Dấu bằng đạt tại x= y= z= .
3 3
Vậy giá trị lớn nhất của P bằng 10.
1
Ví dụ 2. Cho x,y,z là các số thực thỏa mãn điều kiện x 2 + y 2 ≤ z 2 .
2
 1 1 
( 1
Tìm giá trị nhỏ nhất của biểu thức P = x 4 + y 4 + z 4  4 + 4 + 4  .
x y z 
)
389
Khám phá tư duy Kỹ thuật giải bất ĐT Bài toán Max – Min – Đặng Thành Nam

Lời giải
1
Sử dụng bất đẳng thức cơ bản: a 2 + b 2 ≥ ( a + b ) ta có:
2
2
1 2 1
z ≥ x2 + y 2 ≥ ( x + y ) ⇒ x + y ≤ z .
2
2 2
1
( )
1 4 1 1 2 32
2
x4 + y 4 ≥ x2 + y 2 ≥ ( x + y ) ; 4 + 4 ≥ 2 2 ≥ .
2 8 x y x y ( x + y )4
 ( x + y )4   32 1  1 x + y 
4
 z 
4
Suy ra P ≥  +z .
4
  +=    + 32   +5.
 8   ( x + y )4 z 4  8  z   x+ y

n
4

.v
 x+ y 1 32
= Đặt t   , ( 0 < t ≤ 1) ta có: P ≥ f (t ) = t + +5.
 z  8 t

om
1 32
Xét hàm số f (t ) = t + + 5 với 0 < t ≤ 1 ta có:
8 t
1 32
< 0, ∀t ∈ ( 0;1] vì vậy f(t) nghịch biến trên ( 0;1] .
f '(t ) = −
8 t2
.c
ok
297 z
Do đó P ≥ f (t ) ≥ f (1) =. Đẳng thức xảy ra khi và chỉ khi x= y= .
8 2
bo

297
Vậy giá trị nhỏ nhất của P bằng đạt tại =z 2= x 2y .
8
Ví dụ 3. Cho a, b, c là các số thực dương thỏa mãn điều kiện
et

1 1 1 1
a≥ ; + + =2.
2 a +1 b +1 c +1
vi

( )
32 b3 + c3 + 10
ng

3
=
Tìm giá trị nhỏ nhất của biểu thức P + .
8a 2 + 1 64b 2 c 2 + 16bc + 1
Lời giải
a

Chú ý. Ta cần đánh giá bc theo a muốn vậy xuất phát từ điều kiện ta có:
kh

2abc + ab + bc + ca =1 ⇒ bc < 1 .
1 1 1 1 + 2a
Và + = 2− = .
b +1 c +1 1+ a 1+ a
1 1 2 1 + 2a 2 1
Với bc < 1 ta có + ≤ ⇒ ≤ ⇒ bc ≤ .
b + 1 c + 1 1 + bc 1 + a 1 + bc ( 2a + 1)2

Mặt khác
(
32 b3 + c3 + 10 ) ≥
6

6
.
64b c + 16bc + 1
2 2 8bc + 1 8
+1
(2a + 1) 2

390
Cty TNHH MTV DVVH Khang Việt

Suy ra P ≥ =
3
+
6 6 ( 2a − 1) 4a 2 + 8a + 1
2
( + 3≥ 3.
)
8a 2 + 1 8
(2a + 1) 2
+1 8 a 2
+ 1(( 2 a +)(
1) 2
+ 8 )
1 1 1 1 1
Dấu bằng đạt tại a =; b = c; + + = 2⇔a= b= c =.
2 a +1 b +1 c +1 2
Ví dụ 4. Cho a,b,c là các số thực dương thỏa mãn điều kiện abc = 1 .
1 1 2
Tìm giá trị nhỏ nhất của biểu thức P = + + .
2a + 1 2b + 1 (2c + 1) 6c + 3
Lời giải

n
1 1 2 1
Ta biết bất đẳng thức phụ + ≥ đúng với ab ≥ . Do vậy ta

.v
1 + 2a 1 + 2b 2 ab 4
chia trường hợp để xử lý.

om
1 1 1 1 1 1
+ TH1: Nếu ab < ⇒ a < , khi đó P > + > + =
1.
4 4b 2a + 1 2b + 1 1 + 1 2b + 1

1
.c2b

+ TH2: Nếu ab ≥ ⇒ c ≤ 4, khi đó vận dụng bất đẳng thức phụ trên ta được:
ok
4
2 2 c 2
P≥ + = + .
bo

1 + 2 ab ( 2c + 1) 6c + 3 c + 2 (2c + 1) 6c + 3
c 2
Xét hàm số f (c) = + trên khoảng ( 0;4] ta được
et

c + 2 (2c + 1) 6c + 3
vi

8
min f=
(c) f=
(1) .
9
ng

8
Dấu bằng đạt tại a= b= c= 1 . Vậy giá trị nhỏ nhất của P bằng .
9
a

4) Đánh giá xoay quanh các đại lượng (a − b),(b − c),(c − a ) .


kh

Ví dụ 1. Cho các số thực không âm a,b,c có tổng bằng 1.


Tìm giá trị lớn nhất, giá trị nhỏ nhất của biểu thức P =( a − b )( b − c )( c − a ) .
Lời giải
( a b) (b − c ) (c − a ) .
Ta có: P 2 =−
2 2 2

Không mất tính tổng quát ta có thể giả sử a ≥ b ≥ c ≥ 0 khi đó


P 2 ≤ ( a + c − b ) b2 ( a + c ) .
2 2

( )
2
Đặt t= a + c ta có P 2 = ( 2t − 1) t 2 (1 − t ) = 2t 3 − 3t 2 + t
2 2
.

391
Khám phá tư duy Kỹ thuật giải bất ĐT Bài toán Max – Min – Đặng Thành Nam

a+b+c 1 1 
Ta có: t = a + c =1 − b ≤ 1, t = a + c ≥ = . Vậy t ∈  ;1 .
2 2 2 

( 2t ) 1 
2
Xét hàm số f (t ) = 3
− 3t 2 + t liên tục trên  ;1 ta có:
2 
 1
1 
t =
t∈ ;1  2
( )( ) 2 
f '(t )= 2 2t 3 − 3t 2 + t 6t 2 − 6t + 1 ; f '(t )= 0 ← → t = 1

.
t = 3 + 3
 6

n
1 3+ 3  1 1 1

.v
Ta có f = f=
(1) 0, f  = . Suy ra 0 ≤ f (t ) ≤ hay P 2 ≤ .
2  6  108 108 108

om
3 3 3+ 3 3− 3 3
Do đó − ≤P≤ . Tại =
a =
, b 0,=
c thì P = . Tại
18 18 6 6 18
3+ 3 3− 3 3 .c
=a = ,b = , c 0 thì P = − .
6 6 18
ok
3 3+ 3 3− 3
Vậy giá trị nhỏ nhất của P bằng − đạt
= tại a = ,b = , c 0 và giá
108 6 6
bo

3 3+ 3 3− 3
trị lớn nhất của P bằng đạt tại=
a =
, b 0,=
c .
18 6 6
Nhận xét. Xuất phát từ đẳng thức
et

(
x3 + y 3 + z 3 − 3 xyz = ( x + y + z ) x 2 + y 2 + z 2 − xy − yz − zx . )
vi

Ta thay x =a − b, y =−
b c, z =−
c a ta có x3 + y 3 + z 3 =
3 xyz .
ng

Thay vì yêu cầu tìm giá trị lớn nhất, nhỏ nhất của P có thể yêu cầu tìm giá trị
lớn nhất, nhỏ nhất của biểu thức
a

P = ( a − b) + (b − c ) + (c − a ) .
3 3 3
kh

Ví dụ 2. Cho a,b,c là các số thực không âm đôi một phân biệt thỏa mãn
ab + bc + ca = 4.
1 1 1
Tìm giá trị nhỏ nhất của biểu thức P = + + .
( a − b ) ( b − c ) ( c − a )2
2 2

Lời giải
Không mất tính tổng quát giả sử a > b > c ≥ 0 .
1 1 1
Khi đó P ≥ + 2+ 2.
(a − b) a b
2

392
Cty TNHH MTV DVVH Khang Việt

Ta có: 4 = ab + bc + ca ≥ ab ⇒ ab ≤ 4 .
 1 1 1 1 a b
P.ab ab 
Khi đó 4 P ≥= + =
+  + + .
 ( a − b ) 2
a 2
b  a − 2 + b b a
2

b a
a b 1 1 
Đặt t =+ , ( t > 2 ) . Khi đó P ≥ f (t ) =  t + .
b a 4 t −2
1 1 
Xét hàm số f =
(t ) t +  với t > 2, ta có
4 t −2

1 1 

n
t >2
f '(=
t) 1 −  ; f '(=
t ) 0 ← →=
t 3.
4  ( t − 2 )2 

.v
 
Ta có f’(t) đổi dầu từ âm sang dương khi đi qua 3 nên tại t = 3 thì f(t) đạt cực

om
tiểu hay f (t ) ≥ f (3) =1.
Vậy giá trị nhỏ nhất của P bằng 1 đạt tại
a b .c
c = 0, ab = 4, + = 3 ⇔ a = 5 − 1, b = 5 + 1, c = 0 và các hoán vị.
b a
ok
Cách 2: Không mất tính tổng quát giả sử a > b > c ≥ 0 . Khi đó viết lại vế trái của
bất đẳng thức và sử dụng bất đẳng thức AM-GM ta có:
bo

2
 1 1 1   1 1 1 1 1 1 
P=  + +  − 2 . + . + . 
 a −b b−c c−a   a −b b−c b−c c−a c−a a −b
et

2
 1 1 1  4  1 1  4 4
=  + +  ≥  + = ≥ ≥1
 a −b b−c c−a  a −bb−c c−a  ( b − c )( a − c ) ab
vi

Nhận xét. Chú ý đẳng thức quen thuộc


ng

1 1 1 1 1 1 c−a + a −b+b−c
. + . + = . = 0.
a −b b−c b−c c−a c−a a −b ( a − b )( b − c )( c − a )
a

Với giả thiết a,b,c để biểu thức sau có nghĩa và cách chứng minh tương tự ta có
các bất đẳng thức tương tự dạng sau
kh

1) Với a, b, c ≥ 0 ta có
 1 1  11 + 5 5
(a + b2 + c2 
2
) +
1
+ ≥
 ( a − b )2 ( b − c )2 ( c − a )2  2
.

2) Với a, b, c ≥ 0 ta có
 1  59 + 11 33
( a + b ) 2 + ( b + c ) 2 + ( c + a ) 2   1 +
1
+ ≥ .
   ( a − b )2 ( b − c )2 ( c − a )2  4
 

393
Khám phá tư duy Kỹ thuật giải bất ĐT Bài toán Max – Min – Đặng Thành Nam

3) Với a ≠ b ≠ c ta có
 1 1  27
(
a 2 + b 2 + c 2 − ab − bc − ca  )
+
1
+ ≥
 ( a − b )2 ( b − c )2 ( c − a )2  4
.

4) Với a ≠ b ≠ c ta có
 1 1  9
(
a 2 + b2 + c2  ) +
1
+ ≥ .
 ( a − b )2 ( b − c )2 ( c − a )2  2
Sau đây ta cùng xét một số bài toán dạng trên
Ví dụ 3. Cho a,b,c là các số thực không âm đôi một phân biệt.

n
Tìm giá trị nhỏ nhất của biểu thức

.v
 1 1 
P= (a 2
+ b2 + c2  ) +
1
+ .
 ( a − b )2 ( b − c )2 ( c − a )2 

om
Lời giải
Không mất tính tổng quát giả sử c = min {a, b, c} . .c
Khi đó ( a − c ) ≤ a 2 , ( b − c ) ≤ b 2 , a 2 + b 2 + c 2 ≥ a 2 + b 2 .
2 2
ok
Do đó
a b
bo

 1 +
1 1  a 2 + b2 a 2 b2
2
(
P≥ a +b 2

2
) + + =2
 + + =+ 2 b a +a + b .
b  b a 
 ( a − b) a b  ( a − b) b
2 2 2 2
a2 a
−2+
et

b a
a b t
Đặt t =+ , ( t > 2 ) do a ≠ b . Khi đó P ≥ f (t ) = + t2 .
vi

b a t−2
ng

2 t >2 3+ 5
Ta có: f '(t )= − + 2t ; f '(t )= 0 ←
→ t= .
(t − 2) 2 2
a

Bảng biến thiên:


kh

t 3+ 5
2 +∞
2
f’(t) 0
f(t) +∞ +∞

11 + 5 5
2

394
Cty TNHH MTV DVVH Khang Việt

 3 + 5  11 + 5
Dựa vào bảng biến thiên suy ra f (t ) ≥ f   = .
 2  2
Dấu bằng xảy ra khi và chỉ khi
3+ 5 a b 3+ 5 3+ 5 ± 6 5 −2
=t ⇔ +
= ⇔
= a b.
2 b a 2 4
11 + 5 5 3+ 5 ± 6 5 − 2
Vậy giá trị nhỏ nhất của P bằng đạt tại=
c 0,=
a b
2 4
hoặc các hoán vị.
Ví dụ 4. Cho a,b,c là các số thực phân biệt thỏa mãn a + b + c =

n
1 và
ab + bc + ca > 0 .

.v
2 2 2 5
Tìm giá trị nhỏ nhất của biểu thức P = + + + .

om
a −b b−c c−a ab + bc + ca
Lời giải
Không mất tính tổng quát giả sử a > b > c khi đó: .c
2 2 2 5 8 2 5
P= + + + ≥ + +
a−b b−c a−c ab + bc + ca a − b + b − c a − c ab + bc + ca
ok
10 5
= +
a−c ab + bc + ca
bo

 1  1 1
 do ab + bc + ca < 3 ( a + b + c ) =
2
Đặt =
t ab + bc + ca ,  0 < t < .
 3 3
et

1 1
Ta có: ( a − b ) + ( b − c ) ≥ ( a − b ) + ( b − c )  = ( a − c )2 .
2 2 2
vi

2 2
Suy ra:
ng

3
( a − c )2 ≤ ( a − b )2 + ( b − c )2 + ( c − a )2 =2 ( a + b + c )2 − 3 ( ab + bc + ca ) =−
2 6t 2
2
a

2 1 − 3t 2 5 3 5
⇒a−c≤ ⇒ P ≥= +
kh

f (t )
3 1 − 3t 2 t

5 3 5  1 
Xét hàm =
số f (t ) + với t ∈  0;  ta có:
1 − 3t t 2 3

f '(t ) =
15 3t

5
; f '(t ) = 0 ⇔ 3 3t 3 = (1 − 3t )
2 3
⇔t=
1
.
(1 − 3t )
2
2 3 t 6

395
Khám phá tư duy Kỹ thuật giải bất ĐT Bài toán Max – Min – Đặng Thành Nam

1
Ta có f’(t) đổi dấu từ âm sang dương khi đi qua t = nên f(t) đạt cực tiểu tại
6
1  1 
t= . Do đó P ≥ f (t ) ≥ f  =10 6 .
6  6
1 1 1 1 1
Đẳng thức xảy ra tại a = + ,b = ,c = − .
3 6 3 3 6
1 1 1 1 1
Vậy giá trị nhỏ nhất của P bằng 10 6 đạt tại a = + ,b = ,c = − hoặc
3 6 3 3 6
các hoán vị.

n
Cách 2: Đánh giá thông qua bất đẳng thức AM-GM.

.v
Không mất tính tổng quát ta giả sử a > b > c ta có:
2 2 2 5
P= + + +

om
.
a −b b−c a −c ab + bc + ca
1 1 4
Sử dụng bất đẳng thức: + ≥ , ∀x, y > 0 ta được:
x y x+ y .c
4 2 5  2 1 
P ≥ 2. + + = 5 +
ok
a −b+b−c a −c 
ab + bc + ca a−c ab + bc + ca 
5.2 2 20
bo

≥ =
( ) (
4 a − c 2 ab + bc + ca
) 4 ( a − c )2 ( 4ab + 4bc + 4ca )
et

20 20 2 20 2
≥ = =
(a − c) 2
+ 4ab + 4bc + 4ca ( a + c ) + 4b ( a + c ) ( a + c )( a + c + 4b )
2
vi

2
ng

20 2 20 6 40 6
= = ≥ = 10 6
(1 − b )(1 + 3b ) ( 3 − 3b )(1 + 3b ) 3 − 3b + 1 + 3b
a

 2+ 6
a =
kh

a − b = b − c 6
 
 2 1  1
Đẳng thức xảy ra khi và chỉ khi
=  = ⇔ b .
a − c ab + bc + ca  3
3 − 3b =+
1 3b  2− 6
c =
 6
1 1 1 1 1
Vậy giá trị nhỏ nhất của P bằng 10 6 đạt tại a = + ,b = ,c = − hoặc
3 6 3 3 6
các hoán vị.

396
Cty TNHH MTV DVVH Khang Việt

Ví dụ 5. Cho a,b,c là các số thực không âm thỏa mãn điều kiện


(
2 a 2 + b2 + c2 = ) ( ab + bc + ca + 1)( ab + bc + ca ) .
Tìm giá trị lớn nhất của biểu thức =
P 2 a 2 + b2 + c2 − a − b − b − c − c − a .
Lời giải
P 2 a 2 + b2 + c2 − 2 ( a − c ) .
Không mất tính tổng quát giả sử a ≥ b ≥ c ⇒=
t ( t + 1)
Đặt t = ab + bc + ca, ( t ≥ 0 ) theo giả thiết ta có: a 2 + b 2 + c 2 = .
2
Nếu t = 0 ⇔ a = b = c = 0 ⇒ P = 0 .

n
Xét t > 0 khi đó sử dụng bất đẳng thức cơ bản ta có:

.v
t ( t + 1)
a 2 + b 2 + c 2 ≥ ab + bc + ca ⇒ ≥ t ⇔ t ≥ 1.

om
2
Ta có
( a − b )2 + ( b − c )2 + ( c − a )2 = ( )
2 a 2 + b 2 + c 2 − 2 ( ab + bc + ca ) = t ( t + 1) − 2t = t ( t − 1) .
.c
( a − c )2 =( a − b ) + ( b − c )  ( a − b ) + ( b − c ) + 2 ( a − b )( b − c ) ≥ ( a − b ) + ( b − c ) .
2 2 2 2 2
=
ok
t ( t − 1)
Suy ra 2 ( a − c ) ≥ ( a − b ) + ( b − c ) + ( c − a ) = t ( t − 1) ⇒ a − c ≥
2 2 2 2
.
bo

2
2 2t
Do đó P ≤ f (=
t) 2t ( t + 1) − 2t ( t − 1=
) .
t +1 + t −1
et

Xét hàm số f (=
t) 2t ( t + 1) − 2t ( t − 1) với t ≥ 1 ta có:
vi

2t + 1 2t − 1
f '(t ) = − ; f '(t ) =0 ⇔ ( 2t + 1) t ( t − 1) =( 2t − 1) t ( t + 1) .
ng

2t ( t + 1) 2t ( t − 1)
t ≥ 1
a

⇔ (vô nghiệm).
( 2t + 1) ( t − 1) =
( 2t − 1)2 ( t + 1)
2
kh

Vậy f’(t) không đổi dấu trên (1;+∞ ) ta có f '(2) < 0 nên f '(t ) < 0, ∀t > 1 .
Vậy f(t) là hàm nghịch biến trên [1;+∞ ) .
1
2 . Đẳng thức xảy ra khi và chỉ khi a= b= c=
Suy ra P ≤ f (t ) ≤ f (1) = .
3
1
Vậy giá trị lớn nhất của P bằng 2 đạt tại a= b= c= .
3

397
Khám phá tư duy Kỹ thuật giải bất ĐT Bài toán Max – Min – Đặng Thành Nam

Ví dụ 6. Cho a,b,c là các số thực thỏa mãn điều kiện a 2 + b 2 + c 2 =


5.
Tìm giá trị lớn nhất và nhỏ nhất của biểu thức
P = ( a − b )( b − c )( c − a )( ab + bc + ca ) .
Lời giải
Ta có: P 2 = ( a − b ) ( b − c ) ( c − a ) ( ab + bc + ca ) .
2 2 2 2

Để tìm giá trị lớn nhất và nhỏ nhất của P ta tìm giá trị lớn nhất của P 2 .
Khi đó ta có thể giả sử a ≥ b ≥ c khi đó:
Sử dụng bất đẳng thức AM-GM ta có:

n
 ( a − b) + (b − c )  (a − c) .
4 4
( a − b) (b − c ) =( a − b )( b − c )  ≤ 
2 2 2
 =

.v
 2  16

om
1
Suy ra P 2 ≤ ( c − a )6 ( ab + bc + ca )2 .
16
Đặt t = ab + bc + ca, ( t ≤ 5 ) do ab + bc + ca ≤ a 2 + b 2 + c 2 =
5.

Ta có: ( a − b ) + ( b − c ) ≥
2 2 1
.c
( a − b ) + ( b − c )  =
2 1
( a − c )2 .
ok
2 2
Suy ra:
3
(
( a − c )2 ≤ ( a − b )2 + ( b − c )2 + ( c − a )2 =2 a 2 + b2 + c 2 − ab − bc − ca =10 − 2t)
bo

2
3
20 − 4t 1  20 − 4t  2 4 2
⇒ (a − c) t (5 − t )
2 3
≤ ⇒P≤  =
et

t
3 16  3  27
vi

4 2
t ( 5 − t ) với t ≤ 5 ta có:
3
=
Xét hàm số f (t )
27
ng

t = 0
f '(t ) =
20
− t ( t − 2 )( t − 5 ) ; f '(t) =⇔
2 t =
0  2.
a

27
t = 5
kh

Bảng biến thiên:


t −∞ 0 2 5
f’(t) 0 0 0
+∞ 16
f(t)

0 0
Dựa vào bảng biến thiên suy ra P 2 ≤ f (t ) ≤ f (2)= 16 ⇒ −4 ≤ P ≤ 4 .

398
Cty TNHH MTV DVVH Khang Việt

Tại =a 2,= b 1,=c 0 thì P = −4 ; tại a = −2, b =−1, c =0 thì P = 4 .


Vậy giá trị nhỏ nhất của P bằng −4 ; giá trị lớn nhất của P bằng 4.
Nhận xét. Phép toán bình phương đưa về đa thức đối xứng với ba biến a,b,c giúp ta
có thể giả sử được a ≥ b ≥ c . Đây là một cách rất hay được áp dụng khi giá trị lớn
nhất và nhỏ nhất của bài toán có giá trị đối nhau.
Bài tập tương tự
Với mọi a,b,c tìm số thực M nhỏ nhất thoả mãn bất đẳng thức

( ) ( ) ( ) ( )
2
ab a 2 − b 2 + bc b 2 − c 2 + ca c 2 − a 2 ≤ M a 2 + b 2 + c 2 .

Ví dụ 7. Cho a,b,c là các số thực thoả mãn điều kiện a 2 + b 2 + c 2 =


1.

n
Tìm giá trị lớn nhất, giá trị nhỏ nhất của biểu thức

.v
P= ( a − b )( b − c )( c − a )( a + b + c ) .

om
Lời giải
Chuyển P về dạng đối xứng
P= ( a − b )2 ( b − c )2 ( c − a )2 ( a + b + c )2 .c .
= ( a − b )2 ( b − c )2 ( c − a )2 1 + 2 ( ab + bc + ca ) 
ok
 1 
Không mất tính tổng quát giả sử a ≥ b ≥ c và đặt x= ab + bc + ca, x ∈  − ;1 .
 2 
bo

Ta có :
2 2
a −b+b−c  a −c
( a − b )( b − c ) ≤ 
1
 ⇒ ( a − b )( b − c )( a − c ) ≤ ( a − c )
3
=
et

 
 2   2  4
Mặt khác lại có
vi

1 1 1
a 2 + b 2 + c 2 − ab − bc − ca= ( a − b )2 + ( b − c )2 + ( a − c )2
ng

2 2 2
1 1
≥ (a − c) + (a − b + b − c)
2 2
a

2 4
kh

3 4 1
Suy ra 1 − x ≥ ( a − c )2 ⇒ a − c ≤ (1 − x ) ; − ≤ x ≤ 1
4 3 2
2
1  4  
3
4
Suy ra P ≤ (1 + 2 x )  
2
(1 − x )  = ( 2 x + 1)(1 − x )3
4 3   27
 .

4  1 
Xét hàm số f ( x=
) ( 2 x + 1)(1 − x )3 trên đoạn  − 2 ;1 ta có
 
27

399
Khám phá tư duy Kỹ thuật giải bất ĐT Bài toán Max – Min – Đặng Thành Nam

x =1
4
0⇔
− ( x − 1) ( 8 x + 1) ; f '( x) =
2
f '( x) =
27 x = − 1
 8.
 1  81
Từ đó dễ có max f ( x) = f  −  = .
 1 
x∈ − ;1  8  512
 2 

81 9 9
Suy ra P 2 ≤ ⇔− ≤P≤
512 16 2 16 2
9
Vậy giá trị nhỏ nhất của P bằng − đạt tại

n
16 2

.v
3 3+ 6 1 6 −3 3
=a = ,b = ,c
6 2 2 3 6 2

om
Ví dụ 8. Cho a,b,c là các số thực thỏa mãn điều kiện a 2 + b 2 + c 2 =
8.
Tìm giá trị lớn nhất, nhỏ nhất của biểu thức
5 5 .c
P = ( a − b) + (b − c ) + (c − a ) .
5
ok
Lời giải
Ta có:
( a − b )5 + ( b − c )5 + ( c − a )5 = 5 ( a − b )( b − c )( c − a ) ( a 2 + b2 + c 2 − ab − bc − ca )
bo

= 5 ( a − b )( b − c )( c − a )( 8 − ab − bc − ca )
et

Vậy để tìm giá trị lớn nhất và nhỏ nhất của P ta tìm giá trị lớn nhất của biểu thức
P 2 = ( a − b ) ( b − c ) ( c − a ) ( 8 − ab − bc − ca ) .
2 2 2 2
vi

Khi đó ta có thể giả sử a ≥ b ≥ c .


ng

Sử dụng bất đẳng thức AM-GM ta có:


 ( a − b) + (b − c )  (a − c) .
4 4
( a − b) (b − c ) =( a − b )( b − c )  ≤ 
2 2 2
 =
a

 2  16
kh

25
Suy ra P 2 ≤ ( c − a )6 (8 − ab − bc − ca )2 .
16
Đặt t = ab + bc + ca, ( t ≤ 8 ) do ab + bc + ca ≤ a 2 + b 2 + c 2 =
8.
1 1
Ta có: ( a − b ) + ( b − c ) ≥ ( a − b ) + ( b − c )  = ( a − c )2 .
2 2 2
2 2
Suy ra:

400
Cty TNHH MTV DVVH Khang Việt

3
2
( )
( a − c )2 ≤ ( a − b )2 + ( b − c )2 + ( c − a )2 =2 a 2 + b2 + c 2 − ab − bc − ca =16 − 2t
3
32 − 4t 25  32 − 4t  100
⇒ (a − c) ≤  ( 8 − t ) = f (t )= (8 − t )5
2 2
⇒P≤ 
3 16  3  27
Mặt khác:
( a + b + c )2 =a 2 + b 2 + c 2 + 2 ( ab + bc + ca ) =+
8 2t ≥ 0 ⇔ t ≥ −4 ⇒ −4 ≤ t ≤ 8 .
100
Xét hàm số=
f (t ) (8 − t )5 trên đoạn [ −4;8] ta có:
27
500
( t − 8)2 ≤ 0, do đó f(t) là hàm nghịch biến trên đoạn [ −4;8] .

n
f '(t ) =

27

.v
Suy ra P ≤ f (t ) ≤ = f (−4) 921600 ⇒ −960 ≤ P ≤ 960 .
Với a = −2, b = 0, c =⇒
2 P= 960 . Với a = −2, b = 2, c = 0 thì P = −960 .

om
Vậy giá trị lớn nhất của P bằng 960 và giá trị nhỏ nhất của P bằng −960 .
Ví dụ 9. Cho a,b,c là các số thực không âm thoả mãn điều kiện ab + bc + ca = 1.

Tìm giá trị nhỏ nhất của biểu thức P =


1
+
1
.c +
1
.
a 2 + b2 b2 + c2 c2 + a2
ok
Lời giải
Không mất tính tổng quát giả sử c = min {a, b, c} , khi đó ta có:
bo

2 2
 c  c
a 2 + b2 ≤  a +  +  b +  ;
et

 2  2
2
 c
vi

a + c ≤ a +  ;
2 2
 2
ng

2
 c
b + c ≤ b +  ;
2 2
 2
a

 c  c
1 = ab + bc + ca ≥  a +  b + 
kh

 2  2
c c
Đặt x =
a+ ,y=
b + suy ra
2 2
1 1 1 xy 1 1 1 x y
P≥ + + ≥ + xy  +  = + + .
x +y
2 2 x y x +y
2 2
x y x
+
y y x
y x
1  1  1
Đặt t
= , 0 < t ≤  khi đó P ≥ f (t ) =
t+ 2 .
x y  2 t
+
y x
401
Khám phá tư duy Kỹ thuật giải bất ĐT Bài toán Max – Min – Đặng Thành Nam

2 t3 − 2  1 
Ta có f '(t ) = 1 − = < 0, ∀t ∈  0;  nên f(t) là hàm nghịch biến trên
 2
3 3
t t
 1 
 0; .
 2
 1  1
Do đó P ≥ f (t ) ≥ f  =2+ .
 2 2
1
Vậy giá trị nhỏ nhất của P bằng 2 + đạt tại a= b= 1, c= 0 hoặc các hoán
2
vị(xem thêm kỹ thuật sử dụng tính đẳng cấp).

n
Cách 3: Đặt=x a=
2
, y b=
2
, z c 2 khi đó xy + yz + zx ≤ 1 . Bài toán đưa về tìm giá

.v
1 1 1
trị nhỏ nhất của biểu thức P = + + . Bài toán này tôi trích

om
x+ y y+z z+x
riêng và trình bày dưới đây.
Ví dụ 10. Cho a,b,c là các số thực không âm thay đổi thỏa mãn điều kiện
ab + bc + ca =1.
1 1 1
.c
Tìm giá trị nhỏ nhất của biểu thức P = + +
ok
.
a+b b+c c+a
Lời giải
bo

Nhận xét. Với điều kiện a,b,c không âm và ab + bc + ca = 1 nghĩ đến P đạt giá trị
nhỏ nhất khi một số bằng 0 và hai số bằng 1. Vậy ta có ghép cặp hai căn lại với
nhau xem có đánh giá được qua căn thức còn lại hay không.
et

2
 1 1  1 1 2
Ta có:  +  = a+b + a+c +
vi

 a+b a+c  ( a + b )( a + c )
ng

2a + b + c 2 2a + b + c 2
= + = + .
a + ab + bc + ca
2
a + ab + bc + ca
2 a +1
2
a +1
2
a

1 1 2a + b + c + 2 a 2 + 1
+ = 2
kh

Suy ra .
a+b a+c a +1
Không mất tính tổng quát giả sử a = max {a, b, c} khi đó:

1 2a + b + c + 2 1 + a 2
=P + .
b+c 1 + a2
Coi đây là hàm số với biến a và b,c là các hằng số ta được:
bc − a 2 − a a 2 + 1
f '(a) ≤0.
( a + 1) ( 2a + b + c + 2 )
3
2
a +1
2

402
Cty TNHH MTV DVVH Khang Việt

1 − bc 1
Do đó f(a) là hàm nghịch biến nên =
từ a ≤ suy ra
b+c b+c
 1 
f (a) ≥ f  .
b+c
 1  1 b+c
Mặt khác: f  = b+c + + .
b+c b+c ( b + c )2 + 1
1 t 1 1 1 1
Đặt t = b + c ⇒ P ≥ t + + =t + + =t + + .
t t +1
4 t 1 t 2
t2 + 2  1
t t +  − 2

n
 t

.v
1 1
t + , ( u ≥ 2 ) . Khi đó P ≥ g (u ) =
Đặt u = u+ trên [ 2;+∞ ) ta có:
t u −2
2

om
u u u −1
g '(u ) = 1 − ≥1− = > 0, ∀u ≥ 2 .
(u )
3
2
−2 u3 u
.c
1
Do đó g(u) là hàm đồng biến trên [ 2;+∞ ) nên P ≥ g (u ) ≥ g(2) =
2+
ok
.
2
1
Vậy giá trị nhỏ nhất của P bằng 2 + đạt tại a= b= 1, c= 0 hoặc các hoán vị.
bo

2
1 t
Nhận xét. Ta có thể khảo sát trực tiếp hàm số g (t ) = t + + trên ( 0;+∞ ) ta được:
et

t t4 +1
vi

2t 4
t4 +1 −
t 4 + 1 =t − 1 + 1− t4
2
1
ng

g '(t ) =−
1 +
t2 t4 +1 t2 (
t4 +1 t4 +1 )
a

 
(t − 1)  (t + 1)
3
 
2 4
− t4 − t2 
kh

= (  1
)
t −1 2 −
2 t2 +1
=   ; g '(= t >0
t ) 0 ← →=t 1
t

 ( 
t4 +1 t4 +1 
 ) t2 (t + 1)
4 3

Ta có g’(t) đổi dấu từ âm sang dương khi đi qua t = 1 nên g(t) đạt cực tiểu tại
1
t = 1 trên khoảng ( 0;+∞ ) hay min g (t )= g(1)= 2 + .
t∈( 0;+∞ ) 2
1
Vậy giá trị nhỏ nhất của P bằng 2 + đạt tại a= b= 1, c= 0 hoặc các hoán vị.
2
Cách 2: Đặt=
x 2 a= , z 2 c. Khi đó x 2 y 2 + y 2 z 2 + z 2 x 2 =
, y 2 b= 1.
403
Khám phá tư duy Kỹ thuật giải bất ĐT Bài toán Max – Min – Đặng Thành Nam

Suy ra
( xy + yz + zx )2 = x 2 y 2 + y 2 z 2 + z 2 x 2 + 2 xyz ( x + y + z ) ≥ x 2 y 2 + y 2 z 2 + z 2 x 2 = 1 .
Bài toán đưa về tìm giá trị nhỏ nhất của biểu thức
1 1 1
P= + + với xy + yz + zx ≥ 1 .
x2 + y 2 y2 + z2 z 2 + x2
Đây chính là bài toán vừa trình bày trước đó.

B. BÀI TẬP RÈN LUYỆN


3
Bài 1. Cho x,y,z là ba số thực dương thỏa mãn điều kiện x + y + z ≤

n
.
2

.v
1 1 1
Tìm giá trị nhỏ nhất của biểu thức P = x3 + y 3 + z 3 + + + .
x y z

om
Bài 2. Cho x,y,z là các số thực không âm thỏa mãn điều kiện x 2 + y 2 + z 2 =
3.
Tìm giá trị lớn nhất và nhỏ nhất của biểu thức .c
5
P = xy + yz + zx + .
x+ y+z
ok
Bài 3. Cho x,y,z là các số thực không âm thỏa mãn điều kiện x + y + z =
1.
bo

Tìm giá trị nhỏ nhất của biểu thức


( )
P 3 x 2 y 2 + y 2 z 2 + z 2 x 2 + 3 ( xy + yz + zx ) + x 2 + y 2 + z 2 .
=
et

Bài 4. Cho các số thực x,y,z thỏa mãn điều kiện x 2 + y 2 + z 2 =


1.
vi

Tìm giá trị lớn nhất và nhỏ nhất của biểu thức
4
P = xy + yz + zx +
ng

.
xy + yz + zx + 2
Bài 5. Cho x,y,z là các số thực không âm thỏa mãn điều kiện
a

( )
3 x 2 + y 2 + z 2 + xy + yz + zx =
12 .
kh

Tìm giá trị lớn nhất và nhỏ nhất của biểu thức
x2 + y 2 + z 2
=P + xy + yz + zx .
x+ y+z
Bài 6. Cho x,y,z là các số thực dương thỏa mãn điều kiện x + y + z =3.
xy + yz + zx
Tìm giá trị nhỏ nhất của biểu thức P = x 2 + y 2 + z 2 + .
x + y2 + z2 + 3
2

Bài 7. Cho x,y,z là các số thực dương thỏa mãn điều kiện x + y + z =3.

404
Cty TNHH MTV DVVH Khang Việt

x3 + y 3 + z 3
Chứng minh rằng 3 + 8 3 xyz ≤ 9 .
3
Bài 8. Cho x,y,z là các số thực dương thỏa mãn điều kiện x 2 + y 2 + z 2 =
3.

=
Tìm giá trị nhỏ nhất của biểu thức P
( x + y + z − 1)2 +
1
.
x y+ y z+z x
2 2 2 x+ y+z
Bài 9. Cho x,y,z là các số thực dương thỏa mãn điều kiện x + y + z =3.
xy + yz + zx
Tìm giá trị nhỏ nhất của biểu thức P = x 2 + y 2 + z 2 + .
x y + y2 z + z2 x
2

n
Bài 10. Cho các số thực dương x,y,z thỏa mãn điều kiện xy + yz + zx =
3.

.v
( ) 324
2
Tìm giá trị nhỏ nhất của biểu thức P = x 2 + y 2 + z 2 + .
x+ y+z

om
Bài 11. Cho x,y,z là các số thực dương thỏa mãn điều kiện xyz = 1 .
72
Tìm giá trị nhỏ nhất của biểu thức P = ( x + y )( y + z )( z + x ) + .c .
x + y + z +1
ok
Bài 12. Cho a,b,c là các số thực dương thỏa mãn điều kiện
( )
3 a 2 + b2 + c2 = 3( a + b + c ) + 4 .
bo

Tìm giá trị nhỏ nhất của biểu thức


a+b+c
P = a 3 + b3 + c 3 − − 6 ( ab + bc + ca ) .
et

a + b2 + c2
2

Bài 13. Cho x,y,z là các số thực thỏa mãn điều kiện x 2 + y 2 + z 2 =
3.
vi

( x + 2 )( y + 2 )( z + 2 ) .
Tìm giá trị nhỏ nhất của biểu thức P =
ng

Bài 14. Cho các số dương x, y, z . Tìm giá trị nhỏ nhất của biểu thức
x2 y y2 z z2 x 13 xyz
a

S= + + + .
z 3
x 3
y 3
(
3 xy + yz 2 + zx 2
2
)
kh

3
Bài 15. Cho x,y,z là các số thực thuộc đoạn [ 0;1] và x + y + z = .
2
Tìm giá trị lớn nhất của biểu thức P = x 2 + y 2 + z 2 .
Bài 16. Cho x,y,z là các số thực dương thỏa mãn x + y + z =
10 và xy + yz + zx =
xyz .
Tìm giá trị lớn nhất và nhỏ nhất của biểu thức P = x 2 + y 2 + z 2 .
x + y + z =4
Bài 17. Cho x, y, z là các số thực thỏa mãn điều kiện  .
 xyz = 2

405
Khám phá tư duy Kỹ thuật giải bất ĐT Bài toán Max – Min – Đặng Thành Nam

Chứng minh rằng 183 − 165 5 ≤ x 4 + y 4 + z 4 ≤ 18 .


Bài 18. Cho a,b,c là các số thực dương. Chứng minh rằng
a + 2014 b + 2014 c + 2014 a b c
+ + ≤ + + .
b + 2014 c + 2014 a + 2014 b c a
Bài 19. Cho x,y,z là các số thực thỏa mãn 1 ≤ x ≤ y ≤ z ≤ 4 .
1 1 1  1 1
Chứng minh rằng ( x + y + z )  + +  ≤ (1 + y + z ) 1 + +  .
x y z  y z
Bài 20. Cho a,b,c là các số thực dương thoả mãn điều kiện a + b + c =3.
2 abc

n
=
Tìm giá trị lớn nhất của biểu thức: P +3 .
3 + ab + bc + ca (1 + a )(1 + b)(1 + c)

.v
1 
Bài 21. Cho x,y,z là các số thực thuộc đoạn  ;1 .

om
2 
y+z z+x x+ y
Tìm giá trị lớn nhất, nhỏ nhất của biểu thức P = + + .
.c x +1 y +1 z +1
Bài 22. Cho a,b,c là các số thực thỏa mãn a + b + c =3.
( )
ok
Chứng minh rằng abc a 2 + b 2 + c 2 ≤ 3 .

Bài 23. Cho x,y,z là các số thực thỏa mãn điều kiện x + y + z =0 và x 2 + y 2 + z 2 =
bo

1.
Tìm giá trị lớn nhất của biểu thức P = x 2 y 2 z 2 .
et

Bài 24. Cho a,b,c là các số thực. Chứng minh rằng

( ) (a )
3
6 ( a + b + c ) a 2 + b 2 + c 2 + 27 abc ≤ 10 2
+ b2 + c2
vi

Bài 25. Cho a,b,c là các số thực dương thỏa mãn điều kiện ab + bc + ca =
ng

3.
1 4
Tìm giá trị nhỏ nhất của biểu thức= P + .
abc ( a + b )( b + c )( c + a )
a

Bài 26. Cho a,b,c,d,e là các số thực dương thay đổi thỏa mãn a + b + c + d + e = 1.
kh

Tìm giá trị lớn nhất của biểu thức P = abc + bcd + cde + eda + eab .
Bài 27. Cho a,b,c là các số thực dương thỏa mãn điều kiện ab + bc + ca = 3.
Tìm giá trị lớn nhất của biểu thức
a b c
= P + + − a 2 − b2 − c2 + 2 ( a + b + c ) .
( a + b )( a + c ) ( b + c )( b + a ) ( c + a )( c + b )
Bài 28. Cho a,b,c là các số thực thuộc đoạn 0; 3  . Tìm giá trị lớn nhất của biểu
8
thức P = − + 31 − a 2 ( a + 1) − b 2 ( b + 1) − c 2 ( c + 1) .
a + b + c +1
406
Cty TNHH MTV DVVH Khang Việt

Bài 29. Cho a,b,c là các số thực không âm thỏa mãn điều kiện a + b + c =3.
Tìm giá trị lớn nhất và nhỏ nhất của biểu thức
a 4 + b4 + c4
=P + 3 ( ab + bc + ca ) .
a 3 + b3 + c 3
Bài 30. Cho a,b,c là các số thực không âm và không có hai số nào đồng thời bằng
0. Tìm giá trị nhỏ nhất của biểu thức
1 1 1
P= + + .
a 2 − ab + b 2 b 2 − bc + c 2 c 2 − ca + a 2
Bài 31. Cho a,b,c là các số thực không âm thỏa mãn điều kiện ab + bc + ca > 0 .

n
(
2 a 2 + b2 + c2 ) + 3.

.v
abc
Tìm giá trị nhỏ nhất=
của biểu thức P .
a+b+c 4 a b + b2c + c 2 a
2

om
Bài 32. Cho x,y,z là các số thực thuộc khoảng ( 0;1) thỏa mãn điều kiện
(1 − x )(1 − y )(1 − z ) .
xyz =
.c
Tìm giá trị nhỏ nhất của biểu thức P = x 2 + y 2 + z 2 .
ok
Bài 33. Cho x,y,z là các số thực thỏa mãn điều kiện x3 + y 3 + z 3 − 3 xyz =
1.
Tìm giá trị nhỏ nhất của biểu thức P = x 2 + y 2 + z 2 .
bo

x + y + z =0

Bài 34. Cho x,y,z là các số thực thoả mãn điều kiện  2 8.
 x + y + z =
2 2
et

3
Tìm giá trị lớn nhất, giá trị nhỏ nhất của biểu thức P = xy 2 + yz 2 + zx 2 .
vi

a 2 + c 2 =
ng

1
Bài 35. Cho a,b,c là các số thực thoả mãn điều kiện  .
b + 2b ( a + c ) =
2
6
Chứng minh rằng b ( c − a ) ≤ 4 .
a
kh

Bài 36. Cho a,b,c số thực không âm thỏa mãn điều kiện
a + 2b + 1 + a + 2c + 1 =4.
Tìm giá trị lớn nhất của biểu thức: P = a (1 + b ) + b (1 + c ) + c (1 + a ) .

Bài 37. Cho x,y,z là các số thực không âm thoả mãn điều kiện x3 + y 3 + z 3 =2 + 3 xyz .
x2 + 2 y 2 + z 2
Tìm giá trị nhỏ nhất của biểu thức P = .
x2 + y 2 + z 2 + 1

407
Khám phá tư duy Kỹ thuật giải bất ĐT Bài toán Max – Min – Đặng Thành Nam

Bài 38. Cho a,b,c là các số thực không âm thỏa mãn điều kiện a + b + c =3.
Tìm giá trị lớn nhất và nhỏ nhất của biểu thức P =( a − 1) + ( b − 1) + ( c − 1) .
3 3 3

Bài 3 9. Cho a,b,c là các số thực không âm đôi một khác nhau.
Tìm giá trị nhỏ nhất của biểu thức
 1 1 1 
P = ( a + b ) + ( b + c ) + ( c + a )  
2 2 2
+ + .
   ( a − b )2 ( b − c )2 ( c − a )2 
 

D. HƯỚNG DẪN GIẢI – ĐÁP SỐ

n
3
 x+ y+z 1 1 1 9
Bài 1. Ta có: x3 + y 3 + z 3 ≥ 3   và + + ≥ .

.v
 3  x y z x+ y+z
3
 x+ y+z

om
9
Suy ra P ≥ 3   + .
 3  x+ y+z
 3 t3 9
Đặt t = x + y + z ,  0 < t ≤  ta có P ≥ + .
.c
 2 9 t
ok
t3 9 3
Xét hàm số f (t=
) + với 0 < t ≤ ta có
9 t 2
bo

t 2 9 t 3 − 27  3  3  51
f '(t )= − 2= < 0, ∀t ∈  0;  ⇒ P ≥ f (t ) ≥ f  = .
3 t 3t 2
 2 2 8
et

51 1
Vậy giá trị nhỏ nhất của P bằng đạt tại x= y= z= .
8 2
vi

t2 − 3
Bài 2. Đặt t =x + y + z ⇒ t 2 =3 + 2 ( xy + yz + zx ) ⇒ xy + yz + zx = .
ng

2
t2 − 3
Do 0 ≤ xy + yz + zx ≤ x 2 + y 2 + z 2 = 3 ⇒ 0 ≤ ≤ 3 ⇔ t ∈  3;3 .
a

2
t2 − 3 5
kh

Khi đó=
P f=
(t ) + .
2 t
t2 − 3 5
Xét hàm số =
f (t ) + với t ∈  3;3 ta có
2 t
5 t3 − 5
f '(t ) = t − = > 0, ∀t ∈  3;3 .
t2 t2
5
Vậy f(t) là hàm đồng biến trên đoạn  3;3 . Do đó P =f (t ) ≥ f ( 3) = .
3
Đẳng thức xảy ra khi và chỉ khi có một số bằng 3 và hai số còn lại bằng 0.

408
Cty TNHH MTV DVVH Khang Việt

14
P ≤ f (3) = . Đẳng thức xảy ra khi và chỉ khi x= y= z= 1 .
3
5
Vậy giá trị nhỏ nhất của P bằng đạt tại x= 3, y= z= 0 hoặc các hoán vị ;
3
14
giá trị lớn nhất của P bằng đạt tại x= y= z= 1 .
3
=
Bài 3. Ta có (
P 3 x 2 y 2 + y 2 z 2 + z 2 x 2 + xy + yz + zx + 1 ≥ 1 . )
Đẳng thức xảy ra khi và chỉ khi x= y= 0; z= 1 hoặc các hoán vị.
 1 

n
Bài 4. HD : Đánh giá t= xy + yz + zx ∈  − ;1 .
 2 

.v
Bài 5. HD : Đánh giá t = x 2 + y 2 + z 2 ∈ [3;4] .

om
Bài 6. HD : Đặt t = x 2 + y 2 + z 2 ∈ [3;9 ) .
Bài 7. HD : Sử dụng đẳng thức kết hợp AM – GM ta có :
x3 + y 3 + z 3 = ( x + y + z ) − 3 ( x + y )( y + z )( z + x )
3
.c
= 27 − 3 ( x + y )( y + z )( z + x ) ≤ 27 − 24 xyz
ok
Đưa về khảo sát hàm số với t = xyz .
bo

Bài 8. Sử dụng bất đẳng thức C-S ta có :


( x + y + z ) ( x 2 + y 2 + z 2 )= (x 3
) ( ) ( )
+ xy 2 + y 3 + yz 2 + z 3 + zx 2 + x 2 y + y 2 z + z 2 x
et

(
≥ 2 x 2 y + 2 y 2 z + 2 z 2 x + x 2 y + y 2 z + z 2 x= 3 x 2 y + y 2 z + z 2 x )
vi

Do đó x y + y z + z
2 2 2
x≤ x+ y+z⇒P≥
( x + y + z − 1)
2
+
1
.
ng

x+ y+z x+ y+z
Đặt t = x + y + z . Ta có

( ) (
a

x2 + y 2 + z 2 < ( x + y + z ) ≤ 3 x2 + y 2 + z 2 ⇒ t ∈ 3;3 .
2
kh

( t − 1)2 + t với
Xét hàm số=
f (t )
t
t∈ ( 3;3 ta có

t 2 − 10
f '(t ) = 1 −
10
t2
=
t2
< 0, ∀t ∈ ( 3;3 .

Do đó f(t) là hàm nghịch biến trên ( 13


3;3 . Suy ra P ≥ f (t ) ≥ f (3) =
3
.

Đẳng thức xảy ra khi và chỉ khi x= y= z= 1 .

409
Khám phá tư duy Kỹ thuật giải bất ĐT Bài toán Max – Min – Đặng Thành Nam

13
Vậy giá trị nhỏ nhất của P bằng đạt tại x= y= z= 1 .
3
Bài 9. Ta có :

( )
3 x 2 + y 2 + z 2 =( x + y + z ) x 2 + y 2 + z 2 ( )
= (x 3
+ xy 2 )+(y 3
) ( )
+ yz 2 + z 3 + zx 2 + x 2 y + y 2 z + z 2 x

(
≥ 2 x 2 y + 2 y 2 z + 2 z 2 x + x 2 y + y 2 z + z 2 x= 3 x 2 y + y 2 z + z 2 x )
⇒ x2 y + y 2 z + z 2 x ≤ x2 + y 2 + z 2
Suy ra :

n
xy + yz + zx ( x + y + z ) − x2 − y 2 − z 2
2

.v
P≥x + y +z +
2 2 2
=x +y +z
2 2 2
+
x2 + y 2 + z 2 (
2 x2 + y 2 + z 2 )

om
.
9 − x2 − y 2 − z 2
= x2 + y 2 + z 2 +
(
2 x2 + y 2 + z 2 ) .c 9−t
Đặt t = x 2 + y 2 + z 2 , ( t ≥ 3) khi đó P ≥ f (t ) =
t+ ≥ f (3) =
4.
ok
2t
Đẳng thức xảy ra khi và chỉ khi x= y= z= 1 .
bo

Bài 10. HD : Đặt t = x + y + z , ( t ≥ 3) .


Bài 11. Sử dụng đẳng thức kết hợp bất đẳng thức AM – GM ta có :
et

( x + y )( y + z )( z + x ) = ( x + y + z )( xy + yz + zx ) − xyz
= ( x + y + z )( xy + yz + zx ) − 1
vi

.
≥ ( x + y + z ) 3 xyz ( x + y + z ) − 1
ng

= ( x + y + z ) 3( x + y + z ) − 1
a

72
Đặt t = x + y + z , ( t ≥ 3) khi đó P ≥ t 3t + −1.
kh

t +1
72
Xét hàm số f (t ) =t 3t + − 1 với t ≥ 3 ta có
t +1

3 3t ( t + 1) − 72
3
=f '(t ) > 0, ∀t ≥ 3 do đó f(t) là hàm đồng biến trên [3;+∞ ) .
2 ( t + 1) 3

44 . Đẳng thức xảy ra khi và chỉ khi x= y= z= 1 .


Vì vậy P ≥ f (t ) ≥ f (3) =
Vậy giá trị nhỏ nhất của P bằng 44 đạt tại x= y= z= 1 .

410
Cty TNHH MTV DVVH Khang Việt

Bài 12. Ta có
6 ( ab + bc + ca )= 3 ( a + b + c ) − a 2 − b 2 − c 2 = 3 ( a + b + c ) − 3 ( a + b + c ) − 4 .
2 2
 
Và theo bất đẳng thức AM – GM dạng luỹ thừa ta có :
1
a 3 + b3 + c 3 ≥ ( a + b + c ) .
3
9

Suy ra P≥
( a + b + c)
3

3( a + b + c )
− 3( a + b + c ) + 3( a + b + c ) + 4 .
2
9 3( a + b + c ) + 4

Theo giả thiết ta có : 3 ( a + b + c ) += (


4 3 a 2 + b2 + c2 ≥ ( a + b + c )) 2

n
⇔ ( a + b + c + 1)( a + b + c − 4 ) ≤ 0 ⇔ a + b + c ≤ 4 .

.v
3( a + b + c ) 4 4 3
Suy ra − =−1 + ≥ −1 + =− .

om
3( a + b + c ) + 4 3( a + b + c ) + 4 3.4 + 4 4

( a + b + c )3 − 3 13
Do đó P ≥ ( a + b + c )2 + 3 ( a + b + c ) +
.c .
9 4
Đặt t = a + b + c, ( 0 < t ≤ 4 ) khi đó :
ok
P≥
t3
9
13 1
4 9
(
− 3t 2 + 3t + = ( t − 4 ) t 2 − 23t − 65 −
923
36
≥−
923
36
)
, ∀0 < t ≤ 4 .
bo

4
Đẳng thức xảy ra khi và chỉ khi a= b= c= .
3
et

923 4
Vậy giá trị nhỏ nhất của P bằng − đạt tại a= b= c= .
36 3
vi

Bài 13. Theo giả thiết ta có − 3 ≤ x, y, z ≤ 3 .


ng

Do đó x + 2 > 0, y + 2 > 0, z + 2 > 0 .


Vì vậy P đạt giá trị nhỏ nhất khi và chỉ khi cả x,y,z đều âm.
Không mất tính tổng quát giả sử x = max { x, y, z} khi đó −1 ≤ x ≤ 0 và
a
kh

P= ( x + 2 ) ( yz + 2 ( y + z ) + 4 ) =
1
2
1
( )
( x + 2 ) ( y + z + 2 )2 + x 2 + 1 ≥ ( x + 2 ) x 2 + 1
2
1
( )
Xét hàm số f ( x) = ( x + 2 ) x 2 + 1 với x ∈ [ −1;0] ta có
2
 x = −1
3 2 1
f '( x) = x + 2 x + ; f '( x) =0 ⇔  .
2 2 x = − 1
 3
 1  25
Lập bảng biến thiên suy ra min f ( x) = f  −  = .
x∈[ −1;0]  3  27

411
Khám phá tư duy Kỹ thuật giải bất ĐT Bài toán Max – Min – Đặng Thành Nam

Đẳng thức xảy ra khi và chỉ khi


 1
x = − 3
  1 1 5  1 5 1
 y + z + 2 =0 ⇔ ( x; y; z ) = − ; − ; −  ;  − ; − ; −  .
 2  3 3 3  3 3 3
 x + y 2
+ z 2
=
3

25 1 5
Vậy giá trị nhỏ nhất của P bằng đạt tại hai biến bằng − và một biến bằng − .
27 3 3
Bài 14. Ta có
xy 2 + yz 2 + zx 2 y z x

n
+ = + +

.v
xyz z x y
x2 y y2 z z2 x y/z z/x x/ y
+ + + = + +

om
z3 x3 y 3
( z / x) 2
( x / y ) ( y / z )2
2

a b c 1 1 1
= 2
+ 2
+ 2
≥ + +.c
b c a a b c
= ab + bc + ca ≥ 3abc ( a + b + c ) = 3( a + b + c )
ok
13 x y z
⇒ S ≥ f (t ) = 3t + , t = a + b + c ≥ 3, a = , b = , c =
3t y z x
bo

3 13 40
+ f '(=
t) − 2 > 0, ∀t ≥ 3 ⇒ S ≥ f (t ) ≥ f (3) =
2 3t 3t 9
et

Bài tập tương tự


Cho x,y,z là các số thực dương. Tìm giá trị nhỏ nhất của biểu thức
vi

x5 y5 z5 9 xyz
P= + + + .
ng

y z 3 2
z x 3 2 3 2
x y 2( x y + y 2 z + z 2 x)
2

1 
Bài 15. Không mất tính tổng quát giả sử x = max { x, y, z} , khi đó x ∈  ;1 .
a

2 
kh

2
3 
Ta có P = x + y + z = x + ( y + z ) − 2 yz ≤ x + ( y + z )
2 2
2 2 2 2 2
= x +  − x .
2
2 
2
3  1 
Xét hàm số f ( x) = x 2 +  − x  trên đoạn  ;1 ta có
2  2 
3 1 5 3 9
f '( x) = 4 x − 3; f '( x) = 0 ⇔ x = . Ta có f =  f= (1) ; f = .
4 2 4 4 8
5 1
Do đó P ≤ đẳng thức xảy ra khi và chỉ khi= x =
, y 0,=z 1 và các hoán vị.
4 2

412
Cty TNHH MTV DVVH Khang Việt

Bài 16. Ta có : P = ( x + y + z ) − 2 ( xy + yz + zx ) = 100 − 2 xyz .


2

 y + z = 10 − x
 y + z = 10 − x 
Từ điều kiện ta có :  ⇒ x (10 − x ) .
 yz ( x − 1)= x ( y + z )= x (10 − x )  yz =
 x −1
2 x 2 (10 − x )
Suy ra =
P f (=
x) 100 − .
x −1
x (10 − x )
Mặt khác : ( y + z ) ≥ 4 yz ⇔ (10 − x ) ≥ 4.
2 2
x −1
⇔ ( x − 1)(10 − x ) ≥ 4 x ⇔ x − 7 x + 10 ≤ 0 ⇔ 2 ≤ x ≤ 5 .

n
2

.v
2 x 2 (10 − x )
Xét hàm số f (=
x) 100 − liên tục trên [ 2;5] ta có
x −1

om
=
f '(x)
(
2 x 2 x 2 − 13 x + 20 ) ; f '( x=) x∈[ 2;5]
0 ←

→
x=
5
2.
( x − 1)2 
x = 4
.c
Bảng biến thiên :
ok
t 5
2 4 5
2
bo

f’(t) + 0 − 0 +
f(t) 75 75
et

2 2
vi
ng

36 36
75 5
Từ bảng biến thiên suy ra giá trị lớn nhất của P bằng đạt tại x= 5, y= z= .
2 2
a

Giá trị nhỏ nhất của P bằng 36 đạt tại x= 2, y= z= 4 (chú ý chỉ có một trường
kh

hợp xảy ra dấu bằng ở bất đẳng thức ( y + z ) ≥ 4 yz trong mỗi trường hợp max
2

và min).

( ) ( )
2
Bài 17. Ta có P = x 4 + y 4 + z 4 = x 2 + y 2 + z 2 − 2 x2 y 2 + y 2 z 2 + z 2 x2
2
= ( x + y + z ) − 2 ( xy + yz + zx )  − 2 ( xy + yz + zx ) + 2 xyz ( xy + yz + zx )
2 2
 
x + y + z =
( )
4
Theo giả thiết ta có  , đặt t = xy + yz + zx ⇒ P = 2 t 2 − 32t + 144
 xyz = 2

413
Khám phá tư duy Kỹ thuật giải bất ĐT Bài toán Max – Min – Đặng Thành Nam

8
Ta có ( y + z )2 ≥ 4 yz ⇒ ( 4 − x )2 ≥ , giải bất phương trình này ta suy ra
x
3− 5 ≤ x ≤ 2.
2 2
Ta có t = x ( y + z ) + yz = x ( 4 − x ) +
, xét hàm số f ( x) = x ( 4 − x ) + trên
x x
 5 5 − 1
đoạn 3 − 5, 2  ta được t ∈ 5, 
 2 

 5 5 − 1
(
Tương tự xét hàm số f (t ) = 2 t 2 − 32t + 144 trên đoạn 5,
 2 
)
 ta có điều

n
phải chứng minh.

.v
Bài 18. Nhận xét. Để ý hai vế của bất đẳng thức nếu bỏ đi số 2014 ta có đẳng thức
và nếu thay t = 2014 ta được ngay một hàm số mà f (t ) ≤ f (0) .

om
t +a t +b t +c
Xét hàm số f (t ) = + + trên [ 0;+∞ ) .
t +b t +c t +a .c
Bất đẳng thức được chứng minh nếu chứng minh được f(t) là hàm nghịch biến
trên [ 0;+∞ ) .
ok
b−a c−b a−c b−c+c−a c−b a−c
f '(t ) = + + = + +
(t + b) 2
(t + c ) 2
(t + a ) 2
(t + b) 2
(t + c ) 2
( t + a )2
bo

 1 1   1 1 
= (a − c)  −  + ( b − c )  −  .
et

 ( t + a )2 ( t + b )2   ( t + b )2 ( t + c )2 

( a − c )( b − a )( 2t + a + b ) − ( b − c )2 ( 2t + b + c )
vi

( t + a )2 ( t + b )2 ( t + b )2 ( t + c )2
ng

Để f '(t ) ≤ 0 ta chỉ cần giả sử a = max {a, b, c} và bài toán được chứng minh.
a

Đẳng thức xảy ra khi và chỉ khi f '(t ) = 0, ∀t ≥ 0 ⇔ a = b = c .


kh

1 1 1  1 1
Bài 19. Xét hàm số f ( x) = ( x + y + z )  + +  − (1 + y + z ) 1 + +  trên
x y z  y z
[1;4] ta có:
1 1 1 x+ y+z
f '( x) = + + −
 1 1  x − yz ( y + z )
= ( y + z ) − 2  =
2
≤0.
( )
x y z x2  yz x  x 2 yz
Do đó f(x) là hàm nghịch biến trên [1;4] suy ra f ( x) ≤ f (1) =
0 hay

414
Cty TNHH MTV DVVH Khang Việt

1 1 1  1 1
( x + y + z) + +  ≤ (1 + y + z ) 1 + +  .
x y z  y z
Bất đẳng thức được chứng minh. Đẳng thức xảy ra khi và chỉ khi x = 1 .
1 1 1
Nhận xét. Để tìm giá trị lớn nhất của P = ( x + y + z )  + +  với cùng điều
x y z
 1 1
kiện ta xét tiếp hàm số g ( y ) = (1 + y + z ) 1 + +  (xem thêm chủ đề kỹ thuật
 y z
khảo sát hàm nhiều biến).

n
Bài 20. Ta có ab + bc + ca ≥ 3 3 (abc)2 .

.v
(1 + a )(1 + b )(1 + c ) =1 + ab + bc + ca + a + b + c + abc ≥ (1 + 3 abc )
3

om
.

2 t
Khi đó P ≤ + = ,t 3
abc ,0 < t ≤ 1 .
3 + 3t 2t +1 .c
f (t )
Xét hàm số = +
t 2
′(t )
, 0 < t ≤ 1 ⇒ f=
(
t ( t + 1) 3t 2 + t − 1
.
)
ok
3 + 3t 2 t + 1
( )
2
3 ( t + 1) 1 + t 2
2
bo

5
Từ đó suy ra MaxP = ⇔ t =1 ⇔ a = b = c =1.
6
Bài 21. Ta có:
et

x + y + z +1 x + y + z +1 x + y + z +1
= P+3 + +
x +1 y +1 z +1
vi

 1 1 1 
( x + y + z + 1) 
ng

= + + 
 x +1 y +1 z +1
Tìm giá trị nhỏ nhất của P.
a

Sử dụng bất đẳng thức C-S ta có:


kh

1 1 1 9
+ + ≥ .
x +1 y +1 z +1 x + y + z + 3
9
Suy ra P + 3 ≥ ( x + y + z + 1) . .
x+ y+ z+3
3  9 ( t + 1)
Đặt t = x + y + z , t ∈  ;3 khi đó P ≥ f (t=
) −3.
2  t +3
9 ( t + 1) 3 
=
Xét hàm số f (t ) − 3 liên tục trên đoạn  ;3 ta có:
t +3 2 

415
Khám phá tư duy Kỹ thuật giải bất ĐT Bài toán Max – Min – Đặng Thành Nam

18 3  3 
=
f '(t ) > 0, ∀t ∈  ;3 do đó f(t) đồng biến trên  ;3 .
( t + 3) 2
2  2 

3 1
Suy ra P ≥ f (t ) ≥ f   = 5 . Đẳng thức xảy ra khi và chỉ khi x= y= z= .
2 2
1
Vậy giá trị nhỏ nhất của P bằng 5 đạt tại x= y= z= .
2
Tìm giá trị lớn nhất của P.
Ta có:

(1 − x )  x −
1 3 1
 ≥ 0 ⇔ − x + x − ≥ 0 ⇔ −2 x + 3x − 1 ≥ 0
2 2

n
 2 2 2

.v
1 x 5
⇔ ( x + 1)( −2 x + 5 ) ≥ 6 ⇔ ≤− +
x +1 3 6

om
1 y 5 1 z 5
Tương tự ta có: ≤− + ; ≤− + .
y +1 3 6 z +1 3 6
Cộng theo vế 3 bất đẳng thức trên ta được:
.c
1 1 1 x+ y+z 5
+ + ≤− + .
ok
x +1 y +1 z +1 3 2
 x+ y+ z 5
Suy ra P ≤ ( x + y + z + 1)  −
bo

+ .
 3 2
3   t 5 1
Đặt t = x + y + z , t ∈  ;3 ta được: P ≤ ( t + 1)  − + = ( 2t − 7 )( 3 − t ) + 6 ≤ 6 .
et


2   3 2 6
Đẳng thức xảy ra khi và chỉ khi x= y= z= 1 .
vi

Vậy giá trị lớn nhất của P bằng 6 đạt tại x= y= z= 1 .


ng

Bài 22. Gọi biểu thức vế trái của bất đẳng thức là P ta có
P= abc ( a + b + c ) − 2 ( ab + bc + ca ) = 9abc − 2abc ( ab + bc + ca ) .
2
a

 
kh

Ta có ( ab + bc + ca ) ≥ 3abc ( a + b + c ) =
2
9abc .
Suy ra
( ab + bc + ca )2 9 − 2 −2 ( ab + bc + ca )
3
P≤  (=
ab + bc + ca )  + ( ab + bc + ca ) .
2
9 9
−2t 3 2
Đặt t = ab + bc + ca suy ra P ≤ f (t ) = +t .
9
−2t 3 2
Xét hàm số =
f (t ) + t với t ∈ ( 0;3] ta có
9

416
Cty TNHH MTV DVVH Khang Việt

2t 2
f '(t ) =− + 2t ; f '(t ) =0 ⇔ t =3 .
3
Ta có f’(t) đổi dấu từ âm sang dương khi đi qua t = 3 nên f(t) đạt cực đại tại
t = 3 . Suy ra P ≤ f (t ) ≤ f (3) = 3 . Đẳng thức xảy ra khi và chỉ khi a= b= c= 1 .

( )
Bài 23. Ta có: 2 yz = ( y + z ) − y 2 + z 2 = x 2 − 1 − x 2 = 2 x 2 − 1 .
2
( )
2
12 2 1
Suy ra P = x  x −  = x 6 − x 4 + x 2 .
 2 4

Mặt khác: ( y + z ) ≥ 4 yz ⇔ x 2 ≥ 2 2 x 2 − 1 ⇔ 0 ≤ x 2 ≤
2
( ) 2
.

n
3

.v
  2 1
Đặt
= t x 2 ,  t ∈  0;   và P = f (t ) = t 3 − t 2 + t .
  3  4

om
1  2
Xét hàm số f (t ) = t 3 − t 2 + t liên tục trên đoạn 0; 3  ta có
4  
1 1 1
f '(t ) = 3t 2 − 2t + ; f '(t ) = 0 ⇔ t = hoặc t = .
4 6 2
.c
ok
1 1 2 1
Ta có =
f (0) f=  0, f=   f=   .
2 6  3  54
bo

1 1
Suy ra max f (t ) = . Do đó P ≤ đẳng thức xảy ra khi
 2 54 54
t∈ 0;  3 
et

2 1
x= ,y= z = −
vi

.
3 6
ng

1 2 1
Vậy giá trị lớn nhất của P bằng đặt tại x = ,y= z = − .
54 3 6
a

Bài 24. Nếu a 2 + b 2 + c 2 =0 ⇔ a =b =c =0 bất đẳng thức trở thành đẳng thức.
kh

+ Nếu a 2 + b 2 + c 2 > 0 viết lại bất đẳng thức dưới dạng :


(
6 ( a + b + c ) a 2 + b 2 + c 2 − 27 abc ) ≤ 10 .
(a 2
+b +c
2 2 3
)
a b c
Đặt x
= = ,y = ,z ⇒ x2=
+ y2 + z2 1 .
a 2 + b2 + c2 a 2 + b2 + c2 a +b +c
2 2 2

10
Bất đẳng thức trở thành: 6 ( x + y + z ) − 27 xyz ≤ 10 ⇔ 2 ( x + y + z ) − 9 xyz ≤ .
3

417
Khám phá tư duy Kỹ thuật giải bất ĐT Bài toán Max – Min – Đặng Thành Nam

Không mất tính tổng quát=


giả sử x 2 max x 2 , y 2 , z 2 ⇒ x 2 ≥ =
3
{
x2 + y 2 + z 2 1
3
. }
Ta có
( x ( 2 − 9 yz ) + 2 ( y + z ) )2 ≤  x2 + ( y + z )2  . ( 2 − 9 yz )2 + 4= ( 2 yz + 1) (81y 2 z 2 − 36 yz + 8) .
y 2 + z 2 1 − x2 1
Đặt t = yz ⇒ t ≤ = ≤ .
2 2 3

(  1 1
Khi đó P ≤ f (t ) = ( 2t + 1) 81t 2 − 36t + 8 với t ∈  − ;  ta có:
 3 3
)

n
 2
 t= −

.v
9
f '(t ) =486t 2 + 18t − 20; f '(t ) =0 ⇔  .
t = 5

om
 27
 1  29  2  100  5  1369  1  25
Ta có f  − =
 ; f − =
 ;f =  ;f=  .
 3 3  9 9  27  243  3  3 .c
 2  100 100 10
Suy ra Max f (t ) = f  −  = ⇒ P2 ≤ ⇒P≤ .
ok
 1 1
t∈ − ;  9  9 9 3
 3 3 

Đẳng thức xảy ra chẳng hạn tại a =


−1, b =
c=2.
bo

Bài 25. Ta có:


( a + b )( b + c )( c + a ) = ( a + b + c )( ab + bc + ca ) − abc = 3 ( a + b + c ) − abc .
et

1 4
Khi đó=
P + .
abc 3 ( a + b + c ) − abc
vi
ng

3
Mặt khác ( ab + bc + ca ) ≥ 3abc ( a + b + c ) ⇒ a + b + c ≤
2
.
abc
1 4 1 4abc
a

Suy ra P ≥ + = + .
− abc abc 9 − ( abc )
abc 9 2
kh

abc
1 4t
Đặt
= t abc, ( 0 < t ≤ 1) khi đó P ≥ f (t) =+ .
t 9 − t2
1 4t
Xét hàm số f (t)= + trên ( 0;1] ta có:
t 9 − t2

( ) ( )
2
1 4t 2 + 36 t 2 4t 2 + 36 − 9 − t 2
f '(t ) =− + = < 0, ∀t ∈ ( 0;1]
t2
( ) (9 − t )
2 2 2
9 − t2 t2

418
Cty TNHH MTV DVVH Khang Việt

3
nên f(t) là hàm nghịch biến trên ( 0;1] . Do đó f (t ) ≥ f (1) = .
2
3
Vậy giá trị nhỏ nhất của P bằng đạt tại a= b= c= 1 .
2
Cách 2: Ta có thể đánh giá nhanh thông qua bất đẳng thức AM-GM như sau:
1 1 4 1 1 4
P= + + ≥ + +
2abc 2abc ( a + b )( b + c )( c + a ) 2 2abc ( a + b )( b + c )( c + a )
1 2 1 2 3
≥ +2 ≥ +2 =
2 ( ac + bc )( ab + ac )( bc + ab ) 2  2ab + 2bc + 2ca 
3 2
 

n
 3 

.v
3
Vậy giá trị nhỏ nhất của P bằng xảy ra khi a= b= c= 1 .
2

om
Bài 26. Không mất tính tổng quát giả sử e = min {a, b, c, d , e} viết lại biểu thức P
= bc ( a + d − e ) + e ( a + c )( b + d ) .
dưới dạng: P
Sử dụng bất đẳng thức AM-GM ta được:
.c
3 3
 b + c + a + d − e   1 − 2e 
ok
bc ( a + d − e ) ≤   =
  và
 3   3 
bo

2 2
a+c+b+d  1− e 
e ( a + c )( b + d ) ≤ e   =
e  .
 2   2 
et

3 2
 1 − 2e  1− e 
Suy ra P ≤   + e  .
 3   2 
vi

1
Với a, b, c, d , e > 0 và a + b + c + d + e = 1, e = min {a, b, c, d , e} ⇒ 0 < e ≤ .
ng

5
3 2
 1 − 2e  1− e   1
Xét hàm=
số f (e)   + e  trên  0;  ta được:
a

 3   2   5
kh

f '(e=
)
1
36
(
−5e2 − 4e + 1=
1
)
36
(1 − 5e )( e + 1) ≥ 0, ∀e ∈  0;  nên f(e) là hàm
 5
1

 1 1 1
đồng biến trên  0;  . Do đó f (e) ≤ f   = .
 5  5  25
1 1
Vậy giá trị lớn nhất của P bằng đạt tại a= b= c= d= e= .
25 5
a b c
Bài 27. Ta có: + +
( a + b )( a + c ) ( b + c )( b + a ) ( c + a )( c + b )

419
Khám phá tư duy Kỹ thuật giải bất ĐT Bài toán Max – Min – Đặng Thành Nam

2 ( ab + bc + ca ) 6
= .
( a + b )( b + c )( c + a ) ( a + b )( b + c )( c + a )

2 ( a + b + c ) − a 2 − b 2 − c 2= 2 ( a + b + c ) − ( a + b + c ) + 2 ( ab + bc + ca )
2

= 2(a + b + c) − (a + b + c) + 6
2

Mặt khác:
( a + b )( b + c )( c + a ) = ( a + b + c )( ab + bc + ca ) − abc
1
≥ ( a + b + c )( ab + bc + ca ) −
( a + b + c )( ab + bc + ca )

n
9

.v
8 8
Suy ra: ( a + b )( b + c )( c + a ) ≥ ( a + b + c )( ab + bc + ca= ) (a + b + c) .

om
9 3
9
+ 2(a + b + c) − (a + b + c) + 6 .
2
Suy ra P ≤
4(a + b + c)

(
Đặt t = a + b + c, t ≥ 3 ( ab + bc + ca ) = 3 ⇒ P ≤ f (t ) =)
.c 9
+ 2t − t 2 + 6 .
ok
4t
9
Xét hàm số f (t ) = + 2t − t 2 + 6 với t ≥ 3 ta có:
4t
bo

9
f '(t ) =− + 2 (1 − t ) < 0, ∀t ≥ 3 . Do đó f(t) là hàm nghịch biến trên [3;+∞ ) .
4t 2
et

15
Suy ra P ≤ f (t ) ≤ f (3) = . Đẳng thức xảy ra khi và chỉ khi a= b= c= 1 .
vi

4
15
đạt tại a= b= c= 1 .
ng

Vậy giá trị lớn nhất của P bằng


4
Bài 28. Sử dụng bất đẳng thức AM-GM ta có:
a

1 3 1
a 2 ( a + 1) + b 2 ( b + 1) + c 2 ( c + 1) = a3 + b3 + c3 + a 2 + b 2 + c 2 ≥ ( a + b + c ) + ( a + b + c ) .
2
kh

9 3

( )
Đặt t =a + b + c, t ∈ 0;3 3  ⇒ P ≤ f (t ) =−
8
t +1
t3 t 2
+ 31 − − .
9 3

8 t3 t 2
Xét hàm số f (t ) =
− + 31 − − liên tục trên đoạn 0;3 3  ta có:
t +1 9 3
 
1 8 t (t + 2) .
=f '(t ) −
2 −t − 3t + 279 
 ( t + 1)3 3 2

420
Cty TNHH MTV DVVH Khang Việt

Ta có: f '(t ) = 0 ⇔ 8 −t 3 − 3t 2 + 279 − t ( t + 2 ) ( t + 1)3 ⇔ t = 3 (do vế trái là


hàm số nghịch biến).
Ta có f '(t ) đổi dấu từ dương sang âm khi đi qua t = 3 nên f(t) đạt cực đại tại
t = 3 . Do đó P ≤ f (t ) ≤ f (3) =
3.
Vậy giá trị lớn nhất của P bằng 3 đạt tại a= b= c= 1 .
Bài 29. Tìm giá trị nhỏ nhất của P
Ta có: x 4 − 3 x3 − 9 x ( x − 3) = x ( x − 3)
2
( x + 3) ≥ 0, ∀ x ∈ [0;3] .
9 9 27

n
Nếu ab + bc + ca ≥ ⇒ P ≥ 3. = > 3.
5 5 5

.v
9
Xét với ab + bc + ca ≤ ta có:

om
5
a 4 + b4 + c4 a ( a − 3) + b ( b − 3) + c ( c − 3)
≥ 3 + 9.
a +b +c
3 3 3
a 3 + b3 + c 3 .c
a 2 + b2 + c2 − 3( a + b + c ) a 2 + b2 + c2 − ( a + b + c )
2
= 3 + 9. = 3 + 9.
ok
a 3 + b3 + c 3 a 3 + b3 + c 3
ab + bc + ca ab + bc + ca ab + bc + ca
= 3 − 18. 3 3 3 ≥ 3 − 18. 3 3 3 = 3 − 18.
a +b +c a + b + c − 3abc 3. 9 − 3 ( ab + bc + ca ) 
bo

ab + bc + ca
= 3 + 2.
ab + bc + ca − 3
et

ab + bc + ca
Vậy P ≥ 3 + 2. + 3 ( ab + bc + ca ) .
vi

ab + bc + ca − 3
ng

  9  2t 6
Đặt t = ab + bc + ca,  t ∈ 0;   khi đó P ≥ f (t ) = 3 + + 3t = 3t + 5 + .
  5  t −3 t −3
a

6
Xét hàm số f (t ) =3t + 3 + ta có:
kh

t −3
3 4t − 6 3
f '(t ) =− +3= ; f '(t ) =0 ⇔ t = .
6  6  2
5+
t −3
( t − 3)  5 + + 1
 t −3 

421
Khám phá tư duy Kỹ thuật giải bất ĐT Bài toán Max – Min – Đặng Thành Nam

Bảng biến thiên:


t 3 9
0
2 5
f’(t) + 0 −
f(t) 11
2

27
3
5

n
Dựa vào bảng biến thiên suy ra P ≥ f (0) =
3.

.v
Vậy giá trị nhỏ nhất của P bằng 3 đạt tại a= 3, b= c= 0 hoặc các hoán vị.

om
Bài 30. Không mất tính tổng quát giả sử c = min {a, b, c} khi đó

b 2 − bc + c 2 =b 2 + c ( c − b ) ≤ b 2 , c 2 − ca + a 2 =a 2 + c ( c − a ) ≤ a 2 .
Suy ra:
.c
a+b+c a+b
ok
1 1 1
P≥ = + + +
a 2 − ab + b 2 a b ( a + b )2 − 3ab ab
bo


a+b
+
( a + b)
=
2
1
+
( a + b)
2

ab
( a + b )2 − 3ab ab 1− 3 ab
et

( a + b) 2

 1
vi

ab 1 1
Đặt t
= ,  0 < t ≤  khi đó P ≥ f (t=
) + .
(a + b) 2
 4 1 − 3t t
ng

1 1  1
Xét hàm số=
f (t ) + trên  0;  ta có
1 − 3t t  4
a

3 1
kh

− 2.
1 3t − 2 (1 − 3t )
2 3
= =
3
− 2 ≤ 16 4 < 0, ∀t ∈  0; 1  .
f '(t )  
2 (1 − 3t )
3 t 2t 2 (1 − 3t )
3
2t 2 (1 − 3t )
3  4

 1 1
Suy ra f(t) nghịch biến trên  0;  do đó P ≥ f (t ) ≥ f   = 6.
 4 4
1
Vậy giá trị nhỏ nhất của P bằng 6 đạt tại a= b= , c= 0 hoặc các hoán vị.
2
Bài 31. Không mất tính tổng quát giả sử b là số nằm giữa a và c.
Khi đó:

422
Cty TNHH MTV DVVH Khang Việt

( b − a )( b − c ) ≤ 0 ⇔ b2 + ac ≤ ab + bc ⇒ b2c + c 2 a ≤ abc + bc 2
( )
.
⇒ a 2b + b 2 c + c 2 a ≤ a 2b + abc + bc 2 = b a 2 + ac + c 2

abc abc ac
Suy ra ≥ = .
a b+b c+c a
2 2 2
(
b a + ac + c
2 2
) a + ac + c 2
2

Ta cần đánh giá


(
2 a 2 + b2 + c2 ) và ac
.
a+b+c a + ac + c 2
2

(
2 a 2 + b2 + c2 ) ab + bc + ca

n
Ta có: = 2 − 4. .
a+b+c ( a + b + c )2

.v
Sử dụng bất đẳng thức AM-GM ta có:

om
( ) ( )
2
4 ( ab + bc + ca ) a 2 + ac + c 2 ≤ ab + bc + ca + a 2 + ac + c 2
2
.
= b ( a + c ) + ( a + c )  = (a + b + c) (a + c)
2 .c 2 2
 
( a + c )2
ok
ab + bc + ca
Suy ra ≤ .
( a + b + c )2 (
4 a 2 + ac + c 2 )
bo

Do đó

( a + c )2 3ac ac 3ac
et

P≥ 2− + =
1− 2 + .
a + ac + c
2 2
(
4 a 2 + ac + c 2 ) a + ac + c 2
(
4 a 2 + ac + c 2 )
vi

ac  2 
Đặt t = 1− ,  ≤ t ≤ 1 khi đó:
ng

a + ac + c  3
2 2

P ≥ f (t ) =t +
3
( 3
) 3
1 − t 2 =− t 2 + t + .
a

4 4 4
kh

3 3  2 
Xét hàm số f (t ) =− t 2 + t + liên tục trên đoạn  ;1 ta có
4 4  3 
3  2   2 
f '(t ) =− t + 1 < 0, ∀t ∈  ;1 . Do đó f(t) nghịch biến trên  ;1 .
2  3   3 
Suy ra P ≥ f (t ) ≥ f (1) = 1 . Đẳng thức xảy ra khi và chỉ khi=
a b=
,c 0 .
Vậy giá trị nhỏ nhất của P bằng 1 đạt tại=
a b=, c 0 hoặc các hoán vị.
Bài 34. Đây là một biểu thức hoán vị cần tìm cách đánh giá chuyển về biểu thức
đối xứng cách thực hiện là xét phần tương ứng với P cụ thể là

423
Khám phá tư duy Kỹ thuật giải bất ĐT Bài toán Max – Min – Đặng Thành Nam

Q = x2 y + y 2 z + z 2 x .
Theo giả thiết ta có
 ( x + y + z )2 − x 2 − y 2 − z 2 = 4
 xy + yz + zx = −
 2 3
 3 3 3
 x + y + z = ( x + y + z ) − 3 ( x + y )( y + z )( z + x ) = 3xyz
3
.

 x3 y 3 + y 3 z 3 + z 3 x3 = ( xy + yz + zx )3 − 3  xyz ( x + y + z )( xy + yz + zx ) − x 2 y 2 z 2  = 3x 2 y 2 z 2 − 64
   27

Khi đó

n
P + Q = xy ( x + y ) + yz ( y + z ) + zx ( x + z ) = −3 xyz

.v


3 3 3 3
(
 P.Q= x y + y z + z x + xyz x + y + z + 3 x y z = 9 x y z −
3 3 3 3 3 2 2 2
)
2 2 2 64 .

om
27
Theo định lý Vi-ét P,Q là 2 nghiệm của phương trình
64
x 2 + 3tx + 9t 2 − = 0 với t = xyz . .c
27
256 256
ok
−3t +
− 27t 2 −3t − − 27t 2
Suy ra P = 27 hoặc P = 27 .
2 2
bo

Khảo sát P như trên ta tìm được Max và Min.


Bài tập tương tự
et

Cho x,y,z là các số thực thỏa mãn x + y + z =5 và x 2 + y 2 + z 2 =


9 . Tìm giá trị
lớn nhất và nhỏ nhất của biểu thức P = x 2 y + y 2 z + z 2 x .
vi

( a + c )2 − 2ac =
1
ng

2
  6 − b2 
Bài 35. Theo giả thiết ta có  ⇒=
2ac   − 1 .
6 − b2
a + c =  2b 
 2b
a

  
kh

2 2
  6−b 
2 2 2 2
(
Do đó P = b ( c − a ) = b c + a − 2ca = b 1 − 

2 2

 2b  

)
− 1  ≤ 16 . 2

    
2
b+c
Bài 36. Ta có P =a + (a + 1)(b + c) + bc ≤ a + (a + 1)(b + c) +   .
 2 
Ta chỉ cần rút được b+c theo a.
 x + 2 = a + 2b + 1
Theo giả thiết ta có  ⇒ x ∈ [ −1;1] ta có
2 − x = a + 2c + 1

424
Cty TNHH MTV DVVH Khang Việt


a + b + c = x + 3
2
 a + b + c = x 2 + 3
b − c 4 x ⇔ .
 2 b − c =
 4x
 a + 2 ( b + c + 1) a + ( 2b + 1)( 2c + 1) = 4 − x
2

Ta có
4 ( a + b + c ) − 3 ( b − c ) = 12 ( ab + bc + ca ) − ( 2a − b − c ) ≥ 12 ( ab + bc + ca ) .
2 2 2

Do đó
1 1
P = a + b + c + ab + bc + ca ≤ a + b + c + ( a + b + c )2 − ( b − c )2
3 4

n
x2 x2 − 3 ( )
1 2
( x4
)
2
+ 6 ≤ 6, ∀x ∈ [ −1;1]

.v
= x + 3 + x + 3 − 4x =
2 2
− x + 6=
2
3 3 3

om
Dấu bằng xảy ra khi và chỉ khi a= b= c= 1 .
Bài 37. Theo giả thiết ta có:
( x + y + z ) ( x 2 + y 2 + z 2 − xy − yz − zx ) =2 .c
3
( 1 2
⇔ ( x + y + z )  x2 + y 2 + z 2 − ( x + y + z )  =2.)
ok
2 2 

⇔ x +=
y +z
2 2 2 ( x + y + z )2 + 4
≥34
bo

3 3( x + y + z )
x2 + 2 y 2 + z 2 x2 + y 2 + z 2 1 1
Khi đó P = ≥ =1− ≥1−
et

.
x + y + z +1
2 2 2
x + y + z +1
2 2 2
x + y + z +1
2 2 2
1+ 3 4
vi

1
Với x= 3
2, y= z= 0 thì P = 1 − .
1+ 3 4
ng

Bài tập tương tự


Cho x,y,z là các số thực không âm thoả mãn điều kiện x3 + y 3 + z 3 =2 + 3 xyz .
a

Tìm giá trị nhỏ nhất của biểu thức P =x 2 + 2 y 2 + 3 z 2 .


kh

Bài 38. Tìm giá trị nhỏ nhất


Không mất tính tổng quát giả sử a = min {a, b, c} ⇒ a ≤ 1 ⇒ b + c = 3 − a ≥ 2 .
Khi đó
( b − 1)3 + ( c − 1)3 = ( b + c − 2 ) ( b − 1)2 − ( b − 1)( c − 1) + ( c − 1)2 
 ( b − 1 + c − 1)2 ( b − 1 + c − 1)2  ( b + c − 2 )3
≥ (b + c − 2)  − =
 2 4  4

425
Khám phá tư duy Kỹ thuật giải bất ĐT Bài toán Max – Min – Đặng Thành Nam

Suy ra
1 1 3 3
P ≥ ( a − 1) +
3
( b + c − 2 )3 =( a − 1)3 + (1 − a )3 = ( a − 1)3 ≥ − , ∀a ∈ [0;1] .
4 4 4 4
3 3
Vậy giá trị nhỏ nhất của P bằng − đạt tại a= 0, b= c= hoặc các hoán vị.
4 2
+ Tìm giá trị lớn nhất.
Sử dụng đẳng thức x3 + y 3 + z 3 = ( x + y + z ) − 3 ( x + y )( y + z )( z + x ) .
3

Ta có: P = ( a + b + c − 3)3 − 3 ( a + b − 2 )( b + c − 2 )( c + a − 2 ) .
=−3 ( a + b − 2 )( b + c − 2 )( c + a − 2 )

n
⇒ P 2= 9 ( b + c − 2 ) ( a + b − 2 ) ( c + a − 2 )
2 2 2

.v
Ta có b + c − 2 ≥ 0, ( a + b − 2 ) + ( a + c − 2 ) = a − 1 ≤ 0 .

om
+ Nếu cả a + b − 2 ≤ 0, a + c − 2 ≤ 0 ⇒ P ≤ 0 .
+ Nếu tồn tại một thừa số dương 1 thừa số âm ta giả sử a + b − 2 ≤ 0, a + c − 2 ≥ 0
khi đó
P= 3 ( b + c − 2 )( 2 − b − a )( a + c − 2 )
.c
ok
2
b+c−2+a+c−2
≤ 3( 2 − b − a )  
 2 
bo

3 3
= ( c − 1) ≤ . ( 3 − 1) = 6
3 3
4 4
et

Đẳng thức xảy ra khi và chỉ khi a= b= 0, c= 3 .


Vậy giá trị lớn nhất của P bằng 6.
vi

=
Bài 39. Giả sử c min {a, b, c} ⇒ 0 ≤ c < a, b khi đó
ng

 1 1 1
P ≥ ( a + b ) + a 2 + b 2  
2
+ + .
   ( a − b )2 a 2 b 2 
a

 
kh

Ta có:
 1 a 2 + ab + b 2 a b
2
a b
( a + b ) 2 + a 2 + b 2   1 +
1
= +  + + + 2 + 
   (a − b)
2. 2  

2
a 2
b 
2
(a − b) 2
b a b a
a b
+1+ 2
= 2. b a + 2 a + b  + 2 a + b 
   
a
−2+
b b a b a
b a
a b  t +1 
Đặt t =+ , ( t > 2 ) khi đó P ≥ f (=
t) 2 t 2 + t + .
b a  t −2

426
Cty TNHH MTV DVVH Khang Việt

 t +1 
Xét hàm số f (=
t) 2 t2 + t +  trên ( 2;+∞ ) ta có
 t −2
 3 
 ; f '(t ) = 0 ⇔ ( 2t + 1)( t − 2 ) − 3 = 0 .
3
f '(t ) = 2  2t + 1 −
 ( t − 2 ) 
2

t >2 5 + 33
⇔ 2t 3 − 7t 2 + 4t +=
1 0 ←
→=t .
4
5 + 33
Tại t = thì f’(t) đổi dầu từ âm sang dương nên f(t) đạt cực tiểu tại
4

n
5 + 33  5 + 33  59 + 11 33
hay f (t ) ≥ f   =.

.v
4  4  4

59 + 11 33 a b 5 + 33

om
Vậy giá trị nhỏ nhất của P bằng đạt tại =+ = ,c 0 .
4 b a 4

CHỦ ĐỀ 4: KỸ THUẬT SỬ DỤNG TÍNH THUẦN NHẤT


.c
ok
A. NỘI DUNG PHƯƠNG PHÁP
Xét một biểu thức P ( x1 ; x2 ;...; xn ) được gọi là biểu thức thuần nhất bậc k khi
bo

P ( tx1 ; tx2 ;...; txn ) = t k P ( x1 ; x2 ;...; xn ) .


Như đã đề cập trong chủ đề Bất đẳng thức AM – GM với bất đẳng thức thuần nhất
et

ta có thể chuẩn hoá điều kiện của các biến để chứng minh cho đơn giản. Với dạng
toán này chúng ta tiếp cận đưa về bài toán ít biến số hơn bất đẳng thức ban đầu và
vi

kết hợp sử dụng tính đơn điệu của hàm số.


ng

 PHƯƠNG PHÁP
Phương pháp chung là giảm bài toán n biến về (n-1) biến hoặc nhỏ hơn bằng
các phép đặt ẩn phụ.
a

Bằng cách đặt


kh

x2 = t1 x1
x3 = t2 x2
.
...
xn = tn −1 xn −1
Ta đưa về bài toán với (n-1) biến t1 , t2 ,..., tn −1 .
Dấu hiện nhận diện:
- Giả thiết bài toán có dạng thuần nhất.
- Biểu thức của P thuần nhất.
427
Khám phá tư duy Kỹ thuật giải bất ĐT Bài toán Max – Min – Đặng Thành Nam

1) Kỹ thuật đặt x = ty hoặc y = tx cho trường hợp bất đẳng thức và bài toán
cực trị hai biến số.
Ví dụ 1. Cho x,y là các số thực dương thỏa mãn điều kiện x ≥ 2y .
x2 + y2
Tìm giá trị nhỏ nhất của biểu thức P = .
xy
Lời giải
Đặt x = ty từ x ≥ 2y ⇒ t ≥ 2 và ta được:
t 2 + 1 t 2 + 1 5 5 2t 2 − 5t + 2 5
P
=
= − += +
t t 2 2 2t 2

n
( t − 2 )( 2t − 1) + 5 ≥ 0, ∀t ≥ 2

.v
=
2t 2

om
5
Vậy giá trị nhỏ nhất của P bằng đạt tại x = 2y .
2
Nhận xét. Ta có thể xử lý bằng cách khác như sau: .c
Với dự đoán dấu bằng xảy ra tại x = 2y ta viết lại P dưới dạng:
ok
3x 2 x 2 + 4y 2 3 x x 2 + 4y 2 3 4xy 5
P= + = . + ≥ .2 + = .
4xy 4xy 4 y 4xy 4 4xy 2
bo

Trong trường hợp không dự đoán được dấu bằng ta sử dụng kỹ thuật đẳng cấp
như trên ta dễ dàng tìm được giá trị lớn nhất, nhỏ nhất của biểu thức của P với
et

việc coi P là hàm của t.


Ví dụ 2. (TSĐH Khối B 2006) Cho hai số thực x, y thay đổi và thỏa mãn điều
vi

kiện x 2 + y 2 =
1.
ng

2 ( x 2 + 6 xy )
Tìm giá trị lớn nhất và giá trị nhỏ nhất của biểu thức P = .
1 + 2 xy + 2 y 2
a

Lời giải
kh

Nhận xét. Biểu thức P chưa đồng bậc nhưng để ý nếu thay x 2 + y 2 =
1 vào biểu
thức của P thì ta đưa P về dạng đồng bậc.
2 ( x 2 + 6 xy ) 2 ( x 2 + 6 xy )
=
Viết lại biểu thức P dưới dạng: P = .
x 2 + y 2 + 2 xy + 2 y 2 x 2 + 3 y 2 + 2 xy
TH1: Nếu y = 0 ta có P = 2 .
2 ( t 2 + 6t )
TH2: Nếu y ≠ 0 ta đặt x = t.y ta có:=
P f=
(t ) .
t 2 + 2t + 3

428
Cty TNHH MTV DVVH Khang Việt

2 ( t 2 + 6t )
Xét hàm số f (t ) = trên  ta có
t 2 + 2t + 3
4 ( 2t 2 − 3t − 9 ) 
t= −
3
f '(t ) =
− ; f '(t ) =
0⇔  2.
( t 2 + 2t + 3) 
2

t = 3
Bảng biến thiên:
t 3
−∞ − 3 +∞
2
− + −

n
f’(t) 0 0
f(t) 2 3

.v
om
−6 2
Dựa vào bảng biến thiên suy ra P=
max
f(3)
= 3 đạt tại .c
 3 1 
3y ⇔ ( x; y ) =
x= ± ;± .
ok
 10 10 
 3
Pmin =
P−  =−6 đạt tại
bo

 2 
3  3 2   3 2 
− y ⇔ ( x; y ) =
et

x= − ; ; ;− .
2  13 13   13 13 
vi

Ví dụ 3. Giả sử x, y là các số thực dương thỏa mãn điều kiện x + y ≤ 2.


ng

Tìm giá trị lớn nhất của biểu thức P = 7( x + 2 y ) − 4 x 2 + 2 xy + 8 y 2 .


Lời giải
a

Vì x, y là các số thực dương nên


kh

 7( x + 2 y ) − 4 x 2 + 2 xy + 8 y 2 
P= ( x + y )  
 x+ y 
 7 y − 4 x 2 + 2 xy + 8 y 2 
= ( x + y)  7 + . (1)
 x+ y 
 
x 7 y − 4 x 2 + 2 xy + 8 y 2 7 − 4 t 2 + 2t + 8
=
Đặt t , t > 0 khi đó = . (2)
y x+ y t +1

429
Khám phá tư duy Kỹ thuật giải bất ĐT Bài toán Max – Min – Đặng Thành Nam

7 − 4 t 2 + 2t + 8
Xét hàm số f (t ) = với t > 0.
t +1
−7 t 2 + 2t + 8 + 28
Ta có f '(t ) = ; f '(t ) = 0 ⇔ t 2 + 2t + 8 = 4 ⇔ t = 2.
(t + 1) 2
t + 2t + 8
2

Suy ra bảng biến thiên t 0 2 +∞


f '(t ) + 0 –

−3
f (t )

n
.v
Từ bảng biến thiên ta suy ra f (t ) ≤ −3 với mọi t > 0.

om
Dấu đẳng thức xảy ra khi và chỉ khi t = 2. (3)
Từ (1), (2) và (3) ta suy ra P ≤ ( x + y )(7 − 3) ≤ 8, dấu đẳng thức xảy ra khi và
x + y =2 .c
 4 2
chỉ khi  x ⇔=
x ,=
y .
=
t = 2 3 3
ok
 y
4 2
=
Vậy giá trị lớn nhất của P là 8, đạt khi x = ,y .
bo

3 3
Ví dụ 4. (TSĐH Khối D 2013) Cho x,y là hai số thực dương thỏa mãn điều kiện
xy ≤ y − 1 . Tìm giá trị lớn nhất của biểu thức
et

x+y x − 2y
=P −
vi

.
x − xy + 3y
2 2 6 (x + y)
ng

Lời giải
x
Nhận xét. Nhận thấy P đồng bậc nên ta tìm cách đánh giá miền giá trị của t =
a


y
kh

đưa về khảo sát hàm số với t.


2
x y −1 1 1 1 1 1 1
Xuất phát từ giả thiết ta có: 0 < ≤ 2 = − 2 =− −  + ≤ .
y y y y y 2 4 4
x 1 t +1 t −2
Đặt t = ≤ và P
⇒ 0 < t= − .
y 4 t2 − t + 3 6 ( t + 1)
t +1 t −2  1
=
Xét hàm số f(t) − trên  0;  ta được:
t2 − t + 3 6 ( t + 1)  4

430
Cty TNHH MTV DVVH Khang Việt

7 − 3t 1
=f '(t) − .
2 ( t + 1)
2

(t )
3
2 2
−t +3

1
Với 0 < t ≤ ta có t 2 − t + 3= t ( t − 1) + 3 < 3; 7 − 3t > 6; t + 1 > 1 .
4
7 − 3t 1 16 1 1
⇒ − > − = − > 0.
2 ( t + 1) 2.3 3 2 2
2

(t ) 3
3
2 2
−t +3

 1
Vậy f(t) là hàm đồng biến trên  0;  .

n
 4

.v
1 5 7
Do đó f(t) ≤ f   = +

om
.
4 3 30
5 7 1
Vậy giá trị lớn nhất của P bằng + đạt tại=x = ,y 2 .
.c
3 30 2
2) Kỹ thuật giảm biến với bất đẳng thức và bài toán cực trị ba biến
ok
=
Ta thường đặt a x=.b; b y.c .
Ví dụ 1. (Bất đẳng thức Schur) Cho a,b,c là các số thực không âm ta luôn có
bo

a3 + b3 + c3 + 3abc ≥ ab ( a + b ) + bc ( b + c ) + ca ( c + a ) .
Chứng minh.
et

Không mất tính tổng quát giả sử a ≥ b ≥ c khi đó tồn tại hai số thực không âm
(
) ( )
vi

x,y sao cho a =+


x 1 c, b =+
y 1 c và bất đẳng thức tương đương với:
ng

( x + 1) + ( y + 1) + 1 + 3 ( x + 1)( y + 1) ≥ ( x + 1)( y + 1)( x + y + 2 ) + ( x + 1)( x + 2 ) + ( y + 1)( y + 2 )


3 3

⇔ ( x − y ) ( x + y + 1) ≥ 0 (luôn đúng).
2
a

Ví dụ 2. Cho x,y,z là các số thực dương thỏa mãn điều kiện z ( x + y ) =


kh

xy .
x2 + y2 z
=
Tìm giá trị nhỏ nhất của biểu thức P + .
z 2
x+y
Lời giải
=
Đặt x a.z,y
= b.z ta có a + b =ab .
Khi đó
1 1 1
=( a + b ) − 2ab + =( a + b ) − 2 ( a + b ) +
2 2
P =a2 + b2 + .
a+ b a+ b a+ b

431
Khám phá tư duy Kỹ thuật giải bất ĐT Bài toán Max – Min – Đặng Thành Nam

( ) ≥ 4ab ⇒ ( a + b ) ≥ 4 ( a + b ) ⇔ a + b ≥ 4 .
2 2
Mặt khác a + b

1
( )
a + b, t ≥ 4 khi đó P = f(t) = t 2 − 2t + .
Đặt t =
t
1 t 3 − 2t 2 − 1
có f '(t)
Xét hàm số f(t) = t 2 − 2t + với t ≥ 4 ta= > 0, ∀t ≥ 4 nên
t t2
33
f(t) là hàm đồng biến trên  4; +∞ ) suy ra P = f(t) ≥ f(4) = .
4
33
đạt tại x= y= 2z .

n
Vậy giá trị nhỏ nhất của P bằng
4

.v
xy x + y
Cách 2: Từ giải thiết suy ra = .

om
z2 z
2
x+y x+y z
Khi đó P =   − 2 + .
 z   z  x+y .c
2
x+y xy  x + y  t2
ok
Đặt t = ⇒t= 2 ≤  = ⇒ t ≥ 4.
z z  2z  4
bo

1
Khi đó P = f(t) = t 2 − 2t + .
t
et

1 t 3 − 2t 2 − 1
có f '(t)
Xét hàm số f(t) = t 2 − 2t + với t ≥ 4 ta= > 0, ∀t ≥ 4 nên
t t2
vi

33
f(t) là hàm đồng biến trên  4; +∞ ) suy ra P = f(t) ≥ f(4) = .
ng

4
33
Vậy giá trị nhỏ nhất của P bằng đạt tại x= y= 2z .
a

4
kh

Ví dụ 3. Cho a,b,c là các số thực dương. Chứng minh


a b c a+b b+c
+ + ≥ + +1 .
b c a b+c a+b
Lời giải
a+b x +1 b + c y +1
=
Đặt .b; c y.b ( x, y > 0 ) khi=
a x= đó = ; .
b+c y +1 a + b x +1
1 y x +1 y +1
Bất đẳng thức trở thành: x + + ≥ + +1
y x y +1 x +1

432
Cty TNHH MTV DVVH Khang Việt

⇔ x3 y 2 + x 2 + x + y 3 + y 2 ≥ x 2 y + 2 xy 2 + 2 xy .
Bất đẳng thức cuối đúng vì là tổng của ba bất đẳng
x3 y 2 + x x3 y 2 + x + y 3 + y 3
≥ x 2 y; ≥ 2 xy 2 ; x 2 + y 2 ≥ 2 xy .
2 2
Bất đẳng thức được chứng minh. Đẳng thức xảy ra khi và chỉ khi a= b= c .
Ví dụ 4. Cho x,y,z là các số thực thuộc đoạn [1; 2] .
Tìm giá trị giá trị lớn nhất của biểu thức
3 3
 x 2 + z 2 + 4 xz   y 2 + 2 yz − 5 z 2 
=P   +  − 3x − x .
2

n
 x +z   yz − 4 z
2 2 2

.v
Lời giải
1 

om
=
Đặt x a=
.z; y b.z; a, b ∈  ; 2  .
2  
3 3
 a 2 + 4a + 1   b 2 + 2b − 5 
=  +  − 3x − x .
Khi đó P 
 a +1   b − 4 
2
2 .c
ok
Xét hàm số
a 2 + 4a + 1 b 2 + 2b − 5 1 
=f (a) = ; g (b ) trên đoạn  2 ; 2 
bo

a2 + 1 b−4
− 3 x − x 2 trên đoạn [1; 2] ta có
và h( x) =
et

4 ( a 2 − 1) 1 
f '(a ) =− ; f '(a ) =0 ⇔ a =∈
vi

1  ; 2 .
( a + 1) 2 
2 2
ng

 1  13
Suy ra max f =
(a) f=
(1) 3; min f =
(a ) f =
(2) f =  .
2 5
a

b 2 − 8b − 3 1 
kh

=
g '(b) < 0, ∀b ∈  ; 2 
(b − 4) 2 
2

 1  15 3
⇒ max g (b) =
g  = ; min g (b) =
g (2) =

 2  14 2
3 − 2x 3
h '( x) =− ; h '( x) =0⇔ x=
2 3x − x 2 2
3 3
⇒ max h( x) =
h(1) =
h(2) − 2; min h( x) =
h  =

2 2

433
Khám phá tư duy Kỹ thuật giải bất ĐT Bài toán Max – Min – Đặng Thành Nam

Do vậy P = ( f (a ))3 + ( g (b))3 + h( x) .


3
 15 
( max f (a) ) + ( max g (b) ) + max h( x) =
= 3 +  − 2 .
3 3 3
Vì vậy Pmax
 14 
 
 
= a 1= x z x = 2
 1  1 
Dấu bằng xảy ra khi và chỉ khi b = ⇔  y = z ⇔  y =1 .
 2  2 
=   x 1= x 1 z = 2
 

n
=   x 2= x 2

.v
ab
Ví dụ 5. Cho 3 số thực dương a,b,c thõa mãn điều kiện a−c + b−c = .

om
c
a b c c2
Tìm giá trị nhỏ nhất của biểu thức P = + + + 2 .
b + c c + a a + b a + b2
.c
Lời giải
( )
ok
=
Đặt a x.c,
= b y.c, x,y ≥ 1 khi đó theo giả thiết ta có:

x − 1 + y −=
1 xy ⇔ ( ( x − 1)( y − 1) − 1)=
2
0
bo

.
⇔ xy =+
x y ≥ 2 xy =
> xy ≥ 4
et

x y 1 1
Ta viết lại biểu thức P dưới dạng: P = + + + 2 .
y +1 x +1 x + y x + y2
vi

Sử dụng bất đẳng thức C-S ta có:


ng

x2 y2 1 1
P= + + +
xy + x xy + y x + y ( x + y ) 2 − 2 xy
a

( x + y)2 1 1
≥ + + 2 2
kh

2 xy + x + y xy x y − 2 xy
x2 y 2 1 1
= + + 2 2
3 xy xy x y − 2 xy
xy 1 1
= + + 2 2
3 xy x y − 2 xy
t 1 1
( )
Đặt t = xy, t ≥ 4 ⇒ P ≥ f(t) = + + 2
3 t t − 2t
.

434
Cty TNHH MTV DVVH Khang Việt

t 1 1
Xét hàm số f(t) = + + 2 trên  4; +∞ ) ta có:
3 t t − 2t

f '(t)
( )
t 2 t 2 − 4t + 1 + 6 ( t − 1)
> 0, ∀t ≥ 4 .
6t 2 ( t − 2 )
2

41
Vì vậy f(t) đồng biến với t ≥ 4 ⇒ P ≥ f(t)= f(4)= .
24
41
Vậy giá trị nhỏ nhất của P bằng đạt tại x= y= 2 .
24

n
Ví dụ 6. Cho a,b,c là các số thực thỏa mãn điều kiện a ≥ b ≥ c > 0 .

.v
( 3ab + bc )
2
121b 2
= +

om
Tìm giá trị nhỏ nhất của biểu thức P .
b4 a 2 + b 2 + c 2 + 8ac
Lời giải
a c
, ( x ≥ 1 ≥ y > 0 ) khi đó:
x
Đặt =
b
,y
=
b
.c
ok
121
P = ( 3x + y ) +
2

x + y + 8xy + 1
2 2
bo

121
= 9x 2 + 6xy + y 2 + 2
x + y + 8xy + 1
2
et

121
≥ x 2 + 8xy + y 2 + 6 + 2
x + y + 8xy + 1
2
vi

121
ng

Đặt t =
x 2 + 8xy + y 2 + 1 ⇒ P ≥ f(t) =
t+ + 5.
t
121
a

Ta có: f '(t) = 1 − ; f '(t) = 0 ⇔ t = 11 .


t2
kh

Ta có f’(t) đổi dấu từ âm sang dương khi đi qua t = 11 nên f(t) đạt cực tiểu tại
t = 11 hay P ≥ f(t) ≥ f(11) =27 . Đẳng thức xảy ra khi và chỉ khi a= b= c .
Vậy giá trị nhỏ nhất của P bằng 27 đạt tại a= b= c .
Ví dụ 7. Cho x,y,z là các số thực dương thỏa mãn điều kiện
7 ( x 2 + y 2 + z 2=
) 11( xy + yz + zx ) .
51 x y z
Chứng minh rằng ≤ + + ≤ 2.
28 y + z z + x x + y

435
Khám phá tư duy Kỹ thuật giải bất ĐT Bài toán Max – Min – Đặng Thành Nam

Lời giải
x y z
Đặt P = + + khi đó
y+z z+x x+ y
x y z
P +=
3 +1+ +1+ +1
y+z z+x x+ y
x+ y+z x+ y+z x+ y+z  1 1 1 
= + + = ( x + y + z) + + 
y+z z+x x+ y  x+ y y+z z+x
( x + y + z ) ( x + y )( x + z ) + ( y + z )( y + x ) + ( z + x )( z + y )
=
( x + y )( y + z )( z + x )

n
.v
( x + y + z ) ( x + y + z ) + xy + yz + zx 
2

=
( x + y )( y + z )( z + x )

om
Theo giả thiết ta có: 7 ( x + y + z=
) 25 ( xy + yz + zx ) . Suy ra
2

( x + y + z ) ( x + y + z )
7 2
+ ( x + y + z) 
2
.c
( x + y + z)
3
 25  32
ok
P+3 =
( x + y )( y + z )( z + x ) 25 ( x + y )( y + z )( z + x )
( x + y )( y + z )( z + x ) .
bo

32 1
Do đó . =
25 P + 3 ( x + y + z)
3
et

a + b + c = 1
x y z 
=
Đặt a = ,b = ,c ⇒
vi

7 .
x+ y+z x+ y+z x+ y+z ab + bc + ca =
25
ng

32 1
. =( a + b )( b + c )( c + a ) =(1 − a )(1 − b )(1 − c )
25 P + 3
a

=1 − ( a + b + c ) + ab + bc + ca − abc .
kh

7 7
= 1 −1 + − abc = − abc
25 25
Vậy để tìm giá trị lớn nhất và nhỏ nhất của P ta tìm giá trị lớn nhất và nhỏ nhất
của Q = abc .
Ta có
 7  7  7  7
 a  − a ( b + c )=
= a  − ab − ac= − ( − )= − +
3 2
Q  a  25 a 1 a  a a a
 25   25 25
Ta có:

436
Cty TNHH MTV DVVH Khang Việt

b + c =1 − a
  2 7  1 3
7 ; ( b + c ) ≥ 4bc ⇔ (1 − a ) ≥ 4  a − a +  ⇔ ≤ a ≤ .
2 2

bc = a − a − 25  25  15
2
5

7  1 3
Xét hàm số f (a ) = a 3 − a 2 + a trên đoạn  ;  ta có:
25 15 5 
 1
 a=
7 5
f '(a ) =3a 2 − 2a + ; f '(a ) =0 ⇔  .
25 a = 7


n
15
1 7 3 1

.v
49 3
Ta có f =
  f=  ; f=
  f=   .
 15   15  3375  5   5  125

om
3 49 51
Suy ra Qmax = ; Qmin = ⇒ Pmax = 2; Pmin = .
125 3375 28
3) Một đánh giá hay sử dụng .c
Với điều kiện các số thực a,b,c không âm ta các bất đẳng thức thường đạt điểm
rơi tại một biến bằng 0 nên ta thường giả sử c = min {a, b,c} và tìm cách đánh
ok
giá đưa bất đẳng thức về hai biến.
bo

Ta có các ước lượng hay sử dụng:


2 2 2 2
 c  c  c  c
a + c ≤  a +  ; b2 + c2 ≤  b +  ;a2 + b2 ≤  a +  +  b +  ;
2 2
et

 2  2  2  2
2 2
vi

 c  c
a + b + c ≤ a+  +b + 
2 2 2

2  2
ng


 c  c
ab + bc + ca ≥  a +   b + 
2  2
a


kh

2
 c
a − ac + c = a + c ( c − a ) ≤ a ≤  a +  ;
2 2 2 2

 2
2
 c
b − bc + c = b + c ( c − b ) ≤ b ≤  b +  ;
2 2 2 2

 2
2 2
 c  c  c  c
a − ab + b ≤  a +  −  a +  b +  +  b + 
2 2

 2  2  2  2

437
Khám phá tư duy Kỹ thuật giải bất ĐT Bài toán Max – Min – Đặng Thành Nam

Ví dụ 1. Cho a,b,c là các số thực không âm thỏa mãn điều kiện a + b + c =3.
(
Tìm giá trị lớn nhất của biểu thức P = a2 − ab + b2 b2 − bc + c2 c2 − ca + a2 . )( )( )
Lời giải
{
Giả sử c = min a, b,c ta có: }
b2 − bc + c2 = b2 + c ( c − b ) ≤ b2 ;
a2 − ac + c2 = a2 + c ( c − a ) ≤ a2
Suy ra
(
P ≤ a2 b2 a2 − ab + b2 )

n
.v
a b

P

(
a2 b2 a2 − ab + b2 ) ≤ a b (a
2 2 2
− ab + b2 ) =b − 1 + a

om
36 ( a + b + c) (a + b)
6 6 3
a b
 b +2+ a
.c  
a b t −1
Đặt t = + , ( t ≥ 2 ) ⇒ P ≤ f(t) = 36. .
b a ( )
3
ok
t + 2
t −1
Xét hàm số f(t) = 36. với t ≥ 2 ta có:
bo

( t + 2)
3

5 − 2t 5
et

f '(t) = 36. ; f '(t) = 0 ⇔ t = .


( t + 2) 2
4
vi

5
ng

Ta có f’(t) đổi dấu từ dương sang âm khi đi qua t = nên f(t) đạt cực đại tại
2
5 5
t= hay P ≤ f(t) ≤ f   =
12 .
a

2 2
kh

a 2,=
Vậy giá trị lớn nhất của P bằng 12 đạt tại = b 1,c
= 0 hoặc các hoán vị.
Cách 2: Thực hiện đánh giá như trên và sử dụng AM-GM ta có:
3
3 3 
 ab + ab + a2 − ab + b2 
4 3 3 4
P ≤ a2 b2 a=
2
(− ab + b2
9 2 2
)
. ab. ab a2 − ab + b2 ≤  2
9
2
( 3
 )
 
 
4 ( a + b ) 4 ( a + b + c)
6 6

=. ≤ 12
=
9 27 9.27
438
Cty TNHH MTV DVVH Khang Việt

Đẳng thức xảy ra khi và chỉ khi


3 =a 2,=
b 1,c
= 0
a + b + c = 3,c = 0, ab = a2 − ab + b2 ⇔ 
2 a 1,=
= b 2,c
= 0

{
Cách 3: Giả sử c = min a, b,c khi đó ta có: }
2
 c
a − ac + c = a + c ( c − a ) ≤ a ≤  a +  ;
2 2 2 2

 2
2
 c
b − bc + c = b + c ( c − b ) ≤ b ≤  b +  ;
2 2 2 2

2

n

2 2

.v
 c  c  c  c
a − ab + b ≤  a +  −  a +  b +  +  b + 
2 2

 2  2  2  2

om
c c
Đặt x = a + ,y = b + , ( x,y ≥ 0 ) ⇒ x + y = a + b + c = 3 khi đó:
2 2 .c
( ) (
P ≤ x 2 y 2 x 2 − xy + y 2 = x 2 ( 3 − x ) x 2 − x ( 3 − x ) + ( 3 − x )
2 2
)
ok
=3x 6 − 27x 5 + 90x 4 − 135x3 + 81x 2 ≤ 12
Ví dụ 2. Cho a,b,c là các số thực không âm không có hai số nào đồng thời bằng 0
bo

thỏa mãn a + b + c = 2.
1 1 1
Tìm giá trị nhỏ nhất của biểu thức P = + +
et

.
4 4 4
a2 + b 2 b 2 + c2 a 2 + c2
vi

Lời giải
Không mất tính tổng quát giả sử c = min a, b,c khi đó ta có: { }
ng

2 2
 c  c
a + b ≤ a+  +b +  ;
2 2
a

 2  2
kh

2
 c
b +c ≤ b+  ;
2 2

 2
2
 c
a + c ≤ a+ 
2 2

 2
1 1 1
Khi đó P ≥ + +
c c  c 
2
c
2
a+ b+
2 2
4
a+ 2  +b + 2 
   
439
Khám phá tư duy Kỹ thuật giải bất ĐT Bài toán Max – Min – Đặng Thành Nam

c c
Đặt x = a + ,y = b + , ( x,y > 0 ) ⇒ x + y = a + b + c = 2 .
2 2
2
x+y
Theo bất đẳng thức AM-GM ta có: xy ≤  1 và
 =
 2 
1 1 1 2 1 1 1  1  1
P≥ + + ≥ + = + +  2 − .
x y 4
x2 + y2 4
xy x + y2
4 2 4
2xy x + y2 
4 2 4
2  4 xy
2 1 2 1 1
≥ +2− ≥ +2− = 2+
8
(
2xy x 2 + y 2 )
 2xy + x 2 + y 2 
24
2 2 4 4
2

n
  8
2

.v
 
Dấu bằng xảy ra khi và chỉ khi x = y = 1 ⇒ a = b = 1,c = 0 .

om
1
Vậy giá trị nhỏ nhất của P bằng 2 + đạt tại a= b= 1,c= 0 hoặc các hoán vị.
4
2 .c
ok
B. BÀI TẬP CHỌN LỌC
Bài 1. (TSĐH Khối A, A1 2013) Cho các số thực dương a,b,c thỏa mãn điều kiện
( a + c)( b + c) =
4c 2
bo

32a3 32b3 a2 + b 2
Tìm giá trị nhỏ nhất của biểu thức P = + − .
et

( b + 3c) ( a + 3c) c
3 3
vi

Lời giải
=
Đặt a cx,
= (
b cy, x,y > 0 từ điều kiện bài toán ta có: )
ng

( x + 1)( y + 1) = 4 ⇔ xy + x + y = 3 .
a

32x3 32y3
P
Ta có: = + − x2 + y2 .
kh

( y + 3) ( x + 3)
3 3

Trước tiên ta chứng minh bất đẳng thức quen thuộc sau đây:
1
( a + b) .
3
Với mọi a,b dương ta luôn có a3 + b3 ≥
4
( )( ) ( )
3
Thật vậy bất đẳng thức tương đương với: 4 a + b a2 − ab + b2 ≥ a + b .

( )
⇔ 4 a2 − ab + b2 ≥ a2 + 2ab + b2 ⇔ 3 ( a − b ) ≥ 0 .
2

440
Cty TNHH MTV DVVH Khang Việt

Bất đẳng thức luôn đung và bài toán phụ được chứng minh.
Áp dụng ta có:
3 3
32x3 32y3  x y   x 2 + y 2 + 3x + 3y 
+ ≥ 8 +  8
=  
( y + 3) ( x + 3)  y+3 x+3  xy + 3x + 3y + 9 
3 3

3 .
 ( x + y )2 + 3 ( x + y ) − 2xy 
= 8 
 xy + 3 ( x + y ) + 9 
 
Thay xy = 3 − x − y vào biểu thức trên ta được:

n
3
 ( x + y )2 − 2 ( 3 − x − y ) + 3 ( x + y ) 

.v
32x 3
32y 3

( x + y − 1) .
3
+ ≥ 8  =
( y + 3) ( x + 3)
3 3
 3 − x − y + 3(x + y) + 9 

om
 
Do đó

P ≥ ( x + y − 1) − x 2 + y 2 = ( x + y − 1) (x + y)
3 3 2
.c − − 2xy

( x + y − 1) (x + y) − 2 (3 − x − y )
3 2
ok
= −

( x + y − 1) (x + y) + 2 (x + y) − 6
3 2
= −
bo

( )
3
Đặt t= x + y ta có P ≥ t − 1 − t 2 + 2t − 6 và
et

2
x+y t2
3 − x − y= xy ≤  ⇒ 3 − t ≤ ⇔ ( t − 2 )( t + 6 ) ≥ 0 ⇔ t ≥ 2 .
vi


 2  4
ng

( )
3
Xét hàm số f(t) = t − 1 − t 2 + 2t − 6 với t ≥ 2 ta được:

3 t 2 + 2t − 6 ( t − 1) − t − 1
2
a

t +1
f '(t) = 3 ( t − 1) −
2
=
kh

t 2 + 2t − 6 t 2 + 2t − 6
3 2 ( t − 1) − t − 1 4 ( t − 1) − t − 1 4t 2 − 9t + 3
2 2

≥ > = > 0, ∀t ≥ 2
t 2 + 2t − 6 t 2 + 2t − 6 t 2 + 2t − 6
Vậy f(t) là hàm đồng biến trên 2; +∞ do đó )
f(t) ≥ f(2) = 1 − 2 ⇒ P ≥ 1 − 2 .
Vậy giá trị nhỏ nhất của P bằng 1 − 2 đạt tại a= b= c .

441
Khám phá tư duy Kỹ thuật giải bất ĐT Bài toán Max – Min – Đặng Thành Nam

Bài 2. Cho a,b,c là các số thực dương đôi một phân biệt thỏa mãn b ≤ 8a ≤ 4c và
ab + bc =
2c2 . Tìm giá trị lớn nhất và nhỏ nhất của biểu thức
a b c
P= + + .
a−b b−c c−a
Lời giải
 1 1 2
a x.c,=
Đặt= b y.c,  0 < y ≤ x ≤  và xy + y = 2 ⇔ y = .
 8 2 x +1
Khi đó
2

n
x y 1 1 x
P= + + = + x +1 +

.v
x − y y −1 1− x 2 2 1− x
x− −1
x +1 x +1

om
x +x
2
3 x + x − 3 ( x + 2 ) x 2 − 2x − 6
2

= 2 + = =
x + x − 2 1− x x2 + x − 2 x2 + x − 2 .c
1 1  −1 + 2 1 
Ta có y ≤ x ≤ ⇒ x∈ ; 
ok
8 2  2 2 

x 2 − 2x − 6  −1 + 2 1 
bo

Xét hàm số f(x) = liên tục trên  ;  ta có


x2 + x − 2  2 2 
et

3x 2 + 8x + 10  −1 + 2 1 
=f '(x) > 0, ∀x ∈  ; 
(x )  2 2 
2
vi

2
+x−2
ng

 −1 + 2 1 
suy ra f(x) là hàm đồng biến trên  ; 
 2 2 
a

 −1 + 2  1 17 + 6 2 27
kh

Suy ra f   ≤ f(x) ≤ f   hay ≤P≤ .


 2   2  7 5
 
27 8
Vậy giá trị lớn nhất của P bằng đạt tại=b 2a,c
= a và giá trị nhỏ nhất
5 3

của P bằng
17 + 6 2
7
tại a
đạt =
−1 + 2
2
= c, b 4 ( )
2 −1 c .

442
Cty TNHH MTV DVVH Khang Việt

Bài 3. Cho a,b,c là các số thực dương thỏa mãn điều kiện a3 + b3 =
c3 .
a 2 + b 2 − c2
Tìm giá trị nhỏ nhất của biểu thức P = .
( c − a )( c − b )
Lời giải
Nhận xét. Điều kiện đề bài cho đồng bậc 3 và biểu thức P bậc 0 nên ta đặt
=a xc,
= b yc, ( x,y > 0 ) .

( x + y ) − 2xy − 1 .
2
x2 + y2 − 1
Khi đó x +
=3
1 và P =
y = 3

(1 − x )(1 − y ) xy − ( x + y ) + 1

n
(x + y) −1 .

.v
3

= ( x + y ) − 3xy ( x + y ) ⇒ xy =
3
Xuất phát từ 1 = x + y 3 3

3(x + y)

om
t3 − 1
( )
2
Đặt t= x + y ta có t > 1 và x + y ≥ 4xy ⇒ t ≥ 4. 2
⇔1< t ≤ 3 4 .
.c 3t
t+2
Ta có P = dễ thấy đây là hàm nghịch biến với 1 < t ≤ 3 4 suy ra
ok
t −1
3
4 +2
P≥
bo

.
3
4 −1
3
4 +2
et

Vậy giá trị nhỏ nhất của P bằng đạt tại= = a3 2 .


a b,c
3
4 −1
vi

Bài 4. Cho a,b,c là các số thực dương thỏa mãn điều kiện a ≥ b ≥ c .

(a )
ng

2
+ c2 ab + bc + ca
Tìm giá trị nhỏ nhất của biểu thức M = .
ac ( a + b + c )
a

Lời giải
kh

(
Nhận xét. Điều kiện cho a ≥ b ≥ c ⇒ b − a b − c ≤ 0 và M bậc 0 nên đặt )( )
= y.b, ( x ≥ 1,0 < y ≤ 1) và
a x.b,c
=

(1 − x )(1 − y ) ≤ 0 ⇔ xy − x − y + 1 ≤ 0 ⇒ x + y ≥ 1 + xy .
Khi đó: M =
(x 2
+ y2 ) x + y + xy
.
xy ( x + y + 1)

443
Khám phá tư duy Kỹ thuật giải bất ĐT Bài toán Max – Min – Đặng Thành Nam

Đây là biểu thức đối xứng với tổng x + y và tích xy nên suy nghĩ ngay đến việc
đặt S =+
x y,P =xy từ điều kiện ta có ngay 0 < P ≤ S − 1 .

Ta có: M =
(S 2
− 2P ) S+ P
.
P ( S + 1)
Coi vế phải là hàm số với P và tham số S ta được:
2S3 + S2 P + 2P 2
f '(P) =− < 0, ∀S,P > 0
2 ( S + 1) P 2 S + P

do đó f(P) là hàm nghịch biến trên 0;S − 1 . (

n
.v
(S 2
− 2S + 2 ) 2S − 1
Do đó f(P) ≥ f S − 1( ) = .

om
S2 − 1
(S 2
− 2S + 2 ) 2S − 1
Xét hàm số g(S) =
S2 − 1
( )
trên 1; +∞ ta được:
.c
(S − 2) (S 3
+ 4S2 − 5S + 2 );
ok
S4 + 2S3 − 13S2 + 12S − 4
g'(S) =
( ) ( )
2 2
2S − 1 S2 − 1 2S − 1 S2 − 1
bo

g'(S)
= 0 ←
S>1
S 2
→=
Ta có g’(S) đổi dấu từ âm sang dương khi đi qua S = 2 nên g(S) đạt cực tiểu tại
et

2
S = 2 trên (1; +∞ ) hay g(S) ≥ g(2) =.
vi

3
ng

2
Vậy giá trị nhỏ nhất của M bằng đạt tại a= b= c .
3
a

Bình luận. Ngoài lời giải trên ta có thể xét hàm số trực tiếp bằng cách coi b là ẩn
và a,c là tham số ta có kết quả tương tự hoặc chứng minh
kh

( a + c) b + 2ac ≥
2 2
b2 + 2ac 4ac 2
≥ . Khi đó M ≥ .
( a + b + c) 3 ( a + c)
2 2
ac ( a + b + c ) 3
Nhưng rõ ràng với dấu hiệu đẳng cấp từ điều kiện cho đến biểu thức M việc sử
dụng kỹ thuật giảm về hai biến x,y tỏ ra hiệu quả. Đây là một bài toán hay đòi
hỏi phải tư duy logic khi gặp tổng và tích đối xứng của S =+
x y,P = xy .

( )
3
Bài 5. Cho a,b,c là các số thực dương thay đổi thỏa mãn điều kiện a + b + c =32abc .

444
Cty TNHH MTV DVVH Khang Việt

a 4 + b 4 + c4
Tìm giá trị lớn nhất và nhỏ nhất của biểu thức P = .
( a + b + c)
4

Lời giải
a b c 1
Theo giả thiết ta có: . . = .
a + b + c a + b + c a + b + c 32
a b c 1
Đặt x
= = ,y = ,z ,x,y,z > 0 và xyz
= ,x + y +=
z 1.
a+ b+c a+ b+c a+ b+c 32
Khi đó P = x 4 + y 4 + z 4 và ta có:

n
(x ) ( )
2
P= 2
+ y2 + z2 − 2 x2 y2 + y2z2 + z2 x2

.v
2
= ( x + y + z ) − 2 ( xy + yz + zx )  − 2 ( xy + yz + zx ) − 2xyz ( x + y + z ) 
2 2

om
   
 1
= 1 − 2 ( xy + yz + zx )  − 2 ( xy + yz + zx ) − 
2 2

16 

.c
9
= 2 ( xy + yz + zx ) − 4 ( xy + yz + zx ) +
2
ok
8
9
bo

Đặt t = xy + yz + zx ta có P = 2t 2 − 4t + .
8
1 1
( ) ( )
et

Ta có: t =x y + z + yz =x 1 − x + =− x 2 + x + .
32x 32x
vi

Mặt khác:
3 − 5 1
ng

 ≤x≤
4
( y + z ) ≥ 4yz ⇒ (1 − x ) ≥ 32x (
⇔ ( 2x − 1) 4x 2 − 6x + 1 ≥ 0 ⇔  4 2.
)
2 2

 3+ 5
a

x ≥
 4
kh

1 3 − 5 1  3 + 5 
Xét hàm số f(x) =− x 2 + x + trên  ;  ; +∞  ta được:

32x  4 2   4 
 1
 x=  3− 5 1   3+ 5 
1 4x∈ ; 
 4 2   4
;+∞ 

f '(x) = −2 x + 1 − →
; f '(x) = 0 ← . 

32x 2
 1+ 5
x =
 8

445
Khám phá tư duy Kỹ thuật giải bất ĐT Bài toán Max – Min – Đặng Thành Nam

Ta có:
 3− 5   1+ 5  5 5 −1  1   1  5  3+ 5  5 5 +1
f = f = ,f   = f  = ,f  = − .
 4   8  32 4  2  16  4  32
   
5 5 5 −1 9  5 5 5 − 1
Suy ra ≤t≤ . Xét hàm số g(t) = 2t 2 − 4t + trên  ; 
16 32 8 16 32 
 5 5 5 − 1
Ta có: g'(t)= 4t − 4 < 0, ∀t ∈  ; .
16 32 

n
 5  5 5 −1  383 − 165 5 9
Suy ra g  ≥ g(t) ≥ g   hay ta có ≤P≤

.v
  32 
.
 16    256 128

om
9
Vậy giá trị lớn nhất của P bằng đạt tại a= 2, b= c= 1 hoặc các hoán vị và
128
383 − 165 5 1+ 5 .c
giá trị nhỏ nhất của P bằng đạt tại a =
3 − 5, b =
c= .
256 2
ok
Bài 6. Cho a,b,c là các số thực thỏa mãn điều kiện a ≥ b ≥ c ≥ 0 và
2b + 2c − a > 0 .
bo

 b c  2b + 2c − a
Tìm giá trị nhỏ nhất của biểu thức P = 2  + − .
 c+a a + b  a

et

Lời giải
vi

b 2b − a
TH1: Nếu c = 0 ⇒ P = 2 − .
a a
ng

b  1 
Đặt t
= , ≤ t ≤ 1 khi đó P =
f(t) =
2t − 2t 2 − 1 .
a

a  2 
kh

 1 
Xét hàm số f(t) =
2t − 2t 2 − 1 liên tục trên  ;1 ta có
 2 

f '(t) = 2 −
2t
=
2 ( 2t 2 − 1 − t )= (
2 t2 −1 )  1 
≤ 0, ∀t ∈  ;1 .
2t 2 − 1 2t 2 − 1 2t 2 − 1 ( 2t − 1 + t
2
)  2 

 1 
Do đó f(t) là hàm nghịch biến trên  ;1 , suy ra P = f(t) ≥ f(1) = 1 .
 2 

446
Cty TNHH MTV DVVH Khang Việt

a b,c
Trường hợp này giá trị nhỏ nhất của P bằng 1 đạt tại= = 0.
TH2: Nếu c > 0 ⇒ a ≥ b ≥ c > 0 đặt c = x.a, b = y.a, 0 < x,y ≤ 1 . ( )
 x y 
=
Khi đó: P 2 +  − 2x + 2y − 1 .
 y +1 x + 1 

x y
Ta chứng minh + ≥ x+y .
y +1 x +1
Thật vậy bất đẳng thức tương đương với:

( x + y )( x + 1)( y + 1)

n
x2 + x + y2 + y ≥

.v
⇔ x 2 + x + y 2 + y + 2 xy ( x + 1)( y + 1) ≥ ( x + y )( x + 1)( y + 1) .

om
⇔ 2 xy ( x + 1)( y + 1) ≥ xy ( x + y + 2 ) ⇔ 4 ( xy + x + y + 1) ≥ xy ( x + y + 2 ) .
2

( ) ≤ 4xy ( x + y + 2 ) và
2
Bất đẳng thức luôn đúng do xy x + y + 2
.c
xy + x + y + 1 − xy ( x + y + 2 ) = ( x + y + 1)(1 − xy ) ≥ 0 .
ok

Vậy P ≥ 2 x + y − 2x + 2y − 1
bo

Đặt t= ( )
x + y, 0 < t ≤ 2 suy ra P ≥ f(t) =2t − 2t 2 − 1 .
et

2t
Ta có: f '(t) = 2 − ; f '(t) = 0 ⇔ t = 1 .
vi

2t − 1 2

Do đó P ≥ f(1) =
1 . Nhưng trường hợp này dấu bằng không xảy ra.
ng

a b,c
Kết hợp hai trường hợp suy ra giá trị nhỏ nhất của P bằng 1 đạt tại= = 0.
Bài 7. (TSĐH Khối A 2009) Chứng minh rằng với mọi số thực dương x,y,z thỏa
a

(
mãn điều kiện x x + y + z = )
3yz ta luôn có
kh

( x + y ) + ( x + z ) + 3 ( x + y )( y + z )( z + x ) ≤ 5 ( y + z )
3 3 3
.
Lời giải
=
Đặt y a.x,
= ( )
z b.x, a, b > 0 ta có a + b + 1 =3ab .

Ta cần chứng minh ( a + 1) + ( b + 1) + 3 ( a + 1)( b + 1)( a + b ) ≤ 5 ( a + b ) .


3 3 3

⇔ ( a + b + 2 ) − 3 ( a + 1)( b + 1)( a + b + 2 ) + 3 ( a + 1)( b + 1)( a + b ) ≤ 5 ( a + b ) .


3 3

447
Khám phá tư duy Kỹ thuật giải bất ĐT Bài toán Max – Min – Đặng Thành Nam

⇔ ( a + b + 2 ) − 6 ( a + 1)( b + 1) ≤ 5 ( a + b )
3 3

⇔ ( a + b + 2 ) − 6 ( ab + a + b + 1) ≤ 5 ( a + b )
3 3

Thay a + b + 1 =3ab vào bất đẳng thức ta cần chứng minh


( 3ab + 1) − 6 ( ab + 3ab ) ≤ 5 ( 3ab − 1) ⇔ ( 3ab − 1)( ab − 1)( 6ab − 1) ≥ 0
3 3

(luôn đúng)
Do 3ab = a + b + 1 ≥ 2 ab + 1 ⇒ ( )( )
ab − 1 3 ab + 1 ≥ 0 ⇔ ab ≥ 1 .
Bất đẳng thức được chứng minh. Đẳng thức xảy ra khi và chỉ khi x= y= z .

n
Bài tập tương tự

.v
1) Cho a,b là hai số thực và c không âm thỏa mãn điều kiện a2 + b2 + ab =
3c2 .
Chứng minh rằng a3 + b3 + 4abc ≤ 6c3 .

om
Cho a,b,c là các số thực dương thoả mãn điều kiện a 2 + 2a ( b + c ) =
5bc .
Chứng minh rằng ( a + b ) + ( a + c ) + ( a + b )( a + c )( b + c ) ≤ 3 ( b + c ) .
3 3 3

Bài 8. Cho a,b,c là các số thực dương thỏa mãn a + b + c =


.c
6 và a 2 + b 2 + c 2 =
14 .
ok
4a + b
Tìm giá trị lớn nhất và nhỏ nhất của biểu thức P = .
c
bo

Lời giải
c ( x + y + 1) = 6
=
Đặt .c, b y.c, ( x, y > 0 ) theo giả thiết ta có: 
a x=
et

.
 c 2
( x 2
+ y 2
+ 1) =
14
vi

 6−c
x+ y =
( x + y ) c =− 6 c 
 c
ng

⇔ ⇔
( 6 − c ) − (14 − c 2 ) c 2 − 6c + 11
.
 ( x + y ) c  − 2 xyc =−
2 2
2 2
14 c 
=  xy =
a

2c 2 c2
Trước hết ta phải có:
kh

6−c c 2 − 6c + 11 6−2 3 6+2 3


2

( x + y) ≥ 4 xy ⇔   ≥ 4. ⇔ ≤c≤
2
.
 c 
2
c 3 3
Suy ra x,y là hai nghiệm dương của phương trình:
 6 − c − −3c 2 + 12c − 8
t1 =
6 − c c − 6c + 11 2
0⇔
2c
t2 − t+ = .
c c 2
 6 − c + −3c 2 + 12c − 8
t 2 =
 2c

448
Cty TNHH MTV DVVH Khang Việt

Ta có: =P 4x + y .
Tìm giá trị lớn nhất của P.
Để tìm giá trị lớn nhất của P ta xét với=x t2=
, y t1 vì t2 ≥ t1 .
Khi đó
6 − c + −3c 2 + 12c − 8 6 − c − −3c 2 + 12c − 8 30 − 5c + 3 −3c 2 + 12c − 8
P= 4. + = .
2c 2c 2c
30 − 5c + 3 −3c 2 + 12c − 8 6 − 2 3 6 + 2 3 
Xét hàm số f (c) = liên tục trên đoạn  ; .
2c  3 3 

( ) ; f '(c) =0 ⇔ 5

n
3 5 −3c 2 + 12c − 8 + 3c − 4
f '(c) =− −3c 2 + 12c − 8 + 3c − 4 =0

.v
c −3c + 12c − 8
2 2

om
4 − 3c ≥ 0 6
⇔ 2 ⇔ c =.
25 ( −3c + 12c − 8 ) = ( 4 − 3c )
2
7

Ta có f’(c) đổi dấu từ dương sang âm khi đi qua c =


.c 6
7
nên f(c) đạt cực đại tại
ok
6  6  31
c= hay P ≤ f (c) ≤ f   = .
7 7 2
bo

19 17 6
=
Đẳng thức xảy ra khi và chỉ khi a = ,b = ,c .
7 7 7
et

31 19 17 6
Vậy giá trị lớn nhất của P bằng =
đạt tại a = ,b = ,c .
2 7 7 7
vi

Tìm giá trị nhỏ nhất của P.


Để tìm giá trị nhỏ nhất của P ta xét với=x t1=
ng

, y t2 .
Khi đó
a

6 − c − −3c 2 + 12c − 8 6 − c + −3c 2 + 12c − 8 30 − 5c − 3 −3c 2 + 12c − 8


P= 4. + = .
kh

2c 2c 2c
30 − 5c − 3 −3c 2 + 12c − 8
Xét hàm số g(c) = liên tục trên đoạn
2c
6 − 2 3 6 + 2 3 
 ; .
 3 3 

g'(c) =−
(
3 5 −3c 2 + 12c − 8 − 3c + 4 ) ;g'(c) =0 ⇔ 5 −3c 2 + 12c − 8 − 3c + 4 =0 .
c 2
−3c + 12c − 8
2

449
Khám phá tư duy Kỹ thuật giải bất ĐT Bài toán Max – Min – Đặng Thành Nam

3c − 4 ≥ 0
⇔ ⇔c=3.
25 ( −3c + 12c − 8 ) = ( 3c − 4 )
2 2

Ta có g’(c) đổi dấu từ âm sang dương khi đi qua c = 3 nên g(c) đạt cực tiểu tại
c = 3 hay P ≥ g (c) ≥ g (3) =2.
Đẳng thức xảy ra khi và chỉ khi =
a 1,=
b 2,=
c 3.
Vậy giá trị nhỏ nhất của P bằng 2 đạt tại =
a 1,=
b 2,=
c 3.
Bài 9. Cho a,b,c là các số thực dương thỏa mãn điều kiện

( 3a + 2b + c ) 
1 2 3
+ + =30 .

n
a b c

.v
b + 2c − 7 72a 2 + c 2
Tìm giá trị lớn nhất của biểu thức P = .

om
a
Lời giải
b c 1 1 
=
Đặt x = ,y . Suy ra: 30 = ( 4 x + 3 y + 3) 
.c + + 1
2a 3a x y 
 1 1  x ( y + 1)
ok
1 1 
= x  + + 1 + 3 ( x + y + 1)  + + 1 ≥ + 28
x y  x y  2y
bo

2y 4y
⇒x≤ ⇒ P = 2 x + 6 y − 21 y 2 + 8 ≤ + 6 y − 21 y 2 + 8 ≤ −55
y +1 y +1
et

Đẳng thức xảy ra khi và chỉ khi 6= a 3= b 2c .


Cách 2: Trước tiên ta biến đổi điều kiện bài toán
vi

( 3a + 2b + c ) 
1 1 1 
+ + =
ng

5
 6a 3b 2c 
 1 1 1   1 1 1 
⇔ ( 6a + 3b + 2c )  + +  + b  + +  =
a

10
 6a 3b 2c   6a 3b 2c 
kh

 1 1 1   1 1  2 4ac
⇒ b + +  ≤1⇒ b +  ≤ ⇔ b ≤
 6a 3b 2c   6 a 2c  3 3a + c
4ac
+ 2c − 7 72a 2 + c 2 2
3a + c 4c 2c c
⇒P≤ = + − 7 72 +  
a 3a + c a a
4t
Xét hàm số f (t ) = + 2t − 7 t 2 + 72 ≤ −55, ( t = c / a ) ta có
3+t

450
Cty TNHH MTV DVVH Khang Việt

4 4t 7t
f '(t ) =
2+ − − .
t + 3 ( t + 3) 2
t 2 + 72
ĐS: Pmax =
−55; 2a =
3b =
6c .
Dưới đây tôi trình bày một bài toán tương tự nhưng đi theo một hướng khác
Bài 10. Cho a,b,c là các số thực dương thoả mãn điều kiện

( a + 5b + 3c ) 
1 1 1  39
+ + =.
 a 4b 3c  4
Tìm giá trị nhỏ nhất của biểu thức
2 ( a + 3c )( a + 3c − 4b ) − 7c 2 + 4c a 2 + 7c 2

n
P= .
2ac

.v
Lời giải

om
Theo giả thiết ta có
39 1 1 1  1 1 1 1 1 1
=( a + 4b + 3c )  + +  + b  + +  ≥ 9 + b  + + 
4  a 4b 3c   a 4b 3c   a 4b 3c 
⇒ 3ac ≥ 2b ( a + 3c )
.c
1 1 1
ok
Chú ý AM – GM ta có ( a + 4b + 3c )  + +  ≥ 9.
 a 4b 3c 
Khi đó
bo

2 ( a + 3c ) − 8b ( a + 3c ) − 7c 2 + 4c a 2 + 7c 2
2

P=
et

2ac
2 ( a + 3c ) − 12ac − 7c 2 + 4c a 2 + 7c 2 2a 2 + 11c 2 + 4c a 2 + 7c 2
2
vi

≥ = .
2ac 2ac
ng

2 x 2 + 11 + 4 x 2 + 7 a
= ,x
2x c
a

2 x 2 + 11 + 4 x 2 + 7
Xét hàm số f ( x) = với x dương ta có
kh

2x

f '( x) =
( 2x 2
− 11) x 2 + 7 − 28
; f '( x) = 0 ⇔ x = 3 .
2 x2 x2 + 7
15
Ta có f’(x) đổi dấu từ âm sang dương khi đi qua x=3 nên f ( x) ≥ f (3) =.
2
Dấu bằng xảy ra khi và chỉ khi =a 4= b 3c .
15
Vậy giá trị nhỏ nhất của P bằng .
2

451
Khám phá tư duy Kỹ thuật giải bất ĐT Bài toán Max – Min – Đặng Thành Nam

Bài tập tương tự


Cho a,b,c là các số thực dương thoả mãn điều kiện

( a + 3b + c ) 
1 1 1  21
+ +  =.
 a 2b c  2
Tìm giá trị nhỏ nhất của biểu thức
a 2 + ( a + c )( a + c − 2b ) + 2a 2 + 2c 2
P= .
ac
= 5;=
ĐS: Pmin a 2=b c.

n
C. BÀI TẬP RÈN LUYỆN

.v
Bài 1. Cho x,y là hai số thực dương. Tìm giá trị lớn nhất của biểu thức
4xy 2
P=

om
.
(x + )
3
x + 4y
2 2

.c
Bài 2. Cho x,y là hai số không âm . Tìm giá trị lớn nhất của biểu thức
xy + x 4 + 9x 2 y 2
P=
ok
.
x 2 + 8y 2
Bài 3. Cho x,y là hai số thực dương thỏa mãn xy ≤ y − 1 .
bo

y3 2x 2
P
Tìm giá trị nhỏ nhất của biểu thức = + .
x3 y 2
et

Bài 4. Cho a,b,c là các số thực dương thay đổi thỏa mãn
vi

( a + 2b )  1b + 1c  =
 
4,c ≤ 3a . Tìm giá trị lớn nhất và giá trị nhỏ nhất của biểu thức
ng

 
a + 2b
2 2
P= .
ac
a

Bài 5. Cho x,y là các số thực dương. Tìm giá trị nhỏ nhất của biểu thức
kh

x3 4y3
=P + .
x3 + 8y3 y3 + ( x + y )
3

Bài 6. Cho a,b,c là các số thực dương thay đổi thỏa mãn

( a + b + c)  1a + 1b + 1c  =16 .
 
 
a2 + 2b2
Tìm giá trị lớn nhất và nhỏ nhất của biểu thức P = .
ab
452
Cty TNHH MTV DVVH Khang Việt

Bài 7. (TSĐH Khối A 2011) Cho các số thực x, y, z ∈ [1;4] , x ≥ y, z ≥ z .


x y z
Tìm giá trị nhỏ nhất của biểu thức P= + + .
2x + 3y y + z z + x

Bài 8. Cho a,b,c là các số thực dương. Tìm giá trị nhỏ nhất của biểu thức
bc 4abc
= P − .
( b + a )( c + a ) ( a + b )( b + c )( c + a )
Bài 9. Cho các số thực dương thỏa mãn điều kiện ( a + c )( b + c ) =
4c 2 .

n
a b ab
Tìm giá trị lớn nhất của biểu thức P = + + .
b + 3c a + 3c bc + ca

.v
Bài 10. Cho a,b,c là các số thực dương thỏa mãn a ≥ 2b .

om
( )
Chứng minh: 14 a 2 + b 2 + c 2 ≥ 5 ( a + b + c ) .
2

Bài 11. Cho a,b là hai số thực và c không âm thỏa mãn a 2 + b 2 + ab =


.c 3c 2 .
Chứng minh rằng a3 + b3 + 4abc ≤ 6c3 .
ok
Bài 12. Cho a,b,c là các số thực dương thay đổi thỏa mãn ( a + b − c ) =
2
ab .
2
ab c2  c 
bo

Tìm giá trị nhỏ nhất của biểu thức P = + 2 +  .


a+b a +b a+b−c
2

Bài 13. Cho các số thực không âm a,b,c thỏa mãn a + b + c > 0 .
et

a3 + b3 + 16c3
Tìm giá trị lớn nhất và nhỏ nhất của biểu thức P = .
( a + b + c )3
vi

Bài 14. Cho các số thực a,b,c.


ng

27
Chứng minh rằng 8a 4 + 8b 4 + 27c 4 ≥ ( a + b + c )4 .
64
a

( )
Bài 15. Cho a,b,c là các số thực thỏa mãn ( a + b + c )= 2 a 2 + b2 + c 2 và a + b + c ≠ 0 .
2
kh

a 3 + b3 + c 3
Tìm giá trị lớn nhất và nhỏ nhất của biểu thức P = .
( a + b + c )( ab + bc + ca )
Bài 16. Cho x,y,z là các số thực không âm đôi một phân biệt. Chứng minh rằng
x+ y y+z z+x 9
+ + ≥ .
( x − y) ( y − z ) ( z − x)
2 2 2 x + y+z

Bài 17. Cho a,b,c là các số thực không âm đôi một phân biệt thỏa mãn a + b + c =
1.

453
Khám phá tư duy Kỹ thuật giải bất ĐT Bài toán Max – Min – Đặng Thành Nam

1 1 1
Tìm giá trị nhỏ nhất của biểu thức P = + + .
(a − b) 2
(b − c ) 2
( c − a )2
Bài 18. Cho a,b,c là các số thực không âm không có hai số nào đồng thời bằng 0.
Tìm giá trị nhỏ nhất của biểu thức

( a + b + c )2  
1 1 1
P= +
. +
a +b b +c
2
c +a 
2 2 2 2 2

Bài 19. Cho a,b,c là các số thực không âm và không có 2 số nào đồng thời bằng 0.
Tìm giá trị nhỏ nhất của biểu thức

( ab + bc + ca )  
1 1 1
P= +. +

n
a +b b +c
2
c +a 
2 2 2 2 2

.v
Bài 20. Cho a,b,c là các số thực không âm không có 2 số nào đồng thời bằng 0 thỏa
mãn a + b + c = 1.

om
1 1 1
Tìm giá trị nhỏ nhất của biểu thức P = 2 + 2 + 2 .
a +b 2
b +c 2
c + a2
Bài 21. Cho các số thực a > 0, b,c ≥ 0 . Tìm giá trị nhỏ nhất của biểu thức
.c
( ab + bc + ca )  
1 1 1
P= + +
ok
.
a a +b 2
a +b +c  2 2 2 2 2

Bài 22. Cho a,b,c là các số thực không âm và c < a, c < b . Chứng minh rằng
bo

2 2
 a   b  a 2 + b2 + c2
 +
   ≥ .
b−c c−a ab + bc + ca
et

Bài 23. Cho x,y,z là các số thực thuộc đoạn [1;2] .


vi

x2 y 2 + y 2 z 2 + z 2 x2
Tìm giá trị lớn nhất và nhỏ nhất của biểu thức P = .
xyz ( x + y + z )
ng

Bài 24. Cho a,b,c là các số thực dương.


4a3 + 3b3 + 2c3 − 3b 2 c
a

Tìm giá trị nhỏ nhất của biểu thức P =


( a + b + c )3
kh

Bài 25. Cho a,b,c là độ dài một tam giác không nhọn. Chứng minh rằng

(a 2
)  1 1 1 
+ b 2 + c 2  2 + 2 + 2  ≥ 10 .
a b c 
Bài 26. Cho x,y,z là các số thực dương thỏa mãn x 2 + y 2 + 6 z 2= 4 z ( x + y ) .

x3 y3 x2 + y 2
Tìm giá trị nhỏ nhất của biểu thức P = + + .
y( x + z) x( y + z)
2 2 z

454
Cty TNHH MTV DVVH Khang Việt

Bài 27. Cho a,b,c là các số thực thỏa mãn ( a + b + c )= 4 ( ab + bc + ca ) và a + b + c ≠ 0 .


2

a 2b + b 2 c + c 2 a
Tìm giá trị lớn nhất và nhỏ nhất của biểu thức P = .
( a + b + c )3
Bài 28. Cho a, b ≥ 0, c > 0 và thoả mãn điều kiện a3 + b3 = c ( c − 1) .
a 2 + b2 + c2
Tìm giá trị lớn nhất và nhỏ nhất của biểu thức P = .
( a + b + c )2
Bài 29. Cho x,y là các số thực dương thay đổi thỏa mãn x + 2 xy ≥ 8 y .

n
 x3 y 3   x 2 y 2 
Tìm giá trị nhỏ nhất của biểu thức P = 4  3 + 3  − 9  2 + 2  .

.v
y x   y x 

Bài 30. Cho a,b,c là các số thực dương thỏa mãn 0 < 4a ≤ c ≤ 9a .

om
12a 12b 25c
Tìm giá trị nhỏ nhất của biểu thức P = + + .
a+b b+c c+a
4 3
Bài 31. Cho a,b,c là các số thực dương thỏa mãn b ≥ a − c ≥ b .
.c
5 5
ok
12 ( a − b ) 12 ( b − c ) 25 ( c − a )
Tìm giá trị lớn nhất của biểu thức P = + + .
c a b
bo

Bài 32. Cho a,b,c là các số thực không âm thay đổi thỏa mãn 2a3 − ( b + c ) ≤ 4abc .
3

7 4
( )
et

Chứng minh a + 3 b 4 + c 4 ≥ 2a 2b 2 − 3b 2 c 2 + 2c 2 a 2 .
16
vi

Bài 33. Cho a,b,c là các số thực dương thay đổi thỏa mãn ab + bc + ca =
3b 2 .
Tìm giá trị lớn nhất và nhỏ nhất của biểu thức
ng

P=
a
+
c
+
abc − ( a + c ) a 2 + c 2
.
( )
b+c a+b ( a + c ) b2
a
kh

Bài 34. Cho a,b,c là các số thực dương thay đổi thỏa mãn

( a + c ) 
1  10 1
= +
, c ≥ 4b .
a b2  b 2

a +c−b
Tìm giá trị lớn nhất và nhỏ nhất của biểu thức P = .
b
Bài 35. Cho a,b,c là các số thực không âm. Chứng minh rằng

(a )
3
≥ 27 ( a − b ) ( b − c ) ( c − a ) .
2 2 2
2
+ b2 + c2
Bài 36. Cho a,b,c là các số thực không âm thỏa mãn ab + bc + ca > 0 .

455
Khám phá tư duy Kỹ thuật giải bất ĐT Bài toán Max – Min – Đặng Thành Nam

1 1 1 11 32
Chứng minh rằng + + + ≥ .
a +b2 2
b +c2 2
c +a2 2
a +b +c
2 2 2
( a + b + c )2
Bài 37. Cho a,b,c là các số thực không âm đôi một khác nhau thỏa mãn a + b + c =0.
 1 33 1 1
Chứng minh rằng ( ab + bc + ca )2  ≥ . + +
 ( a − b )4 ( b − c )4 ( c − a )4  16
Bài 38. Tìm hằng số k tốt nhất sao cho bất đẳng thức sau đúng với mọi số thực
không âm a,b,c
( a + b + c )5 ≥ k ( a 2 + b2 + c 2 ) ( a − b )( b − c )( c − a ) .

n
Bài 39. Cho a,b,c là các số thực đôi một phân biệt. Chứng minh rằng

.v
2 2 2
 a −b  b−c   c−a 
  +  +  ≥ 5.

om
 b−c  c−a   a −b
Bài 40. Cho a,b,c là các số thực dương. Chứng minh rằng

1
+
1
+
.c
1

9
a + ab + b
2 2
b + bc + c
2 2
c + ca + a
2 2
( a + b + c )2
ok
Bài 41. (Iran 1996) Cho a,b,c là các số thực không âm. Chứng minh rằng
bo

1 1 1 9
+ + ≥ .
( a + b) (b + c ) (c + a )
2 2 2 4 ( ab + bc + ca )
et

1 1 1
Bài 42. Cho a,b,c là các số thực dương thỏa mãn 2
+ =2 .
2
a b 2c
vi

a b c
Tìm giá nhỏ nhất của biểu thức P = + + .
b+c c+a
ng

a 2 + b2 + c2
Bài 43. Cho a,b,c là các số thực dương. Chứng minh rằng
( )
27 ( a + b + c ) a 2 + b 2 + c 2 + 45abc ≥ 32 ( a + b + c )( ab + bc + ca ) .
a
kh

Bài 44. Cho a,b,c là các số thực dương. Tìm giá trị lớn nhất của biểu thức
ab bc ca 4abc
P= + + − .
( a + b ) ( b + c ) ( c + a ) ( a + b )( b + c )( c + a )
2 2 2

Bài 45. Cho x,y,z là các số thực không đồng thời bằng 0 thỏa mãn
( )
2 x 2 + y 2 + z 2 = 3 ( xy + yz + zx ) .

Tìm giá trị nhỏ nhất và lớn nhất của biểu thức P =
( y − 2 z )2 .
xy + yz + zx

456
Cty TNHH MTV DVVH Khang Việt

Bài 46. Cho a,b,c là các số thực thỏa mãn a 2 + b 2 + c 2 + 4ab + 4bc + 4ca =
0 và
a + b + c ≠ 0 . Tìm giá trị lớn nhất và giá trị nhỏ nhất của biểu thức
a3 + b3 + c3 − abc
P= .
( a + b + c )3
Bài 47. Cho a,b,c là các số thực thỏa mãn a 2 + b 2 + c 2= 4 ( ab + bc + ca ) và
ab + bc + ca ≠ 0 . Tìm giá trị lớn nhất và giá trị nhỏ nhất của biểu thức
( a − b )( b − c )( c − a ) ( a 2 + b2 + c 2 − ab − bc − ca )
P= .
( a + b + c )5

n
(
Bài 48. Cho x,y,z là các số thực dương thỏa mãn 5 x 2 + y 2 + z 2 = 6 ( xy + yz + zx ) . )

.v
Tìm giá trị lớn nhất và giá trị nhỏ nhất của biểu thức

om
1 1 1
P = ( x + y + z) + +  .
x y z
Bài 49. Cho x,y,z là các số thực không đồng thời bằng 0 và thỏa mãn
.c
( )
8 x 2 + y 2 + z 2= 11( xy + yz + zx ) .
ok
x3 + y 3 + z 3
Tìm giá trị lớn nhất, giá trị nhỏ nhất của biểu thức P = .
( x + y + z ) ( x2 + y 2 + z 2 )
bo

Bài 50. Cho a,b,c là các số thực thỏa mãn a 2 + b 2 + c 2 > 0 .


et

Tìm giá trị lớn nhất và giá trị nhỏ nhất của biểu thức
(
2 ( a + b + c ) a 2 + b 2 + c 2 − 9abc )
vi

P= .
(a )
ng

2 3
2
+b +c
2
a

C. HƯỚNG DẪN GIẢI – ĐÁP SỐ


kh

4t
Bài 1. Đặt
= x t. y, ( t > 0 ) ta có=
P f=
(t ) ta có
(t + )
3
t +42

f '(t ) =
4 ( t 2 + 4 − 3t ) ; f '(t ) = 0 ⇔ t 2 + 4 = 3t ⇔ t =
1
.
( )
3
2
t2 + 4 t + t2 + 4

457
Khám phá tư duy Kỹ thuật giải bất ĐT Bài toán Max – Min – Đặng Thành Nam

1
Ta có f’(t) đổi dấu từ âm sang dương khi đi qua t = trên khoảng ( 0;+∞ ) nên
2
1  1  1
tại t = f(t) đạt cực tiểu hay P =f (t ) ≥ f   =.
2  2 8
1
Vậy giá trị nhỏ nhất của P bằngđạt tại y = 2 x .
8
Bài 2. Nếu x =0 ⇒ P =0 . Xét x > 0 ta đặt=y t .x ( t ≥ 0 ) .

t + t 4 + 9t 2
Khi đó=
P f=
(t ) .
t2 + 8

n
.v
t + t 4 + 9t 2
Xét hàm số f (t ) = trên [ 0;+∞ ) ta có
t2 + 8

om
=
7t 2 + 72 + 8 − t 2 t 2 + 9
f '(t ) = ; f '(t ) 0
( )
( )
2
t + 9t t + 8
4 2 2
.c
⇔ 7t 2 + 72 + 8 − t 2 ( ) t +9 = 0⇔t = 6
2
2
ok
3 2
Lập bảng biến thiên suy ra P =
f (t ) ≤ f (6 2) = .
bo

4
3 2
Vậy giá trị lớn nhất của P bằng đạt tại x = 6 2 y .
et

4
2
x y −1 1  1 1  1 y
≤ 2 = − −  ≤ ⇒t = ≥ 4 .
vi

Bài 3. Theo giả thiết ta có:


y y 4  y 2 4 x
ng

2 2
Khi đó P= f (t=
) t3 + 2
. Xét hàm số f (t=
) t3 + với t ≥ 4 ta có
t t2
a

4 3t 5 − 4
f '(t=
) 3t 2 − 3= > 0, ∀t ≥ 4 do đó f(t) là hàm đồng biến trên
kh

t t3
513
[ 4;+∞ ) suy ra P = f (t ) ≥ f (4) = .
8
513 1
Vậy giá trị nhỏ nhất của P bằng đạt tại=x = ,y 2.
8 2
Bài 4. Đặt = c y.b, ( 3 x ≥ y > 0 ) khi đó từ điều kiện ta có:
a x.b,=
 1 2 ( y − 1)
( x + 2 ) 1 + =4⇒ x = .
 y y +1

458
Cty TNHH MTV DVVH Khang Việt

2 ( y − 1)
Mặt khác: 3 x ≥ y ⇔ 3. ≥ y ⇔ y2 − 5 y + 6 ≤ 0 ⇔ 2 ≤ y ≤ 3 .
y +1

 2 ( y − 1) 
2

  +2
x + 2  y +1 
2
3y2 − 2 y + 2
Khi đó
= P = = f=
( y) .
xy 2 ( y − 1) y3 − y
.y
y +1
3y2 − 2 y + 2
Xét hàm số f ( y ) = liên tục trên đoạn [ 2;3] ta có:
y3 − y

n
−3 y 4 + 4 y 3 + 3 −3 y ( y − 2 ) − 2 y + 3
3 3
=f '(y) = < 0, ∀y ∈ [ 2;3] .

.v
( ) ( )
2 2
y3 − y y3 − y

om
Do đó f(y) là hàm nghịch biến trên [ 2;3] .
11 2
Suy ra P=
max f=
(2) đạt tại
= a = b, c 2b

P=
min f=
6
(3) 1 đạt tại=a b=
3
, c 3b .
.c
ok
 1 1
Bài 6. Đặt
= .a, b y.a, ( x, y > 0 ) theo giả thiết ta có: (1 + x + y ) 1 + +  =
c x= 16 .
 x y
bo

1 + 2 y2
Khi đó
= P f=
( y) .
y
et

Để khảo sát hàm f(y) ta cần tìm miền giá trị của y trước tiên.
 1  
vi

1
Viết lại đẳng thức trên dưới dạng: 1 +  x 2 +  y + − 13  x + y + 1 =0 (1) .
 y  y 
ng

Phương trình (1) phải có nghiệm x,y dương do đó điều kiện là:

a

y > 0

kh

2
  1   1 7−3 5 7+3 5
 ∆
=  y + − 13  − 4  1 +  ( y + 1) ≥ 0 ⇔ ≤ y≤ .
  y   y  2 2
 1
 y + − 13 > 0
 y
7 − 3 5 7 + 3 5 
Việc còn lại ta chỉ khảo sát hàm số f(y) trên  ;  ta có:
 2 2 

 7 − 3 5  21 − 3 5  1 
=
Pmax f  =  = , Pmin f=
  2 2.
 2  2  2
459
Khám phá tư duy Kỹ thuật giải bất ĐT Bài toán Max – Min – Đặng Thành Nam

Bài 7. Nhận xét. Biểu thức P đẳng cấp bậc 0 và có x ≥ y, x ≥ z nên ta đặt ẩn phụ
giảm biến bằng cách biểu diễn y theo x và z theo x.
1 
Đặt
= y a= .x, z b.x do x, y, z ∈ [1;4] , x ≥ y, x ≥ z ⇒ a, b ∈  ;1 .
4 
1 a b
Khi đó P = + + .
2 + 3a a + b b + 1
Ta chưa có mối liên hệ giữa a và b nên cứ mạnh dạn xét hàm số với a hoặc b
xem sao và hy vọng được một hàm luôn đồng biến hoặc luôn ngịch biến.
1 a b 1 
Xét hàm số f (a) = + + liên tục trên  ;1 ta có:
2 + 3a a + b b + 1

n
4 

.v
3 b ( 2 + 3a ) b − 3 ( a + b ) .
2 2
f '(a ) =
− + = 2
( 2 + 3a ) ( a + b )
2 2
( 2 + 3a ) ( a + b )2

om
Ta có:
( 2 + 3a )2 b − 3 ( a + b )2= 9a 2b + 6ab + 4b − 3a 2 − 3b 2
.c
≥ 9a 2b + 6a 2b + 4b − 3a 2 − 3b 2 .
ok
1 
= 3a 2 ( 5b − 1) + b ( 4 − 2b ) > 0, ∀a, b ∈  ;1
4 
bo

1 
Do đó f(a) là hàm đồng biến trên  ;1 suy ra
4 
et

1 4 1 b
P= f (a ) ≥ f   =+ + .
 4  11 1 + 4b 1 + b
vi

4 1 b 1 
Xét hàm số g (b) =+ + liên tục trên đoạn  ;1 ta có:
ng

11 1 + 4b 1 + b 4 
1 
b∈ ;1
1 4 1
; g '(b)= 0 ⇔ (1 + 4b ) = 4 ( b + 1) ← 4 
2 2
g '(b)= − → b=
a

.
( b + 1) 2
(1 + 4b ) 2 2
kh

1
Ta có g’(b) đổi dấu từ âm sang dương khi đi qua b = nên g(b) đạt cực tiểu tại
2
1  1  34 34
b= hay g (b) ≥ g   = . Do đó P ≥ .
2  2  33 33
34 1 1
Vậy giá trị nhỏ nhất của P bằng đạt tại=
a = ,b =
hay x 4=
y, x 2 z và do
33 4 2
x, y, z ∈ [1;4] ⇒ x = 4, y = 1, z = 2 .
Bài 8. Đặt
= .a, c y.a, ( x, y > 0 ) khi đó:
b x=

460
Cty TNHH MTV DVVH Khang Việt

xy 4 xy xy  4 
P= − = 1 −
( x + 1)( y + 1) ( x + y )( x + 1)( y + 1) ( x + 1)( y + 1)  x + y 
.

Với x + y > 4 ⇒ P > 0 ta xét với x + y ≤ 4 .


xy  4 
Khi đó P
= 1 −  khi đó nếu coi vế phải làm một hàm của xy
xy + x + y + 1  x + y 
2
 x+ y
ta có một hàm nghịch biến mặt khác xy ≤   suy ra:
 2 
2
 x+ y
  ( x + y )( x + y − 4 ) .

n
 2   4 
P≥ 1 −  =2
 x + y  ( x + y) + 4( x + y) + 4

.v
2
x+ y
  + x + y +1
 2 

om
t (t − 4)
Đặt t = x + y, ( 0 < t ≤ 4 ) ⇒ P ≥ f (t ) = .
( t + 2 )2 .c
t (t − 4)
Xét hàm số f (t ) = trên ( 0;4] ta có:
( t + 2 )2
ok
( 2t − 4 )( t + 2 ) − 2 ( t 2 − 4t ) 8t − 8
bo

f '(t ) = = ; f '(t ) = 0 ⇔ t = 1 .
(t + 2)3
( t + 2 )3
Ta có f’(t) đổi dấu từ âm sang dương khi đi qua t = 1 nên f(t) đạt cực tiểu tại
et

1
t = 1 do đó P ≥ f (t ) ≥ f(1) =
− .
vi

3
1 1
Vậy giá trị nhỏ nhất của P bằng − đạt tại x = y = ⇔ a = 2b = 2c .
ng

3 2
Bài 9. Đặt
= .c, b y.c, ( x, y > 0 ) theo điều kiện bài toán ta có:
a x=
a

( x + 1)( y + 1) = 4.
kh

x y xy x2 + y 2 + 3( x + y ) xy
Khi đó: P = + + = +
y+3 x+3 x+ y xy + 3 ( x + y ) + 9 x + y

( x + y )2 + 3 ( x + y ) − 2 xy xy
+
xy + 3 ( x + y ) + 9 x+ y
Do ( x + 1)( y + 1) =4 ⇒ xy =3 − ( x + y ) .

461
Khám phá tư duy Kỹ thuật giải bất ĐT Bài toán Max – Min – Đặng Thành Nam

Đặt t = x + y, ( 0 < t < 3) ⇒ xy = 3 − t và


2
 x+ y t2 t >0
3=
− t xy ≤  = ⇔ t 2 + 4t − 12 ≥ 0 ← →t ≥ 2 .
 2  4
t 2 + 3t − 2 ( 3 − t )
3−t t 3 3
Khi đó P = = + − . +
3 − t + 3t + 9 t 2 t 2
t 3 3 Bảng biến thiên:
Xét hàm số f (t ) = + − với
2 t 2 t 2 6 3
t ∈ [ 2;3) ta có f’(t) − 0 +

n
1 3 f(t) 1 0
f '(t ) = − ; f '(t ) = 0 ⇔ t = 6 .
2 t2

.v
Dựa vào bảng biến thiên suy ra 3
6−

om
max f = = 1.
(t ) f(2) 2
t∈[ 2;+3)

Vậy giá trị lớn nhất của P bằng 1 đạt tại a= b= c .


Bài 10. Đặt a= x.c, b= y.c, ( 0 < 2 y ≤ x ) ta cần chứng minh .c
( )
14 x 2 + y 2 + 1 ≥ 5 ( x + y + 1) ⇔ 9 y 2 − 10 ( x + 1) y + 9 x 2 − 10 x + 9 ≥ 0 .
2
ok
Coi vế trái là hàm của y và x là tham số
bo

f ( y ) = 9 y 2 − 10 ( x + 1) y + 9 x 2 − 10 x + 9 ta có
18 y 10 ( x + 1) ≤ 9 x − 10 ( x + 1) =
f '( y ) =− − x − 10 < 0 nên f(y) là hàm nghịch
et

2
 x 5 
biến suy ra f ( y ) ≥ f   =  x − 3  ≥ 0 .
vi

  
2 2 
Bất đẳng thức được chứng minh.
ng

Đẳng thức xảy ra khi và chỉ khi= a 2= b,5a 6c .


2
 b  3b 2
a

Bài 11. Nếu c = 0 ⇒ a 2 + ab + b 2 = 0 ⇔  a +  + = 0 ⇔ a = b = 0 bất đẳng


 2 4
kh

thức trở thành đẳng thức.


Nếu c > 0 đặt
= a x=.c, b y.c khi đó theo điều kiện ta có:
3 và bất đẳng thức cần chứng minh trở thành:
x 2 + y 2 + xy =

x3 + y 3 + 4 xy ≤ 6 ⇔ ( x + y ) − 3 xy ( x + y ) + 4 xy ≤ 6 .
3

Đặt
t2
t = x + y ⇒ 3 = x 2 + y 2 + xy = ( x + y )2 − xy = t 2 − xy ⇒ t 2 − 3 = xy ≤ ⇔ −2 ≤ t ≤ 2 .
4

462
Cty TNHH MTV DVVH Khang Việt

Thay xy= t 2 − 3 vào bất đẳng thức trên ta cần chứng minh:

( ) ) (
t 3 − 3 t 2 − 3 t + 4 t 2 − 3 ≤ 6, ∀t ∈ [ −2;2] .

⇔ 2t 3 − 4t 2 − 9t + 18 ≥ 0, ∀t ∈ [ −2;2] ⇔ ( t − 2 ) ( 2t 2
)
− 9 ≥ 0, ∀t ∈ [ −2;2] (luôn đúng).
Bất đẳng thức được chứng minh. Đẳng thức xảy ra khi và chỉ khi a= b= c .
xy 1 1
.c, b y.c từ điều kiện ta có: ( x + y − 1) =
2
Bài 12. Đặt
= a x= xy và P = + 2 2 + .
x + y x + y xy
2
 x+ y  
Ta có: ( x + y − 1) =xy ≤   ⇒ ( x + y ) − 4 ( x + y ) − 2 ( x + y ) + 1 ≥ 0 .
2 2 2

n
 2 

.v
2
⇔ 3( x + y ) − 8 ( x + y ) + 4 ≤ 0 ⇔ ≤ x + y ≤ 2 .
2
3

om
2 
Đặt t = x + y,  ≤ t ≤ 2  thay xy= ( t − 1) vào biểu thức của P ta được:
2

3 

P=
( t − 1)2
+
1
+
1
=
.c
( t − 1)2
+
1
+
1
t − 2 ( t − 1) ( t − 1) −t + 4t − 2 ( t − 1)2
2 2 2
ok
t 2 t

( t − 1)2 1 1 1
= + + +
bo

2 ( t − 1) −t + 4t − 2 2 ( t − 1)
t 2 2 2

2 4 2 4 2 
et

≥ + = + ≥ 2, ∀t ∈  ;2 
t t − 1 −t 2 + 4t − 2 + 2 ( t − 1)2 t t −1 t2 3 
vi

Vậy giá trị nhỏ nhất của P bằng 2 đạt tại a= b= c= 1 .


Nhận xét. Ta có thể xử lý theo cách khác sau đây:
ng

ab c2 c2 ab c2 c2 c2
P= + 2 + = + + +
a + b a + b 2 ab a + b 2ab a 2 + b 2 2ab
a

2
c 2 ab 4c 2 2c  2c 
kh

≥2 + 2 ≥ + 
ab ( a + b ) a + b + 2ab a + b  a + b 
2

2
 c  ab 1 1 c 3
Xuất phát từ giả thiết ta có: 1 − =  ≤ ⇒ ≤ ≤ .
 a+b (a + b) 2 4 2 a+b 2

c 1 3
Đặt t = ) 2t + 4t 2 trên  ;  ta có kết quả tương tự như
. Xét hàm số g (t=
a+b 2 2
lời giải trên. Qua bài toán này ta có nhận xét là đôi khi sử dụng kỹ thuật giảm
biến đối với dạng đồng bậc đòi hỏi chúng ta phải biến đổi và vận dụng tinh tế
các bất đẳng thức.
463
Khám phá tư duy Kỹ thuật giải bất ĐT Bài toán Max – Min – Đặng Thành Nam

3 3 3
 a   b   c 
Bài 13. Ta có: P =   +  + 16   .
a+b+c a+b+c a+b+c
a b c
= Đặt x = ,y = ,z , ( x, y, z ≥ 0 ) và x + y + z =
1.
a+b+c a+b+c a+b+c
Khi đó P = x3 + y 3 + 16 z 3 .
Để tìm giá trị lớn nhất ta thực hiện như sau:
x3 y 3 + z 3 + 15 z 3 ≤ ( x + y + z ) + 15 z 3 =
3
P =+ 1 + 15 z 3 ≤ 16 .
Vậy giá trị lớn nhất của P bằng 16 đạt tại a= b= 0 .
Để tìm giá trị nhỏ nhất của P ta thực hiện như sau:

n
P = x3 + y 3 + 16 z 3 = ( x + y ) ( x + y ) − 3xy  + 16 z 3

.v
2
 

om
 3 2 1
≥ ( x + y ) ( x + y ) − ( x + y )  + 16 z 3 = (1 − z )3 + 16 z 3
2

 4  4
1
Xét hàm số f ( z ) = (1 − z ) + 16 z 3 liên tục trên đoạn [ 0;1] ta được:
3
4
.c
3 z∈[ 0;1] 1
f '( z )= − (1 − z ) + 48 z 2 ; f '( z )= 0 ←
ok
2
→ z= .
4 9
1  1  16  1  16
bo

=
Ta có f (0) =
,f  = , f (1) 16 . Suy ra P ≥ f ( z ) ≥ f   = .
4  9  81  9  81
16
đạt tại
et

Vậy giá trị nhỏ nhất của P bằng


81
1 c 1
vi

x = y , z = ⇔ a = b, = , a = b = 4c .
9 a+b+c 9
ng

Bài 14. Nếu a + b + c =0 , bất đẳng thức hiển nhiên đúng.


Nếu a + b + c ≠ 0 , khi đó viết lại bất đẳng thức dưới dạng:
a

4 4 4
 a   b   c  27
8  + 8  + 27   ≥ .
kh

a+b+c a+b+c a+b+c 64


a b c
Đặt x =
= ,y = ,z suy ra x + y + z =
1.
a+b+c a+b+c a+b+c
Khi đó vế trái của bất đẳng thức bằng P = 8 x 4 + 8 y 4 + 27 z 4 .
Sử dụng bất đẳng thức C-S ta có:

( ) ( ) ( )
2 2
=  2 x 2 + y 2  ≥ ( x + y ) =(1 − z ) .
4 4
8 x4 + y 4 ≥ 4 x2 + y 2
 
Suy ra P ≥ 27 z 4 + (1 − z ) .
4

464
Cty TNHH MTV DVVH Khang Việt

) 27 z 4 + (1 − z ) ta được:
4
Xét hàm số f ( z=
1
f '( z ) =−4 (1 − z ) + 27.4 z 3 ; f '( z ) =0 ⇔ (1 − z ) =27 z 3 ⇔ z = .
3 3
4
1 1
Ta có f’(z) đổi dấu từ âm sang dương khi đi qua z = nên tại z = thì f(z) đạt
4 4
 
1 27
cực tiểu hay P ≥ f ( z ) ≥ f   = .
 4  64
3c
Vậy bất đẳng thức được chứng minh. Đẳng thức xảy ra khi và chỉ khi a= b= .
2

n
Nhận xét. Trong trường hợp a,b,c dương và các hệ số của a 4 , b 4 , c 4 đôi một khác

.v
nhau ta sử dụng kỹ thuật cân bằng hệ số cho bất đẳng thức AM-GM hoặc dùng AM

om
– GM dạng luỹ thừa.

( )
Bài 15. Ta có: ( a + b + c ) = 2 a 2 + b 2 + c 2 ⇔ a 2 + b 2 + c 2= 2 ( ab + bc + ca ) .
2

⇔ ( a + b + c )= 4 ( ab + bc + ca ) .
2 .c
(
4 a 3 + b3 + c 3 ) = 4 3 3 3
ok
a   b   c 
Khi đó P =   + 4  + 4  .
( a + b + c )3 a+b+c a+b+c a+b+c
bo

a b c
Đặt x
= = ,y = ,z ta có x + y + z =
1 và
a+b+c a+b+c a+b+c
ab + bc + ca 1
et

xy +=
yz + zx = .
( a + b + c )2 4
vi

 y + z =1 − x  y + z =1 − x
 
ng

Suy ra  1⇔ 1.
 yz + x ( y + z ) = 4  yz = x − x + 4
2
a

 1 2
Mặt khác ( y + z ) ≥ 4 yz ⇒ (1 − x ) ≥ 4  x 2 − x +  ⇔ 0 ≤ x ≤ .
2 2
kh

 4  3
Khi đó :

( )
P= 4 x3 + y 3 + z 3 = 4  x3 + ( y + z ) − 3 yz ( y + z ) 

3

  1 
= 4  x3 + (1 − x ) − 3  x 2 − x +  (1 − x )=
3
 12 x − 12 x + 3x + 1
3 2
  4  
 2
Xét hàm số f ( x)= 12 x3 − 12 x 2 + 3 x + 1 liên tục trên 0;  ta được:
 3

465
Khám phá tư duy Kỹ thuật giải bất ĐT Bài toán Max – Min – Đặng Thành Nam

 1
 2

x∈0; x=
 3 6
f '( x)= 36 x 2 − 24 x + 3; f '( x)= 0 ←→  .
x = 1
 2
1 2  1  11 11
Ta có =
f (0) f=
  1, f=
  f=
  . Suy ra 1 ≤ f ( x) =≤
P .
2 3 6 9 9
Vậy giá trị nhỏ nhất của P bằng 1 đạt tại=
a 0,=
b c . Giá trị lớn nhất của P
11 a 5
bằng đạt tại b= c= hoặc b= c= a.
9 2 2

n
giả sử z min { x, y, z} ⇒ x > y > z ≥ 0 .
Bài 16. Không mất tính tổng quát =

.v
 x+ y y+z z+x 
cần chứng minh: ( x + y + z )  + +  ≥9.

om
 ( x − y )2 ( y − z )2 ( z − x )2 
Đặt P là biểu thức vế trái của bất đẳng thức.
Ta có
.c
x y
+2+
ok
 x+ y y x ( x + y )2 + x + y= y x x y
P ≥ ( x + y)  + +=  +2 + + +2.
 ( x − y )2 y 2 x 2  ( x − y )2 y x x
−2+
y y x
bo

y x
x y t+2
Đặt t =+ , ( t > 2 ) do x > y khi đó P ≥ f (=
t) +t +2.
et

y x t−2
t+2
vi

Xét hàm số f (=
t) + t + 2 trên khoảng ( 2;+∞ ) ta có
t−2
ng

4
+ 1; f '(t )= 0 ⇔ ( t − 2 ) = 4 ←
2 t >2
f '(t )= − → t= 4 .
(t − 2) 2
a

Ta có f’(t) đổi dấu từ âm sang dương khi đi qua t = 4 nên f(t) đạt cực tiểu tại
kh

9 do đó P ≥ 9 .
t = 4 hay f (t ) ≥ f (4) =
Bất đẳng thức được chứng minh.
x y
Đẳng thức xảy ra khi và chỉ khi + = 4, z = 0 hoặc các hoán vị.
y x
giả sử c min {a, b, c} ⇒ a, b > c ≥ 0 .
Bài 17. Không mất tính tổng quát =
Khi đó:

466
Cty TNHH MTV DVVH Khang Việt

 1 1 1 
P= ( a + b + c )2  + + 
 ( a − b ) 2
(b − c ) 2
(c − a) 2

a b .
  +2+ 2 2
≥ ( a + b) 
2 1 1
+ 2 +=
1
 b a + a + b + 2 a + b  + 2
 
 ( a − b ) b  b a
2 2
a a
−2+
b b2 a 2
b a
a b
Đặt t =+ , ( t > 2 ) do a,b dương phân biệt.
b a
t+2
+ ( t + 1) + 1 .
2
Khi đó P ≥ f (t=

n
)
t−2

.v
t+2
+ ( t + 1) + 1 với t > 2
2
Xét hàm số f (t=)
t−2

om
4
Ta có: f '(t ) =
− + 2 ( t + 1) ;
( t − 2 )2
f '(=
t ) 0 ←
→=
t >2
t 1+ 3 .
.c
ok
Bảng biến thiên:
t 2 1+ 3 +∞
− +
bo

f’(t) 0
f(t) +∞ +∞
et
vi

11 + 6 3
Dựa vào bảng biến thiên suy ra Pmin =min f (t ) =f(1 + 3) =11 + 6 3 đạt tại
ng

t >2
a b
+ =1 + 3, c =0 hoặc các hoán vị.
a

b a
Bài 18. Giả sử c = min {a, b, c} khi đó a, b > c ≥ 0 ta có:
kh

2 2
 c  c
a 2 + b2 ≤  a +  +  b +  ;
 2  2
2
 c
b2 + c2 ≤  b +  ;
 2
2
 c
a2 + c2 ≤  a +  ;
 2

467
Khám phá tư duy Kỹ thuật giải bất ĐT Bài toán Max – Min – Đặng Thành Nam

2
 c c  c  c
(a + b + c) 2
=  a + + b +  ≥ 4  a +  b +  .
 2 2  2  2
 
 
 c  c
Suy ra: P ≥ 4  a +  b +   .
1 1 1
+ +
 2  2   2 2 2
c 
2
  a +  
c c  c 
b +  a +  + b +  
  2  2  2  2  

c c  1 1 1  x y 4
Đặt x = b + khi đó P ≥ 4 xy  2 + 2 + 2
a+ ,y=  = 4 + + .
2 2 x y x + y 2   y x x + y

n
y x

.v
x y 4 4
Đặt t = ) 4t + với t ≥ 2 ta có f '(t ) = 4 − 2 > 0, ∀t ≥ 2
+ ≥ 2 xét hàm số f (t=
y x t t

om
nên f(t) là hàm đồng biến với t ≥ 2 . Do đó P ≥ f (t ) ≥ f (2) =
10 . Đẳng thức xảy
ra khi và chỉ khi x = y, c = 0 ⇒ a = b, c = 0 .
Vậy giá trị nhỏ nhất của P bằng 10 đạt tại=
a b=
.c
, c 0 hoặc các hoán vị.
Bài 19. Giả sử c = min {a, b, c} ta có:
ok
2 2
 c  c
a 2 + b2 ≤  a +  +  b +  ;
bo

 2  2
2
 c
b2 + c2 ≤  b +  ;
et

 2
2
 c
vi

a2 + c2 ≤  a +  ;
 2
ng

c c2 c  c c c2  c  c
ab + bc + ca = ab + ( )
a + b + +  a + b −  ≥ ab + ( )
a + b + =  a +  b + 
2 4 2 2 2 4  2  2
a

 
kh

 
 c  c  1 1 1 
Suy ra: P ≥  a +  b +  + + .
 2  2   c
2
 c
2

2
c  c 
2
 a +  b +  a +  + b +  
  2  2  2  2  

c c  1 1 1   x y 1
Đặt x = b + khi đó P ≥ xy  2 + 2 + 2
a+ ,y=  =  +  + .
2 2 x y x +y   y x x + y
2

y x

468
Cty TNHH MTV DVVH Khang Việt

x y 1 1
Đặt t = + ≥ 2 xét hàm số f (t ) = t + với t ≥ 2 ta có f '(t ) = 1 − 2 > 0, ∀t ≥ 2
y x t t
5
nên f(t) là hàm đồng biến với t ≥ 2 . Do đó P ≥ f (t ) ≥ f (2) = . Đẳng thức xảy
2
ra khi và chỉ khi x = y, c = 0 ⇒ a = b, c = 0 .
5
Vậy giá trị nhỏ nhất của P bằng đạt tại=a b= , c 0 hoặc các hoán vị.
2
Cách 2: Coi P là hàm số của c ta chứng minh P đạt giá trị nhỏ nhất khi c = 0 .
Thật vậy ta cần chứng minh:

n
( ab + bc + ca )  2 2 + 2 2 + 2 2  ≥ ab  2 2 + 2 + 2 
1 1 1 1 1 1
a +b b +c c +a  a +b b 

.v
a
 1 1 1   1 1 1 1 
⇔ c (a + b) 2 + 2 + 2 ≥ ab  2 + 2 − 2 − 2

om
2  .
a +b 2
b +c 2
c +a  a b b +c 2
a + c2 
 
 1 1 1   c2 c2 
⇔ c (a + b) 2 + +  ≥ ab +
(
 a2 a2 + c2
) ( ) 
 a + b2 b2 + c2 c2 + a 2 

.c b2 b2 + c2

ok
Bất đẳng thức cuối đúng do:
c (a + b)
≥ 0;
bo

a 2 + b2
c (a + b) abc 2 ac  a 2 + b ( a − c ) 
− =   ≥0
( ) ( )
et

a +c
2 2
a a +c
2 2 2
a a +c
2 2 2

bc b 2 + a ( b − c ) 
vi

c (a + b) abc 2   ≥0
− =
(
b2 + c2 b2 a 2 + c2 ) ( )
ng

b2 b2 + c2

 1 1 1 
Vậy ta tìm giá trị nhỏ nhất=
của Q ab  2 + 2 + 2 .
a

a +b 2
a b 
kh

1 a b 5
Tương tự trên ta có: =Q + + ≥ .
a b b a 2
+
b a
5
Vậy giá trị nhỏ nhất của P bằng đạt tại= a b= , c 0 hoặc các hoán vị.
2
Bài 20. Nhận xét. Bài toán này thực chất là một trường hợp riêng của hai bất đẳng
thức trên:
Ta có:

469
Khám phá tư duy Kỹ thuật giải bất ĐT Bài toán Max – Min – Đặng Thành Nam

( a + b + c )2  
1 1 1
P= + + 
a +b2 2
b +c
2 2
c +a 
2 2

 
2 
 c c  1 1 1 .
≥ a + +b +  + +
 2  2 2 2 2 
  a +  +  b +  
2 c c c  c
b +   a +  
 2   2   2   2  
2 1 1 1 
=( x + y )  2 + 2 + 2  ≥ 10
x y x + y 2 

n
c c
Với x = a + , y = b + , ( x, y > 0, x + y =1) và c = min {a, b, c} .
2 2

.v
1
Vậy giá trị nhỏ nhất của P bằng 10 đạt tại a= b= ,c= 0 hoặc các hoán vị.

om
2
Bài 21. Giả sử c = min {a, b, c} khi đó a 2 + b 2 ≤ a 2 + ( b + c ) và
2

a 2 + b2 + c2 ≤ a 2 + ( b + c ) .
.c 2

Suy ra:
ok

1 
bo

1 1
P ≥ ( ab + bc + ca )  2 + + 
 a ( b + c )2 a 2 + ( b + c )2 
.
1   
et

1 1 1 1 1
= bc + a ( b + c )   2 + +  ≥ a (b + c )  2 + + 
 a ( b + c ) a + ( b + c ) 
2 2 2
 a ( b + c ) a + ( b + c )2 
2 2
vi

Đặt x= a, y= b + c đưa về tìm giá trị nhỏ nhất của biểu thức quen thuộc
ng

 1 1 1 
=
Q xy  2 + 2 + 2  .
x y x + y 2 
a

Ta có:
kh

2
 x y  x y
2 +  − 5 +  + 2
= 
x y 1 y x  y x 5
Q= + + +
y x x+y  x y 2
2 + 
y x  y x
x y  2 x 2 y 
 + − 2  + − 1
=  y x  y x +5≥5
 x y 2 2
2 + 
 y x
470
Cty TNHH MTV DVVH Khang Việt

Đẳng thức xảy ra khi và chỉ khi x = y, c = 0 ⇒ a = b, c = 0 .


5
Vậy giá trị nhỏ nhất của P bằng đạt tại=
a b=
, c 0 hoặc=
a c=
,b 0 .
2
Bài 22. Bất đẳng thức cần chứng minh tương đương với:
2 2
 a   b  a +b +c
2 2 2
  +  − ≥ 0.
b−c c−a ab + bc + ca
Coi vế trái là hàm số của c ta có:

2a 2 2b 2 ( b + c ) ( a 2 + b2 + c 2 ) − 2c ( ab + bc + ca )
f '(c) = + +
( b − c )3 ( a − c )3 ( ab + bc + ca )2

n
( ) c ( a − b ) + ( b − c ) + ( c − a ) 

.v
2c a 2 + b 2 + c 2 − 2c ( ab + bc + ca )
2 2 2

> =   ≥0
( ab + bc + ca )2 ( ab + bc + ca )

om
2

Vậy vế trái là hàm đồng biến với c vậy ta chỉ cần chứng minh với c = 0 .
a2 b2 a 2 + b2
.c
Khi đó ta cần chứng minh: + ≥
b2 a2 ab

( ) ( )
ok
⇔ a 4 + b 4 ≥ ab a 2 + b 2 ⇔ ( a − b ) a 2 + ab + b 2 ≥ 0 (luôn đúng).
2

Bất đẳng thức được chứng minh. Đẳng thức xảy ra khi và chỉ khi=
a b=
,c 0 .
bo

2 2
 a   b  a 2 b2 a b a 2 + b2 a 2 + b2 + c2
Cách 2: Ta có   +  ≥ + ≥ += ≥ .
b−c c−a b2 a 2 b a ab ab + bc + ca
et

Bất đẳng thức được chứng minh.


y b.z, (1 ≤ b ≤ a ≤ 2 ) , khi đó
vi

Bài 23. Giả sử x ≥ y ≥ z. Đặt =


x a.z,=
a 2b 2 + a 2 + b 2
ng

=P f=
(a) .
ab ( a + b + 1)
a

( a + b + ab ) b2 ( a − 1) + a − b 
Ta có f '(a ) ≥ 0, ∀a ≥ b ≥ 1 nên f(a) là hàm
kh

a 2b ( a + b + 1)
2

đồng biến trên [1;2] suy ra f (b) ≤ f (a ) ≤ f (2) .

Mặt khác f=
(b)
b 4 + 2b 2
=
b2 + 2
=
( b − 1)2 + 1 ≥ 1 và
b 2 ( 2b + 1) 2b + 1 2b + 1

=
f (2)
5b 2 + 4
=
(13b − 10 )( b − 2 ) + 6 ≤ 6 .
2b ( b + 3) 10b ( b + 3) 5 5

471
Khám phá tư duy Kỹ thuật giải bất ĐT Bài toán Max – Min – Đặng Thành Nam

6
Do đó 1 ≤ P ≤ .
5
Vậy giá trị nhỏ nhất của P bằng 1 đạt tại x= y= z và giá trị lớn nhất của P bằng
6
đạt tại x= y= 2, z= 1 hoặc các hoán vị.
5
Nhận xét. Ta có thể tìm nhanh Min của P như sau:
xy xyz ( x + y + z ) suy ra P ≥ 1 .
x 2 y 2 + y 2 z 2 + z 2 x 2 ≥ xy. yz + yz.zx + zx.=
Nếu thay điều kiện x, y, z ∈ [1;2] bằng đoạn [ 2;4] ta có kết quả tương tự.

Bài 24. Sử dụng bất đẳng thức AM-GM ta được: b3 + b3 + c3 ≥ 3b 2 c .

n
Suy ra:

.v
(
4a3 + b3 + c3 + 2b3 + c3 − 3b 2 c ) ≥ 4a 3
+ b3 + c 3

om
P
( a + b + c )3 ( a + b + c )3
1
4a 3 + ( b + c )3 16a3 + ( b + c )3  a 3 1  b + c 3
.c
≥ 4 = =4  +  
( a + b + c )3 4(a + b + c)
3
 a +b+c 4 a +b+c
ok
a b+c
Đặt x
= = ,y , ( y > x > 0 ) khi đó x + y = 1 và
a+b+c a+b+c
bo

1 3 1
P ≥ 4 x3 + y 3 = f ( y ) = 4 (1 − y ) + y 3 .
4 4
et

3 1
Xét hàm số f ( y ) = 4 (1 − y ) + y 3 với y ∈ ( 0;1) ta có
4
vi

3 4
f '( y )= −12 (1 − y ) + y 2 ; f '(y)= 0 ←
2 0< y <1
→ y= .
ng

4 5
4
Ta có f’(y) đổi dấu từ âm sang dương khi đi qua y = nên f(y) đạt cực tiểu tại
5
a

4 4 4
y = hay P ≥ f ( y ) ≥ f   =
kh

.
5  5  25
4
Vậy giá trị nhỏ nhất của P bằng đạt tại b= c= 2a .
25
Bài 25. Không mất tính tổng quát giả sử a là độ dài cạnh lớn nhất khi đó góc A lớn nhất:
Tam giác không nhọn nên cos A ≤ 0 ⇒ b 2 + c 2 − a 2 ≤ 0 ⇔ b 2 + c 2 ≤ a 2 .
Đặt
= .a 2 , c 2 y.a 2 , ( x, y > 0 ) và ta có: x + y ≤ 1 .
b 2 x=
Khi đó đặt P là biểu thức vế trái của bất đẳng thức.

472
Cty TNHH MTV DVVH Khang Việt

 1 1  xy + x + y 
P = (1 + x + y ) 1 + +  = ( x + y + 1)  
 x y  xy 
( x + y )2 + x + y
≥ ( x + y + 1) . 4 =
( x + y + 1)( x + y + 4 ) .
( x + y) 2 x+ y
4

=
( x + y + 1)( x =
+ y + 4)
− 10 + 10
( x + y − 1)( x + y − 4 ) + 10 ≥ 0
x+ y x+ y
Bất đẳng thức được chứng minh.

n
Đẳng thức xảy ra khi và chỉ khi tam giác đó vuông.

.v
Bài 26. Đặt
= .z, x b.z, ( a, b > 0 ) khi đó ta có a 2 + b 2 + 6= 4 ( a + b ) và
y a=

om
a3 b3
=
P + + a 2 + b2 .
b ( a + 1) a ( b + 1)
2 2

Mặt khác a 2 + b 2 ≥
1
2
1
( a + b )2 ⇒ ( a + b )2 + 6 ≤ 4 ( a + b ) .
2
.c
ok
⇔ ( a + b − 2 )( a + b − 6 ) ≤ 0 ⇔ 2 ≤ t = a + b ≤ 6 và
4 ( a + b ) = a 2 + b 2 + 6 ≥ 2ab + 6 ⇒ ab ≤ 2 ( a + b ) − 3 .
bo

1
Sử dụng bất đẳng thức AM-GM ta có: a 2 + b2 ≥ ( a + b ) ≥ 2 và
2
et

a3 a + 1 b ( a + 1) 3a
+ + ≥
vi

b ( a + 1)
2 8 8 4

b + 1 a ( b + 1) 3b
ng

b3
+ + ≥
a ( b + 1)
2 8 8 4
a

a + b ab 1 a + b 2(a + b) − 3 1 1
Suy ra P ≥ 2 + − − ≥ 2+ − − = + 2.
kh

2 4 4 2 4 4 2
1
Vậy giá trị nhỏ nhất của P bằng + 2 đạt tại x= y= z .
2
Nhận xét. Ta có thể đánh giá như sau
3
1 1  2a + 2b + 2  1
b ( a +=
1) .2b ( a + 1)( a + 1) ≤   ≤ ( a + b ) và
2 3
2 2 3  2
1
a ( b + 1) ≤ ( a + b )3 .
2
2

473
Khám phá tư duy Kỹ thuật giải bất ĐT Bài toán Max – Min – Đặng Thành Nam

 a3 b3  a 3 + b3 1 1
Do đó P ≥ 2  +  + 2 = 2. + 2 ≥ 2. + 2 = + 2 .
 ( a + b )3
( a + b ) 
3
(a + b) 3 4 2
Bài 27. HD: Từ giả thiết ta có:
a b b c c a 1
. + . + . = .
a+b+c a+b+c a+b+c a+b+c a+b+c a+b+c 4
a b c
Đặt x =
= ,y = ,z ta có
a+b+c a+b+c a+b+c
1
x + y + z= 1, xy + yz + zx= khi đó P = x 2 y + y 2 z + z 2 x .
4

n
( a + b + c ) − 2 ( ab + bc + ca )
2
ab + bc + ca

.v
Bài 28. Ta có: P= =
1 − 2. ≤ 1.
(a + b + c) 2
( a + b + c )2

om
Vậy giá trị lớn nhất của P bằng 1 đạt tại a= b= 0, c= 1 .
2
 1+ c −1 c3
Từ điều kiện ta có: a3 + b3 = c ( c − 1) ≤ c   = ..c
 2  4
Mặt khác
ok
2  (a + b)
3
 3
a3 + b3 =( a + b ) ( a + b ) − 3ab  ≥ ( a + b ) ( a + b ) − ( a + b )  =
2 2
.
   4  4
bo

Suy ra ( a + b ) ≤ c3 ⇔ a + b ≤ c .
3

Đặt
= .c, b y.c ( x, y ≥ 0 ) ta có x + y ≤ 1 và
a x=
et

x2 + y 2 + 1 ( x + y ) − 2 xy + 1 ( x + y ) + 1
2 2
vi

2 xy
P= = ≥ −
( x + y + 1) 2
( x + y + 1) 2
( x + y + 1) ( x + y + 1)2
2
ng

1
( x + y + 1)2 xy 1 1  x + y 2 1 1 3
≥2 − ≥ −   ≥ − =
a

( x + y + 1)2 2 2 2  2  2 8 8
kh

3
Vậy giá trị nhỏ nhất của P bằng đạt tại a= b= 1, c= 2 .
8
Bài 35. Giả sử c = min {a, b, c} khi đó ( a − c ) ≤ a 2 , ( b − c ) ≤ b 2 .
2 2

( ) ≥ 27 ( a − b ) a b .
3 2
Vậy ta đi chứng minh: a 2 + b 2 + c 2 2 2

Để ý ( a + b + c ) ≥ ( a + b ) . Vậy bất đẳng thức được chứng minh nếu bất


2 2 2 3 2 2 3

đẳng thức sau đúng: ( a + b ) ≥ 27 ( a − b ) a b .


2 3
2 2 2 2

474
Cty TNHH MTV DVVH Khang Việt

Nếu b = 0 bất đẳng thức trở thành đẳng thức.


Nếu b > 0 đặt
= a x.b, ( x ≥ 0 ) đưa về chứng minh bất đẳng thức.

( x + 1) ( ) (x )
3 2
≥ 27 ( x − 1) x 2 ⇔ x 2 − 3 x + 1
2
2 2
+ 6 x + 1 ≥ 0 (luôn đúng).

3± 5 3± 5
Đẳng thức xảy ra khi x = ,c = 0 ⇔ a = b,c = 0 .
2 2
3± 5
Bài toán được chứng minh. Đẳng thức xảy =
ra khi a = b,c 0 hoặc các
2
hoán vị.

n
Nhận xét. Viết lại bất đẳng thức dưới dạng.

(a )

.v
3
− 27 ( a − b ) ( b − c ) ( a − c ) ≥ 0 .
2 2 2
f (c ) = 2
+ b2 + c2

om
Coi vế trái bất đẳng thức là hàm số của c.

( )
3
+ 54 ( a − b ) [b − c + a − c ] ≥ 0 (do
2
Ta có f '(=
c) 6c a 2 + b 2 + c 2

c min {a, b, c} ≥ 0 ).
=
.c
( )
3
− 27 a 2b 2 ( a − b ) .
ok
2
Vậy f (c) ≥ f (0) = a 2 + b 2

( )
3
− 27 a 2b 2 ( a − b ) ≥ 0.
2
Bài toán đưa về chứng minh bất đẳng thức a 2 + b 2
bo

Đây chính là kết quả của cách trên. Trong trường hợp a,b,c là các số thực bất kỳ
ta có bất đẳng thức sau:
et

(a )
3
≥ 2 ( a − b) (b − c ) (c − a ) .
2 2 2
2
+ b2 + c2
vi

Để chứng minh bất đẳng thức trên ta tìm cách đánh giá ( a − c ) ( b − c ) ≤ a 2b 2 .
2 2
ng

Không mất tính tổng quát giả sử c là số nằm giữa a và b.


TH1: Nếu c ≥ 0 ta có ngay điều phải chứng minh.
a

TH2: Nếu c ≤ 0 và a, b ≤ 0, ta đặt x = − a, y =


−b, z =−c đưa bất đẳng thức về
kh

trường hợp 1 và có ngay điều phải chứng minh.


TH3: Nếu c ≤ 0, ab ≤ 0, ta có thể giả sử c ∈ [ a; b ] , khi đó xét
f (c) = ( c − a )( c − b ) + ab = c 2 − ( a + b ) c + 2ab trên đoạn [ a; b ] ta có:
f (c) ≤ max { f (a), f (b)} =
ab tức ( c − a )( c − b ) ≤ −ab . Mặt khác
( c − a )( c − b )= c ( c − a − b ) + ab ≥ ab .
Vậy ( c − a )( c − b ) ≤ ab ⇒ ( c − a ) ( c − b ) ≤ a 2b 2 .
2 2

475
Khám phá tư duy Kỹ thuật giải bất ĐT Bài toán Max – Min – Đặng Thành Nam

( )
3
≥ 2 ( a − b ) a 2b 2 .
2
Vậy đưa về chứng minh bất đẳng thức: a 2 + b 2 + c 2
Đặt a = x.b và đưa về chứng minh bất đẳng thức:

( x + 1) (x )
3
≥ 2 ( x − 1) x 2 ⇔ ( x + 1)
2 2
2 4
− 2 x3 + 4 x 2 − 2 x + 1 ≥ 0 .

2
( ) 
2
⇔ ( x + 1)  x 2 − x + 3 x 2 − 2 x + 1 ≥ 0 (luôn đúng).
 
Bài toán được chứng minh. Đẳng thức xảy ra khi a = −b, c =
0 hoặc các hoán vị.
Bài 36. Không mất tính tổng quát giả sử c = min {a, b, c} , khi đó ta có:

n
2 2
 c  c
a 2 + b2 ≤  a +  +  b +  ;

.v
 2   2 
2
 c

om
b2 + c2 ≤  b +  ;
 2
2
 c
a2 + c2 ≤  a +  ; .c
 2
2 2
 c  c
ok
a 2 + b2 + c2 ≤  a +  +  b +  .
 2  2
bo

c c
Đặt x =a+ ,y= b + , ( x, y > 0 ) bất đẳng thức được chứng minh nếu ta chứng
2 2
1 1 12 32
minh được: 2 + 2 + 2 ≥
et

.
x y x +y 2
( x + y )2
vi

Sử dụng bất đẳng thức AM-GM ta có:


 1 4 
ng

1 1 12 1 8 4 8
+ + =  2 + 2 + 2 2 
 + 2 ≥ + 2
x 2
y 2
x +y
2
x
2
y x +y  x +y 2 xy x + y 2
a

8 8 4 32
= + 2 ≥ 8. =
2 xy x + y 2
2 xy + x + y
2 2
( x + y )2
kh

Bài toán được chứng minh. Đẳng thức xảy ra khi và chỉ khi=
a b=
, c 0 hoặc các
hoán vị.
Bài 39. Không mất tính tổng quát giả sử a > b > c, đặt
a = x + y + c, b = y + c, ( x, y > 0 ) .
Gọi P là biểu thức vế trái của bất đẳng thức cần chứng.
y2
Ta có P = +
x2
+
( x + y) . 2

x 2 ( x + y )2 y2

476
Cty TNHH MTV DVVH Khang Việt

Đặt y = t.x ta có P =
t + 2 1
+
( t + 1)
2
.
( t + 1)2 t2

Ta cần chứng minh: t + 2 1


+
( t + 1)
2
≥ 5.
( t + 1)2 t2

( )
2
⇔ t 6 + 2t 5 − 3t 4 − 6t 3 + 2t 2 + 4t + 1 ≥ 0 ⇔ t 3 + t 2 − 2t − 1 ≥ 0 (luôn đúng).

a −b b−c c−a
Đặt x
Cách 2:= = ,y = ,z , khi đó:
b−c c−a a −b

n
 1
x + y + 1 = 0

.v
  xy + y + 1 = 0
 1 
 y + + 1 = 0 ⇔  yz + z + 1 = 0 ⇒ xy + yz + zx + x + y + z + 3 = 0 .

om
 z 
 1  zx + x + 1 = 0
 z + + 1 =0
 x .c
Ta cần chứng minh x 2 + y 2 + z 2 ≥ 5 ⇔ ( x + y + z ) − 2 ( xy + yz + zx ) − 5 ≥ 0
2
ok
⇔ ( x + y + z ) − 2 ( −3 − x − y − z ) − 5 ≥ 0 ⇔ ( x + y + z + 1) ≥ 0 (luôn đúng).
2 2

Nhận xét. Tử cách 2 ta có một đẳng thức đẹp mặt sau đây:
bo

2 2 2 2
 a −b  b−c   c−a   a −b b−c c−a 
  +  +  = 5+ + + + 1 .
 b−c  c−a   a −b  b−c c−a a −b 
et

Bài 41. Nhận xét. Đây là một bài toán khó và đã có nhiều lời giải dành cho bất
vi

đẳng thức này.


Dưới đây trình bày một lời giải khác dựa vào dấu hiệu đồng bậc của bất đẳng
ng

thức trên.
Giả sử c max {a, b, c} ⇒ c > 0 .
=
a

Đặt
= .x, b c.y, ( x, y ∈ [ 0;1]) khi đó ta cần chứng minh
a c=
kh

1 1 1 9
+ + ≥ .
( x + y) 2
( x + 1) 2
( y + 1) 2 4 ( xy + x + y )
Quy đồng hai vế và rút gọn đưa về bất đẳng thức:

 (  ) ( )
4  x + y + x5 + y 5 + xy x 4 + y 4  + 2 xy 1 + x3 + y 3 + 6 x 2 y 2

≥ x2 + y 2 + x4 + y 4 + 6( x 3
+ y + x y ) + 2 xy  x + y + x
3 3 3 2
(
+ y 2 + xy ( x + y )  + x 2 y 2 x 2 + y 2 )
 

(
Ta có: 4 x + x5 − x 2 ) − x − 6 x = x ( x − 1) ( 4 x
4 3 2 2
)
+ 7 x + 4 ≥ 0, ∀x ∈ [ 0;1] do đó

477
Khám phá tư duy Kỹ thuật giải bất ĐT Bài toán Max – Min – Đặng Thành Nam

( )
4 x + y + x5 + y 5 ≥ x 2 + y 2 + x 4 + y 4 + 6 x3 + y 3 . ( )
Vậy ta chỉ cần chứng minh
( ) ( ) (
4 x 4 + y 4 + 2 1 + x3 + y 3 + 6 xy ≥ 2 x + y + x 2 + y 2 + xy ( x + y ) + 6 x 2 y 2 + xy x 2 + y 2 . ) ( )
Ta có:
(
4 x +y4 4
) = 3( x 4
+y 4
)+ x 4
( ) (
+ y ≥ 3.2 x 2 y 2 + xy x 2 + y 2 = 6 x 2 y 2 + xy x 2 + y 2 .
4
)
Vậy ta chứng minh 1 + x3 + y 3 + 3 xy ≥ x + y + x 2 + y 2 + xy ( x + y ) .
Ta có thể giả sử x ≥ y ⇒ 0 ≤ y ≤ x ≤ 1 và bất đẳng thức tương đương với:

n
y ( x − 1) + (1 − x )( x − y ) + (1 − y )  + (1 − x ) 1 − x + 2 ( x − y )  ≥ 0 (luôn đúng).
2 2 2
 

.v
Vậy bất đẳng thức được chứng minh.

om
Bài 42. Đặt
= .c, b y.c, ( x, y > 0 ) , khi đó theo giả thiết ta có:
a x=
1 1 1 x2 y 2
+ = ⇒ x 2
+ y 2
= .
x2 y2 2 2 .c
 2 x2 y 2
ok
 x + y 2
≥ 2 xy ⇒ ≥ 2 xy ⇔ xy ≥ 4
 2
Sử dụng bất đẳng thức AM-GM ta có:  .
2 2
x y 2 1
 2 = x + y ≥ 2 ( x + y ) ⇔ x + y ≤ xy
2
bo

x y 1
Khi đó: P = + +
et

.
y +1 x +1 x2 + y 2 + 1
vi

x y x2 + y 2 + x + y x2 y 2 + 2 ( x + y ) x 2 y 2 + 2 xy x 2 y 2 + 2 xy
=
+ = ≥ =
y +1 x +1 xy + x + y + 1 2 xy + 2 ( x + y ) + 2 2 xy + 2 xy + 2 4 xy + 2
ng

(hàm nghịch biến với x+y).


x 2 y 2 + 2 xy 1
a

Do đó P ≥ + .
4 xy + 2 x2 y 2
kh

+1
2
t 2 + 2t 1
Đặt xy = t , ( t ≥ 4 ) ⇒ P ≥ f (t ) = + .
4t + 2 2
t
+1
2
t 2 + 2t 1
Xét hàm số=
f (t ) + với t ≥ 4 ta có:
4t + 2 t 2
+1
2

478
Cty TNHH MTV DVVH Khang Việt

(t ) (t ) − 2t ( 2t + 1)
3 2
t2 + t +1 2t
2
+ t +1 2
+2
=
f '(t ) − = > 0, ∀t ≥ 4 .
( 2t + 1)2 ( t2 + 2
3
) ( 2t + 1) 2
(t + 2)
2 3

5
Do đó f(t) là hàm đồng biến với t ≥ 4 ⇒ P ≥ f (t ) ≥ f (4) = .
3
5
Vậy giá trị nhỏ nhất của P bằng đạt tại a= b= 2c .
3
Bài 43. Chia hai vế bất đẳng thức cho ( a + b + c ) đưa về chứng minh bất đẳng
3

( )

n
thức: 27 x 2 + y 2 + z 2 + 45 xyz ≥ 32 ( xy + yz + zx )

.v
( )
P 27 x 2 + y 2 + z 2 + xy ( 45 z − 32 ) − 32 z ( x + y ) ≥ 0 .
⇔=

om
a b c
Trong
= đó x = ,y = ,z ; x=
+ y + z 1.
a+b+c a+b+c a+b+c
1 .c
giả sử z min { x, y, z} ⇒ z ≤ ⇒ 45 z − 32 < 0 .
Không mất tính tổng quát =
3
ok
Khi đó sử dụng bất đẳng thức AM-GM ta được:
2 ( x + y)
2
27
P ≥ 27 z + ( x + y ) +
2
( 45 z − 32 ) − 32 z ( x + y )
bo

2 4
2 (1 − z )
2
27 1
= 27 z + (1 − z ) +
2
( 45 z − 32 ) − 32 z (1 − z=) ( 3z − 1)2 ( 5 z + 22 ) ≥ 0
et

2 4 4
Bài toán được chứng minh. Đẳng thức xảy ra khi và chỉ khi a= b= c .
vi

Bài 44. Đặt


= .c, b y.c, ( x, y > 0 ) ta có:
a x=
ng

xy x y 4 xy
P= + + − .
( x + y) 2
( x + 1) 2
( y + 1) 2
( x + y )( x + 1)( y + 1)
a

Nhận xét. Do biểu thức của P đối xứng với a,b,c nên dự đoán đẳng thức xảy ra khi
kh

a = b = c ⇔ x = y =1 .
1
Vậy ta chứng minh P ≤ .
4
xy x y 4 xy 1
+ + − ≤
( x + y) 2
( x + 1) 2
( y + 1) 2
( x + y )( )( ) 4
x + 1 y + 1

xy ( x + 1) ( y + 1) + x ( y + 1) ( x + y ) + y ( x + 1) ( x + y ) − 4 xy ( x + y )( x + 1)( y + 1) 1
2 2 2 2 2 2
⇔ ≤
( x + y )2 ( x + 1)2 ( y + 1)2 4

479
Khám phá tư duy Kỹ thuật giải bất ĐT Bài toán Max – Min – Đặng Thành Nam


(x 2
y + xy + x + y 2 )( xy 2
+ xy + y + x 2 ) ≤ 1 ⇔ ( x − 1) ( y − 1)2 ( x − y )2 ≥ 0
2

( x + y )2 ( x + 1)2 ( y + 1)2 4 ( x + y )2 ( x + 1)2 ( y + 1)2


(luôn đúng).
1
Vậy giá trị lớn nhất của P bằng đạt tại a= b= c .
4
Cách 2: Ta biến đổi đưa P về dạng:
3 ab 1 bc 1 ca 1 4abc
P= + − + − + − −
4 (a + b) 2 4 (b + c ) 2 4 (c + a) 2 4 ( a + b )( b + c )( c + a )

3 1  (a − b) ( b − c )2 + ( c − a )2 + 

n
2
16abc
=−  + 
4 4  ( a + b )2 ( b + c )2 ( c + a )2 ( a + b )( b + c )( c + a ) 

.v
 
a −b b−c c−a

om
2a 2b 2c
Đặt=
x =
,y =
,z ⇒ x +=
1 , y +=
1 , z +=
1 .
a+b b+c c+a a+b b+c c+a
Do đó :
3 1
P = −  x 2 + y 2 + z 2 + 2 ( x + 1)( y + 1)( z + 1) 
4 4 
.c
ok
3 1
= −  x 2 + y 2 + z 2 + 2 (1 + x + y + z + xy + yz + zx + xyz ) 
4 4 

(
bo

3 1 1
= − x 2 + y 2 + z 2 + 2 xy + 2 yz + 2 zx + 2 = − ( x + y + z ) ≤
4 4 4
) 2 1
4
Nhận xét. Lời giải trên ta sử dụng đẳng thức quen thuộc:
et

a −b b−c c−a a−b b−c c−a


x + y + z + xyz
= + + + . . = 0
vi

a+b b+c c+a a+b b+c c+a


Bài 45. HD: Theo giả thiết ta có:
ng

xy + yz + zx 2
2 ( x + y + z )= 7 ( xy + yz + zx ) ⇔
2
= .
( x + y + z )2 7
a

a + b + c = 1
kh

x y z 
Đặt a
= = ,b = ,c ⇒ 2 (1) .
x+ y+z x+ y+z x+ y+z ab + bc + ca =
7
7 ( y − 2z )
2
7
Khi đó=
P = ( b − 2c )2 .
2( x + y + z )
2 2
ab + bc + ca 1
Bài 46. Theo giả thiết ta có: ( a + b + c ) =−2 ( ab + bc + ca ) ⇒
2
=− .
(a + b + c) 2 2

480
Cty TNHH MTV DVVH Khang Việt

x + y + z = 1
a b c 
Đặt x
= = ,y = ,z ⇒ 1.
a+b+c a+b+c a + b + c  xy + yz + zx =

 2
 1  1 − 10 1 + 10
Ta có: ( y + z ) ≥ 4 yz ⇒ (1 − x ) ≥ 4  − − x (1 − x )  ⇔
2 2
≤x≤ .
 2  3 3
Khi đó :
P = x3 + y 3 + z 3 − xyz = x3 + ( y + z ) − 3 yz ( y + z ) − xyz
3

 1   1 
= x3 + (1 − x ) − 3  − − x (1 − x )  (1 − x ) − x  − − x (1 − x ) 
3

 2   2 

n
4 x3 − 4 x 2 − 2 x + 5

.v
=
2

om
4 x3 − 4 x 2 − 2 x + 5 1 − 10 1 + 10 
Xét hàm số f ( x) = liên tục trên đoạn  ;  ta có:
2  3 3 
.c 2 − 10
x =
6
f '( x) =6 x 2 − 4 x − 1; f '( x) =0 ⇔  .
ok
 2 + 10
x =
 6
bo

Bài 47. HD: Ta có: P =


( a − b )5 + ( b − c )5 + ( c − a )5 .
5( a + b + c)
5
et

a −b b−c c−a
Đặt
= x = ,y = ,z ⇒ x +=
y+z 0.
vi

a+b+c a+b+c a+b+c


Theo giả thiết ta có:
ng

( a − b )2 + ( b − c )2 + ( c − a )2 = ( a + b + c )2 ⇒ x 2 + y 2 + z 2 = 1.
Vậy bài toán đưa về tìm giá trị lớn nhất và nhỏ nhất của biểu thức
a

 x + y + z =0
kh

Q = x5 + y 5 + z 5 với  2 .
 x + y + z =
2 2
1

Bài 48. Theo giả thiết ta có: 5 ( x + y + z=


) 16 ( xy + yz + zx )
2

x y y z z x 5
⇔ . + . + . = .
x + y + z x + y + z x + y + z x + y + z x + y + z x + y + z 16
 5
x y z ab + bc + ca =
Đặt a
= = ;b = ;c ⇒ 16 (1) .
x+ y+z x+ y+z x+ y+z a + b + c =
1

481
Khám phá tư duy Kỹ thuật giải bất ĐT Bài toán Max – Min – Đặng Thành Nam

Khi đó P
=
( x + y + z )( xy
=
+ yz + zx ) 5( x + y + z )
=
3
5
.
xyz 16 xyz 16abc
Bài toán đưa về tìm giá trị lớn nhất, giá trị nhỏ nhất của biểu thức P với điều
kiện (1).
b + c =1 − a

Giả thiết ta có:  5 5 5 .
bc = 16 − a ( b + c ) = 16 − a (1 − a ) = a − a + 16
2

5
) a  − a (1 − a=
) a  a 2 − a +  .
5 5
Ta có: =Q abc = a  − a ( b + c=
 16   16   16 

n
Mặt khác:

.v
 5 1 1
4bc ≤ ( b + c ) ⇔ 4  a 2 − a +  ≤ (1 − a ) ⇔ 12a 2 − 8a + 1 ≤ 0 ⇔ ≤ a ≤ .
2 2

 16  6 2

om
 5 1 1 
a ) a  a 2 − a +  liên tục trên đoạn  ;  ta có:
Xét hàm số f (=
 16  6 2

 a=
1
.c
5 4 .
ok
f '(a ) =3a 2 − 2a + ; f '(a ) =0 ⇔ 
16 a = 5
 12
bo

1  5  25 1 1 1


Ta có f=  f=   ; f=  f=   .
6  12  864  2   4  32
et

1 25 54
Suy ra MaxQ = ⇒ MinP = 10 và MinQ = ⇒ MinP = .
32 864 5
vi

Vậy giá trị nhỏ nhất của P bằng 10 đạt tại y= z= 2 x .


ng

54 5
Giá trị lớn nhất của P bằng đạt tại y= z= x.
5 2
a

Bài 49. Theo giả thiết ta có:


kh

xy yz zx 8
8 ( x + y + z=
) 27 ( xy + yz + zx ) ⇔
2
+ + = .
(x + y + z) 2
(x + y + z) 2
( x + y + z ) 27
2

 8
x y z ab + bc + ca =
Đặt a
= = ;b = ;c ⇒ 27 (1) .
x+ y+z x+ y+z x+ y+z a + b + c =
1

Khi đó P
=
8
.
x3 + y 3 + z 3
= =
27 x3 + y 3 + z 3 27 a + b + c
.
3 3 3
.
( )
13 ( x + y + z )( xy + yz + zx ) 13 ( x + y + z )3 13

482
Cty TNHH MTV DVVH Khang Việt

 8 8 8
bc = − a (b + c ) = − a (1 − a ) = a 2 − a +
Từ hệ điều kiện (1) ta có:  27 27 27 .
b + c =1 − a

 8  1 5
Mặt khác: ( b + c ) ≥ 4bc ⇔ (1 − a ) ≥ 4  a 2 − a +  ⇔ ≤ a ≤ .
2 2

 27  9 9
Khi đó:
27  3 27  3  8 
c )
a + ( b + c ) − 3bc ( b + = a + (1 − a ) − 3 (1 − a )  a 2 − a +  
3 3
=
P 
13   13   27  

=
3
(
27 a3 − 27 a 2 + 8a + 1 )

n
13

.v
Xét hàm số =
f (a)
3
13
( ) 1 5
27 a3 − 27 a 2 + 8a + 1 liên tục trên đoạn  ;  ta có:
9 9

om
 2
 a=
3
13
( )
f '(a ) = 81a 2 − 54a + 8 ; f '(a ) =⇔
0 
a = 4
9
. .c
 9
ok
1  4  43  5   2  47
Ta có f=   f=   ; f=
  f=   .
 
9  
9 117  9  9  119
bo

47 43
Suy= ra Pmax = ;Pmin .
117 117
et

Bài 50. Đặt


a b c
= = = ⇒ x2=
+ y2 + z2 1 .
vi

x ,y ,z
a +b +c
2 2 2
a +b +c
2 2 2
a +b +c
2 2 2
ng

Khi đó P= 2 ( x + y + z ) − 9 xyz= x ( 2 − 9 yz ) + 2 ( y + z ) .

Không mất tính tổng quát= { }


x2 + y 2 + z 2 1
giả sử x 2 max x 2 , y 2 , z 2 ⇒ x 2 ≥ =
a

.
3 3
kh

Ta có
 x ( 2 − 9 yz ) + 2 ( y + z )  ≤  x 2 + ( y + z )  . ( 2 − 9 yz ) + 4=
 ( 2 yz + 1) (81y 2 z 2 − 36 yz + 8)
2 2 2
  
y 2 + z 2 1 − x2 1
Đặt t = yz ⇒ t ≤ = ≤ .
2 2 3

( )
 1 1
Khi đó P ≤ f (t ) = ( 2t + 1) 81t 2 − 36t + 8 với t ∈  − ;  ta có:
 3 3

483
Khám phá tư duy Kỹ thuật giải bất ĐT Bài toán Max – Min – Đặng Thành Nam

 2
 t= −
9
f '(t ) =486t 2 + 18t − 20; f '(t ) =0 ⇔  .
t = 5
 27
 1  29  2  100  5  1369  1  25
Ta có f  − =
 ; f − =
 ;f =  ;f=  .
 3 3  9 9  27  243  3  3
 2  100  5  1369
Suy ra Max f (t ) = f  −  = ; Min f (t ) = f   = .
 1 1
t∈ − ;  9  9  1
t∈ − ;
1   27  243
 3 3   3 3 

n
10
Vậy giá trị lớn nhất của P bằng đạt tại chẳng hạn a =
−1, b =
c=2 và giá trị
3

.v
10
nhỏ nhất của P bằng − .

om
3

CHỦ ĐỀ 5: KỸ THUẬT SỬ DỤNG BẤT ĐẲNG THỨC TIẾP TUYẾN .c


A. NỘI DUNG PHƯƠNG PHÁP
ok
Định lý 1(Bất đẳng thức tiếp tuyến) Cho hàm số f(x) liên tục và có đạo hàm cấp
2 trên đoạn [ a; b ] .
bo

Nếu f ''( x) ≥ 0, ∀x ∈ [ a; b ] ta luôn có f ( x) ≥ f '( x0 ) ( x − x0 ) + f ( x0 ), ∀x0 ∈ [ a; b ] .


Nếu f ''( x) ≤ 0, ∀x ∈ [ a; b ] ta luôn có f ( x) ≤ f '( x0 ) ( x − x0 ) + f ( x0 ), ∀x0 ∈ [ a; b ] .
et

Đẳng thức xảy ra trong hai bất đẳng thức trên ⇔ x =x0 .
vi

Chứng minh.
) f ( x) − f '( x0 ) ( x − x0 ) − f ( x0 ), x ∈ [ a; b ] , ∀x0 ∈ [ a; b ] .
Xét hàm số g ( x=
ng

'( x) f '( x) − f '( x0 ) do f ''( x) ≥ 0, ∀x ∈ [ a; b ] nên f’(x) là hàm đồng


Ta có g=
biến trên [ a; b ] . Do đó g’(x) đổi dấu từ âm sang dương khi đi qua x0 nên g(x)
a
kh

đạt cực tiểu tại x0 hay g ( x) ≥ g ( x0 ) .


⇔ f ( x) − f '( x0 ) ( x − x0 ) − f ( x0 ) ≥ 0 ⇔ f ( x) ≥ f '( x0 ) ( x − x0 ) + f ( x0 ), ∀ x ∈ [ a; b ] .
Chứng minh tương tự.
=
Nhận xét. Hệ thức y f '( x0 ) ( x − x0 ) + f ( x0 ) chính là phương trình tiếp tuyến của
đồ thị hàm số f(x) tại điểm x0 . Do vậy nếu f ''( x) ≥ 0, ∀x ∈ [ a; b ] thì tiếp tuyến với
đồ thị hàm số tại điểm bất kỳ trên đoạn [ a; b ] luôn nằm phía dưới đồ thị hàm số.
Nếu f ''( x) ≤ 0, ∀x ∈ [ a; b ] thì tiếp tuyến với đồ thị hàm số tại điểm bất kỳ trên đoạn
[ a; b] luôn nằm phía trên đồ thị hàm số.
484
Cty TNHH MTV DVVH Khang Việt

Dấu hiệu nhận biết


Thông thường bài toán được phát biểu dưới dạng
Cho các số thực x1 , x2 ,..., xn thỏa mãn g ( x1 ) + g ( x2 ) + ... + g ( xn ) ≥ k bài toán
yêu cầu chứng minh
f ( x1 ) + f ( x2 ) + ... + f ( xn ) ≥ M hoặc f ( x1 ) f ( x2 )... f ( xn ) ≥ M .
Ta có thể áp dụng trực tiếp các định lý 1 và 2 hoặc lấy logarit hóa tự nhiên hai
vế bất đẳng thức.
Kỹ năng cần vận dụng
Viết phương trình tiếp tuyến của hàm số f(x) tại điểm x0 (điểm dấu bằng xảy ra

n
k
thường là x0 = ) có phương trình:= y f '( x0 )(x − x 0 ) + f ( x0 ) .
n

.v
Xét tính dương âm của f ( x) − [ f '( x0 )(x − x 0 ) + f ( x0 ) ] .

om
Thay x bởi x1 , x2 ,..., xn cuối cùng cộng lại theo vế n bất đẳng thức trên ta có
ngay điều phải chứng minh.
Chú ý. Trong một số trường hợp không áp dụng trực tiếp được cách trên ta cần xét
.c
trường hợp hoặc xây dựng một hàm phụ phù hợp với điều kiện( f ''(x) ≥ 0,f''(x) ≤ 0 )
và điều kiện dấu bằng xảy ra.
ok
Ví dụ 1. Cho a,b,c là các số thực thỏa mãn điều kiện a + b + c = 6.
( )
bo

Chứng minh rằng a 4 + b 4 + c 4 ≥ 2 a3 + b3 + c3 .


Lời giải
et

Nhận xét. Dấu bằng xảy ra khi a= b= c= 2 nên ta viết phương trình tiếp tuyến với
đồ thị hàm số f ( x=
) x 4 − 2 x3 tại điểm x = 2 .
vi

Phương trình tiếp tuyến là =


y 8 x − 16 .
ng

Ta có: f ( x) − 8 x + 16= ( x − 2 )2 ( x 2 − 2 x + 4 ) ≥ 0, ∀x ∈  .
Do đó f (a ) + f (b) + f (c) ≥ 8 ( a + b + c ) − 3.16 =
a

0.

( )
kh

Vì vậy a 4 + b 4 + c 4 ≥ 2 a3 + b3 + c3 .
Bài toán được chứng minh. Đẳng thức xảy ra khi và chỉ khi a= b= c= 2 .
Ví dụ 2. Cho a,b,c là các số thực không âm thỏa mãn điều kiện a + b + c =
1.
a b c 9
Chứng minh rằng 2 + 2 + 2 ≤ .
a +1 b +1 c +1 10
Lời giải

Xét hàm số f ( x) =
x
với x ∈ [ 0;1] ta có
= f ''( x)
(
2 x x2 − 3 ) ≤ 0, ∀x ∈ [0;1] .
x +1
( x + 1)
2 2 2

485
Khám phá tư duy Kỹ thuật giải bất ĐT Bài toán Max – Min – Đặng Thành Nam

Áp dụng bất đẳng thức ii.định lý 1 ta được:


 1  1 1
f (a) ≤ f'   a −  + f  
 3  3 3
 1  1 1
f (b) ≤ f '   b −  + f  
 3  3 3
 1  1 1
f (c) ≤ f'   c −  + f  
 3  3 3
Cộng theo vế 3 bất đẳng thức trên ta được:
1 1 9
= f (a ) + f (b) + f (c) ≤ f '   ( a + b + c − 1) + 3 f
VT  =  .

n
3  3  10

.v
1
Đẳng thức xảy ra khi và chỉ khi a= b= c= .
3

om
3
Nhận xét. Bất đẳng thức vẫn đúng trong trường hợp a, b, c ≥ − .
4
1
.c
Thật vậy xét tiếp tuyến của f(x) tại điểm x = có phương trình là:
3
ok
18 3
= y x+ .
25 50
bo

18
Ta có: f ( x) − x −
3
=−
( 3x − 1) ( 4 x + 3) ≤ 0, ∀x ≥ − 3 .
2

25 50 50 x 2 + 1 ( ) 4
et

18 3 9
Do đó VT
= f (a ) + f (b) + f (c) ≤ ( a + b + c ) + 3. = .
vi

50 50 10
ng

Định lý 2 (Bất đẳng thức cát tuyến) Cho hàm số f(x) liên tục và có đạo hàm cấp 2
trên đoạn [ a; b ] .
a

f (a) − f (b)
Nếu f ''( x) ≥ 0, ∀x ∈ [ a; b ] ta luôn có f ( x) ≥ ( x − a ) + f (a), ∀x ∈ [ a; b] .
kh

a −b
f (a) − f (b)
Nếu f ''( x) ≤ 0, ∀x ∈ [ a; b ] ta luôn có f ( x) ≤ ( x − a ) + f (a), ∀x ∈ [ a; b] .
a −b
Đẳng thức xảy ra trong hai bất đẳng thức trên ⇔ x = a hoặc x = b .
Việc chứng minh hai bất đẳng thức trên chỉ cần chuyển vế và khảo sát hàm
trực tiếp.
Ta có một bất đẳng thức khác hay được sử dụng

486
Cty TNHH MTV DVVH Khang Việt

Định lý 3.(Bất đẳng thức Jensen) Cho hàm số f ( x) liên tục và có đạo hàm cấp
hai trên khoảng (a;b) và n số thực dương α k , k = 1, n có tổng bằng 1 ta có
+ Nếu f ''( x) > 0, ∀x ∈ ( a; b ) thì ta có
α1 f ( x1 ) + α 2 f ( x2 ) + ... + α n f ( xn ) ≥ f (α1 x1 + α 2 x2 + ... + α n xn ) .
Với mọi xk ∈ ( a; b ) , k =
1, n . Đẳng thức xảy ra khi và chỉ khi x1= x2= ...= xn .
+ Nếu f ''( x) < 0, ∀x ∈ ( a; b ) thì ta có
α1 f ( x1 ) + α 2 f ( x2 ) + ... + α n f ( xn ) ≤ f (α1 x1 + α 2 x2 + ... + α n xn ) .
Với mọi xk ∈ ( a; b ) , k =
1, n . Đẳng thức xảy ra khi và chỉ khi x1= x2= ...= xn .

n
.v
Chứng minh.
a) Đặt y= α1 x1 + α 2 x2 + ... + α n xn . Vì f ''( x) > 0 nên áp dụng bất đẳng thức tiếp

om
tuyến ta có
f ( xk ) ≥ f '( y ) ( xk − y ) + f ( y ), k =
1, n
.
⇒ α k f ( xk ) ≥ f '( y ) (α k xk − α k y ) + α k f ( y ), k =
.c1, n
Cộng lại theo vế n bất đẳng thức trên ta được
ok
n n
∑α k f ( xk ) ≥ ∑ ( f '( y) (α k xk − α k y ) + α k f ( y)=) f ( y=) f (α1x1 + α 2 x2 + ... + α n xn ) .
k 1=
bo

k 1
Bất đẳng thức được chứng minh.
Nhận xét. Ta có thể viết lại bất đẳng thức dưới dạng
et

 α1 x1 + α 2 x2 + ... + α n xn 
α1 f ( x1 ) + α 2 f ( x2 ) + ... + α n f ( xn ) ≥ (α1 + α 2 + ... + α n ) f  
 α1 + α 2 + ... + α n
vi


Trong đó α1 ,α 2 ,...,α n là các số thực dương bất kỳ.
ng

b) Chứng minh tương tự.


Từ đây ta có ngay kết quả của bất đẳng thức AM – GM suy rộng.
a

(Bất đẳng thức AM – GM suy rộng) Cho α1 ,α 2 ,...,α n là các số thực dương có
kh

tổng bằng 1, khi đó với mọi số thực không âm bất kỳ x1 , x2 ,..., xn ta có


α1 x1 + α 2 x2 + ... + α n xn ≥ x1α1 xα2 2 ...xαn n .
Chứng minh.
Bất đẳng thức tương đương với:
ln (α1 x1 + α 2 x2 + ... + α n xn ) ≥ α1 ln x1 + α 2 ln x2 + ... + α n ln xn .
1
f ( x) ln x, x > 0 ta có f ''( x) =−
Xét hàm số = < 0, ∀x > 0 .
x2
Khi đó áp dụng bất đẳng thức Jensen ta có ngay điều phải chứng minh.

487
Khám phá tư duy Kỹ thuật giải bất ĐT Bài toán Max – Min – Đặng Thành Nam

Ví dụ 1. Cho x,y,z,t là các số thực dương thoả mãn điều kiện


x ≤ 2, x + y ≤ 6, x + y + z ≤ 12, x + y + z + t ≤ 24 .
1 1 1 1
Chứng minh rằng + + + ≥ 1.
x y z t
Lời giải
1 2
Xét hàm số f (t ) = với t dương ta có f ''(t )= 3 > 0, ∀t > 0 .
t t
Do đó ta có
1 1 1 1 1  x 1  y 1  z 1  t 
+ +=+ f  + f  + f  + f 

n
x y z t 2  2  4  4  6  6  12  12 

.v
1 x 1 y 1 z 1 t  144
≥ f  . + . + . + . =
 2 2 4 4 6 6 12 12  36 x + 9 y + 4 z + t

om
144
=
27 x + 5 ( x + y ) + 3 ( x + y + z ) + ( x + y + z + t )


144
27.2 + 5.6 + 3.12 + 24
=
1
.c
ok
Bất đẳng thức được chứng minh. Đẳng thức xảy ra khi và chỉ khi
=
x 2,=y 4,=z 6,=
t 12 .
bo

Bài tập tương tự


Cho x,y,z là các số thực dương thoả mãn điều kiện x ≤ 4, y ≤ 9, x + y + z ≤ 49 .
et

1 1 1
Chứng minh rằng + + ≥1.
x y z
vi

Ví dụ 2. Cho a,b,c là các số thực dương thoả mãn điều kiện abc = 1 .
ng

a b c
Chứng minh rằng a b .b c .c a ≥ 1 .
Lời giải
a

a b c
Bất đẳng thức đã cho tương đương với ln a + ln b + ln c ≥ 0 .
kh

b c a
1
Xét hàm số= f ( x) x ln x, x > 0 ta có f '( x) = ln x + 1; f ''( x) = > 0, ∀x > 0 .
x
a b c 1 1 1
Vì vậy ln a + ln b + ln c = f (a ) + f (b) + f (c)
b c a b c a
 a b c
 + + 
1 1 1
≥ + +  f  b c a 
b c a  1 + 1 + 1 
a b c

488
Cty TNHH MTV DVVH Khang Việt

a b c 1 1 1
Chú ý f(x) đồng biến và + + ≥ + + .
b c a a b c
Thật vậy theo bất đẳng thức AM – GM ta có
a b a b2 3
+ 2. ≥ 33 . 2 =
b c b c c
b c b c2 3
+ 2. ≥ 33 . 2 =
c a c a a
c a c a2 3
+ 2. ≥ 33 . 2 =

n
a b a b b
Cộng lại theo vế ba bất đẳng thức trên ta có đpcm.

.v
Từ đó suy ra

om
a b c 1 1 1
ln a + ln b + ln c ≥  + +  f (1) =
0.
b c a b c a
Bất đẳng thức được chứng minh. Đẳng thức xảy ra khi và chỉ khi a= b= c= 1 .
.c
Chú ý. Bất đẳng thức trên vẫn đúng khi abc ≥ 1 .
ok
B. BÀI TOÁN CHỌN LỌC
Bài 1. Chứng minh rằng với mọi số thực dương a,b,c ta có
bo

( b + c − a )2 + ( c + a − b )2 + ( a + b − c )2 ≥ 3 .
a2 + (b + c ) b2 + ( c + a ) c2 + ( a + b )
2 2 2 5
et

Lời giải
vi

Bất đẳng thức có dạng thuần nhất chuẩn hoá a + b + c =


1.
Bất đẳng thức trở thành
ng

(1 − 2a )2 + (1 − 2b )2 + (1 − 2c )2 ≥ 3
a 2 + (1 − a ) b 2 + (1 − b ) c 2 + (1 − c )
2 2 2
a

5
.
kh

1 1 1 27
⇔ 2 + + ≤
2a − 2a + 1 2b 2 − 2b + 1 2c 2 − 2c + 1 5
1 1 54 x + 27
Tiếp tuyến của hàm số y = 2 tại điểm x = là y = .
2x − 2x + 1 3 25
1 54 x + 27
Vậy ta chứng minh: ≤ .
2x − 2x + 1
2 25
2 ( 3 x − 1) ( 6 x + 1)
2
Thật vậy bất đẳng thức tương đương với: ≥ 0 , luôn đúng.
(
25 2 x 2 − 2 x + 1 )
489
Khám phá tư duy Kỹ thuật giải bất ĐT Bài toán Max – Min – Đặng Thành Nam

Thay x bởi a,b,c rồi cộng lại theo vế ta có đpcm.


Bài tập tương tự
1) Cho a,b,c là các số thực dương. Chứng minh
( 2a + b + c )2 + ( 2b + c + a )2 + ( 2c + a + b )2 ≤ 8 .
2a 2 + ( b + c ) 2b 2 + ( c + a ) 2c 2 + ( a + b )
2 2 2

2) Cho a,b,c là các số thực dương có tổng bằng 3. Chứng minh


a2 + 9 b2 + 9 c2 + 9
+ + ≤ 5.
2a 2 + ( b + c ) 2b 2 + ( c + a ) 2c 2 + ( a + b )
2 2 2

3) Cho a,b,c là các số thực không âm thoả mãn điều kiện a + b + c > 0 .

n
a2 b2 c2

.v
1
Chứng minh rằng + + ≤ .
5a + ( b + c ) 5b + ( c + a ) 5c + ( a + b )
2 2 2 2 2 2 3

om
4) Cho a,b,c là các số thực không âm thoả mãn điều kiện a + b + c > 0 .
Chứng minh rằng
1 a2 b2 c2 2 .c
≤ + + ≤ .
2 2a + ( b + c )
2 2
2b + ( c + a )
2 2
2c + ( a + b )
2 2 3
ok
5) Cho a,b,c là các số thực dương. Chứng minh
a (b + c ) b (c + a) c (a + b) 6
bo

+ + ≤ .
a + (b + c ) b + (c + a) c + (a + b)
2 2 2 2 2 2 5
Bài 2. Cho a,b,c là các số thực dương thỏa mãn điều kiện
et

a 2 − ab + b 2 + b 2 − bc + c 2 + c 2 − ca + a 2 =
12 .
vi

a b c
Tìm giá trị lớn nhất của biểu thức P = + + .
ng

a2 + 9 b2 + 9 c2 + 9
Lời giải
a

Chú ý: a 2 − ab + b 2 + b 2 − bc + c 2 + c 2 − ca + a 2 ≥ a + b + c .
kh

Chứng minh.
3( a − b ) + ( a + b ) (a + b) a + b .
2 2 2
=
Ta có: a − ab + b
2 2
= ≥
4 4 2
b+c c+a
Tương tự: b 2 − bc + c 2 ≥
, c 2 − ca + a 2 ≥ .
2 2
Cộng lại theo vế 3 bất đẳng thức trên ta có ngay điều phải chứng minh.
Từ đó suy ra: a + b + c ≤ 12 .

490
Cty TNHH MTV DVVH Khang Việt

x
Biểu thức P có dạng P = f (a ) + f (b) + f (c) trong đó f ( x) = .
x2 + 9
x
Xét hàm số f ( x) = trên ( 0;+∞ ) ta có:
x2 + 9
9 27 x
f '(x) = ; f ''( x) =− < 0, ∀x > 0 .
(x ) (x )
3 5
2
+9 2
+9

Do đó áp dụng bất đẳng thức ii ta được:


f (a ) ≤ f'(4) ( a − 4 ) + f (4)

n
f (b) ≤ f '(4) ( b − 4 ) + f (4)

.v
f (c) ≤ f '(4) ( c − 4 ) + f (4)

om
Cộng lại theo vế 3 bất đẳng thức trên ta được:
12
P f (a) + f (b) + f (c) ≤ f '(4) ( a + b + c − 12 ) + 3 f (4) ≤ 3f(4)
= = .
.c 5
Đẳng thức xảy ra khi và chỉ khi a= b= c= 4 .
ok
12
Vậy giá trị lớn nhất của P bằng đạt tại a= b= c= 4 .
5
Bài tập tương tự
bo

Cho a,b,c là các số thực dương thoả mãn điều kiện a 2 + b 2 + c 2 =


3.
1 1 1
et

Chứng minh rằng + + ≥1.


1 + 8a 1 + 8b 1 + 8c
vi

Bài 3. Cho a,b,c là các số thực không âm thỏa mãn điều kiện a + b + c =3.
Tìm giá trị lớn nhất của biểu thức
ng

( ) (b + ) (c + )
b c a
P =a + a 2 + 1 b2 + 1 c2 + 1 .
a

Lời giải
kh

Lấy logarit hóa tự nhiên P ta được:


ln= ( ) (
P b ln a + a 2 + 1 + c ln b + b 2 + 1 + a ln c + c 2 + 1 . ) ( )
Xét hàm số f ( x= (
) ln x + x 2 + 1 , ∀ x ≥ 0 ta có )
x
1+
f '( x) = x +1 = 1
2
> 0, f ''( x) =−
x
≤ 0, ∀x ≥ 0 .
x + x2 + 1 x2 + 1
( x + 1)
2 3

491
Khám phá tư duy Kỹ thuật giải bất ĐT Bài toán Max – Min – Đặng Thành Nam

Áp dụng định lý 1.ii ta được:


f (a ) ≤ f '(1) ( a − 1) + f (1);
f (b) ≤ f '(1) ( b − 1) + f (1);
f (c) ≤ f '(1) ( c − 1) + f (1).
Lấy tổng ba bất đẳng thức trên ta được
ln P = b. f (a ) + c.f(b) + a .f(c)
≤ b ( f '(1) ( a − 1) + f (1) ) + c ( f '(1) ( b − 1) + f (1) ) + a ( f '(1) ( c − 1) + f (1) )
= ( ab + bc + ca − a − b − c ) f '(1) + ( a + b + c ) f (1)
1 
( )

n
≤  ( a + b + c ) − a − b − c  f '(1) + 3 f (1)
2
= 3f(1)
= 3ln 1 + 2
3 

.v
( )
3
Do đó P ≤ 1 + 2 .

om
( )
3
Vậy giá trị lớn nhất của P bằng 1 + 2 đạt tại a= b= c= 1 .
Bài 4. Cho tam giác ABC nhọn tìm giá trị lớn nhất của biểu thức
.c
P = sin A.sin 2 B.sin 3 C .
ok
Lời giải
Ta có ln P =ln(sin A) + 2ln(sin B ) + 3ln(sin C ) .
bo

 π
Xét hàm =
số f ( x) ln(sin x), x ∈  0;  ta có
 2
et

1  π
f '( x) =cot x; f ''( x) =− < 0, ∀x ∈  0;  .
vi

2
sin x  2
Vì vậy
ng

f ( A) ≤ f '( M ) ( A − M ) + f ( M ) =( A − M ) cot M + ln(sin M )


f ( B ) ≤ f '( N ) ( B − N ) + f ( N ) = ( B − N ) cot N + ln(sin N ) .
a

f (C ) ≤ f '( P ) ( C − P ) + f ( P ) = ( C − P ) cot P + ln(sin P )


kh

Với M,N,P là ba góc nhọn trong một tam giác.


Khi đó
tan M . f ( A) + tan N . f ( B) + tan P. f (C ) ≤ ( A − M + B − N + C − P )
+ tan M .ln(sin M ) + tan N .ln(sin N ) + tan P.ln(sin P)
= tan M .ln(sin M ) + tan N .ln(sin N ) + tan P.ln(sin P)
tan M tan N tan P
Do vậy ta chỉ cần chọn các góc M,N,P sao cho = = = k.
1 2 3
Mặt khác

492
Cty TNHH MTV DVVH Khang Việt

tan M + tan N + tan P =


tan M .tan N .tan P
⇒ k + 2k + 3k= 6k 3 ⇒ k= 1 .
tan M 1 2 3
⇒ sin M
= = =
;sin N ;sin =
P
1 + tan M2 2 5 10
Thay vào bất đẳng thức trên ta có ngay
1 2 3 27 27
= f ( A) + 2 f ( B) + 3 f (C ) ≤ ln
ln P + 2ln + 3ln = ln ⇒P≤ .
2 5 5 25 5 25 5
1 2 3
Dấu bằng đạt tại sin
= A sin=
M =
;sin =
B sin N =
;sin =
C sin P .
2 5 10

n
Tổng quát

.v
Cho tam giác ABC nhọn tìm giá trị lớn nhất của biểu thức
P = sin m A.sin n B.sin n C .

om
Với m,n,p là các số thực dương.
Bài tập tương tự
Cho tam giác ABC nhọn, tìm giá trị nhỏ nhất của biểu thức
P =tan A + 2 tan B + 3tan C .
.c
ok
Bài 5. Cho a,b,c là các số thực không âm thỏa mãn điều kiện a + b + c =3.
Tìm giá trị lớn nhất và nhỏ nhất của biểu thức
bo

P=
a 2 + a + 4 + b2 + b + 4 + c2 + c + 4 .
Lời giải
et

Tìm giá trị nhỏ nhất của P


Xét hàm số f ( x= x 2 + x + 4 với x ≥ 0 ta có:
vi

)
2x + 1 15
ng

=f '( x) = ; f ''( x) > 0, ∀x ≥ 0 .


2 x2 + x + 4
(x )
3
4 2
+x+4
a

Theo bất đẳng thức i.định lý 1 ta có:


f (a ) ≥ f '(1) ( a − 1) + f (1)
kh

f (b) ≥ f'(1) ( b − 1) + f (1)


f(c) ≥ f'(1) ( c − 1) + f (1)
Cộng theo vế ba bất đẳng thức trên ta được:
P f (a ) + f (b) + f (c) ≥ f '(1) ( a + b + c − 3) + 3 f (1)
= = 3 6.
Vậy giá trị nhỏ nhất của P bằng 3 6 đạt tại a= b= c= 1 .

493
Khám phá tư duy Kỹ thuật giải bất ĐT Bài toán Max – Min – Đặng Thành Nam

2
1   15 
2

Cách 2: Ta phân tích được thừa số a + a + 4 =  a +  + 
2
 thành tổng của
 2   2 
hai bình phương nên nghĩ ngay đến việc áp dụng bất đẳng thức Mincopski khi
đó a= b= c= 1 .
Sử dụng bất đẳng thức Mincopski ta có:
2 2 2
1   15  1   15  1   15 
2 2 2
  
P =  a +  +   +  b +  +  +  c +  + 
 2   2   2   2   2   2 
2
3  15 
2

n
≥  a + b + c +  +  3.  =3 6
 2  2 

.v
Đẳng thức xảy ra khi và chỉ khi a= b= c= 1 .

om
Cách 3: Ta có a 2 + a + 4= ( a − 1)2 + 3 ( a + 1)2
Tìm giá trị lớn nhất của P
.c
Như tôi đã nhận xét những bất đẳng thức với điều kiện các số thực không âm
thông thường nếu không đánh giá qua các biến đối xứng(dấu bằng không đạt tại
ok
tâm) thì dấu bằng của bất đẳng thức thường xảy ra khi có một biến đạt tại biên.
Với bài toán này dấu bằng xảy ra khi có hai biến bằng 0 khi đó biến còn lại
bo

bằng 3.
Vậy để tìm giá trị lớn nhất của P lúc này ta cần xây dựng hàm f(x) có
f ''( x) ≤ 0 để áp dụng bất đẳng thức ii.định lý 2.
et

15
Với hàm số lúc đầu f ( x=
) x 2 + x + 4=
ta có f ''( x) > 0, ∀x ≥ 0
(x )
vi

3
4 2
+x+4
ng

vậy để tạo ra một hàm có đạo hàm cấp 2 âm ta xử lý như nào. Điều này xuất
15
phát từ phát hiện dấu bằng xảy ra như trên và f ''( x) ≤ < 1, ∀x ≥ 0 .
a

4.8
Giả sử dấu bằng xảy ra khi a= 3, b= c= 0 khi đó xét hàm số như sau:
kh

15
g ( x=
) x 2 + x + 4 − ( x − 3) x. =
Ta có: g ''( x) − 2 < 0, ∀x ≥ 0 .
(x )
3
4 2
+x+4

Áp dụng bất đẳng thức ii.định lý 2 ta được:


g (3) − g (0) 4
g (a) ≤ ( a − 3) + g (3)= ( a − 3) + 4
3−0 3
g (3) − g (0) 4
g (b) ≤ ( b − 0 ) + g (0) = b + 2
3−0 3

494
Cty TNHH MTV DVVH Khang Việt

g (3) − g (0) 4
g (c ) ≤ ( c − 0 ) + g (0) = c + 2
3−0 3
Cộng theo vế 3 bất đẳng thức trên ta được:
P= g (a ) + g (b) + g (c) + a ( a − 3) + b ( b − 3) + c ( c − 3)
4
( a + b + c − 3) + 8 =
≤ g (a ) + g (b) + g (c) ≤ 8
3
Đẳng thức xảy ra khi và chỉ khi a= 3, b= c= 0 .
Vậy giá trị lớn nhất của P bằng 8 đạt tại a= 3, b= c= 0 hoặc các hoán vị.
Cách 2: Ngoài ra có thể áp dụng bất đẳng thức

n
x2 + x + 4 + y 2 + y + 4 ≤ 2 + ( x + y )2 + x + y + 4, ∀x, y ≥ 0; x + y ≥ 1 .

.v
Chứng minh.
Bất đẳng thức đã cho tương đương với

om
( )( )
x2 + y 2 + x + y + 8 + 2 x2 + x + 4 y 2 + y + 4 ≤ 4 + 4 ( x + y ) + x + y + 4 + ( x + y ) + x + y + 4
2 2

(x )( )
⇔ 2
+ x + 4 y 2 + y + 4 ≤ xy + 2
.c
( x + y )2 + x + y + 4
ok
( )( )
⇔ x 2 + x + 4 y 2 + y + 4 ≤ x 2 y 2 + 4 xy ( x + y )2 + x + y + 4 + 4 ( x + y )2 + x + y + 4
bo

⇔ xy  4 ( x + y ) + x + y + 4 + x + y − 7  ≥ 0 (luôn đúng do x + y ≥ 1 ).
2
 
Áp dụng vào bài toán
et

Trong ba số a,b,c luôn tồn tại hai số có tổng không nhỏ hơn 1 giả sử hai số đó là
a và b khi đó áp dụng bất đẳng thức phụ trên ta được
vi

P≤2+ ( a + b )2 + a + b + 4 + c2 + c + 4 .
ng

Mặt khác a + b + c = 3 > 1 nên suy ra


a

( a + b )2 + a + b + 4 + c2 + c + 4 ≤ 2 + ( a + b + c )2 + a + b + c + 4 =6.
kh

Do đó P ≤ 8 . Đẳng thức xảy ra khi và chỉ khi a= 3, b= c= 0 hoặc các hoán vị.
Bài 6. Cho a,b,c là các số thực không âm thỏa mãn điều kiện a + b + c =
1.
( ) (
Chứng minh rằng 10 a3 + b3 + c3 − 9 a5 + b5 + c5 ≥ 1 . )
Lời giải
Bài toán này bất đẳng thức xảy ra có hai trường hợp của dấu bằng là
1
a= b= c= hoặc a= 1, b= c= 0 và các hoán vị.
3
( x) 10 x3 − 9 x5 với x ∈ [ 0;1] ta có:
Xét hàm số f=

495
Khám phá tư duy Kỹ thuật giải bất ĐT Bài toán Max – Min – Đặng Thành Nam

f '( x) =30 x 2 − 45 x 4 ; f ''( x) =60 x − 180 x3 = ( )


60 x 1 − 3 x 2 .

 1   1 
Suy ra f ''( x) ≥ 0, ∀x ∈ 0;  , f ''( x) ≤ 0, ∀x ∈  ;1 .
 3  3 
Vậy ta cần phân chia trường hợp của các biến, không mất tính tổng quát giả sử
a≥b≥c.
1
TH1: Nếu a ≤ khi đó áp dụng bất đẳng thức i.định lý 1 ta được:
3
 1  1 1
f (a) ≥ f '   a −  + f   ;
 3  3 3

n
 1  1 1

.v
f (b) ≥ f '   b −  + f   ;
 3  3 3

om
 1  1 1
f (c) ≥ f '   c −  + f   .
 3  3 3
Cộng theo vế 3 bất đẳng thức trên ta được: .c
1 1
= f (a ) + f (b) + f (c) ≥ f '   ( a + b + c − 1) + 3 f  =
VT  1.
ok
 
3 3
1
Đẳng thức xảy ra khi và chỉ khi a= b= c= .
bo

3
1 1 2
TH2: Nếu a ≥ khi đó nếu b ≥ thì a + b + c ≥ > 1 vô lý.
3 3 3
et

1
Vậy a ≥ ≥ b ≥ c . Khi đó xét hàm g ( x) = 10 x3 − 9 x5 + 20 x 2 ( x − 1) ta có
vi

3
( )
x) 180 1 − x 2 ≥ 0, ∀x ∈ [ 0;1] .
g ''(=
ng

Áp dụng bất đẳng thức i.định lý 1 ta được:


g (a ) ≥ g '(1) ( a − 1) + g (1) =
a

a
kh

g (1) − g (0)
g (b) ≥ ( b − 0 ) + g (0) =b
1− 0
g (1) − g (0)
g(c) ≥ ( c − 0 ) + g (0) =c
1− 0
Cộng theo vế 3 bất đẳng thức trên ta được:
VT= g (a ) + g (b) + g (c) − a 2 ( a − 1) − b 2 ( b − 1) − c 2 ( c − 1)
≥ g (a ) + g (b) + g (c) ≥ a + b + c =1
Đẳng thức xảy ra khi và chỉ khi a= 1, b= c= 0 .
Bài toán được chứng minh.

496
Cty TNHH MTV DVVH Khang Việt

1
Đẳng thức xảy ra khi và chỉ khi a= b= c= hoặc a= 1, b= c= 0 hoặc các hoán vị.
3
1 1 1 1
Bài 7. Cho a, b, c > 2 thoả mãn điều kiện 2 + 2 + 2 = .
a −4 b −4 c −4 7
1 1 1 3
Chứng minh rằng + + ≤ .
a+2 b+2 c+2 7
Lời giải
1 9 1
Với mọi t > 2 ta có ≤ + .
t + 2 10 t 2 − 4 10( )

n
( t − 5 )2

.v
Thật vậy bất đẳng thức tương đương với: ≥ 0.
(
10 t 2 − 4 )

om
Áp dụng bất đẳng thức trên ta có
1 1 1 9 1 1 1  3 3
+ + ≤  2 + 2 + 2  + =.
a + 2 b + 2 c + 2 10  a − 4 b − 4 c − 4  10 7 .c
Bất đẳng thức được chứng minh. Đẳng thức xảy ra khi và chỉ khi a= b= c= 5 .
ok
Chú ý. Bất đẳng thức trên có thể chứng minh bằng BĐT Chebyshev.
bo

C. BÀI TẬP RÈN LUYỆN


Bài 1. Cho x,y,z là các số thực dương thoả mãn điều kiện
x ≤ 4, y ≤ 9, x + y + z ≤ 49 .
et

1 1 1
Chứng minh rằng + + ≥1.
vi

x y z
Bài 2. Cho x,y,z là các số thực dương thoả mãn điều kiện x ≤ 1, y ≤ 2, x + y + z ≤ 6 .
ng

Chứng minh rằng ( x + 1)( y + 1)( z + 1) ≥ 4 xyz .


a

Bài 3. Cho a,b,c là các số thực dương thỏa mãn a 2 + b 2 + c 2 =


1.
kh

a b 3 3 c
Chứng minh rằng + + . ≥
b +c
2 2
c +a 2
a +b 2 2 2 2

Bài 4. Cho a,b,c là các số thực không âm thỏa mãn điều kiện a + b + c =
1.
Tìm giá trị lớn nhất và nhỏ nhất của biểu thức
P= a 2 + 11a + 4 + b 2 + 11b + 4 + c 2 + 11c + 4 .
Bài 5. Cho a,b,c là các số thực dương thoả mãn điều kiện a 2 + b 2 + c 2 =
1.
1 1 1 3 3+9
Chứng minh rằng + + ≥ .
1− a 1− b 1− c 2

497
Khám phá tư duy Kỹ thuật giải bất ĐT Bài toán Max – Min – Đặng Thành Nam

Bài 6. Cho a,b,c là các số thực dương có tích bằng 1 chứng minh
a b c 3 2
+ + ≥ .
1+ a 1+ b 1+ c 2
3
Bài 7. Cho a,b,c là các số thực dương thoả mãn điều kiện a + b + c ≥ .
2

( )( )(
Chứng minh rằng 1 + a 2 1 + b 2 1 + c 2 ≥ ) 125
64
.

9
Bài 8. Cho a,b,c là các số thực dương thoả mãn điều kiện a + b + c ≤ .
4

n
Tìm giá trị lớn nhất của biểu thức

( )( )( )

.v
P =a + a 2 + 1 b + b 2 + 1 c + c 2 + 1 .

om
Bài 9. Cho x,y,z là các số thực dương thoả mãn điều kiện x + y + z =3.
4+ x 4+ y 4+ z
Chứng minh rằng + + ≥5.
4− x 4− y 4− z .c
Bài 10. Cho a,b,c là độ dài ba cạnh một tam giác chứng minh
ok
1 1 1 9  1 1 1 
+ + + ≥ 4 + + .
a b c a+b+c a+b b+c c+a
bo

Bài 11. Cho a,b,c là các số thực dương thoả mãn điều kiện a + b + c =3.
1 1 1
Chứng minh rằng 2 + 2 + 2 ≥ a 2 + b 2 + c 2 .
et

a b c
Bài 12. Cho a,b,c là các số thực dương thoả mãn điều kiện a 4 + b 4 + c 4 =
3.
vi

1 1 1
Chứng minh rằng + + ≤ 1.
ng

4 − ab 4 − bc 4 − ca
Bài 13. Cho a,b,c là các số thực dương. Chứng minh
( 3a + b + c )3 + ( 3b + c + a )3 + ( 3c + a + b )3 ≤ 375 .
a

3a3 + ( b + c ) 3b3 + ( c + a ) 3c3 + ( a + b )


kh

3 3 3 11
Bài 14. Cho a,b,c là các số thực dương. Chứng minh
3+ 3 2
9
( )
1 1 1
a + b2 + c2  + +  ≥ a + b + c + a 2 + b2 + c2 .
a b c
Bài 15. Cho a,b,c là các số thực dương. Chứng minh
a b c 3( a + b + c )
+ + ≥ .
b+c c+a a+b 2

498
Cty TNHH MTV DVVH Khang Việt

D. HƯỚNG DẪN GIẢI – ĐÁP SỐ


3
f (t ) 1 / t , t > 0 ta có f=
Bài 1. Xét hàm số= ''(t ) 2
> 0, ∀t > 0 .
4t t
Do đó ta có
1 1 1 1  x 1  y 1  z 
+ + = f  + f  + f 
x y z 2 4 3 9 6  36 
1 x 1 y 1 z  216
≥ f  . + . + . =
 2 4 3 9 6 36  27 x + 8 y + z
216 216
= ≥ = 1

n
26 x + 7 y + ( x + y + z ) 26.4 + 7.9 + 49

.v
Bất đẳng thức được chứng minh.

om
Đẳng thức xảy ra khi và chỉ khi=
x 4,=
y 9,=
z 36 .
Bài 2. Bất đẳng thức đã cho tương đương với
 1  1  1 .c
ln 1 +  + ln 1 +  + ln 1 +  ≥ ln 4 .
 x  y  z
ok
 1 1 2t + 1
Xét hàm số f (t ) = ln 1 +  , t > 0 ta có f '(t ) =− 2 ; f ''(t ) = > 0, ∀t > 0 .
 t t +t
( )
2
t +t
2
bo

Do đó sử dụng bất đẳng thức tiếp tuyến ta có


 1 1
ln 1 +  ≥ f '(1) ( x − 1) + f (1) =− ( x − 1) + ln 2
et

 x 2
vi

 1 1 3
ln 1 +  ≥ f '(2) ( y − 2 ) + f (2) =− ( y − 2 ) + ln
 y 6 2
ng

 1 1 4
ln 1 +  ≥ f '(3) ( z − 3) + f (3) =− ( z − 3) + ln
 z 12 3
a

Cộng theo vế ba bất đẳng thức trên ta được


kh

 1  1  1 1 1 1
ln 1 +  + ln 1 +  + ln 1 +  ≥ ln 4 − ( x − 1) − ( y − 2 ) − ( z − 3)
 x  y  z 2 6 12
 6 x + 2 y + z  13
= ln 4 −  +
 12  4
5 x + y + ( x + y + z ) 13
= ln 4 − + ≥ ln 4
12 4
Bất đẳng thức được chứng minh. Đẳng thức xảy ra khi và chỉ khi= x 1,=
y 2,=
z 3.

499
Khám phá tư duy Kỹ thuật giải bất ĐT Bài toán Max – Min – Đặng Thành Nam

a b c 3 2
Bài 3. Viết lại bất đẳng thức dưới dạng: + + ≥ .
1− a 2
1− b 2
1− c 2 2
2 2
a a a 3 2a
Ta có: = = ≥ .
b +c22
1− a 2
a 1− a 2 2 ( )
3 2b 2 b c 3 2c 2
Tương tự ta có: ; ≥ ≥ .
c2 + a2 2 a 2 + b2 2
Cộng theo vế 3 bất đẳng thức trên ta được:
a
+
b
+
c

3 2 2
( 3 2
a + b2 + c2 = . )

n
b +c
2 2
c +a
2 2
a +b
2 2 2 2

.v
1
Đẳng thức xảy ra khi và chỉ khi a= b= c= .
3

om
Bài 4. Tìm giá trị nhỏ nhất của P
105
Xét hàm số f ( x) = x 2 + 11x + 4 với x ≥ 0.=
Ta có f ''( x) > 0, ∀x ≥ 0 .
( )
.c 3
4 x + 11x + 42

Áp dụng bất đẳng thức i.định lý 1 ta được:


ok
 1  1 1
f (a ) ≥ f '   a −  + f  
 3  3 3
bo

 1  1 1
f (b) ≥ f '   b −  + f  
 3  3 3
et

 1  1 1
f (b) ≥ f '   c −  + f  
vi

 3  3 3
Cộng theo vế 3 bất đẳng thức trên ta được:
ng

1 1
P f (a ) + f (b) + f (c) ≥ f '   ( a + b + c − 1) + 3 f  =
=  2 10 .
3 3
a

1
Dấu bằng xảy ra khi và chỉ khi a= b= c=
kh

.
3
Tìm giá trị lớn nhất của P
Xét hàm số g ( x=
) x 2 + 11x + 4 − 2 x ( x − 1) với x ≥ 0 ta có:
105
=g ''( x) − 4 < 0, ∀x ≥ 0 .
(x )
3
4 2
+ 11x + 4

Áp dụng bất đẳng thức ii.định lý 2 ta được:

500
Cty TNHH MTV DVVH Khang Việt

g (1) − g (0)
g (a) ≤ ( a − 1) + g (1)= 2 ( a − 1) + 4
1− 0
g (1) − g (0)
g (b) ≤ ( b − 0 ) + g (0) = 2b + 2
1− 0
g (1) − g (0)
g (c ) ≤ ( c − 0 ) + g (0) = 2c + 2
1− 0
Cộng theo vế 3 bất đẳng thức trên ta được:
P= g (a ) + g (b) + g (c) + 2a ( a − 1) + 2b ( b − 1) + 2c ( c − 1)
≤ g (a ) + g (b) + g (c) ≤ 2 ( a + b + c − 1) + 8 =8

n
Đẳng thức xảy ra khi và chỉ khi a= 1, b= c= 0 .
Vậy giá trị lớn nhất của P bằng 8 đạt tại a= 1, b= c= 0 hoặc các hoán vị.

.v
1 9+6 3 2 3
x + , ∀x ∈ ( 0;1) .

om
Bài 5. HD: Chứng minh ≥
1− x 4 4
x 3 2 2
Bài 6. HD: Chứng minh ≥ x− , ∀x > 0 .
1+ x 4 4 .c
Bài 7. HD: Chứng minh ln 1 + x 2 ≥ ( ) 4 5 2  3
x + ln − , ∀x ∈ 0;  .
ok
5 4 5  2
Bài toán tổng quát xem chương 4.
bo

Bài 8. Lấy Logarit tự nhiên P ta được

( ) ( ) (
ln P= ln a + a 2 + 1 + ln b + b 2 + 1 + ln c + c 2 + 1 . )
et

Xét hàm số f ( x) = ln ( x + x + 1 ) , x > 0 ta có


2
vi

1 x
ng

f '( x) = ; f ''( x) =− < 0, ∀x > 0 .


x2 + 1 ( x + 1)
2
x2 + 1
a

 3  3 3 4 3
Do đó f ( x) ≤ f '   x −  + f   = x + ln 2 − .
kh

 4  4 4 5 5
Từ đó suy ra

( ) (
ln P= ln a + a 2 + 1 + ln b + b 2 + 1 + ln c + c 2 + 1 ) ( )
4 9
≤ ( a + b + c ) + 3ln 2 − ≤ 3ln 2
5 5
⇒ P≤8
3
Với a= b= c= thì P bằng 8. Vậy giá trị lớn nhất của P bằng 8.
4

501
Khám phá tư duy Kỹ thuật giải bất ĐT Bài toán Max – Min – Đặng Thành Nam

4+ x
Bài 9. Phương trình tiếp tuyến của hàm số y = tại điểm x = 1 là
4− x
13 17
=y x+ .
18 18
4 + x 13 17
Ta đi chứng minh: ≥ x+ .
4 − x 18 18
Thật vậy bất đẳng thức tương đương với:

( )  13
 18
17 
( ) (
4 + t ≥ 4 − t 2  t 2 +  , t = x ∈ 0; 3 ⇔ ( t − 1) 13t 2 + 26t + 4 ≥ 0 .
18 
2
)

n
Ta có điều phải chứng minh.
Xây dựng hai bất đẳng thức tương tự cho y,z rồi cộng lại theo vế ta có đpcm.

.v
Bài 10. Bất đẳng thức có dạng thuần nhất chuẩn hoá a + b + c =1.

om
1 4 1 4 1 4
Bài toán đưa về chứng minh − + − + − ≥ −9 .
a 1− a b 1− b c 1− c
.c  1
Chú ý do a,b,c là độ dài ba cạnh một tam giác nên a, b, c ∈  0;  .
 2
ok
1 4 1
Tiếp tuyến của hàm số y= − tại điểm x = là y= 3 − 18 x .
x 1− x 3
bo

1 4  1
Vậy ta chứng minh: − ≥ 3 − 18 x, ∀x ∈  0;  .
x 1− x  2
( 3x − 1)2 (1 − 2 x ) ≥ 0
et

Thật vậy bất đẳng thức tương đương với: luôn đúng.
x (1 − x )
vi

1 4 1 4 1 4
Do đó − ≥ 3 − 18a; − ≥ 3 − 18b; − ≥ 3 − 18c .
ng

a 1− a b 1− b c 1− c
Cộng lại theo vế ba bất đẳng thức trên ta có đpcm.
Đẳng thức xảy ra khi và chỉ khi a= b= c .
a
kh

CHỦ ĐỀ 6: KỸ THUẬT KHẢO SÁT HÀM NHIỀU BIẾN


A. NỘI DUNG PHƯƠNG PHÁP
Bài toán. Tìm giá trị lớn nhất và nhỏ nhất của biểu thức P ( x, y, z ) thỏa mãn điều
kiện K cho trước.
Xuất phát từ giả thiết bài toán tìm được mối liên hệ(điều kiện x so với y,z) giữa
biến x với hai biến còn lại. Coi f ( x) = P ( x, y, z ) là hàm số với biến x và y,z là
tham số. Khảo sát hàm số với biến x tìm ra giá trị nhỏ nhất, lớn nhất của f(x) chẳng

502
Cty TNHH MTV DVVH Khang Việt

hạn đạt tại x = x0 . Khi đó tiếp tục khảo sát hàm số g ( y ) = P ( x0 , y, z ) và tìm ra giá
trị lớn nhất, nhỏ nhất của g(y) từ đó suy ra kết quả bài toán.
Các điểm cần lưu ý
- Bài toán xử lý được bằng phương pháp này thường cho mối ràng buộc các biến
cùng thuộc đoạn [ a; b ] cho trước và dấu bằng của bất đẳng thức đạt tại biên.
- Với bài toán có hai biến số x,y ta thực hiện tương tự.
- Nếu bất đẳng thức có dạng đối xứng 3 biến ta thường giả sử x ≥ y ≥ z để thuận
tiện cho việc đánh giá tính đơn điệu của hàm số.
- Đánh giá cuối cùng là hàm một biến ta có thể khảo sát hàm số hoặc biến đổi
tương đương để có nhanh kết quả.

n
.v
Một bài toán sử dụng phương pháp này hay gặp trong đề thi
Bài toán. Cho a,b,c và x,y,z là các số thực dương thoả mãn điều kiện

om
ax + by + cz =
xyz .
Tìm giá trị nhỏ nhất của biểu thức P = x + y + z .
 PHƯƠNG PHÁP
Theo giả thiết ta có:
.c
ok
ax + by ax + by
ax + by + cz = xyz ⇒ z = , ( xy > c ) ⇒ P = f ( y ) = x + y + .
xy − c xy − c
bo

ax + by
Xét hàm số f ( y ) = x + y + ta có
xy − c
et

bc + ax 2
0 ( xy − c ) =
2
f '( y ) =
1− ; f '( y ) =⇔ bc + ax 2
( xy − c ) 2
vi

c + bc + ax 2
ng

⇔ y = y0 =
x
Ta có f’(y) đổi dấu từ âm sang dương khi đi qua y0 nên f(y) đạt cực tiểu tại y0 .
a

Vì vậy
kh

c + bc + ax 2
ax + b.
c + bc + ax 2 x
P ≥ f ( y0 ) =
x+ +
x bc + ax 2
c + bc + ax 2 b
=x + + + bc + ax 2
x x
x 2 + b + c + ( x + 1) bc + ax 2
=
x

503
Khám phá tư duy Kỹ thuật giải bất ĐT Bài toán Max – Min – Đặng Thành Nam

x 2 + b + c + ( x + 1) bc + ax 2
Thực hiện xét tính đơn điệu của hàm số g ( x) = ta
x
có kết quả bài toán.
Ví dụ 1. Cho x,y,z là các số thực dương thoả mãn điều kiện 2 x + 4 y + 7 z =
2 xyz .
Tìm giá trị nhỏ nhất của biểu thức P = x + y + z .
Lời giải
2x + 4 y 2x + 4 y
Theo giả thiết ta có z = . Khi đó P = f ( y ) = x + y + .
2 xy − 7 2 xy − 7
2x + 4 y 7
Xét hàm số f ( y ) = x + y + với y >

n
ta có
2 xy − 7 2x

.v
4 x 2 + 28
0 ( 2 xy − 7 ) =
2
f '( y ) =
1− ; f '( y ) =⇔ 4 x 2 + 28

om
( 2 xy − 7 ) 2

7 + 2 x2 + 7 7
⇔ y = y0 = > .c
2x 2x
Ta có f’(y) đổi dầu từ âm sang dương khi đi qua y0 nên f(y) đạt cực tiểu tại y0 .
ok
Vì vậy
7 + 2 x2 + 7
bo

2 x + 4.
7 + 2 x2 + 7 2x 11 2 x 2 + 7
P ≥ f ( y0 ) =x + + =x + + .
2x 2 x2 + 7 2x x
et

11 2 x 2 + 7
Xét hàm số g ( x) =x + + với x > 0 ta có
vi

2x x
28
ng

2 x 2 − 11 −
g '( x) = x 2 + 7 ; g '( x) =0 ⇔ 2 x 2 − 11 = 28
2
2x x2 + 7
a

 11
kh

 x ≥
⇔ 2 ⇔x=
3
 2
( ) (x )
2
 2 x − 11 + 7 − 282 =
2
0

15 15
Từ đó suy ra g min = g (3) = ⇒ P ≥ . Đẳng thức xảy ra khi và chỉ khi
2 2
5
=
x 3,=
y =,z 2 .
2
5
Vậy giá trị nhỏ nhất của P bằng 15/2 đạt tại=
x 3,=
y =,z 2 .
2
504
Cty TNHH MTV DVVH Khang Việt

Chú ý. Sử dụng bất đẳng thức AM – GM ta có:

x+
11 2 x 2 + 7
+ =x +
11 (
+
)
x2 + 7 (9 + 7 )
2x x 2x 2x
11 3 x + 7 9 3 9 3 15
≥ x+ + = x + + ≥ 2 x. + =
2x 2x x 2 x 2 2
Ta có kết quả tương tự trên.
Kỹ thuật cố định biến số
Ví dụ 1. Cho a,b,c là các số thực không âm thoả mãn điều kiện ab + bc + ca > 0 .
 4a  4b  4c 
Chứng minh rằng 1 + 1 + 1 +  ≥ 25 .

n
 b + c  c + a  a +b

.v
Lời giải
Bất đẳng thức được viết dưới dạng

om
( 4a + b + c )( 4b + c + a )( 4c + a + b ) − 25 ≥ 0 ⇔ f = (ab)
α ab + β
− 25 ≥ 0 .
( a + b )( c + a )( c + b ) λ ab + γ
Trong đó α , β , λ , γ là các hằng số khi ta cố định c và a + b .
.c
Rõ ràng f(ab) là hàm luôn đồng biến hoặc luôn nghịch biến do đó với
ok
 ( a + b )2 
ab ∈ 0;  thì f(ab) đạt giá trị nhỏ nhất hoặc tại 0(khi đó có ít nhất một
 4 

bo

số bằng 0) hoặc tại


( a + b )2 (khi đó 2 số bằng nhau).
4
et

Vậy ta chỉ cần chứng minh bất đẳng thức trong hai trường hợp.
+ TH1: Nếu có một số bằng 0 giả sử là a. Ta cần chứng minh
vi

 4b  4c 
1 + 1 +  ≥ 25 .
ng

 c  b 
Bất đẳng thức này là hiển nhiên theo C–S, thật vậy
2
a

 4b  4c   4b 4c 
1 + 1 +  ≥ 1 + .  = 25 .
c b 
kh

 c  b  
Đẳng thức xảy ra khi và chỉ khi b = c .
+ TH2: Khi 2 số bằng nhau giả sử là a và b, khi đó ta cần chứng minh.
 4a  4a  4c 
1 + 1 + 1 +  ≥ 25
 a + c  a + c  2a 
2
 

2
4 a   2c   4   2c 
⇔ 1 +  1 +  ≥ 25 ⇔ 1 +  1 +  − 25 ≥ 0
 a+c  a   1+ c   a 
 a

505
Khám phá tư duy Kỹ thuật giải bất ĐT Bài toán Max – Min – Đặng Thành Nam

c
Đặt x= ≥ 0 ta phải chứng minh
a
2

1 +

4   ( 
 (1 + 2 x ) − 25 ≥ 0 ⇔ x x − 2 x + 5 ≥ 0 ⇔ x ( x − 1) + 4  ≥ 0 .
x +1
2 2
)
Bất đẳng thức cuối luôn đúng.
Bất đẳng thức được chứng minh. Đẳng thức xảy ra khi và chỉ khi 1
=
a b= , c 0 hoặc các hoán vị.
2a 2b 2c
Cách 2: Đặt x= , y= ,=
z ⇒ xy + yz + zx + xyz= 4 .
b+c c+a a+b

n
Theo bất đẳng thức đã được chứng minh ta có
x + y + z ≥ xy + yz + zx .

.v
Vậy bài toán đưa về chứng minh

om
(1 + 2 x )(1 + 2 y )(1 + 2 z ) ≥ 25 ⇔ 2 xyz + x + y + z ≥ 4 ⇔ ( x + y + z + xyz ) + xyz ≥ 4 .
Bất đẳng thức cuối đúng bởi vì x + y + z + xyz ≥ xy + yz + zx + xyz =4.
Ví dụ 2. Cho a,b,c là các số thực dương có tổng bằng 3. Chứng minh .c
a b c 3(a 2 + b 2 + c 2 )
+ + ≤
ok
.
1 + (b + c) 2 1 + (a + c)2 1 + ( a + b) 2 a 2 + b 2 + c 2 + 12abc
Lời giải
bo

Bất đẳng thức đã cho tương đương với:


 a   b   c  3(a 2 + b 2 + c 2 )
a − + − + − ≥ −
et

  b   c  3
 1 + (b + c)   1 + (c + a )   1 + (a + b)  a 2 + b 2 + c 2 + 12abc
2 2 2
vi

a (b + c)2 b (c + a ) 2 c ( a + b) 2 36abc
⇔ + + ≥
1 + (b + c) 2
1 + (c + a ) 2
1 + (a + b) 2
a + b + c 2 + 12abc
2 2
ng

x
tăng do đó với ( b + c ) ≥ 4bc; ( c + a ) ≥ 4ca; ( a + b ) ≥ 4ab
2 2 2
Xét hàm số y =
x +1
a

Ta có:
kh

a (b + c ) 4abc b ( c + a ) 4abc c ( a + b )
2 2 2
4abc
≥ ; ≥ ; ≥ .
1 + (b + c ) 1 + 4bc 1 + ( c + a ) 1 + 4ca 1 + ( a + b ) 1 + 4ab
2 2 2

Do đó ta chỉ cần chứng minh


4abc 4abc 4abc 36abc 36abc
+ + ≥ 2 = .
1 + 4bc 1 + 4ca 1 + 4ab a + b + c + 12abc ∑ a 2 (1 + 4bc)
2 2

cyc

Bất đẳng thức cuối đúng bởi theo C –S ta có

506
Cty TNHH MTV DVVH Khang Việt

1
∑ 1 + 4bc ∑ a 2 (1 + 4bc) ≥ (a + b + c)2 =9 .
cyc cyc

Bất đẳng thức được chứng minh. Đẳng thức xảy ra khi và chỉ khi a= b= c= 1 .
Ví dụ 3. Cho x,y,z là các số thực thuộc đoạn [1;2] .
x2 y + y 2 z + z 2 x
Tìm giá trị lớn nhất và giá trị nhỏ nhất của biểu thức P = .
x4 + y 4 + z 4
Lời giải
Tìm giá trị lớn nhất
Do điều kiện bài toán x, y, z ∈ [1;2] ⇒ x 2 y ≤ x 2 y 2 ; y 2 z ≤ y 2 z 2 ; z 2 x ≤ z 2 x 2 .

n
.v
x2 y 2 + y 2 z 2 + z 2 x2
Do đó P ≤ .
x4 + y 4 + z 4

om
x4 + y 4 + z 4 − x2 y 2 − y 2 z 2 − z 2 x2
= 1−
x4 + y 4 + z 4

(x − y ) + ( y − z ) + (z − x )
2 2 2 2 2 2 2
.c 2 2
=
1− ≤1
2( x + y + z )
ok
4 4 4

Đẳng thức xảy ra khi và chỉ khi x= y= z= 1 .


bo

Vậy giá trị lớn nhất của P bằng 1 đạt tại x= y= z= 1 .


Tìm giá trị nhỏ nhất
et

Với giả thiết bài toán x,y,z thuộc đoạn nên thường đạt cực trị tại biên. Thử x,y,z
chọn trong 2 số (1,2) thay vào P ta thấy giá trị nhỏ nhất
vi

7 7
x = y =1; z =2 ⇒ P = = .
ng

2 + 16 18
7
Vậy ta đi chứng minh P ≥ .
a

18
Trước tiên chuyển P về biểu thức đối xứng của x,y,z.
kh

Theo giả thiết ta có: ( x − 1)( x − 2 ) ≤ 0 ⇔ x 2 ≤ 3 x − 2; y 2 ≤ 3 y − 2; z 2 ≤ 3z − 2 .


 y2 + 2   z 2 + 2  2  x2 + 2 
Suy ra: x 2 y + y 2 z + z 2 x ≥ x 2   + y2  +z 
 3   3   3 
     

=
(
x2 y 2 + y 2 z 2 + z 2 x2 + 2 x2 + y 2 + z 2 )
3
Do vậy ta chỉ cần chứng minh.

507
Khám phá tư duy Kỹ thuật giải bất ĐT Bài toán Max – Min – Đặng Thành Nam

(
x2 y 2 + y 2 z 2 + z 2 x2 + 2 x2 + y 2 + z 2 )≥ 7
3 18
(x + y + z )
4 4 4

(
⇔7 x +y +z 4 4 4
) − 6( x y + y2 z2
2 2
+ z x ) − 12 ( x + y + z ) ≤ 0
2 2 2 2 2

Để cho đơn giản ta đặt a =


x2 , b = z 2 ⇒ a, b, c ∈ [1;4] và bất đẳng thức trở
y2 , c =

( )
thành: 7 a 2 + b 2 + c 2 − 6 ( ab + bc + ca ) − 12 ( a + b + c ) ≤ 0 .
Vế trái ký hiệu là P (a, b, c) là tam thức bậc hai của a với hệ số của a2 dương
nên đạt max tại 1 hoặc 4.
Do đó P (a, b, c) ≤ max { P (1, b, c); P(4, b, c)} .

n
Bằng cách tương tự P (1, b, c); P(4, b, c) là tam thức bậc hai với hệ số b2; c2

.v
dương nên
P (1, b, c) ≤ max { P (1,1, c); P (1, 4, c)}

om
≤ max {max { P(1,1,1); P(1,1, 4)} ;max { P(1, 4,1); P(1, 4, 4)}} =
0
Và .c
P (4, b, c) ≤ max { P (4,1, c); P (4, 4, c)}
≤ max {max { P(4,1,1); P(4,1, 4)} ;max { P(4, 4,1); P(4, 4, 4)}} =
ok
0
Từ đó suy ra điều phải chứng minh.
bo

Với x= y= 1, z= 2 thì P bằng 7/18. Vậy giá trị nhỏ nhất của P bằng 7/18.
Nhận xét. Thực chất ta có đánh giá trên dựa vào tính chất của hàm lồi, đó là nếu
f ''( x) ≥ 0; ∀x ∈ [ a; b ] thì f(x) đạt max tại a hoặc tại b.
et
vi

B. BÀI TOÁN CHỌN LỌC


1 
ng

Bài 1. Cho a,b là hai số thực thuộc đoạn  ;1 . Tìm giá trị nhỏ nhất của biểu thức
4 
1 a b
P= + +
a

.
2 + 3a a + b b + 1
kh

Lời giải
1 a b 1 
Xét hàm số f (a) = + + trên đoạn  ;1 ta có:
2 + 3a a + b b + 1 4 
b ( 2 + 3a ) − 3 ( a + b )
2 2
3 b
f '(a ) =
− + = 2 > 0 vì
( 2 + 3a ) ( a + b )
2 2
( 2 + 3a ) ( a + b )2
b ( 2 + 3a ) − 3 ( a + b ) = 9a 2b + 6ab + 4b − 3a 2 − 3b 2 ≥ 9a 2b + 6a 2b + 4b − 3a 2 − 3b 2
2 2

1 
= 3a 2 ( 5b − 1) + b ( 4 − 3b ) > 0, ∀a, b ∈  ;1
4 

508
Cty TNHH MTV DVVH Khang Việt

1 
Vậy f(a) là hàm đồng biến trên đoạn  ;1 suy ra
4 

1 4
P = f (a) ≥ f   = g (b) = +
1
+
b
=
( 2b − 1) +
34 34

2
.
4 11 1 + 4b b + 1 3 ( b + 1)( 4b + 1) 33 33
1 1
Đẳng thức xảy ra khi và chỉ khi=
a =,b .
4 2
34 1 1
Vậy giá trị nhỏ nhất của P bằng đạt tại=
a = ,b .
33 4 2
34

n
Nhận xét. Để có biến đổi g (b) ≥ thực chất ta đã xét hàm một biến với b tìm ra
33

.v
1
g(b) đạt cực tiểu tại b = .

om
2
Bài 2. Cho x,y là hai số thực thuộc đoạn [1;4] . Tìm giá trị lớn nhất của biểu thức
y −1 1− x
P= x− y+ + .c
.
x y
ok
Lời giải

Ta có: x −=
y+
y −1 1− x
+
( x − y )( =
y − 1)( x − 1)
⇒P
( x − y )( y − 1)( x − 1) .
bo

x y xy xy
Do P là biểu thức đối xứng với x và y nên không mất tính tổng quát ta giả sử
( x −=
y )( x − 1)( y − 1) y − 1  x 2 − ( y + 1) x + y 
et

=
x≥ y⇒P  .
xy y  x 
vi

y − 1  x − ( y + 1) x + y 
2
 liên tục trên đoạn [1;4] ta có:
ng

Xét hàm số f ( x) = 
y  x 

( y − 1) ( x 2 − y )
a

y −1 y 
f=
'( x) 1 −=  ≥ 0, ∀x ≥ y ≥ 1 .
kh

y  x2  x2 y
Do đó f(x) là hàm đồng biến trên đoạn [1;4] .

Suy ra P = f ( x) ≤ f (4) = g ( y ) =
( y − 1)(12 − 3 y ) .
4y
( y − 1)(12 − 3 y ) liên tục trên đoạn
Xét hàm số g ( y ) = [1;4] ta có:
4y

509
Khám phá tư duy Kỹ thuật giải bất ĐT Bài toán Max – Min – Đặng Thành Nam

12 − 3 y 2
g '( y ) = ; g '( y ) =0 ⇔ y =2 ∈ [1;4] . Ta có g’(y) đổi dấu từ dương sang
4 y2
3
âm khi đi qua y = 2 nên g(y) đạt cực đại tại y = 2 . Do đó P ≤ g ( y ) ≤ g (2) = .
4
Đẳng thức xảy ra khi và chỉ khi=x 4,= y 2.
3
Vậy giá trị lớn nhất của P bằng đạt tại=
x 2,= y 4 hoặc= x 4,= y 2.
4
Bài 3. Cho hai số thực x ≥ 1 ≥ y > 0 . Tìm giá trị nhỏ nhất của biểu thức

(x 2
+ y2 ) xy + x + y

n
A= .
xy ( x + y + 1)

.v
Lời giải
Đây là biểu thức đối xứng với tổng x + y và tích xy nên suy nghĩ ngay đến việc

om
đặt S =+ xy từ điều kiện (1 − x )(1 − y ) ≤ 0 ⇔ x + y − 1 ≥ xy ta có ngay
x y, P =
0 < P ≤ S −1 . .c
Ta có: A =
(S 2
− 2P ) S+P
.
ok
P ( S + 1)
Coi vế phải là hàm số với P và tham số S ta được:
bo

2S 3 + S 2 P + 2 P 2
f '( P ) =− < 0, ∀S , P > 0 do đó f(P) là hàm nghịch biến trên
2 ( S + 1) P 2 S + P
et

( 0; S − 1] .
(S )
vi

2
− 2S + 2 2S − 1
Do đó f ( P ) ≥ f ( S − 1) = .
S 2 −1
ng

Xét hàm số g ( S ) =
(S 2
− 2S + 2 ) 2S − 1
trên (1;+∞ ) ta được:
a

S 2 −1
( S − 2 ) ( S 3 + 4 S 2 − 5S + 2 )
kh

S 4 + 2 S 3 − 13S 2 + 12 S − 4
g '( S ) = ;
( ) ( )
2 2
2S − 1 S 2 − 1 2S − 1 S 2 − 1
S >1
g '( S=
) 0 ←
→=S 2.
Ta có g’(S) đổi dấu từ âm sang dương khi đi qua S = 2 nên g(S) đạt cực tiểu tại
2
S = 2 hay g ( S ) ≥ g (2) =.
3
2
Vậy giá trị nhỏ nhất của A bằng đạt tại x= y= 1 .
3

510
Cty TNHH MTV DVVH Khang Việt

Bài 4. Cho x,y,z là các số thực thỏa mãn 1 ≤ x ≤ y ≤ z ≤ 4 .


1 1 1
Tìm giá trị lớn nhất của biểu thức P = ( x + y + z )  + +  .
x y z
Lời giải
1 1 1
Xét hàm số f ( x) = ( x + y + z )  + +  trên [1;4] ta có:
x y z

1 1 1 x+ y+z
f '( x) = + + −
 1 1  x − yz ( y + z )
= ( y + z ) − 2  =
2
(
≤0.
)
x y z x2  yz x  x 2 yz

n
Do đó f(x) là hàm nghịch biến trên [1;4] suy ra f ( x) ≤ f (1) hay

.v
 1 1
P = f ( x) ≤ f (1) = g ( y ) = (1 + y + z ) 1 + +  .

om
 y z
 1 1
Xét hàm số g ( y ) = (1 + y + z ) 1 + +  trên đoạn [1;4] ta có:
 y z .c
1 1 1 + y + z z + 1 1 + z ( y − z )( z + 1)
g '( y ) =1 + + − = − 2 = ≤ 0.
ok
y z y2 yz y y2 z
Do đó g(y) là hàm nghịch biến trên đoạn [1;4] suy ra
bo

 1 2
P ≤ g ( y ) ≤ g(1) =( 2 + z )  2 +  =h( z ) =5 + 2 z + .
 z z
et

2
Xét hàm số h( z ) =5 + 2 z + trên đoạn [1;4] ta có:
vi

z
(
2 z2 −1 )
ng

2
h '( z ) =2 − 2 = 2 ≥ 0 nên h(z) là hàm đồng biến trên đoạn [1;4] .
z z
27
a

Do đó P ≤ h( z ) ≤ h(4) = .
2
kh

Đẳng thức xảy ra khi và chỉ khi x= y= 1, z= 4 .


27
Vậy giá trị lớn nhất của P bằng đạt tại x= y= 1, z= 4 .
2
Bài 5. Cho a,b,c là các số thực dương thoả mãn điều kiện abc + a + c = b.
2 2 3
Tìm giá trị lớn nhất của biểu thức P = 2 − 2 + 2 .
a +1 b +1 c +1
Lời giải
1 a+c
Theo giả thiết ta có b (1 − ac ) = a + c > 0 ⇒ 1 − ac > 0 ⇒ a < ; b = .
c 1 − ac
511
Khám phá tư duy Kỹ thuật giải bất ĐT Bài toán Max – Min – Đặng Thành Nam

( )
2
2 2 3 2 a 2 + 2ac + 2c 2 + 1 3
Khi đó P = − + = + − 2.
a +1  a + c 
2
  +1
c +1 2 2
( a + 1)( c + 1)
2 2 c +1
2

 1 − ac 

( )
2
2 a 2 + 2ac + 2c 2 + 1 3
=
Xét hàm số f (a ) + − 2 ta có
( a + 1)( c + 1) c 2 2 2
+1

−4c ( a + 2ac − 1)
2
f '(a ) = ; f '(a ) = 0 ⇔ a = c2 + 1 − c .
( a + 1) ( c + 1)
2 2 2

n
.v
Ta có f’(a) đổi dấu từ dương sang âm khi đi qua c2 + 1 − c .

( )

om
2c 3
Vì vậy f (a ) ≤ f c 2 + 1=
−c + .
c2 + 1 c +12

2c 3
=
Xét hàm số g (c ) + với c dương ta có .c
c2 + 1 c +12

( )
ok
2 1 − 8c 2 1
g '(c) = ; g '(c) = 0 ⇔ c = , ( c > 0) .
( c2 + 1 )
2
( 3c + c 2 + 1 ) 2 2
bo

1
Ta có g’(c) đổi dấu từ dương sang âm khi đi qua c = .
et

2 2
 1  10
Vì vậy g (c) ≤ g  =
vi

.
2 2 3
ng

1 1 1 1
Đẳng thức xảy ra khi và chỉ khi c= , a= +1 − = , b= 2.
2 2 8 2 2 2
a

Vậy giá trị lớn nhất của P bằng 10/3 đạt tại
kh

1 1 1 1
c= , a= +1 −
, b= 2 . =
2 2 2 8 2 2
Nhận xét. Ta có thể giải bài toán bằng lượng giác xem chương sau.
Bài 6. Cho a,b,c là các số thực không âm thỏa mãn a + b + c =3.
Tìm giá trị nhỏ nhất của biểu thức P =a + b 2 + c3 .
Lời giải
Thay a = 3 − b − c vào biểu thức của P ta được:
2
 1 11 11
P = b 2 − b + c 3 − c + 3 =  b −  + c 3 − c + ≥ f (c ) = c 3 − c + .
 2 4 4
512
Cty TNHH MTV DVVH Khang Việt

11
Xét hàm số f (c) = c3 − c + với c ∈ [ 0;3] ta có:
4
1
f '(c) = 3c 2 − 1; f '(c) = 0 ⇔ c = ∈ [ 0;3] .
3
1
Ta có f’(c) đổi dấu từ âm sang dương khi đi qua c = nên f(c) đạt cực tiểu tại
3
1
c= .
3
 1  11 2 3
Suy ra P ≥ f (c) ≥ f  = − 9 .

n
 3 4

.v
5 1 1 1
Đẳng thức xảy ra khi và chỉ khi a = − ,b = ,c =.
2 3 2 3

om
11 5 3 5 1 1 1
Vậy giá trị nhỏ nhất của P bằng − đạt tại a = − ,b = ,c =.
4 9 2 3 2 3
Bài 7. Cho a, b, c là độ dài ba cạnh một tam giác. Chứng minh rằng
.c
a b c a c b
3 + +  ≥ 2  + +  + 3 .
ok
b c a c b a
Lời giải
bo

Không mất tính tổng quát giả sử a = max {a, b, c}


- Nếu a ≥ c ≥ b thì
et

a b c a b c a b c a b c a b c
+ + ≥ + + ≥ 3 ⇒ 3 + +  ≥ 3 + +  ≥ 2  + +  + 3
b c a c a b b c a c a b c a b
vi

- Nếu a ≥ b ≥ c thì
ng

a b c a c b
Xét hàm số f (a )= 3  + +  − 2  + +  − 3
b c a c b a
(
a 2 − bc ( 3c − 2a ))
a

3 3c 2 2b
Ta có f '(a ) = − 2− + 2 =
kh

b a c a a 2bc
Do a ≥ b ≥ c ⇒ a 2 ≥ bc
b c
Khả năng 1. Nếu 3c > 2a hàm số đồng biến suy ra f (a ) ≥ f (b) = + −2≥0
c a
Khả năng 2. Nếu 3c < 2a hàm số nghịch biến suy ra
( b − c )2 3c − 2b 2 ( b − 2c ) + c ( b − c )
2
f (a ) ≥ f (b =
+ c) + = ≥0
bc b+c ( b + c )2
Bất đẳng thức được chứng minh.

513
Khám phá tư duy Kỹ thuật giải bất ĐT Bài toán Max – Min – Đặng Thành Nam

Bài 8. (TSĐH Khối A 2011) Cho x,y,z là ba số thực thuộc đoạn [1;4] và
x ≥ y, x ≥ z .
x y z
Tìm giá trị nhỏ nhất của biểu thức P= + + .
2x + 3y y + z z + x
Lời giải
x y z
Xét hàm số f ( z )= + + trên đoạn [1;4] ta có:
2x + 3y y + z z + x

f '(z) =

y
+
x
=
(
( x − y ) z 2 − xy
.
)
( y + z )2 ( x + z )2 ( x + z )2 ( y + z )2

n
.v
1 y z 6
+ Nếu x = y ⇒ P = + + = .
5 y+z z+ y 5

om
+ Nếu x > y khi đó f '(z) = 0 ⇔ z = xy ta có f’(z) đổi dấu từ âm sang dương khi
đi qua z = xy nên f(z) đạt cực tiểu tại z = xy . .c x
ok
x 2 y y 2
Do đó P =f (z) ≥ f( xy ) = + = + .
2x + 3y x + y 2 +3
x x
+1
y
bo

y
x
Đặt
= t , (1 < t ≤ 2 ) ta có
et

( 2 − t ) ( 35t 2 − 27t + 48) 34 34


vi

t2
2
P ≥ g=
(t) + = + ≥ .
2t + 3 t + 1 33 ( t + 1) ( 2t 2 + 3)
2 33 33
ng

Đẳng thức xảy ra khi và chỉ khi


z = xy , t = 2 ⇔ x = 4 y, z = 2 y ⇔ x = 4, y = 1, z = 2 .
a
kh

34
đạt tại =
Vậy giá trị nhỏ nhất của P bằng x 4,=
y 1,=
z 2.
33
Nhận xét. Do P dạng đẳng cấp bậc 0 nên ta có thể đặt=y a=
.x, z b.x,
1 
a, b ∈  ;1 đưa bài toán về tìm giá trị nhỏ nhất của biểu thức
4 
1 a b
P= + + đã được trình bày ở trên(xem thêm chủ đề sử dụng bất
2 + 3a a + b b + 1
đẳng thức phụ).

514
Cty TNHH MTV DVVH Khang Việt

1 
Bài 9. Cho các số thực a, b, c ∈  ;3 .
3 
a b c
Tìm giá trị lớn nhất của biểu thức S = + + .
a+b b+c c+a
Lời giải
a b c
Đặt f ( a ) = + + . Xét hai trường hợp sau:
a+b b+c c+a

b c ( b − c ) ( a 2 − bc )
• TH1: a ≥ b ≥ c . Ta có f ′ ( a ) = − = ≥0
( a + b )2 ( a + c )2 ( a + b )2 ( a + c )2

n
3 b c
Suy ra f ( a ) ≤ f ( 3) = = g (c) .

.v
+ +
3+b b + c c +3
( b − 3) ( 3b − c 2 )

om
−b 3
Mặt khác g ′ ( c ) = + = ≤0
( c + b )2 ( c + 3)2 ( c + 3)2 ( b + c )2
1
Suy ra g ( c ) ≤ g  =
3
+
3b 1
+ = h (b)
.c
 3  3 + b 3b + 1 10
ok
Ta có h′ ( b ) =
3 3 (1 − b )(1 + b ) 0
− = ≤
( 3b + 1) 2
( b + 3) 2
( 3b + 1)( b + 3)
bo

 1 8
Từ bảng biến thiên suy ra f ( a; b; c ) ≤ f  3;1;  = .
 3 5
et

8
• TH2: c ≥ b ≥ a . Từ TH1 ta có f ( c; b; a ) ≤ .
vi

5
( a − b )( b − a )( a − c ) ≤ 0 .
ng

Mặt khác f ( a; b; c ) −=
f ( c; b; a )
( a + b )( b + a )( a + c )
a

8
Suy ra f ( a; b; c ) ≤ .
kh

5
8  1 1   1 
Vậy max S = , đạt được khi và chỉ khi ( a, b, c ) ∈  3,1,  ,  ,3,1 ,  1, ,3   .
5  3  3   3 
Bài 10. Cho a, b, c ∈ [ 0;1] . Tìm giá trị lớn nhất của biểu thức
a b c
S= + + .
b +c +6
c + a + 6 a + b3 + 6
3 3 3 3 3

Lời giải
a b c
Đặt f ( c ) = 3 3 + 3 + 3 .
b + c + 6 c + a + 6 a + b3 + 6
3

515
Khám phá tư duy Kỹ thuật giải bất ĐT Bài toán Max – Min – Đặng Thành Nam

1 3ac 2 3c 2
Ta có f ′ ( c ) =3 − −
a + b3 + 6 b3 + c 3 + 6
(
) ( a + c + 6)
2 3 3 2

6ac ( 6 + b − 2c ) 6bc ( 6 + a − 2c )
3 3 3 3
f ′′ ( c ) =
− − ≤0
(b + c + 6) ( a + c + 6)
3 3 2 3 3 2

Nên f’(c) giảm trên [0; 1]. Suy ra


1 3a 3b 1 3
f ′ ( c )=
≥ f ′ (1) − − ≥ − 2. > 0
6+a +b
(7 + b ) (7 + a )
3 3 3 2 3 2 8 49

n
Suy ra f(c) tăng trên [0; 1].

.v
a b 1
Do đó S = f ( c ) ≤ f (1) = + + = g (a) .

om
b +73
a +73
a + b3 + 6
3

1 2a 2 b 3a 2
Ta có g ′ ( a ) =3 − −
b + 7 a3 + 7
(
) (a + b + 7)
2 3 3 .c 2

6ab ( 7 − 2a ) 6a ( b + 6 − 2a )
ok
3 3 3
g ′′ ( a ) =
− − ≤0
(a + 7) (a + b + 7)
3 3 3 3 3
bo

Nên g’(a) giảm trên [0; 1]. Suy ra


et

1 3b 3  1 1  5 3  5 − 3b
g ′ ( a ) ≥ g ′ (1=
) − − =  3 −  − 3 + >0
b +7
(
 b + 7 8  8 b + 7 
)
3 64 7 + b3 2 64
vi

2
b
S g ( a ) ≤ g (1=
)
ng

Suy ra g(a) tăng trên [0; 1]. Do đó = + = h(b) .


b +7 8
3

(b )
2
+7 − 48b 2
a

3
1 6b 2
Ta có h′ ( b ) = − − > 0, ∀b ∈ [ 0;1] .
( ) ( )
kh

8 b3 + 7 2 2
8 b3 + 7

3 3
Suy ra h(b) tăng trên [0; 1], nên h ( b ) ≤ h (1) = ⇒ S ≤ .
8 8
3
Với a = b = c = 1 thì max S =
8

516
Cty TNHH MTV DVVH Khang Việt

Bài 11. Xét các số thực dương x, y, z thỏa mãn hệ điều kiện
2
 5 ≤ z ≤ min { x, y} (1)

 4
 xz ≥ ( 2)
 15
 1
 yz ≥ 5 ( 3)

1 2 3
Hãy tìm giá trị lớn nhất của biểu thức P ( x, y, z ) = + + .
x y z

n
Lời giải
 4 

.v
Từ điều kiện (1) và (2) suy ra x ≥ max  z ,  (4)
 15 z 

om
1 1 2
a) Xét hàm số f ( x )= + với x > 0 và tham số z ≥ . Xét hai trường hợp
x z 5
2 4 1 1 2
• Nếu z ≥ thì x ≥ z ≥ theo (4) nên f ( x ) ≤ + = ≤ 15 (5)
15 15 z
.c z z z
ok
2 2 4 15 z 1
• Nếu ≤z≤ thì x ≥ ≥ z theo (4) nên f ( x ) ≤ + =g ( z) .
5 15 15 z 4 z
bo

2 2
Xét hàm số g(z) với ≤z≤ . Ta có
5 15
et

15 1 2
g′( z ) = − 2 <0⇔ z< .
4 z 15
vi

2
Do đó g(z) là hàm giảm và f ( z ) ≤ g ( z ) ≤ g   =
4 (6)
ng

5
So sánh (5) và (6) ta có
1 1 1 1 2 2
a

+ ≤ 4 và + =4 ⇔ x = ,z = (7)
x z x z 3 5
kh

1 1 2
b) Xét hàm số h ( y =
) + với tham số z ≥
y z 5
 1
Từ điều kiện (1) và (3) suy ra y ≥ max  z ,  (8)
 5z 
Lập luận tương tự phần a) ta được
1
• Nếu z ≥ thì h ( y ) ≤ 2 5 (9)
5

517
Khám phá tư duy Kỹ thuật giải bất ĐT Bài toán Max – Min – Đặng Thành Nam

2 1 9
• Nếu ≤z≤ thì h ( y ) ≤ (10)
5 5 2
So sánh (9) và (10) ta có
1 1 9 1 1 9 2 1
+ ≤ và + = ⇔ x= ,y= (11)
y z 2 x z 2 5 2
So sánh kết quả phần a) và b) ta có
1 1 1 1 9
P ( x, y, z ) =  +  + 2  +  ≤ 4 + 2. = 13
x y  y z 2
2 1 2
Đẳng thức xảy ra khi và chỉ khi= x = ,y =

n
,z
3 2 5

.v
Vậy giá trị lớn nhất của P bằng 13.
1 

om
Bài 12. Cho x,y,z là các số thực thuộc đoạn  ;2  . Tìm giá trị nhỏ nhất của biểu
2 
x y z y x z
thức P= 8  + +  − 5  + +  . .c
 y z x  x z y
Lời giải
ok
Nhận xét. Ta thử đạo hàm P theo biến x ta được:

 1 z   y 1  8 x 2 z − 8 z 2 y + 5 yz − 5 x 2 y (x 2
)
− yz ( 8 z − 5 y )
bo

8 =− 2  − 5 − 2 +  = .
y x   x z x 2 yz x 2 yz
et

Để P min suy nghĩ đến đạo hàm không âm. Vậy trước tiên cần có x 2 ≥ yz ta chỉ
cần giả sử x = max { x, y, z} .
vi

Không mất tính tổng quát giả sử x = max { x, y, z} .


ng

8z − 5 y 8z − 5 y 8 y 5z
Viết lại P dưới dạng: P= f ( x)= .x + + − với yz ≤ x ≤ 2
yz x z y
a

(x 2
)
− yz ( 8 z − 5 y )
kh

Ta có f '( x) = .
x 2 yz
TH1: Nếu 8 z − 5 y > 0 ⇒ f '( x) ≥ 0 nên f(x) là hàm đồng biến trên  yz ;2  do đó

8 y 5z z y
f ( x) ≥ f ( yz ) = − + 16 − 10 .
z y y z
y  1  5 16
Đặt
= t ,  t ∈  ;2   . Khi đó P = f ( x) ≥ f ( yz ) = g (t ) = 8t 2 − 2 + − 10t .
z  2  t t

518
Cty TNHH MTV DVVH Khang Việt

5 16 1 
Xét hàm số g (t ) = 8t 2 − + − 10t liên tục trên  ;2  ta có:
t 2 t 2 

10 16 ( t − 1)(8t − 5) ( t 2 + t + 1) t = 1
1 
t∈ ;2 
g '(t=
) 16t + 3 − 2 − 10
= ; g '(t= → 5 .
2 
) 0 ←
t t t3 t =
 8
1  5  387 75
Ta có g =  g= (1) 9, g =  , g=
(2) .
2  8  40 4
1
Suy ra min g (t ) = 9 xảy ra tại t = 1 hoặc t = hay x= y= z hoặc
= z 4=
y, x 2 y .
2

n
TH2: Nếu 8 y − 5 z ≤ 0 ⇒ f '( x) ≤ 0 suy ra

.v
 2 y z   y z 2
f ( x) ≥ f (2)= 8  + +  − 5  + + = g ( y ) .

om
 y z 2  2 y z 
Thực hiện tương tự.
1 
Bài 13. Cho a,b,c là các số thực thuộc đoạn  ;1 . Tìm giá trị lớn nhất của biểu
.c
2 
a −b b−c c−a
ok
thức P = + + .
c a b
Lời giải
bo

Ta có:
a − b b − c c − a ab ( a − b ) + bc ( b − c ) + ca ( c − a ) ( a − b )( b − c )( c − a )
et

P= + + = = .
c a b abc abc
vi

Do P là biểu thức đối xứng với ba biến a,b,c nên không mất tính tổng quát giả
sử a ≥ b ≥ c . Khi đó:
ng

(=
a − b )( b − c )( a − c ) b − c  a − ( b + c ) a + bc 
2
P  .
abc bc  a 
a

b − c  a − ( b + c ) a + bc 
kh

2
1
Xét hàm số f (a ) =   với ≤ c ≤ b ≤ a ≤ 1 ta có:
bc  a  2

b − c  bc  ( b − c ) ( a 2 − bc )
f '(a=
) 1 − =
 ≥0.
bc  a 2  a 2bc
1 
Do đó f(a) là hàm đồng biến trên  ;1 .
2 
b−c 1− b 1  1
= f (a ) ≤ f (1)
Suy ra P = (1 − b − c + bc=) g (c=) + c  − 1 + b − .
bc c b  b

519
Khám phá tư duy Kỹ thuật giải bất ĐT Bài toán Max – Min – Đặng Thành Nam

1− b 1  1 1 
Xét hàm số g (c=
) + c  − 1 + b − trên đoạn  ;1 ta có:
c b  b 2 

g '(c) =−
1− b 1
+ − 1 =(1 − b )  − 2  =
( ≤
)
 1 1  (1 − b ) c − b (1 − b )( c − b )
2
≤0.
c 2 b b c  bc 2 bc 2
1 
Do đó g(c) là hàm nghịch biến trên  ;1 .
2 
1 3 1
Suy ra P ≤ g (c) ≤ g   = h(b) = − b − .
2 2 2b

n
3 1 1 
Xét hàm số h(b) = −b− liên tục trên đoạn  ;1 ta có:
2 

.v
2 2b
1 1
h '(b) =−1 + 2 ; h '(b) =0 ⇔ b = .

om
2b 2
1
Ta có h(b) đối dấu từ dương sang âm khi đi qua b = nên h(b) đạt cực đại tại
.c2
1  1  3− 2 2
b= . Do đó P ≤ h(b) ≤ h  = .
ok
2  2 2
1 1
Đẳng thức xảy ra khi và chỉ khi =
a 1,=
b =
,c và các hoán vị.
bo

2 2
3− 2 2 1 1
Vậy giá trị lớn nhất của P bằng đạt tại =a 1,= b =
,c và các hoán vị.
et

2 2 2
Nhận xét. Điểm quan trọng trong lời giải trên là phát hiện đẳng thức
vi

a −b b−c c−a
+ + = −
( a − b )( b − c )( c − a ) .
ng

c a b abc
Ngoài ra dễ thấy P là biểu thức đẳng cấp bậc 0 và dạng đối xứng nên không mất
tính tổng quát ta có thể giả sử a ≥ b ≥ c . Ta đặt = a x.c,= b y.c, ( 2 ≥ x ≥ y ≥ 1) .
a

( x − 1)( y − 1)( x − y ) .
kh

Bài toán đưa về tìm giá trị lớn nhất của biểu thức P =
xy
Đây chính là bài toán 2 tôi đã trình bày.
Bài 14. Cho a,b,c là các số thực thuộc đoạn [1;2] và 4a + 2b + c =
11 .
1 2 3
Tìm giá trị lớn nhất và nhỏ nhất của biểu thức P = + + .
a b c
Lời giải
4 2 3
Theo giả thiết ta có 4a =11 − 2b − c suy ra P= f (b=
) + + .
11 − 2b − c b c

520
Cty TNHH MTV DVVH Khang Việt

4 2 3
=
Xét hàm số f (b) + + liên tục trên [1;2] ta có
11 − 2b − c b c
2 8b − 2 (11 − 2b − c ) 2 ( 4b + c − 11)(11 − c )
2 2
8
=f '(b) = − = < 0, ∀b, c ∈ [1;2] .
(11 − 2b − c )2 b2 b2 (11 − 2b − c )2 b 2 (11 − 2b − c )
2

Do đó f(b) là hàm nghịch biến trên đoạn [1;2] suy ra f (2) ≤ f (b) ≤ f (1) .
Tìm giá trị lớn nhất của P.
4 3
Xét hàm số g (c)= f(1)= + 2 + ta có:
9−c c
3 c 2 + 54c − 243

n
4
=
g '(c) =
− < 0, ∀c ∈ [1;2] .
( 9 − c )2 c 2 c2 ( 9 − c )

.v
11
Vậy g(c) là hàm nghịch biến trên đoạn [1;2] ⇒ P= f (b) ≤ f (1)= g(c) ≤ g(1)=

om
.
2
Đẳng thức xảy ra khi và chỉ khi b= c= 1, a= 2 .
11 .c
Vậy giá trị lớn nhất của P bằng đạt tại a= 2, b= c= 1 .
2
ok
Tìm giá trị nhỏ nhất của P.
4 3
Xét hàm số h(= =
c) f (2) + 1 + ta có:
7−c
bo

c
4 3 c 2 + 42c − 147
=
h '(c) =
− < 0, ∀c ∈ [1;2] .
( 7 − c )2 c 2 c2 ( 7 − c )
2
et

Do đó h(c) là làm nghịch biến trên đoạn [1;2] .


vi

33
Suy ra P = f (b) ≥ f (2) = h(c) ≥ h(2) =
ng

.
10
5
Đẳng thức xảy ra khi và chỉ khi b= c= 2, a= .
a

4
kh

33 5
Vậy giá trị nhỏ nhất của P bằng đạt tại a= , b= c= 2 .
10 4
Bài tập tương tự
Cho a,b,c là các số thực thuộc đoạn [1;3] thỏa mãn a + 2b + 3c =
12 .
3 2 1
Tìm giá trị nhỏ nhất của biểu thức P =+ + .
a b c
Bài 15. Cho a,b,c là các số thực thuộc đoạn [1;2] . Tìm giá trị nhỏ nhất của biểu
1 1 1
thức P = + + .
4 + a − ab 4 + b − bc 4 + c − ca
521
Khám phá tư duy Kỹ thuật giải bất ĐT Bài toán Max – Min – Đặng Thành Nam

Lời giải
Coi P là hàm số của a và b,c là tham số ta có:
1 1 1
P= f (a) = + + .
4 + a − ab 4 + b − bc 4 + c − ca
b −1 c
=f '(a ) + ≥ 0, ∀a, b, c ∈ [1;2] .
( 4 + a − ab ) ( 4 + c − ca )2
2

Do đó f(a) là hàm đồng biến trên đoạn [1;2] .


1 1 1
Suy ra f (a ) ≥ f (1) = g (b) = + + .
5 − b 4 + b − bc 4

n
1 1 1
Xét hàm số g (b) = + + liên tục trên đoạn [1;2] ta có:

.v
5 − b 4 + b − bc 4
1 c −1
g= + ≥ 0, ∀b, c ∈ [1;2] . Do đó g(b) là hàm đồng biến

om
'(b)
( 5 − b ) ( 4 + b − bc )2
2

trên đoạn [1;2] .

Suy ra P ≥ g (b) ≥ g (1) =


1
+
1
≥ +
1 1 3
= .
.c
2 5 − c 2 5 −1 4
ok
3
Vậy giá trị nhỏ nhất của P bằng đạt tại a= b= c= 1 .
4
bo

Bài 16. (Việt Nam TST 2001) Cho a,b,c là các số thực dương thỏa mãn
21ab + 2bc + 8ca ≤ 12 .
et

1 2 3
Tìm giá trị nhỏ nhất của biểu thức P = + + .
a b c
vi

Lời giải
2 8 21 12
ng

Ta có: 21ab + 2bc + 8ca ≤ 12 ⇔ + + ≤ .


a b c abc
1 1 1
Đặt= x = ,y = , z , ( x, y, z > 0 ) và 2 x + 8 y + 21z ≤ 12 xyz . Khi đó P =x + 2 y + 3 z .
a

a b c
kh

2x + 8 y 7
Ta có: z (12 xy − 21) ≥ 2 x + 8 y > 0 ⇒ z ≥ và x > .
12 xy − 21 4y
2x + 8 y 2x + 8 y
Khi đó P ≥ x + 2 y + 3. =x + 2 y + .
12 xy − 21 4 xy − 7
2x + 8 y  7 
Xét hàm số f ( x) =x + 2 y + trên khoảng  ; +∞  ta được:
4 xy − 7  4y 
14 + 32 y 2 7 + 32 y 2 + 14
; f '( x) =0 ⇔ ( 4 xy − 7 ) =14 + 32 y 2 ⇔ x =x0 =
2
f '( x) =1 − .
( 4 xy − 7 ) 2 4y

522
Cty TNHH MTV DVVH Khang Việt

Ta có f’(x) đổi dầu từ âm sang dương khi đi qua x0 nên f(x) đạt cực tiểu tại x0 .

9 32 y 2 + 14
Do đó f ( x) ≥ f ( x0 ) = 2 y + + .
4y 2y

9 32 y 2 + 14
Xét hàm số g ( y ) = 2 y + + trên ( 0;+∞ ) ta có:
4y 2y

g '( y ) = 2 −
9
4y 2

7
y 2 32 y 2 + 14
(
; g '( y ) = 0 ⇔ 8 y 2 − 9 ) 5
32 y 2 + 14 − 28 = 0 ⇔ y = .
4

5 5
Ta có g’(y) đổi dấu từ âm sang dương khi đi qua nên tại y = g(y) đạt cực

n
4 4

.v
 5  15
tiểu. Do đó g ( y ) ≥ g   = .
4 2

om
 5  15
Suy ra P ≥ f ( x) ≥ f ( x0 ) =g(y) ≥ g   = .
4 2

Vậy giá trị nhỏ nhất của P bằng


15
đạt tại =
y
5
=
, x 3,=
z
2
hay
.c
2 4 3
ok
1 4 3
=a = ,b = ,c .
3 5 2
bo

Bài 17. Cho a,b,c là các số thực thuộc đoạn [1;2] . Tìm giá trị lớn nhất của biểu
10a 11b 2014c
thức P = + +
et

.
bc ca ab
Lời giải
vi

2014c 1  10a 11b 


Ta lần lượt coi P là hàm của c: f (c) = +  + .
ng

ab c b a 
Ta có:
a

2014 1  10a 11b  2014c 2 − 10a 2 − 11b 2 2014.12 − 10.22 − 11.22


f '(c) = −  + = ≥ >0.
kh

ab c 2  b a  abc 2 abc 2
Do đó f(c) là hàm đồng biến trên [1;2] .
4028 5a 11b
Do đó f (c) ≤ f (2) = g (b) =+ + .
ab b 2a
4028 5a 11b
Xét hàm số g (b)= + + trên đoạn [1;2] ta có:
ab b 2a
11 4028 11b 2 − 2.4028 11.22 − 2.4028
g '(b) = − = ≤ <0.
2a ab 2 2ab 2 2ab 2

523
Khám phá tư duy Kỹ thuật giải bất ĐT Bài toán Max – Min – Đặng Thành Nam

Do đó g(b) là hàm nghịch biến trên đoạn [1;2] suy ra


4028 11
g (b) ≤ g (1)= h(a )= + 5a + .
a 2a
4028 11
Xét hàm số h(a= ) + 5a + trên đoạn [1;2] ta có:
a 2a
4028 11
h '(a) =− 2 + 5 − 2 < 0, ∀a ∈ [1;2]. Do đó h(a) là hàm nghịch biến trên [1;2]
a 2a
8077 8077
Vì vậy h(a ) ≤ h(1)
= ⇒P≤ . Đẳng thức xảy ra khi và chỉ khi
2 2

n
a= b= 1, c= 2 .

.v
8077
Vậy giá trị lớn nhất của P bằng đạt tại a= b= 1, c= 2 .
2

om
C. BÀI TẬP RÈN LUYỆN
Bài 1. Cho 2 số thực x,y thuộc [ 0;1] . Tìm giá trị lớn nhất của biểu thức
.c
( )
P= 2 x3 + y 3 − x 2 y − y 2 − x .
ok
Bài 2. Cho x,y là hai số thực dương thỏa mãn x, y ≤ 4 .
bo

x y 12
Tìm giá trị nhỏ nhất của biểu thức P = + + .
2 y x xy
Bài 3. Cho hai số thực x,y thoả mãn điều kiện x ≥ y ≥ 1 .
et

2 ( x + 1) 2 x ( x + 1)
Chứng minh rằng x 2 + x + 4 ≥ + + 2y .
vi

x+ y y +1
ng

Bài 4. Cho các số thực 0 ≤ a ≤ b ≤ c ≤ 1 . Tìm giá trị lớn nhất của biểu thức
P= (a 2
)
− b 2 ( b − c ) + c 2 (1 − c ) .
a

Bài 5. Cho a,b,c là các số thực dương thỏa mãn a + b + c = 8.


kh

1 3 8
Tìm giá trị nhỏ nhất của biểu thức P = + + .
a b 3c + 2a
Bài 6. Cho x,y,z là các số thực dương thỏa mãn x ≥ y ≥ z .
x z y x y z
Chứng minh rằng + + ≥ + + .
z y x y z x
Bài 7. Cho x,y,z là ba số thực thuộc đoạn [1;3] . Tìm giá trị lớn nhất của biểu thức
x y z y x z
P= + + + + + .
y z x x z y

524
Cty TNHH MTV DVVH Khang Việt

1 
Bài 8. Cho x,y,z là các số thực thuộc đoạn  ;1 .
2 
Tìm giá trị lớn nhất và giá trị nhỏ nhất của biểu thức
21 1 1
P = ( x + y + z)  + +  .
x y z
Bài 9. Cho x,y,z là ba số thực thuộc đoạn [1;9] và x ≥ y, x ≥ z .
x y z
Tìm giá trị giá trị nhỏ nhất của biểu thức P = + + .
x + 2y y + z z + x
1 

n
Bài 10. Cho x,y,z là các số thực thuộc đoạn  ;3 .
3 

.v
x y z
Tìm giá trị lớn nhất và giá trị nhỏ nhất của biểu thức P = + + .

om
x+ y y+z z+x
Bài 11. Cho x,y,z là các số thực thuộc đoạn [1;3] . Tìm giá trị nhỏ nhất của biểu
x y z x z y .c
thức P= 5  + +  − − − .
 y z x z y x
ok
Bài 12. Cho a,b,c là các số thực thuộc đoạn [1;2] .

Chứng minh rằng a3 + b3 + c3 ≤ 5abc .


bo

Bài 13. Cho a,b,c là các số thực thuộc đoạn [1;2] .


et

1 1 1
Chứng minh ( a + b + c )  + +  ≤ 10 .
vi

a b c
Bài 14. Cho x,y,z là các số thực thuộc đoạn [1;2] .
ng

10 x 11 y 12 z
Tìm giá trị lớn nhất của biểu thức P = + + .
a

yz zx xy
Bài 15. Cho x,y,z là các số thực thuộc đoạn [1;3] .
kh

36 x 2 y z
Tìm giá trị nhỏ nhất của biểu thức P = + + .
yz zx xy
Bài 16. Cho a,b,c là các số thực thuộc đoạn [1;3] thỏa mãn a + 2b + 3c =
12 .
3 2 1
Tìm giá trị lớn nhất và nhỏ nhất của biểu thức P = + + .
a b c
Bài 17. Cho x,y là hai số thực thuộc đoạn [1;2] . Tìm giá trị lớn nhất của biểu thức

525
Khám phá tư duy Kỹ thuật giải bất ĐT Bài toán Max – Min – Đặng Thành Nam

y −1 1− x
P= x− y+ + .
x y
1 
Bài 18. Cho a,b,c là các số thực thuộc đoạn  ;2  .
2 
a −b b−c c−a
Tìm giá trị lớn nhất của biểu thức P = + + .
c a b
Bài 19. Cho a,b,c là các số thực dương thoả mãn điều kiện 21ab + 2bc + 8ca ≤ 12 .
1 2 3
Tìm giá trị nhỏ nhất của biểu thức P = + + .
a b c

n
Bài 20. Cho x,y,z là các số thực dương thoả mãn điều kiện x + y + z ≤ xyz .

.v
Tìm giá trị nhỏ nhất của biểu thức P =x + 2 y + 5 z .

om
5
Bài 20. Cho x,y,z là các số thực dương thoả mãn điều kiện x + y + z = xyz .
3
Tìm giá trị nhỏ nhất của biểu thức P = 6 ( x + y ) + z .
.c
Bài 21. Cho x,y,z là các số thực dương thoả mãn điều kiện x 2 + y 2 + z 2 =
6.
ok
x 2 y 5z
Tìm giá trị nhỏ nhất của biểu thức P = + + .
yz zx xy
bo

7
Bài 22. Cho a,b,c là các số thực dương thỏa mãn điều kiện ab > và
3
100
et

3a + 57b + 7c= 3abc + .


a
Tìm giá trị nhỏ nhất của biểu thức P = a + b + c .
vi

Bài 23. Cho a, b, c là độ dài ba cạnh của một tam giác. Chứng minh rằng
ng

a ( b − c ) + b ( c − a ) + c ( a − b ) + 4abc > a 3 + b3 + c3 .
2 2 2

Bài 24. Cho a, b, c là độ dài ba cạnh của một tam giác (có thể suy biến).
a

Đặt T =
( a − b )( b − a )( a − c ) . Tìm maxT và chứng minh rằng 1
max T <
kh

.
( a + b )( b + a )( a + c ) 21
Bài 25. Xét các số thực dương x, y, z thỏa mãn hệ điều kiện


0 < z ≤ y ≤ x ≤ 3 (1)
 3 2
 + 2 ≥1 ( 2)
 xy y
 18 4 18
 2 + 2 + 2 ≥3 ( 3)
 x y y z z x

526
Cty TNHH MTV DVVH Khang Việt

1 80 3 18 3
Hãy tìm GTLN của biểu thức P ( x, y, z ) = xyz + x + y .
2 27 8
D. HƯỚNG DẪN GIẢI – ĐÁP SỐ
Bài 1. Xét hàm số f ( x)= 2 x3 − x 2 y − x + 2 y 3 − y 2 trên đoạn [ 0;1] ta có:
 y − y2 + 6
=  x ∉ [ 0;1]
f '( x) =6 x 2 − 2 xy − 1; f '( x) =⇔
0  6
.

 y + y 2
+ 6
=  x ∈ [ 0;1]
6

n
y + y2 + 6
Ta có f’(x) đổi dấu từ âm sang dương khi đi qua x = nên f(x) đạt

.v
6
y + y2 + 6

om
cực tiểu tại x = .
6
{ }
Do đó P ≤ Max { f (0); f (1)}= Max 2 y 3 − y 2 ;2 y 3 − y 2 − y + 1 = 2 y 3 − y 2 − y + 1 vì
.c
2 y 3 − y 2 − y + 1 ≥ 2 y 3 − y 2 , ∀y ∈ [ 0;1] .
ok
Mặt khác 2 y 3 − y 2 − y += ( )
1 y ( y − 1)( 2 y + 1) + 1 ≤ 1, ∀y ∈ [ 0;1] .
1 y 2 y 2 − y − 1 +=
Suy ra P ≤ 1 . Đẳng thức xảy ra khi và chỉ khi=
x 1,=
y 0 hoặc x= y= 1 .
bo

Vậy giá trị lớn nhất của P bằng 1 đạt tại=


x 1,=
y 0 hoặc x= y= 1 .
et

x y 12
Bài 2. Xét hàm số f ( x) = + + với x ∈ ( 0;4] ta có:
2 y x xy
vi

1 y 12 x 2 − 2 y 2 − 24 x 2 − 24
f '( x) = − 2− 2 = < < 0.
ng

2y x yx 2 yx 2 2 yx 2
Do đó f(x) là hàm nghịch biến trên khoảng ( 0;4] suy ra
a

P = f ( x) ≥ f (4) =
y 5
+ =
( y − 4 )( y − 5) + 9 ≥ 9 , ∀0 < y ≤ 4 .
kh

4 y 4y 4 4
Đẳng thức xảy ra khi và chỉ khi x= y= 4 .
9
Vậy giá trị nhỏ nhất của P bằng đạt tại x= y= 4 .
4
Bài tập tương tự
Cho a,b,x,y là các số thực dương thỏa mãn a3 + b3 =
1 và x, y ≤ 4 .
x 2 + 2 y 2 + 24
Tìm giá trị nhỏ nhất của biểu thức P = .
(
xy a 2 + b 2 )
527
Khám phá tư duy Kỹ thuật giải bất ĐT Bài toán Max – Min – Đặng Thành Nam

2 ( x + 1) 2 x ( x + 1)
Bài 3. Xét hàm số f ( x) = x 2 + x + 4 − − − 2 y với x ≥ y ≥ 1 ta có
x+ y y +1

2 ( 2 x + 1) 2 ( y − 1)  2x + 1 2 
f '( x) = 2 x + 1 − − = ( y − 1)  − ≥0.
y +1 ( x + y) 2

y +1 ( x + y )2 
1
Vì vậy f(x) là hàm đồng biến suy ra f ( x) ≥ f ( y ) = y 2 − 3 y + 3 − =
( y − 1)3 ≥ 0 .
y y
Bất đẳng thức được chứng minh. Đẳng thức xảy ra khi và chỉ khi x= y= 1 .
1 1 1 z y

n
Bài 6. Xét hàm số f ( x)= x  −  + ( y − z ) . + − ta có:
z y x y z

.v
1 1 y−z ( y − z ) ( x 2 − yz )

om
f '( x) = − − 2 = ≥ 0, ∀x ≥ y ≥ z > 0 .
z y x x 2 yz
x z y x y z
Vậy f(x) là hàm đồng biến suy ra f ( x) ≥ f ( y ) =
0 hay .c + + ≥ + + .
z y x y z x
Đẳng thức xảy ra khi và chỉ khi x = y .
ok
Bài 7. Không mất tính tổng quát giả sử x ≥ y ≥ z . Coi P là hàm của x ta được:
1 1 y+z y z
bo

P = f ( x) =  +  x + + + với x ∈ [1;3] ta có:


 y z x z y

1 1 y + z ( y + z ) x − yz ( )
et

2
f '( x) = + − 2 = ≥0.
y z x x 2 yz
vi

Do đó f(x) là hàm đồng biến trên đoạn [1;3] .


ng

1 1 y+ z y z
Suy ra P= f ( x) ≤ f (3)= g ( z )= 3  +  + + + .
 y z 3 z y
a

1 1 y+ z y z
+ + trên đoạn [1;3] ta có:
kh

Xét hàm số g ( z )= 3  +  +
 y z 3 z y

3 1 y 1 ( y + 3) z − 3 y
g '( z ) =− 2 + − 2 + =
2
(
≤ 0, ∀1 ≤ z ≤ y ≤ 3 .
)
z 3 z y 3 yz 2
Suy ra g(z) là hàm nghịch biến trên đoạn [1;3] hay
 1  y +1 1 4 ( y − 1)( y − 3) 26 26
P ≤ g (z) ≤ g(1)
= 3  + 1 + + y += + ≤ , ∀y ∈ [1;3] .
y  3 y 3y 3 3
Đẳng thức xảy ra khi và chỉ khi x= 3, y= z= 1 hoặc x= y= 3, z= 1 .

528
Cty TNHH MTV DVVH Khang Việt

26
Vậy giá trị lớn nhất của P bằng đạt tại x= 3, y= z= 1 hoặc x= y= 3, z= 1 và
3
các hoán vị.
Cách 2: Không mất tính tổng quát giả sử x ≥ y ≥ z khi đó:
x y x
 z +1≥ +
( x − y )( y − z ) ≥ 0 ⇔ xy + yz ≥ y + zx ⇒ 
2 z y
.
1 + z ≥ y
+
z
 x x y
x z
Suy ra P ≤ 2  +  + 2 .

n
 z x

.v
x 3
Đặt=t , t ∈ [1;3] suy ra ( t − 1)( t − 3) ≤ 0 ⇔ t 2 + 3 ≤ 4t ⇔ t + ≤ 4 .
z t

om
 1  2 4 4 26
Suy ra P ≤ 2  t +  + 2 ≤ 2  4 −  + 2 = 10 − ≤ 10 − = .
 t  t t 3 3
.c  1 1 1  35
Nhận xét. Ta có thể viết lại bất đẳng thức dưới dạng: ( x + y + z )  + +  ≤ .
x y z 3
ok
Bất đẳng thức tổng quát của dạng toán này đã đề cập đến trong chủ đề trước.
Bài tập tương tự
bo

Cho x,y,z là các số thực thuộc đoạn [1;2] . Chứng minh rằng
1 1 1
( x + y + z )
et

+ +  ≤ 10 .
x y z
vi

Bài 8. Tìm giá trị nhỏ nhất của P.


3 1 1 1 3 27
ng

Ta có: x + y + z ≥ ; ( x + y + z )  + +  ≥ 9 ⇒ P ≥ .9 = .
2 x y z 2 2
1
a

Đẳng thức xảy ra khi và chỉ khi x= y= z= .


2
kh

27 1
Vậy giá trị nhỏ nhất của P bằng đạt tại x= y= z= .
2 2
Tìm giá trị lớn nhất của P.
Không mất tính tổng quát giả sử x ≥ y ≥ z khi đó:
21 1 1 1 
Xét hàm số f ( x) = ( x + y + z )  + +  trên đoạn  ;1 ta có:
x y z 2 

529
Khám phá tư duy Kỹ thuật giải bất ĐT Bài toán Max – Min – Đặng Thành Nam

 1 1 1  ( x + y + z)
f '( x) = 2 ( x + y + z )  + +  −
2
= ( x + y + z)
( )
 xyz + 2 x 2 − yz ( y + z ) 
>0
 x y z  x 2  x 2
yz 
 
vì x 2 ≥ yz .
1 
Do đó f(x) là hàm đồng biến trên  ;1 suy ra
2 
 1 1
P = f (x) ≤ f(1) = g ( y ) = 1 + +  (1 + y + z ) .
2

 y z
 1 1 1 

n
Xét hàm số g ( y ) = 1 + +  (1 + y + z ) trên đoạn  ;1 ta có:
2

 y z 2 

.v
(1 + y + z )2 + 2 1 + y + z 1 + 1 + 1  =1 + y + z  yz + ( 2 y )
− z ( z + 1) 
2

om
( ) >0
g '( y ) =− 2  ( ) 
y  y z y2 z
 
vì 2 y = 2 y. y ≥ y ≥ z .
2
.c
1 
Vậy g(y) là hàm đồng biến trên đoạn  ;1 suy ra
ok
2 

 1 (
( z − 1) 2 z 2 + 11z − 4 ) 1 
bo

P ≤ g ( y ) ≤ g (1)=  2 +  ( 2 + z ) =
2
+ 27 ≤ 27, ∀z ∈  ;1 .
 z z 2 
Đẳng thức xảy ra khi và chỉ khi x= y= z= 1 .
et

Vậy giá trị lớn nhất của P bằng 27 đạt tại x= y= z= 1 .


vi

Nhận xét. Sử dụng bất đẳng thức AM-GM ta tìm giá trị lớn nhất của P như sau:
3
1 1 1 1 
ng

P≤  + + + 2 ( x + y + z ) .
27  x y z 
( x − 1)( 2 x − 1) + 3 ≤ 3, ∀x ∈  1 ;1 .
a

1
Ta=
có: 2 x + 2 
 
kh

x x
1 1 1
≤ 3;2 z + ≤ 3 . Do đó P ≤ ( 3 + 3 + 3) =
3
Tương tự ta có: 2 y + 27 .
y z 27
Đẳng thức xảy ra khi và chỉ khi x= y= z= 1 .
x y z
Bài 9. Xét hàm số f ( z ) = + + trên đoạn [1;9] ta có:
x + 2y y + z z + x

f '(z) =

y
+
x
=
(
( x − y ) z 2 − xy
.
)
( y + z )2 ( x + z )2 ( x + z )2 ( y + z )2
530
Cty TNHH MTV DVVH Khang Việt

1 y z 4
+ Nếu x = y ⇒ P = + + = .
3 y+z z+ y 3
+ Nếu x > y khi đó f '(z) = 0 ⇔ z = xy ta có f’(z) đổi dấu từ âm sang dương khi
đi qua z = xy nên f(z) đạt cực tiểu tại z = xy .
x
x 2 y y 2
Do đó P =f (z) ≥ f( xy ) = + = + .
x + 2y x + y x +2 x
+1
y y

n
x
Đặt
= t , (1 < t ≤ 3) ta có

.v
y

( 3 − t ) ( 7t 2 − 16t + 10 ) 29 29

om
t2 2
P ≥ g (t)= 2 + = + ≥ .
t + 2 t +1 22 ( t + 1) ( t 2 + 2 ) 22 22

Đẳng thức xảy ra khi và chỉ khi .c


z = xy , t = 3 ⇔ x = 9 y, z = 3 y ⇔ x = 9, y =1, z = 3 .
ok
29
Vậy giá trị nhỏ nhất của P bằng đạt tại =
x 9,= y 1,= z 3.
22
bo

x y z
Bài 10. Giả sử z = min { x, y, z} khi đó xét hàm số f ( z ) = + + ta có:
x+ y y+z z+x

( )
et

y x ( x − y ) z 2 − xy x = y
f '( z ) =
− + = ; f '( z ) =
0 ⇔  .
( y + z )2 ( x + z )2 ( x + z )2 ( y + z )2
vi

 z = xy
ng

1 y z 3
+ Nếu x = y ⇒ P = + + = .
2 y+z z+ y 2
+ Nếu x > y ⇒ f '( z ) ≤ 0 nên f(z) là hàm nghịch biến do đó
a
kh

x 2 y t2 2 x
P=f ( z ) ≥ f ( xy ) = + = + =
với t , (1 < t ≤ 3) .
x+ y x + y t +1
2 t + 1 y

t2 2 ( 3 − t ) ( 2t 2 − 2t + 1) 7 7
=
Mặt khác: + + ≥ , ∀t ∈ (1;3] .
t +1 t +1 5 ( t + 1) ( t 2 + 1)
2 5 5

Đẳng thức xảy ra khi và chỉ khi


1
z = xy , t = 3 ⇔ x = 9 y, z = 3 y ⇔ x = 3, y = , z =1 .
3

531
Khám phá tư duy Kỹ thuật giải bất ĐT Bài toán Max – Min – Đặng Thành Nam

7 1
Vậy giá trị nhỏ nhất của P bằng đạt tại=
x 3,=
y =
, z 1 hoặc các hoán vị.
5 3
Tìm giá trị lớn nhất thực hiện tương tự
Bài 12. Không mất tính tổng quát giả sử 1 ≤ c ≤ b ≤ a ≤ 2 khi đó bất đẳng thức
tương đương với: a3 − 5bca + b3 + c3 ≤ 0 .
Xét hàm số f (a ) = a3 − 5bca + b3 + c3 trên đoạn [1;2] ta có
f '(a ) =
3a 2 − 5bc; f ''(a ) =
6a > 0 .
Suy ra f (a ) ≤ max { f (1); f (2)= { }
} max 1 − 5bc + b3 + c3 ;8 − 10bc + b3 + c3 .

n
Ta chỉ cần chỉ ra rằng f (1) ≤ 1, f (2) ≤ 0 .

.v
+ Xét hàm số g (b) = f (1) = b3 − 5cb + c3 + 1 trên đoạn [1;2] ta có

om
g '(b) =
3b 2 − 5c; g ''(b) =
5b > 0 .
Suy ra g (b) ≤ max { g (1); g (2)
= {
} max c3 − 5c + 2; c3 − 10c + 9 . }
− 5c + 2 = ( c − 2 ) ( c − 1 + 2c ) ≤ 0, ∀c ∈ [1;2]
Chú ý c3 2
.c
− 10c + 9 = ( c − 1) ( c + c − 9 ) ≤ 0, ∀c ∈ [1;2]
ok
c3 2

Do đó f (1) ≤ 0 . Tương tự ta có f (2) ≤ 0 .


bo

Bài toán được chứng minh. Đẳng thức xảy ra khi và chỉ khi a= 2, b= c= 1 và
các hoán vị.
et

Bài tập tương tự


Chứng minh a,b,c là các số thực thuộc đoạn [1;2] ta có
vi

( )
2 a3 + b3 + c3 ≤ ( a + b + c )( ab + bc + ca ) .
ng

Bài 13. Bất đẳng thức cần chứng minh tương đương với:
a b c b c a
+ + + + + ≤7.
a

b c a a b c
kh

Không mất tính tổng quát giả sử a ≥ b ≥ c . Khi đó


a a b c c b
( a − b )( b − c ) ≥ 0 ⇔ ab + bc ≥ ac + b2 ⇔ + 1 ≥ + và + 1 ≥ + .
c b c a b a
a b c b c a a c
Do đó + + + + + ≤ 2 + 2  +  .
b c a a b c c a
a c 5 a  a 
Vậy ta chứng minh + ≤ ⇔  − 2  − 1 ≤ 0 luôn đúng do 1 ≤ c ≤ a ≤ 2 .
c a 2 c  c 
Bất đẳng thức được chứng minh. Đẳng thức xảy ra khi a= b= 2, c= 1 hoặc
a= 2, b= c= 1 và các hoán vị.

532
Cty TNHH MTV DVVH Khang Việt

Tổng quát. Cho các số thực x1 , x2 ,..., xn ∈ [ p; q ] , ( p, q ≥ 0 ) . Chứng minh


1 1 1  ( p − q) . 2
( x1 + x2 + ... + xn )  + + ... +  ≤ n2 + kn
 x1 x2 xn  4 pq

trong đó kn = n 2 nếu n chẵn và k=


n n 2 − 1 nếu n lẻ.
3 2 3
Bài 16. Từ điều kiện ta có: 3c = 12 − a − 2b khi đó P = + + .
a b 12 − a − 2b
3 2 3
Xét hàm số f (a) = + + trên đoạn [1;3] ta có:
a b 12 − a − 2b
3 3 12 ( 6 − b )( a + b − 6 )
≤ 0, ∀a, b ∈ [1;3] .

n
f '(a) =− 2+ =
a (12 − a − 2b )2 a 2 (12 − a − 2b )2

.v
Do đó f(a) là hàm nghịch biến trên đoạn [1;3] suy ra

om
2 3 2 3
g (b) =1 + + = f (3) =P = f (a ) ≤ f (1) =h(b) =3 + + .
b 9 − 2b b 11 − 2b
1 2 3 .c
Bài 19. Đặt= x = ,y = ,z ta có 2 x + 4 y + 7 z ≤ 2 xyz và ta cần tìm giá trị nhỏ
a b c
nhất của biểu thức P = x + y + z . Thực hiện tương tự bài toán trên ta có giá trị
ok
1 4 3
nhỏ nhất của P bằng 15/2 đạt tại= a = ,b = ,c .
3 5 2
bo

100
3a + 57b −
100
Bài 22. Theo giả thiết ta có: c ( 3ab − 7 ) = 3a + 57b − ⇒c= a .
et

a 3ab − 7
100
3a + 57b −
vi

a = a + b + 3a + 57 ab − 100
2
Khi đó P = a + b +
3ab − 7 a ( 3ab − 7 )
ng

57
3a 2 + ( 3ab − 7 ) + 33 3a 2 + 33 57
= a+b+ 3 = a+b+ +
a

a ( 3ab − 7 ) a ( 3ab − 7 ) 3a
kh

1 3a 2 + 33 64
=a + ( 3ab − 7 ) + +
3a a ( 3ab − 7 ) 3a

64 1 3a 2 + 33 64 2 a 2 + 11 35
≥a+ +2 ( 3ab − 7 ) . =a + + ≥
3a 3a a ( 3ab − 7 ) 3a a 3
Việc xuất hiện 3ab − 7 ta suy nghĩ đến việc khử cái đó khỏi biểu thức của P
bằng cách tách nhóm hợp lý và triệu tiêu bằng AM-GM.
Chú ý. Khảo sát hàm số f(a) sau cùng ta có ngay kết quả bài toán.
5 35 35
Với= a 5,=b thì P bằng . Vậy giá trị nhỏ nhất của P bằng .
3 3 3
533
Khám phá tư duy Kỹ thuật giải bất ĐT Bài toán Max – Min – Đặng Thành Nam

CHỦ ĐỀ 7: KỸ THUẬT SỬ DỤNG TÍNH CHẤT CỦA


NHỊ THỨC BẬC NHẤT VÀ TAM THỨC BẬC HAI
A. NỘI DUNG PHƯƠNG PHÁP
Ta mở đầu kỹ thuật này bằng các định lý sau đây
1. Định lý 1. Nếu f(x) là hàm bậc nhất thoả mãn điều kiện f (a ) ≥ 0, f (b) ≥ 0 khi
đó f ( x) ≥ 0, ∀x ∈ [ a; b ] .
2. Định lý 2. Nếu f(x) là hàm bậc nhất khi đó
min { f (a ); f (b)} ≤ f ( x) ≤ max { f (a ); f (b)} , ∀x ∈ [ a; b ] .

n
3. Định lý 3. Nếu f ( x) = ax 2 + bx + c, ( a ≠ 0 ) khi đó với mọi x ∈ [α ; β ] .

.v
b
Ta có f(x) đạt Max, Min tại x = α hoặc x = β hoặc x = − .

om
2a
B. BÀI TẬP MẪU
Bài 1. Cho x,y,z là các số thực thuộc đoạn [ 0;2] . .c
Chứng minh rằng 2 ( x + y + z ) − ( xy + yz + zx ) ≤ 4 .
ok
Lời giải
Phân tích tìm lời giải: Nếu coi y và z là tham số thì bất đẳng thức có dạng nhị
thức bậc nhất của x nên ta đánh giá theo định lý 2.
bo

Viết lại bất đẳng thức đã cho dưới dạng:


( 2 − y − z ) x + 2 ( y + z ) − yz − 4 ≤ 0 .
f ( x) =
et

Do x ∈ [ 0;2] nên f ( x) ≤ max { f (0); f (2)} .


vi

Ta có f (0) = 2 ( y + z ) − yz − 4 = ( y − 2 )( 2 − z ) ≤ 0, ∀y, z ∈ [ 0;2] .


ng

f (2) =2 ( 2 − y − z ) + 2 ( y + z ) − yz − 4 =− yz ≤ 0 .
Suy ra f ( x) ≤ 0 . Bất đẳng thức được chứng minh đẳng thức đạt tại chẳng hạn
a

x= 0, y= z= 2 .
kh

Bài 2. Cho a,b,c,d là các số thực thuộc đoạn [ 0;1] .


Chứng minh rằng (1 − a )(1 − b )(1 − c )(1 − d ) + a + b + c + d − 1 ≥ 0 .
Lời giải
Ta cần chứng minh P = (1 − a )(1 − b )(1 − c )(1 − d ) + a + b + c + d − 1 ≥ 0 .
Rõ ràng biểu thức vế trái là hàm bậc nhất đối với mỗi biến a,b,c,d.
+ Nếu coi là hàm bậc nhất của a thì biểu thức P đạt giá trị nhỏ nhất tại a = 0 hoặc
a =1.
+ Tương tự P đạt giá trị nhỏ nhất tại b, c, d ∈ {0;1} .

534
Cty TNHH MTV DVVH Khang Việt

Nếu một trong 4 số bằng 1 thì P ≥ 0 .


Nếu cả 4 số bằng 0 thì P = 0 .
Vậy P ≥ 0, ∀a, b, c, d ∈ [ 0;1] ta có điều phải chứng minh.
Bài 3. Cho 2015 số thực x1 , x2 ,..., x2015 thuộc đoạn [ −2015;2015] .
Tìm giá trị nhỏ nhất của biểu thức P= x1 x2 + x2 x3 + ... + x2014 x2015 + x2015 x1 .
Lời giải
Tương tự bài toán trên ta có P đạt giá trị nhỏ nhất khi
xi ∈ {−2015;2015} , i =1, 2015 .
Trong 2015 số đã cho luôn tồn tại ít nhất 2 số cùng dấu không mất tính tổng

n
quát giả sử x1 x2 > 0 ⇒ x1 x2 =20152 .

.v
Khi đó

om
P=
20152 + x2 x3 + ... + x2015 x1 ≥ 20152 − 20152 − ... − 20152 =
−2013.20152 .
Với x1 =
x2 =
2015, x3 =
−2015, x4 =
2015,..., x2014 =
2015, x2015 =
−2015 thì
P = −2013.20152 .
.c
Vậy giá trị nhỏ nhất của P bằng −2013.20152 .
ok
Bài 4. Cho a,b,c là các số thực thuộc đoạn [ 0;1] .
bo

a b c
Chứng minh rằng + + + (1 − a )(1 − b )(1 − c ) ≤ 1 .
b + c +1 c + a +1 a + b +1
Lời giải
et

Gọi P là biểu thức vế trái của bất đẳng thức.


vi

Không mất tính tổng quát giả sử


b b c c
=a max {a, b, c} ⇒ ≤ ≤
ng

; .
c + a +1 b + c +1 a + b +1 b + c +1
a+b+c
Suy ra P ≤ + (1 − a )(1 − b )(1 − c ) .
a

b + c +1
kh

Vậy để chứng minh bất đẳng thức trên ta chỉ cần chứng minh
a+b+c
= f (a) + (1 − a )(1 − b )(1 − c ) − 1 ≤ 0 .
b + c +1
Ta có f (a ) ≤ max { f (0); f (1)} .
b+c
Ta có f=
(1) 0; f =
(0) + (1 − b )(1 − c ) − 1
b + c +1

=
( )
bc ( b + c ) − b 2 + c 2 − bc − 1
b + c +1

535
Khám phá tư duy Kỹ thuật giải bất ĐT Bài toán Max – Min – Đặng Thành Nam

(b + c ).
b2 + c2
2
( )
− b 2 + c 2 − bc − 1

b + c +1
(b 2
)
+ c 2 ( b + c − 2 ) − 2 ( bc + 1)
≤0
2 ( b + c + 1)
Suy ra điều phải chứng minh.
Cách 2: Ta có thể chứng minh theo cách khác như sau:
a+b+c
+ (1 − a )(1 − b )(1 − c ) ≤ 1
b + c +1

n
a+b+c 1− a
⇔ (1 − a )(1 − b )(1 − c ) ≤ 1 − =

.v
b + c +1 b + c +1
⇔ (1 − b )(1 − c )( b + c + 1) ≤ 1

om
luôn đúng theo AM-GM.
Bài 5. Cho a,b,c là các số thực không âm thỏa mãn điều kiện a + b + c =
1.
Tìm giá trị lớn nhất của biểu thức P = ab + bc + ca − 2abc . .c
Lời giải
ok
Do P là biểu thức đối xứng với a,b,c nên không mất tính tổng quát ta có thể giả
a+b+c 1
sử c = min {a, b, c} ⇒ c ≤ = ⇒ 1 − 2c > 0 .
bo

3 3
Khi đó viết lại P dưới dạng: P = ab (1 − 2c ) + ( a + b ) c = ab (1 − 2c ) + c (1 − c ) .
Sử dụng bất đẳng thức AM-GM ta có:
et

2
 a + b  1− c 
2   1 − c 2 
vi

0 ≤ ab ≤  =    ⇒ ab ∈ 0;   .
 2   2    2  
ng

Coi ab là ẩn thì P là hàm bậc nhất của ab.


Suy ra
a

  1 − c 2    1− c 
2 
= Max  P(0);P  =   Max c (1 − c ) ; (1 − 2c )   + c (1 − c )  .
kh

MaxP
  2      2  
2
1− c 
(1 2c )   1
1 1 1
Xét hàm số f (c) =−  + c (1 − c ) =
− c3 + c 2 + với c ∈ 0; 
 2  2 4 4  3
ta có:

f '(c)=
1
2
(
−3c 2 + c =
1
2
)  1
c (1 − 3c ) ≥ 0, ∀c ∈ 0;  .
 3
 1
Do đó f(c) là hàm đồng biến trên đoạn 0;  .
 3

536
Cty TNHH MTV DVVH Khang Việt

1 7
Suy ra Max = f (c ) f=
  .
 1
c∈ 0;  3  27
 3 

1
Đẳng thức xảy ra khi và chỉ khi a= b= c= .
3
2c. (1 − 2c ) 1  2c + 1 − 2c 2 1 7
Xét Q =c (1 − 2c ) = ≤   =< .
2 2 2  8 27
 1− c 
2  1 7
Vậy Max c (1 − c ) ; (1 − 2c )   + c (1 − c ) = f  =  .
  2    3  27

n
7

.v
Vì vậy giá trị lớn nhất của P bằng đẳng thức xảy ra khi và chỉ khi
27

om
1
a= b= c= .
3
Nhận xét. Vì 1 − 2c > 0 nên ta có thể đánh giá trực tiếp:

P≤
a+b
2

 (1 − 2c ) + c (1=
1− c 
− c) 
2
.c
 (1 − 2c ) + c (1 − c ) .
 2   2 
ok
Bài 6. Cho x, y, z là các số thực không âm có tổng bằng 3.
bo

Chứng minh rằng x 2 + y 2 + z 2 + xyz ≥ 4 .


Lời giải
et

1
Không mất tính tổng quát ta giả sử x = min { x, y, z} ⇒ 3x ≤ x + y + z = 3 ⇒ x ≤ .
3
vi

Đặt biểu thức vế trái bất đẳng thức là P, khi đó ta có:


P − 4 = x 2 + ( y + z ) − 2 yz + xyz − 4 = ( x − 2 ) yz + x 2 + ( 3 − x )2 − 4
2
ng

= f (t ) = ( x − 2 ) t + 2 x 2 − 6 x + 5
a

2 2
 y + z  3− x 
Với 0 ≤ t = yz ≤   = 
kh


 2   2  .
  3 − x 2 
Vậy ta tìm giá trị nhỏ nhất của f (t ) trên 0;    , ta có f (t ) là hàm số
  2  
nghịch biến do x − 2 < 0 .
  3 − x 2  1
Vậy P − 4= f (t ) ≥ f  
  2   4

= ( x − 1)2 ( x + 2 ) ≥ 0 ⇒ P ≥ 4.
 
Từ đó ta có điều phải chứng minh. Đẳng thức xảy ra khi và chỉ khi x= y= z= 1 .

537
Khám phá tư duy Kỹ thuật giải bất ĐT Bài toán Max – Min – Đặng Thành Nam

Bài 7. Cho a, b, c là các số thực dương thỏa mãn điều kiện a + b + c =


1.

( ) (
Chứng minh rằng 5 a 2 + b 2 + c 2 ≤ 6 a3 + b3 + c3 + 1 . )
Lời giải
1
giả sử a min {a, b, c} ⇒ a ≤
Không mất tính tổng =
3
Bất đẳng thức đã cho tương đương với:
5  a 2 + ( b + c ) − 2bc  ≤ 6  a3 + ( b + c ) − 3bc ( b + c )  + 1
2 3
   
⇔ 5  a 2 + (1 − a ) − 2bc  ≤ 6  a3 + (1 − a ) − 3bc (1 − a )  + 1
2 3
   

n
⇔ ( 9a − 4 ) bc + ( 2a − 1) ≥ 0

.v
2

om
2 2
 b + c  1− a 
Ta đặt t = bc ⇒ 0 < t ≤   =  
 2   2 
 1− a  
2
( 9a − 4 ) t + ( 2a − 1)2 ≥ 0, ∀t ∈ 0; 
Vậy ta chỉ cần chứng minh: f (t=
)
.c  .
  2  
ok
  1 − a 2  1
a ( 3a − 1) ≥ 0.
2
Do f (t ) là hàm nghịch biến nên f (t ) ≥ f   = 
  2   4
 
bo

1
Ta có đpcm. Đẳng thức xảy ra khi và chỉ khi a= b= c= .
3
et

C. BÀI TẬP RÈN LUYỆN


Bài 1. Cho a,b,c là các số thực thuộc đoạn [ 0;1] .
vi

Chứng minh rằng a + b + c − ab − bc − ca ≤ 0 .


ng

Bài 2. Cho là các số thực không âm thỏa mãn điều kiện a + b + c =


1.
1
a) Chứng minh rằng a3 + b3 + c3 + 6abc ≥ .
a

4
kh

b) 7 ( ab + bc + ca ) ≤ 2 + 9abc .
 1
Bài 3. Cho a,b,c là các số thực thuộc đoạn 0;  và a + b + c =
1.
 2
9
Chứng minh rằng a3 + b3 + c3 + 4abc ≤ .
32
Bài 4. Cho a,b,c là các số thực thuộc đoạn [1;2]. Chứng minh a3 + b3 + c3 ≤ 5abc .
Bài 5. Cho các số thực không âm x,y,z có tổng bằng 1.

538
Cty TNHH MTV DVVH Khang Việt

7
Chứng minh rằng 0 ≤ xy + yz + zx − 2 xyz ≤ .
27
D. HƯỚNG DẪN GIẢI – ĐÁP SỐ
1
giả sử a max {a, b, c} ⇒ a ≥
Bài 2. a) Không mất tính tổng quát = .
3
Bất đẳng thức tương đương với:
1
a3 + ( b + c ) − 3 ( b + c ) bc + 6abc ≥
3
4
1
⇔ a3 + (1 − a ) − 3 (1 − a ) bc + 6abc −
3
≥ 0.
4

n
⇔ 4 ( 3a − 1) bc + ( 2a − 1) ≥ 0
2

.v
1
Bất đẳng thức cuối đúng. Đẳng thức xảy ra khi a= b= , c= 0 hoặc các hoán vị.

om
2
giả sử a min {a, b, c} ⇒ 7 − 9a > 0 .
c) Không mất tính tổng quát =
Bất đẳng thức đã cho tương đương với: .c
( 7 − 9a ) bc + 7a ( b + c ) − 2 ≤ 0
ok
⇔ ( 7 − 9a ) bc + 7 a (1 − a ) − 2 ≤ 0
2
b+c
bo

Vế trái là hàm đồng biến với bc và bc ≤   , do đó ta chỉ cần chứng minh


 2 
2
b+c
( 7 − 9a ) 
et

 − 7a + 7a − 2 ≤ 0
2
 2 
vi

2
1− a 
⇔ ( 7 − 9a )   − 7a + 7a − 2 ≤ 0
2
 2 
ng

( a + 1)( 3b − 1)2
⇔ ≥0
a

4
kh

1
Bất đẳng thức cuối đúng. Đẳng thức xảy ra khi và chỉ khi a= b= c= .
3
1
Bài 5. HD: Giả sử x = min { x, y, z} ⇒ 3 x ≤ x + y + z =1 ⇒ x ≤ .
3
Khi đó ta có: P = xy + yz + zx − 2 xyz = yz (1 − 2 x) + xy + zx ≥ 0
Đẳng thức xảy ra khi và chỉ khi x= 1, y= z= 0 .
Vậy giá trị nhỏ nhất của P bằng 0 đạt tại x= 1, y= z= 0 hoặc các hoán vị.
2
1− x 
Mặt khác ta lại có P= yz (1 − 2 x) + x( y + z ) ≤ x (1 − x ) +   (1 − 2 x )= f ( x)
 2 
539
Khám phá tư duy Kỹ thuật giải bất ĐT Bài toán Max – Min – Đặng Thành Nam

 1
Ta tìm giá trị lớn nhất của f ( x) trên đoạn 0;  .
 3
3 1   1
Ta có f '( x=
) x  − x  ≥ 0 , do đó f ( x) đồng biến trênđoạn 0; 3  .
2 3   
1 7
=
Vậy max P max=
f ( x) f=
  .
 3  27
1
Đẳng thức xảy ra khi và chỉ khi x= y= z= .
3

n
.v
CHỦ ĐỀ 8: BẤT ĐẲNG THỨC PHỤ ĐÁNG CHÚ Ý
VÀ ÁP DỤNG GIẢI ĐỀ THI

om
Nội dung chủ đề này tôi đề cập đến một số bài toán cơ bản và kết hợp sử dụng
bất đẳng thức cơ bản như AM – GM và Cauchy – Schwarz trong chứng minh bất
.c
đẳng thức và tìm cực trị. Hy vọng chủ đề này sẽ giúp ích cho bạn đọc nhìn nhận
ok
tiếp cận bài toán bất đẳng thức và cực trị linh hoạt và bao quát hơn.
Bài toán 1. Với mọi số thực a,b thỏa mãn điều kiện ab ≥ 1 ta luôn có
bo

1 1 2
+ ≥ .
1+ a 2
1+ b 2 1 + ab
Đẳng thức xảy ra khi và chỉ khi a = b hoặc ab = 1 .
et

Với ab ≤ 1 thì bất đẳng thức đổi chiều.


vi

Chứng minh.
Nhận xét. Bằng cách chưng minh tương tự trên ta có các bất đẳng thức cùng dạng
ng

như sau
Với mọi số thực dương a,b thay đổi thỏa mãn ab ≥ 1 ta luôn có
a

1 1 2
+ ≥ .
kh

1 + a2 1 + b2 1 + ab
Đẳng thức xảy ra khi và chỉ khi a = b hoặc ab = 1 .
Với ab ≤ 1 bất đẳng thức đổi chiều.
Một số bất đẳng thức phụ khác cùng dạng
1 1 2
 + ≤ , ( −1 < xy ≤ 1) .
1+ x 2
1+ y 2 1 + xy
1 1 2
 + ≥ , ( xy ≥ 1) .
1 + x2 1 + y2 1 + xy

540
Cty TNHH MTV DVVH Khang Việt

1 1 2
 + ≤ , ( −1 < xy ≤ 1) .
1 + x2 1 + y2 1 + xy
1 1 2
 Cho hai số thực dương x,y ta luôn có + ≥ .
x + xy
2
y + xy
2
x + y2
2

Đẳng thức xảy ra khi và chỉ khi x = y .


Chứng minh.
Sử dụng bất đẳng thức AM-GM ta có
1 1 2
+ ≥
x + xy y + xy ( )( )
2 2

n
x 2 + xy y 2 + xy

.v
2 2 2
≥ = ≥
( x + y )2

om
x 2 + xy + y 2 + xy x2 + y 2
2 2
1 1 1 n
 + + ... + ≥ , xk ≥ 1, k =
.c 1, n .
1 + x1 1 + x2 1 + xn 1 + n x1 x2 ...xn
ok
Từ bất đẳng thức này ta có thể đưa ra các bất đẳng thức cùng dạng chẳng hạn
4
Chứng minh rằng với mọi số thực x,y thoả mãn điều kiện xy ≥ ta có
bo

5
1 1 2
+ ≥
4 + 5x 2
4 + 5y 2 4 + 5 xy
et

1 1 2
+ ≥
vi

4 + 5x2 4 + 5 y2 4 + 5 xy
ng

a b 2
Với mọi số dương a,b ta có + ≥ .
b + ab a + ab 1 + ab
a

Chứng minh.
kh

Sử dụng bất đẳng thức C –S ta có


a
+
b

( a + b )= (a +=b)
2 2
1
b + ab a + ab ab + a b + ab + b a ab ( a + b ) + 2ab
2 2 ab
+
2ab
a + b ( a + b )2
.
1 2
≥ =
ab 1 1 + ab
+
2 ab 2
Dấu bằng xảy ra khi và chỉ khi a = b .

541
Khám phá tư duy Kỹ thuật giải bất ĐT Bài toán Max – Min – Đặng Thành Nam

 CÁC VÍ DỤ
Ví dụ 1. Cho x,y,z là các số thực dương thoả mãn điều kiện y 2 ≥ xz; z 2 ≥ xy .
x y z
Tìm giá trị nhỏ nhất của biểu thức P = + + 2014. .
x+ y y+z z+x
Lời giải
1 1 2
Chú ý z / x ≥ 1 khi đó sử dụng bất đẳng thức + ≥ , ( ab ≥ 1)
1+ a 2
1+ b 2 1 + ab
x y 1 1 2
Ta có + = + ≥ .
x + y y + z 1+ y 1+ z z
1+

n
x y x

.v
z
2 2 t2
Do đó P ≥ + 2014. x = + 2014. 2

om
+1 1+ t t +1
z z
1+
x x

=
(
2 ( t − 1) 503t 2 + 1007t + 503 ) z
+ 1008 ≥ 1008, t = ≥ 1
.c
( t + 1) t + 1
2
( ) x
ok
Vậy giá trị nhỏ nhất của P bằng 1008 đạt tại x= y= z .
Ví dụ 2. Cho x,y,z là các số thực dương thỏa mãn điều kiện x ≤ z .
bo

2 x2 2z ( 2 y + z ) 3z
Tìm giá trị nhỏ nhất của biểu thức: P =2+ − + .
( x + y) 2
( y + z) 2 z+x
et

Lời giải
vi

Biến đổi biểu thức trong căn và sử dụng bất đẳng thức C –S ta có
2 x2 2 z (2 y + z ) 2 x2 2 y2
ng

2+ − = +
( x + y )2 ( y + z )2 ( x + y )2 ( y + z )2
 
a

2
   
   1 1 
kh

1 1
= 2 + 2
≥ + 

 1+ y 
2
 z    1 +
y
1 +
z
  x
 1 +    x y 
 y 
y z z
Mặt khác do: . = ≥ 1.
x y x
2 2
   
 1 1   2 
Nên áp dụng bài toán 1 ta có  +  ≥ .
1+ y 1+ z   z 
 1+
 x y   x

542
Cty TNHH MTV DVVH Khang Việt

2 x2 2 z (2 y + z ) 2
Từ đó ta suy ra 2+ − ≥ .
( x + y) 2
( y + z) 2
z
1+
x
z 2 3t 2
Đặt=t ≥ 1 . Khi đó ta suy ra P ≥ + 2 .
x 1+ t t +1
2 3t 2 −2 6t
Xét hàm f=(t ) + 2 trên [1; + ∝) ta có =f '(t ) + 2 2 > 0 ∀t ∈ [1; + ∞)
1+ t t +1 (1 + t ) (t + 1)
2

5
Do đó Min P = f (1) =
2

n
Bài tập tương tự

.v
Cho x,y,z là các số thực dương thoả mãn điều kiện x = min { x, y, z} .

om
x y 3z
Tìm giá trị nhỏ nhất của biểu thức P = + + .
x+ y y+z z+x
1
Ví dụ 3. Cho các số thực ≤ x ≤ 1; y, z ≥ 1 thỏa mãn điều kiện xyz = 1 .
.c
4
1 1 1
Tìm giá trị nhỏ nhất của biểu thức P = + +
ok
.
1+ x 1+ y 1+ z
Lời giải
bo

1 1 2 1 2 1 2
Với yz ≥ 1 ta có + ≥ ⇒P≥ + = +
1 + y 1 + z 1 + yz 1 + x 1 + yz 1 + 1 1 + yz
et

yz
1 t2 2
= yz ⇒ 1 ≤ = ≤ 2 . Suy ra P ≥ f (t ) = 2 +
vi

t t .
x t +1 1+ t

( )
ng

2
2t ( t + 1) − 2 t 2 + 1
2
2t 2
Ta có =
: f '(t ) − = ≤ 0, ∀t ∈ [1;2] .
( ) (1 + t )2 ( )
2 2
t 2 + 1 ( t + 1)
2
t2 +1
a
kh

22
f (t ) ≥ f (2) =
15
22 1
Vậy giá trị nhỏ nhất của P bằng đạt tại x= ; y= z= 2 .
15 4
Ví dụ 4. Cho a,b,c là các số thực dương.
2a 2b 2c
Chứng minh rằng + + ≤ 3.
a+b b+c c+a
Lời giải
Nhận xét. Ta đã chứng minh bất đẳng thức trên bằng bất đẳng thức C –S dưới đây
tiếp cận theo phương pháp hàm số.

543
Khám phá tư duy Kỹ thuật giải bất ĐT Bài toán Max – Min – Đặng Thành Nam

1 1 1 3
Ta cần chứng minh: + + ≤ .
b c a 2
1+ 1+ 1+
a b c
b c a
Đặt x = ,y= ,z = ⇒ xyz = 1 .
a b c
1 1 1 3
Bất đẳng thức trở thành: + + ≤ .
1+ x 2
1+ y 2
1+ z 2 2
1
Không mất tính tổng quát giả sử z = max { x, y, z} ⇒ z ≥ 1 ⇒ xy = ≤1.
z

n
Khi đó sử dụng bất đẳng thức C-S và bài toán phụ ta được:

.v
1 1  1 1  4 2 z
+ ≤ 2  +  ≤ =.

om
1 + x2 1 + y2 1+ x
2
1+ y 
2 1 + xy z +1

2 z 1 3
Chứng minh hoàn tất nếu ta chứng minh được: + ≤ .
.c z +1 1 + z2 2
2 z 1 2 z 2 3
ok
Thật vậy + ≤ + ≤ (vì bất đẳng thức tương đương
z +1 1+ z 2 z +1 1+ z 2

( )
bo

2
với 2z − z + 1 ≥ 0 ).
Bài toán được chứng minh.
et

Ví dụ 5. Cho các số thực dương a, b thỏa mãn điều kiện a + b =2ab .


1 3 3 2 2 1
vi

Tìm giá trị nhỏ nhất của biểu thức:=


Q +3 +
a +b +a+b+4 .
a +1 b +1
2
4 2
ng

Lời giải
Từ giả thiết ta có: 2ab = a + b ≥ 2 ab ⇒ ab ≥ 1 ⇒ ab ≥ 1.
a

1 2 1
Với ab ≥ 1 ta có BĐT: + . ≥
kh

a + 1 b + 1 1 + ab
2 2

Mặt khác ta có:


3 3 2 3 3 2
3
a + b 2 + a + b=
+4 a + b 2 + 2ab + 4
3
4 4
3
= 3 3 (a + b) + 4
2
4
3
≥ 3 3 4ab +=4 3 3 ab + 1
4

544
Cty TNHH MTV DVVH Khang Việt

2 3 23 2  2 − 23 2
Do đó : Q ≥ + 33 ab +=
 ab + 1 + ab + 1 + ab + 1 +
13 3
 +
ab + 1  ab + 1  ab + 1
Áp dụng BĐT AM - GM ta có:
23 2
3
ab + 1 + 3 ab + 1 + 3 ab + 1 + ≥ 44 23 2 =43 2
ab + 1
Từ giả thiết ta có:
1 1
2ab = a + b ≥ 2 ab ⇒ ab ≥ 1 ⇒ ab ≥ 1 ⇒ ab + 1 ≥ 2 ⇒ ≤
ab + 1 2
2 − 23 2 2 − 23 2
Mà 2 − 2 3 2 < 0 ⇒ ≥ =1 − 3 2

n
ab + 1 2

.v
Do đó Q ≥ 4 3 2 + 1 − 3 2 = 1 + 3 3 2 . Đẳng thức xảy ra khi a = b = 1.

om
Vậy giá trị nhỏ nhất của Q là bằng 1 + 3 3 2 đạt được khi a = b = 1.
Ví dụ 6. Cho x,y,z là các số thực dương thỏa mãn điều kiện
4 4
 x  y z x .c
  +   + = + 2.
 y  z x z
ok
2 y2 2z2 3z
Tìm giá trị lớn nhất của biểu thức P = + − .
x +y
2 2
y +z2 2 2x + z
bo

Lời giải
4 4 4 4
x  x  y z  x  y z x2 z
et

Theo giả thiết ta có: +=


2   +   + ≥ 2   .  + = 2 2 + .
z  y  z x  y  z x z x
vi

x 1
Đặt=
t , ( t > 0 ) ⇒ t + 2 ≥ 2t 2 + ⇔ 2t 3 − t 2 − 2t + 1 ≤ 0
z t
ng

( ) 1
⇔ ( t − 1) 2t 2 + t − 1 ≤ 0 ⇔ ( t − 1)( t + 1)( 2t − 1) ≤ 0 ⇔ ≤ t ≤ 1 .
2
a

x y x 1 1 2 2
Khi đó . = = t ≤1⇒ + ≤ =
kh

.
y z z x
2
 y
2 x y 1+ t
1 + .
  +1  z  +1 y z
 y  
y2 z2 1 1 2 2 2
Ta có: + = 2 + ≤
= = .
x +y
2 2
y +z
2
x
2

2
y x
1+ .
y
1+
x 1+ t
  +1   +1 y z z
 y  z
4 3
Suy ra P ≤ f (t ) =− .
t 2t + 1

545
Khám phá tư duy Kỹ thuật giải bất ĐT Bài toán Max – Min – Đặng Thành Nam

4 3 1 
Xét hàm số f (t )= − liên tục trên đoạn  ;1 ta có:
t 2t + 1 2 

f '(t ) =
6

4
=−
(
2 5t 2 + 2t − 1 ) 1 
≤ 0, ∀t ∈  ;1 .
( 2t + 1) 2
( t + 1) 2
( t + 1) ( 2t + 1)
2 2
2 

1  1 7
Do đó f(t) là hàm nghịch biến trên đoạn  ;1 . Suy ra P ≤ f (t ) ≤ f   = .
2  2 6
x y x 1
Đẳng thức xảy ra khi và chỉ khi = , = ⇔ z = 2 x, y = 2 x .
y z z 2

n
7
Vậy giá trị lớn nhất của P bằng đạt tại
= z 2=
x, y 2x .

.v
6
Ví dụ 7. (TSĐH Khối A 2011) Cho các số thực x, y, z ∈ [1;4] thỏa mãn điều kiện

om
x y z
x ≥ y, x ≥ z . Tìm giá trị nhỏ nhất của biểu thức P= + + .
2x + 3y y + z z + x

1
Lời giải
1 1
.c
Viết lại P dưới dạng: P = + +
ok
.
3y z x
2+ 1+ 1+
x y z
bo

z x x
Do x ≥ y ⇒ . = ≥ 1. Áp dụng bất đẳng thức trong bài toán 1, ta được
y z y
et

1 1 2 1 2
+ ≥ ⇒P≥ + .
z x x 3y x
1+ 1+ 2+
vi

1+ 1+
y z y x y
ng

x 1 2 t2 2
Đặt
= t , (1 ≤ t ≤ 2 ) suy ra P ≥ + = + .
2 + 2 1 + t 2t + 3 1 + t
y 3 2
a

t
kh

t2 2
Xét hàm số=
f (t ) + trên [1;+∞ ) , ta được:
2t + 3 1 + t
2

6t 2
=
f '(t ) − < 0, ∀t ∈ [1;2] .
( 2t ) (1 + t )2
2
2
+3

34
Suy ra min f = =
(t ) f(2) .
t∈[ 2;2] 33
34
Vậy giá trị nhỏ nhất của P bằng đạt tại =
x 4,=
y 1,=
z 2.
33

546
Cty TNHH MTV DVVH Khang Việt

Nhận xét. Ngoài ra có thể chứng minh bằng tam thức bậc hai, khảo sát hàm nhiều
biến(chương 1 – chương 3).
Ví dụ 8. Cho a,b,c là các số thực dương thỏa mãn điều kiện abc = 1 và
max {a, b, c} ≤ 4 . Tìm giá trị lớn nhất của biểu thức
1 1 1
P= + + .
1 + a2
1 + b2 1+ c
Lời giải
Nhận xét. Với vai trò của a và b như nhau làm ta nghĩ đến bất đẳng thức quen
1 1 2
thuộc + ≤ .
1 + ab

n
1 + a2 1 + b2

.v
1
Để có được điều này ta cần có ab ≤ 1 . Do đó =
c ≥ 1 . Vậy ta cần chứng
ab

om
minh rằng P đạt giá trị lớn nhất trong trường hợp c = max {a, b, c} .
Thật vậy giả sử a = max {a, b, c} . Khi đó a ≥ 1 và ta cần chứng minh
1
+
1
+
1
1+ c

1
+
1
+
1 .c
1+ a
1 + a2 1 + b2 1 + c2 1 + b2
ok
1 1 1 1
⇔ − ≤ − (1) .
1+ c 1 + c2 1+ a 1 + a2
bo

+ Nếu c < 1 ⇒ VT(1) < 0 < VP(1) bất đẳng thức luôn đúng.
1 1
et

+ Nếu c ≥ 1 khi đó xét hàm số=


f ( x) − trên đoạn [1;4] ta có
1+ x 1 + x2
vi

( )
3
2 x (1 + x ) − 1 + x 2
3
1 x
ng

f '( x) =
− + = >0.
2 (1 + x ) (1 + x ) ( )
3 2 3 3
2 (1 + x ) 1 + x 2
3
a

Do
kh

2 x (1 + x ) ≥ x x . (1 + x ) =
3 3
x3 (1 + x ) =
3
(x + x ) 2 3
≥ (1 + x ) , ∀x ∈ [1;4] .
2 3

Do đó f(x) là hàm đồng biến trên đoạn [1;4] . Nên c ≤ a ⇒ f (c) ≤ f(a) . Ta có
(1) đúng.
Vậy P đạt giá trị lớn nhất khi và chỉ khi c = max {a, b, c} và ta có c ≥ 1, ab ≤ 1 .
Áp dụng bất đẳng thức đã phân tích ở trên ta được
2 1 2 1 2 c +1
P≤ + = + =g (c ) = .
1 + ab 1+ c 1 1+ c 1+ c
1+
c
547
Khám phá tư duy Kỹ thuật giải bất ĐT Bài toán Max – Min – Đặng Thành Nam

2 c +1
Xét hàm số g (c) = liên tục trên đoạn [1;4] ta có
1+ c
2− c
=g '(c) ≥ 0, ∀c ∈ [1;4] .
2 (1 + c ) c 2 + c
Do đó g(c) là hàm đồng biến trên đoạn [1;4] suy ra P ≤ g (c) ≤ g (4) =5.
1
Đẳng thức xảy ra khi và chỉ khi a =b, c =4, abc =1 ⇔ a =b = , c =4 .
2
1 
Ví dụ 9. Cho x,y,z là các số thực thuộc đoạn  ;3 .

n
3 

.v
Tìm giá trị lớn nhất và giá trị nhỏ nhất của biểu thức
x y z
P= + +

om
.
x+ y y+z z+x
Lời giải
Trong chương 1 tôi đã trình bày bằng cách sử dụng tam thức bậc hai dưới đây
.c
chúng ta áp dụng bài toán đã nêu.
+ Tìm giá trị nhỏ nhất.
ok
z
Không mất tính tổng quát = giả sử z max { x, y, z} ⇒ ≥ 1 .
x
bo

Khi đó sử dụng bất đẳng thức phụ đã cho ta có


x y 1 1 2 2
+ = + ≥ = .
et

x + y y + z 1+ y 1+ z y z z
1+ . 1+
x y x y x
vi

2 1 2 t2 z
Khi đó P ≥ + = + 2 ,= ,1 ≤ t ≤ 3 .
ng

t
z x 1+ t t +1 x
1+ 1+
x z
a

2 t2
Xét hàm số f=
(t ) + 2 liên tục trên đoạn [1;3] ta có
kh

1+ t t +1

f '(t ) =−
(
2 ( t − 1) t 2 + t + 1
2
) ≤ 0, ∀t ∈ [1;3] .
( t + 1) ( t )
2 2
2
+1

7
Do đó f(t) nghịch biến trên đoạn [1;3] vì vậy f (t ) ≥ f (3) = .
5
1
Đẳng thức xảy ra khi và chỉ khi z = 9 x, x = y ⇔ x = y = , z = 3 .
3
Vậy giá trị nhỏ nhất của P bằng 7/5.

548
Cty TNHH MTV DVVH Khang Việt

+ Tìm giá trị lớn nhất


y x y x z 2
Ta có . = ≤1⇒ + ≤ .
x z z x+ y z+x y
1+
z
y 2 2 t2 y 1 
Do đó P ≤ + = + 2 , t = ∈  ;1 .
y+z y t +1 t +1 z 3 
1+
z
2 t2 1 8
Xét hàm số tương tự trên ta có P ≤ g (t ) = + 2 ≤ g  = .
t +1 t +1 3 5

n
1
Vậy giá trị lớn nhất của P bằng 8/5 đạt tại x= z= 3, y= .

.v
3
1

om
Ví dụ 10. Cho các số thực a, b, c ≥ thoả mãn điều kiện abc = 1 .
9
a b 1
Tìm giá trị nhỏ nhất của biểu thức P = + + .
b + ab c + ab 1 + c .c
Lời giải
ok
Sử dụng bất đẳng thức C –S ta có
a
+
b

(a + b) 2
bo

b + ab a + ab ab + a 2b + ab + b 2 a

= =
( a + b)
2
1
et

ab ( a + b ) + 2ab ab
+
2ab
a + b ( a + b )2
vi

1 2
≥ =
ng

ab 1 1 + ab
+
2 ab 2
Dấu bằng xảy ra khi và chỉ khi a = b .
a
kh

2 1 2 ab
Khi đó P ≥ + = + .
1 + ab 1 + c 1 + ab ab + 1
1 1 1 2 t2
Đặt t = ab . Do a, b, c ≥ ⇒ ≤ t= ab= ≤ 3; P ≥ + 2 .
9 9 c 1+ t t +1
2 t2
Xét hàm số f=
(t ) + 2 liên tục trên đoạn [1/9;3] ta có
1+ t t +1

f '(t ) =−
(
2 ( t − 1) t 2 + t + 1
2
) ≤ 0, ∀t ∈ [1;3] .
( t + 1) ( t )
2 2
2
+1

549
Khám phá tư duy Kỹ thuật giải bất ĐT Bài toán Max – Min – Đặng Thành Nam

7
Do đó f(t) nghịch biến trên đoạn [1/9;3] vì vậy f (t ) ≥ f (3) = .
5
1
Đẳng thức xảy ra khi và chỉ khi a= b= 3, c= .
9
Vậy giá trị nhỏ nhất của P bằng 7/5.

 BÀI TẬP RÈN LUYỆN


Bài 1. Cho x,y,z là các số thực dương thỏa mãn điều kiện x ≥ z .
Tìm giá trị lớn nhất của biểu thức
x y z

n
P= + + .
x +y
2 2
y +z2 2 x+z

.v
1 1 z 2 z/x

om
HD: P = + + ≤ + ≤ 5.
2 2 x+z 1+ z / x 1+ z / x
 y z
1+  1+  
x  y .c
Bài 2. Cho các số thực không âm thay đổi thỏa mãn điều kiện a ≤ b ≤ c và abc = 1 .
ok
1 1 2
Tìm giá trị lớn nhất của biểu thức P = + + .
1+ a 2
1+ b 1+ c
2

2 2
bo

HD: P ≤ + =
2.
1 + ab 1 + c
et

Bài 3. Cho x,y,z là các số thực dương thỏa mãn điều kiện xy ≥ 1, z ≥ 1 .
z3 + 2
vi

x y
Tìm giá trị nhỏ nhất của biểu thức P = + + .
y + 1 x + 1 3 ( xy + 1)
ng

HD: P = ( x + y + 1) 
 1
+
1  z3 + 2 2 2 xy + 1 (1 3 )
+ −2≥ + −2≥ .
 x + 1 y + 1  3 ( xy + 1) xy + 1
a

1 + xy 2
kh

Bài 4. Cho x,y,z là các số thực dương thỏa mãn điều kiện x 2 + y 2 + z 2 =
1.
1 1 2 3
Tìm giá trị nhỏ nhất của biểu thức P = + + .
x + xy
2
y + xy
2 1+ z

1 1 2
HD: Sử dụng bất đẳng thức: + ≥ .
x 2 + xy y 2 + xy x2 + y 2
2 3 2 8
Suy ra P ≥ + ≥ .
1+ z 1 − z2 3

550
Cty TNHH MTV DVVH Khang Việt

Bài 5. Cho a,b,c là các số thực dương không nhỏ hơn 1.


Tìm giá trị nhỏ nhất của biểu thức
1 2 3
P= + + + 6 1 + abc ( abc − 1) .
1 + a 1 + b 1 + c2
6 3

HD: Sử dụng bất đẳng thức phụ sau:


1 1 2 1 1 1 3
+ ≥ , ∀ab ≥ 1 và + + ≥ , ∀a, b, c ≥ 1
1+ a 2
1+ b 2 1 + ab 1 + a 1 + b 1 + c 1 + abc
3 3 3

 1 1  4 1 1 2
Ta có 2  + 2 
≥ ; + ≥ .
1+ b 1+ c  1+ c b 1+ a 1+ c
3 3 6 2
1 + a 3c

n
Suy ra

.v
 1 1 1  6
P ≥ 2 + +  + 6 1 + abc ( abc − 1) ≥ + 6 1 + abc ( abc − 1) .
 1 + ca 3  1 + abc
 1 + c b3 1 + c b3 

om
6
Đặt
= t abc, ( t ≥ 1) ta có: P ≥ f (t ) = + 6 1 + t ( t − 1) .
t +1

Xét hàm số f (t ) =
6
.c
+ 6 1 + t ( t − 1) với t ≥ 1 ta có
t +1
ok
3 ( 2t − 1) 3 ( 2t − 1)( t + 1) − 6 t 2 − t + 1
2
6
f '(t ) =− + = > 0, ∀t ≥ 1 .
( t + 1)2 ( t + 1) t − t + 1
bo

2
t − t +1
2 2

Do đó f(t) là hàm đồng biến trên [1;+∞ ) .


et

9 . Đẳng thức xảy ra khi và chỉ khi a= b= c= 1 .


Suy ra P ≥ f (t ) ≥ f (1) =
Vậy giá trị nhỏ nhất của P bằng 9 đạt tại a= b= c= 1 .
vi

Bài 6. Cho a,b,c là các số thực không âm thỏa mãn điều kiện a 2 + b 2 + c 2 =
1.
ng

1 1 1
Tìm giá trị lớn nhất của biểu thức P = + + .
a +1 b +1 c+2
a

HD:
kh

2
 1 1  1 1 2 a+b+2 2
 +  = a +1 + b +1 + = +
 a +1 b +1  (1 + a )(1 + b ) ab + a + b + 1 ab + a + b + 1
2
a+b+2 2  1 
≤ + = 1+ 
a + b +1 a + b +1  a + b +1 
1 1 1
Suy ra + ≤1+ .
a +1 b +1 a + b +1
Mặt khác: ( a + b + c ) ≥ a 2 + b 2 + c 2 =⇒
2
1 a + b ≥1− c .

551
Khám phá tư duy Kỹ thuật giải bất ĐT Bài toán Max – Min – Đặng Thành Nam

1 1
Suy ra P ≤ f (c) =
1+ + .
2−c c+2
1 1
Xét hàm số f (c) =
1+ + liên tục trên đoạn [ 0;1] ta có:
2−c c+2
 
1 1 1  ≥ 0, ∀c ∈ [ 0;1]. Do đó f(c) là hàm đồng biến
=f '(c) −
2 3
 ( 2 − c )3 ( 2 + c ) 
trên [ 0;1] .
1
Suy ra P ≤ f (c) ≤ f (1) =
2+

n
.
3

.v
Đẳng thức xảy ra khi và chỉ khi c= 1, a= b= 0 .

om
1
Vậy giá trị lớn nhất của P bằng 2 + đạt tại a= b= 0, c= 1 .
3
1 1
Nhận xét. Ta có thể đánh giá + bằng biến đổi tương đương như sau:
.c
2−c c+2
ok
2
1 1  1 1  1 1 2
+ =  +  = + +
2−c c+2  2−c c+2 2−c 2+c 4 − c2
bo

4 2 4 2 1
= 2+ ≤ + =
1+
4−c 4 − c2 4 −1 4 −1 3
et

Bài 7. Cho a,b,c,d là các số thực dương thỏa mãn điều kiện a ≥ b ≥ c ≥ d và
abcd = 1 .
vi

1 1 1 3
Tìm giá trị nhỏ nhất của biểu thứ P = + + +
ng

.
1+ a 1+ b 1+ c 1+ d
HD: Do a ≥ b ≥ c ≥ d và abcd = 1 nên ab ≥ 1 ≥ cd ≥ d 2 .
a

1 1 2 2 2 2d
Với ab ≥ 1 ta có: + ≥ = ≥ = .
kh

1 + a 1 + b 1 + ab 1 + 1 1+
1 1+ d
cd d
1 1 1 1 1 1 c
Vì cd ≤ 1 ⇒ d ≤ ⇒ + ≥ + = + = 1.
c 1+ c 1+ d 1+ c 1+ 1 1+ c 1+ c
c
2d 2
Mặt khác: + =2.
1+ d 1+ d

552
Cty TNHH MTV DVVH Khang Việt

 1 1   1 1  2 2d 2
Suy ra P =  + + + + ≥ +1+ = 3.
1+ a 1+ b  1+ c 1+ d  1+ d 1+ d 1+ d
Vậy giá trị nhỏ nhất của P bằng 3 đạt tại a= b= c= d= 1 .
Bài toán 2. Cho a,b,c là các số thực không âm thoả mãn điều kiện
k , trong đó k ≥ 1 .
a 2 + b2 + c2 =
Chứng minh rằng
a) k + 2bc ≥ 2a ( b + c ) .

b) ( a + b + c ) ≤ 2k (1 + bc ) .
2 2

n
2
c) a + b + c − abc ≤ 2k .
k

.v
Chứng minh.

om
a) Theo giả thiết ta có: k + 2bc =a 2 + b 2 + c 2 + 2bc =a 2 + ( b + c ) .
2

Sử dụng bất đẳng thức AM – GM ta có:


a 2 + ( b + c ) ≥ 2a ( b + c ) ⇒ k + 2bc ≥ 2a ( b + c ) .
2 .c
Bất đẳng thức được chứng minh.
ok
k k
Đẳng thức xảy ra khi và chỉ khi a= b= , c= 0 hoặc a= c= , c= 0 .
2 2
bo

b) Ta cần chứng minh: k + 2a ( b + c ) + 2bc ≤ 2k 1 + b 2 c 2 + 2bc ( )


⇔ 2kb 2 c 2 + 2 ( 2k − 1) bc + k ≥ 2a ( b + c )
et

Sử dụng bất đẳng thức đã chứng ở câu a) ta chỉ cần chứng minh
vi

2kb 2 c 2 + 2 ( 2k − 1) bc + k ≥ k + 2bc
ng

⇔ kb 2 c 2 + 4 ( k − 1) bc ≥ 0 ⇔ bc ( kbc + 4 ( k − 1) ) ≥ 0
Bất đẳng thức cuối luôn đúng do k ≥ 1 .
a

a 2k a
Nhận xét. Từ b) ta có một bất đẳng thức rất hay sử dụng là: ≤
kh

.
1 + bc a + b + c
c) Sử dụng bất đẳng thức C –S ta có
  2 2 
 2 
a 1 − bc  + b + c ≤
 k 
(a 2
+ (b + c )
2
)  1 − bc  + 1
 k 


( k + 2bc )  
4 4
= b 2 c 2 − bc + 2 
k 2 k 
Vậy ta chỉ cần chứng minh

553
Khám phá tư duy Kỹ thuật giải bất ĐT Bài toán Max – Min – Đặng Thành Nam

( k + 2bc )  
4 4
b 2 c 2 − bc + 2  ≤ 2k
k 2 k 
8b3c3
4b 2 c 2
⇔ 2
− ≤ 0 ⇔ 4b 2 c 2 ( 2bc − k ) ≤ 0
k k
Bất đẳng thức luôn đúng vì theo AM – GM ta có
2bc ≤ b 2 + c 2 ≤ a 2 + b 2 + c 2 =
k.
Các bất đẳng thức dạng này đã trình bày trong chương 2 (Kỹ thuật sử dụng bất
đẳng thức C-S).

n
 CÁC VÍ DỤ
Ví dụ 1. (TSĐH Khối A,A1/2014) Cho x,y,z là các số thực không âm thỏa mãn

.v
điều kiện x 2 + y 2 + z 2 =
2.

om
x2 y+z 1 + yz
Tìm giá trị lớn nhất của biểu thức P= + − .
x + yz + x + 1
2 x + y + z +1 9
Lời giải
.c
Nhận xét. Với các bất đẳng thức và bài toán cực trị với điều kiện các biến không
ok
âm thông thường dấu bằng xảy ra tại một biến bằng 0 và để ý để P lớn nhất ta thấy
yz = 0 là hợp lý nhất. Vậy ta tìm cách đánh giá P theo đại lượng 1 + yz và
bo

y = 0 hoặc z = 0 .

Ta có 2 (1 + yz ) =x 2 + ( y + z ) ≥ 2 x ( y + z ) .
2
et

Dấu bằng xảy ra khi và chỉ khi x= y + z .


vi

Suy ra x 2 + yz + x + 1 ≥ x 2 + x + x ( y + z )= x ( x + y + z + 1) .
ng

x2 y+z 1 + yz
Do đó P ≤ + −
x ( x + y + z + 1) x + y + z + 1 9
a

x+ y+z 1 + yz  1 1 + yz 
= − =
1−  + 
kh

x + y + z +1 9  x + y + z +1 9 
Theo bất đẳng thức C-S ta có

x + y + z ≤ 2  x 2 + ( y + z ) = 2 1 + yz .
2
 
1 1 + yz
Suy ra P ≤ 1 − − .
2 1 + yz + 1 9
1 t2
Đặt t =1 + yz , ( t ≥ 1) ta có P ≤ f (t ) =
1− − .
2t + 1 9

554
Cty TNHH MTV DVVH Khang Việt

1 t2
Xét hàm số f (t ) =
1− − với t ≥ 1 ta có
2t + 1 9
2t 18 − 2t ( 2t + 1)
2
2
; f '(t) = 0 ⇔ 18 − 2t ( 2t + 1) = 0 ⇔ t = 1 .
2
f '(t ) = − =
( 2t + 1)2 9 9 ( 2t + 1)
2

Ta có f’(t) đổi dấu từ dương sang âm khi đi qua t = 1 .


Do đó f(t) đạt cực đại tại t = 1 .
5
Do đó P ≤ f (t ) ≤ f (1) = . Đẳng thức xảy ra khi và chỉ khi
9
1 + yz = 1

n
 x= y + z

.v
 2 ⇔=
x 1,=
y 1,=
z 0 hoặc=
x 1,=y 0,=z 1.
x + y + z =
2 2
2

om
 x, y , z ≥ 0

5
Vậy giá trị lớn nhất của P bằng đạt tại =x 1,=
y 0,=
z 1 hoặc
9 .c
=
x 1,= y 0,= z 1.
ok
Ví dụ 2. Cho a,b,c là các số thực không âm thoả mãn điều kiện a 2 + b 2 + c 2 =
1.
2a + c 2b + c a + b + c
Tìm giá trị lớn nhất của biểu thức P = + + .
bo

1 + bc 1 + ca 1 + 2abc
Lời giải
et

Ta có
2a + c 2a + c
( a + b + c )2 ≤ 2 (1 + bc )2 ⇒ ≤ 2.
vi

1 + bc a+b+c
2b + c 2b + c
ng

( a + b + c )2 ≤ 2 (1 + ca )2 ⇒ ≤ 2.
1 + ca a+b+c
2a + c 2b + c 2a + c 2b + c
⇒ + ≤ 2. + 2. =
a

2 2
1 + bc 1 + ca a+b+c a+b+c
kh

Theo bất đẳng thức đã chứng minh tại câu c) bài toán phụ ta có:
a + b + c − 2abc ≤ 2 .
Thật vậy sử dụng bất đẳng thức C –S ta có
a + b + c − 2abc= a (1 − 2bc ) + ( b + c )

≤ (a 2
+ (b + c )
2
) ((1 − 2bc ) + 1)
2

= (1 + 2bc ) ( 4b2c 2 − 4bc + 2 )


Vậy ta chỉ cần chứng minh

555
Khám phá tư duy Kỹ thuật giải bất ĐT Bài toán Max – Min – Đặng Thành Nam

(1 + 2bc ) ( 4b2c 2 − 4bc + 2 ) ≤ 2 ⇔ 4b2c 2 ( 2bc − 1) ≤ 0 .


Bất đẳng thức cuối đúng vì theo AM – GM ta có
2bc ≤ b 2 + c 2 ≤ a 2 + b 2 + c 2 =
1.
Bất đẳng thức được chứng minh. Từ đó suy ra P ≤ 3 2 .
1
Dấu bằng đạt tại a= b= , c= 0 .
2
Bài tập tương tự
Cho a,b,c là các số thực không âm thoả mãn điều kiện a 2 + b 2 + c 2 =
2.

n
2a + c 2b + c a + b + c
Chứng minh rằng + + ≤5.
1 + bc 1 + ca abc + 2

.v
Ví dụ 3. Cho x,y,z là các số thực không âm thoả mãn điều kiện x 2 + y 2 + z 2 =
1.

om
x y z
Chứng minh rằng 1 ≤ + + ≤ 2.
1 + yz 1 + zx 1 + xy

Chứng minh bất đẳng thức vế phải


Lời giải
.c
ok
x 2x
Chú ý ( x + y + z ) ≤ 2 (1 + yz ) ⇒
2 2
≤ .
1 + yz x + y + z
bo

y 2y z 2z
Tương tự ta có: ≤ ; ≤ .
1 + zx x + y + z 1 + xy x + y + z
et

Cộng theo vế ba bất đẳng thức trên ta có đpcm.


vi

1
Đẳng thức xảy ra khi và chỉ khi x= y= , z= 0 hoặc các hoán vị.
2
ng

Ví dụ 4. Cho x,y,z là các số thực không âm thoả mãn điều kiện x 2 + y 2 + z 2 =


2.
x2 y2 z2
a

Chứng minh rằng + + ≤ 1.


x 2 + yz + 1 y 2 + zx + 1 z 2 + xy + 1
kh

Lời giải
Ta có 2 + 2 yz = x 2 + ( y + z ) ≥ 2 x ( y + z )
2

⇒ 1 + yz ≥ x ( y + z ) ⇒ x 2 + yz + 1 ≥ x ( x + y + z )
x2 x
⇒ ≤
x + yz + 1
2 x+ y+z
y2 y z2 z
Tương tự ta có: ≤ ; 2 ≤
y 2 + zx + 1 x + y + z z + xy + 1 x + y + z

556
Cty TNHH MTV DVVH Khang Việt

Cộng lại theo vế ba bất đẳng thức trên ta có đpcm.


Đẳng thức xảy ra khi và chỉ khi x= y= 1, z= 0 hoặc các hoán vị.
Ví dụ 5. Cho x,y,z là các số thực không âm thỏa mãn điều kiện x 2 + y 2 + z 2 =
2.
Tìm giá trị lớn nhất của biểu thức
x2 y+z 1
M= + + .
x + yz + x + 1
2 x + y + z + 1 xyz + 3
Lời giải
Sử dụng bất đẳng thức AM – GM ta có:
2 + 2 yz =x 2 + y 2 + z 2 + 2 yz =x 2 + ( y + z ) ≥ 2 x ( y + z )
2

n
⇒ 1 + yz ≥ x ( y + z ) ⇒ x 2 + x + yz + 1 ≥ x ( x + y + z + 1)

.v
x2 x
Do đó ≤

om
.
x + x + yz + 1
2 x + y + z +1
Ta chứng minh x + y + z − xyz ≤ 2 .
Thật vậy sử dụng bất đẳng thức C –S ta có .c
x + y + z − xyz = x (1 − yz ) + ( y + z ) ≤ (x 2
+ ( y + z)
2
) ((1 − yz ) + 1)
2
ok
= (
2 (1 + yz ) y 2 z 2 − 2 yz + 2= ) y 2 z 2 ( yz − 1) + 4 ≤ 2
bo

Bất đẳng thức cuối đúng vì theo AM – GM ta có


y 2 + z 2 x2 + y 2 + z 2
et

yz ≤ ≤ =1.
2 2
vi

x y+z 1
Từ đó suy ra M ≤ + + =
1.
x + y + z +1 x + y + z +1 x + y + z +1
ng

Với x= y= 1, z= 0 thì M bằng 1. Vậy giá trị lớn nhất của M bằng 1.
Bài tập tương tự
a

Cho x,y,z là các số thực không âm thoả mãn điều kiện x 2 + y 2 + z 2 =


8.
kh

x2 y+z 2
Chứng minh rằng + + ≤1.
x + x + 4 + yz
2 x + y + z + 1 xyz + 5 2

 BÀI TẬP RÈN LUYỆN


Bài 1. Cho x,y,z là các số thực không âm thoả mãn điều kiện x 2 + 4 y 2 + z 2 =
8.
Tìm giá trị nhỏ nhất của biểu thức
4 yz + 8
= P +2 ( y − 1)2 + ( z − 1)2 + 3 yz .
xz + 2 yx + 2 yz

557
Khám phá tư duy Kỹ thuật giải bất ĐT Bài toán Max – Min – Đặng Thành Nam

HD: Chú ý
4 yz + 8 = 4 yz + 4 y 2 + z 2 + x 2 = x 2 + ( 2 y + z ) ≥ 2 x ( 2 y + z )
2

4 yz + 8 4 yz + 8 yz + 2 .
⇒ 2 yz + 4 ≥ x ( 2 y + z ) ⇒ ≥ =
xz + 2 yx + 2 yz 4 yz + 4 yz + 1
Nhóm thành hằng đẳng thức ta có:
( y − 1)2 + ( z − 1)2 + 3 yz =( y + z − 1)2 + 1 + yz ≥ 1 + yz .
yz + 2
Suy ra P ≥ + 4 (1 + yz ) ≥ 4 .
yz + 1

n
Bài 2. Cho x,y,z là các số thực không âm thoả mãn điều kiện x 2 + y 2 + z 2 =
2.

.v
x y z
Chứng minh rằng + + ≤2.
1 + yz 1 + zx 1 + xy

om
x 2x y 2y z 2z
HD: Chứng minh ≤ ; ≤ ; ≤ .
1 + yz x + y + z 1 + zx x + y + z 1 + xy x + y + z
.c
Bài 3. Cho x,y,z là các số thực không âm thoả mãn điều kiện x 2 + y 2 + z 2 =
2.
ok
x y z
Chứng minh rằng + + ≤2.
1 + yz 1 + zx 2 (1 + xy )
bo

HD: Sử dụng bất đẳng thức AM – GM ta có:


x y 1 x y 
+ ≤  + + 2 .
1 + yz 1 + zx 2  1 + yz 1 + zx
et


x 2x y 2y z 2z
≤ ≤ ≤
vi

Chứng minh ; ; .
1 + yz x + y + z 1 + zx x + y + z 1 + xy x + y + z
ng

Bài 4. Cho a,b,c là các số thực không âm thoả mãn điều kiện a 2 + b 2 + c 2 = 2.
2a + c 2b + c a + b + c
Chứng minh rằng + + ≤5.
a

1 + bc 1 + ca abc + 2
kh

a+b+c a+b+c
HD: Chứng minh a + b + c − abc ≤ 2;1 + bc ≥ ;1 + ca ≥ .
2 2
5
Bài 5. Cho a,b,c là các số thực không âm thoả mãn điều kiện a 2 + b 2 + c 2 =.
2
18
Tìm giá trị lớn nhất của biểu thức P = ab + bc + ca − .
9 − 5 (1 + bc ) + 1
2

5
( a + b + c )2 −
=
HD: ab + bc + ca 2 ≤ 5 (1 + bc )2 − 5
2 2 4

558
Cty TNHH MTV DVVH Khang Việt

 
1 36  − 5 ≤ − 33 .
⇒ P ≤ 5 (1 + bc ) −
2
2 
9 − 5 (1 + bc ) + 1  4
2 4

Bài 6. Cho x,y,z là các số thực không âm thỏa mãn điều kiện x 4 + y 4 + z 4 =
2.
Tìm giá trị lớn nhất của biểu thức

x4 ( y + z )2 1 + yz ( 2 y 2
+ 2 z 2 + 3 yz )
P + −
x 4 + x 2 + yz ( 2 y 2 + 2 z 2 + 3 yz ) + 1 x 2 + 1 + ( y + z )
2 9
HD: Ta có
( ) ( )

n
2 + 2 yz 2 y 2 + 2 z 2 + 3 yz =x 4 + y 4 + z 4 + 2 yz 2 y 2 + 2 z 2 + 3 yz

.v
= x4 + ( y + z ) ≥ 2 x2 ( y + z )
4 2

om
(
⇒ 1 + yz 2 y 2 + 2 z 2 + 3 yz ≥ x 2 ( y + z ) ) 2

(
⇒ x 4 + x 2 + yz 2 y 2 + 2 z 2 + 3 yz + 1 ) .c
≥ x4 + x2 + x2 ( y +=
z) x2  x2 + 1 + ( y + z ) 
2 2
 
ok
x4 x2
⇒ ≤
(
x 4 + x 2 + yz 2 y 2 + 2 z 2 + 3 yz + 1 ) x2 + 1 + ( y + z )
2
bo

x4 + ( y + z )
4
Mặt khác: 1 + yz 2 y 2 + 2 =
z 2 + 3 yz( ) ≥
1 2 2 2
x + ( y + z)  .
et

2 4 

(x + ( y + z)
2 2
)
vi

2
x2 + ( y + z )
2
Suy ra P ≤ − .
x +1+ ( y + z)
ng

2 2 36

t t2
Đặt t = x 2 + ( y + z ) , ( t > 0 ) ta có P ≤ f (t ) =
2
− .
a

t + 1 36
kh

t t2
Xét hàm số f=
(t ) − với t > 0 ta có
t + 1 36
1 t
f '(t ) = − ; f '(t ) = 0 ⇔ t = 2 (do t > 0 ).
( t + 1) 2 18
Ta có f’(t) đổi dấu từ dương sang âm khi đi qua t = 2 nên f(t) đạt cực đại tại t = 2 .
5
Do đó P ≤ f (t ) ≤ f (2) = .
9
Đẳng thức xảy ra khi và chỉ khi ( x; y; z ) = (1;1;0 ) ; (1;0;1) .

559
Khám phá tư duy Kỹ thuật giải bất ĐT Bài toán Max – Min – Đặng Thành Nam

5
Vậy giá trị lớn nhất của P bằng .
9
Đẳng thức xảy ra khi và chỉ khi ( x; y; z ) = (1;1;0 ) ; (1;0;1) .

Cách 2: Ta có:
2 + 2 yz (2 y 2 + 2 z 2 + 3 yz ) =x 4 + y 4 + z 4 + 2 yz (2 y 2 + 2 z 2 + 3 yz )
= x 4 + ( y + z )4 ≥ 2 x 2 ( y + z )2
Do đó:
x2 ( y + z )2 x 4 + ( y + z )4

n
P≤ + −
x 2 + ( y + z )2 + 1 x 2 + ( y + z )2 + 1 18

.v
x 2 + ( y + z )2 x 4 + ( y + z )4

om
x 2 + ( y + z )2 + 1 18
.
 1 x 4 + ( y + z )4 
=1−  2 + 
 x + ( y + z ) + 1
2 18  .c
 2 x 4 + ( y + z )4 + 4 4  5
ok
≤1−  4 + − ≤
 x + ( y + z ) + 4
4 18 18  9
5
bo

=
Vậy MaxP khi =
x 1,=
y 1,=
z 0 hoặc=
x 1,=
y 0,=
z 1.
9
Bài 7. Cho x,y,z là các số thực không âm thỏa mãn điều kiện x3 + y 3 + z 3 =
2.
et

Tìm giá trị lớn nhất của biểu thức


2 (1 + yz ( y + z ) )
vi

x2 y+z
=P + − .
x 2 + x + yz ( y + z ) + 1 x + y + z +1
ng

27
HD: Ta có
3 + 3 yz ( y + z ) =+
x3 y 3 + z 3 + 3 yz ( y + z ) + 1 =+
x3 ( y + z ) + 1 ≥ 3x ( y + z )
a

⇒ x ( y + z ) ≤ 1 + yz ( y + z )
kh

⇒ x 2 + x + yz ( y + z ) + 1 ≥ x 2 + x + x ( y + z )= x ( x + y + z + 1) .
x2 x
⇒ ≤
x + x + yz ( y + z ) + 1
2 x + y + z +1

x+ y+z 2 (1 + yz ( y + z ) )
⇒P≤ −
x + y + z +1 27

560
Cty TNHH MTV DVVH Khang Việt

Mặt khác ta có
( x + y + z )3 = x3 + y 3 + z 3 + 3x ( y + z )( x + y + z )
= 2 + 3x ( y + z )( x + y + z )
≤ 2 + ( x + y + z ) ( 3 + 3 yz ( y + z ) ) .

⇒ 1 + yz ( y + z ) ≥
( x + y + z) − 2
3

3( x + y + z )

2 (x + y + z) − 2
3
x+ y+z
Do đó P ≤ − . .
1 + x + y + z 81 x+ y+z

n
2 t3 − 2 ( )

.v
t
Đặt t = x + y + z vì t ≥ x + y + z ⇒ t ≥ 2 và P ≤ f (t ) =
3 3 3 3
− 3
.
t +1 81t

om
Xét hàm số f=
(t )
t

2 t3 − 2
với t ≥ 3 2 ta có
( )
t +1 81t
( ) ; f '(t ) = 0 ⇔ t = 2 .
f '(t ) =
1

4 t3 + 1
.c
( t + 1)
ok
2 2
81t
Ta có f’(t) đổi dấu từ dương sang âm khi đi qua t = 2 nên f(t) đạt cực đại tại
bo

t=2.
16
Do đó P ≤ f (t ) ≤ f (2) = .
et

27
16
Với x= y= 1, z= 0 thì P = .
vi

27
16
xảy ra khi ( x; y; z ) = (1;1;0 ) ; (1;0;1) .
ng

Vậy giá trị lớn nhất của P bằng


27
Bài 8. Cho a,b,c là các số thực không âm thoả mãn điều kiện a 2 + b 2 + c 2 =
1.
a

a b (1 + bc) 2
kh

Tìm giá trị lớn nhất của biểu thức P = + − .


1 + bc 1 + ac 3 + 2 2

Kết hợp đánh giá bất đẳng thức AM – GM và Cauchy – Schwarz


Bất đẳng thức hoặc bài toán cực trị có dạng đối xứng giữa hai biến số ta thường
sử dụng đánh giá bằng bất đẳng thức AM – GM hoặc Cauchy – Schwarz giữa hai
biểu thức đối xứng về biểu thức còn lại đưa về khảo sát hàm số.
 CÁC VÍ DỤ
Ví dụ 1. Cho các số thực thoả mãn điều kiện x > y > z > 0 .

561
Khám phá tư duy Kỹ thuật giải bất ĐT Bài toán Max – Min – Đặng Thành Nam

y z x2
Tìm giá trị nhỏ nhất của biểu thức P = + + .
x − y y − z 8x ( xz − z )
Lời giải
y z x
Đặt a = , b = , c = ⇒ abc =
1.
x y z
a b c2
Khi đó P = + + .
1− a 1− b 8 c −1 ( )
Sử dụng bất đẳng thức AM – GM cho hai số dương ta có

n
a b 2 ab 4 ab 4 ab 2 ab
+ ≥ ≥ ≥ = .

.v
1− a 1− b (1 − a )(1 − b ) 2 − a − b 2 − 2 ab 1 − ab

om
2 ab c2 c 2 + 16
Suy ra P ≥ + = .
1 − ab 8 c − 1 8 c − 1 ( ) ( )
c 2 + 16 .c
Xét hàm số f (c) = trên khoảng (1; +∞) ta có
8 ( c −1 )
ok
3c 2 − 4c c − 16
f '(c) = ; f '(c) = 0 ⇔ 3c 2 − 16 = 4c c ⇔ c = 4 .
( )
2
bo

16 c c −1
Lập bảng biến thiên ta có kết quả P=
min min f=
(c) f=
(4) 4 .
et

Đẳng thức xảy ra khi và chỉ khi a = b, abc = 1, c = 4 ⇔ x = 2 y = 4 z .


Ví dụ 2. (TSĐH Khối B 2013) Cho a,b,c là các số thực dương.
vi

Tìm giá trị lớn nhất của biểu thức


ng

4 9
= P − .
a + b + c + 4 ( a + b ) ( a + 2c )( b + 2c )
2 2 2
a

Lời giải
kh

Nhận xét. Nhận thấy a,b đối xứng với nhau nên vai trò của a + 2c, b + 2c như nhau
ta sử dụng AM-GM cho hai số đó
Sử dụng bất đẳng thức AM-GM ta được:
a + 2c + b + 2c
( a + b ) ( a + 2c )( b + 2c ) ≤ ( a + b ) .
2

=
( a + b )2 + 4c ( a + b ) ≤ ( a + b )2 + 4c 2 + ( a + b )2
2 2
( ) (
=( a + b ) + 2c 2 ≤ 2 a 2 + b 2 + 2c 2 =2 a 2 + b 2 + c 2
2
)
562
Cty TNHH MTV DVVH Khang Việt

4 9
Vậy P ≤ − .
a 2 + b2 + c2 + 4 (
2 a + b2 + c2
2
)
4 9
Đặt t= a 2 + b 2 + c 2 + 4, ( t > 2 ) ⇒ P ≤ − .
2
t 2 t −4 ( )
4 9
Xét hàm số f (t )= − trên khoảng ( 2;+∞ ) ta được:
2
(
t 2 t −4 )
4 9t ( 4 − t ) ( 4t 3
+ 7t 2 − 4t − 16 ) ; f '(t) =0 ⇔ t =4 .
f '(t ) =− + =
(t ) ( )

n
2 3 3
t 2
−4 t2 t2 − 4

.v
Ta có f’(t) đổi dấu từ dương sang âm khi đi qua t = 4 nên f(t) đạt cực đại tại
t = 4 trên khoảng ( 2;+∞ ) .

om
5
Suy ra P ≤ f (t ) ≤ f (4) = .
8 .c
Đẳng thức xảy ra khi và chỉ khi a= b= c= 2 .
ok
5
Vậy giá trị lớn nhất của P bằng đạt tại a= b= c= 2 .
8
Bài tập tương tự
bo

Cho a,b,c là các số thực dương. Tìm giá trị nhỏ nhất của biểu thức
9 16
= −
et

P .
ac ( a + 2b )( c + 2b ) a 2 + b2 + c2 + 1
Ví dụ 3. Cho các số thực x, y, z ∈ ( 0;1] thoả mãn điều kiện x + y ≥ z + 1 .
vi
ng

x y z
Tìm giá trị nhỏ nhất của biểu thức P = + + .
y + z z + x xy + z 2
a

Lời giải
Sử dụng bất đẳng thức C-S và AM-GM ta được:
kh

P=
x2
+
y2
+
z

( x + y) + z 2

x ( y + z ) y ( z + x ) xy + z 2 2 xy + z ( x + y ) xy + z 2


( x + y)
2
+
z
=
2( x + y)
2
+
4z
( x + y ) + z x + y  x + y  + z 2 ( x + y ) + 2 z ( x + y ) ( x + y )2 + 4 z 2
2 2 2
( )  2 
2  
2t 2 4z
Đặt t = x + y, (1 + z ≤ t ≤ 2 ) khi đó P ≥ f (t=
) + .
t + 2tz
2
t + 4z2
2

563
Khám phá tư duy Kỹ thuật giải bất ĐT Bài toán Max – Min – Đặng Thành Nam

2t 2 4z
=
Xét hàm số f (t ) + liên tục trên [1 + z;2] ta có
t + 2tz
2
t + 4z2
2

   
 t 2  2 2 
f '(t=
) 4 zt  −  ≤ 4 zt  − =
 0
( ) ( ) ( ) ( )
2 2 2 2 2
 t + 2 zt t 2 + 4z2   t + 4 z t + 4z
2 2 2 2
 

Do đó f(t) là hàm nghịch biến trên [1 + z;2] .


Vì vậy
2 z 3z 2 + z + 2
P ≥ f (t ) ≥ f (2) = + =
1 + z 1 + z2 (
1 + z 2 (1 + z ) )

n
.v
=
−3 z 3 + 3 z 2 − z + 1 3 (1 − z ) 3 z + 1 3 3
= +
2
+ ≥ , ∀z ∈ ( 0;1]
( )
( ) ( )

om
2 1 + z 2 (1 + z ) 2 2 1 + z 2 (1 + z ) 2 2

3
Vậy giá trị nhỏ nhất của P bằng đạt tại x= y= z= 1 . .c
2
Ví dụ 4. Cho các số thực dương a, b, c thỏa mãn điều kiện a + b + 1 =c.
ok
a2 b2 c 2 + 16
Tìm giá trị nhỏ nhất của biểu thức: F = + + .
a + bc b + ca c + ab
bo

Lời giải
Sử dụng bất đẳng thức C –S ta có:
et

F≥
( a + b)
2
+
c 2 + 16 c − 1 2c 2 + 32 2c 2 + c + 31
= + = .
( a + b )( c + 1) ( a + b )2 c +1 c +1 c +1
vi

c+
4
ng

2c 2 + c + 31
Xét f ( c ) = , c > 1 ⇒ f ′(c) = 0 ⇒ c = 3 .
c +1
a

43
Từ đó suy ra : MinF = ⇔ c = 3, a = b = 1 .
kh

4
Ví dụ 5. Cho x > 1, y > 0, z > 0 . Tìm giá trị lớn nhất của biểu thức
1 2
P − .
x2 + y 2 + z 2 − 2 x + 2 x ( y + 1)( z + 1)
Lời giải
Nhận xét. Dự đoán dấu bằng xảy ra khi x = y + 1 = z + 1 nên ta sử dụng bất
đẳng thức AM-GM cho x ( y + 1)( z + 1) .
Sử dụng bất đẳng thức AM-GM ta có:

564
Cty TNHH MTV DVVH Khang Việt

3
 x+ y+ z+2
x ( y + 1)( z + 1) ≤   .
 3 
Sử dụng bất đẳng thức C-S ta được:
1 1
x 2 + y 2 + z 2 − 2 x + 2 = ( x − 1) + y 2 + z 2 + 1 ≥ ( x − 1 + y + z + 1)2 = ( x + y + z )2 .
2
4 4
2 54
Suy ra P ≤ − .
x + y + z ( x + y + z + 2 )3
2 54
Đặt t = x + y + z , ( t > 1) khi đó P ≤ f (t ) =− .
t ( t + 2 )3

n
.v
2 54
Xét hàm số f (t )= − trên (1;+∞ ) ta có:
t ( t + 2 )3

om
162 2
f '(=
t) − ; f '(=
t ) 0 ⇔ 81=
t2 ( t + 2 )4 ←
t >1
→=t 4.
(t + 2) 4
t 2

.c
Ta có f’(t) đổi dấu từ dương sang âm khi đi qua t = 4 nên f(t) đạt cực đại tại
1
t = 4 hay P ≤ f (t ) ≤ f (4) =
ok
.
4
1
Vậy giá trị lớn nhất của P bằng đạt tại x= 2, y= z= 1 .
bo

4
Bài tập tương tự
Cho x,y,z là các số thực không âm thoả mãn điều kiện x + y > 0 .
et

Tìm giá trị lớn nhất của biểu thức


vi

1 4
= F − .
ng

x 2 + y 2 + z 2 + 2 z + 2 3 ( x + y )3 ( z + 2)3
Ví dụ 6. Cho a,b,c là các số thực dương thỏa mãn điều kiện
a

a 2 + b 2 + c 2 + 2ab= 3 ( a + b + c ) .
kh

2014 2014
Tìm giá trị nhỏ nhất của biểu thức P = 6 ( a + b ) + c 2 + + .
a+c b+2
Lời giải
1
Theo giả thiết ta có: 3 ( a + b + c ) = ( a + b )2 + c 2 ≥ ( a + b + c )2 ⇔ a + b + c ≤ 6 .
2
1 1 4 4 4
Ta có + ≥ ≥ = .
a+c b+2 a+c + b+2 2 ( a + c + b + 2) 2(a + b + c) + 4

6 ( a + b ) + c 2 ≥ 6 ( a + b ) + 6c − 9= 6 ( a + b + c ) − 9 .

565
Khám phá tư duy Kỹ thuật giải bất ĐT Bài toán Max – Min – Đặng Thành Nam

2014.4
Suy ra P ≥ 6 ( a + b + c ) + −9.
2(a + b + c) + 4
2014.4
Đặt t = a + b + c, ( 0 < t ≤ 6 ) ta có P ≥ f (t ) =6t + .
2t + 4
2014.4
Xét hàm số f (t=
) 6t + với t ∈ ( 0;6] ta có
2t + 4

8056 6 ( 2t + 4 )3 − 8056
f '(t ) = 6 − = < 0, ∀t ∈ ( 0;6] .
( 2t + 4 )3 ( 2t + 4 )3

n
Do đó f(t) nghịch biến trên ( 0;6] ⇒ P ≥ f (t ) ≥ f (6) =
2050 .

.v
a = 1
a + b = c = 3 

om
Đẳng thức xảy ra khi và chỉ khi  ⇔ b = 2.
a + c = b + 2 c = 3

Vậy giá trị nhỏ nhất của P bằng 2050 đạt tại =
a 1,=b 2,= c 3. .c
Bài tập tương tự
ok
Cho a,b,c là các số thực dương thỏa mãn điều kiện
a 2 + b 2 + c 2 = 5 ( a + b + c ) − 2ab .
bo

3 48
Tìm giá trị nhỏ nhất của biểu thức P = a + b + c + 48 +3 .
a + 10 b+c
et

Ví dụ 7. Cho a,b,c là các số thực dương thỏa mãn điều kiện a + b + 1 =c.
Tìm giá trị nhỏ nhất của biểu thức
vi

a3 b3 c3 14
P= + + + .
ng

a + bc b + ca c + ab ( c + 1) ( a + 1)( b + 1)
Lời giải
a

Sử dụng bất đẳng thức AM-GM ta có:


kh

c + ab =a + b + 1 + ab =( a + 1)( b + 1) ≤
( a + b + 2 )2 =( c + 1)2 .
4 4
Mặt khác sử dụng bất đẳng thức C –S ta có:

(a )
2
a3
b 3
a 4
b 4 2
a 2 + b2 + b2
+ = + ≥ 2 =
a + bc b + ca a ( a + bc ) b ( b + ac ) a + b 2 + 2abc 1 + 2abc
a 2 + b2 .
a 2 + b2 1 ( a + b ) ( c − 1)
2 2
≥ ≥ =
1+ c 2 c +1 2 ( c + 1)

566
Cty TNHH MTV DVVH Khang Việt

Do đó P ≥ f (c=
( c − 1)2 4c3 28
) + + .
2 ( c + 1) ( c + 1) 2
( c + 1)2
( 3c − 5) ( 3c 2 + 14c + 23) 5
Ta có f '(c) = ; f '(c) = 0 ⇔ c = .
2 ( c + 1)
3 3
5
Ta có f’(c) đổi dấu từ âm sang dương khi đi qua c = nên f(c) đạt cực tiểu tại
3
5  
5 53
c = hay P ≥ f (c) ≥ f   = .
3 3 8

n
1 5
Đẳng thức xảy ra khi và chỉ khi a= b= , c=

.v
.
3 3

om
53 1 5
Vậy giá trị nhỏ nhất của P bằng đạt tại a= b= , c= .
8 3 3

 BÀI TẬP RÈN LUYỆN .c


Bài 1. Cho các số thực dương a,b,c.
ok
2
a b  c 
Tìm giá trị nhỏ nhất của biểu thức P = + +  .
b+c c+a a+b
bo

HD. Sử dụng bất đẳng thức C-S ta được:


a
+
b
=
a2
+
b2

(a + b) = 2
1

1
et

.
b + c c + a a ( b + c ) b ( a + c ) c ( a + b ) + 2ab c
+
2ab c
+
1
a + b ( a + b )2 a + b 2
vi

2
1  c 
ng

Suy ra P ≥ +
1  a + b 
.
c
+
a+b 2
a

c 1
Đặt t
= , ( t > 0 ) ta có: P ≥ f (t ) = + t2 .
kh

a+b t+
1
2
1 1
Ta có: f '(t ) =− 2
+ 2t ; f '(t ) =0 ⇔ t = . Ta có f’(t) đổi dấu từ âm sang
 1 2
t + 
 2
1 1
dương khi đi qua t = nên tại t = thì f(t) đạt cực tiểu trên ( 0;+∞ ) hay
2 2
1 5
f (t ) ≥ f   = .
2 4
567
Khám phá tư duy Kỹ thuật giải bất ĐT Bài toán Max – Min – Đặng Thành Nam

5
Vậy giá trị nhỏ nhất của P bằng đạt tại a= b= 2c .
4
Bài 2. Cho a,b,c là các số thực dương. Tìm giá trị lớn nhất của biểu thức
1 1
= P − .
2 a + b + c +1 (
2 2 2 a + 1)( + 1)( c + 1)
b
HD. Sử dụng bất đẳng thức C-S và AM-GM ta có:
1
a 2 + b 2 + c 2 + 1 ≥ ( a + b + c + 1)
2
4
3
a +b + c +3
( a + 1)( b + 1)( c + 1) ≤  

n
 3 

.v
1 27
Suy ra P ≤ − .
a + b + c + 1 ( a + b + c + 3)3

om
1 27
Đặt t = a + b + c + 1, ( t > 1) khi đó P ≤ f (t ) =− .
t ( t + 2 )3

Xét hàm số f (t )=
1

27
với t > 1 ta có:
.c
t ( t + 2 )3
ok
1 81
f '(=
t) − + ; f '(=
t ) 0 ⇔ 81=
t2 ( t + 2 )4 ←
t >1
9t ( t + 2 ) ⇔
→=
2
=t 4.
bo

t 2
(t + 2) 4

Ta có f’(t) đổi dầu từ dương sang âm khi đi qua t = 4 nên tại t = 4 f(t) đạt cực
et

1
đại suy ra P ≤ f (t ) ≤ f (4) = .
8
vi

1
Vậy giá trị lớn nhất của P bằng đạt tại a= b= c= 1 .
ng

8
Bài 3. Cho a,b,c là các số thực dương.
( a + b + c + 3)2 .
a

2
=
Tìm giá trị lớn nhất của biểu thức P −
3 ( a + 1)( b + 1)( c + 1)
kh

a 2 + b2 + c2 + 1
Bài 4. Cho a,b,c là các số thực dương thỏa mãn điều kiện
a 2 + b 2 + c 2 = 5 ( a + b + c ) − 2ab .
3 48
Tìm giá trị nhỏ nhất của biểu thức P = a + b + c + 48 +3 .
a + 10 b+c
HD: Từ điều kiện ta có: ( a + b ) + c 2= 5 ( a + b + c ) .
2

Mặt khác:

568
Cty TNHH MTV DVVH Khang Việt

1 1
( a + b )2 + c 2 ≥ ( a + b + c )2 ⇒ ( a + b + c )2 ≤ 5 ( a + b + c ) ⇔ a + b + c ≤ 10 .
2 2
Dự đoán được a + b = c, a + b + c = 10 ⇒ c = 5, a + b = 5 ta sử dụng bất đẳng thức
AM-GM cho hai căn thức:
Sử dụng bất đẳng thức AM-GM ta có:
a + 10
+4
a + 10 1 a + 10 a + 22
= .4 ≤ 3 =
3 2 3 4 22
13 b + c + 16
3
=b+c 8.8 ( b + c ) ≤
4 12

n
 1 1  2304
Suy ra P ≥ a + b + c + 48.12  + ≥a+b+c+

.v
.
 a + 22 b + c + 16  a + b + c + 38

om
2304
Đặt t = a + b + c, ( 0 < t ≤ 10 ) khi đó P ≥ f (t ) =
t+ .
t + 38
2304 ( t + 38)2 − 2304 ≤ 0, ∀ t ∈ 0;10 suy
Ta có f '(t ) = 1 − = .c ( ] ra f(t) nghịch
( t + 38)2 ( t + 38)2
ok
biến trên ( 0;10] .
Do đó P ≥ f (t ) ≥ f (10) =
58 .
bo

Vậy giá trị nhỏ nhất của P bằng 58 đạt tại =


a 2,=
b 3,=
c 5.
et

Bài 5. Cho a,b,c là các số thực dương. Tìm giá trị lớn nhất của biểu thức
5 4
= −
vi

P .
a 2 + 2b 2 + 5c 2 + 3 + 1 ab + bc + ca + 1
ng

HD: Sử dụng bất đẳng thức C-S ta có:


a 2 b2 c2 ( a + b + c )
2
2a 2 + 3b 2 + 6c 2 = + + ≥ = ( a + b + c )2
a

1 1 1 1 1 1
+ +
kh

2 3 6 2 3 6
⇒ a 2 + 2b 2 + 5c 2 ≥ 2 ( ab + bc + ca )
Đẳng thức xảy ra khi và chỉ khi 2= a 3=b 6c .
5 4
Do đó P ≤ − .
2 ( ab + bc + ca ) + 3 + 1 ab + bc + ca + 1
Bài 8. Cho a,b,c là các số thực thuộc đoạn [1;2] .

Tìm giá trị nhỏ nhất của biểu thức P =


( a + b )2 .
c 2 + 4 ( ab + bc + ca )
569
Khám phá tư duy Kỹ thuật giải bất ĐT Bài toán Max – Min – Đặng Thành Nam

HD : Sử dụng bất đẳng thức AM-GM ta có 4ab ≤ ( a + b ) .


2

2
a b
( a + b )2  + 
Khi đó P ≥ = c c
c2 + 4 ( a + b ) c + ( a + b )
2 2
a b a b
1+ 4 +  +  + 
c c c c
a+b a+b
Ta đặt t = thì do a, b, c ∈ [1;2] ⇒=t ∈ [1;4]
c c
t2 4t 2 + 2t
Xét hàm số f (t ) = = có f '(t ) > 0, ∀t ∈ [1;4] .
1 + 4t + t 2
(t )
2

n
2
+ 4t + 1

.v
1
Do đó P ≥ f (t ) ≥ f (1) = . Đẳng thức xảy ra khi và chỉ khi a= b= 1, c= 2 .

om
6
1
Vậy giá trị nhỏ nhất của P bằng đạt tại a= b= 1; c= 2 .
6

Bài 9. Cho a,b,c là các số thực dương. Tìm giá trị nhỏ nhất của biểu thức
.c
ok
9 16
= P − .
ac ( a + 2b )( c + 2b ) a 2 + b2 + c2 + 1
bo

HD: Sử dụng bất đẳng thức AM-GM ta có:


4
1 1  4a + 4c + 4b  1
ac ( a + 2b )( c + 2b ) ≤ .3a.3c. ( a + 2b )( c + 2b ) ≤   = (a + b + c) .
et

4
9 9 4  9

( ).
vi

2
Và sử dụng bất đẳng thức C-S ta có: ( a + b + c ) ≤ 9 a 2 + b 2 + c 2
4
ng

9 16
Suy ra P ≥ − .
a +b +c
2 2 2
a 2 + b2 + c2 + 1
a

916
Đặt t = a 2 + b 2 + c 2 + 1, ( t > 1) khi đó P ≥ f (t ) = . −
kh

t −1 t
2

9
16
Xét hàm số =
f (t ) trên (1;+∞ ) ta có:

t −1 t
2

f '(t )= −
18t
+ 2=
(
16 2 ( t − 2 ) 8t + 7t − 2t − 4
3 2
t >1
; f '(t )= 0 ←
)
→ t= 2 .
(t ) ( )
2 2
2
−1 t t t −1
2 2

Ta có f’(t) đổi dấu từ âm sang dương khi đi qua t = 2 nên f(t) đạt cực tiểu tại
t = 2, do đó P ≥ f (t) ≥ f(2) =
−5 .
Vậy giá trị nhỏ nhất của P bằng −5 đạt tại a= b= c= 1 .

570
Cty TNHH MTV DVVH Khang Việt

Bài 10. Cho a,b,c là các số thực dương thỏa mãn điều kiện a + b + c =
1.
Tìm giá trị nhỏ nhất của biểu thức
a2 b2 3
=P + − ( a + b )2 .
(b + c ) 2
+ 5bc (c + a) 2
+ 5ca 4
HD: Sử dụng bất đẳng thức AM-GM ta có:
a2 a3 2a 3
= =
( b + c )2 + 5bc a ( b + c )2 + 5abc 2a ( b + c )( b + c ) + 10abc
2a 3
≥ 3 3
=
3a3

n
 2a + b + c + b + c  a+b+c
  + 10  
   

.v
3 3
b2
Tương tự:

om
≥ 3b3 .
(c + a) 2
+ 5ca

(
Suy ra P ≥ 3 a3 + b3 −
3
4
) 3 3
( a + b )2 ≥ ( a + b )3 − ( a + b )2 .
4 4
.c
3
Đặt t = a + b, ( 0 < t < 1) khi đó P ≥ f (t ) = t 3 − t 2 . ( )
ok
4

Xét hàm số =
3 3 2
( )
t − t với t ∈ ( 0;1) ta có:
bo

f (t )
4
3
( ) 2
f '(t ) = 3t 2 − 2t ; f '(t ) = 0 ⇔ t = (t > 0) .
et

4 3
2
Ta có f’(t) đổi dấu từ âm sang dương khi đi qua t = nên f(t) đạt cực tiểu tại
vi

3
ng

2 2 1
t = . Do đó P ≥ f (t ) ≥ f   = − .
3 3 9
a

1
Đẳng thức xảy ra khi và chỉ khi a= b= c= .
3
kh

1 1
Vậy giá trị nhỏ nhất của P bằng − đạt tại a= b= c= .
9 3
( )
Cách 2: Ta có ( b + c ) + 5bc ≤ 2 b 2 + c 2 + 5bc =( b + 2c )( c + 2b ) .
2

Suy ra
a2 a2 3a3 3a3
≥ = ≥ = 3a3 .
( b + c )2 + 5bc ( b + 2c )( c + 2b ) 3a ( b + 2c )( c + 2b )  3a + b + 2c + c + 2b  3
 
 3 

571
Khám phá tư duy Kỹ thuật giải bất ĐT Bài toán Max – Min – Đặng Thành Nam

b2
Tương tự ta có: ≥ 3b3 . Ta có kết quả tương tự lời giải trên.
(c + a) 2
+ 5ca
Cách 3: Sử dụng bất đẳng thức AM-GM ta có
( b + c )2 a2 4a 2 4 a 
2
bc ≤ ⇒ ≥ =   .
4 ( b + c )2 + 5bc 9 (b + c )
2 9 1− a 

Mặt khác
a 9 1
≥ a− ⇔
( 3a − 1) ≥ 0 . 2

1− a 4 4 4 (1 − a )
2
a2 4 9 1

n
Suy ra ≥  a−  .
( b + c ) + 5bc 9  4 4 
2

.v
2
b2 4 9 1

om
Tương tự ta có: ≥  b−  .
( c + a ) + 5ca 9  4 4 
2

4  9 1  3
2 2
1 9
 a −  +  b −   − ( a + b )
Do đó P ≥
9  4 4 4 4   4
2 .c
ok
2  9 1  3
2
1 9
 a − + b −   − ( a + b )
2

9  4 4 4 4   4
bo

3c 2 c 5 1
= − − ≥−
8 4 72 9
et

Bài toán 3. Cho a,b,c là các số thực dương ta luôn có


vi

4
a + ab + 3 abc ≤ ( a + b + c ) .
3
ng

Chứng minh.
Sử dụng bất đẳng thức AM-GM ta có:
a

1 1
a + ab + 3 abc = a+ a.4b + 3 a.4b.16c
kh

2 4
1 a + 4b 1 a + 4b + 16c 4
≤a+ . + . = (a + b + c)
2 2 4 3 3
Bất đẳng thức được chứng minh. Đẳng thức xảy ra khi và chỉ khi = a 4=
b 16c .
Nhận xét. Đây là chỉ là một trường hợp đơn giản của đánh giá AM – GM từ trung
bình nhân sang trung bình cộng kết hợp với tham số hoá ta có thể vận dụng bất
đẳng thức này để xây dựng các bài toán cực trị.
Bài tập tương tự
1) Cho a,b,c là các số thực không âm thỏa mãn a + b + c = 3.

572
Cty TNHH MTV DVVH Khang Việt

3
Chứng minh rằng 2a + b + ab + bc + 3 abc ≤ 7 .
4
2) Cho a,b,c là các số thực dương. Chứng minh
6 ab + 7c + 8 ca ≤ 9 ( a + b + c ) .

 CÁC VÍ DỤ
Ví dụ 1. Cho a,b,c là các số thực dương. Tìm giá trị nhỏ nhất của biểu thức
1 3
= P − .
a + ab + abc
3
a+b+c

n
Lời giải
Sử dụng bất đẳng thức AM-GM ta được:

.v
1 a + 4b 3 13 a + 4b + 16c
ab =a.4b ≤ , abc = a.4b.16c ≤ .

om
2 4 4 12
4 3 3
Suy ra ab + bc + 3 abc ≤ ( a + b + c ) do đó P ≥ − .
3 .c2(a + b + c) a+b+c
2
3 3 1 
3 3 3
Đặt t = a + b + c , ( t > 0 ) khi đó P ≥
−=  − 1 − ≥ − .
ok
2t 2 t 2t  2 2
3 16 4 1
Vậy giá trị nhỏ nhất của P bằng − đạt tại
= a = ,b =
bo

,c .
2 21 21 21

Bài tập tương tự


et

Cho a,b,c là các số thực dương. Tìm giá trị nhỏ nhất của biểu thức
vi

2 3
= P − .
a + ab + abc
3
a+b+c
ng

Ví dụ 2. Cho a,b,c là các số thực dương thỏa mãn điều kiện a + b + c ≤ 1 .


Tìm giá trị nhỏ nhất và lớn nhất của biểu thức
a

1 1
= P − .
kh

6 ab + 7c + 8 ca 9 a + b + c
Lời giải
Sử dụng bất đẳng thức AM-GM ta có
6 ab + 7c + 8 =
ca 2 a.9b + 7c + 4 c.4a
≤ a + 9b + 7c + 2 ( c + 4a )
= 9(a + b + c)
2
1 1 1 1 1 1 1
Suy ra P ≥ −=  −  − ≥− .
9 ( a + b + c ) 9 a + b + c 9  a + b + c 2  36 36

573
Khám phá tư duy Kỹ thuật giải bất ĐT Bài toán Max – Min – Đặng Thành Nam

 18
a = 23

 a + b + c =2  2
Đẳng thức xảy ra khi và chỉ khi  ⇔ b = .
= a 9=
b; c 4a  23
 72
c = 23

1 18 2 72
Vậy giá trị nhỏ nhất của P bằng − đạt tại
= a = ,b = ,c .
36 23 23 23
Bài tập tương tự
Cho a,b,c là các số thực dương. Tìm giá trị nhỏ nhất của biểu thức

n
.v
abc 1
=P − .
6ab c + 7 abc3 + 8ca b 9 a+b+c

om
Ví dụ 3. Cho a,b,c là các số thực dương. Tìm giá trị lớn nhất của biểu thức

P=
8a + 3b + 4 ( ab + bc + 3 abc
.c ).
1 + (a + b + c)
2
ok
Lời giải
Sử dụng bất đẳng thức AM-GM ta được:
bo

4=
ab 2 a.4b ≤ a + 4b
4= bc 2 b.4c ≤ b + 4c
et

a + 4b + 16c
= 4 3 abc 3 a.4b.16c ≤
3
vi

Cộng theo vế 3 bất đẳng thức trên ta được:


ng

8a + 3b + 4 ( ab + bc + 3 abc ≤ ) 28
3
(a + b + c) .
a

28 ( a + b + c ) 14 14 ( a + b + c − 1)
2
14
Suy ra P ≤ = − ≤ .
kh

3 + 3( a + b + c )
2
(
3 3 ( a + b + c )2 + 1 3
)
16 4 1
Đẳng thức xảy ra khi và chỉ khi a + b + c = 1, a = 4b = 16c ⇔ a = ,b = ,c = .
21 21 21
14
Vậy giá trị lớn nhất của P bằng .
3

574
Cty TNHH MTV DVVH Khang Việt

 BÀI TẬP RÈN LUYỆN


Bài 1. Cho a,b,c là các số thực dương. Tìm giá trị nhỏ nhất của biểu thức
1 8
= P − .
2a + b + 2 2bc 2a + 2 ( a + c ) + 3
2 2

Bài 2. Cho a,b,c là các số thực không âm thỏa mãn a + b + c =3.


3
Chứng minh rằng 2a + b + ab + bc + 3 abc ≤ 7 .
4
Bài 3. Cho a,b,c là các số thực dương. Chứng minh
6 ab + 7c + 8 ca ≤ 9 ( a + b + c ) .

n
.v
Bài toán 4. Chứng minh rằng với mọi số thực x ≥ −1 ta có
x2 + 2

om
x3 + 1 ≤
.
2
Chứng minh.
Sử dụng bất đẳng thức AM – GM ta có
.c
( x + 1) ( x 2 − x + 1) ≤
x + 1 + x2 − x + 1 x2 + 2
ok
x3 + 1= = .
2 2
Bất đẳng thức được chứng minh. Dấu bằng xảy ra khi và chỉ khi hoặc
bo

x = 0 hoặc x = 2 .
Bất đẳng thức khác cũng hay được dùng
et

( ) x4 + 1
2
Chứng minh với mọi số thực x ta có x 2 − x + 1 ≥ .
2
vi

 CÁC VÍ DỤ
ng

Ví dụ 1. Cho a,b,c là các số thực dương thỏa mãn điều kiện


2a 2 + 3b 2 + 5ab + 3bc + 2ac + c ≤ 3 + 5a + 8b .
a

1 1 1
kh

Chứng minh rằng + + ≥ 1.


8a + 1 8b + 1 8c + 1
Lời giải
Viết lại điều kiện của bài toán thành: ( a + b + c − 3)( 2a + 3b + 1) ≤ 0 .
Vì a, b, c > 0 nên a + b + c ≤ 3 .
Đặt
= x 2=
a
, y 2=
b
, z 2c , ( x, y, z > 0 ) và=
xyz 2a +b + c ≤ 8 .
1 1 1
Ta cần chứng minh + + ≥1.
x +1
3
y +1
3
z +1
3

575
Khám phá tư duy Kỹ thuật giải bất ĐT Bài toán Max – Min – Đặng Thành Nam

( x + 1) ( x 2 − x + 1) ≤
x + 1 + x2 − x + 1 x2 + 2
Ta có x3 + 1= = .
2 2
1 2 1 2 1 2
Suy ra ≥ . Tương tự: ≥ 2 ; ≥ 2 .
x3 + 1 x +2
2
y3 + 1 y + 2 z3 + 1 z + 2
Đẳng thức xảy ra khi và chỉ khi x= y= z= 2 .
1 1 1 2 2 2
Do đó + + ≥ + + .
x3 + 1 y3 + 1 z3 + 1 x +22
y +2
2
z +2
2

2 2 2
Ta chỉ cần chứng minh + + ≥1.

n
x +22
y +22
z +22

( )( ) ( )( ) ( )( ) ( )( )( )

.v
⇔ 2 y 2 + 2 z 2 + 2 + 2 x2 + 2 z 2 + 2 + 2 x2 + 2 y 2 + 2 ≥ x2 + 2 y 2 + 2 z 2 + 2

⇔ 2 ( x y + y z + z x ) + 8 ( x + y + z ) + 24 ≥

om
2 2 2 2 2 2 2 2 2

≥ x y z + 4( x + y + z ) + 2( x y + y z + z x ) + 8
2 2 2 2 2 2 2 2 2 2 2 2

⇔ 4 ( x + y + z ) + 16 ≥ x y z
2 2 2 2 2 2
.c
ok
 1 1 1  16
⇔ 4  2 2 + 2 2 + 2 2  + 2 2 2 ≥ 1 (luôn đúng).
x y y z y z  x y z
bo

Vì theo bất đẳng thức AM-GM ta có


 1 1 1  16 1 16 12 16
et

4  2 2 + 2 2 + 2 2  + 2 2 2 ≥ 12 3 4 4 4 + 2 ≥ + = 1.
x y y z y z  x y z x y z 8 3 4
8 64
vi

Bài toán được chứng minh. Đẳng thức xảy ra khi và chỉ khi a= b= c= 1 .
Ví dụ 2. Cho x,y,z là các số thực dương thỏa mãn điều kiện xyz = x + y + z + 2 .
ng

1 1 1
Tìm giá trị lớn nhất của biểu thức P = + + .
x2 + 2 y2 + 2 z2 + 2
a
kh

Lời giải
Viết lại đẳng thức đề bài đã cho dưới dạng:
(1 + x )(1 + y )(1 + z ) = (1 + x )(1 + y ) + (1 + y )(1 + z ) + (1 + z )(1 + x ) .
1 1 1
⇔ + + =
1.
1+ x 1+ y 1+ z
Sử dụng bất đẳng thức C-S ta có:

(x 2
) 1
+ 2 1 +  ≥ ( x + 1) ⇒
 2
2 1
x2 + 2

3 1
.
2 x +1
.

576
Cty TNHH MTV DVVH Khang Việt

1 3 1 1 3 1
Tương tự ta có: ≤ . ; ≤ . .
y2 + 2 2 y + 1 z2 + 2 2 z +1
Đẳng thức xảy ra khi và chỉ khi x= y= z= 2 .
Cộng theo vế 3 bất đẳng thức trên ta được
1 1 1 3 1 1 1  3
+ + ≤  + + = .
x +2 2
y +22
z +22 2 1+ x 1+ y 1+ z  2
Đẳng thức xảy ra khi và chỉ khi x= y= z= 2 .
6
Vậy giá trị lớn nhất của P bằng đạt tại x= y= z= 2 .

n
2

.v
Ví dụ 3. Cho a,b,c là các số thực dương thỏa mãn điều kiện a 2 + b 2 = c 2 + 1 .
Tìm giá trị nhỏ nhất của biểu thức

om
a3 b3 2c 3 + 1
P= + + .
a3 + ( b + c ) b3 + ( c + a )
3 3 27
Lời giải
.c
ok
Sử dụng bất đẳng thức AM-GM ta có:

(a ) ( ) ( ) ( )( )
2 2
2
+ b2 + c2 =a 4 + 2a 2 b 2 + c 2 + b 2 + c 2 =a 4 + b 2 + c 2 b 2 + c 2 + 2a 2
bo

( b + c )2  ( b + c )2 + 2a 2 
1 1
≥ a4 +
2 2 
et

1 1
≥ a4 + ( b + c )2 .2 ( b + c )2 .2a 2 = a 4 + a ( b + c )3
vi

2 2
a3 a4 a3 a2
ng

Do đó ≥ ⇒ ≥ .
a3 + ( b + c ) (a ) a3 + ( b + c ) a 2 + b2 + c2
3 2 2 2 3
+b +c 2
a

b3 b2
Tương tự ta có: ≥
kh

.
b3 + ( c + a ) a 2 + b2 + c2
3

Cộng theo vế 2 bất đẳng thức trên ta được:


a 2 + b2 2c3 + 1 c 2 + 1 2c3 + 1
P≥ + = 2 +
a 2 + b2 + c2 27 2c + 1 27
( c − 1)2 ( 4c3 + 8c 2 + 14c + 7 ) 7 7.
+ ≥
(
27 2c + 1 2
) 9 9

Dấu bằng xảy ra khi và chỉ khi a= b= c= 1 . Vậy giá trị nhỏ nhất của P bằng 7/9.

577
Khám phá tư duy Kỹ thuật giải bất ĐT Bài toán Max – Min – Đặng Thành Nam

Ví dụ 4. Cho a,b,c là các số thực dương thỏa mãn điều kiện a + b = c + 1 .


Tìm giá trị nhỏ nhất của biểu thức
9a 2 9b 2 a + b + c2
P=3 +3 + .
( b + c )2 + 5bc ( c + a )2 + 5ca 9
Lời giải
Sử dụng bất đẳng thức C-S và AM-GM ta có:
9a 2 9a 2 9a 2 27 a3
≥ = =
( b + c )2 + 5bc (
2 b 2 + c 2 + 5bc ) ( b + 2c )( c + 2b ) 3a ( b + 2c )( c + 2b )

n
27 a3 27 a3
≥ =

.v
 3a + b + 2c + c + 2b 
3
( a + b + c )3
 
 3 

om
9a 2 3a
Suy ra 3 ≥ .
(b + c ) 2
+ 5bc a+b+c
.c
9b 2 3b
Tương tự ta có: 3 ≥
ok
.
( c + a ) + 5ca 2 a+b+c

3( a + b ) a + b + c2 3 ( c + 1) c 2 + c + 1
bo

Do đó P≥ + = + .
a+b+c 9 2c + 1 9
3 ( c + 1) c2 + c + 1
et

=
Xét hàm số f (c ) + với c > 0 ta có
2c + 1 9
vi

2c + 1 ( 2c + 1) − 27
3
3
; f '(c) =0 ⇔ ( 2c + 1) =27 ⇔ c =
2
f '(c) =− + = 1.
( 2c + 1)2 9 ( 2c + 1)
ng

9 2

Ta có f’(c) đổi dấu từ âm sang dương khi đi qua c = 1 nên f(c) đạt cực tiểu tại c = 1 .
a

7
Do đó P ≥ f (c) ≥ f (1) = .
kh

3
Đẳng thức xảy ra khi và chỉ khi a= b= c= 1 .
7
Vậy giá trị nhỏ nhất của P bằng đạt tại a= b= c= 1 .
3

 BÀI TẬP RÈN LUYỆN


Bài 1. Chứng minh rằng với mọi số thực dương a,b,c ta có
a3 b3 c3
+ + ≥1.
a3 + ( b + c ) b3 + ( c + a ) c3 + ( a + b )
3 3 3

578
Cty TNHH MTV DVVH Khang Việt

Bài 2. Cho các số thực dương x, y, z thỏa mãn điều kiện x, y ≥ 1 và xyz = 1 .
Tìm giá trị lớn nhất của biểu thức
1 1 1
P= + + .
( ) ( ) ( )
2 2 2
x2 − x + 1 y2 − y + 1 z2 − z + 1

( ) x4 + 1
2
HD: Sử dụng bất đẳng thức: x 2 − x + 1 ≥ , ∀x ∈  .
2

(
y4 + 1
) z4 + 1
( )
2 2
Tương tự ta có: y 2 − y + 1 ≥ và z 2 − z + 1 ≥ .
2 2
2 x4 y 4

n
2 2 2 2 2
Suy ra P ≤ 4 + 4 + 4 = 4 + 4 + 4 4 .
x +1 y +1 z +1 x +1 y +1 x y +1

.v
2 2 2 x4 y 4
+ + ≤ 3.

om
Ta sẽ chứng minh
x4 + 1 y 4 + 1 x4 y 4 + 1
Biến đổi tương đương ta thư được:
( )(
⇔ 2 x4 y 4 − 1 x4 − 1 y 4 − 1 ≥ 0 . )( ) .c
Bất đẳng thức này luôn đúng do x, y ≥ 1 .
ok
Vậy giá trị lớn nhất của P bằng 3 , khi và chỉ khi x= y= z= 1 .
Bài toán 5. Chứng minh rằng với mọi số thực x,y cùng dấu ta có
bo

k2 + x + k2 + y ≥ k + k2 + x + y .
et

Chứng minh.
Bình phương hai vế đưa về chứng minh bất đẳng thức:
vi

2 (k 2
)( )
+ x k 2 + y ≥ 2k k 2 + x + y
ng

⇔ (k 2
+ x )( k 2
+ y) ≥ k (k 2 2
)
+ x + y ⇔ xy ≥ 0
Bất đẳng thức cuối luôn đúng. Đẳng thức xảy ra khi và chỉ khi x = 0 hoặc
a

y = 0.
kh

Hay được dùng: 1 + x + 1 + y ≥ 1 + 1 + x + y .


Ta có một kết quả cùng dạng
Với mọi số thực không âm a,b,c thỏa mãn điều kiện a + b + c =
1 ta có
a + b2 + b + c2 ≥ a + b + c + b2 .

 CÁC VÍ DỤ
Ví dụ 1. Cho a,b,c là các số thực không âm có tổng bằng 1. Chứng minh
4a + 1 + 4b + 1 + 4c + 1 ≥ 5 + 2 .

579
Khám phá tư duy Kỹ thuật giải bất ĐT Bài toán Max – Min – Đặng Thành Nam

Lời giải
Ta có
1 + x + 1 + y ≥ 1 + 1 + x + y với mọi x,y không âm.
1
giả sử a max {a, b, c} ⇒ a ≥
Không mất tính tổng quát = .
3
Khi đó
4b + 1 + 4c + 1 ≥ 1 + 1 + 4a + 4b = 1 + 5 − 4a .
Gọi P là biểu thức vế trái ta có
P ≥ 4a + 1 + 1 + 5 − 4a .

n
1 
Xét hàm số f (a= 4a + 1 + 1 + 5 − 4a liên tục trên đoạn  ;1 ta có

.v
)
3 

om
2 2 1
f '(a ) = − ; f '(a ) = 0 ⇔ 5 − 4a = 4a + 1 ⇔ a = .
4a + 1 5 − 4a 2
1 7 11  1 
Ta có f   =
1+ + ;f  =1 + 2 3; f (1) =5 + 2 . .c
3 3 3 2
ok
Vì vậy P ≥ f min =f (1) = 5 + 2 .
Bất đẳng thức được chứng minh. Đẳng thức xảy ra khi và chỉ khi a = 1,
bo

b= c= 0 hoặc các hoán vị.


Ví dụ 2. Cho a,b,c là các số thực không âm thay đổi thỏa mãn điều kiện
et

1 + a 2 + 1 + 2b + 1 + 2c =
5.
Tìm giá trị lớn nhất của biểu thức P = 2a3 + b3 + c3 .
vi

Lời giải
ng

Theo giả thiết và sử dụng bài toán phụ trên ta có


1 + a 2 + 1 + 2b + 1 + 2c ≥ 1 + a 2 + 1 + 1 + 2b + 2c
a

≥ 1 + 1 + 1 + a 2 + 2b + 2c
kh

=2 + 1 + a 2 + 2b + 2c
a2
Suy ra: 1 + a 2 + 2b + 2c ≤ 3 ⇔ a 2 + 2b + 2c ≤ 8 ⇔ b + c ≤ 4 − .
2
3
 a2 
Ta có: P = 2a + ( b + c ) − 3bc ( b + c ) ≤ 2a + ( b + c )
3 3
3 3
≤ 2a +  4 −  .
3
 2 

Từ giả thiết a,b,c không âm thỏa mãn điều kiện 1 + a 2 + 1 + 2b + 1 + 2c =
5
suy ra 5 ≥ 1 + a 2 + 1 + 1 ⇒ a ≤ 2 2 ⇒ 0 ≤ a ≤ 2 2 .

580
Cty TNHH MTV DVVH Khang Việt

3
 a2 
Xét hàm số f (a ) = 2a +  4 −  liên tục trên 0;2 2  ta được:
3
 2 

2
 a2 
 2 
3
4  ( ) (
f '(a ) = 6a − 3a  4 −  = a ( a − 2 )  a 12 − a 2 + 2 16 − a 2  ;
2
 )
 .
0≤ a ≤ 2 2  a = 0
f '(a )= 0 ← →
a = 2 2
Ta có: f (0) =
64, f (2) =
24, f (2 2) =
32 2 ⇒ max f (a) =
f(0) =
64 .
a∈0;2 2 

n
Vậy giá trị lớn nhất của P bằng 64 đạt tại=
a 0,= = 4 hoặc=
b 0,c a 0,=
b 4,=
c 0.

.v
Bài tập tương tự

om
Cho x,y,z là các số thực không âm thoả mãn điều kiện 1 + x + 1 + y + 1 + z =
5.
Tìm giá trị lớn nhất của biểu thức P = x + y − 2 z + 12 1 + z .
Ví dụ 3. Cho x,y,z là các số thực không âm thoả mãn điều kiện x 2 + y 2 + z 2 =
.c 1.

( x + y )2 ( y + z )2 ( z + x) 2
ok
Chứng minh rằng 1− + 1− + 1− ≥ 6.
4 4 4
Lời giải
bo

Trong ba số
( x + y )2 − 1 ; ( y + z )2 − 1 ; ( z + x )2 − 1 luôn tồn tại hai số cùng
4 3 4 3 4 3
et

 ( x + y )2 1   ( z + x )2 1 
dấu, không mất tính tổng quát giả sử  −  − ≥0.
vi

 4 3  4

3

ng

2  ( x + y )2 1  2  ( z + x) 2 1 
Ta có − − + − − 
3  4 3  3  4 3 
a

2  ( x + y )2 1 ( z + x)2 1 
kh

6
≥ + − − + − 
3 3  4 3 4 3 

4 2x + 2x ( y + z ) + y + z
2 2 2
6
= + −
3 3 4


6
+ −
2 2 2 2
(
4 2x + 2x 2 y + z + y + z
2
)
3 3 4

581
Khám phá tư duy Kỹ thuật giải bất ĐT Bài toán Max – Min – Đặng Thành Nam

2 2
4  y + z + 2 x  4  1 − x2 + 2 x 
2 2
6 6
= + − = + −  .
3 3  2  3 3  2 
   
Sử dụng bất đẳng thức AM – GM ta có

1−
( y + z )2 ≥ 1−
y2 + z2
=
1 + x2
.
4 2 2
2
4  1 − x2 + 2 x  1 + x2 2 6
Vậy ta chỉ cần chứng minh −  + ≥ .
3  2  2 3
 

n
Bất đẳng thức cuối luôn đúng(bạn đọc kiểm chứng).

.v
1
Bất đẳng thức được chứng minh. Đẳng thức xảy ra khi và chỉ khi x= y= z= .

om
3
Ví dụ 4. Cho a,b,c là các số thực thuộc đoạn [ −1;1] và thoả mãn điều kiện
a+b+c =0. .c
Tìm giá trị nhỏ nhất của biểu thức P = 1 + a + b 2 + 1 + b + c 2 + 1 + c + a 2 .
ok
Lời giải
Ta có: a + b 2 , b + c 2 , c + a 2 ≥ −1 và trong 3 số này có ít nhất hai số hạng cùng
bo

( )(
dấu, không mất tính tổng quát ta giả sử a + b 2 b + c 2 ≥ 0 . )
Khi đó áp dụng bài toán phụ ta được
et

1 + a + b2 + 1 + b + c2 + 1 + c + a 2 ≥ 1 + 1 + a + b2 + b + c2 + 1 + c + a 2
vi

=+
1 1 + b2 + c2 − c + 1 + c + a 2
ng

( ) +b ( )
2 2
=1 + c2 − c + 1 2
+ 1+ c + a2
a

( ) + (a + b)
2
2
≥1+ c2 − c + 1 + 1 + c
kh

( 1+ c ) + c
2
=1 + c2 − c + 1 + 2

Rút gọn vế phải ta được: P ≥ 1 + 2c 2 + 2 1 − c3 + 2 .

Ta chứng minh 1 + 2c 2 + 2 1 − c3 + 2 ≥ 3
⇔ 2c 2 + 2 1 − c3 + 2 ≥ 4 ⇔ 1 − c3 ≥ 1 − c 2

582
Cty TNHH MTV DVVH Khang Việt

( )
⇔ 1 − c3 ≥ 1 − 2c 2 + c 4 ⇔ c 2 c 2 + c − 2 ≤ 0 ⇔ c 2 ( c − 1)( c + 2 ) ≤ 0 (luôn đúng

với c ∈ [ −1;1] ).
Vậy giá trị nhỏ nhất của P bằng 3 đạt tại a= b= c= 0 .
Ví dụ 5. Cho a,b,c là các số thực không âm thay đổi thỏa mãn điều kiện
a+b+c = 1 . Tìm giá trị nhỏ nhất của biểu thức
P= a + b2 + b + c2 + c + a 2 .
Lời giải
Ta có:

n
( )
2
+ (a + b)
2
P ≥ a + b + c + b2 + c + a 2 ≥ a + b + c+ c

.v
=1 − c + 4c + (1 − c ) =1 − c + ( c + 1)2
2
=2

om
1
Vậy giá trị nhỏ nhất của P bằng 2 đạt tại a= b= c= hoặc a= 1, b= c= 0 và
3
các hoán vị. .c
Ví dụ 6. Chứng minh rằng với x,y là hai số thực không âm thỏa mãn x + y ≥ 1 ta
ok
luôn có x2 + x + 4 + y 2 + y + 4 ≤ 2 + ( x + y )2 + x + y + 4 .
bo

Lời giải
Bất đẳng thức đã cho tương đương với
x2 + y 2 + x + y + 8 + 2 (x 2
)(
+ x + 4 y2 + y + 4 )
et

≤4+4 ( x + y )2 + x + y + 4 + ( x + y )2 + x + y + 4
vi

(x )( ) ( x + y )2 + x + y + 4
ng

⇔ 2
+ x + 4 y 2 + y + 4 ≤ xy + 2

( )( )
⇔ x 2 + x + 4 y 2 + y + 4 ≤ x 2 y 2 + 4 xy ( x + y )2 + x + y + 4 + 4 ( x + y )2 + x + y + 4
a
kh

⇔ xy  4 ( x + y ) + x + y + 4 + x + y − 7  ≥ 0 (luôn đúng do x + y ≥ 1 ).
2
 
Ta có một số kết quả cùng dạng
+ Chứng minh rằng với mọi số thực không âm x,y ta luôn có
x2 − x + 1 + y 2 − y + 1 ≤ 1 + ( x + y )2 − x − y + 1 .
Đẳng thức xảy ra khi và chỉ khi x = 0 hoặc y = 0 .
+ Chứng minh rằng với mọi số thực không âm x,y ta luôn có
x2 + x + k 2 + y 2 + y + k 2 ≤ k + ( x + y )2 + x + y + k 2 .

583
Khám phá tư duy Kỹ thuật giải bất ĐT Bài toán Max – Min – Đặng Thành Nam

Đẳng thức xảy ra khi và chỉ khi x = 0 hoặc y = 0 .


Bài toán áp dụng
Cho a,b,c là các số thực không âm thỏa mãn điều kiện a + b + c =3.
Tìm giá trị lớn nhất và nhỏ nhất của biểu thức
P= a 2 + a + 4 + b2 + b + 4 + c2 + c + 4 .

Ví dụ 7. Cho x,y,z là các số thực không âm thoả mãn điều kiện xy + yz + zx > 0 .
Tìm giá trị nhỏ nhất của biểu thức
1 1
= P + + 4 z +1 + 2 x + 2y + 4 .

n
x( y + z ) + 2 z 2 y( x + 4 z)

.v
Lời giải

om
Sử dụng kết quả bài toán phụ ta có
 x y 
4 z + 1 + 2 x + 2 y +=
4 4  z + 1 + + + 1 
 4 2 .c
 x y 
≥ 4 1 + + + z + 1 
ok
 4 2 
=4 + 2 x + 2 y + 4 z + 4
bo

1 2 2 2 2
Chú ý= ≥ ;
x( y + z ) + 2 z 2 8 xy + 8 xz + 16 z 2 x + 2 y + 4z
et

1 2 2 2 2
=
và ≥ .
vi

y ( x + 4 z )) 2 2 y ( x + 4 z ) x + 2 y + 4 z
ng

4 2
Đặt t =x =2 y + 4 z > 0 ta suy ra: P ≥ + 2 t + 4 + 4.
t
a

4 2
Xét hàm f (t ) = + 2 t + 4 + 4 ta có ngay Min f (t )= 4 + 5 2 .
kh

t
Vậy giá trị nhỏ nhất của P bằng 4 + 5 2 đạt tại ( x, y, z ) = ( 2;1;0 ) .

 BÀI TẬP RÈN LUYỆN

Bài 1. Cho x,y,z là các số thực không âm thỏa mãn điều kiện
x 2 + 4 + 3 y + 4 + 3z + 4 =
8.

(
Tìm giá trị lớn nhất của biểu thức P =2 x3 + 9 y 3 + z 3 . )
584
Cty TNHH MTV DVVH Khang Việt

HD: Dùng bất đẳng thức phụ đánh giá


8= x 2 + 4 + 3 y + 4 + 3z + 4 ≥ x 2 + 4 + 2 + 4 + 3 y + 3z
12 − x 2
≥ 4 + 4 + x 2 + 3 y + 3z ⇒ y + z ≤
3
1
( )
3
⇒ P ≤ 2 x3 + 9 ( y + z ) ≤ 2 x3 +
3
12 − x 2 ≤ 576
3
Bài 2. Cho x,y,z là các số thực không âm thoả mãn điều kiện
1+ x + 1+ y + 1+ z =
5.
Tìm giá trị lớn nhất của biểu thức P = x + y − 2 z + 12 1 + z .

n
Bài 3. Cho x,y,z là các số thực không âm thoả mãn điều kiện

.v
1 + 2 x2 + 1 + 2 y 2 + 1 + 2 z 2 =
5.

om
Chứng minh rằng 4 2 x3 + y 6 + z 6 ≤ 64 .
Bài toán 6. Cho x,y,z là các số thực không âm thỏa mãn điều kiện
max { x, y, z}
xy + yz + zx > 0, z =
.c
ok
x y x+ y
Ta luôn có + ≥ .
y+z z+x z
bo

Chứng minh.
Thật vậy ta có thể giả sử x ≤ y . Nếu x = 0 bất đẳng thức trở thành đẳng thức.
et

Với z ≥ y ≥ x > 0 khi đó viết lại bất đẳng thức dưới dạng:
vi

xz yz
+ ≥1.
( x + y )( y + z ) ( x + y )( x + z )
ng

Ta có
a

xz 2 xz 2 xz 2 xz
= ≥ =
( x + y )( y + z ) 2 z ( x + y ) .x ( y + z ) z ( x + y ) + x ( y + z ) xy + yz + 2 zx
kh

.
yz 2 yz 2 yz 2 yz
= ≥ =
( y + z )( x + y ) 2 y ( x + z ) .z ( x + y ) y ( x + z ) + z ( x + y ) xy + xz + 2 yz
Mặt khác xy + xz + 2 yz ≥ xy + yz + 2 zx suy ra
xz yz 2 xz 2 yz 2 ( xz + yz )
+ ≥ + = ≥ 1.
( x + y )( y + z ) ( x + y )( x + z ) xy + yz + 2 zx xy + yz + 2 zx xy + yz + 2 zx
Vì yz ≥ xz .
Bất đẳng thức được chứng minh.
585
Khám phá tư duy Kỹ thuật giải bất ĐT Bài toán Max – Min – Đặng Thành Nam

Đẳng thức xảy ra khi và chỉ khi x= y= z hoặc x = 0 hoặc y = 0 .


Ngoài ra ta có một đánh giá yếu hơn như sau:
x y 2( x + y)
+ ≥ .
y+z z+x x+ y+z
Một bất đẳng thức có dạng tương tự
=
Cho x,y,z là các số thực không âm và z min { x, y, z} ; xy + yz + zx > 0 ta có

x y x+ y
+ ≥2 .
y+z z+x x + y + 2z

n
Chứng minh.

.v
Sử dụng bất đẳng thức Holder ta có
2
 y 

om

x
+ 
z + x 
(x 2
( y + z ) + y 2 ( z + x ) ) ≥ ( x + y )3
 y+z
.

x
+
y

( x + y) 3 .c
y+z z+x x 2
( y + z ) + y2 ( z + x)
ok
Chú ý sử dụng bất đẳng thức AM – GM ta có
x 2 ( y + z ) + y 2 ( z + x=
) xy ( x + y − 2 z ) + z ( x + y ) 2
bo

2
 x+ y
≤  ( x + y − 2 z ) + z ( x + y )2 .
et

 2 
( x + y )4 + 2 z ( x + y )2
vi

=
4
ng

x
+
y

( x + y )3 =
x+ y
Suy ra 2 .
y+z z+x ( x + y )4 + 2 z ( x + y )2 x + y + 2z
a

4
kh

Bất đẳng thức được chứng minh. Đẳng thức xảy ra khi và chỉ khi x = y .

 CÁC VÍ DỤ
Ví dụ 1. Cho x,y,z là các số thực không âm thỏa mãn điều kiện
max { x, y, z}
xy + yz + zx > 0, z =

x y z
Tìm giá trị nhỏ nhất của biểu thức P = + 2 + 33 .
y+z z+x x+ y

586
Cty TNHH MTV DVVH Khang Việt

Lời giải
x x
Sử dụng bất đẳng thức AM-GM ta có: +1≥ 2 .
y+z y+z
 x y  z
Suy ra P ≥ 2  +  + 33 −1 .
 y+z z+x x+ y

x y 2( x + y)
Ta chứng minh. + ≥ .
y+z z+x x+ y+z
Thật vậy nếu x = 0 hoặc y = 0 giả sử y = 0 bất đẳng thức trở thành:

n
x 2x
⇔ x ( x + z ) ≥ 4 x 2 z ⇔ x ( x − z ) ≥ 0 (luôn đúng).
2 2

.v
z x+z
Với x, y > 0 sử dụng bất đẳng thức AM-GM ta có:

om
x y x y 2x 2y 2( x + y)
+ = + ≥ + = .
y+z z+x x( y + z) y ( z + x) .c x+ y+z x+ y+z x+ y+z

4( x + y) z 4 z
Vậy P ≥ + 33 −1 = + 33 −1 .
ok
x+ y+z x+ y 1+
z x+ y
x+ y
bo

z  31 4
Đặt=
t 3 ,  t ≥  ⇒ P ≥ f (=
t) + 3t − 1 .
x+ y  2 1 + t3
et

4 1
Xét hàm số f (t=
) + 3t − 1 với t ≥ 3 ta có:
vi

1+ t 3 2

( )(
3 ( t − 1) t 2 + t − 1 t 3 + 2t + 1 )
ng

12t 2  1
f '(t ) =− +3= ; f '(t ) =0 ⇔ t =1, 
 t ≥ 3 
2 
(1 + t )
3 2
( )
2
1 + t3 
a

Ta có f’(t) đổi dấu từ âm sang dương khi đi qua t = 1 nên f(t) đạt cực tiểu tại
kh

4 . Đẳng thức xảy ra khi và chỉ khi=


t = 1 hay P ≥ f (t ) ≥ f (1) = y 0,=
x z.
Vậy giá trị nhỏ nhất của P bằng 4 đạt tại=
x z=
,y 0.
Bài tập tương tự
Cho x,y,z là các số thực không âm thỏa mãn điều kiện
max { x, y, z}
xy + yz + zx > 0, z =

x y z
Tìm giá trị nhỏ nhất của biểu thức P = 2 +2 + 33 .
y+z z+x x+ y

587
Khám phá tư duy Kỹ thuật giải bất ĐT Bài toán Max – Min – Đặng Thành Nam

Ví dụ 2. Cho x,y,z là các số thực không âm thoả mãn điều kiện xy + yz + zx > 0 .
48 x 48 y 48 z
Chứng minh rằng 1 + + 1+ + 1+ ≥ 15 .
y+z z+x x+ y
Lời giải
Không mất tính tổng quát giả sử z = min { x, y, z} ta có

x y x+ y
+ ≥2 .
y+z z+x x + y + 2z
Sử dụng bất đẳng thức Mincopski và bất đẳng thức trên ta được

n
2
48 x 48 y  x y 

.v
1+ + 1+ ≥ 4 + 48  + 
y+z z+x  y+z z + x 

om
.
48 ( x + y )
2
 x+ y 
≥ 4 + 48  2  = 2 1 +
 x + y + 2z .c  x + y + 2z

48 ( x + y ) z
Vậy ta chỉ cần chứng minh 2 1 + + 1 + 48 ≥ 15 .
ok
x + y + 2z x+ y
48 z
Bất đẳng thức khá đơn giản bằng cách đặt t = 1+ ,1 ≤ t ≤ 5 .
bo

x+ y
Bạn đọc tự chứng minh.
et

Bất đẳng thức được chứng minh. Đẳng thức xảy ra khi và chỉ khi x= y= z hoặc
=
x y=
vi

; z 0 hoặc các hoán vị.


Bài tập tương tự
ng

Cho x,y,z là các số thực dương chứng minh rằng


16 x 16 y 16 z
a

1+ + 1+ + 1+ ≥9.
y+z z+x x+ y
kh

Ví dụ 3. Xét các số thực a ≥ b ≥ c > 0 thoả mãn điều kiện a + b + c =


1.
a b 24
Tìm giá trị nhỏ nhất của biểu thức P = + + .
b+c c + a 5 5(a + b)
Lời giải
Theo giả thiết kết hợp bất đẳng thức phụ ta có

588
Cty TNHH MTV DVVH Khang Việt

a b 24
P= + +
b+c c + a 5 5(a + b)
a+b 24
≥2 + .
a + b + 2c 5 5 ( a + b )

1− c 24
= 2 +
1 + c 5 5 (1 − c )

1− c 24  1
Xét hàm số=
f (c ) 2 + trên nửa khoảng  0;  ta có
1 + c 5 5 (1 − c )  3

n
−10 5 (1 − c ) + 12 (1 + c ) 1 + c

.v
f '(c) = ;
(
5 5 1− c 2
) 1− c 2

om
f '(c) = 0 ⇔ −10 5 (1 − c ) + 12 (1 + c ) 1 + c = 0 .
⇔ 125 (1 − c ) = 36 (1 + c ) ⇔ 36c − 17c + 358c − 89= 0
2 3 3 2
.c
( )
⇔ ( 4c − 1) 9c 2 − 2c + 89 = 0 ⇔ c =
1
ok
4
1 78
Từ đó suy ra f (c) ≥ f   =.
bo

 4  5 15
3 1 78
Dấu bằng đạt tại a= b= , c= . Vậy giá trị nhỏ nhất của P bằng .
et

8 4 5 15
vi

Ví dụ 4(TSĐH Khối B/2014) Cho a,b,c là các số thực không âm thoả mãn điều
kiện c ( a + b ) > 0 . Tìm giá trị nhỏ nhất của biểu thức
ng

a b c
P= + + .
a

b+c c + a 2(a + b)
kh

Lời giải
a 2a b 2b
Chú ý ≥ ; ≥ .
b+c a+b+c c+a a+b+c
2(a + b) c 2 c
Từ đó suy ra P ≥ + = + .
a+b+c 2(a + b) 1 + c 2(a + b)
a+b
c
Đặt t
= , t > 0 khi đó sử dụng bất đẳng thức AM – GM ta có
a+b

589
Khám phá tư duy Kỹ thuật giải bất ĐT Bài toán Max – Min – Đặng Thành Nam

2 t 2 t +1 1 2 t +1 1 3
P≥ + = + − ≥2 . − = .
1+ t 2 t +1 2 2 t +1 2 2 2
Đẳng thức xảy ra khi và chỉ khi=
a c=
, b 0 hoặc=
a 0,=
b c.
Bài tập tương tự
Cho a,b,c là các số thực không âm thoả mãn điều kiện a 2 + b 2 + c 2 =
2.
1 + bc 1 + ca c
Tìm giá trị nhỏ nhất của biểu thức P = + + .
(b + c ) 2
(a + c) 2 2(a + b)

Ví dụ 5. Cho a,b,c là các số thực không âm thoả mãn điều kiện ab + bc + ca > 0 .

n
a b c
Tìm giá trị nhỏ nhất của biểu thức P = + +2 .

.v
b+c 2c + a a+b+c

om
Lời giải
a b c
Ta sẽ chứng minh: + +2 ≥ 2.
b+c 2c + a a+b+c
.c
+ Nếu c bằng 0 ta chỉ cần sử dụng bất đẳng thức AM – GM cho hai số dương có
điều phải chứng minh.
ok
+ Nếu a bằng 0 khi đó ta cần chứng minh
bo

b c b 1
+2 ≥2⇔ +2 ≥2
2c b+c 2c b
+1
c
et

2  b 
⇔ x+ ≥ 2,  =
x ≥ 0  .
vi

2 x2 + 1  2c 
( ) (
⇔ 4 ≥ ( 2 − x ) 2 x 2 + 1 ⇔ x 2 x3 − 8 x 2 + 9 x − 4 ≤ 0
2
)
ng

Bất đẳng thức cuối đúng với x ≤ 2 .


a

+ Nếu b bằng 0 chứng minh tương tự trường hợp trên.


kh

+ Ta xét tất cả các số đều dương khi đó sử dụng bất đẳng thức AM – GM ta có
a b c a b 2c
+ +2 = + +
b+c 2c + a a+b+c a (b + c ) b ( 2c + a ) c (a + b + c)
2a 2b 4c 2a + 2b + 4c
≥ + + > =
2
a + b + c a + b + 2c a + b + 2 c a + b + 2c
Bất đẳng thức được chứng minh. Đẳng thức xảy ra khi và chỉ khi=
a b=
,c 0 .
25
Ví dụ 6. Cho các số thực x, y, z ∈ [ 0;1) thỏa mãn điều kiện x + y + z = .
24

590
Cty TNHH MTV DVVH Khang Việt

x y z
Tìm giá trị nhỏ nhất của biểu thức P = + + .
1− x 1− y 1− z
Lời giải
25 25 2
Không mất tính tổng quát giả sử z = min { x, y, z} ⇒ z ≤ ⇒ x + y= −z> .
72 24 3
Sử dụng bất đẳng thức Holder ta có:
2
 y 

x
+ 
1 − y 
(x 2
(1 − x ) + y 2 (1 − y ) ) ≥ ( x + y )3 .
 1− x

n
x
+
y

( x + y )3

.v
Suy ra .
1− x 1− y x 2 (1 − x ) + y 2 (1 − y )

om
Chú ý. Sử dụng bất đẳng thức AM – GM ta có
x 2 (1 − x ) + y 2 (1 − y ) = x 2 + y 2 − x3 − y 3
= ( x + y ) − ( x + y ) + xy ( 3 ( x + y ) − 2 )
2 3 .c
2
3  x+ y
ok
.
≤ ( x + y) − ( x + y) +   (3( x + y ) − 2)
2

 2 
bo

1 1
= ( x + y) − ( x + y)
2 3
2 4

( x + y )3
et

x y 25 − 24 z
Suy ra + ≥ =
2 .
1− x 1− y 1 1 23 + 24 z
( x + y )2 − ( x + y )3
vi

2 4
ng

25 − 24 z z
Khi đó P ≥=
f ( z) 2 + .
23 + 24 z 1− z
a

25 − 24 z z  25 
=
Xét hàm số f ( z ) 2 + trên 0;  ta có
23 + 24 z 1− z
kh

 72 
1 1152
=f '( z ) − ;
z 25 − 24 z
2 ( z − 1) ( 23 + 24 z )
2 2
.
1− z 23 + 24 z
0 ⇔ 23042 (1 − z ) z =
( 23 + 24 z ) ( 25 − 24 z )
3 3
f '( z ) =

 25  25
Phương trình cuối có 2 nghiệm thuộc đoạn 0;  đó là 0 < z1 < z2 = .
 72  72

591
Khám phá tư duy Kỹ thuật giải bất ĐT Bài toán Max – Min – Đặng Thành Nam

10
Lập bảng biến thiên suy ra min f=
( z ) f=
(0) .
23
10 25
Vậy giá trị nhỏ nhất của P bằng đạt tại x= y= , z= 0 hoặc các hoán vị.
23 48
Bài tập tương đương tự
1) Cho a,b,c là các số thực không âm thoả mãn điều kiện
1 1 11
+ + = .
a + 47
2
b + 47
2
c + 47 24
2

47

n
Chứng minh rằng a + b + c ≥ 10 .
23

.v
1 1− x 1 1− y 1 1− z
HD: Tồn tại x, y, z ∈ [ 0;1) sao =
cho 2 = ; 2 = ; 2 .

om
a + 47 47 b + 47 47 c + 47 47
25 x y z
Khi đó x + y +=
z ;=
P + + .
24 1− x 1− y 1− z .c
Đây chính là bài toán trên.
ok
1 1 1 1
2) Cho các số thực a, b, c ≥ 1 và + + = .
a + 47 2
b + 47
2
c + 47 24
2
bo

Chứng minh rằng a + b + c ≥ 15 .


et

 BÀI TẬP RÈN LUYỆN


Bài 1. Cho x,y,z là các số thực không âm thỏa mãn điều kiện
vi

max { x, y, z}
xy + yz + zx > 0, z =
ng

x y z
Tìm giá trị nhỏ nhất của biểu thức P = 2 +2 + 33 .
y+z z+x x+ y
a

x y 2( x + y)
+ ≥
kh

HD: Ta chứng minh. .


y+z z+x x+ y+z
Thật vậy nếu x = 0 hoặc y = 0 giả sử y = 0 bất đẳng thức trở thành:
x 2x
⇔ x ( x + z ) ≥ 4 x 2 z ⇔ x ( x − z ) ≥ 0 (luôn đúng).
2 2

z x+z
Với x, y > 0 sử dụng bất đẳng thức AM-GM ta có:
x y x y 2x 2y 2( x + y)
+ = + ≥ + = .
y+z z+x x( y + z) y ( z + x) x+ y+z x+ y+z x+ y+z

592
Cty TNHH MTV DVVH Khang Việt

4( x + y) z 4 z
Vậy P ≥ + 33 = + 33 .
x+ y+z x + y 1+ z x+ y
x+ y

z  31 4
Đặt=
t 3 ,  t ≥  ⇒ P ≥ f (=
 t) + 3t .
x+ y  2 1 + t3

4 1
Xét hàm số =
f (t ) + 3t với t ≥ 3 ta có:
1+ t 3 2

f '(t ) =−
12t 2
+3=
( )(
3 ( t − 1) t 2 + t − 1 t 3 + 2t + 1 )  1
; f '(t ) =0 ⇔ t =1,  t ≥ 3  .

n
(1 + t )
3 2
( )
2 2
1 + t3 

.v
Ta có f’(t) đổi dấu từ âm sang dương khi đi qua t = 1 nên f(t) đạt cực tiểu tại

om
5 . Đẳng thức xảy ra khi và chỉ khi=
t = 1 hay P ≥ f (t ) ≥ f (1) = y 0,=
x z.
Vậy giá trị nhỏ nhất của P bằng 5 đạt tại=
x z=
,y 0.
.c
Bài 2. Cho x,y,z là các số thực không âm thoả mãn điều kiện xy + yz + zx > 0 .
ok
x y z xyz
Chứng minh rằng + + ≥ 2 1+ .
y+z z+x x+ y ( x + y )( y + z )( z + x )
HD: Giả sử z = min { x, y, z} khi đó ta có
bo

x y x+ y
+ ≥2
et

.
y+z z+x x + y + 2z
Chú ý sử dụng bất đẳng thức AM – GM ta có
vi

( x + y) . 2
ng

xy 1 1
= ≤ =
( x + z )( y + z ) z ( x + y + z ) + 1 4 z ( x + y + z ) + 1 ( x + y + 2 z )2
xy ( x + y )2
a

z ( x + y)
kh

x+ y z
Do vậy ta chỉ cần chứng minh 2 + ≥ 2 1+ .
x + y + 2z x+ y ( x + y + 2 z )2
z
Đặt t = đưa về chứng minh bất đẳng thức một biến.
x+ y

Bài 3. Cho a,b,c là các số thực không âm thoả mãn điều kiện a 2 + b 2 + c 2 =
2.
1 + bc 1 + ca c
Tìm giá trị nhỏ nhất của biểu thức P = + + .
(b + c ) 2
(a + c) 2 2(a + b)

593
Khám phá tư duy Kỹ thuật giải bất ĐT Bài toán Max – Min – Đặng Thành Nam

HD: Chú ý
a b c 3
1 + bc ≥ a ( b + c ) ;1 + ca ≥ b ( a + c ) ⇒ P ≥ + + ≥ .
b+c c + a 2(a + b) 2

x2 xn
Bài toán 7. Chứng minh rằng với mọi số thực x ta có e x ≥ 1 + x + + ... + .
2! n!
Đẳng thức xảy ra khi và chỉ khi x = 0 .
Chứng minh. Xem chủ đề 1 Khảo sát hàm một biến.
 x2 xn 
Từ bất đẳng thức trên ta có x ≥ ln 1 + x + + ... +  .
 n1 

n
 2!

.v
Thông thường sử dụng bất đẳng thức x ≥ ln(1 + x), ∀x ≥ 0 .
x2 xn

om
Chú ý. Với x là số thực không âm và a ≥ e ta có a x ≥ 1 + x +
+ ... + .
2! n!
Ví dụ 1(TSĐH Khối A,A1/2012) Cho x,y,z là ba số thực thỏa mãn điều kiện
x+ y+z= 0 . Tìm giá trị nhỏ nhất của biểu thức
.c
x− y y−z z−x
ok
P= 3 +3 +3 − 6 x2 + 6 y 2 + 6 z 2 .
Lời giải
bo

Nhận xét. Biểu thức P và điều kiện các biến có vai trò như nhau nên dự đoán dấu
bằng xảy ra tại x= y= z suy ra x= y= z= 0 khi đó P = 3 . Vậy ta đi chứng
et

minh P không nhỏ hơn 3.


Thay z =− x − y vào P ta được:
vi

x− y x+2 y y+2 x
− 6 x2 + 6 y 2 + 6 ( − x − y ) .
2
P =3 +3 +3
ng

Suy ra :

P ≥ x − y + 1 + x + 2 y + 1 + y + 2 x + 1 − 6 x2 + 6 y 2 + 6 ( x + y )
a

2
.
kh

≥ 4 x + 2 y + 3 − 12 x 2 + 12 y 2 + 12 xy
Ta chứng minh P ≥ 3 vậy ta đi chứng minh:

4 x + 2 y + 3 − 12 x 2 + 12 y 2 + 12 xy ≥ 3 ⇔ 2 x + y ≥ 3x 2 + 3 y 2 + 3xy .

⇔ 4 x 2 + 4 xy + y 2 ≥ 3 x 2 + 3 y 2 + 3 xy
.
⇔ x 2 + xy − 2 y 2 ≥ 0 ⇔ ( x − y )( x + 2 y ) ≥ 0

594
Cty TNHH MTV DVVH Khang Việt

Để bất đẳng thức cuối đúng nếu ta có x ≥ y ≥ z ⇒ 2x + y ≥ 0 và


x + 2 y =x + y + y =− z + y ≥ 0 (chỉ việc giả sử x ≥ y ≥ z từ đầu) do đó bất đẳng
thức được chứng minh tức P ≥ 3 .
Vậy giá trị nhỏ nhất của P bằng 3 đạt tại x= y= z= 0 .
Nhận xét. Như vậy bằng việc dự đoán dấu bằng xảy ra và biến đổi bất đẳng thức
tương đương ta nghĩ đến việc giả sử x ≥ y ≥ z đây chính là nút thắt đưa đến lời giải
hết sức tự nhiên như trên.
Ví dụ 2. Cho x,y,z là các số thực dương thỏa mãn điều kiện xy + yz + zx =
3.
Tìm giá trị nhỏ nhất của biểu thức

n
x2 y 2 + y 2 z 2 + z 2 x2 x2 + y 2 + z 2

.v
x− y y−z z−x
P = 10 + 10 + 10 −4 − .
3 3( x + y + z )

om
Lời giải

( )
Ta có: x 2 y 2 + y 2 z 2 + z 2 x 2 ≤
1 2
3
x + y2 + z2
.c 2
ok
⇒ −4
x2 y 2 + y 2 z 2 + z 2 x2
3
4
(
≥ − x2 + y 2 + z 2
3
)
bo

x2 + y 2 + z 2 x2 + y 2 + z 2
x + y + z ≥ 3 ( xy + yz + zx ) = 3 ⇒ − ≥− .
3( x + y + z ) 9
et

Suy ra:
vi

P ≥ 10
x− y
+ 10
y−z
+ 10
z−x

13 2
9
(
x + y2 + z2 )
ng

13
− ( x − y ) + ( y − z ) + ( z − x ) + 6  .
x− y y−z z−x 2 2 2
= 10 + 10 + 10
18  
a

13 13
− ( x − y ) + ( y − z ) + ( z − x )  −
x− y y−z z−x 2 2 2
= 10 + 10 + 10
18   3
kh

13 2
Xét hàm số f (=
t ) 10t − t với t ≥ 0 ta có
18
13 13 13
f=
'(t ) 10t ln10 − t; f =
''(t ) 10t ln 2 10 − > ln 2 10 − > 0 nên f’(t) là hàm
9 9 9
đồng biến trên [0; +∞ ) ⇒ f '(t ) ≥ f '(0) = 0 nên f(t) là hàm đồng biến trên
[0;+∞ ) do đó f (t ) ≥ f(0) =
1.

595
Khám phá tư duy Kỹ thuật giải bất ĐT Bài toán Max – Min – Đặng Thành Nam

13 2
Hay 10t − t ≥ 1, ∀t ≥ 0 .
18
Áp dụng với t =−
x y , y − z , z − x ta có:
13 
10
x− y
+ 10
y−z
+ 10
z−x
− ( x − y )2 + ( y − z )2 + ( z − x )2  ≥ 3 .
18 
4
Do đó giá trị nhỏ nhất của P bằng − đạt tại x= y= z= 1 .
3
Ví dụ 3. Cho x,y,z là các số thực thỏa mãn điều kiện x + y + z =0.
Tìm giá trị nhỏ nhất của biểu thức

n
(
P = 2 x − y + 2 y − z + 2 z − x − ln  14 x 2 + y 2 + z 2 + 1 . )

.v
 

om
Lời giải
Nhận xét. Dự đoán dấu bằng đạt tại x = y = z ⇒ x = y = z = 0 ⇒ P = 0 .
Vậy ta chứng minh P đạt giá trị nhỏ nhất bằng 0. .c
Thay z =− x − y vào biểu thức của P ta được:
ok

( 
P= 2 x − y + x + 3 y + 2 y + 3 x − ln  14 x 2 + y 2 + ( − x − y ) + 1

2


)
bo


(
≥ 2 x − y + x + 3 y + 2 y + 3 x − ln  28 x 2 + y 2 + xy + 1

. )
(
= 6 x + 4 y − ln  28 x 2 + y 2 + xy + 1 )
et

 
vi

(
Ta cần chứng minh: 6 x + 4 y − ln  28 x 2 + y 2 + xy + 1 ≥ 0 .
 
)
ng

(
⇔ 6 x + 4 y ≥ ln  28 x 2 + y 2 + xy + 1 .
 
)
a

Sử dụng bất đẳng thức quen thuộc: ln ( x + 1) ≤ x, ∀x ≥ 0 ta có:


kh

( ) (
ln  28 x 2 + y 2 + xy + 1 ≤ 28 x 2 + y 2 + xy .
 
)
Vậy nhận định được chứng minh nếu ta chứng minh được bất đẳng thức sau đúng:

( )
6 x + 4 y ≥ 28 x 2 + y 2 + xy ⇔ 36 x 2 + 48 xy + 16 y 2 ≥ 28 x 2 + 28 y 2 + 28 xy .

⇔ 8 x 2 + 20 xy − 12 y 2 ≥ 0 ⇔ ( x + 3 y )( 8 x + 4 y ) ≥ 0 .

596
Cty TNHH MTV DVVH Khang Việt

Bất đẳng thức cuối luôn đúng nếu ta có xy ≥ 0 . Thật vậy với ba số x,y,z luôn có
hai số cùng dấu và bài toán kết thúc nếu ta giả sử từ đầu x, y cùng dấu.
Vậy giá trị nhỏ nhất của P bằng 0 đạt tại x= y= z= 0 .

Ví dụ 4. Cho a,b là hai số thực dương thoả mãn điều kiện a + b = a 4 + b 4 .


3 3
Chứng minh rằng a a bb ≤ 1 ≤ a a bb .
Lời giải
Bất đẳng thức đã cho tương đương với: a ln a + b ln b ≤ 0 ≤ a3 ln a + b3 ln b .
Để chứng minh bất đẳng thức vế trái ta sử dụng bất đẳng thức

n
ln x ≤ x − 1, ∀x > 0 ta có a ln a + b ln b ≤ a ( a − 1) + b ( b − 1) = a 2 + b 2 − a − b .

.v
Vậy ta chỉ cần chứng minh a 2 + b 2 ≤ a + b .

om
Thật vậy sử dụng bất đẳng thức Holder ta có

(a ) ( )
3
+ b4 ( a + b ) ≥ a 2 + b2
2
4
⇒ a 2 + b2 ≤ a + b .
.c
Ta có điều phải chứng minh. Đẳng thức xảy ra khi và chỉ khi a= b= 1 .
ok
Chứng minh bất đẳng thức vế phải
Theo giả thiết kết hợp sử dụng bất đẳng thức AM – GM ta có
bo

1
a + b = a 4 + b4 ≥ ( a + b )4 ⇒ a + b ≤ 2 ⇒ a, b ∈ ( 0;2 ) .
8
et

x − x4
Khi đó xét hàm số f=
( x) 3ln x + trên khoảng (0;2) ta có
x3
vi

3
2
(
2 ( x − 1) − x + 2 x + 2 )
; f '( x) = 0 ⇔ x = 1∈ ( 0;2 ) .
ng

f '( x) = − 1 − 3 =
x x x3
Khi đó f’(x) đổi dấu từ âm sang dương khi đi qua x = 1 nên f(x) đạt cực tiểu tại
a

x =1.
kh

x4 − x
Vì vậy f ( x) ≥ f (1) =
0 ⇔ x3 ln x ≥ .
3
a 4 − a b4 − b
Do đó a3 ln a + b3 ln b ≥ + =0.
3 3
Bất đẳng thức được chứng minh. Đẳng thức xảy ra khi và chỉ khi a= b= 1 .

597
Khám phá tư duy Kỹ thuật giải bất ĐT Bài toán Max – Min – Đặng Thành Nam

 BÀI TẬP RÈN LUYỆN


Bài 1. Cho a,b,c là các số thực thay đổi thỏa mãn a, b ≥ 0, c > 0 và a + b + c =
1.
Tìm giá trị lớn nhất của biểu thức

=P
1
ln 1 + a 2 +
1
(ln 1 + b 2 +) ( )
(
8 1 − c2 + 1 ).
2013 2014 3c
HD:
Do a, b ≥ 0, c > 0, a + b + c =1 ⇒ a, b ∈ [ 0;1] , c ∈ ( 0;1] .
Sử dụng kết quả bài toán phụ ta có

n
1
(
ln 1 + a 2 +
1
)
ln 1 + b 2 ≤
1 2
a + (1 2
b )

.v
2013 2014 2013 2014
.
1 1 4 4 4
≤ a+ b ≤ a + b = (1 − c )

om
2013 2014 3 3 3

4
Do đó: P ≤ (1 − c ) +
(
8 1 − c2 + 3 ). .c
3 3c

(
8 1 − c2 + 3 ) liên tục trên ( 0;1] , ta có:
ok
4
Xét hàm số g (c) = (1 − c ) +
3 3c
bo

2
c
− c2 + 3
− − 2 − . c +3 2
4 8 8 2 4 8 8
g '(c) = =− − 2+ .
et

3 3c 3 c 3 3c c c2 + 3
2
vi

24 − 4c 2 c 2 + 3 − 8 c 2 + 3
g '(c) ≥ 0, ∀c ∈ ( 0;1] .
3c 2 c 2 + 3
ng

8
Nên g (c) là hàm đồng biến trên ( 0;1] . Suy ra g (c) ≤ g (1) =
− .
3
a

8
kh

Vậy giá trị lớn nhất của P bằng − khi a= b= 0, c= 1 .


3
Bài 2. Cho x,y,z là các số thực. Tìm giá trị nhỏ nhất của biểu thức

P= 3
x− y
+3
y−z
+3
z−x

( x − y) + ( y − z ) + ( z − x)
2 2 2
−2 x 2 + y 2 + z 2 − xy − yz − zx .
2
x2
HD: Sử dụng bất đẳng thức a x ≥ 1 + x + , ∀x, a ≥ e ta có
2

3
x− y
+3
y−z
+3
z−x

( x − y) + ( y − z ) + ( z − x)
2 2 2
≥ 3+ x− y + y−z + z−x .
2
598
Cty TNHH MTV DVVH Khang Việt

Do đó P ≥ 3 + x − y + y − z + z − x − 2 x 2 + y 2 + z 2 − xy − yz − zx .

Ta chứng minh x − y + y − z + z − x − 2 x 2 + y 2 + z 2 − xy − yz − zx ≥ 0 .
Không mất tính tổng quát giả sử x ≥ y ≥ z bất đẳng thức trở thành

1 1 1
( x − y) + ( y − z) + ( x − z) ≥ 2 ( x − y )2 + ( y − z )2 + ( z − x )2
2 2 2

(
⇔ 2( x − z ) ≥ 2 ( x − y) + ( y − z ) + ( z − x)
2 2
)
2

⇔ 4( x − z ) ≥ 2 (( x − y ) + ( y − z) + ( z − x) )
2 2 2 2

n
.v
⇔ ( x − z) ≥ ( x − y) + ( y − z)
2 2 2
.
⇔ ( ( x − y ) + ( y − z ) ) ≥ ( x − y ) + ( y − z ) ⇔ 2 ( x − y )( y − z ) ≥ 0

om
2 2 2

Bất đẳng thức cuối luôn đúng ta có đpcm. Suy ra P ≥ 3 .


Vậy giá trị nhỏ nhất của P bằng 3 đạt tại x= y= z . .c
Bài 3. Cho x,y,z là các số thực không âm thỏa mãn điều kiện
ok
( x + y )2 + ( y + z )2 + ( z + x )2 ≤ 18 .
x y z
bo

1
Tìm giá trị lớn nhất của biểu thức P = 4 3 +4 3 +4 3 − ( x + y + z )4 .
108
et

x
Bài 3. HD : Ta chứng minh 4 3 ≤ x + 1, ∀ x ∈ [ 0;3] .
vi

x
Xét hàm số f ( x) = 4 3 − x − 1 với x ∈ [ 0;3] ta có
ng

x
1 3
f '( x) = ln 4.4 3 − 1; f '( x) = 0 ⇔ x = x0 = 3log 4 .
a

3 ln 3
Ta có f’(x) đổi dấu từ âm sang dương khi đi qua x0 nên f(x) đạt cực tiểu tại x0 .
kh

x
Do đó f ( x) ≤ max { f (0); f (3)} 0 hay 4 ≤ x + 1 với
= 3 mọi x ∈ [ 0;3] .
Đẳng thức xảy ra khi và chỉ khi x = 0 hoặc x = 3 .

Suy ra P ≤ ( x + y + z ) + 3 −
( x + y + z )4 .
108

599
Khám phá tư duy Kỹ thuật giải bất ĐT Bài toán Max – Min – Đặng Thành Nam

2
Bài toán 7. Cho x,y là hai số thực không âm thỏa mãn x + y ≤ ta luôn có
3
1− x 1− y 1− ( x + y)
+ ≤1+ .
1+ x 1+ y 1+ ( x + y)
Đẳng thức xảy ra khi và chỉ khi x = 0 hoặc y = 0 .
Chứng minh.
Bất đẳng thức đã cho tương đương với:
1− x 1− y (1 − x )(1 − y ) 1 1 − x − y 2 1− x − y
+ +2 ≤ + +
1+ x 1+ y (1 + x )(1 + y ) 1+ x + y 1+ x + y

n
.
 (1 − x )(1 − y ) 1− x − y 

.v
1− x − y 1− x 1− y
⇔ 2 −  ≤1+ − −

 (1 + x )(1 + y ) 1+ x + y 

1+ x + y 1+ x 1+ y

om
Chú ý
(1 − x )(1 − =y) 1− x − y

2 xy ( x + y )
≥0.
(1 + x )(1 + y ) 1 + x + y (1 + x )(1 + y )(1 + x + y )
.c
1− x − y 1− x 1− y 2 xy ( x + y + 2 )
Và 1 + − − = .
1 + x + y 1 + x 1 + y (1 + x )(1 + y )(1 + x + y )
ok
2 xy ( x + y )
bo

(1 − x )(1 − y ) 1 − x − y (1 + x )(1 + y )(1 + x + y )


Suy ra − =
(1 + x )(1 + y ) 1 + x + y (1 − x )(1 − y ) + 1 − x − y
et

(1 + x )(1 + y ) 1 + x + y
2 xy ( x + y )
vi


(1 + x )(1 + y )(1 + x + y )
ng

1− x − y
2
1+ x + y .
a

xy ( x + y )
=
kh

1− x − y
(1 + x )(1 + y )(1 + x + y )
1+ x + y
Vậy ta chỉ cần chứng minh
xy ( x + y ) xy ( x + y + 2 )

1− x − y (1 + x )(1 + y )(1 + x + y )
(1 + x )(1 + y )(1 + x + y )
1+ x + y .
1− x − y x+ y
⇔ ≥
1+ x + y x + y + 2

600
Cty TNHH MTV DVVH Khang Việt

Bất đẳng thức cuối luôn đúng do


1− x − y x+ y 1− 2 / 3 2/3 1 1
− ≥ − = − >0.
1+ x + y x + y + 2 1+ 2 / 3 2 / 3 + 2 5 4

 CÁC VÍ DỤ
Ví dụ 1. Cho x,y,z là các số thực không âm thỏa mãn điều kiện x + y + z =
1.
Tìm giá trị lớn nhất và nhỏ nhất của biểu thức
1− x 1− y 1− z
P= + + .
1+ x 1+ y 1+ z

n
Lời giải

.v
Tìm giá trị nhỏ nhất

( ( )) ≥ 0

om
Chú ý với x, y, z ∈ [ 0;1] ta có (1 − x ) 1 − 1 − x 2

1− x
⇒ ≥1− x . .c
1+ x
ok
1− y 1− z
Tương tự ta có ≥ 1 − y; ≥1− z .
1+ y 1+ z
bo

Cộng theo vế ba bất đẳng thức trên ta được:


1− x 1− y 1− z
=
P + + ≥ 3 − x − y=
− z 2.
et

1+ x 1+ y 1+ z
Đẳng thức xảy ra khi và chỉ khi x= 1, y= z= 0 hoặc các hoán vị.
vi

Cách 2: Nếu để ý ta có
ng

y+z z+x x+ y
P= + +
( x + y) + ( x + z) ( y + z) + ( y + z) ( z + x) + ( z + y )
a

.
2( y + z) 2( z + x) 2( x + y)
≥ + + =
kh

2
2( x + y + z) 2( x + y + z ) 2( x + y + z )
Tìm giá trị lớn nhất
1 2
Không mất tính tổng quát=giả sử z max { x, y, z} ⇒ z ≥ ; x + y ≤ .
3 3
Khi đó sử dụng bất đẳng thức phụ ta có
1− x 1− y 1− x − y z
+ ≤1+ =1+ .
1+ x 1+ y 1+ x + y z+2

601
Khám phá tư duy Kỹ thuật giải bất ĐT Bài toán Max – Min – Đặng Thành Nam

z 1− z
Suy ra P ≤ 1 + + .
z+2 1+ z
z 1− z 1 
Xét hàm số f ( z ) =
1+ + trên đoạn  3 ;1 ta có
z+2 1+ z  
1 1
=f '( z ) − ;
z 1− z
(2 − z) 2
(1 + z ) 2
2− z 1+ z .
1 1 
f '( z ) = 0 ⇔ (1 + z ) (1 − z ) = ( 2 − z ) z ⇔ z = ∈  ;1
3 3
2 3 

n
 1 1  1 2

.v
So sánh các giá trị suy ra max f (=
z ) max  f   ; f   ; f (1)=
 f= +1 .
 3  2  2 3

om
1
Đẳng thức xảy ra khi và chỉ khi x= 0, y= z= hoặc các hoán vị.
2
2 .c
Vậy giá trị lớn nhất của P bằng 1 + .
3
ok
Bài tập tương tự
bo

Cho x,y,z là các số thực không âm thoả mãn điều kiện


=z max { x, y, z=
}; x + y + z 1 .
et

1− x 1− y 2− z
Tìm giá trị lớn nhất của biểu thức P = + + .
1+ x 1+ y
vi

z
 BÀI TẬP RÈN LUYỆN
ng

Bài 1. Cho a,b,c là các số thực không âm thoả mãn điều kiện
1 1 1
+ = +
a

2.
1+ a 1 + b 1 + c2
2 2
kh

Tìm giá trị nhỏ nhất và lớn nhất của biểu thức P = a + b + c .
Bài toán 8. Cho x,y là hai số thực dương thỏa mãn x + y < 1 ta luôn có
2
 1  1   2 
 − 1  − 1 ≥  − 1 .
 x  y   x + y 
Đẳng thức xảy ra khi và chỉ khi x = y .
Chứng minh.
1 1 1 4 4
Bất đẳng thức đã cho tương đương với: − − ≥ −
xy x y ( x + y ) 2 x+ y
602
Cty TNHH MTV DVVH Khang Việt

1 4 1 1 4
⇔ − ≥ + −
xy ( x + y ) 2 x y x+ y

( x − y )2 ( x − y )2
⇔ ≥ .
xy ( x + y )
2 xy ( x + y )

⇔ ( x − y ) (1 − x − y ) ≥ 0
2

Đẳng thức xảy ra khi và chỉ khi x = y .

 CÁC VÍ DỤ

n
Ví dụ 1. Cho a,b,c là các số thực dương thỏa mãn điều kiện a + b + c =
1.

.v
 1  1  18
Tìm giá trị nhỏ nhất của biểu thức P=  − 1 − 1 + .

om
 a  b  a + b + 2c
Lời giải
 1  1  (1 − a )(1 − b ) 1 − ( a + b ) + ab c
.c
Ta có  − 1 − 1 = = = +1.
 a  b  ab ab ab
ok
2 2
 a + b  1− c 
Ta có ab ≤   =
  . Đẳng thức xảy ra khi và chỉ khi a = b .
 2   2 
bo

2
 1  1  c  c +1 c +1
Suy ra  − 1 − 1 ≥ = +1 =  .
 a  b  1− c 
2
1− c  1− c
et

 
 2 
vi

c +1 18 c + 1 18
Do đó P ≥ + = + .
1 − c a + b + 2c 1 − c 1 + c
ng

4 ( 2c − 1)
2
c + 1 18  c + 1 18 
Mặt khác + =  + − 15  + 15
= + 15 ≥ 15 .
a

1− c 1+ c 1− c 1+ c  1 − c2
kh

1 1− c 1
Đẳng thức xảy ra khi và chỉ khi c= , a= b= = .
2 2 4
1 1
Vậy giá trị nhỏ nhất của P bằng 15 đạt tại a= b= , c= .
4 2
Cách 2: Sử dụng bất đẳng thức phụ ta có
2 18 2 18
P≥ −1+ = + −1.
a+b a + b + 2c 1 − c 1 + c
Thực hiện đánh giá như lời giải trên ta có kết quả bài toán.
Nhận xét. Lời giải 2 cho ta đánh giá trong trường hợp tổng ba số khác 1.
603
Khám phá tư duy Kỹ thuật giải bất ĐT Bài toán Max – Min – Đặng Thành Nam

Bài toán 9. Chứng minh rằng với mọi số thực không âm a,b thoả mãn điều kiện ta
1 1 1
có + ≥1+ .
1 + kx 1 + ky 1+ k ( x + y)
2 2 2

Chứng minh.
Bất đẳng thức đã cho tương đương với:
1 − k 2 x2 y 2 1

(1 + kx )(1 + ky )
2 2
1+ k ( x + y)
2

2kxy − k 3 x 2 y 2 ( x + y ) − 2k 2 x 2 y 2
2
⇔ ≥0

n
( )( )(
1 + kx 2 1 + ky 2 1 + k ( x + y )
2
)

.v
( ( ))
.
kxy 2 − kxy k ( x + y ) + 2
2

om
⇔ ≥0
( )(
1 + kx 2 1 + ky 2 1 + k ( x + y ))( 2
)
 CÁC VÍ DỤ
.c
ok
Ví dụ 1. Chứng minh rằng với mọi số thực không âm a,b thoả mãn điều kiện
2 1 1 1
a+b≤ ta có + ≥1+ .
bo

1 + 3a 1 + 3b 1 + 3( a + b )
2 2 2
7
Lời giải
et

 9ab ( a + b ) 
2
ab 1 − 3ab − 
vi

 2 
Bất đẳng thức đã cho tương đương với:   ≥0.
(1 + 3a )(1 + 3b ) (1 + 3( a + b ) )
ng

2 2 2

9ab ( a + b )
2
a

Ta chỉ cần chứng minh 1 − 3ab − ≥0


2
kh

(
⇔ 2 ≥ 3ab 2 + 3 ( a + b )
2
).
Bất đẳng thức đúng bởi vì
2
 2  
( ) ( )
2 2
a+b  1 
3ab 2 + 3 ( a + b )  2 + 3( a + b ) ≤ 3  2 + 3  <2.
2 2
≤ 3 
 2   7   7  

Bất đẳng thức được chứng minh. Đẳng thức xảy ra khi và chỉ khi a = 0 hoặc b = 0 .

604
Cty TNHH MTV DVVH Khang Việt

Bài tập tương tự


Cho a,b là các số thực không âm thoả mãn điều kiện a + b ≤ 2 .
1 1 1
Chứng minh rằng + ≥1+ .
1+ a 1+ b 1 + (a + b)
2 2 2

Ví dụ 2. Cho a,b,c là các số thực không âm thoả mãn điều kiện a + b + c =


1.
Tìm giá trị nhỏ nhất của biểu thức
1 1 1
P= + + .
1 + 3a 2
1 + 3b 2
1 + 3c 2

n
Lời giải
Không mất tính tổng quát giả sử a = max {a, b, c}

.v
1 2
⇒ a ≥ ;b + c ≤ .

om
3 3
1 1 1
Khi đó ta có + ≥1+ .
1 + 3b 1 + 3c 1 + 3(b + c )
2 2 .c 2

1 1
ok
Vì vậy P ≥ +1+ .
1 + 3a 1 + 3(b + c )
2 2
bo

1 1
= 2+ +1
1 + 3a 1 + 3 (1 − a )
2

3 ( 2a − 1) (1 + 6a (1 − a ) ) 15 15
et

2
= + ≥
( )(
7 3a 2 + 1 3a 2 − 6a + 4 7 7 )
vi

1
ng

Dấu bằng xảy ra khi và chỉ khi a= b= , c= 0 hoặc các hoán vị.
2
Bài tập tương tự
a

Cho a,b,c,d là các số thực không âm thoả mãn điều kiện a + b + c + d =2.
kh

Chứng minh rằng


1 1 1 1 16
+ + + ≥ .
1 + 3a 2
1 + 3b 2
1 + 3c 2
1 + 3d 2 7

605
Khám phá tư duy Kỹ thuật giải bất ĐT Bài toán Max – Min – Đặng Thành Nam

MỘT SỐ BẤT ĐẲNG THỨC DẠNG LƯỢNG GIÁC HOÁ GIẢI BẰNG ĐÁNH
GIÁ ĐẠI SỐ
Dưới đây đề cập một số bất đẳng thức dạng lượng giác hoá có thể đánh giá bằng
các bất đẳng thức cơ bản hoặc sử dụng tính đơn điệu của hàm số (xem chương 4).

Bài 1. Cho a,b,c là các số thực dương thỏa mãn ab + bc + ca =


1 ta luôn có
a b 1
+ ≤ .
1+ a 2
1+ b 2
1 + c2
Lời giải

n
1 nên 1 + a 2 = ab + bc + ca + a 2 = ( a + b )( a + c )
Vì ab + bc + ca = .

.v
1 + b 2 = ( b + c )( b + a ) ;1 + c 2 = ( c + a )( c + b )

om
Suy ra ( a + b )( b + c )( c + a ) = (1 + a )(1 + b )(1 + c ) .
2 2 2

Khi đó sử dụng bất đẳng thức C –S ta có .c


a b a b a (b + c ) + b (c + a )
+= + =
( a + b )( a + c ) ( b + c )( b + a ) ( a + b )( b + c )( c + a )
ok
1 + a 1 + b2
2

1 + ab (1 + a )(1 + b ) = 2 2 .
bo

1
= ≤
(1 + a )(1 + b )(1 + c
2 2 2
) (1 + a )(1 + b )(1 + c )
2 2 2
1 + c2
et

a = b
Đẳng thức xảy ra khi và chỉ khi  .
ab + bc + ca =
1
vi

Bài 2. Cho a,b,c là các số thực dương thỏa mãn điều kiện ab + bc + ca =
1.
ng

a b 3c
Tìm giá trị lớn nhất của biểu thức P = + + .
1+ a 2
1+ b 2
1 + c2
a

Lời giải
kh

a b 1
Sử dụng + ≤ .
1+ a 2
1+ b 2
1 + c2

Suy ra P ≤
3c + 1

( c + 1)(3 + 1) =
2

10 .
2

1 + c2 c2 + 1
c = 3
 a =b =−3 + 10
Đẳng thức xảy ra khi và =
chỉ khi a b ⇔ .
ab + bc + ca = c = 3
 1

606
Cty TNHH MTV DVVH Khang Việt

Vậy giá trị lớn nhất của P bằng 10 .


Bài tập tương tự
Cho a,b,c là các số thực dương thỏa mãn điều kiện ab + bc + ca =
1.
a b 3c
Tìm giá trị lớn nhất của biểu thức P = + + .
1+ a 2
1+ b 2
1 + c2
Bài 3. Cho các số thực không âm x, y, z thỏa mãn điều kiện xz + yz + 1 =xy.
2x 2y z2 −1
Tìm giá trị lớn nhất của=
biểu thức P + + .
x2 + 1 y2 + 1 z2 + 1

n
Lời giải

.v
Theo giả thiết ta có xz + yz = xy − 1 ≥ 0 ⇔ xy ≥ 1 .

om
z z 1
Khi đó chia hai vế cho xy ta được: + + =
1.
y x xy
1 1 2a 2b c2 − 1
Đặt= =,b = , c z ta có ab + bc + ca =
1 và P = + +
a
x y
.c
1 + a 2 1 + b2 c2 + 1
.
ok
Ta có 1 + a 2 = ab + bc + ca + a 2 = ( a + b )( a + c ) .
1 + b 2 = ( b + c )( b + a ) ;1 + c 2 = ( c + a )( c + b )
bo

Suy ra ( a + b )( b + c )( c + a ) = (1 + a )(1 + b )(1 + c ) .


2 2 2
et

a b a b
Ta có =
+ +
1+ a 1+ b
2 2
ab + bc + ca + a 2
ab + bc + ca + b 2
vi

a b a (b + c ) + b (c + a )
ng

= + =
( a + b )( a + c ) ( b + c )( b + a ) ( a + b )( b + c )( c + a )
(1 + a )(1 + b ) =1
2 2 .
a

1 + ab
= ≤
( a + b )( b + c )( a + c ) (1 + a )(1 + b )(1 + c ) 1 + c
kh

2 2 2 2

2 c2 − 1
Vậy P ≤ f (c=
) + .
1 + c2 c2 + 1

2 c2 − 1
Xét hàm số=
f (c ) + với c ≥ 0 ta có
1 + c2 c2 + 1

607
Khám phá tư duy Kỹ thuật giải bất ĐT Bài toán Max – Min – Đặng Thành Nam

f '(c) =

(
2c c 2 + 1 − 2
; f '( c ) =
0 ⇔ 
)
c = 0
.
1+ c 2 2
( )  c = 3

3
Lập bảng biến thiên suy ra max=
f (c) f=
( 3) .
c ≥0 2
Đẳng thức xảy ra khi và chỉ khi c = 3, a = b = 2 − 3 ⇔ x = y = 2 + 3, z = 3 .
3
Vậy giá trị lớn nhất của P bằng đạt tại x = y= 2 + 3, z =3.
2
Bài toán 10. Với mọi số thực dương a,b,c ta luôn có

n
.v
1 1 1
+ ≥ .
(a + b) 2
(a + c) 2
a + bc
2

om
Đẳng thức xảy ra khi và chỉ khi a= b= c .
Chứng minh.
Sử dụng bất đẳng thức C-S cho hai số dương ta được: .c
(
 2
) 1 + bc  ≥ ( a + b ) 2 1 1 c
ok
 a + bc ⇒ ≥ =
 (a + b) 2
(  b
a 2 + bc 1 + 
2
)
a + bc ( b + c ) ( )
  c

bo

.
(
 a 2 + bc
) 1 + bc  ≥ ( a + c ) 2

1

1
=
b

 ( a + c )2 (  c
a 2 + bc 1 +  )
a 2 + bc ( b + c ) ( )
et

  b
Cộng theo vế hai bất đẳng thức trên ta có điều phải chứng minh.
vi

Đẳng thức xảy ra khi và chỉ khi a= b= c .


ng

Đặc biệt. Khi a = 1 ta có bất đẳng thức hay dùng sau đây:
1 1 1
+ Với mọi số thực dương a, b ta luôn có: + ≥ .
a

( a + 1) 2
( b + 1) 2 1 + ab
kh

1 1 2
+ Với mọi số thực dương a,b ta luôn có: + ≥ .
(1 + a ) (1 + b ) a+b+2
2 2

Chứng minh.
1 1 1 1 2
Ta có: + ≥ ≥ =.
(1 + a ) (1 + b ) 1 + ab a+b a+b+2
2 2
1+
2

608
Cty TNHH MTV DVVH Khang Việt

 CÁC VÍ DỤ
Ví dụ 1. Cho a,b,c là các số thực dương thay đổi thỏa mãn abc = 1 .
1 1 1 3
Chứng minh rằng + + ≥ .
(1 + a ) 2
(1 + b ) 2
(1 + c ) 2 4
Lời giải
Bài toán phụ. Với mọi số thực dương a, b ta luôn có:
1 1 1
+ ≥ .
( a + 1) ( b + 1) 1 + ab
2 2

Chứng minh. Ta có:

n
  a 1 1 b

.v
(1 + ab ) 1 + b  ≥ (1 + a ) ⇒
2
≥ =
   ( a + 1) (1 + ab ) 1 + a  (1 + ab )( a + b )
2

om
  a
 .
(1 + ab ) 1 + b  ≥ (1 + b )2 ⇒ 1

1
=
a
 
  a (1 + b)
2
 b  (1 + ab )( a + b )
 (1 + ab ) 1 +  .c
  a
Cộng lại theo vế hai bất đẳng thức trên ta có điều phải chứng minh.
ok
 1 1  1 1 1
Vậy ta có: P=  + + ≥ +
bo

 (1 + b )  (1 + c ) 1 + ab (1 + c )2
 (1 + a )
2 2 2

1 1 c2 + c + 1
et

= + = .
1 (1 + c )2 ( + ) 2
1+ c 1
vi

c
Ta chứng minh
ng

c2 + c + 1
( c + 1) 2

3
4
( 2
)
⇔ 4c 2 + 4c + 4 ≥ 3 c 2 + 2c + 1 ⇔ ( c − 1) ≥ 0 (luôn đúng).
a

3
kh

Vậy giá trị nhỏ nhất của P bằng đạt tại a= b= c= 1 .


4
Ví dụ 2. Cho x,y,z là các số thực dương thỏa mãn xyz = 1 .
1 1 4
Tìm giá trị nhỏ nhất của biểu thức P = + + .
(1 + x ) 2
(1 + y ) 2
3 (1 + z )
3

Lời giải
1 1 1 1 z
Sử dụng bất đẳng thức: + ≥ = = .
(1 + x ) 2
(1 + y ) 2 1 + xy 1 + 1 1+ z
z

609
Khám phá tư duy Kỹ thuật giải bất ĐT Bài toán Max – Min – Đặng Thành Nam

z 4 3z 3 + 6 z 2 + 3z + 4
Khi đó P ≥ + = .
1 + z 3 (1 + z )3 3 ( z + 1)
3

2
Dự đoán đẳng thức xảy ra khi x =y =z =1 ⇒ P = .
3
Ta chứng minh
3z 3 + 6 z 2 + 3z + 4 2
⇔ z 3 − 3 z + 2 ≥ 0 ⇔ ( z + 2 )( z − 1) ≥ 0 (luôn đúng).
2

3 ( z + 1)
3 3

2
Vậy giá trị nhỏ nhất của P bằng đạt tại x= y= z= 1 .

n
3

.v
Ví dụ 3. Cho x,y,z là các số thực dương thay đổi thỏa mãn xyz = 1 .

om
1 1 1
Tìm giá trị nhỏ nhất của biểu thức P = + + .
(1 + x )
3
(1 + y )
3
(1 + z )3
Lời giải
.c
Sử dụng bất đẳng thức AM-GM ta được:
ok
1 1 1 3
+ + ≥
(1 + x ) 3
(1 + x ) 3 8 2 (1 + x )2
bo

1 1 1 3
+ + ≥ .
(1 + y ) 3
(1 + y ) 3 8 2 (1 + y )2
et

1 1 1 3
+ + ≥
vi

(1 + z )3
(1 + z ) 3 8 2 (1 + z )2
ng

Cộng theo vế ba đất đẳng thức trên ta được:

3 3 1 1 1  3 1 1  3 z2 + z + 1 9
2P + ≥  + +  ≥  + =  ≥
a

.
8 2  (1 + x )2 (1 + y )2 (1 + z )2  2  1 + xy (1 + z )2  2 ( z + 1)2 8
   
kh

3
Vậy giá trị nhỏ nhất của P bằng đạt tại x= y= z= 1 .
8
b c a
Nhận xét. Thay=x =,y = ,z ta có bài toán trong đề thi Kỳ thi chọn HSG
a b c
quốc gia năm 2006 như sau:
Bài toán. (VMO 2006) Cho a,b,c là các số thực dương.
3 3 3
 a   b   c  3
Chứng minh rằng   +  +  ≥ .
a+b b+c c+a 8

610
Cty TNHH MTV DVVH Khang Việt

Ví dụ 4. Cho x, y, z là các số thực dương thay đổi thỏa mãn x ≤ y ≤ z và xyz = 1 .


3 2 1
Tìm giá trị nhỏ nhất của biểu thức P = + + .
( x + 1) 2
( y + 1) 2
( z + 1)2
Lời giải

1  1 1 1 1 
Do x ≤ z ⇒ ⇒ P≥≥ 2  + + .
( ) ( )  2 
( ) ( ) ( ) 
2 2 2 2
x + 1 z + 1 x + 1 y + 1 z + 1
Sử dụng bất đẳng thức phụ ta có
2 2 2 2 z2 + z + 1

n
P≥ + = + =
2. .
1 + xy (1 + z )2 1 + 1 (1 + z )2 (1 + z )2

.v
z
z2 + z + 1

om
3
⇔ ( z − 1) ≥ 0 (luôn đúng).
2
Ta chứng minh 2. ≥
(1 + z ) 2 2

3
Vậy GTNN của P bằng khi x= y= z= 1 . .c
2
ok
Ví dụ 5. Cho các số thực dương a,b,c,d thay đổi thỏa mãn a 2 + b 2 + c 2 + d 2 =
1.
Chứng minh rằng (1 − a )(1 − b )(1 − c )(1 − d ) ≥ abcd .
bo

Lời giải
1− a 1− b 1− c 1− d
Ta phải chứng minh ≥1.
et

. . .
a b c d
1− a 1− b 1− c 1− d
vi

Đặt
= x = ,y = ,z = ,t ta có 0 < x, y, z , t < 1 .
a b c d
ng

1 1 1 1
Theo giả thiết ta có: + + + =
1.
(1 + x ) 2
(1 + y ) 2
(1 + z ) 2
(1 + t )2
a

1 1 1 1 1 1 2 + xy + zt
+ + + ≥ + =
kh

Ta có: .
(1 + x ) 2
(1 + y ) 2
(1 + z ) 2
(1 + t ) 2 1 + xy 1 + zt 1 + xy + zt + xyzt

2 + xy + zt
Suy ra: ≤ 1 ⇔ xyzt ≥ 1 ta có điều phải chứng minh.
1 + xy + zt + xyzt
1
Đẳng thức xảy ra khi và chỉ khi a= b= c= d= .
2

611
Khám phá tư duy Kỹ thuật giải bất ĐT Bài toán Max – Min – Đặng Thành Nam

 BÀI TẬP RÈN LUYỆN


Bài 1. Cho a,b,c,d là các số thực dương . Chứng minh
2 2 2 2
 a   b   c   d 
  +  +  +  ≥ 1.
a+b b+c c+d  d +a
HD: Viết lại bất đẳng thức dưới dạng:
1 1 1 1
2
+ 2
+ 2
+ 2
≥1.
 b  c  d  a
1 +  1 +  1 +  1 + 
 a  b  c  d
b c d a

n
Đặt=x =,y = ,z = ,t ta có xyzt = 1 và đưa và bất đẳng thức quen thuộc
a b c d

.v
1 1 1 1
+ + + ≥ 1 với x,y,z,t là các số thực dương thỏa

om
(1 + x ) 2
(1 + y ) 2
(1 + z ) 2
(1 + t )2
mãn xyzt = 1 .

Bài 2. Cho a,b,c là các số thực dương. Tìm giá trị nhỏ nhất của biểu thức
.c
ok
2 2 2
 a   b   c 
P=  +  +  .
a+b b+c c+a
bo

HD: Viết lại P dưới dạng:


1 1 1 1 1 1
P= + + = + +
et

 b
2
 c
2
 a
2
(1 + x ) 2
(1 + y ) 2
(1 + z )2
1 +  1 +  1 + 
 a  b  c
vi

b c a
ng

với=x =,y = ,z nên ta có xyz = 1 .


a b c
1 1 1
Sử dụng bất đẳng thức phụ ta được: + ≥
a

.
(1 + x ) 2
(1 + y ) 2 1 + xy
kh

1 1 1 1 z2 + z + 1 3
Suy ra: P ≥ + = + = ≥ .
1 + xy (1 + z )2 1 + 1 (1 + z )2 ( z + 1)2 4
z
3
Vậy giá trị nhỏ nhất của P bằng đạt tại a= b= c .
4
Nhận xét. Từ kết quả trên ta chứng minh được với mọi a,b,c dương ta luôn có
3 3 3
 a   b   c  3
  +  +  ≥ .
a+b b+c c+a 8
612
Cty TNHH MTV DVVH Khang Việt

a b c
đặt A
Thật vậy,= = ,B = ,C theo chứng minh trên ta có:
a+b b+c c+a
3
A2 + B 2 + C 2 ≥ .
4
Sử dụng bất đẳng thức AM-GM ta được:
1 3 1 3 1 3
A3 + A3 + ≥ A2 , B3 + B3 + ≥ B 2 , C 3 + C 3 + ≥ C 2 .
8 2 8 2 8 2
Cộng lại theo vế ba đất đẳng thức trên ta được:

( )
3 3
( 9 3
2 A3 + B3 + C 3 + ≥ A2 + B 2 + C 2 ≥ ⇒ A3 + B3 + C 3 ≥ . )

n
8 2 8 8

.v
Bài toán được chứng minh. Đẳng thức xảy ra khi a= b= c .

om
Bài 3. Cho a,b,c là các số thực dương thay đổi thỏa mãn a ≤ b ≤ c và abc = 1 .
5 4 3
Tìm giá trị nhỏ nhất của biểu thức P = + + .
(1 + a ).c 2
(1 + b ) 2
(1 + c )2
Bài 4. Cho a,b,c là các số thực dương thỏa mãn a ≥ b ≥ c .
ok
Tìm giá trị nhỏ nhất của biểu thức
1 2 3 ab + bc + ca
bo

P= + + + .
( a + 1) 2
( b + 1) 2
( c + 1) 2 4
Bài 5. Cho x, y, z là các số thực dương thay đổi thỏa mãn x ≤ y ≤ z và xyz = 1 .
et

3 2 1
Tìm giá trị nhỏ nhất của biểu thức P = + +
vi

.
( x + 1) 2
( y + 1) 2
( z + 1)2
ng

1 1
HD: Do x ≤ z ⇒ ≥
( x + 1) 2
( z + 1)2
a

 1 1 
kh

1
⇒ P ≥ 2 + + .
 ( x + 1)2 ( y + 1)2 ( z + 1)2 
 
Vậy ta có:
 1 1  1  2 2 2 2 z2 + z + 1
P ≥ 2  + + ≥ + = + =
2. .
 (1 + x )2 (1 + y )2  (1 + z )2  1 + xy (1 + z )2 1 + 1 (1 + z )2 (1 + z ) 2
  
z
z2 + z + 1 3
⇔ ( z − 1) ≥ 0 (luôn đúng).
2
Ta chứng minh 2. ≥
(1 + z ) 2 2

613
Khám phá tư duy Kỹ thuật giải bất ĐT Bài toán Max – Min – Đặng Thành Nam

3
Vậy GTNN của P bằng khi x= y= z= 1 .
2
Bài 6. Cho a,b,c,d là các số thực dương thay đổi thỏa mãn abcd = 1 .
Tìm giá trị nhỏ nhất của biểu thức
1 1 1 1
P= + + + .
(1 + a ) 2
(1 + b ) 2
(1 + c ) 2
(1 + d )2
HD: Sử dụng bất đẳng thức phụ ta có:
1 1 1 1 1 1 ab + cd + 2
+ + + ≥ =+ = 1.
(1 + a ) 2
(1 + b ) 2
(1 + c ) 2
(1 + d ) 2 1 + ab 1 + cd 1 + ab + cd + abcd

n
Vậy giá trị nhỏ nhất của P bằng 1 đạt tại a= b= c= d= 1 .

.v
1 1 8
Bài toán 11. Với mọi số thực dương a,b ta có + ≥

om
.
a 2
b 2
( a + b )2
Dấu bằng xảy ra khi và chỉ khi a = b .
Chứng minh.
Sử dụng bất đẳng thức AM-GM cho hai số dương ta được:
.c
ok
1 1 2 2 8
+ ≥ ≥ = 2.
a 2
b 2 ab  a + b  2
(a + b)
bo

 
 2 
Đẳng thức xảy ra khi và chỉ khi a = b .
et

Đây chỉ là một bất đẳng thức rút ra từ bất đẳng thức AM – GM cho hai số
dương ta cùng xét một số bài toán áp dụng.
vi
ng

 CÁC VÍ DỤ
Ví dụ 1. Cho x, y, z là các số thực không âm thỏa mãn điều kiện x 2 + y 2 + z 2 ≤ 3 y .
a

1 4 8
Tìm giá trị nhỏ nhất của biểu thức P = + +
kh

.
( x + 1) 2
( y + 2) 2
( z + 3)2
Lời giải
Ta có:
1 4 1 1 2 8
+ = + ≥ ≥ .
( x + 1) 2
( y + 2) 2
( x + 1) 2
y 
2
y   
( x + 1)  + 1  x + 1 + + 1
y
2
 + 1 2  
2  2 

614
Cty TNHH MTV DVVH Khang Việt

8 8 16 64
+ ≥ ≥ .
 y 
2
( z + 3) 2

 + +
y 
+ ( +  
)  x + 1 + + 1 + z + 3
y
2
 x + 1 + + 1
x 1 1 z 3
 2   2   2 
Mặt khác: 2 x + 4 y + 2 z ≤ x 2 + 1 + y 2 + 4 + z 2 + 1 = x 2 + y 2 + z 2 + 6 ≤ 3 y + 6
y 64
⇒ 2x + y + 2z ≤ 6 ⇒ x + + z ≤ 3⇒ P ≥ =
1 .
2 ( 3 + 2 + 3)2
Vậy GTNN của P bằng 1 khi x= z= 1, y= 2 .
Ví dụ 2. Cho a,b,c là các số thực không âm và không đồng thời bằng 0 thay đổi

n
thỏa mãn điều kiện a 2 + b 2 + c 2 ≤ 6b .

.v
1 8 1
Tìm giá trị nhỏ nhất của biểu thức P= + + .
(a + b + c) 2
( b + 11) 2
( c + 6 )2

om
Lời giải
1 1 8
Sử dụng bất đẳng thức phụ + ≥ ta được:
x 2
y 2
( x + y )2
.c
ok
1 1 8
+ ≥ .
(a + b + c) 2
( c + 6) 2
( a + b + 2c + 6 ) 2
bo

 1 1  64
Suy ra P ≥ 8  + ≥ .
 ( a + b + 2c + 6 ) 2
( ) 2  ( a + 2b + 2c + 17 )2
et

 b + 11 
Mặt khác theo bất đẳng thức AM-GM ta có:
vi

( ) ( ) (
2a + 10b + 4c ≤ a 2 + 1 + b 2 + 25 + c 2 + 4 ≤ 6b + 30 ⇒ a + 2b + 2c ≤ 15 . )
ng

64 1
Do đó P ≥ = .
(15 + 17 ) 2 16
a
kh

1
Vậy giá trị nhỏ nhất của P bằng đạt tại =
a 1,=
b 5,=
c 4.
16

 BÀI TẬP RÈN LUYỆN


Bài 1. Cho a,b,c là các số thực đôi một phân biệt.
 1 1 
Tìm giá trị nhỏ nhất của biểu thức P = a 2 + b 2 + c 2  +
1
( + .
 ( a − b )2 ( b − c )2 ( c − a )2 
)
 

615
Khám phá tư duy Kỹ thuật giải bất ĐT Bài toán Max – Min – Đặng Thành Nam

HD: Không mất tính tổng quát giả sử a > b > c khi đó ta có:
1 1 2 8
+ ≥ ≥
(a − b) 2
( a − b )( b − c ) ( a − c )2
(b − c ) 2
.
a 2 + b2 + c2 ≥ a 2 + c2 ≥
( a − c ) 2

( a − c)   9
2
8 1
Suy ra P≥  + = .
2  ( a − c )2 ( a − c )  2
2

Đẳng thức xảy ra khi và chỉ khi b = 0, a = −c .

n
9
đạt tại a =
− c, b =

.v
Vậy giá trị nhỏ nhất của P bằng 0 hoặc các hoán vị.
2
Bài 2. Cho a,b,c là các số thực đôi một phân biệt thuộc đoạn [ 0;2] .

om
Tìm giá trị nhỏ nhất của biểu thức
1 1 .c 1
P= + + .
(a − b) 2
(b − c ) 2
( c − a )2
ok
HD: Không mất tính tổng quát giả sử a > b > c ta có:
1 1 2 8
+ ≥ ≥
bo

.
(a − b) 2
(b − c ) 2
( a − b )( b − c ) ( a − c )2
9 9
≥ , ∀a, c ∈ [ 0;2] .
et

Do đó P ≥
(a − c) 2 4
vi

9
Vậy giá trị nhỏ nhất của P bằng đạt tại =
a 2,=
b 1,=
c 0 hoặc các hoán vị.
ng

4
a
kh

616
Cty TNHH MTV DVVH Khang Việt

CHỦ ĐỀ 9: BÀI TOÁN CHỌN LỌC


BẤT ĐẲNG THỨC VÀ CỰC TRỊ BA BIẾN
Trong chủ đề này tôi tổng hợp lại một số bài toán bất đẳng thức và cực trị ba
biến hay trong các đề thi thử, đề thi chọn Học sinh giỏi cấp tỉnh và thành phố các
năm cùng một số bài toán đề xuất trên Diễn đàn học tập trực tuyến Mathlinks.vn.
Và sau một số bài toán có bổ sung thêm một số bài toán tương tự dành cho bạn đọc
rèn luyện.
Bài 1. Cho x,y,z là các số thực dương thỏa mãn điều kiện x(1 – yz) = y + z.

(z + z )
2
xy

n
2z
=
Tìm giá trị lớn nhất của biểu thức P + .
( x + y ) ( z 2 + 1) ( )

.v
1 + z2 1 + z2

om
Lời giải
Theo giả thiết ta có z (1 + xy ) = x − y ⇒ x > y .
Sử dụng bất đẳng thức C –S ta có

(z + z xy )
2
( )
.c
= z 2 1 + 2 xy + xy = z 2 (1 + xy ) + 2 z 2 xy = z ( x − y ) + 2 z 2 xy
ok
≤ (z 2
 )
+ z 4 ( x − y ) + 4 xy  =
2

( x + y ) z z2 + 1
bo

Khi đó P ≤
( x + y ) z z2 + 1 + 2z
= +
z 2z
.
( x + y ) ( z 2 + 1) (1 + z 2 ) z 2 + 1 z 2 + 1 (1 + z 2 ) z 2 + 1
et

z 2z
=
Xét hàm số g ( z) + với z dương ta có
vi

z +1
2
(
1+ z 2
) z2 + 1
ng

g '( z ) =
(
3 1 − z2 ) ; g '( z ) = 0 ⇔ z = 1 .
(z )
2
2
+1 z +12
a
kh

Ta có g’(z) đổi dấu từ dương sang âm khi đi qua 1 vì vậy g ( z ) ≤ g (1) =2.
Với z = 1, x = 2 + 1, y = 2 − 1 thì P bằng 2.
Vậy giá trị lớn nhất của P bằng 2 .
Bài 2. Cho a, b, c là các số không âm thoả mãn điều kiện ab + bc + ca =
4.
Tìm giá trị nhỏ nhất của biểu thức
P= a 2 + 9bc + b 2 + 9ac + c 2 + 9ba .
Lời giải
Ta chứng minh a 2 + 9bc + b 2 + 9ac + c 2 + 9ba ≥ 5 ab + bc + ca .

617
Khám phá tư duy Kỹ thuật giải bất ĐT Bài toán Max – Min – Đặng Thành Nam

Không mất tính tổng quát giả sử c = min {a, b, c} .


Sử dụng bất đẳng thức Mincopski ta có

( a + b ) 2 + 9c ( )
2
a 2 + 9bc + b 2 + 9ac ≥ a+ b .

Và c 2 + 9ab ≥ 3 ab .
Vậy ta chỉ cần chứng minh

( a + b )2 + 9c ( )
2
a+ b + 3 ab ≥ 5 ab + bc + ca

( a + b )2 + 9c ( )
2
⇔ a+ b ≥ 5 ab + bc + ca − 3 ab

n
( )
2

.v
⇔ ( a + b ) + 9c ≥ 9ab + 25 ( ab + bc + ca ) − 30 ab ( ab + bc + ca )
2
a+ b
.
⇔ ( a + b ) + 18c ab + 30 ab ( ab + bc + ca ) ≥ 34ab + 16c ( a + b )

om
2

⇔ ( a + b ) + 18c ab − 4ab − 6c ( a + b )
2

 c ( a + b ) 

+30  ab ( ab + bc + ca ) −  ab +  ≥ 0
.c
  3  
ok
c (a + b) 
2

ab ( ab + bc + ca ) −  ab + 
 c (a + b) 
bo

ab ( ab + bc + ca ) −  ab +  3 
Ta có = c (a + b)
 3 
ab ( ab + bc + ca ) + ab +
3
et

c ( a + b )( 3ab − bc − ca )
≥0.
vi

9 ab ( ab + bc + ca ) + 9ab + 3c ( a + b )
ng

Vậy ta chứng minh ( a + b ) + 18c ab − 4ab − 6c ( a + b ) ≥ 0 .


2

Bất đẳng thức có dạng thuần nhất nên chuẩn hoá a + b =


1.
a

a+b 1
Đặt= x ab ,0 ≤ c ≤ x ≤ = .
kh

2 2
Ta cần chứng minh f ( x) =−4 x 2 + 18cx − 6c + 1 ≥ 0 .
Vế trái là tam thức bậc hai với hệ số x2 âm nên chỉ cần chỉ ra rằng
  1 
min  f (c); f    ≥ 0 .
  2 
1
Thật vậy ta có f   =3c ≥ 0; f (c) =5c 2 + ( 3c − 1) > 0 .
2

2
Bất đẳng thức được chứng minh.

618
Cty TNHH MTV DVVH Khang Việt

Vậy ta có a 2 + 9bc + b 2 + 9ac + c 2 + 9ba ≥ 10 .


Dấu bằng đạt tại a= b= 2, c= 0 hoặc các hoán vị.
Vậy giá trị nhỏ nhất của P bằng 10.
Bài 3. Cho a,b,c là độ dài ba cạnh của một tam giác và thỏa mãn điều kiện
a 2 + b2 + c2 =3 . Tìm giá trị nhỏ nhất của biểu thức
a+b b+c c+a
P= + + .
a+b−c b+c−a c+ a −b
Lời giải
Ta chứng minh giá trị nhỏ nhất của P bằng 6.

n
Thật vậy chỉ cần chứng minh rằng

.v
a+b b+c c+a a 2 + b2 + c2
+ + ≥ 64 .

om
a+b−c b+c−a c+ a −b 3
Bất đẳng thức có dạng thuần nhất nên chuẩn hoá a + b + c =3.
3− a 3−b 3−c .c a 2 + b2 + c2
Ta cần chứng minh rằng + + ≥ 64 .
3 − 2a 3 − 2b 3 − 2c 3
ok
3− x x2 + 3  3
Ta chứng minh ≥ , ∀x ∈  0;  .
3 − 2x 2  2
bo

Thật vậy bất đẳng thức đã cho tương đương với:

( ) ( 3 − 2 x ) ⇔ ( x − 1)2 ( 2 x3 + x 2 + 12 x + 9 ) ≥ 0 .
2
4 (3 − x ) ≥ x2 + 3
2
et

Bất đẳng thức được chứng minh.


Áp dụng bất đẳng thức kết hợp bất đẳng thức AM – GM ta có
vi

3− a 3−b 3−c a 2 + b 2 + c 2 + 3.3


ng

+ + ≥
3 − 2a 3 − 2b 3 − 2c 2

( ) a 2 + b2 + c2
a

≥ 2 4 33 a 2 + b 2 + c 2 = 6 4
3
kh

Ta có đpcm.
Vậy giá trị nhỏ nhất của P bằng 6 đạt tại a= b= c= 1 .
Bài 4. Cho x,y,z là các số thực dương thoả mãn điều kiện
x2 + y 2 + z 2 + 1 ≤ 2 x + 3 y .
Tìm giá trị nhỏ nhất của biểu thức
x3 + x 2 + 36 y 3 + y 2 + 36 2 z 3 + z 2 + 9
P= + + .
2x + 2 4y + 4 2z + 1

619
Khám phá tư duy Kỹ thuật giải bất ĐT Bài toán Max – Min – Đặng Thành Nam

Lời giải
2 2
x y 18 9 9
Ta có P = + + z2 + + + .
2 4 x + 1 y + 1 2z + 1
Dựa vào biểu thức của P trên ta dự đoán dấu bằng đạt tại
x + 1 = y + 1 = 2z + 1 ⇒ x = y = 2z .
Thay vào điều kiện ta được: x= y= 2, z= 1 .
Dựa vào đó ta có các đánh giá sau đây:
Sử dụng bất đẳng thức C –S ta có
x2 y 2 2 x2 + y 2 + 4 z 2 1
≥ ( 2x + y + 2z ) .
2
= + + z2

n
2 4 4 16

.v
18 9 9 9 9 9 9
+ + = + + +
x + 1 y + 1 2z + 1 x + 1 x + 1 y + 1 2z + 1

om
9.42 144
≥ =
2 ( x + 1) + y + 1 + 2 z + 1 2 x + y + 2 z + 4

Do đó P ≥
144 1
+ ( 2x + y + 2z ) .
2
.c
2 x + y + 2 z + 4 16
ok
Chú ý giả thiết bài toán ta có 2 x + 3 y ≥ x 2 + y 2 + z 2 + 1

(x ) ( ) ( )
bo

= 2
+ 4 + y2 + 4 + z2 + 1 − 8
≥ 4x + 4 y + 2z − 8
et

⇒ 2x + y + 2z ≤ 8

( t − 8)2 ( t + 20 ) + 16 ≥ 16 .
vi

1 2 144
Đặt t = 2 x + y + 2 z , t ∈ ( 0;8] ta có P ≥ = t +
16 t+4 16 ( t + 4 )
ng

Đẳng thức xảy ra khi và chỉ khi x= y= 2, z= 1 .


Vậy giá trị nhỏ nhất của P bằng 16.
a

Bài 5. Cho a,b,c là các số thực dương thoả mãn điều kiện a 2 + b 2 + c 2 =
14 .
kh

Tìm giá trị lớn nhất của biểu thức


4(a + c) 4a 5 3
=
P + − − .
a + 3c + 28
2 2
a + bc + 7
2
(a + b) 2 a (b + c )
Lời giải
Chú ý đến giả thiết bài toán và tích a(b+c) ta có đánh giá như sau:
14 + 2bc = a 2 + ( b + c ) ≥ 2a ( b + c ) ⇒ a ( b + c ) ≤ 7 + bc .
2

620
Cty TNHH MTV DVVH Khang Việt

4a 4a 4
Suy ra ≤ = .
a + bc + 7
2
a + a (b + c ) a + b + c
2

2
a+b+c
Sử dụng bất đẳng thức AM – GM ta có: a ( b + c ) ≤  
 2 
3 3 12
⇒− ≤− =− .
a (b + c ) a+b+c
2
( a + b + c )2
 
 2 
Cộng theo vế hai bất đẳng thức trên ta được:
4a 3 12 4

n
− ≤− +
a + bc + 7 a ( b + c )
2
(a + b + c) a + b + c
2

.v
2
 1 1 1 1

om
= −12  −  + ≤
a+b+c 6 3 3
Đẳng thức xảy ra khi và chỉ khi a = b + c; a + b + c = 6 .
Sử dụng bất đẳng thức C –S ta có: .c
 1 2
( 2
) 2 2 2 3
1 +  a + 3c ≥ ( a + c ) ⇒ a + 3c ≥ ( a + c ) .
2
ok
 3  4
4(a + c) 4(a + c) 1 2
≤ ≤ ( a + c − 4) + .
bo

Suy ra
a + 3c + 28
2 2 3
( a + c )2 + 28 25 5
4
et

5 2 1
Và − ≤ ( a + b − 5) − .
( a + b )2 25 5
vi
ng

Chú ý ở hai bất đẳng thức trên ta sử dụng bất đẳng thức dạng tiếp tuyến.
Cộng theo vế hai bất đẳng thức trên và sử dụng bất đẳng thức C –S ta được:
4(a + c)
a

5 3a + 2b + c − 9
− ≤
a + 3c + 28 ( a + b )
kh

2 2 2 25


(a 2
)( )
+ b 2 + c 2 32 + 22 + 12 − 9 1
=
25 5
1 1 8
Từ đó suy ra Pmax = + = đạt tại= a 3,= b 2,= c 1.
3 5 15
Nhận xét. Ta có thể đánh giá phân thức đầu tiên bằng cách khác như sau:
a 2 + 3c 2 + 28 = 2a 2 + b 2 + 4c 2 + 14 ≥ 2a 2 + 4bc + 14 .
Mặt khác: 2a 2 + 2bc + 2 ( bc + 7 ) ≥ 2a 2 + 2bc + 2a ( b + c ) = 2 ( a + b )( a + c ) .

621
Khám phá tư duy Kỹ thuật giải bất ĐT Bài toán Max – Min – Đặng Thành Nam

4(a + c) 2
Suy ra ≤ .
a + 3c + 28
2 2 a+b
2 4 5 12
Do đó P ≤ + − −
a + b a + b + c ( a + b )2 ( a + b + c )2
2 2
8 1  a +b−5 1  a +b+c−6 8
=
− ⋅  − ⋅  ≤
15 5  a + b  3  a + b + c  15
Ta có kết quả tương tự trên.
Bài tập tương tự
1) Cho a,b,c là các số thực dương thoả mãn điều kiện a + b + c =3.

n
Tìm giá trị lớn nhất của biểu thức

.v
( 2a + b + c )2 + ( 2b + c + a )2 + 8 (a + b − 3 c + 3) .
2

om
P=
2a 2 + ( b + c ) 2b 2 + ( c + a )
2 2 9c

2) Cho a,b,c là các số thực dương thoả mãn điều kiện a 2 + b 2 + c 2 =


26 .
.c
Tìm giá trị lớn nhất của biểu thức
4(a + c)
ok
4a 4 7
=P + − − .
2a + 4bc + 26
2
a + bc + 13
2 a ( b + c ) ( a + b )2
bo

Bài 6. Cho a, b, c là các số thuộc đoạn [0, 3].


Tìm giá trị lớn nhất của biểu thức P = ( a − b )( b − c )( c − a )( a + b + c ) .
et

Lời giải
vi

Giả sử a = max {a, b, c} .


ng

Nếu a ≥ b ≥ c ⇒ A ≤ 0 .
Nếu a ≥ c ≥ b .
Khi đó
a

P (a, b, c) − P (a,0, c) = ( a − b )( b − c )( c − a )( a + b + c ) − ac ( a − c )( a + c )
kh

= ( a − c ) ( a − b )( c − b )( a + b + c ) − ac ( a + c ) .

(
= ( a − c ) b b 2 − ac − c 2 − a 2 ≤ 0 )
Do đó P (a, b, c) ≤ P (a,0, c) = ac ( a − c )( a + c ) = a3c − ac3 .

a ) a3c − ac3 trên đoạn [0;3] ta có


Xét hàm số f (=
f= ( )
'(a ) c 3a 2 − c 2 ≥ 0; ∀3 ≥ a ≥ c ≥ 0 .
Do đó f(a) đồng biến trên đoạn [0;3] suy ra

622
Cty TNHH MTV DVVH Khang Việt

f (a ) ≤ f (3) =g (c) =27c − 3c3 .


c) 27c − 3c3 trên đoạn [0;3] ta có
Xét hàm số g (=
g '(c) = 27 − 9c 2 ; g '(c) = 0 ⇔ c = 3 .
Ta có g’(c) đổi dấu từ dương sang âm khi đi qua c = 3 nên g(c) đạt cực đại tại
c= 3.
Vì vậy g (c) ≤ g ( 3) =
18 3 .
Suy ra Pmax = 18 3 . Dấu bằng đạt tạị=
a 3,=
b 0,=
c 3.
Bài tập tương tự

n
Cho a, b, c là các số thuộc đoạn [0, 2].

.v
Tìm giá trị lớn nhất của biểu thức P = ( a − b )( b − c )( c − a )( a + b + c ) .

om
Bài 7. Cho x,y,z là các số thực dương thỏa mãn điều kiện x + 3 y ≤ 9 z và x > y > z .
2 2 2
 x   y   z 
Tìm giá trị nhỏ nhất của biểu thức P =  + 3  + 3
.c  .
 y−z  x−z  x− y
Lời giải
ok
Ta có ( a − 3) ≥ 0 ⇒ a 2 ≥ 6a − 9 . Đẳng thức xảy ra khi và chỉ khi a = 3 .
2

( b − 1)2 ≥ 0 ⇒ b2 ≥ 2b − 1 . Đẳng thức xảy ra khi và chỉ khi b = 1 .


bo

Áp dụng vào bài toán ta có:


et

2 2 2
 x  x  y  y  z  z
Ta có:   ≥ 6. − 9;   ≥ 2. − 1;   ≥ 2. −1.
 y − z  y − z  x − z  x − z  x − y  x − y
vi

 x y z 
ng

Suy ra: P ≥ 6  + +  − 15 = 6Q − 15 .
 y−z x−z x− y
x y z x2 z2 y
a

Với Q = + + = + + .
y − z x − z x − y x( y − z) z ( x − y) x − z
kh

Sử dụng bất đẳng thức C-S ta có:


x2
+
z2

( x + z) .
2

x( y − z) z ( x − y) x( y − z) + z ( x − y)

Suy ra: Q ≥
( x + z )2 +
y
=
( x + z )2 + y
x( y − z) + z ( x − y) x − z y( x − z) x − z

y   x+z 
2 2
 x+z y y
=   . + = 1 +   
 y  x − z x − z x − z   y  

623
Khám phá tư duy Kỹ thuật giải bất ĐT Bài toán Max – Min – Đặng Thành Nam

2 2
 x+z   x+z x+z
Mặt khác  − 2 ≥ 0 ⇒   ≥ 4. −4.
 y   y  y
y  x+z  x+z y 4x + 4z − 3y
Do đó Q ≥ 1 + 4. −=
4  4. − 3. = ≥5.
x−z y  x−z x−z x−z
Vì x + 3 y ≤ 9 z .
Tại=
x 3,=
y 2,=
z 1 thì P = 15 .
Vậy giá trị nhỏ nhất của P bằng 15 đạt tại=
x 3,=
y 2,=
z 1.
Cách 2: Theo giả thiết tồn tại các số dương a,b sao cho
( a 1) z, y =+
x =+ ( b 1) z, ( a > b ) .

n
Ta có: x + 3 y ≤ 9 z ⇔ ( x − z ) + 3 ( y − z ) ≤ 5 z ⇔ a + 3b ≤ 5 .

.v
2 2 2
 a +1  b +1  1 

om
Khi đó P =  + 3  + 3  .
 b   a   a −b
Sử dụng bất đẳng thức C-S ta có

( 9 + 3 + 3) 
 a + 1 2
+
 b +1
2
+
 1  
2
 a +1 b +1
.c
1 
 3   3    ≥ 3 + + .
 b   a   a − b    b a −b
ok
a
a +1 b +1 1 1 a − b b +1 b +1
Xét Q = + + = + + +
bo

b a a −b a −b b b a
1 a − b b +1 b +1 b +1 b +1 2
≥2 . + + = + +
a −b b 5 − 3b 5 − 3b
et

b b b

( ) (13b + 20 ) ≥ 5, ∀b ∈  0; 5  .
2
b +1 b +1 b −1 b +5
vi

2
Mặt khác + + = 5+  
b 5 − 3b b b ( 5 − 3b )  3
ng

Suy ra Q ≥ 5 ⇒ P ≥ 15 .
Đẳng thức xảy ra khi và chỉ khi b =1 ⇒ a =2 ⇔ 2 x =3 y =6 z .
a

Nhận xét. Đây là một bài toán hay và khó biểu thức của P là dạng đồng bậc nên
kh

chắc hẳn nhiều học sinh sẽ nghĩ đến phép đặt như trên.
Bài 8. Với mọi a,b,c tìm số thực M nhỏ nhất thoả mãn bất đẳng thức

( ) ( ) ( ) ( )
2
ab a 2 − b 2 + bc b 2 − c 2 + ca c 2 − a 2 ≤ M a 2 + b 2 + c 2 .
Lời giải
(
Chú ý ab a − b 2 2
) + bc (b 2
−c 2
) + ca ( c 2
− a2 = ) ( a − b )( b − c )( c − a )( a + b + c ) .
Vậy ta cần tìm số thực M nhỏ nhất sao cho
( a − b )( b − c )( c − a )( a + b + c ) ≤ M ( a 2 + b2 + c 2 )
2
với mọi a,b,c.

624
Cty TNHH MTV DVVH Khang Việt

Bất đẳng thức thuần nhất nên chuẩn hoá a 2 + b 2 + c 2 =


1.
Ta cần tìm M nhỏ nhất sao cho
M ≥ ( a − b )( b − c )( c − a )( a + b + c ) ⇔=
M max ( a − b )( b − c )( c − a )( a + b + c ) .
Ký hiệu P = ( a − b )( b − c )( c − a )( a + b + c ) .
Không mất tính tổng quát giả sử a ≥ b ≥ c khi đó
2
a−c 1
(a − b)(b − c) ≤   ⇒ P ≤ (a − c) ( a + b + c ) .
3
 2  4
( a − b ) + ( b − c ) 
2
Và (a − c) 2 =

n
=( a − c ) + ( b − c ) + 2(a − b)(b − c)
2 2

.v
1
≤ ( a − c ) + (b − c ) + ( a − c )2
2 2

om
2
3
⇒ (a − c) 2 ≤ (a − b) 2 + (b − c) 2 + (c − a ) 2
2

⇒P≤
1 2
 ( a − b )(2
+ (b − c ) 2
+ ( c − a )
.c
2 3
 )
.( a + b + c )
2
4 3 
ok
Đẳng thức xảy ra khi và chỉ khi a − b = b − c ⇔ a + c = 2b .
Khi đó sử dụng bất đẳng thức AM – GM ta có
bo

4
 (a − b) 2 + (b − c) 2 + (c − a ) 2 
2  ( a + b + c ) + 3.
2

P≤  3 
et

2  4 
 
 
vi

2
( 9 2
)
4
= . 34 a 2 + b 2=
+ c2
ng

32 32

a 2 + b 2 + c 2 =
a

1

kh

Đẳng thức xảy ra khi và chỉ khi a + c = 2b



( a + b + c )2 =(a − b) 2 + (b − c) 2 + (c − a ) 2
 3
9 2
Vậy giá trị cần tìm của M là .
32
Bài 9. Cho a,b,c là các số thực dương thỏa mãn điều kiện a + 6 ( b + c ) ≤ 15 .
Tìm giá trị nhỏ nhất của biểu thức P= 3 ( a + b + c ) − 5abc .

625
Khám phá tư duy Kỹ thuật giải bất ĐT Bài toán Max – Min – Đặng Thành Nam

Lời giải
Nhận xét. Điều kiện đối xứng với b và c nên ta nhóm lại biểu thức của P như sau
P = a ( 3 − 5bc ) + 3 ( b + c ) .
+ Nếu 3 − 5bc > 0 ⇒ P > 0 .
3 5
+ Nếu 3 − 5bc ≤ 0 ⇔ bc ≥ khi đó theo giả thiết ta có a ≤ 15 − 6 ( b + c ) ; b + c < .
5 2
Suy ra
P ≥ 15 − 6 ( b + c )  ( 3 − 5bc ) + 3 ( b + c ) =−15 ( b + c ) + 30bc ( b + c ) + 45 − 75bc
15
=−15 ( b + c ) + 15bc ( 2b + 2c − 5 ) + 45 ≥ −15 ( b + c ) + ( b + c )2 ( 2b + 2c − 5) + 45

n
4

.v
15 75
= ( b + c )3 − ( b + c )2 − 15 ( b + c ) + 45
2 4

om
3 3 15 3 75 2
Đặt t = b + c ≥ 2 bc ≥ 2 > khi đó P ≥ f (t ) = t − t − 15t + 45 .
5 2 4 4
15 3 75 2 3
.c
Xét hàm số f (t ) = t − t − 15t + 45 với t > ta có
4 4 2
ok
f '(t ) =
15 2
2
( )
15
3t − 5t − 2 = ( t − 2 )( 3t + 1) ; f '(t ) = 0 ⇔ t = 2 .
2
Ta có f’(t) đổi dấu từ âm sang dương khi đi qua t = 2 nên f(t) đạt cực tiểu tại t = 2 .
bo

Do đó P ≥ f (t ) ≥ f (2) = 0 . Đẳng thức xảy ra khi và chỉ khi b= c= 1, a= 3 .


Vậy giá trị nhỏ nhất của P bằng 0 đạt tại a= 3, b= c= 1 .
et

Bài 10. Cho x,y,z là các số thực không âm thỏa mãn điều kiện
( )
vi

5 x 2 + y 2 + z 2 = 6 ( xy + yz + zx ) .

( )
ng

Tìm giá trị lớn nhất của biểu thức =


P 2( x + y + z ) − y2 + z2 .
a

Lời giải
kh

Nhận xét. Biểu thức của P đối xứng với 2 biến y và z nên ta nghĩ đến việc đánh giá
biểu thức P theo y + z và đẳng thức điều kiện có dạng đẳng cấp nên làm ta suy
nghĩ đến việc đánh giá x theo y + z . Để tìm giá trị lớn nhất của P ta tìm cận trên
của x theo y + z .
Ta có
5 1
5 x 2 + ( y + z ) 2 ≤ 5 x 2 + 5( y 2 + z 2 )= 6( xy + yz + zx) ≤ 6 x( y + z ) + 6. ( y + z ) 2 .
2 4
Do đó 5 x 2 − 6 x( y + z ) + ( y + z ) 2 ≤ 0,

626
Cty TNHH MTV DVVH Khang Việt

Suy ra x + y + z ≤ 2( y + z ) .
1 1 1
Khi đó P ≤ 2( x + y + z ) − ( y + z )2 ≤ 4( y + z ) − ( y + z )2 = 2 y + z − ( y + z )2 .
2 2 2
t4
Đặt t , khi đó t ≥ 0 và P ≤ 2t −
y+z = . (1)
2
1
Xét hàm số f (t=
) 2t − t 4 với t ≥ 0.
2
Ta có f '(t ) = 2 − 2t 3 ; f '(t ) = 0 ⇔ t = 1.
Suy ra bảng biến thiên:

n
t 0 1 +∞

.v
f '(t ) + 0 –

om
3
2
f (t )
.c
ok
3
Dựa vào bảng biến thiên ta có f (t ) ≤ f (1) = với mọi t ≥ 0. (2)
2
 x= y + z x = 1
bo

3  
Từ (1) và (2) ta có P ≤ , dấu đẳng thức xảy ra khi=  y z ⇔  1
2 y + z =  y= z=
 1  2
et

3 1
Vậy giá trị lớn nhất của P là , đạt tại x= 1, y= z= .
vi

2 2
Bài tập tương tự
ng

Cho x,y,z là các số thực không âm thỏa mãn điều kiện

( )
8 x2 + y 2 + z 2 = 3( x + y + z ) .
2
a

(
2( x + y + z ) − y2 + z2 . )
kh

Tìm giá trị lớn nhất của biểu thức =


P
Bài 11. Cho x,y,z là các số thực dương thay đổi thỏa mãn điều kiện
x2 + y 2 + z 2 =
1.
xy yz x3 y 3 + y 3 z 3
Tìm giá trị lớn nhất của biểu thức P = + − .
1 + z2 1 + x2 24 x3 z 3
Lời giải
Ta có:

627
Khám phá tư duy Kỹ thuật giải bất ĐT Bài toán Max – Min – Đặng Thành Nam

xy x3 y 3 + y 3 z 3
yz xy yz 1  y3 y3 
+ −
= + −  + .
1 + z 2 1 + x2 24 x3 z 3 x 2 + y 2 + 2 z 2 y 2 + z 2 + 2 x 2 24  x3 z 3 
Sử dụng bất đẳng thức AM-GM ta được:
xy
+
yz

( x + y )2 +
( y + z )2 .
x2 + y 2 + 2 z 2 y 2 + z 2 + 2 x2 (
4 x2 + y 2 + 2 z 2 ) 4( y 2
+ z 2 + 2 x2 )
Sử dụng bất đẳng thức C-S ta được:
( x + y )2 +
( y + z )2 ≤
x2
+
y2
+
z2
+
y2
x2 + y 2 + 2 z 2 y 2 + z 2 + 2 x2 x2 + z 2 y2 + z2 x2 + z 2 x2 + y 2

n
y2 y2 1  y2  1  y2 

.v
=
1+ + ≤ 1 + 1 +  + 1 + 
x2 + y 2 y2 + z2 4  x 2  4  z 2 

om
3 1  y2 y2 
= +  + 
2 4  x 2 z 2 

1  3 1  y 2 y 2  1  y3 y3 
Suy ra: P ≤  +  2 + 2   −  3 + 3 
.c
4  2 4  x z   24  x z 
ok
3 1  3 y 2 3 y 2 2 y3 2 y3 
=+  2 + 2 − 3 − 3 
8 48  x z 
bo

z x
3 1  3 y 2 2 y3  1  y 2 2 y3 
=+  − 3  + 3 2 − 3 
8 48  x 2 x  48  z z 
et

t ) 3t 2 − 2t 3 trên ( 0;+∞ ) , ta được:


Xét hàm số f (=
vi

f '(t ) =6t − 6t 2 =6t (1 − t ) suy ra f’(t) đổi dấu từ dương sang âm khi đi qua
ng

t = 1 do đó f(t) đạt cực đại tại t = 1 trên ( 0;+∞ ) .


Suy ra f (t ) ≤ f (1) = 1, ∀ t > 0 .
a
kh

3 1   y  y  3 1 5
Tức P ≤ +  f  + f    ≤ + (1 + 1) = .
8 48  x  z   8 48 12
1 5
Vậy giá trị lớn nhất của P bằng . đạt tại x= y= z=
312
Bài 12. Cho a,b,c là các số thực không đồng thời bằng 0 thoả mãn điều kiện
a+b+c = 0.
13a 2b 2 c 2 − 2abc − 2
Tìm giá trị lớn nhất của biểu thức P = .
(a 2
+b +c
2
)
2 3

628
Cty TNHH MTV DVVH Khang Việt

Lời giải
Theo giả thiết a,b,c là ba nghiệm của phương trình: x3 + qx − r =0 với
p = a + b + c = 0, q = ab + bc + ca, r = abc .
Điều kiện để phương trình có ba nghiệm là hàm số y = x3 + qx − r có
yCD . yCT ≤ 0 .
q ≤ 0
q 
Ta có y ' =3 x + q; y ' =
2
0⇔ x =− ⇒ 2
−q .
3 x = ±
 3

n
Khi đó

.v
 −q   −q   2q −q  2q q  2 4q 3 27
y  −  . y   = −  r + 
 − − r  = r + ≤ 0 ⇒ −q3 ≥ r 2 .
 3   3   3 3  3 3  27 4

om
Ta chỉ cần tìm giá trị lớn nhất của P dương vậy xét 13r 2 − 2r − 2 > 0 .
Khi đó

Chú ý a 2 + b 2 + c 2 =p 2 − 2q =
−2q ⇒ P = =
.c
13r 2 − 2r − 2 13r 2 − 2r − 2
.
(−2q )3 −8q3
ok
2
13r 2 − 2r − 2 13r 2 − 2r − 2 13 1 1 1 1 1 1 1
P= ≤ = − − =−  +  ≤ .
bo

−8q 3 27
8. r 2 54 27 r 27 r 2 4 27  r 2  4
4

et

=  p 0= p 0
 
vi

Đẳng thức xảy ra khi và chỉ khi r =−2 ⇔ q = −3 .


 
27 r = −2
ng

−q3 = r 2
 4
Do đó a,b,c là ba nghiệm của phương trình
a

x =1
x 2 − 3 x + 2 = 0 ⇔ ( x − 1) ( x + 2) = 0 ⇔ 
2
.
kh

 x = −2
Vậy giá trị lớn nhất của P bằng ¼ đạt tại a = b = 1, c = −2 hoặc các hoán vị.
Bài 13. Cho a,b,c là các số thực dương thỏa mãn điều kiện
a 2 + b 2 + ( a + b ) c + 4c 2 =
4.
Tìm giá trị lớn nhất của biểu thức
a (b + c ) b(a + c)
2 2
8 − 7c 2 − c
P= + − .
a+c b+c c (a + b)

629
Khám phá tư duy Kỹ thuật giải bất ĐT Bài toán Max – Min – Đặng Thành Nam

Lời giải
1
Ta có: a 2 + b 2 + c ( a + b ) + 4c 2 =4 ⇒ 4 ≥ 4c 2 + c ( a + b ) + ( a + b )2 .
2
⇔ t 2 + 2tc + 8c 2 − 8 ≤ 0 ⇔ ( t + c ) ≤ 8 − 7c 2 ⇔ t ≤ −c + 8 − 7c 2 .
2

a (b + c ) b(a + c)
2 2
1
Do đó P ≤ − .+
a+c b+c c
Đặt a + c= x, b + c= y khi đó theo giả thiết ta có:

( x − c )2 + ( y − c )2 + ( x + y − 2c ) c + 4c 2 =4 ⇔ x 2 + y 2 − c ( x + y ) =4 − 4c 2 .

n
Mặt khác:

.v
a (b + c ) b(a + c) y2 ( x − c) x2 ( y − c )  x2 y 2 
2 2
+ = + = x2 + y 2 − c  +  .
a+c b+c  y x 

om
x y 
Sử dụng bất đẳng thức AM-GM ta có:
x2 y 2 1 1
+ ≥ x + y ⇒ P ≤ x 2 + y 2 − c ( x + y ) − =−4c 2 − + 4 . .c
y x c c
ok
1
Xét hàm số f (c) = −4c 2 − + 4 với c > 0 ta có:
c
bo

1 1 1
f '(c) =−8c + 2 ; f '(c) =0 ⇔ 8c = 2 ⇔ c = .
c c 2
1
et

Ta có f’(c) đổi dấu từ dương sang âm khi đi qua c = nên f(c) đạt cực tiểu tại
2
vi

1 1
c= hay P ≤ f (c) ≤ f   =
1.
2 2
ng

1
Vậy giá trị lớn nhất của P bằng 1 đạt tại a= b= 1, c= .
2
a

Bài tập tương tự


kh

1) Cho a,b,c là các số thực dương thoả mãn điều kiện a > b; ( a + c )( b + c ) =
1.
1 6 12
Tìm giá trị nhỏ nhất của biểu thức P = + + .
(a − b) 2
(b + c ) 2
( c + a )2
2) Cho a,b,c là các số thực dương thoả mãn điều kiện a 2 + b 2 + ( a + b ) c + 4c 2 =
4.

a (b + c ) b(a + c)
2 2
1
Tìm giá trị lớn nhất của biểu thức P = + − .
a+c b+c c

630
Cty TNHH MTV DVVH Khang Việt

Bài 14. Cho a,b,c là các số thực dương. Tìm giá trị nhỏ nhất của biểu thức

=P
( a + b + c )( ab + bc + ca ) + 4bc
.
abc ( b + c )2
Lời giải
Sử dụng bất đẳng thức AM – GM ta có:
( a + b + c )( ab + bc + ca ) =
3+
b + c a ( b + c ) b2 + c2
+ +
abc a bc bc
2 ( b + c ) b2 + c2
≥ 3+ +
bc bc

n
2 (b + c ) (b + c )
2 2
b+c 

.v
= 1+ + =+
 1
bc bc  bc 

om
b+c 4bc
Do đó P ≥ +1+ .
bc ( b + c )2
( b + c )2 , .c4
Đặt t
= ( t ≥ 4 ) ta có P ≥ f (t ) =+
1 t+ .
bc t
ok
4
Xét hàm số f (t ) =+
1 t + với t ≥ 4 ta có
t
bo

1 4 t t −8
f '(=
t) −= ≥ 0, ∀t ≥ 4 do đó f(t) là hàm đồng biến trên [ 4;+∞ ) suy ra
2 t t2 2t 2
et

4 . Đẳng thức xảy ra khi và chỉ khi a= b= c .


P ≥ f (t ) ≥ f (4) =
Cách 2: Ta có thể đánh giá biểu thức trong căn như sau:
vi

( a + b + c )( ab + bc + ca ) = ( a + b + c )  a ( b + c ) + bc 
ng

= a 2 ( b + c ) + bc ( b + c ) + a ( b + c ) + abc
2

≥ 2a ( b + c ) bc + abc + a ( b + c )
2
a

( a + b + c )( ab + bc + ca ) ≥ 2a ( b + c ) bc + abc + a ( b + c )
kh

2
b+c
Suy ra = +1.
abc abc bc
Ta có kết quả tương tự trên.
1 2
Bài 15. Cho x,y,z,t là các số thực thuộc đoạn  ;  .
2 3
Tìm giá trị lớn nhất và giá trị nhỏ nhất của biểu thức
2 2
 x+z  z+t 
=P 9  + 16   .
 x+t   x+ y

631
Khám phá tư duy Kỹ thuật giải bất ĐT Bài toán Max – Min – Đặng Thành Nam

Lời giải
Sử dụng bất đẳng thức AM-GM ta có:

 x+z
2
 z+t 
2
 x+z
2
 z+t  ( x + z )( z + t )
2
9  + 16   ≥ 2 9   .16   =
24.
 x+t   x+ y  x+t   x+ y ( x + t )( x + y )
 1  1 
 x +  t +  ( 2 x + 1)( 2t + 1)
≥ 24. 
2  2 
=
18. .
4( x + t )
( x + t )  + 
2 2
3 3
 1 4 xt + 1  1 1
= 18  +  ≥ 18  + = 18

n
 2 4 ( x + t )  2 2

.v
Do 2 ( x + t ) − 4 xt − 1 = (1 − 2 x )( 2t − 1) ≤ 0 .

om
2 1
Vậy giá trị nhỏ nhất của P bằng 18 đạt tại x = y = , z= t= .
3 2
Tìm giá trị lớn nhất của P .c
2 1 2 2
x+ + x+
x+z 7
ok
Ta có: ≤ 3 ≤ 2 3 = (do 3 là hàm nghịch biến với x).
x+t x+t + t 3 ( 2t + 1) x+t
1
2
bo

2
+t 2
z+t 2 49  2
Tương tự ta có: ≤ 3 =t + . Suy ra P ≤ f (=
t) + 16  t +  .
et

x+ y 1 + 1 3 ( 2t + 1)2  3
2 2
vi

2
49  2 1 2
Xét hàm số f (=
t) + 16  t +  liên tục trên đoạn  2 ; 3  ta có:
ng

( 2t + 1) 2
 3  

 2 196 1 2 196 77
f '(t ) = 32  t +  − ≥ 32  +  − = >0.
a

 3  ( 2t + 1)3
2 3  1  3 6
 2. + 1
kh

 2 
1 2
Do đó f(t) là hàm đồng biến trên  ;  .
2 3
 2  337
Do đó P ≤ f (t ) ≤ f   =.
3 3
1 2
Đẳng thức xảy ra khi và chỉ khi x = y = , z= t= .
2 3
337 1 2
Vậy giá trị lớn nhất của P bằng đạt tại x = y = , z= t= .
3 2 3
632
Cty TNHH MTV DVVH Khang Việt

Bài 16. Cho x,y,z là các số thực thỏa mãn điều kiện x 2 + y 2 + z 2 =
3.

Tìm giá trị lớn nhất của biểu thức P= 3x 2 + 7 y + 3x 2 + 7 z + 5 y + 5 z .


Lời giải
Sử dụng bất đẳng thức C-S ta có:
( )
P 2 ≤ 3 6 x 2 + 7 y + 7 z + 5 y + 5 z= 18  x 2 + 2 ( y + z ) 
 
.
( )
≤ 18  x 2 + 2 2 y 2 + z 2  = 18  x 2 + 2 2 3 − x 2 
   
( )
( )
Xét hàm số f ( x) =18  x 2 + 2 2 3 − x 2  liên tục trên  − 3; 3  ta có:

n
 

.v
  x = 0 x = 0
 
⇔  x =
2x
f '( x) =
36  x − 0⇔
 ; f '( x) = −1 .
( )

om
( )  − =
2
 2 3 − x2  2 3 x 2
    x = 1

Ta có f (0)= 2 6, f (− 3)= f ( 3)= 3, f (−1)= f (1)= 5 . .c


Suy ra P ≤ f ( x) ≤ max f ( x) =
5.
x∈ − 3; 3 
ok
Đẳng thức xảy ra khi và chỉ khi x =
±1, y =
z=1.
Vậy giá trị lớn nhất của P bằng 5 đạt tại x =
−1, y =
z=1 hoặc x= y= z= 1 .
bo

Bài 17. Cho x,y,z là các số thực dương thỏa mãn điều kiện y +=
z x y2 + z2 . ( )
et

Tìm giá trị nhỏ nhất của biểu thức


1 1 1 4
vi

P= + + + .
( x + 1) ( y + 1) ( z + 1) ( x + 1)( y + 1)( z + 1)
2 2 2
ng

Lời giải

Theo giả thiết ta có: y +=


1
( 2
) 2
z x y2 + z2 ≥ x ( y + z ) ⇒ y + z ≤ .
a

2 x
kh

Sử dụng bất đẳng thức AM-GM ta có:


1 1 2 8 8 2 x2
+ ≥ ≥ ≥ =
( y + 1)2 ( z + 1)2 ( y + 1)( z + 1) ( y + z + 2 )2 2
+

2
( x + 1)2
 2 
x 
1 4 4 x2
≥ ≥ = .
(1 + x )(1 + y )(1 + z ) (1 + x )( y + z + 2 )2 2 
2
( x + 1)
3
(1 + x )  + 2 
x 

633
Khám phá tư duy Kỹ thuật giải bất ĐT Bài toán Max – Min – Đặng Thành Nam

1 2 x2 4 x2 2 x3 + 6 x 2 + x + 1
Suy ra P ≥ + + = 3 .
(1 + x )2 (1 + x )2 (1 + x )3 (1 + x )
2 x3 + 6 x 2 + x + 1
Xét hàm số f ( x) = với x > 0 ta có:
(1 + x )3
2 ( 5 x − 1) 1
f '( x) = ; f '( x) = 0 ⇔ x = .
(1 + x ) 4 5
1
Ta có f’(x) đổi dấu từ âm sang dương khi đi qua x = nên f(x) đạt cực tiểu tại
5

n
1  1  91
x= hay P ≥ f ( x) ≥ f   =.

.v
5  5  108

om
1
Đẳng thức xảy ra khi và chỉ khi x= , y= z= 5 .
5
91 1
Vậy giá trị nhỏ nhất của P bằng đạt tại x= , y= z= 5 . .c
108 5
Bài 18. Cho a,b,c là các số thực không âm thỏa mãn điều kiện a > b > c và
ok
3ab + 5bc + 7ca ≤ 9 . Tìm giá trị nhỏ nhất của biểu thức
32 1 1
P= + + .
bo

( a − b ) ( b − c ) ( c − a )4
4 4

Lời giải
et

Theo giả thiết ta có a > b > c ≥ 0 ⇒ 9 ≥ 7ca + 5bc + 3ab ≥ 3ab ⇒ ab ≤ 3 .


Khi đó:
vi

32 a 2b 2  321 11 1
P≥ + ++ + 

ng

( a − b )4 a 4 b4 9  ( a − b )4 a 4 b4 
 
a

 2 2 2 2
  2 
1 32a b a b  1 32 a b 
= =
+ + + + −
kh

    2
( 
) b a 
2 2 2 2
a  9 a b
  + − 2 
9 a 2 − 2ab + b 2 b
  
  b a  

a b 1  32 
Đặt t =+ , ( t > 2 ) do a > b khi đó P=
≥ f (t )  + t 2
− 2 .
b a 9  ( t − 2 )2 

1  32 
=
Xét hàm số f (t )  + t 2
− 2  với t > 2 ta có:
9  ( t − 2 )2 

634
Cty TNHH MTV DVVH Khang Việt

1 64 
f '(t ) = − + 2t  ; f '(t ) =
0
9  ( t − 2 )3 
 
⇔ 2t ( t − 2 ) = 64 ⇔ t = 4
3

Ta có f’(t) đổi dấu từ âm sang dương khi đi qua t = 4 nên tại t = 4 f(t) đạt cực
10
tiểu hay P ≥ f (t ) ≥ f (4) = .
9
Đẳng thức xảy ra khi và chỉ khi:
 =
c = 0 a 6+3 3

n

ab = 3

⇔ b = 2 + 3 2 3 − 3 . ( )

.v
a b 
 + = 4 c = 0

om
b a
10
Vậy giá trị nhỏ nhất của P bằng đạt tại
9 .c
(
a = 6 + 3 3 , b = 2 + 3 2 3 − 3 , c =0 . )
ok
Bài 19. Cho a,b,c là các số thực không âm thoả mãn c = min {a, b, c}
và a 2 + b 2 + c 2 =
bo

3 . Tìm giá trị lớn nhất của biểu thức


6
P = 2ab + 3bc + 3ca + .
a+b+c
et

Lời giải
Trước hết ta có: 2ab + 3bc + 3ca ≤ ( a + b + c ) − 1
vi

2
ng

6
Suy ra P ≤ ( a + b + c ) +
2
−1 .
a+b+c

( ) 6
a

Đặt t = a + b + c, 3 < t ≤ 3 khi đó P ≤ f (t ) = t 2 + −1.


t
kh

Xét hàm số f (t ) = t 2 +
6
t
− 1 với t ∈ ( 3;3 ta có:

f '(t )= 2t −
t2
6
(
> 0, ∀t ∈ 3;3 ⇒ P ≤ f (t ) ≤ f (3)= 10 .

Vậy giá trị lớn nhất của P bằng 10 đạt tại a= b= c= 1 .


Nhận xét. Bất đẳng thức 2ab + 3bc + 3ca ≤ ( a + b + c ) − 1 có được nhờ dự đoán
2

dấu bằng đạt tại a= b= c= 1 và ý tưởng đưa về khảo sát hàm số với t = a + b + c .

635
Khám phá tư duy Kỹ thuật giải bất ĐT Bài toán Max – Min – Đặng Thành Nam

Chứng minh.
( a + b + c )2 − 1 − 2ab − 3bc − 3ca = 2 − bc − ca

( )
= 2 − c ( a + b ) ≥ 2 − c 2 a 2 + b2 = 2 − c 2 3 − c2 ( )
(
= 2 − 2c 2 3 − c 2 ≥ 2 − ) 2c 2 + 3 − c 2 1 − c 2
2
=
2
≥0

Bài tập tương tự


Cho a,b,c là các số thực không âm có c = min {a, b, c} và a 2 + b 2 + c 2 =
3.
2ab + 3bc + 3ca 6

n
Tìm giá trị lớn nhất=
của biểu thức P − .
2 ( a + b + 2c )2 − 17

.v
Bài 20. Cho a,b,c là các số thực dương thỏa mãn điều kiện abc ( a + b + c ) =4.

om
1 8bc
=
Tìm giá trị nhỏ nhất của biểu thức P − .
( a + b )( a + c ) ( )
bc b + c 2 + 8
2

Lời giải
.c
Nhận xét. Nhìn vào biểu thức P cho ta thấy P đối xứng với b và c nên ta tìm cách
ok
đánh giá xoay quanh hai biến này. Và dễ thấy áp dụng AM-GM cho a + b, a + c .
Sử dụng bất đẳng thức AM-GM ta có:
bo

a + b + a + c 2a + b + c
( a + b )( a + c ) ≤ = .
2 2
et

Vậy tìm cách đánh giá theo 2a + b + c .


8bc 8 8
= = 2
vi

Ta có:
( )
bc b 2 + c 2 + 8 b 2 + c 2 + 8 b + c + 2a ( a + b + c )
2
ng

bc
8 16
≤ =
( b + c ) 2 + 2a ( b + c ) + 2a 2 ( 2a + b + c )
1 2
a

2
kh

2 16
Suy ra P ≥ − .
2a + b + c ( 2a + b + c )2
2 16
Đặt t = 2a + b + c, ( t > 0 ) khi đó P ≥ f (t ) =− 2 .
t t
16 2 16
Ta có: t 2 = 4a 2 + 4a ( b + c ) + ( b + c ) =
+ ( b + c ) ≥ + 4bc ≥ 16 ⇒ t ≥ 4 .
2
bc bc
2 16 2 32
Xét hàm số f (t )= − với t ≥ 4 ta có f '(t ) =− 2 + 3 ; f '(t ) =0 ⇔ t =16 .
t t2 t t

636
Cty TNHH MTV DVVH Khang Việt

1
Lập bảng biến thiên suy ra P ≥ f (t ) ≥ f (4) =− .
2
Đẳng thức xảy ra khi a = 2 − 2, b =c= 2.
Bài 21. Cho a,b,c là các số thực dương thỏa mãn điều kiện a ≥ b ≥ c .

Tìm giá trị nhỏ nhất=


của biểu thức P
a+b
+
(
b a 2 + ac + c 2 ).
ab 2 + bc 2 + ca 2 b+c
Lời giải
Do a ≥ b ≥ c > 0 nên ta có:
( b − a )( b − c ) ≤ 0 ⇔ b2 + ac ≤ ab + bc ⇒ ab2 + ca 2 ≤ a 2b + abc

n
⇒ ab 2 + bc 2 + ca 2 ≤ a 2b + abc + bc 2 = b a 2 + ac + c 2 ( )

.v
Mặt khác: a + b ≥ 2b, b + c ≤ 2b suy ra:

om
P≥
2b
+
(
b a 2 + ac + c 2 )
(
b a 2 + ac + c 2 ) 2b .c
1 a 2 + ac + c 2
ok
b
=2 + ≥2
a 2 + ac + c 2 2 b
bo

b 1 4
Đẳng thức xảy ra khi và chỉ khi a = b = c, = ⇔ a =b =c = .
a + ac + c 2 2 2 3
4
et

Vậy giá trị nhỏ nhất của P bằng 2 đạt tại a= b= c= .


3
Bài 22. Cho a,b,c là các số thực dương thỏa mãn điều kiện a + b + c =
vi

1.
a2 b2 4c 2
Tìm giá trị nhỏ nhất của biểu thức P = + +
ng

.
1 + b 1 + a 2 + 2a 2 + 2b 2
Lời giải
a

( )
2
4 1 − 2 a 2 + b 2 
kh

( )
2
≥  
2 4c
Ta có c = 1 − ( a + b ) ≥ 1 − 2 a 2 + b ⇒
(
2 + 2 a 2 + b2 ) 2+ 2(a 2
+b )
2

( )
2
a2 b2 a4 b4 a 2 + b2
+ = + ≥
1 + b 1 + a a 2 + a 2b b 2 + ab 2 a 2 + b 2 + ab ( a + b )

(a ) ( )
2
2
+ b2 2 a 2 + b2
≥ =
a 2 + b2 + (a
1
2
2
+ b2 ) 2(a 2
+ b2 ) 2 + 2 a 2 + b2 ( )
637
Khám phá tư duy Kỹ thuật giải bất ĐT Bài toán Max – Min – Đặng Thành Nam

( )
2

(
2 a +b
2 2
) 4 1 − 2 a 2 + b 2 
Suy ra P ≥ +  
(
2 + 2 a 2 + b2 ) 2+ 2(a 2
+b ) 2

t 2 + 4 (1 − t )
2
Đặt t = ( )
2 a 2 + b 2 , ( t > 0 ) ta có P=
≥ f (t ) =
t+2
5t 2 − 8t + 4
t+2
5t 2 − 8t + 4
Xét hàm số f (t ) = với t > 0 ta có
t+2
(
5 t 2 + 4t − 4 )
; f '(t ) =0 ⇔ t =−2 + 2 2 ( t > 0 ) .

n
f '(t ) =
( t + 2 )2

.v
Ta có f’(t) đổi dấu từ âm sang dương khi đi qua t =−2 + 2 2 nên f(t) đạt cực

om
tiểu tại t =−2 + 2 2 hay P ≥ f (t ) ≥ f −2 + 2 =
2 20 2 − 28 . ( )

 2 a + b =−2 + 2 2
2 2
(
a =b =−1 + 2
)
.c

Đẳng thức xảy =
ra khi và chỉ khi a b ⇔
ok
a + b + c =
1 c= 3 − 2 2


bo

Vậy giá trị nhỏ nhất P bằng 20 2 − 28 đạt tại a =b =−1 + 2, c =3 − 2 2 .


Bài 23. Cho x,y,z là các số thực không âm. Tìm giá trị lớn nhất của biểu thức
et

1 5
= P − .
x + 2 y + 3 z + 1 ( x + 2 y )( 3z + 1)
vi

Lời giải
ng

Sử dụng bất đẳng thức AM-GM ta có:


2
x + 2 y + 3z + 1 
( x + 2 y )( 3z + 1) ≤  
a

 2 
kh

Đẳng thức xảy ra khi và chỉ khi x + 2 y =3 z + 1


1 20
⇒P≤ −
x + 2 y + 3 z + 1 ( x + 2 y + 3 z + 1)2
2
 1 1  1 1
=
−20  −  + ≤
 x + 2 y + 3 z + 1 40  80 80
19 1 1
Với x + 2 y = 20; z = thì P bằng . Vậy GTLN của P bằng .
3 80 80

638
Cty TNHH MTV DVVH Khang Việt

1 
Bài 24. Cho a,b,c là các số thực dương thuộc đoạn  ;1 .
2 
a b c
=
Tìm giá trị lớn nhất của biểu thức P . + .
b+c a+c a+b
Lời giải

( c + a )( c + b ) ≥ ( c + )=
2
ab 2 2
 c   2c 
 + 1 ≥  + 1
ab ab  ab   a + b 
c 1
⇒P≤ +
a + b  2c 
2

n
 + 1
a+b 

.v
c 1
Đặt
= t , ≤ t ≤ 1.

om
a+b 4
1
Ta có P ≤ f (t ) =
f (t ) =
t+ .
( 2t + 1)2 .c
1 1 
Xét hàm số f (t ) = t + liên tục trên đoạn  ;1 ta có
ok
( 2t + 1) 2
4 

( 2t + 1)3 − 4 0, t  1 ;1 .
bo

4
f '(t ) = 1 − = > ∀ ∈ 
( 2t + 1)3 ( 2t + 1)3 4 
et

10 1
Do đó Pmax = f (1) = ⇔ a = b = ; c =1
9 2
vi

Bài 25. Cho a,b,c là các số thực dương thỏa mãn điều kiện a + b + c =
1.
P a3b3c3 − ( a − bc )( b − ca )( c − ab ) .
Tìm giá trị nhỏ nhất của biểu thức=
ng

Lời giải
Ta chứng minh 8a b c ≥ ( a − bc )( b − ca )( c − ab ) .
a

2 2 2
kh

a − bc > 0
Không mất tính tổng quát giả sử a ≥ b ≥ c khi đó do a, b, c ∈ ( 0;1) ⇒ 
b − ca > 0
Nếu c − ab < 0 bất đẳng thức luôn đúng.
Nếu c − ab ≥ 0 khi đó ta chứng minh 2ab ≥ ( a − bc )( b − ca )
Thật vậy 2ab ≥ ( a − bc )( b − ca ) ⇔ 4a 2b2 ≥ ( a − bc )( b − ca )
( )
⇔ 4a 2b 2 ≥ ab − a 2 c − b 2 c + abc 2 ⇔ 4a 2b 2 + c a 2 + b 2 − ab c 2 + 1 ≥ 0 ( )
2
( )
⇔ 4a 2b 2 + c ( a − b ) + ab 2c − c 2 − 1 ≥ 0 ⇔ 4a 2b 2 + c ( a − b ) − ab ( c − 1) ≥ 0
2 2

639
Khám phá tư duy Kỹ thuật giải bất ĐT Bài toán Max – Min – Đặng Thành Nam

⇔ ab  4ab − ( a + b )  + c ( a − b ) ≥ 0 ⇔ ( c − ab )( a − b ) ≥ 0 (luôn đúng).


2 2 2
 
Tương tự ta có 2bc ≥ ( b − ca )( c − ab ) ;2ca ≥ ( c − ab )( a − bc ) .
Nhân theo vế 3 bất đẳng thức trên ta có ngay điều phải chứng minh.
1
Đẳng thức xảy ra khi và chỉ khi a= b= c= .
3
Khi đó P ≥ a3b3c3 − 8a 2b 2 c 2 .
 a+b+c
3
1 
Đặt
= t abc,  0 < t ≤  =  ta có P ≥ f (t ) =
t 3 − 8t 2 .
  3  
27 

n
 1

.v
) t 3 − 8t 2 với t ∈  0;  ta có
Xét hàm số f (t=
 27 

om
 1
f '(t= t t ( 3t − 16 ) < 0, ∀t ∈  0;  .
) 3t 2 − 16=
 27 
 1 .c
Do đó f(t) là hàm nghịch biến trên  0;  .
 27 
ok
 1  215
Suy ra P ≥ f (t ) ≥ f   = − .
 27  19683
bo

1
Đẳng thức xảy ra khi và chỉ khi a= b= c= .
3
et

215 1
Vậy giá trị nhỏ nhất của P bằng − đạt tại a= b= c= .
19683 3
vi

Bài 26. Cho a,b,c là các số thực dương. Tìm giá trị lớn nhất của biểu thức
ng

25 5 36
=P − .
( a + 1)(b + 1)( c + 1)
2 2 2 a+b+c
a

Lời giải
kh

( )( )(
Sử dụng bất đẳng thức: a 2 + 1 b 2 + 1 c 2 + 1 ≥ ) 5
16
( a + b + c + 1)2 .
Chứng minh xem chương 2.
100 36
Khi đó P ≤ − .
a + b + c +1 a + b + c
100 36
Đặt t = a + b + c khi đó P ≤ f (t ) = − .
t +1 t
100 36 3
Ta có f '(t ) =− + 2 ; f '(t ) =0 ⇔ 6 ( t + 1) =10t ⇔ t = .
( t + 1) t
2 2

640
Cty TNHH MTV DVVH Khang Việt

3 3
Ta có f’(t) đổi dấu từ dương sang âm khi đi qua t =
nên f(t) đạt cực đại tại t = .
2 2
3 1
16 . Đẳng thức xảy ra khi và chỉ khi a= b= c=
Suy ra P ≤ f (t ) ≤ f   = .
2 2
1
Vậy giá trị lớn nhất của P bằng 16 đạt tại a= b= c= .
2
Bài 27. Cho x,y,z là độ dài 3 cạnh một tam giác.
Tìm giá trị nhỏ nhất của biểu thức
24 ( y + z − x ) 24 ( z + x − y ) 24 ( x + y − z )
P = 1+ + 1+ + 1+ .

n
x y z

.v
Lời giải
Đặt a = x + y − z , b = y + z − x, c = x + z − y khi đó:

om
48a 48b 48c
P = 1+ + 1+ + 1+ .
b+c c+a a+b
Không mất tính tổng quát giả sử c = min {a, b, c} . .c
Sử dụng bất đẳng thức Mincopski ta có:
ok
2 2
48a 48b  48a 48b   a b 
1+ + 1+ ≥ 22 +  +  = 4 + 48  +  .
b+c c+a + + + +
bo

 b c c a   b c c a 
a b a+b
Mặt khác: + ≥2 .
b+c c+a a + b + 2c
et

2
 a b   2
  a ( b + c ) + b ( c + a )  ≥ ( a + b ) .
vi

3
Thật vậy ta có:  + 2

 b+c c+a 
ng

a (b + c ) + b (c + a )
2 2

( a + b)
3

c (a + b)
2
=−
( a − b ) ( a + b − 2c )
2
≤0
4 2 4
a

⇒ a 2 ( b + c ) + b2 ( c + a ) ≤
(a + b) 3
+
c (a + b)
2
kh

4 2

Do đó
(a + b) 3

( a + b )3 =
4(a + b)
.
a 2 ( b + c ) + b 2 ( c + a ) ( a + b )3 c ( a + b )2 a + b + 2c
+
4 2
a b a+b
Suy ra + ≥2 .
b+c c+a a + b + 2c
48 ( a + b ) 48c
Vậy P ≥ 2 1 + + 1+ .
a + b + 2c a+b

641
Khám phá tư duy Kỹ thuật giải bất ĐT Bài toán Max – Min – Đặng Thành Nam

48c 48 ( a + b ) t 2 + 2t + 1176
Đặt t + 1= 1+ , (0 < t ≤ 4) ⇒ 1 + =
a+b a + b + 2c t 2 + 2t + 24
t 2 + 2t + 1176
Suy ra P ≥ 2 + t + 1.
t 2 + 2t + 24
t 2 + 2t + 1176 t 2 + 2t + 1176
Ta chứng minh 2 + t + 1 ≥ 15 ⇔ 2 ≥ 14 − t
t 2 + 2t + 24 t 2 + 2t + 24
( ) 2
( )
⇔ 4 t 2 + 2t + 1176 ≥ (14 − t ) t 2 + 2t + 24 ⇔ t (18 − t )( t − 4 ) ≥ 0 (luôn đúng).
2

Đẳng thức xảy ra khi và chỉ khi t = 4 ⇔ 2c = a + b ⇔ a = b = c ⇔ x = y = z .

n
Vậy giá trị nhỏ nhất của P bằng 15 đạt tại x= y= z .

.v
1 3
Bài 28. Cho a,b,c là các số thực thuộc đoạn  ;  và thoả mãn điều kiện

om
2 2
a+b+c = 3 . Tìm giá trị nhỏ nhất của biểu thức
1 1 1
(
P= 7  + +  − 6 a 4 + b 4 + c 4 .
a b c
.c )
ok
Lời giải
1 3
Vì a, b, c ∈  ;  ⇒ a + b − c = 3 − 2c ≥ 0 . Tương tự b + c − a ≥ 0, c + a − b ≥ 0 .
2 2
bo

Do đó
( ) ( )
2 a 2b 2 + b 2 c 2 + a 2 a 2 − a 4 + b 4 + c 4 = ( a + b + c )( a + b − c )( b + c − a )( c + a − b ) ≥ 0 .
et

( ) (
Suy ra 2 a 2b 2 + b 2 c 2 + c 2 a 2 + a 4 + b 4 + c 4 ≥ 2 a 4 + b 4 + c 4 . )
vi

1 2
( )
ng

2
⇔ a 4 + b4 + c4 ≤ a + b2 + c2 .
2
1 1 1
( )
2
a

Suy ra P ≥ 7  + +  − 3 a 2 + b 2 + c 2 .
a b c
kh

3
Đặt a = y + z , b = z + x, c = x + y ⇒ x, y, z ≥ 0 và x + y + z = .
2
Ta có
1 1 1 1 1 1 2  1 1 1 
+ += + + = .( x + y + z )  + + 
a b c y+z z+x x+ y 3  y+z z+x x+ y
2 x y z  2 x y z 
=  + + + 3 ≥ 2 +  + + 
3 y + z z + x x+ y  3 y + z z + x x+ y 

642
Cty TNHH MTV DVVH Khang Việt

x x2 x2
= ≥
y + z x ( y + z ) xy + yz + zx
y y2 y2
= =
z + x y ( z + x ) xy + yz + zx
z z2 z2
= =
x + y z ( x + y ) xy + yz + zx
x y z x2 + y 2 + z 2 9
Suy ra + + ≥ = −2
y + z z + x x + y xy + yz + zx 4 ( xy + yz + zx )

n
( ) ( )
2 2
Và a 2 + b 2 + c 2 = 4 x 2 + y 2 + z 2 + xy + yz + zx

.v
2
2 9 
= 4 ( x + y + z ) − ( xy + yz + zx )  = 4  − xy − yz − zx 
2

om
  4 
 2 9  9 
2
Suy ra P ≥ 7  2 +  − 2   − 12  − xy − yz − zx 
3  4 ( xy + yz + zx )
   4
.c 
2
ok
14 63 9 
= + − 12  − xy − yz − zx 
3 6 ( xy + yz + zx ) 4 
bo

 ( x + y + z)
2
3
Đặt t = xy + yz + zx,  0 < t ≤ =  khi đó
 3 4
 
et

2
14 63 9 
P ≥ f (t ) = + − 12  − t  .
vi

3 6t 4 
ng

2
14 63 9   3
Xét hàm số f (t ) = + − 12  − t  với t ∈  0;  ta có
3 6t 4   4
( ) ; f '(t ) = 0 ⇔ 16t
a

3 16t 3 − 36t 2 + 7 1 3
f '(t ) = − 3
− 36t 2 + 7 = 0 ⇔ t = (do 0 < t ≤ ).
kh

2 2 4
2t
1
Ta có f’(t) đổi dấu từ âm sang dương khi đi qua t = nên f(t) đạt cực tiểu tại
2
1 1 133
t= hay P ≥ f (t ) ≥ f   =
− .
2 2 12

643
Khám phá tư duy Kỹ thuật giải bất ĐT Bài toán Max – Min – Đặng Thành Nam

 3
x + y + z = 2

 1  1 
Đẳng thức xảy ra khi và chỉ khi  xy + yz + zx = ⇔ ( x; y; z ) = 0; ;1 hoặc
 2  2 
 xyz = 0


1 3
các hoán vị suy ra ( a; b; c ) =  ;1;  hoặc các hoán vị.
2 2
133 1 3
Vậy giá trị nhỏ nhất của P bằng − đạt tại =
a ,=
b 1,=
c hoặc các hoán vị.

n
12 2 2
Bài 29. Cho a,b,c là các số thực thuộc đoạn [ 0;2] .

.v
Tìm giá trị lớn nhất của biểu thức

om
P= ab(a 2 − b 2 ) + bc(b 2 − c 2 ) + ca (c 2 − a 2 ) .
Lời giải .c
Không mất tính tổng quát, giả sử a = max{a, b, c}
ok
Ta có :
P= ab(a 2 − b 2 ) + c(b3 − a3 ) + c3 (a − b)= ab(a 2 − b 2 ) + c(b − a )(a 2 + ab + b 2 − c 2 )
bo

Vì a ≥ b ≥ 0, a ≥ c ≥ 0 nên c(b − a )(b 2 + ab + a 2 − c 2 ) ≤ 0

Suy ra: P ≤ ab(a 2 − b 2 ) ≤ 2b(4 − b 2 ) (do b ∈ [ 0;2] )


et

Mặt khác : F = 2b(4 − b 2 ) ≥ 0 .


vi

Ta có F 2= 4b 2 (2 − b 2 )(4 − b 2 )= 2.  2b 2 .(4 − b 2 ).(4 − b 2 ) 


 
ng

Áp dụng bất đẳng thức AM-GM ta có:


a

3
 2b 2 + 4 − b 2 + 4 − b 2  512 32 3
F ≤ 2. 
2
= 2. ⇔0≤ F ≤

kh

 3  27 9

2 3
Dấu bằng xảy ra khi=
a 2,=
b =
, c 0.
3
32 3
Vậy giá trị lớn nhất của biểu thức P = , đạt được chẳng hạn khi
9
2 3
=
a 2,=
b =
, c 0.
3

644
Cty TNHH MTV DVVH Khang Việt

Bài 30. Cho a,b,c là các số thực dương thuộc đoạn [1;2] và thỏa mãn điều kiện
( a + b + c )=
2
4 ( ab + bc + ca − 1) . Tìm giá trị nhỏ nhất của biểu thức
a+b−c b+c−a c+ a−b 2
P= + + +2 2 .
c a b a + b + c2 − 2
2

Lời giải
Đặt x = a + b − c, y = b + c − a, z = a + c − b, ( x, y, z > 0 )
Theo giả thiết ta có: xy + yz + zx =
4.
 x y z  4
Khi đó=
P 2  + + + .
x + y 

n
 y+z z+x x2 + y 2 + z 2

.v
2
 x y z 
Ta có:  + + 

om
 y+z z+x x + y 

x y z  xy yz zx 
= + + + 2 + + .
y+z z+x x+ y 
 ( y + z )( x + z ) .c ( y + x )( z + x ) ( z + y )( x + y ) 
 x z 
ok
x y z 1 y
Ta có: + + = ( xy + yz + zx )  + + 
y+z z+x x+ y 4  y+z z+x x+ y
bo

x 2 + y 2 + z 2 xyz  x y z  x2 + y 2 + z 2
= +  + + ≥ .
4 4  y+z z+x x+ y 4
et

xy xy 2 xy xy
= ≥ ≥ .
( y + z )( x + z ) ( xy + zx )( xy + yz ) 2 xy + yz + zx xy + yz + zx
vi

yz yz zx zx
Tương tự: ≥ ≥
ng

, .
( y + x )( z + x ) xy + yz + zx ( z + y )( x + y ) xy + yz + zx
Cộng theo vế ba bất đẳng thức trên ta được:
a

xy yz zx
+ + ≥ 1.
kh

( y + z )( x + z ) ( y + x )( z + x ) ( z + y )( x + y )
x y z x2 + y 2 + z 2
Do đó + + ≥ +2 .
y+z z+x x+ y 4

x2 + y 2 + z 2 4
Suy ra: P ≥ +4+ .
2 x + y2 + z2
2

Đặt t = ( )
x 2 + y 2 + z 2 , t ≥ xy + yz + zx = 2 khi đó P ≥ f (t )=
t2
2
4
+4+ .
t

645
Khám phá tư duy Kỹ thuật giải bất ĐT Bài toán Max – Min – Đặng Thành Nam

t2 4
Xét hàm số f (t )= + 4 + với t ≥ 2 ta có:
2 t

f '(t ) =
t

4
(
; f '(t ) = 0 ⇔ t 3 = 4 2 t 2 + 8 ⇔ t = 2 2 . )
( )
2
2 t +8
2 t

Ta có f’(t) đổi dấu từ âm sang dương khi đi qua t = 2 2 nên f(t) đạt cực tiểu tại
t = 2 2 hay P ≥ f (t ) ≥ f (2 2) =
3 2.
Vậy giá trị nhỏ nhất của P bằng 3 2 đạt tại = a 1,= b 2,= c 1 hoặc các hoán vị.
Bài 31. Cho các số thực dương a, b, c thỏa mãn điều kiện abc = 1 .

n
1
Tìm giá trị nhỏ nhất của biểu thức P = a 2b + b 2 c + c 2 a +

.v
.
a + b3 + c 3
6 3

om
Lời giải
a b c
Đặt x = a 2b =a.ab = ; y = b2c = b.bc = ; z = c2a = c.ca = ⇒ xyz = 1.
c a b .c
1
Vì abc = 1 nên ta biến đổi được P thành P = x + y + z +
xy + yz 2 + zx 2
ok
6 2

Ta có x 2 y + y 2 z + z 2 x ≥ 3 3 x 2 y. y 2 z.z 2 x =
3.
bo

Sử dụng bất đẳng thức AM – GM ta có


1 6
3.6 x 2 y + y 2 z + z 2 x
et

=
6
xy 2 + yz 2 + zx 2 6
( xy 2
+ yz 2 + zx 2 )( x 2
)
y + y 2 z + z 2 x .3xyz
vi

3
3

ng

6
( xy 2
+ yz + zx
2 2
)( x 2
)
y + y 2 z + z 2 x .3xyz
Sử dụng bất đẳng thức AM-GM ta được:
a

3
 x 2 y + y 2 x + y 2 z + z 2 y + z 2 x + x 2 z + 3xyz 
kh

( xy 2
+ yz + zx
2 2
)( x y + y z + z x .3xyz ≤ 
2 2

2
) 3

 
3
 ( x + y + z )( xy + yz + zx)  ( x + y + z )
9
=   ≤
 3  27 2
3.3 3
Suy ra P ≥ x + y + z +
( x + y + z )3/2
3.3 3
Đặt t =x + y + z ,(t ≥ 3) ⇒ P ≥ f (t ) =t +
t 3/2

646
Cty TNHH MTV DVVH Khang Việt

3.3 3 1
t + 3/2 ; t ≥ 3 ta thu được: P ≥ 3 + 6 .
Xét hàm số f (t ) =
t 3
Đẳng thức xảy ra khi và chỉ khi a= b= c= 1 .
1
Vậy giá trị nhỏ nhất của P bằng 3 + 6 đạt tại a= b= c= 1 .
3
Bài 32. Cho a,b,c là các số thực không âm thoả mãn điều kiện a 2 + b 2 + c 2 =
2.
Tìm giá trị lớn nhất của biểu thức
a b a + b − 2ab
P= 2 + 2 − .
a + b + c b + c + a ( a + b + c )2

n
Lời giải

.v
( )
Sử dụng bất đẳng thức C –S ta có a 2 + b + c (1 + b + c ) ≥ ( a + b + c )
2

om
a a (1 + b + c )
⇒ ≤ .
a2 + b + c ( a + b + c )2 .c
b b (1 + c + a )
Tương tự ta có ≤ .
( a + b + c )2
ok
b +c+a
2

a ( b + c ) + b ( c + a ) + 2ab
Từ đó suy ra P≤ ≤1.
bo

( a + b + c )2
Bởi vì 2 + 2bc = a 2 + ( b + c ) ≥ 2a ( b + c ) ⇒ a ( b + c ) ≤ 1 + bc
2
et

2 + 2ca = b 2 + ( c + a ) ≥ 2b ( c + a ) ⇒ b ( c + a ) ≤ 1 + ca
2
vi

⇒ a ( b + c ) + b ( c + a ) + 2ab ≤ 2 + c ( a + b ) + 2ab
ng

= a 2 + b 2 + c 2 + c ( a + b ) + 2ab ≤ ( a + b + c )
2

Vậy giá trị lớn nhất của P bằng 1 đạt tại a= b= 1, c= 0 .


a

Bài tập tương tự


kh

Cho a,b,c là các số thực dương có tổng bằng 3. Chứng minh


1 1 1
+ 2 + 2 ≤ 1.
a +b+c b +c+a c +a+b
2

Bài 33. Cho a,b,c là các số thực dương thoả mãn điều kiện

( a + 5b + 3c ) 
1 1 1  39
+ + =.
 a 4b 3c 4

2 ( a + 3c )( a + 3c − 4b ) − 7c 2 + 4c a 2 + 7c 2
Tìm giá trị nhỏ nhất của biểu thức P = .
2ac

647
Khám phá tư duy Kỹ thuật giải bất ĐT Bài toán Max – Min – Đặng Thành Nam

Lời giải
Theo giả thiết ta có
39 1 1 1  1 1 1  1 1 1 
=( a + 4b + 3c )  + +  + b + +  ≥ 9 + b + + 
4  a 4b 3c   a 4b 3c   a 4b 3c  .
⇒ 3ac ≥ 2b ( a + 3c )
1 1 1
Chú ý. AM – GM ta có ( a + 4b + 3c )  + + ≥9.
 a 4b 3c 
Khi đó
2 ( a + 3c ) − 8b ( a + 3c ) − 7c 2 + 4c a 2 + 7c 2
2

n
P=
2ac

.v
2 ( a + 3c ) − 12ac − 7c 2 + 4c a 2 + 7c 2
2
2a 2 + 11c 2 + 4c a 2 + 7c 2
≥ =

om
2ac 2ac
2 x 2 + 11 + 4 x 2 + 7 a
= ,x
2x c .c
2 x 2 + 11 + 4 x 2 + 7
Xét hàm số f ( x) = với x dương ta có
ok
2x

( 2x 2
− 11) x 2 + 7 − 28
bo

f '( x) = ; f '( x) = 0 ⇔ x = 3 .
2 x2 x2 + 7
et

15
Ta có f’(x) đổi dấu từ âm sang dương khi đi qua x=3 nên f ( x) ≥ f (3) = .
2
vi

Dấu bằng xảy ra khi và chỉ khi =a 4= b 3c .


15
ng

Vậy giá trị nhỏ nhất của P bằng .


2
Bài tập tương tự
a

1) Cho a,b,c là các số thực dương thoả mãn điều kiện


kh

( a + 3b + c ) 
1 1 1  21
+ +  =.
 a 2b c  2
a 2 + ( a + c )( a + c − 2b ) + 2a 2 + 2c 2
Tìm giá trị nhỏ nhất của biểu thức P = .
ac
ĐS: Pmin
= 5;=
a 2=
b c.
2) Cho a,b,c là các số thực dương thoả mãn điều kiện

( 3a + 2b + c ) 
1 2 3
+ + =30 .
a b c

648
Cty TNHH MTV DVVH Khang Việt

b + 2c − 7 72a 2 + c 2
Tìm giá trị nhỏ nhất của biểu thức P = .
a
ĐS: Pmax =
−55;2a =
3b =
6c .
Bài 34. Cho x, y, z là các số thực không âm thỏa mãn điều kiện

x2 + y 2 + 2 x + 2 y + 1 + z =3.

Tìm giá trị lớn nhất của biểu thức P = x 4 + y 4 + z 4 .


Lời giải
Trước hết ta chứng minh: x2 + y 2 + 2 x + 2 y + 1 ≥ 1 + x2 + y 2 .

n
Thật vậy bất đẳng thức tương đương với:

.v
x2 + y 2 + 2 x + 2 y + 1 ≥ 1 + x2 + y 2 + 2 x2 + y 2 .

om
⇔ x + y ≥ x 2 + y 2 ⇔ xy ≥ 0 , bất đẳng thức luôn đúng.
Dấu bằng xảy ra khi và chỉ khi xy = 0 .
.c
Suy ra 3 − z ≥ 1 + x 2 + y 2 ⇔ x 2 + y 2 ≤ 2 − z ⇔ x 2 + y 2 ≤ ( 2 − z ) và 0 ≤ z ≤ 2 .
2

( ) ( )
ok
2 2
+ z4 ≤ (2 − z ) + z4 .
4
P=x 4 + y 4 + z 4 =x 2 + y 2 + z 4 − 2 x2 y 2 ≤ x2 + y 2

(
Xét hàm số f ( z ) =( 2 − z ) + z 4 , ta có f '( z ) = 4 z 3 − ( 2 − z )
4 3
) = 0 ⇔ z =1.
bo

Ta có= f (0) 16,= f (1) 2,=


f (2) 16 .
et

Suy ra P ≤ f ( z ) ≤ 16 .
 xy = 0
vi

 x = 0
  z = 2 
ng

Dấu bằng xảy ra khi và chỉ khi   ⇔ y = 0 hoặc


  z = 0 z = 2
 2 
 x + y 2
+ 2 x + 2 y + 1 + z =3
a

các hoán vị.


kh

Vậy P lớn nhất bằng 16 khi và chỉ khi ( x, y, z ) = ( 0,0, 2 ) và các hoán vị.
Bài 35. Cho a,b,c là các số thực thỏa mãn điều kiện abc = 1 .
Tìm giá trị nhỏ nhất của biểu thức
5 −3 1 1
=P . 2 + 2 .
3 (a − a + 1)(b − b + 1) c − c + 1
2

Lời giải
Ta chứng minh với mọi số thực a và b ta luôn có
( )( ) (
3 1 − a + a 2 1 − b + b 2 ≥ 2 1 − ab + a 2b 2 . )
649
Khám phá tư duy Kỹ thuật giải bất ĐT Bài toán Max – Min – Đặng Thành Nam

Chứng minh. Viết lại bất đẳng thức dưới dạng:


(a 2
) ( )
− 3a + 3 b 2 − 3a 2 − 5a + 3 b + 3a 2 − 3a + 1 ≥ 0 .

Vế trái là tam thức bậc hai của b có a 2 − 3a + 3 > 0, ∀a ∈  và;

( ) ( )( ) ( )
2 2
∆b =3a 2 − 5a + 3 − 4 a 2 − 3a + 3 3a 2 − 3a + 1 =
− a 2 − 3a + 1 ≤ 0, ∀a ∈  .
Do đó vế trái luôn không âm và bài toán được chứng minh.
 3− 5
 a =
  2
a − 3a + 1 =   3− 5

n
2
0  b =
 
3a 2 − 5a + 3 ⇔ 
2

.v
Đẳng thức xảy ra khi và chỉ khi  .
b = 

 2 a 2 − 3a + 3  a = 3 +( ) 5

om
 2

 b = 3 + 5
  2 .c
(
Khi đó a 2 − a + 1 b 2 − b + 1 ≥ )( ) 23 ( a b
2 2
)
− ab +=
1
2 1 1 
 2 − +=
1 2.
c2 − c + 1
.
ok
3c c  3c 2
1 1 3kc 2 + 1 5 −3
Suy ra P ≤ + 2k . = với k = .
bo

c2 − c + 1 c − c +1 c − c +1
2 2 6
2.
3c 2
et

3kc 2 + 1
Xét hàm số f (c) = với c > 0 ta có
c2 − c + 1
vi

−3kc 2 + ( 6k − 2 ) c + 1
f '(c) = ; f '(c) = 0 ⇔ −3kc 2 + ( 6k − 2 ) c + 1= 0 .
(c )
ng

2
2
− c +1

7+3 5 3− 5
a

⇔c= hoặc c = .
2 2
kh

Bảng biến thiên:


c 3− 5 7+3 5
0 +∞
2 2
f’(c) + 0 − 0 +
5 −3
5 −1
2

f(c)
1− 5
1
3
650
Cty TNHH MTV DVVH Khang Việt

 7 + 3 5  1− 5
Dựa vào bảng biến thiên suy ra P ≥ f (c) ≥ f   = .
 2  3
3− 5 7+3 5
Đẳng thức xảy ra khi và chỉ khi a= b= , c= .
2 2
1− 5 3− 5 7+3 5
Vậy giá trị nhỏ nhất của P bằng đạt tại a= b= , c= .
3 2 2
Bài 36. Cho x,y,z là các số thực dương thỏa mãn điều kiện
( x − y )2 + ( y − z )2 + ( z − x )2 =
2.

n
Tìm giá trị nhỏ nhất của biểu thức
 1 1 1  2 ( x + y + z − 1)

.v
P = ( x + y + z ) + +  − .
x y z 3 xyz

om
Lời giải
Không mất tính tổng quát giả sử x = max { x, y, z} . Đặt y + z = 2t ⇒ x ≥ t .

Theo giả thiết ta có x 2 + y 2 + z 2 − xy − yz − zx =


1.
.c
⇔ ( x + y + z ) =1 + 3 ( xy + yz + zx ) (1) .
2
ok
xy + yz + zx 2 ( x + y + z − 1)
Khi đó P = ( x + y + z ) . −
bo

xyz 3 xyz
( x + y + z ) ( ( x + y + z )2 − 1) − 2 ( x + y + z − 1)
et

=
3 xyz
vi

=
( x + y + z )3 − 3 ( x + y + z ) + 2
ng

3 xyz

Và (1) ⇒ x 2 + ( y + z ) − x ( y + z ) − 3 yz =
2
1.
a

⇔ 3 yz = x 2 + 4t 2 − 2tx − 1 .
kh

Suy ra P
(=
x + 2t ) − 3 ( x + 2t ) + 2
3
( )
x3 + 6tx 2 + 12t 2 − 3 x + 8t 3 − 6t + 2
.
(
x x + 4t − 2tx − 1
2 2
) (
x x − 2tx + 4t 2 − 1
2
)
Mặt khác ta có:
1
2 = ( x − y) + ( x − z) + ( y − z) ≥ ( x − y) + ( x − z) ≥ ( 2 x − y − z )2 .
2 2 2 2 2
2
⇒ ( 2x − y − z ) ≤ 4 ⇔ 2x − y − z ≤ 2 ⇔ x ≤ t + 1 .
2

Ta chứng minh P ≥ 9 .

651
Khám phá tư duy Kỹ thuật giải bất ĐT Bài toán Max – Min – Đặng Thành Nam

Thật vậy bất đẳng thức tương đương với


( )
x3 + 6tx 2 + 12t 2 − 3 x + 8t 3 − 6t + 2
≥9
(
x x 2 − 2tx + 4t 2 − 1 )
( ) (
⇔ x3 + 6tx 2 + 12t 2 − 3 x + 8t 3 − 6t + 2 ≥ 9 x x 2 − 2tx + 4t 2 − 1 )
⇔ 8 x3 − 24 x 2t + 24 xt 2 − 8t 3 − 6 ( x − t ) − 2 ≤ 0 .

⇔ 4 ( x − t ) − 3 ( x − t ) − 1 ≤ 0 ⇔ ( x − t − 1)  2 ( x − t ) + 1 ≤ 0 (luôn đúng vì x ≤ t + 1 ).


3 2

y = z

n

 y+z x = 2

.v
Đẳng thức xảy ra khi và chỉ khi  x = t + 1 = +1 ⇔ .
 2  y= z= 1

om
( x − y )2 + ( y − z )2 + ( z − x )2 =
2

Vậy giá trị nhỏ nhất của P bằng 9 đạt tại ( x; y; z ) = ( 2;1;1) hoặc các hoán vị.
Cách 2: Ta chứng minh P ≥ 9 . .c
Viết lại bất đẳng thức dưới dạng:
ok
 1 1 1  2 ( x + y + z − 1)
( x + y + z ) + + − ≥9
x y z 3 xyz
bo

⇔ 3 ( x + y + z )( xy + yz + z ) + 2 − 27 xyz − 2 ( x + y + z ) ≥ 0
⇔ 3 ( x + y + z )( xy + yz + zx ) − 9 xyz  + 2 − 2 ( x + y + z ) ≥ 0
et

⇔ 3  x 2 ( y + z ) + y 2 ( z + x ) + z 2 ( x + y ) − 6 xyz  + 2 − 2 ( x + y + z ) ≥ 0
 
vi

⇔ 3  x 2 ( y + z ) + y 2 ( z + x ) + z 2 ( x + y ) − 6 xyz  +
ng

 
+2 − ( x + y + z ) ( x − y ) + ( y − z ) + ( z − x )  ≥ 0
2 2 2
 
a

⇔ ( 2 z − x − y )( x − y ) + ( 2 x − z − y )( z − y ) + ( 2 y − x − z )( x − z ) + 2 ≥ 0
2 2 2
kh

a + b + c =0
Đặt a = x − y; b = y − z; c = z − x ⇒  2 .
a + b + c =
2 2
2

Ta cần chứng minh ( c − b ) a 2 + ( b − a ) c 2 + ( a − c ) b 2 + 2 ≥ 0 .


⇔ M = ( a − b )( b − c )( a − c ) ≤ 2 .

( a b) (b − c ) (c − a ) .
Ta có M 2 =−
2 2 2

Trong 3 số a,b,c luôn có hai số cùng dấu không mất tính tổng quát giả sử hai số
đó là a và b, khi đó ab ≥ 0 .
652
Cty TNHH MTV DVVH Khang Việt

Và điều kiện trở thành: a 2 + b 2 + ( a + b ) =


2
2

⇔ a 2 + ab + b 2 =
1.
Khi đó
M2 =( a − b ) ( 2a + b ) ( 2b + a ) =(1 − 3ab )( 2 + 3ab )
2 2 2 2

3
2 + 3ab 2 + 3ab  1 − 3ab + 2 + 3ab 
= 4.(1 − 3ab ) . . ≤ 4  =
4
2 2  3 
Suy ra M ≤ 2 . Đẳng thức xảy ra khi và chỉ khi
= a 0  x= −y 0
ab = 0⇔ ⇔

n
= b 0  y= −z 0.

.v
⇒ ( x; y; z ) =
(1;1;2 ) ; ( 2;1;1)
Vậy giá trị nhỏ nhất của P bằng 9 đạt tại ( x; y; z ) = ( 2;1;1) hoặc các hoán vị.

om
.c
ok
bo
et
vi
a ng
kh

653
Khám phá tư duy Kỹ thuật giải bất ĐT Bài toán Max – Min – Đặng Thành Nam

Chương 4:
SỐ PHƯƠNG PHÁP CHỨNG MINH BẤT ĐẲNG THỨC KHÁC

CHỦ ĐỀ 1: KỸ THUẬT LƯỢNG GIÁC HÓA


Trong chủ đề này tôi đề cập đến một số phép lượng giác hoá cơ bản và một số
bất đẳng thức trong tam giác và ứng dụng. Một chú ý là các bài toán trình bày trong
chủ đề được giải bằng phương pháp lượng giác hoá có thể được giải bằng phương
pháp đại số thông thường.

n
A. NỘI DUNG PHƯƠNG PHÁP

.v
1. Các phép lượng giác hoá cơ bản

om
 −π π 
Nếu x ≤ R thì đặt x = Rcos α , α ∈ [ 0; π ] ; hoặc x = Rsin α , α ∈  ,  .
 2 2
R  π
Nếu x ≥ R thì đặt x = α ∈ [ 0, c ) ∪ π ,3  .
.c
cos α  2
ok
 x= a + R cos α
Nếu ( x − a ) + ( y − b=
)2 R 2 , ( > 0) thì đặt 
2
, (α = 2π ) .
 y= b + R sin α
bo

 x −α   y − β   x= α + aR cos α
2 2
Nếu   +  =R , a, b > 0 thì đặt 
2
, (α = 2π ) .
 a   b   y= β + bR sin α
et

b  π π
Nếu trong bài toán xuất hiện ( ax ) + b 2 , ( a, b > 0 ) =
2
đặt x tgα , α ∈  − ,  .
vi

a  2 2
Ví dụ 1. Cho bốn số thực x, y, u, v thoả mãn điều kiện x 2 + y 2 = u 2 + v 2 = 1 .
ng

Chứng minh rằng − 2 ≤ u ( x − y ) + v ( x + y ) ≤ 2 .


a

Lời giải
kh

= =
Đặt x cos =
a, y sin =
a; u cos b, v sin b .
Ta có u ( x – y ) + v ( x + y ) = cosb ( cosa − sina ) + sinb ( cosa + sina )
= cos ( a − b ) + sin ( b − a )
 π
= 2 cos  b − a −  ∈  − 2; 2 
 4
Bất đẳng thức được chứng minh.

654
Cty TNHH MTV DVVH Khang Việt

Ví dụ 2. Chứng minh rằng với mọi số thực a,b thuộc đoạn [-1;1] ta có

(
a 1 − b 2 + b 1 − a 2 + 3 ab − (1 − b )(1 − a ) ) ≤ 2 .
2 2

Lời giải
a = cos α
Do a ≤ 1, b ≤ 1 nên ta đặt:  (α , β ∈ [0,π ])
b = cos β

(
Khi đó : a 1 − b 2 + b 1 − a 2 + 3 ab − (1 − b )(1 − a ) )
2 2

= cos α .sin β + cos β .sin α + 3 ( cos α .cos β − sin α .sin β )

n
π
= sin(α + β ) + 3.cos(α + β=
) 2 cos(α + β − )

.v
6

om
⇒ a 1 − b 2 + b 1 − a 2 + 3  ab −

(1 − b )(1 − a )  ≤ 2
2 2

Bất đẳng thức được chứng minh. .c


Bài tập tương tự
1) Chứng minh rằng với mọi số thực x,y thuộc đoạn [ −2; 2] ta có
ok
x 4 − y 2 + y 4 − x2 ≤ 4 .
bo

2) Cho a , b ≥ 1 . Chứng minh rằng : a b − 1 + b a − 1 ≤ ab .


Ví dụ 3. Cho a,b là hai số thực thoả mãn điều kiện ab ≠ 0 .
et

a 2 − (a − 4b) 2
Chứng minh rằng −2 2 − 2 ≤ ≤ 2 2 − 2.
vi

a 2 + 4b 2
ng

Lời giải
 π π
Đặt a 2btgα , α ∈  − ,  khi đó
=
 2 2
a

a 2 − (a − 4b) 2 tg 2α − (tgα − 2) 2
kh

= = 4(tgα − 1).cos 2 α
a + 4b
2 2
1 + tg α
2

= 2sin 2α − 2(1 + cos 2=α ) 2(sin 2α − cos 2α ) − 2


π
= 2 2 sin(2α − ) − 2 ∈  −2 2 − 2, 2 2 − 2 
2
Bất đẳng thức được chứng minh.

655
Khám phá tư duy Kỹ thuật giải bất ĐT Bài toán Max – Min – Đặng Thành Nam

Bài tập tương tự


1 ( x − y )(1 + xy ) 1
Cho x,y là các số thực chứng minh rằng − ≤ ≤ .
2 (1 + x 2 )(1 + y 2 ) 2
Ví dụ 4. Cho a,b,x,y,z là các số thực dương có a + b =
1.
Chứng minh rằng ( x + y ) ( y + z) ≥ 64abxy 2 z ( ax + y + bz ) .
3 3 2

Lời giải
Các đẳng thức và bất đẳng thức trong tam giác
A B C
sin A  sin B  sin C  4 cos cos cos

n
2 2 2
sin 2 A  sin 2 B  sin 2 C  2 1  cos A cos B cos C 

.v
sin 2 A  sin 2 B  sin 2C  4sin A sin B sinC

om
tan A  tan B  tan C  tan A tan B tan C
A B B C C A
1  tan tan  tan tan  tan tan
2 2 2 2 2 2 .c
1  cot A cot B  cot B cot C  cot C cot A
1  cos 2 A  cos 2 B  cos 2 B  2 cos A cos B cos C
ok
Một số phép lượng giác hoá đưa về chứng minh bất đẳng thức trong tam giác
+ Nếu a,b,c là các số thực dương thỏa mãn a + b + c =abc khi đó tồn tại ba góc
bo

một tam giác sao=cho a tan = =


A, b tan B, c tan C .
+ Nếu a,b,c là các số thực dương thỏa mãn ab + bc + ca =
1 khi đó tồn tại ba góc
et

A B C
một tam giác sao= =
cho a tan =
, b tan , c tan .
vi

2 2 2
+ Nếu a,b,c là các số thực dương thỏa mãn a 2 + b 2 + c 2 + 2abc =1 khi đó tồn tại
ng

ba góc nhọn của một tam giác sao=cho a cos= A, b cos = B, c cos C .
Đẳng thức quen thuộc:
a

(a 2
+ 1)( b 2 + 1)( c 2 + 1)= ( ab + bc + ca − 1) + ( a + b + c − abc )
2 2
kh

.
Bài toán 1. Chứng minh rằng với mọi tam giác ABC ta có
3
cos A + cos B + cos C ≤ .
2
Chứng minh.
A+ B A− B A+ B
=VT 2 cos cos − 2 cos 2 +1.
2 2 2
A+ B A− B
Do A,B,C là các góc của tam giác nên 1 > cos > 0;0 ≤ cos ≤ 1.
2 2
656
Cty TNHH MTV DVVH Khang Việt

A+ B A+ B A+ B 1  3
2
3 
Do đó, VT ≤ −2 cos 2
+ 2 cos + 1= − 2  cos −  ≤ .
2 2 2  2 2 2
Dấu bằng xảy ra khi và chỉ khi tam giác đều.
Tổng quát với x,y,z là các số thực dương và A,B,C là các góc của một tam giác ta
cos A cos B cosC x 2 + y 2 + z 2
có + + ≤ .
x y z 2 xyz
Chứng minh.
Bất đẳng thức tương đương với:
2 yz cos A + 2 zx cos B + 2 xy cos C ≤ x 2 + y 2 + z 2

n
⇔ x 2 + y 2 + z 2 + 2 yz cos ( B + C ) − 2 zx cos B − 2 xy cos C ≥ 0

.v
⇔ x 2 + y 2 + z 2 + 2 yz cos B cos C − 2 yz sin B sin C − 2 zx cos B − 2 xy cos C ≥ 0

om
⇔ ( x − z cos B − y cos C ) + ( z sin B − y sinC ) ≥ 0 .
2 2

Bất đẳng thức được chứng minh. .c


sin A sin B sin C
Đẳng thức xảy ra khi và chỉ khi = =
ok
.
x y z
Ví dụ 1. Cho a,b,c là độ dài ba cạnh một tam giác và x,y,z là các số thực thoả mãn
bo

cy + bz =
a

az + cx =
b.
et

bx + ay =
 c
vi

3
Chứng minh rằng x + y + z ≤ .
ng

2
Lời giải
Dễ tính được
a

b2 + c2 − a 2 c2 + a 2 − b2 a 2 + b2 − c2
kh

=x = ;y = ;z .
2bc 2ca 2ab
3
Bài toán đưa về chứng minh cos A + cos B + cos C ≤ .
2
Bất đẳng thức luôn đúng đẳng thức xảy ra khi và chỉ khi
1
x= y= z= ; a= b= c .
2

657
Khám phá tư duy Kỹ thuật giải bất ĐT Bài toán Max – Min – Đặng Thành Nam

Cách 2: Từ giả thiết ta có:



 c b
y+ x=
1
 a a

 a c
z+ x=
1
 b b
 b a
 x+ y=
1
 c c
b c a c a b
Cộng theo vế ba phương trình trên ta được: x( + ) + y ( + ) + z ( + ) =
3.
c b c a b a

n
Sử dụng AM-GM ta có ngay đpcm.

.v
Ví dụ 2. Cho a, b, c là các số thực khác 0 và c > b .

om
bc + a 2 − b a 2 + c 2 + c a 2 + b 2
Tìm giá trị lớn nhất của biểu thức P = .
a 2 + b2 a 2 + c2
Lời giải .c
bc + a 2
−b c
= + +
ok
P
(a 2 + c 2 )(a 2 + b 2 ) a 2 + b2 a2 + c2
bc + a 2 −b(c − b) c (c − b )
bo

= + +
(a 2 + c 2 )(a 2 + b 2 ) (c − b) a 2 + b2 (c − b) a2 + c2
Vì 3 số a,b,c khác O nên trong hệ trục tọa độ Oxy chọn 3 điểm
et

A(0; a ); B (b;0); C (c;0), b < c là ba đỉnh một tam giác.


vi

  


Ta có AB = (b; −a ); AC = (c; −a ); BC = (c − b;0) .
ng

Xét tam giác ABC có


  bc + a 2
= =
a

cos A cos( AB, AC ) ;


(a 2 + b 2 )(a 2 + c 2 )
kh

  −b ( c − b )
= =
cos B cos( BA; BC ) ;
| c − b | a 2 + b2
  c (c − b )
= =
cos C cos(CA, CB) ;
| c − b | a2 + c2
3
⇒=
P cos A + cos B + cos C ≤ .
2

658
Cty TNHH MTV DVVH Khang Việt

Dấu bằng xảy ra khi và chỉ khi tam giác ABC đều.
3
Vậy giá trị lớn nhất của P bằng khi đó c > 0; b =
−c; a =
± 3c .
2
Ví dụ 3. Cho a,b,c là các số thực dương thỏa mãn điều kiện a + b + c =abc .
1 1 1 29
Chứng minh rằng + + ≤ .
2 a +1
2
3 b +1
2
4 c +1
2 48
Lời giải
Theo giả thiết tồn tại 3 góc nhọn của một tam giác thỏa mãn
= a tan= A, b tan= B, c tan C .

n
Khi đó:

.v
1 1 cos A
= =
2 a 2 + 1 2 tan 2 A + 1 2

om
1 1 cos B
= =
3 b 2 + 1 3 tan 2 B + 1 3
.c
1 1 cos C
= =
ok
4 c + 1 4 tan C + 1
2 2 4
cos A cos B cos C 29
Ta cần chứng minh: + + ≤
bo

2 3 4 48
Bất đẳng thức này luôn đúng bởi vì
cos A cos B cos C 22 + 32 + 42 29
et

+ + ≤ =
2 3 4 2.2.3.4 48
vi

sin A sin B sin C


Đẳng thức xảy ra khi và chỉ khi: = =
2 3 4
ng

tan A tan B tanC


⇔ = =
2 tan 2 A + 1 3 tan 2 B + 1 4 tan 2 C + 1
a

a b c
kh

⇔ = =
2 a + 1 3 b + 1 4 c2 + 1
2 2

1 1 1
Ví dụ 4. Cho a,b,c là các số thực dương thỏa mãn điều kiện + + =6
3a 2b c
3 a b c
Chứng minh rằng ≥ + + .
2 a + 4bc b + 9ca c + 36ab
Lời giải
Phân tích biểu thức vế phải trước

659
Khám phá tư duy Kỹ thuật giải bất ĐT Bài toán Max – Min – Đặng Thành Nam

1 1 1
VP = + +
4bc 9ca 36ab
1+ 1+ 1+
a b c
1 1 1
= + +
2 2 2
 bc   ca   ab 
1+  2  1+ 3  1+ 6 
 a   b   c 

bc ca ab
Đặt =
x 2 ,=
y 3 ,=
z 6 ⇒ xy= 6c; yz= 18a; zx
= 12b .
a b c

n
Theo giả thiết ta có:

.v
1 1 1 1 1 1
+ + =6 ⇔ + + =1
3a 2b c 18a 12b 6c

om
1 1 1
⇔ + + =1 ⇔ x + y + z = xyz
yz zx xy
Cho ta thấy dấu hiệu của lượng giác hoá vậy tồn tại ba góc một tam giác sao cho
.c
=x tan= =
A, y tan B, z tan C .
ok
Ta cần chứng minh
1 1 1 3
+ + ≤
bo

1 + tan 2 A 1 + tan 2 B 1 + tan 2 C 2

3
⇔ cos A + cos B + cos C ≤
et

2
Bất đẳng thức cuối đúng và ta có điều phải chứng minh.
vi

 bc
2 = 3  3
ng

 a bc = 4 a
 a = 1 / 6
 ca  1 
Đẳng thức xảy ra khi và chỉ khi 3 = 3 ⇔ ca = b ⇔ b = 1 / 4 .
a

 b  3 c = 1 / 2
  
kh

ab 1
6 = 3 ab = 12 c
 c 

Bài tập tương tự


1 1 1
Cho a,b,c là các số thực dương thoả mãn điều kiện + + = 6.
a 2b 3c
a b c 1
Chứng minh rằng . . ≤ .
a + 36bc b + 9ca c + 4ab 27

660
Cty TNHH MTV DVVH Khang Việt

Bài toán 2. Chứng minh rằng với mọi tam giác ABC ta có
3 3
cos A + cos B + sin C ≤ .
2
Chứng minh.
A+ B A− B
cos A +=
cos B + sin C 2cos cos + sin C
2 2
A+ B C C
≤ 2cos + sin=
C 2sin 1 + cos 
2 2 2
2 3
 C  C  C  C
= 2 1 − cos 2 1 + cos  = 2 1 − cos 1 + cos 

n
 2  2  2  2

.v
Chú ý sử dụng bất đẳng thức AM – GM ta có
4

om
  C  C 
3  3  1 − cos  + 3  1 + cos   4
 C  C 2  3
3 1 − cos 1 + cos  ≤  
2 
=
 
 2  2  4  2
 .c 
 
3
ok
 C  C 3 3
⇒ 2 1 − cos 1 + cos  ≤
 2  2 2
bo

Bất đẳng thức được chứng minh.


π 2π
et

Đẳng thức xảy ra khi và chỉ khi A= B= , C= .


6 3
Ví dụ 1. Cho a,b,c là các số thực dương có tổng bằng 1. Chứng minh rằng
vi

a b abc 3 3
ng

+ + ≤1+ .
a + bc b + ca c + ab 4
Lời giải
a

Gọi P là biểu thức vế trái của bất đẳng thức.


kh

ab
1 1 c .
Ta có P = + +
bc ca ab
1+ 1+ 1+
a b c
Chú ý theo giả thiết ta có
ab ca ab bc bc ca
a=
+b+c . + . + = . 1.
c b c a a b

661
Khám phá tư duy Kỹ thuật giải bất ĐT Bài toán Max – Min – Đặng Thành Nam

Suy ra tồn tại 3 góc một tam giác sao cho


bc A ca B ab C
= tan
= , =
tan , tan .
a 2 b 2 c 2
C
tan
1 1 2
Khi đó P = + +
A 2 B C
1 + tan 2
1 + tan 1 + tan 2
2 2 2
A B 1
= cos 2 + cos 2 + sin C
2 2 2
1 3 2
= 1 + ( cos A + cos B + sin C ) ≤ 1 +

n
2 4

.v
Bất đẳng thức được chứng minh. Đẳng thức xảy ra khi và chỉ khi

om
a=b=2 3 − 3, c =7−4 3.
Bằng cách chứng minh tương tự các bất đẳng thức trên ta chứng minh được
Chứng minh rằng với mọi tam giác ABC ta có .c
3 3
sin A + sin B + sin C ≤ ;
ok
2
1 A B C
cos A.cos B.cos C ≤ ≤ sin .sin .sin
8 2 2 2
bo

3
cos 2 A + cos 2 B + cos 2 C ≥ ;
4
et

3
sin A + sin B − cos C ≤ ;
2
vi

5
3 cos A − sin B + 3 sin C ≤ ;
ng

2
3
sin 2 A + sin 2 B + cos 2C ≤ ;
2
a

5
cos 2 A + 3 ( cos 2 B + cos 2C ) ≥ −
kh

2
Bài toán 3. Chứng minh rằng với mọi tam giác ABC ta có
cos 2 A + cos 2 B + cos 2 C + 2cos A.cos B.cos C =
1.
Chứng minh.
Đẳng thức đã cho tương đương với:

662
Cty TNHH MTV DVVH Khang Việt

cos 2 B + cos 2 C + 2cos A.cos B.cos C =


sin 2 A
⇔ ( cos B + cos C ) − 2cos B.cos C + 2cos A.cos B.cos C =
2
sin 2 A
B+C B −C A A A
⇔ 4cos 2 cos 2 − 4cos B cos C sin 2 =4sin 2 cos 2
2 2 2 2 2
B − C A
⇔ cos 2 − cos B cos C = cos 2
2 2
1 + cos( B − C ) 1 + cos A
⇔ − cos B.cos C =
2 2
⇔ cos( B − C ) − 2cos B.cos C = cos A

n
⇔ cos( B − C ) + cos( B + C ) − 2cos B.cos C = 0

.v
Đẳng thức cuối đúng và ta có đpcm.
Như vậy với các số thực dương x,y,z thoả mãn điều kiện

om
x 2 + y 2 + z 2 + 2 xyz =
1.
Khi đó tồn tại ba góc nhọn trong một tam giác sao cho
= x cos= A; y cos= B; z cos C .
.c
Thật vậy ta luôn có
ok
cos 2 A + cos 2 B + cos 2 C + 2cos A.cos B.cos C =1
A+ B +C A+ B −C B+C − A C + A− B
bo

⇔ cos .cos .cos .cos =


0
2 2 2 2
 π
et

Với A, B, C ∈  0;  thì A + B + C = π.
 2
vi

Ví dụ 1. Cho a,b,c là các số thực dương thoả mãn điều kiện


1 1 1
ng

+ + =
2.
a +1 b +1 c +1
2 2 2

3
Chứng minh rằng ab + bc + ca ≤ .
a

2
kh

Lời giải
Giả thiết bài toán tương đương với:
1 ⇒ ab, bc, ca ∈ ( 0;1) .
a 2 b 2 + b 2 c 2 + c 2 a 2 + 2a 2 b 2 c 2 =
Do đó tồn tại ba góc nhọn trong một tam giác thoả mãn:
= ab cos= =
A, bc cos B, ca cos C .
3
Bất đẳng thức trở thành: cos A + cos B + cos C ≤ .
2
Bất đẳng thức cuối luôn đúng và ta có đpcm.

663
Khám phá tư duy Kỹ thuật giải bất ĐT Bài toán Max – Min – Đặng Thành Nam

1
Đẳng thức xảy ra khi và chỉ khi a= b= c= .
2
Ví dụ 2. Cho a,b,c là các số thực dương thoả mãn điều kiện a + b + c + 1 =4abc .
4bc − 1 4ca − 1 1
Tìm giá trị lớn nhất của biểu thức P = + + .
bc ca ab
Lời giải
1 1 1 1
Giả thiết bài toán tương đương với + + + 2. = 1.
4bc 4ca 4ab 8abc
Do đó tồn tại ba góc nhọn trong một tam giác thoả mãn:

n
1 1 1
= 2cos =
A; =
2cos B; 2cos C .

.v
bc ca ab
Khi đó

om
1 1
=P 4cos 2 A 2
− 1 + 4cos 2 B 2
− 1 + 4cos 2 C
cos A cos B
= 2sin 2 A + 2sin 2 B − 4sin C + 4
2 .c
= 4sin( A + B) cos( A − B) − 4sin 2 C + 4
ok
≤ 4sin C − 4sin 2 C + 4 =− ( 2sin C − 1) + 5 ≤ 5
2
bo

 π 
C = 6 a= b=
3
Đẳng thức xảy ra khi và chỉ khi  ⇔ 3 .
 A= B= 5π =
et

c 2 3 + 3
 12 
vi

3
Vậy giá trị lớn nhất của P bằng 5 đạt tại a= b= , c= 2 3 + 3 .
ng

3
Ví dụ 3. Cho a,b,c là các số thực dương thoả mãn điều kiện abc = a + b + c + 2 .
( )
ab + bc + ca ≤ 4 + abc .
a

Chứng minh rằng 2


kh

Lời giải
Giả thiết bài toán tương đương với:
1 1 1 1
+ + + 2. =1.
bc ca ab abc
Do đó tồn tại ba góc nhọn một tam giác sao cho
1 1 1
= cos
= A, =
cos B, cos C .
bc ca ab
 1 1 1  1
Ta cần chứng minh 2  + + ≤4+
 cos A cos B cos C  cos A.cos B.cos C
664
Cty TNHH MTV DVVH Khang Việt

⇔ 2 ( cos A.cos B + cos B.cos C + cos C.cos A ) ≤ 4cos A.cos B.cos C + 1 .


Để chứng minh bất đẳng thức cuối ta chứng minh như sau
1
=
2cos A.cos B sin 2 A.cot A.sin 2 B.cot B ≤ ( sin 2 A.cot B + sin 2 B.cot A )
2
1
2cos B.cos C ≤ ( sin 2 B.cot C + sin 2C.cot B )
2
1
2cos C.cos A ≤ ( sin 2C.cot A + sin 2 A.cot C )
2
Cộng lại thế vế và khai triển vế phải ta có ngay điều phải chứng minh.
Đẳng thức xảy ra khi và chỉ khi A = B = C ⇔ a = b = c = 2 .

n
Bài tập tương tự

.v
Cho a,b,c là các số thực dương thoả mãn điều kiện ab + bc + ca + abc = 4.

om
Tìm giá trị nhỏ nhất của biểu thức P = ab + bc + ca + ab + bc + ca .
Ví dụ 4. Cho a,b,c,x,y,z là các số thực dương thoả mãn điều kiện
ax 2 + by 2 + cz 2 + xyz =
.c
4abc .
Chứng minh rằng a + b + c ≥ x + y + z .
ok
Lời giải
x2 y2 z2 xyz
bo

Giả thiết đã cho tương đương với: + + + 2. =


1.
4bc 4ca 4ab 8abc
Do đó tồn tại ba góc một tam giác sao cho:
et

x y z
= cos=A; =
cos B; cos C .
vi

2 bc 2 ca 2 ab
Ta cần chứng minh a + b + c ≥ 2 bc cos A + 2 ca cos B + 2 ab cos C .
ng

Bất đẳng thức cuối luôn đúng và ta có đpcm.


a

B. BÀI TẬP CHỌN LỌC


kh

Bài 1. Cho x,y,z là các số thực không âm thỏa mãn điều kiện xz + yz + 1 =xy .
2x 2y z2 −1
Tìm giá trị lớn nhất của biểu thức P = + + .
x2 + 1 y2 + 1 z2 + 1
Lời giải
2 xy ( x + y ) + 2 x + 2 y 4x + 4 y
TH1: Nếu xy = 1 ⇒ z = 0 ⇒ P = −1 = − 1 ≤ 1.
x y + x + y +1
2 2 2 2
( x + y )2
xy − 1
TH2: Nếu xy > 1 ⇒ z = .
x+ y
665
Khám phá tư duy Kỹ thuật giải bất ĐT Bài toán Max – Min – Đặng Thành Nam

xy − 1  π π
Đặt x =
tan a, y =
tan b, z = = − cot ( a + b ) với a, b ∈  0;  , a + b > .
x+ y  2 2

Khi đó : P =sin 2a + sin 2b + cos(2a + 2 b) =1 − 2sin 2 (a + b) + 2sin(a + b)cos(a − b)


2
 1 3 3
≤ 1 − 2sin 2 (a + b) + 2sin(a + b) =−2 sin(a + b) −  + ≤ .
 2 2 2

a = b

 1 5π
sin(a + b) = ⇔ a =b =

n
2 12
Đẳng thức xảy ra khi và chỉ khi 
  π π

.v
a, b ∈  0;  , a + b >
  2  2

om
5π 5π
⇒ x =y =tan = 2 + 3, z = − cot =3
12 6
3
Vậy giá trị lớn nhất của P bằng đạt tại x = y= 2 + 3, z =
.c 3
2
Bài 2. Cho a,b,c là các số thực dương thỏa mãn điều kiện ab + bc + ca =
ok
1
a b 3c
Tìm giá trị lớn nhất của biểu thức P = + +
1+ a 2
1+ b 2
bo

1 + c2
Lời giải
et

Theo giả thiết tồn tại ba góc của một tam giác thỏa mãn
A B C
= a tan = , b tan= , c tan .
vi

2 2 2
Khi đó:
ng

A
tan
a 2 A A 1 b 1 c C
= = sin= cos =
sinA và =sin B; sin .
a

1+ a 2
1 + tan 2
A 2 2 2 1+ b 2 2 1 + c2 2
kh

2
Từ đó suy ra
1 C A+ B A− B C
P= ( sin A + sin B ) + 3sin = sin cos + 3sin
2 2 2 2 2
A− B
(1 + 9 )  cos 2 2C
C C C C C
= cos cos + 3sin ≤ cos + 3sin ≤ + sin
=  10
2 2 2 2 2  2 2

666
Cty TNHH MTV DVVH Khang Việt

 A− B
cos 2 = 1

Đẳng thức xảy ra khi và chỉ khi  C
 C
sin
2
cos =
 2 3
⇔ a = b = 10 − 3, c = 3 .
Vậy giá trị lớn nhất của P bằng 10 đạt tại a =b = 10 − 3, c =3.
Cách 2: Ta xử lý bằng đại số như sau
Sử dụng giả thiết ab + bc + ca =
1 ta có:

n
a b a b
=
+ +

.v
1+ a 1+ b
2 2
ab + bc + ca + a 2
ab + bc + ca + b 2
a (b + c ) + b (c + a )

om
a b
= + =
( a + b )( a + c ) ( b + a )( b + c ) ( a + b )( b + c )( c + a )
2ab + c ( a + b ) ab ( a + b ) + c ( a + b )
.c
= ≤
( a + b )( b + c )( c + a ) ( a + b )( b + c )( c + a )
ok
c + ab 1 1
= ≤ =
( c + a )( c + b ) ( c + a )( c + b ) 1 + c2
bo

1 3c  1 c2 
Do đó P ≤ + ≤ (1 + 9 )  + =10 .
1 + c2 1 + c2 1+ c
2
1 + c 2 
et

Đẳng thức xảy ra khi và chỉ khi a =b = 10 − 3, c =3.


vi

Bài 3. Cho x,y,z là các số thực dương thỏa mãn điều kiện xyz + x + y =z và z ≥ 1 .
ng

Tìm giá trị lớn nhất của biểu thức


 1 1  z
a

P = 4  + 2 
 + .
1+ x 1 + y  1 + z2
2
kh

Lời giải
y x
Theo giả thiết ta có: xy + + =1 nên tồn tại 3 góc của một tam giác sao cho:
z z
A B 1 C
= =
x tan =
, y tan , tan .
2 2 z 2
Khi đó :

667
Khám phá tư duy Kỹ thuật giải bất ĐT Bài toán Max – Min – Đặng Thành Nam

  C
  tan
P = 4
1
+
1
+ 2 = 4  cos 2 A + cos 2 B  + 1 sinC
  
 1 + tan 2 A 1 + tan 2 B  1 + tan 2 C  2 2 2
 2 2 2
1 A+ B A− B 1
= 4 + 2 ( cos A + cos B ) + sinC = 4 + 4cos cos + sin C
2 2 2 2
C A− B 1 C 1
= 4 + 4sin cos + sin C ≤ 4 + 4sin + sin C
2 2 2 2 2
C C 9+4 2
Mặt khác z ≥ 1 ⇒ tan ≤ 1 ⇔ ≤ 450 ⇔ C ≤ 900 ⇒ P ≤ .
2 2 2

n
Đẳng thức xảy ra khi và chỉ khi z= 1, x= y= 2 −1.

.v
Bài 4. Cho x, y, z là các số thực dương thay đổi thỏa mãn điều kiện xyz + x + z =y.

om
Tìm giá trị lớn nhất của biểu thức
2 2 4z 3z
P= − − + .
x +1
2
y +1
2
z +1
2 .c (z 2
)
+1 z2 + 1
Lời giải
ok
 π
Đặt x = tan A, y = tan B, z = tan C ,  0 < A, B, C <  .
 2
bo

Theo giả thiết ta có:


y−z tan B − tan C
x= ⇔ tan A = = tan ( B − C ) ⇔ A = B − C + kπ .
et

1 + yz 1 + tan B tan C
π
vi

Do − < A − B + C < π ⇒ k =⇒
0 A=B −C ⇔ A− B =−C .
2
ng

Khi đó:
 1 1  4 tan C 3tan C
P = 2 − 2 
− +
( )
a

 1 + tan A 1 + tan B 
2
1 + tan 2 C 1 + tan 2 C 1 + tan 2 C
kh

( )
= 2 cos 2 A − cos 2 B − 4sin C + 3sin C cos 2 C

= cos 2 A − cos 2 B − 4sin C + 3sinCcos 2 C


−2sin ( A + B ) sin ( A − B ) − 4sin C + 3sinCcos 2 C
=
= 2sin C sin ( A + B ) − 4sin C + 3sinCcos 2 C
≤ 2sin C − 4sin C + 3sinCcos 2= (
C sin C 3cos 2 C −=
2 sin C 1 − 3sin 2 C ) ( )
3 3
Nếu sin C > ⇒ P < 0 xét với sin C ≤ sử dụng bất đẳng thức AM – GM ta có:
3 3
668
Cty TNHH MTV DVVH Khang Việt

(
6sin 2 C 1 − 3sin 2 C 1 − 3sin 2 C )( )
( )
2
P = sin C 1 − 3sin C
2 2
=
6
3
 6sin 2 C + 1 − 3sin 2 C + 1 − 3sin 2 C 
 
 3  2
≤ =
6 9
Đẳng thức xảy ra khi và chỉ khi
1 2 2 2 2
=
sin C ⇔=
tan C =
, tan A =
, tan B =
2⇔x =
2, y = ,z .
3 4 2 2 4

n
2 2 2
đạt tại
= = =

.v
Vậy giá trị lớn nhất của P bằng x 2, y ,z .
9 2 4
Cách 2: Theo giả thiết ta có:

om
y−z 2 2 4z 3z
=x ⇒=P − 2 − +
1 + yz  y−z 

2

 +1
y +1 z2 + 1 z2 + 1 z2 + 1
.c ( )
 1 + yz 
2 (1 + yz )
2
ok
2 4z 3z
=P − − +
(y 2
)(
+1 z +1 2
) y +1
2
z +12
(z 2
+1 ) z2 + 1
bo

=
2 z 2 y + y 2 − 1 z 
  − 4z +( 3z )
2
(
y +1 z +1 2
z +1
2
)(
z + 1 z2 + 1
2
) ( )
et

Sử dụng bất đẳng thức C –S ta có


vi

 2
( ) ( )
2
( )
2 z 2 y + y 2 − 1 z  2 z 4 y + y − 1  1 + z
2 2
ng

 ≤   2z
=
( y +1 z +1
2 2
)( )
y +1 z +1
2 2
1 + z2 ( )( )
a

3
 z 
kh

2z 4z 3z z
Suy ra P ≤ − + =
−3   +
1 + z2 z2 + 1 (z 2
+1 ) z2 + 1
 2
 z +1  z2 + 1

, ( t ∈ ( 0;1) ) khi đó P ≤ f (t ) =
z
Đặt t
= −3t 3 + t .
z +1
2

t∈( 0;1) 1
Ta có f '(t )= −9t 2 + 1; f '(t )= 0 ←
→ t= .
3

669
Khám phá tư duy Kỹ thuật giải bất ĐT Bài toán Max – Min – Đặng Thành Nam

1 1
Ta có f’(t) đổi dấu dương sang âm khi đi qua t = nên f(t) đạt cực đại tại t =
3 3
1 2
trên khoảng ( 0;1) hay f (t ) ≤ f   = .
3 9
2 2 2
Vậy giá trị lớn nhất của P bằng đạt tại
= x = ,y = 2, z .
9 2 4
Bài 5. Cho x,y,z là các số thực dương thỏa mãn điều kiện
4 x 2 + 4 y 2 + 4 z 2 + 16 xyz =
1.
x + y + z + 4 xyz

n
Tìm giá trị nhỏ nhất của biểu thức P = .
1 + 4 ( xy + yz + zx )

.v
Lời giải

om
Theo giả thiết tồn tại 3 góc nhọn của một tam giác sao cho:
= 2 x cos= A, 2 y cos= B, 2 z cos C .
3
 A B B C C A
.c
 tan tan + tan tan + tan tan 
A B C 2 2 2 2 2 2 =1
Khi đó tan 2 tan 2 tan 2 ≤   .
ok
2 2 2  3  27
 
bo

A B C A B C
⇔ cos 2 cos 2 cos 2 ≥ 27sin 2 sin 2 sin 2
2 2 2 2 2 2
⇔ (1 + cos A )(1 + cos B )(1 + cos C ) ≥ 27 (1 − cos A )(1 − cos B )(1 − cos C )
et

⇔ (1 + 2 x )(1 + 2 y )(1 + 2 z ) ≥ 27 (1 − 2 x )(1 − 2 y )(1 − 2 z )


vi

⇔ 28 ( x + y + z + 4 xyz ) ≥ 13 (1 + 4 xy + 4 yz + 4 zx )
ng

x + y + z + 4 xyz 13
=⇔P ≥
1 + 4 xy + 4 yx + 4 zx 28
a

1
Bất đẳng thức được chứng minh. Đẳng thức xảy ra khi và chỉ khi x= y= z= .
kh

4
Bài 6. Cho a,b,c là các số thực dương có tổng bằng 1. Chứng minh
 bc abc  abc 2 3 − 5
3  −  + ≥ .
 a + bc c + ab  b + ca 4
Lời giải
ab ca ab bc bc ca
Chú ý theo giả thiết ta có a =
+b+c . + . + = . 1.
c b c a a b
Suy ra tồn tại 3 góc một tam giác sao cho

670
Cty TNHH MTV DVVH Khang Việt

bc A ca B ab C
= tan = , =
tan , tan .
a 2 b 2 c 2
Khi đó
 2 A C  B
 tan 2 tan
2  tan
2
=
P 3 − +
 1 + tan A C  1 + tan 2 B
2
1 + tan 2
 2 2 2
 A 1  1
= 3  sin 2 − sin C  + sin B
 2 2  2

( )

n
1 3 5 3
=− 3 cos A − sin B + 3 sin C + ≥− +

.v
2 2 4 2
5
Bất đẳng thức cuối đúng do 3 cos A − sin B + 3 sin C ≤ .

om
2
 π
A= B=
 6 a= b= 2 3 − 3
Đẳng thức xảy ra khi và chỉ khi  ⇔ ..c
C = 2π c= 7 − 4 3

ok
3
Bài 7. Cho a,b,c là các số thực dương thỏa mãn điều kiện a 2 + b 2 + c 2 + 2abc =
1.

( )
bo

Chứng minh rằng a 2 + b 2 + c 2 ≥ 4 a 2b 2 + b 2 c 2 + c 2 a 2 .

Lời giải
et

Theo giả thiết tồn tại ba góc nhọn của một tam giác thỏa mãn:
= a cos= A, b cos= B, c cos C .
vi

Ta cần chứng minh:


ng

(
cos 2 A + cos 2 B + cos 2 C ≥ 4 cos 2 A cos 2 B + cos 2 B cos 2 C + cos 2 C cos 2 A )
cot 2 A cot 2 A cot 2 B
a

⇔∑ ≥ 4∑
cyc 1 + cot
2
(1 + cot A)(1 + cot B )
2 2
kh

A cyc

cot 2 A cot 2 A cot 2 B


⇔∑ ≥ 4∑
cyc ( cotA + cotB )( cot A + cot C ) cyc ( cot A + cotB ) ( cot C + cot A )( cotC + cot B )
2

cot 2 A cot 2 B
⇔ ∑ ( cot B + cot C ) cot 2 A ≥ 4∑ (luôn đúng).
cyc cyc cot A + cot B

cot 2 A cot 2 B
Do 4∑ ≤ ∑ cot A cot B ( cot A + cot B=
) ∑ ( cot B + cot C ) cot 2 A .
cyc cot A + cot B cyc cyc

671
Khám phá tư duy Kỹ thuật giải bất ĐT Bài toán Max – Min – Đặng Thành Nam

1
Đẳng thức xảy ra khi và chỉ khi a= b= c= .
2
Cách 2: Ta đưa về chứng minh

(a 2
)( ) (
+ b 2 + c 2 + 2abc a 2 + b 2 + c 2 ≥ 4 a 2b 2 + b 2 c 2 + c 2 a 2 )
(
⇔ a 4 + b 4 + c 4 + 2abc a 2 + b 2 + c ) ≥ 2(a b
2 2 2
+ b2c2 + c2 a 2 )
Ta có:
1
8
3
( )
abc ≤ , a + b + c ≤ ⇒ 2 a 2 + b 2 + c 2 − a − b − c = 2 − 4abc − a − b − c ≥ 0 .
2

n
(
Do đó 2abc a 2 + b 2 + c 2 ) ≥ abc ( a + b + c ) .

.v
Ta đưa về chứng minh bất đẳng thức:

om
(
a 4 + b 4 + c 4 + abc ( a + b + c ) ≥ 2 a 2b 2 + b 2 c 2 + c 2 a 2 )
⇔ a 2 ( a − b )( a − c ) + b 2 ( b − c )( b − a ) + c 2 ( c − a )( c − b ) ≥ 0
.c
Không mất tính tổng quát giả sử a ≥ b ≥ c khi đó c 2 ( c − a )( c − b ) ≥ 0 và
ok
a 2 ( a − b )( a − c ) + b 2 ( b − c )( b − a ) = ( a − b )  a 2 ( a − c ) − b 2 ( b − c ) 
 
bo

≥ ( a − b ) a ( b − c ) − b ( b − c )
2 2
 
(
=( a − b )( b − c ) a 2 − b 2 ≥ 0 )
et

Bất đẳng thức được chứng minh. Đẳng thức xảy ra khi và chỉ khi a= b= c
vi

Bài tập tương tự


ng

Cho a,b,c là các số thực không âm thỏa mãn a 2 + b 2 + c 2 + abc =


4.

( )(
Chứng minh rằng a 2 + b 2 − abc b 2 + c 2 − abc c 2 + a 2 − abc ≥ a 2b 2 c 2 .)( )
a
kh

Bài 8. Cho x,y,z là các số thực dương thoả mãn điều kiện 6 x + 3 y + 2 z =
xyz .

x yz
Tìm giá trị lớn nhất của biểu thức P = .
x 2
+ 14 (y 2
+4 z +9 )( 2
)
Lời giải
2 6 3
Giả thiết bài toán tương đương với + + =1.
xy yz zx

672
Cty TNHH MTV DVVH Khang Việt

1 2 3
Đặt a = , b = , c = ⇒ ab + bc + ca =1. Khi đó tồn tại ba góc một tam giác
x y z
A B C
sao= =
cho a tan =
, b tan , c tan .
2 2 2
1 1 1 A B C 4 3
Khi đó P
= .= . cos cos cos ≤ .
1+ a 1+ b
2 2
1+ c 2 2 2 2 9

Đẳng thức xảy ra khi và chỉ khi x= y= 2 2, z= 3 2 .

n
C. BÀI TẬP RÈN LUYỆN

( a − b )(1 − ab )

.v
1
Bài 1. Cho a,b là hai số thực bất kỳ chứng minh rằng ≤ .
(1 + a )(1 + b )
2 2 2

om
Bài 2. Cho x,y là hai số thực thoả mãn điều kiện 36 x 2 + 16 y 2 =
5.
Tìm giá trị lớn nhất và giá trị nhỏ nhất của biểu thức: Z =y − 2 x + 5 .
.c
5 5
ok
Bài 2. Đặt x = sin a thì từ giả thiết ta có y = cos a .
6 4
Bài 3. Chứng minh rằng với mọi số thực a,b,c ta có
bo

a −b b−c c−a
+ ≥ .
a 2 + 1. b 2 + 1 b 2 + 1. c 2 + 1 c 2 + 1. a 2 + 1
et

Bài 4. Cho a, b, c là các số thực dương thoả mãn điều kiện abc + c + 2b =
vi

2a .

b2 c2
ng

1 3
Chứng minh rằng + + ≤ .
1+ a 1+ b
2
4+c 2 2 2
Bài 5. Cho a, b, c là các số thực dương thỏa mãn điều kiện a.b + bc + ca =
1.
a
kh

a b 3 3 c
Chứng minh rằng + +
. ≥
1+ a 1+ b
2
1+ c 2
2 2

Bài 6. Cho a,b,c là các số thực thuộc khoảng (0;1). Chứng minh rằng
abc + (1 − a )(1 − b )(1 − c ) < 1 .
Bài 7. Cho a,b,c là các số thực dương thoả mãn điều kiện a + b + c =
1.
a b c 9
Chứng minh rằng + + ≤ .
a + bc b + ca c + ab 4
Bài 8. Cho a,b,c là các số thực dương thỏa mãn điều kiện a + b + c =abc .

673
Khám phá tư duy Kỹ thuật giải bất ĐT Bài toán Max – Min – Đặng Thành Nam

2 1 1
Tìm giá trị lớn nhất của biểu thức P = + + .
a2 + 1 b2 + 1 c2 + 1
Bài 9. Cho x,y,z là các số thực dương thỏa mãn điều kiện xyz + x − y + z =0.
2 2 3
Tìm giá trị lớn nhất của biểu thức P = − + .
x +1
2
y +12
z +1
2

Bài 10. Cho a,b,c là các số thực dương thỏa mãn điều kiện

ab + bc + ca
= ( a + 1)(b + 1)( c + 1) + 1 .
2 2 2

Chứng minh rằng a + b + c ≥ 3 .

n
Bài 11. Cho a,b,c là các số thực dương thoả mãn điều kiện a + b + c =abc .

.v
a b c
Tìm giá trị nhỏ nhất của biểu thức P = + + .

om
a + bc b + ca c + ab
Bài 12. Cho a,b,c là các số thực dương thoả mãn điều kiện a + b + c =abc .
a  b c 
Tìm giá trị nhỏ nhất của biểu thức P = + 3  + .c
.
a + bc  b + ca c + ab 
ok
1 1 1
Bài 13. Cho a,b,c là các số thực dương thoả mãn điều kiện + + = 6.
a 2b 3c
bo

a b c 1
Chứng minh rằng . . ≤ .
a + 36bc b + 9ca c + 4ab 27
Bài 14. Cho a,b,c là các số thực dương thoả mãn điều kiện
et

2009ac + ab + bc =
2009 .
vi

2 2b 2 3
Tìm giá trị lớn nhất của biểu thức P = − + 2 .
a + 1 b + 2009 c +1
ng

2 2 2

Bài 15. Cho a,b,c là các số thực không âm thỏa mãn a 2 + b 2 + c 2 + abc =
4.

( )( )( )
a

Chứng minh rằng a 2 + b 2 − abc b 2 + c 2 − abc c 2 + a 2 − abc ≥ a 2b 2 c 2 .


kh

Bài 16. Cho a,b,c là các số thực dương thoả mãn điều kiện ab + bc + ca + abc =
4.
Chứng minh rằng ab + bc + ca ≤ 3 .
Bài 17. Chox,y,z là các số thực dương thoả mãn điều kiện xy + yz + zx =
xyz .
2 1 1 9
Chứng minh rằng + + ≤ .
1 + x2 1 + y2 1 + z2 4

Bài 18. Cho a,b,c là các số thực dương thoả mãn điều kiện a + b + c =6.
Tìm giá trị lớn nhất của biểu thức

674
Cty TNHH MTV DVVH Khang Việt

1 1 1
P=(3 − a )(3 − b)(3 − c)( 2 2
+ 2 2
+ 2 2
).
b c c a a b
Bài 19. Cho a,b,c là các số thực dương thoả mãn điều kiện a 2 + b 2 + c 2 + abc =
4.
Chứng minh rằng a 4 − a 2 + b 4 − b 2 + c 4 − c 2 ≤ 3 3 .
Bài 20. Cho a,b,c là các số thực thuộc khoảng (0;2) thoả mãn điều kiện
ab + bc + ca + abc =4.
Chứng minh rằng 4 − a 2 + 4 − b2 + 4 − c2 ≤ 3 3 .
1+ z ( x + y) .
Bài 21. Cho x,y,z là các số thực dương thoả mãn điều kiện xy =

n
2 xy ( xy + 1) z
= +

.v
Tìm giá trị lớn nhất của biểu thức P .
(1 + x )(1 + y )
2 2
z +12

om
D. HƯỚNG DẪN GIẢI – ĐÁP SỐ
Bài 1. Đặt
= =
a tanx; b tany ta có

VT=
( tan x − tan y )(1 − tan x.tan y )=
.c
sin( x − y ) cos( x − y )=
1 1
sin 2( x − y ) ≤ .
(1 + tan x )(1 + tan y )
2 2
ok
2 2

Bất đẳng thức được chứng minh.


bo

5 5 5 5
Nên Z = sin a − cos a + 5 ⇔ sin a − cos a = Z − 5 (*)
6 2 6 2
et

5 5 49 23 37
Phương trình (*) có nghiệm khi và chỉ khi ( Z − 5 ) ≤ + =
2
⇔ ≤Z ≤ .
36 4 36 6 6
vi

Đặt a tanx
Bài 3. = = =
; b tany ; c tan z .
ng

Bất đẳng thức đã cho tương đương với:


tan x − tan y tan y − tan z tan x − tan z
+ ≥
a

1 + tan 2 x . 1 + tan 2 y 1 + tan 2 y . 1 + tan 2 z 1 + tan 2 x . 1 + tan 2 z


kh

⇔ sin( x − y ) + sin( y − z ) ≥ sin( x − z )


Bất đẳng cuối đúng bởi vì
sin( x − y ) + sin( y − z ) ≥ sin( x − y ) + sin( y − z ) ≥ sin( x − z ) .
Bất đẳng thức được chứng minh.
Bài tập tương tự
Chứng minh rằng với mọi số thực a,b,c ta có
a −b b−c c−a
+ ≥ .
a 2 + 2015. b 2 + 2015 b 2 + 2015. c 2 + 2015 c 2 + 2015. a 2 + 2015

675
Khám phá tư duy Kỹ thuật giải bất ĐT Bài toán Max – Min – Đặng Thành Nam

Bài 4. Theo giả thiết tồn tại ba góc một tam giác sao cho:
1 2
a = tanA ; = tan B ; = tan C .
b c
3
Bất đẳng thức đã cho tương đương với: cos A + cos B + cos C ≤ .
2
Bất đẳng thức cuối luôn đúng và ta có đpcm.
A B C
Đặt a tan
Bài 5.= = ; b tan . Từ giả thiết suy ra c = tan ; với A, B, C là 3 góc
2 2 2
của tam giác nhọn ABC
Bất đẳng thức tương đương với:

n
.v
1 3 3
(tan A + tan B + tan C ) ≥ (luôn đúng).
2 2

om
Đặt a sin
Bài 6.= = 2
x; b =
sin 2
y; c sin 2 z .
Ta cần chứng minh sin x.sin y.sin z + cos x.cos y.cos z < 1 .
Bất đẳng thức đúng bởi vì
.c
sin x.sin y.sin z + cos x.cos y.cos z ≤ sin x.sin y + cos x.cos =
y cos( x − y ) ≤ 1 .
ok
Dấu bằng không xảy ra và ta có đpcm.
1 1 1 9
+ + ≤
bo

Bài 7. Ta cần chứng minh


bc ca ab 4
1+ 1+ 1+
a b c
et

ab bc bc ca ca ab
Theo giả thiết ta có . + . + . =
1.
c a a b b c
vi

Do đó tồn tại ba góc một tam giác sao cho


ng

bc A ca B ab C
= tan
= ; =
tan ; tan .
a 2 b 2 c 2
a

1 1 1 9
Bất đẳng thức trở thành: + + ≤ .
kh

A B C 4
1 + tan 2 1 + tan 2 1 + tan 2
2 2 2
A B C 9
⇔ cos 2 + cos 2 + cos 2 ≤
2 2 2 4
3 + cos A + cos B + cos C 9
⇔ ≤
2 4
3
⇔ cos A + cos B + cos C ≤
2
1
Bất đẳng cuối đúng và ta có đpcm. Đẳng thức xảy ra khi và chỉ khi a= b= c= .
3
676
Cty TNHH MTV DVVH Khang Việt

Bài 8. Theo giả thiết tồn tại 3 góc nhọn của một tam giác sao cho:
= a tan= =
A, b tan B, c tan C .
Khi đó:
B+C B −C
P = 2cosA + cos B + cos C = 2cosA + 2cos cos .
2 2
2
C C  C 1 7 7
≤ 2 − 4sin 2 + 2sin =−4  sin −  + ≤
2 2  2 4 4 4
x z
Bài 9. Từ giả thiết ta có: xz + + =1.
y y

n
A 1 B C
Do đó tồn tại 3 góc của một tam giác sao= = =

.v
cho x tan , tan , z tan .
2 y 2 2
Khi đó ta có:

om
2 2 3 A B C
P= − + = 2cos 2 − 2sin 2 + 3cos 2
A B C 2 2 2
1 + tan 2 1 + cot 2 1 + tan 2 .c
2 2 2
C A+ B A− B C
= cos A + cos B + 3 − 3sin 2 = 2cos + 3 − 3sin 2
ok
cos
2 2 2 2
2
C C  C 1  10 10
≤ 2sin − 3sin 2 + 3 =−3  sin −  + ≤
bo

2 2  2 3 3 3
C 1
Đẳng thức xảy ra khi và chỉ khi sin= = ,A B
et

2 3
1 1
vi

⇒=x ,=
y 2,=
z .
2 2 2
ng

10 1 1
Vậy giá trị lớn nhất của P bằng đạt tại
= =
,y = x
2, z .
3 2 2 2
a

Cách 2: Từ điều kiện cho phép ta giảm biến biểu thức P về hai biến x và y.
kh

y−x
xyz + x − y + z = 0 ⇒ z = ≥0⇒ y ≥ x.
1 + xy
Khi đó ta có:

P=
(
2 y 2 − x2 ) +
3
=
(
2 y 2 − x2 ) +
( )
3 x 2 y 2 + 2 xy + 1

(1 + x )(1 + y )
2 2
 y−x 

2

 +1
(1 + x )(1 + y
2 2
) (1 + x )(1 + y )
2 2

 1 + xy 

677
Khám phá tư duy Kỹ thuật giải bất ĐT Bài toán Max – Min – Đặng Thành Nam

(
2 y 2 − x2 ) +
( ) (
3 x 2 y 2 + x 2 + y 2 + 1 − 3 x 2 − 2 xy + y 2 )
(1 + x )(1 + y )
2 2
(1 + x 2
)(1 + y ) 2

2( y − x )
2 2
3( x − y ) ( y − x )( 5 x − y ) + 3
2
− = +3
(1 + x )(1 + y ) (1 + x )(1 + y ) (1 + x )(1 + y )
2 2 2 2 2 2

Sử dụng bất đẳng thức AM-GM và C-S ta có:


 3( y − x ) + 5x − y 
2
3 ( y − x )( 5 x − y ) ≤ 
 2
 = ( x + y) ≤ 1+ x 1+ y .

2 2 2
( )( )

n
1 10
Suy ra P ≤ + 3 = .

.v
3 3
3 y − 3 x = 5 x − y 

om
1
 x = 1
Đẳng thức xảy ra khi và chỉ khi  x 1 ⇔ 2 ⇒z= .
1 = y y = 2 2 2
 
Bài tập tương tự
.c
1) Cho a,b,c là các số thực dương thỏa mãn điều kiện b = a + c + abc .
ok
2 2 3
Tìm giá trị lớn nhất của biểu thức P = . − +
a +1 b +1 c +1 2 2 2
bo

2) Cho a,b,c là các số thực dương thỏa mãn điều kiện 2014ac + bc + ca =
2014 .
2 2b 2 3
et

Tìm giá trị lớn nhất của biểu thức P = − + 2 .


a + 1 b + 2014 c + 1
2 2

3) Cho x,y,z là các số thực dương thỏa mãn điều kiện xy + yz + zx =


vi

1.
x y 2z
ng

Tìm giá trị lớn nhất của biểu thức P = + + .


x +1
z +1
2
y +1
2 2

4) Cho x,y,z là các số thực dương thỏa mãn điều kiện xz − yz − xy =


1.
a

2 x2 2 y2 3z 2
kh

Tìm giá trị nhỏ nhất của biểu thức P = − + .


1 + x2 1 + y2 1 + z2
Bài 10. Sử dụng đẳng thức:

( a + 1)(b + 1)( c + 1)=


2 2 2
( ab + bc + ca − 1)2 + ( a + b + c − abc )2 .
Suy ra a + b + c =abc .
Do đó tồn tại 3 góc của một tam giác thỏa mãn
= =
a tan =
A, b tan B, c tan C .
Bài toán đưa về chứng minh tan A + tan B + tan C ≥ 3 3 .
Bất đẳng thức cuối luôn đúng và ta có đpcm.

678
Cty TNHH MTV DVVH Khang Việt

Bài 11. Điều kiện bài toán tương đương với:


ab bc bc ca ca ab ab bc ca
. + . + . = . . .
c a a b b c c a b
Do đó tồn tại ba góc trong một tam giác sao cho
ab bc ca
= tan
= A, =
tan B, tan C .
c a b
1 1 1 3
Khi đó P = + + = cos 2 A + cos 2 B + cos 2 C ≥ .
1 + tan A 1 + tan B 1 + tan C
2 2 2 4
Đẳng thức xảy ra khi và chỉ khi a= b= c= 3 .

n
Bài 12. Điều kiện bài toán tương đương với:

.v
ab bc bc ca ca ab ab bc ca
. + . + . = . . .

om
c a a b b c c a b
Do đó tồn tại ba góc trong một tam giác sao cho
bc ca ab
= tan
= A, =
tan B, tan C . .c
a b c
 
ok
1 1 1
Khi đó P = 2 + 3  + 
1 + tan A  1 + tan B 1 + tan C 
2 2

= (
cos 2 A + 3 cos 2 B + cos 2 C )
bo

cos 2 A + 3 ( cos 2 B + cos 2C ) + 1 + 2 3 4 3 −3



et

2 4
5
Chú ý cos 2 A + 3 ( cos 2 B + cos 2C ) ≥ − .
vi

2
ng

3+ 2 3
Đẳng thức xảy ra khi và chỉ khi a = 7 + 4 3, b =
c= .
3
Bài 13. Giả thiết đã cho tương đương với:
a

6bc + 3ac + 2ab 36abc


kh

6bc + 3ac + 2ab= 36abc ⇔ =


abc abc
bc ac ab bc ac ab
⇔6 +3 +2 =
6 .3 .2
a b c a b c
A B C A B C
Trong tam giác ta có: cot + cot + cot = cot .cot .cot .
2 2 2 2 2 2
Suy ra tồn tại ba góc trong một tam giác sao cho:
ab C ca B bc A
=
2 =
cot ;3 cot ; 6 = cot .
c 2 b 2 a 2

679
Khám phá tư duy Kỹ thuật giải bất ĐT Bài toán Max – Min – Đặng Thành Nam

1 1 1
Vì vậy VT = . . .
A B 2C
1 + cot 2
1 + cot 2
1 + cot
2 2 2
2
A 2B C 1 A− B A+ B  C
= sin 2 =
sin sin  cos − cos  sin
2 2 2 4 2 2  2
2 2
1 A− B C C 1 C C
=  cos − sin  sin ≤ 1 − sin  sin
4 2 2 2 4 2 2
3
 C  C  C 
  1 − sin  + 1 − sin  +  2sin  
1 C  C C 1  2  2  2 1

n
=− 1 sin 1 − sin  2sin ≤   =
8 2  2 2 8 3  27

.v
 

om
Bất đẳng thức được chứng minh.
b b
Bài 14. Từ giả thiết ta có: ac + a. + .c = 1.
2009 2009 .c
Suy ra tồn tại ba góc một tam giác sao cho:
ok
A b B C
= a tan = ; tan ; c = tan .
2 2009 2 2
Khi đó
bo

2 2 3 A B C
P= − + = 2cos 2 − 2sin 2 + 3cos 2
A 1 C 2 2 2
1 + tan 2 1+ 1 + tan 2
et

2 2 B 2
tan
2
vi

2
C 1 A− B  C 1 A − B  10
=cos A + cos B + 3(1 − sin2
) =3 + cos 2 −  3 sin − ≤
2 
cos
ng

2 3 2  2 3 3
Bài tập tương tự
a

1) Cho a,b,c là các số thực dương thoả mãn điều kiện abc + a + b =c.
kh

a b c2
Tìm giá trị lớn nhất của biểu thức P = + . −
a 2 + 1 b2 + 1 c2 + 1
2) Cho a,b,c là các số thực dương thoả mãn điều kiện abc + a + b =c.
2 (1 + ab ) c
=
Tìm giá trị lớn nhất của biểu thức P + .
(1 + a )(1 + b )
2 2
c +12

3) Cho a,b,c là các số thực dương thoả mãn điều kiện ab =1 + bc + ca .


a2 b2 c
Tìm giá trị lớn nhất của biểu thức P = + + .
a +1 b +1 c +1
2 2 2

680
Cty TNHH MTV DVVH Khang Việt

4) Cho a,b,c là các số thực dương thoả mãn điều kiện ab =1 + bc + ca .


3 b 3c
Tìm giá trị nhỏ nhất của biểu thức P = . + −
a +1 b +1 c +1 2 2 2

5) Cho a,b,c là các số thực dương thoả mãn điều kiện ab =1 + bc + ca .


1 3 3
Tìm giá trị nhỏ nhất của biểu thức P = + − .
a +1 b +1 c +1
2 2 2

yz zx xy
Bài 15. Đặt a 2=
= , b 2= ,c 2 .
( x + y )( x + z ) ( y + z )( y + z ) ( z + x )( z + y )
( )≥ ( )

n
4 y 2 z + yz 2 + x 2 z + xz 2 − 2 xyz 4 yz 2 + xz 2
Khi đó a + b − abc
= 2 2
.
( x + y )( y + z )( z + x ) ( x + y )( y + z )( z + x )

.v
Tương tự suy ra:

om
(
4 yz 2 + xz 2 )
∏(a 2
+ b − abc ≥ ∏
2
) ( x + y )( y + z )( z + x )
≥ a 2b 2 c 2
.c 3
Bài 16. Bất đẳng tương đương với: cos A + cos B + cos C ≤ .
ok
2
ab bc ca
=
Với cos A = ;cos B = ;cos C .
bo

2 2 2
Bài 17. Theo giả thiết tồn tại các góc nhọn trong một tam giác sao cho
= = =
et

x tan A; y tan B; z tan C .


Khi đó gọi P là biểu thức vế trái ta có
vi

2 1 1
P= + +
ng

1 + tan 2 A 1 + tan 2 B 1 + tan 2 C


= 2cos A + cos B + cos C
B −C
a

 A A
= 2 1 − 2sin 2  + 2sin cos
 2
kh

2 2
 A A
≤ 2 1 − 2sin 2  + 2sin
 2 2
2
9  A 1 9
=− 4  sin −  ≤
4  2 4 4
Bất đẳng thức được chứng minh.
Bài 18. Trước hết,ta biến đổi P về dạng:
(a + b − c)(b + c − a )(c + a − b)( a 2 + b 2 + c 2 )
8P =
a 2b 2 c 2
681
Khám phá tư duy Kỹ thuật giải bất ĐT Bài toán Max – Min – Đặng Thành Nam

Bây giờ ta sẽ chứng minh rằng:


(a + b + c)(a + b − c)(b + c − a )(c + a − b)(a 2 + b 2 + c 2 ) ≤ 9a 2b 2 c 2 (1)
Thật vậy,nếu (a + b − c)(b + c − a)(c + a − b) ≤ 0 bất đẳng thức trên đúng
Nếu (a + b − c)(b + c − a)(c + a − b) > 0 ta thấy rằng 3 số a + b – c; b + c – a;
c + a – b không thể có đồng thời 2 số âm.
Vì thế, ta có thể quy bài toán về chứng minh bất đẳng thức đúng với 3 cạnh 1
tam giác.
Ta xét tam giác ABC với bán kính ngoại tiếp= R; a BC= ; b CA=; c AB .
9a 2 b 2 c 2
Ta có : 9R 2 ≥ a 2 + b 2 + c 2 hay ≥ (a 2 + b 2 + c 2 ) .

n
16 S 2

.v
hay 9a 2b 2 c 2 ≥ (a + b + c)(a + b − c)(b + c − a )(c + a − b)(a 2 + b 2 + c 2 )
Vậy ta có (1) được chứng minh.

om
9 3 3
Suy ra 8 P ≤ = hay P ≤ .
6 2 16

= a 2cos= =
A; b 2cos B; c 2cos C .
.c
Bài 19. Theo giả thiết tồn tại ba góc nhọn trong một tam giác sao cho
ok
Bất đẳng thức trở thành:
cos sA 4 − cos 2 A + cos B 4 − cos 2 B + cos C 4 − cos 2 C ≤ 3 3
bo

.
3 3
⇔ cos A sin A + cos B sin B + cos C sin C ≤
2
et

cos A ≤ cos B ≤ cos C


Không mất tính tổng quát giả sử A ≥ B ≥ C ⇒  .
vi

sin A ≥ sin B ≥ sin C


Theo bất đẳng thức Chebyshev ta có
ng

1
( cos A + cos B + cos C )( sin A + sin B + sin C )
cos A sin A + cos B sin B + cos C sin C ≤
3
a

1 3 3 3 3 3
≤ . . =
kh

3 2 2 4
Bất đẳng thức cuối đúng và ta có đpcm.
Bài 20. Sử dụng bất đẳng thức C –S ta có

(
4 − a 2 + 4 − b 2 + 4 − c 2 ≤ 3 12 − a 2 − b 2 − c 2 . )
Ta chỉ cần chứng minh a 2 + b 2 + c 2 ≥ 3 .
Thật vậy theo giả thiết tồn tại ba góc nhọn một tam giác sao cho
bc ca ab
=
cos A = ,cos B = ,cos C .
2 2 2
682
Cty TNHH MTV DVVH Khang Việt

2cos B cos C 2cos C cos A 2cos A cos B


=
Suy ra a = ,b = ,c .
cos A cos B cos C
2 2 2
 2cos B cos C   2cos C cos A   2cos A cos B 
Vậy ta chứng minh   +  +  ≥ 3.
 cos A   cos B   cos C 
Bất đẳng thức đúng do
2 2 2
 cos B cos C   cos C cos A   cos A cos B  3
 +  +  ≥ cos A + cos B + cos C ≥
2 2 2

 cos A   cos B   cos C  4
Ta có đpcm.
1 1 1

n
Bài 21. Theo giả thiết ta có + z +  =
1.
xy x y

.v
1 A 1 B C
Do đó tồn tại ba góc một tam giác sao=
cho =
tan , =
tan , z tan .

om
x 2 y 2 2
Khi đó sử dụng bất đẳng thức C –S và AM – GM ta có
 1  .c
2 1 + 
=P  xy 
+ 2
z

2
+ 2
z
ok
 1  1  z +1  1  1  z +1
1 + 2  1 + 2   1 +   1 + 
 x  y   x2   y2 
bo

C
tan
1 1 z 1 1 2
≤ + + 2 = + + .
et

1+
1
1 +
1 z + 1 1 + tan 2 A 1 + tan 2 B 1 + tan 2 C
x2 y2 2 2 2
vi

A B 1
= cos 2 + cos 2 + sin C
ng

2 2 2
3
1+
1 4+3 3
= 1 + ( cos A + cos B + sin C ) ≤ 1 + 2 =
a

.
2 2 4
kh

4+3 3
Vậy giá trị lớn nhất của P bằng ,
4
π 2π
đạt tại A = B = ,C = ⇒ x = y = 2 + 3; z = 3 .
6 3

683
Khám phá tư duy Kỹ thuật giải bất ĐT Bài toán Max – Min – Đặng Thành Nam

CHỦ ĐỀ 2: KỸ THUẬT SỬ DỤNG BẤT ĐẲNG THỨC SCHUR

Nội dung chủ đề này đề cập đến bất đẳng thức Schur và một số bài toán áp
dụng dạng cơ bản nhất. Để chi tiết hơn các bạn có thể tìm đọc một số bài viết trong
[1].[2].

A. NỘI DUNG PHƯƠNG PHÁP


Với mọi a,b,c,k là các số thực không âm ta có
a k ( a − b )( a − c ) + b k ( b − c )( b − a ) + c k ( c − a )( c − b ) ≥ 0

n
Với k = 1 ta có bất đẳng thức Schur bậc 3

.v
a3 + b3 + c3 + 3abc ≥ ab ( a + b ) + bc ( b + c ) + ca ( c + a )
.

om
Một số dạng tương đương:
( x + y + z )3 + 9 xyz ≥ 4 ( x + y + z )( xy + yz + zx ) ;
( x + y + z ) ( x 2 + y 2 + z 2 ) + 9 xyz ≥ 2 ( x + y + z )( xy + yz + zx ) .
.c
ok
Với k = 2 ta có bất đẳng thức Schur bậc 4:
( ) ( )
a 4 + b 4 + c 4 + abc ( a + b + c ) ≥ ab a 2 + b 2 + bc b 2 + c 2 + ca c 2 + a 2 . ( )
bo

Trong quá trình sử dụng bất đẳng thức thường đưa về chứng minh các bất đẳng
thức ràng buộc giữa p,q,r được gọi là phương pháp bất đẳng thức Schur kết hợp
et

đổi biến p, q, r.
Các hằng đẳng thức cần lưu ý
vi

Ta đặt p = x + y + z; q = xy + yz + zx; r = xyz , khi đó


ng

ab ( a + b ) + bc ( b + c ) + ca ( c + a ) = pq − 3r ;
( a + b )( b + c )( c + a ) = pq − r ;
a

( ) ( ) ( )
ab a 2 + b 2 + bc b 2 + c 2 + ca c 2 + a 2 = p 2 q − 2q 2 − pr ;
kh

( a + b )( a + c ) + ( b + c )( b + a ) + ( c + a )( c + b ) = p 2 + q;
a 2 + b 2 + c 2 = p 2 − 2q;
a3 + b3 + c3 = p3 − 3 pq + 3r ;
a 4 + b 4 + c 4 = p 4 − 4 p 2 q + 2q 2 + 4 pr.
Nếu đặt L = p 2 q 2 + 18 pqr − 27 r 2 − 4q3 − 4 p3 r , khi đó

684
Cty TNHH MTV DVVH Khang Việt

pq + 3r ± L
a 2b + b 2 c + c 2 a =
2
( a − b )( b − c )( c − a ) =± L
Ngoài ra để đánh giá r với p,q ta thường sử dụng hai bất đẳng thức:

(
 p 4 p − q2

r ≥ max 0;
) 

 9 

( )(
 p2 − q 4 p − q2 

r ≥ max 0;
 )

n
 6p 

.v
B. BÀI TẬP MẪU
Bài 1. Cho a,b,c là các số thực dương có tổng bằng 3. Chứng minh

om
12
abc + ≥ 15 .
ab + bc + ca
Lời giải
Sử dụng bất đẳng thức Schur bậc 3 ta có
.c
ok
( a + b + c )3 + 9abc ≥ 4 ( ab + bc + ca )( a + b + c ) ⇒ 3abc ≥ 4 ( ab + bc + ca ) − 9
36
Ta cần chứng minh 4 ( ab + bc + ca ) − 9 +
bo

≥ 15
ab + bc + ca
⇔ 4 ( ab + bc + ca ) − 24 ( ab + bc + ca ) + 36 ≥ 0 ⇔ 4 ( ab + bc + ca − 3) ≥ 0
2 2
et

Bất đẳng thức luôn đúng. Đẳng thức xảy ra khi và chỉ khi a= b= c= 1 .
vi

Bài 2. Cho a,b,c là các số thực dương có tích bằng 1. Chứng minh
ng

2 1 3
+ ≥ .
a + b + c 3 ab + bc + ca
Lời giải
a

Sử dụng bất đẳng thức Schur bậc 3 ta có


kh

( ab + bc + ca )3 + 9a 2b2c 2 ≥ 4 ( ab + bc + ca ) abc ( a + b + c )
( ab + bc + ca )3 + 9
⇔ a+b+c≤
4 ( ab + bc + ca )
8 ( ab + bc + ca ) 1 3
Vậy ta chỉ cần chứng minh + ≥
( ab + bc + ca ) + 9 3 ab + bc + ca
3

( )
Đặt t = ab + bc + ca, t ≥ 3 3 a 2b 2 c 2 = 3 bất đẳng thức trở thành

685
Khám phá tư duy Kỹ thuật giải bất ĐT Bài toán Max – Min – Đặng Thành Nam

8t 1 3
(
+ ≥ ⇔ 24t 2 + t t 3 + 9 ≥ 9 t 3 + 9
t +9 3 t
3 ) ( )
(
⇔ t 4 − 9t 3 + 24t 2 + 9t − 81 ≥ 0 ⇔ ( t − 3) t 3 − 6t 2 + 6t + 27 ≥ 0 )
Luôn đúng với t ≥ 3 . Bất đẳng thức được chứng minh.
Đẳng thức xảy ra khi và chỉ khi a= b= c= 1 .
Bài 3. Cho a,b,c là các số thực dương thoả mãn điều kiện a 2 + b 2 + c 2 =
3.
Chứng minh rằng 12 + 9abc ≥ 7 ( ab + bc + ca ) .
Lời giải
( )

n
Đặt t = a + b + c, 3≤t ≤3

.v
( a + b + c )2 − a 2 − b 2 − c 2 t2 − 3
=
⇒ ab + bc + ca =

om
.
2 2
Sử dụng bất đẳng thức Schur bậc 3 ta có
( a + b + c )3 + 9abc ≥ 4 ( a + b + c )( ab + bc + ca ) ⇒ 9abc ≥ 2t ( t 2 − 3) − t 3 .
.c
Vậy ta chỉ cần chứng minh
ok
(
7 t2 − 3 ) ⇔ ( t − 3)
( )
12 + 2t t − 3 − t ≥
2 3
( 2t + 5) ≥ 0 . 2
bo

2
Bất đẳng thức được chứng minh. Đẳng thức xảy ra khi và chỉ khi a= b= c= 1 .
Bài 4. Cho a,b,c là các số thực dương thoả mãn điều kiện ab + bc + ca =
et

3.
Chứng minh rằng a3 + b3 + c3 + 7 abc ≥ 10 .
vi

Lời giải
ng

Ta có a3 + b3 + c3 = ( a + b + c ) − 3 ( a + b )( b + c )( c + a )
3

= ( a + b + c )3 − 3 ( a + b + c )( ab + bc + ca ) + 3abc
a

( )
Đặt t = a + b + c, t ≥ 3 ( ab + bc + ca ) = 3 bất đẳng thức trở thành
kh

10abc + t 3 − 9t − 10 ≥ 0 .
Theo bất đẳng thức Schur bậc 3 ta có
( a + b + c )3 + 9abc ≥ 4 ( a + b + c )( ab + bc + ca ) ⇒ 9abc ≥ 12t − t 3 .
+ Nếu t ≥ 2 3 ⇒ 10abc + t 3 − 9t − 10 > 12t − 9t − 10 = 3t − 10 > 0 .
12t − t 3
+ Nếu 3 ≤ t ≤ 2 3 ⇒ abc ≥ .
9

686
Cty TNHH MTV DVVH Khang Việt

12t − t 3 3
Ta cần chứng minh 10. + t − 9t − 10 ≥ 0
9
(
⇔ ( t − 3) −t 2 − 3t + 30 ≥ 0)
( ( ) )
⇔ ( t − 3) 3 ( 4 − t ) + 16 − t 2 + 2 ≥ 0

Luôn đúng. Bất đẳng thức được chứng minh. Đẳng thức xảy ra khi và chỉ khi
a= b= c= 1 .
Bài 5. Cho a,b,c là các số thực dương. Chứng minh

( a + b + c )( ab + bc + ca ) ( a3 + b3 + c3 ) ≤ ( a 2 + b2 + c 2 )
3
.

n
.v
Lời giải
Chuẩn hoá p = a + b + c = 1 . Đặt q = ab + bc + ca; r = abc .

om
Ta có a 2 + b 2 + c 2 =1 − 2q; a3 + b3 + c3 =1 − 3q + 3r .

Khi đó ta cần chứng minh: q (1 − 3q + 3r ) ≤ (1 − 2q ) .


3

Sử dụng q 2= ( ab + bc + ca )2 ≥ 3abc ( a + b + c )=
.c 3r .
ok
Do đó q (1 − 3q + 3r ) =−
q 3q 2 + 3qr ≤ q − 3q 2 + q3 .

Vậy chỉ cần chứng minh q − 3q 2 + q3 ≤ (1 − 2q ) ⇔ (1 − q )( 3q − 1) ≥ 0 .


bo

3 2

Bất đẳng thức luôn đúng và ta có đpcm. Đẳng thức xảy ra khi và chỉ khi a= b= c .
et

Bài 5. Cho x,y,z là các số thực không âm thỏa mãn ( x + y )( y + z )( z + x ) > 0 và


xy + yz + zx =
1 . Tìm giá trị nhỏ nhất của biểu thức
vi

x ( x + y )( x + z ) + y ( y + z )( y + x ) + z ( z + x )( z + y )
ng

P= .
x+ y+z
Lời giải
a

x ( x + y )( x + =
z) (
x x 2 + x ( y + z ) + yz
= ) x 2 ( x + y + z ) + xyz .
kh

Ta có:

x ( x + y )( x + z ) xyz
Suy ra = x2 + .
x+ y+z x+ y+z
Tương tự ta có:
y ( y + z )( y + x ) xyz z ( z + x )( z + y ) xyz
=
y2 + ; =
z2 + .
x+ y+z x+ y+z x+ y+z x+ y+z

687
Khám phá tư duy Kỹ thuật giải bất ĐT Bài toán Max – Min – Đặng Thành Nam

xyz xyz xyz


Suy ra P =x2 + + y2 + + z2 + .
x+ y+z x+ y+z x+ y+z
Sử dụng bất đẳng thức Mincopski ta có:
2
 xyz  9 xyz
P ≥ ( x + y + z ) +  3 ( x + y + z )2 +
2
 = .
 x+ y+z  x+ y+z
Theo bất đẳng thức schur ta có:
( x + y + z )3 + 9 xyz ≥ 4 ( x + y + z )( xy + yz + zx )
9 xyz
⇒ (x + y + z) + ≥ 4 ( xy + yz + zx ) =
2

n
4
x+ y+z

.v
1
Do đó P ≥ 2 . Đẳng thức xảy ra khi và chỉ khi x= y= z= .

om
3
Bài 6. Cho x,y,z là các số thực không âm thỏa mãn x + y + z =
1 và xy + yz + zx > 0 ;
k là một số thực dương. .c
Tìm giá trị nhỏ nhất của biểu thức P =
( )
k x 2 + y 2 + z 2 + 9 xyz
.
ok
xy + yz + zx
Lời giải
bo

Ta có:
k ( x + y + z ) − 2k ( xy + yz + zx ) + 9 xyz
2
k 9 xyz
P= = + − 2k .
et

xy + yz + zx xy + yz + zx xy + yz + zx
Theo bất đẳng thức Schur ta có:
vi

( x + y + z )3 + 9 xyz ≥ 4 ( x + y + z )( xy + yz + zx )
ng

9 xyz 1
⇔ 1 + 9 xyz ≥ 4 ( xy + yz + zx ) ⇒ ≥4−
zy + yz + zx xy + yz + zx
a

k 1 k −1
kh

Suy ra P ≥ +4− = − 2k + 4 − 2k .
xy + yz + zx xy + yz + zx xy + yz + zx

TH1: Nếu k > 1 ⇒ k − 1 > 0; xy + yz + zx ≤


( x + y + z)
2
=
1
⇒ P ≥ 3 ( k − 1) + 4 − 2k = k + 1 .
3 3
1
Đẳng thức xảy ra khi và chỉ khi x= y= z= .
3
9 xyz
TH2: Nếu 0 < k ≤ 1 khi đó sử dụng: x 2 + y 2 + z 2 + ≥ 2 ( xy + yz + zx ) .
x+ y+z

688
Cty TNHH MTV DVVH Khang Việt

Ta có:
( )
x + y 2 + z 2 + 9 xyz ≥ 2 ( xy + yz + zx ) ⇔ k x 2 + y 2 + z 2 + 9kxyz ≥ 2k ( xy + yz + zx ) .
2

( )
Mặt khác: k ≤ 1 ⇒ k x 2 + y 2 + z 2 + 9 xyz ≥ k x 2 + y 2 + z 2 + 9kxyz . ( )
Do đó P
(
k x 2 + y 2 + z 2 + 9kxyz ) ≥ 2k . Đẳng thức xảy ra khi và chỉ khi
xy + yz + zx
1
x= y= , z= 0 hoặc các hoán vị.
2
Bài 7. Cho a,b,c là các số thực không âm thoả mãn điều kiện a 2 + b 2 + c 2 =

n
3.
Chứng minh rằng ( 2 − ab )( 2 − bc )( 2 − ca ) ≥ 1 .

.v
Lời giải

om
Bất đẳng thức cần chứng minh tương đương với
( 2 − ab )( 2 − bc )( 2 − ca ) − 1 ≥ 0
⇔ 7 − 4 ( ab + bc + ca ) + 2abc ( a + b + c ) − a 2b 2 c 2 ≥ 0
.c
⇔ ( a + b + c ) − ( a + b + c − abc ) − 4 ( ab + bc + ca ) + 7 ≥ 0
2 2
ok
( a + b + c )2 − a 2 − b 2 − c 2 t2 − 3
Đặt t = a + b + c, ( )
3 ≤ t ≤ 3 ⇒ ab + bc + ca = =
bo

.
2 2
Sử dụng bất đẳng thức Schur bậc 3 ta có
et

( a + b + c )3 + 9abc ≥ 4 ( a + b + c )( ab + bc + ca )
vi

( )
⇒ 9abc ≥ 2t t 2 − 3 − t 3 = t 3 − 6t ⇒ abc ≥
t 3 − 6t
9
ng

3
 a 2 + b2 + c2 
Chú ý abc ≤   =
1

a

 3 
kh

a + b + c − abc ≥ 3 3 abc − abc ≥ 3abc − abc = 2abc ≥ 0


t 3 − 6t 15t − t 3
a + b + c − abc ≤ t − =
9 9
2
 15t − t 3 
Vậy ta chứng minh t − 
 9 
2
− 2 t2 − 3 + 7 ≥ 0 ( )
 

( )
2
⇔ 81.13 − 81t 2 − t 2 t 2 − 15 ≥0

( ) (
⇔ 9 − t 2 9 + 9 − t 2 12 − t 2  ≥ 0
  )( )
689
Khám phá tư duy Kỹ thuật giải bất ĐT Bài toán Max – Min – Đặng Thành Nam

Luôn đúng do 3 ≤ t ≤ 3 . Bất đẳng thức được chứng minh đẳng thức xảy ra khi
và chỉ khi a= b= c= 1 .
Bài 8. Cho a,b,c là các số thực dương thoả mãn điều kiện abc = 1 .
b + c 3 c + a 3 a + b 5( a + b + c) + 9
Chứng minh rằng 3 + + ≤ .
2a 2b 2c 8
Lời giải
Đặt a =x3 , b =y 3 , c =z 3 ⇒ xyz =1 .
Bất đẳng thức đã cho trở thành:

n
y3 + z3 z 3 + x3
3
(
x3 + y 3 5 x + y + z + 9
3 3
)

.v
yz 3 + zx 3 + xy 3 ≤ .
2 2 2 8

om
Sử dụng bất đẳng thức AM – GM ta có

=
yz 3
y3 + z3
2
3
y+z
2
(
. yz. yz. yz. y 2 − yz + z 2 )
.c
4
y + z  3 yz + y 2 − yz + z 2 
3
 y+z
ok
≤3 .  =
 
2  4   2 
bo

3 3
z 3 + x3  z + x  3 x + y ≤ x+ y .
3 3
Tương tự ta có zx 3 ≤  ; xy  
2  2  2  2 
et

Cộng theo vế ba bất đẳng thức trên ta được

x3 + y 3 ( x + y ) + ( y + z ) + ( z + x )
3 3 3
vi

y3 + z3 z 3 + x3
yz 3 + zx 3 + xy 3 ≤ .
2 2 2 8
ng

Vậy ta chỉ cần chứng minh

( x + y )3 + ( y + z )3 + ( z + x )3 ≤ 5 ( x3 + y3 + z 3 ) + 9
a
kh

⇔ x3 + y 3 + z 3 + 3 ≥ xy ( x + y ) + yz ( y + z ) + zx ( z + x )
⇔ x3 + y 3 + z 3 + 3 xyz ≥ xy ( x + y ) + yz ( y + z ) + zx ( z + x )
Bất đẳng thức cuối là bất đẳng thức Schur bậc 3 nên ta có đpcm.
Đẳng thức xảy ra khi và chỉ khi a= b= c= 1 .

690
Cty TNHH MTV DVVH Khang Việt

C. BÀI TẬP RÈN LUYỆN


Bài 1. Cho a,b,c là các số thực dương có tổng bằng 1. Chứng minh

( ) ( 5
2 a3 + b3 + c3 + 3 a 2 + b 2 + c 2 + 12abc ≥ .
3
)
Bài 2. Tìm hằng số dương k lớn nhất thoả mãn bất đẳng thức với mọi a,b,c là các
số không âm có tổng bằng 3.
a3 + b3 + c3 + kabc ≥ 3 + k .
Bài 3. Chứng minh với a,b,c là các số thực dương có tích bằng 1, ta có
a+b b+c c+a  1 1 1
+ + + 6 ≥ 2 a + b + c + + +  .

n
c a b  a b c

.v
Bài 4. Cho a,b,c là các số thực dương. Chứng minh
a 2 + bc b 2 + ca c 2 + ab

om
9
+3 +3 ≥ .
( ) ( ) ( )
3
abc b + c2 2
abc c + a
2 2
abc a + b
2 2 a+b+c

Bài 5. Cho a,b,c là các số thực dương có tổng bằng 3. Chứng minh .c
1 1 1 3
+ + ≤ .
ok
9 − ab 9 − bc 9 − ca 8
Bài 6. Cho a,b,c là các số thực dương thoả mãn điều kiện a 2 + b 2 + c 2 =
3.
bo

3
Chứng minh rằng 5 ( a + b + c ) + ≥ 18 .
abc
et

Bài 7. Cho a,b,c là các số thực không âm thoả mãn điều kiện a 2 + b 2 + c 2 =
3.
Chứng minh rằng 21 + 18abc ≥ 13 ( ab + bc + ca ) .
vi

Bài 8. Cho a,b,c là các số thực không âm thoả mãn điều kiện a 2 + b 2 + c 2 =
3.
ng

1 1 1
Chứng minh rằng + + ≤1.
5 − 2ab 5 − 2bc 5 − 2ca
a
kh

D. HƯỚNG DẪN GIẢI – ĐÁP SỐ


Bài 1. Bất đẳng thức tương đương với

( ) ( ) 5
2 a3 + b3 + c3 + 3 a 2 + b 2 + c 2 ( a + b + c ) + 12abc ≥
3
(a + b + c) 3

⇔ ( a + b + c ) + 2abc ≥ 2ab ( a + b ) + 2bc ( b + c ) + 2ca ( c + a )


10 3 3 3
3
Bất đẳng thức trên là tổng của hai bất đẳng thức

691
Khám phá tư duy Kỹ thuật giải bất ĐT Bài toán Max – Min – Đặng Thành Nam

(
4 3
3
a + b3 + c3 ≥ 4abc )
2(a 3
+ b3 + c ) + 6abc ≥ 2ab ( a + b ) + 2bc ( b + c ) + 2ca ( c + a )
3

1
Bất đẳng thức được chứng minh. Đẳng thức xảy ra khi và chỉ khi a= b= c= .
3
3 27 15
Bài 2. Cho a = b = ,c = 0 ⇒ k + 3 ≤ ⇒k≤ .
2 4 4
15
Ta chứng minh k = là giá trị cần tìm.
4

n
Thật vậy bất đẳng thức trở thành

.v
15 27
a3 + b3 + c3 + abc ≥

om
4 4
( )
⇔ 4 a 3 + b3 + c3 + 15abc ≥ ( a + b + c )
3
.

⇔ 3( a 3
)
+ b3 + c3 + 3abc ≥ 3  ab ( a + b ) + bc ( b + c ) + ca ( c + a ) 
.c
Đây chính là bất đẳng thức Schur bậc 3.
ok
x y z
Bài 3. Theo giả thiết tồn tại các số thực dương x,y,z sao cho=a = ,b = ,c .
y z x
bo

Bất đẳng thức trở thành


x3 + y 3 + z 3 + 3 xyz ≥ xy ( x + y ) + yz ( y + z ) + zx ( z + x ) .
et

Đây chính là bất đẳng thức Schur bậc 3.


vi

Bài 4. Sử dụng bất đẳng thức AM – GM ta có


(
a b2 + c2 )
ng

3
(
abc b 2 + c 2 )≤ b+c+
a + bc
2 ab ( a + b ) + bc ( b + c ) + ca ( c + a )
=
a + bc
2 3 (
3 a 2 + bc )
a

( )
kh

a 2 + bc 3 a 2 + bc
⇒3 ≥
(
abc b + c 2 2
) ab ( a + b ) + bc ( b + c ) + ca ( c + a )

Tương tự cho 2 căn thức còn lại

Và ta chỉ cần chứng minh


(
3 a 2 + b 2 + c 2 + ab + bc + ca ) ≥
9
ab ( a + b ) + bc ( b + c ) + ca ( c + a ) a+b+c

⇔ a3 + b3 + c3 + 3abc ≥ ab ( a + b ) + bc ( b + c ) + ca ( c + a ) .
Bất đẳng thức cuối chính là bất đẳng thức Schur bậc 3.

692
Cty TNHH MTV DVVH Khang Việt

Bài 5. Quy đồng và rút gọn với điều kiện a + b + c =3 bất đẳng thức đã cho tương
đương với a 2b 2 c 2 − 19abc + 33 ( ab + bc + ca ) − 81 ≥ 0 .
Đặt p = a + b + c = 3; q = ab + bc + ca; r = abc .
Ta cần chứng minh r 2 − 19r + 33q − 81 ≥ 0 .
Theo bất đẳng thức Schur bậc ba ta có

r ≥ max 0;
(
 p 4p − q
2
)  =  12 − q  2

max 0; . 
 9   3 

n
+ Nếu q ≥ 2 3 ⇒ 33q + r 2 > 33q > 100 ≥ 19r + 81 bất đẳng thức đúng.

.v
12 − q 2
+ Nếu q ≤ 2 3 ⇒ r ≥ ⇒ q ≥ 3( 4 − r ) .

om
3
Vậy ta chỉ cần chứng minh r 2 − 19r + 33 3 ( 4 − r ) − 81 ≥ 0
⇔ 33 3 ( 4 − r ) ≥ 81 + 19r − r 2 .c
( )
2
⇔ 332.3 ( 4 − r ) ≥ 81 + 19r − r 2
ok
(
⇔ (1 − r ) r 3 − 37 r 2 + 162r + 6507 ≥ 0 )
bo

3
a+b+c
Bất đẳng thức cuối đúng vì r ≤   = 1.
 3 
et

Bất đẳng thức được chứng minh. Đẳng thức xảy ra khi và chỉ khi a= b= c= 1 .
vi

t2 − 3
Bài 7. Đặt t = a + b + c, ( )
3 ≤ t ≤ 3 ⇒ ab + bc + ca =
2
.
ng

Theo bất đẳng thức Schur bậc 3 ta có


( a + b + c )3 + 9abc ≥ 4 ( a + b + c )( ab + bc + ca ) ⇒ 9abc ≥ 2t ( t 2 − 3) − t 3 = t 3 − 6t .
a
kh

3 13.3
+ Nếu t ≤ 6 ⇒ ab + bc + ca ≤ ⇒ 21 + 18abc − 13 ( ab + bc + ca ) ≥ 21 − >0.
2 2
+ Nếu 6 ≤ t ≤ 3 ⇒ 21 + 18abc − 13 ( ab + bc + ca ) ≥

(
13 t 2 − 3 )
(
≥ 21 + 2 t − 6t −
3
) 2

=
4t 3 − 13t 2 − 24t + 81 ( t − 3) 4t − t + 27
=
2
≥ 0, ∀t ≥ 3
( )
2 2
Bất đẳng thức được chứng minh. Đẳng thức xảy ra khi và chỉ khi a= b= c= 1 .

693
Khám phá tư duy Kỹ thuật giải bất ĐT Bài toán Max – Min – Đặng Thành Nam

CHỦ ĐỀ 3: KỸ THUẬT DỒN BIẾN


A. NỘI DUNG PHƯƠNG PHÁP
Một bất đẳng thức đối xứng 3 biến a,b,c ta ký hiệu là P ( a, b, c ) ta phải chứng
minh P ( a, b, c ) ≥ 0 .
Dồn biến chính là một cách tách bất đẳng thức cần chứng minh thành hai bất
đẳng thức đơn giản hơn tức là đánh giá P (a, b, c) qua một biểu thức trung gian

 b + c b + c   b2 + c2 b2 + c2 
(
chẳng hạn P a, bc , bc , P  a,

)2
, 
2  
, P a ,
2
,
2
 ,... sau

 

n
đó đánh giá biểu thức trung gian.

.v
Như vậy tóm tắt theo 2 bước như sau :
Bước 1 : Chứng minh P ( a, b, c ) ≥ PTG và cần sắp thứ tự các biến để bất đẳng thức đúng.

om
Bước 2 : Chứng minh PTG ≥ 0 .
Thông thường thực hiện bước 2 trước để kiểm tra tính đúng của phương pháp
.c
dồn biến, sau đó thực hiện bước 1, chú ý sắp thứ tự các biến chẳng hạn
a = min {a, b, c} hoặc a = max {a, b, c} .
ok
Dưới đây tôi trình bày một số phương pháp dồn biến cụ thể phụ thuộc vào điều
bo

kiện bài toán.


1. Nếu điều kiện bài toán cho tổng 3 số a + b + c =k
 b+c b+c
Ta tìm cách đánh giá P ( a, b, c ) − P  a,
et

, ≥0
 2 2 
vi

 b+c b+c  k −a k −a


=
P  a, ,  P  a, , ≥0
 2 2   2 2 
ng

Khi đó ta cần sắp thứ tự a = min {a, b, c} hoặc a = max {a, b, c} .


Ví dụ 1. Cho a, b, c là các số thực dương thoả mãn điều kiện a + b + c = 1.
a
kh

1 1 1
Chứng minh rằng + + + 48(ab + bc + ca ) ≥ 25 .
a b c
Lời giải
1 1 1
Ta cần chứng minh: P (a, b, c) = + + + 48(ab + bc + ca ) − 25 ≥ 0 (1) .
a b c
Không mất tính tổng quát, có thể giả sử a = max {a, b, c} .
Để chứng minh (1), ta sẽ lần lượt chứng minh
b+c b+c
i) P (a, b, c) ≥ P (a, , ) với a, b, c > 0, a + b + c = 1, a = max {a, b, c} .
2 2

694
Cty TNHH MTV DVVH Khang Việt

ii) P ( a, t , t ) ≥ 0 với a + 2t =
1.
Để chứng minh i), ta xét
b+c b+c 1 1 4  b+c 
2
P ( a , b, c ) − P ( a , , )= + − + 48 bc −   
2 2 b c b+c   2  
 1 
=
(b − c) 2  − 12 
 bc(b + c) 

1 2
Vì a max {a, b, c} ⇒ a ≥ , b + c ≤ ⇒ bc ( b + c ) ≤
=
(b + c ) ≤ 2 . 3

3 3 4 27

n
1 27
− 12 ≥ − 12 > 0 .

.v
Suy ra
bc(b + c) 2
Vậy i) đã được chứng minh.

om
Để chứng minh ii), ta có
1 2
P ( a, t , t ) =
+ + 48(2at + t 2 ) − 25 .c
a t
1 2
= + + 48(2(1 − 2t )t + t 2 ) − 25
ok
1 − 2t t
2(4t − 1) 2 (3t − 1) 2
= ≥0
bo

(1 − 2t )t
1 1
Dấu bằng xảy ra khi và chỉ khi t = hoặc t = tương ứng với
et

4 3
1 1 1
vi

a= , b= c= hoặc a= b= c= .
2 4 3
ng

Bất đẳng thức được chứng minh.


Nhận xét.
1) Khi thực hành phương pháp dồn biến, nên bắt đầu từ bất đẳng thức ii) trước với
a

các lý do sau:
kh

+ Tìm được các điểm nghi vấn xảy ra dấu bằng. Biết được điểm xảy ra dấu bằng,
chúng ta có thể tìm được các cách tiếp cận thích hợp.
+ Nếu không chứng minh được ii) thì việc dồn biến là vô ích. Vì vậy phải làm
bước này trước.
b+c b+c
2) Bất đẳng thức P (a, b, c) ≥ P (a, , ) nói chung không đúng với mọi a, b, c.
2 2
Sử dụng tính đối xứng của bất đẳng thức, ta có thể sắp xếp thứ tự a, b, c để bất
đẳng thức này đúng.

695
Khám phá tư duy Kỹ thuật giải bất ĐT Bài toán Max – Min – Đặng Thành Nam

3) Việc chọn giá trị để dồn biến đến phụ thuộc vào biểu thức của P và điều kiện
ràng buộc. Trong trường hợp bài toán trên, do có điều kiện a + b + c = 1 nên ta
bắt buộc phải dồn biến đến các biến mà điều kiện này không thay đổi.
4) Khéo léo đánh giá điều kiện của biến ta hoàn toàn dồn được biến về chứng minh
bất đẳng thức trong trường hợp hai biến bằng nhau(xem ví dụ 2).

Ví dụ 2. Cho a,b,c là các số thực dương. Chứng minh rằng


a 2 + b 2 + c 2 + abc + 5 ≥ 3 ( a + b + c ) .
Lời giải

n
Ta cần chứng minh P (a, b, c) = a 2 + b 2 + c 2 + abc + 5 − 3 ( a + b + c ) ≥ 0 .

.v
Giả sử c là số nhỏ nhất trong ba số a, b, c. Xét hiệu

( ) ( ) 

om
2 2
P (a, b, c) − P ( ab , ab , c) = a − b a+ b − 3

 a + b a + b  (a − b) (2 − c)
2
P ( a , b, c ) − P  , ,c  = .c
 2 2  4
+ Nếu c ≥ 1 ⇒ a + b ≥ 2 ⇒ P (a, b, c) ≥ P ( ab , ab , c) .
ok
a+b a+b 
+ Nếu c < 1 ⇒ P(a, b, c) ≥ P  , ,c  .
bo

 2 2 
Do vậy ta chỉ cần chứng minh bất đẳng thức trong trường hợp hai số bằng nhau.
et

P ( ab , ab , c)= t 2 ( c + 2 ) − 6t + c 2 − 3c + 5, t= ab .

( )
Ta có ∆ 't =9 − ( c + 2 ) c 2 − 3c + 5 =− ( c − 1) ( c + 1) ≤ 0 .
vi

Điều đó chứng tở bất đẳng thức được chứng minh. Đẳng thức xảy ra khi và chỉ
ng

khi a= b= c= 1 .
Nhận xét. Ngoài ra ta có thể chứng minh đơn giản bằng Nguyên lý Dirichlet.
a

Trong ba số (a − 1),(b − 1),(c − 1) luôn có hai số cùng dấu giả sử là


kh

( a − 1)( b − 1) ≥ 0 ⇒ ab ≥ a + b − 1 ⇒ abc ≥ ac + bc − c .
Vậy ta chỉ cần chứng minh a 2 + b 2 + c 2 + 5 + ac + bc − c ≥ 3 ( a + b + c ) .
1
Ta có VT ≥ ( a + b )2 − 2ab + ( c − 3)( a + b ) + c 2 − 4c + 5 ≥ 0 .
2
Chú ý ∆ a +b =
2
(
( c − 3) − 2 c 2 − 4c + 5 = )
− ( c − 1) ≤ 0 ta có đpcm.
2

2. Nếu điều kiện bài toán cho tích ba số bằng 1


(
Ta tìm cách đánh giá P ( a, b, c ) ≥ P a, bc , bc ≥ 0 )
696
Cty TNHH MTV DVVH Khang Việt

(
P a,=
bc , bc )
 1 
P  2 , x, x  ≥ 0 với x = bc .
x 
Khi đó ta cần sắp thứ tự a = min {a, b, c} hoặc a = max {a, b, c} .
 b 
Chú ý. Một số trường hợp dồn biến về f (ta, b / t , c) với t ∈  ;1 .
 a 
Ví dụ 1. Cho a,b,c là các số thực dương có tích bằng 1.
1 1 1 6
Chứng minh rằng + + + ≥5.
a b c a+b+c

n
Lời giải

.v
1 1 1 6
Ta cần chứng minh: P (a, b, c) = + + + − 5 ≥ 0 (1) .
a b c a+b+c

om
giả sử a max {a, b, c} ⇒ a ≥ bc .
Không mất tính tổng quát =
Để chứng minh bất đẳng thức (1) ta thực hiện chứng minh 2 bất đẳng thức sau:
( )
i) P ( a, b, c ) ≥ P a, bc , bc với abc a max {a, b, c} .
= 1,=
.c
ok
 1 
ii) P  2 , x, x  ≥ 0 với x = bc .
x 
Chứng minh bất đẳng thức i)
bo

Ta có:
1 
et

1 1 1 6 2 6
P (a, b, c) − P (a, bc , bc ) = + + + − + + 
a b c a+b+c a bc a + 2 bc 
vi

=
1 1
+ −
2
+
6 2 bc − b − c ( )
( )
ng

b c bc ( a + b + c ) a + 2 bc

( ) ( )
2 2
b− c 6 b− c
a

= −
(a + b + c)(a + 2 )
kh

bc bc

( ) ( )
2
b− c ( a + b + c ) a + 2 bc − 6bc 
 
(
bc ( a + b + c ) a + 2 bc )
Do a ≥ bc , b + c ≥ 2 bc .
( ) (
⇒ ( a + b + c ) a + 2 bc − 6bc ≥ 3 bc a + 2 bc − 6bc = 3a bc > 0 )
 1 
Do đó P (a, b, c) ≥ P (a, bc , bc ) =
f  2 , x, x  với x = bc .
x 
697
Khám phá tư duy Kỹ thuật giải bất ĐT Bài toán Max – Min – Đặng Thành Nam

Chứng minh bất đẳng thức ii)


2 6
Ta chỉ cần chứng minh x 2 + 2
+ −5≥ 0 .
x 1
2x +
x2

( )
⇔ ( x − 1) 2 x 4 + 4 x3 − 4 x 2 − x + 2 ≥ 0 (luôn đúng).
2

Bất đẳng thức được chứng minh.


Đẳng thức xảy ra khi và chỉ khi x =1 ⇔ a =b =c =1 .
3. Nếu điều kiện bài toán cho tổng bình phương của 3 số a 2 + b 2 + c 2 =
k
 b2 + c2 b2 + c2 

n
Ta tìm cách đánh giá P ( a, b, c ) − P  a, , ≥0
 2 

.v
2
 
 b2 + c2 b2 + c2   

om
k − a2 k − a2
P =a, ,  P  a, , ≥0
 2 2   2 2 
   
Khi đó ta cần sắp thứ tự a = min {a, b, c} hoặc a = max {a, b, c} . .c
Ví dụ 1. Cho a,b,c là các số thực không âm thỏa mãn a 2 + b 2 + c 2 =
ok
3.
Chứng minh rằng a + b + c ≥ a 2b 2 + b 2 c 2 + c 2 a 2 .
bo

Lời giải
Không mất tính tổng quát giả sử
et

a ≤ b ≤ c ⇒ a ≤ 1, b 2 + c 2 ≥ 2 ⇒ b + c ≥ b 2 + c 2 =2 .
Ta cần chứng minh:
vi

(
f ( a , b, c ) = a + b + c − a 2 b 2 − b 2 c 2 − c 2 a 2 = a + b + c − a 2 b 2 + c 2 − b 2 c 2 ≥ 0 . )
ng

Xét
 b2 + c2 b2 + c2  b2 + c2  b2 + c2 
2
a

f ( a , b, c ) − f  a , =b+c−2 −b c +
2 2
 2 
,
 2 2  2  
kh

 

(b ) ( b + c )2 − 2 ( b 2 + c 2 )
2
2
− c2
= +
4
(
b + c + 2 b2 + c2 )
 
( b + c )2 −
( b − c )  
2 1
= ≥0


4 b + c + 2 b2 + c2 ( ) 

698
Cty TNHH MTV DVVH Khang Việt


( b + c )2

1
=
( b + c )3 + ( b + c )2
2 b2 + c2 − 4 ( )
4
(
b + c + 2 b2 + c2 ) 4  b + c + 2 b 2 + c 2 
 
( )
2 2 + 2 2.2 − 4
≥ >0
(
4  b + c + 2 b 2 + c 2 
 
)
Vậy ta chỉ cần chứng minh
 b2 + c2 b2 + c2 
f  a, , ≥0
 2 

n
2
 

.v
 3 − a2 3 − a2 
⇔ f  a, , ≥0
 2 

om
2
 

( )
⇔ a + 2 3 − a2 ≥ a2 3 − a2 + ( ) 14 (3 − a ) 2 2
.c
 
3 3 
⇔ ( a − 1)  ( a + 1) −
ok
2 2
 ≥ 0 (luôn đúng).


4 3 − a + 2 3 − a2 ( ) 

bo


3
( a + 1)2 −
3 3 3 3 2
≥ ( a + 1) − =
2
(
a + 2a ≥ 0, ∀a ∈ [ 0;1] . )
4 3 − a + 2 3 − a2( 4
) 4 4
et

Bất đẳng thức được chứng minh. Đẳng thức xảy ra khi và chỉ khi a= b= c= 1 .
vi

4. Dồn biến bằng hàm số


Dưới đây tôi trình bày kỹ thuật dồn biến bằng hàm số dạng đơn giản nhất khi
ng

điều kiện bài toán cho tổng các số không đổi (a + b + c =k ) dấu bằng đạt tại
một biến bằng 0 hoặc 2 số bằng nhau.
a

Khi đó sắp thứ tự lại biến số giả sử a ≥ b ≥ c lúc đó dấu bằng đạt tại
kh

2
b+c
c =0 ⇒ bc =0 hoặc b =c ⇒ bc =  .
 2 
t= b + c t2
nên ta đặt  ,0 ≤ s ≤ .
 s = bc 4
Đưa bất đẳng thức cần chứng minh về dạng hàm số f(s) và chỉ ra rằng f(s)
 t2 
nghịch biến hoặc đồng biến trên 0;  .
 4 

699
Khám phá tư duy Kỹ thuật giải bất ĐT Bài toán Max – Min – Đặng Thành Nam

+ Nếu dấu bằng đạt tại một số bằng 0 ta cần chỉ ra rằng f(s) nghịch biến tức
f ( s ) ≤ f (0) , lúc đó đưa về chứng minh bất đẳng thức với 2 biến a và t với
a+t =k . Vậy bài toán đưa về chứng minh bất đẳng thức một biến số.
t2
+ Nếu dấu bằng đạt tại 2 số bằng nhau khi đó b = c ⇒ s = tức f(s) là hàm đồng biến.
4
Ví dụ 1. Cho a,b,c là các số thực không âm có tổng bằng 1. Chứng minh

(a 2
)(
+ b2 b2 + c2 c2 + a 2 ≤ )( ) 1
32
.

Lời giải

n
Không mất tính tổng quát giả sử a ≥ b ≥ c .

.v
Gọi P là biểu thức vế trái ta có

(
P = ( b + c ) − 2bc  .  a 4 + a 2 ( b + c ) − 2bc + b 2 c 2  .
2 2
)

om
   
 t2 
Đặt b + c = t , bc = s, s ∈ 0;  ta có
 4  .c
( ) (
P = f ( s) = t 2 − 2s  a 4 + a 2 t 2 − 2s + s 2  . )
ok
 
 t2 
( ) ( )
Xét hàm số f ( s ) = t 2 − 2 s  a 4 + a 2 t 2 − 2 s + s 2  với s ∈ 0;  ta được
bo

 
 4 

( ) (
−2  a 4 + a 2 t 2 − 2 s + s 2  + 2 t 2 − 2 s s − a 2 .
f '( s ) = )( )
et

 
2
t2 1  2  1
vi

Chú ý s ≤ ≤   = ≤ a 2 ⇒ s − a 2 ≤ 0 ⇒ f '( s ) ≤ 0 .
4 4 3 9
ng

 t2 
Do đó f(s) là hàm nghịch biến trên đoạn 0;  .
 4 
a

( )
Suy ra P = f ( s ) ≤ f (0) = t 2 a 4 + a 2t 2 = a 2t 2 a 2 + t 2 ( )
kh

1
= t 2 (1 − t ) t 2 + (1 − t )  ≤
2 2
  32
 2
Chú ý khảo sát hàm một biến g (t )= t 2 (1 − t ) t 2 + (1 − t )  trên đoạn 0;  ta
2 2
   3
có kết quả trên.
Bất đẳng thức được chứng minh.
1
Đẳng thức xảy ra khi và chỉ khi a= b= , c= 0 hoặc các hoán vị.
2

700
Cty TNHH MTV DVVH Khang Việt

2 2
 c  c
=
Cách 2: Giả sử c min {a, b, c} ⇒ b + c ≤  b +  ; a 2 + c 2 ≤  a +  và
2 2
 2  2
2 2
 c  c
a 2 + b2 ≤  a +  +  b +  .
 2  2
c
2
c
Đặt x = a + , y = b + ⇒ x + y = 1; P ≤ x 2 y 2 x 2 + y 2 .
2
( )
Khi đó sử dụng bất đẳng thức AM – GM cho 2 số dương ta được
2
2
 2 xy + x 2 + y 2 
P≤x y 2 2
( x +
2
= y
xy
) 1 x+ y
.2 xy x 2 + y 2 ≤ 
2
 ( )  = 
1
.

n
2 2 2   2  32

.v
Bài tập tương tự
Cho a,b,c là các số thực không âm có tổng bằng 1. Chứng minh

om
(a 3 1
)(
+ b3 b3 + c 3 c 3 + a 3 ≤
256
. )( )
Ví dụ 2. Cho a,b,c là các số thực dương có tổng bằng 3. Chứng minh .c
bc ca ab 9 21
+ + + abc ≥ .
ok
a b c 4 4
Lời giải
bo

Gọi P là biểu thức vế trái và giả sử a = max {a, b, c} ta có

bc  b2 + c2  9 s a t − 2s 9
2
( )
et

P =+ a  + abc =+ + as .
a  bc  4 a s 4
 
vi

s a t − 2s 9
Xét hàm số f ( s ) = +
2
(
+ as trên nửa khoảng
)  t2 
 0;  ta có
ng

a s 4  4 
t2
( ) ( )
a

s 2
92a 2
+ 4 − 4 a 2
t 9a 2 + 4 − 4a 2 t
1 at 9
f '( s ) = − 2 + a = ≤ 16
kh

a s 4 4as 2 4as 2

=
t.
(9a + 4) t − 64a
2 2
≤ t.
(
2 9a 2 + 4 − 64a 2 ) =
t.
8 − 46a 2
<0
64as 2 64as 2 64as 2
 t2 
Vậy f(s) là hàm nghịch biến trên  0;  suy ra
 4
 
 t2  t2 9at 2 ( 3 − a ) 9a ( 3 − a )
2 2
P = f (s) ≥ f   = + 2a + = + 2a + .
 4  4a 16 4a 16
 

701
Khám phá tư duy Kỹ thuật giải bất ĐT Bài toán Max – Min – Đặng Thành Nam

Chú ý.
( 3 − a ) 2 + 2a =
+
9a ( 3 − a )
2
9 ( a − 1) ( a − 2 )
2 2
+
21 21
≥ .
4a 16 16a 4 4
Bất đẳng thức được chứng minh.
 1 1
Đẳng thức xảy ra khi và chỉ khi ( a; b; c ) = (1;1;1) ;  2; ;  hoặc các hoán vị.
 2 2
Ví dụ 3. Cho a,b,c là các số thực dương thoả mãn điều kiện a 2 + b 2 + c 2 =
3.
7  1 1 1  81
Chứng minh rằng a + b + c +  + + ≥ .
20  a b c  20
Lời giải

n
Không mất tính tổng quát giả sử a = max {a, b, c} .

.v
t
Đặt t =b2 + c2 ; s =bc, s ≤

om
.
2
Gọi P là biểu thức vế trái ta có
7 1 b 2 + c 2 + 2bc  .c
P =a + b 2 + c 2 + 2bc + + 
20  a bc 
 
ok
7 1 t + 2s 
=a + t + 2 s +  + 
20  a s 
bo

7 1 t + 2s 
Xét hàm số f ( s ) =a + t + 2 s +  +  ta có
s 
et

20  a

7 (t + s ) 20 s 2 − 7 ( t + s )
vi

1
f '( s ) = − =2 <0
t + 2 s 20 s 2 t + 2 s 20 s t + 2 s
ng

t
20 s 2 − 7 ( t + 2 s ) ≤ 20 s 2 − 7.4=
s 4 s ( 5s − 7 ) < 0, s ≤ ≤ 1
2
a

Vì vậy f(s) là hàm nghịch biến do đó


kh

t 7 1 2 2
P=f ( s ) ≥ f   =+
a 2t +  + 
2 20  a t 
7  1 2 2  81
= 3 − t + 2t +  + ≥
20  3 − t t  20
Ví dụ 4. Cho a,b,c là các số thực không âm có tổng bằng 3. Chứng minh
(a 3
)( )
+ b3 + c3 a3b3 + b3c3 + c3 a3 ≤ 36 ( ab + bc + ca ) .

Lời giải
Không mất tính tổng quát giả sử a ≥ b ≥ c .
702
Cty TNHH MTV DVVH Khang Việt

Viết lại bất đẳng thức đã cho dưới dạng

   ( 3
)
P =  a3 + ( b + c ) − 3bc ( b + c )  .  a3 ( b + c ) − 3bc ( b + c ) + b3c3  − 36a ( b + c ) − 36bc ≤ 0
3

 t2 
Đặt b + c = t , bc = s, s ∈ 0;  ta có
 4 

P= (a 3
)( )
+ t 3 − 3ts a3t 3 − 3a3ts + s 3 − 36at − 36 s .

 t2 
Xét hàm số f ( s ) = ( )( )
a3 + t 3 − 3ts a3t 3 − 3a3ts + s3 − 36at − 36s với s ∈ 0;  ta có
 4 

n
f '( s ) = ( ) ( )(
−3t a3t 3 − 3a3ts + s 3 + a3 + t 3 − 3ts 3s 2 − 3a 3t − 36 ≤ 0 . )

.v
Vì s 2 − a3t ≤ 0 với t ≤ 2, a ≥ 1, s ≤ 1 .

om
 t2 
Do đó f(s) là hàm nghịch biến trên 0;  suy ra
 4  .c
( )
f( s ) ≤ f(0) = a3 + t 3 a3t 3 − 36at = ( a + t ) − 3at ( a + t )  a3t 3 − 36at

3

ok
= 27 − 9t ( 3 − t )  t 3 ( 3 − t ) − 36t ( 3 − t )
3
bo

Vậy ta chỉ cần chứng minh  27 − 9t ( 3 − t )  t 2 ( 3 − t ) − 36 ≤ 0, ∀t ∈ [ 0;2] .


2

9
Đặt x = t ( 3 − t ) ≤ , ta chứng minh
et

4
( 27 − 9 x ) x 2 ≤ 36 ⇔ x3 − 3x 2 + 4 ≥ 0 ⇔ ( x − 2 )2 ( x + 1) ≥ 0 (luôn đúng).
vi

Bất đẳng thức được chứng minh.


ng

Đẳng thức xảy ra khi và chỉ khi =


a 2,=
b 1,=
c 0 hoặc các hoán vị.
Ví dụ 5. Cho x,y,z là các số thực không âm. Chứng minh
a

1
x4 ( y + z ) + y 4 ( z + x ) + z 4 ( x + y ) ≤ ( x + y + z )5 .
kh

12
Lời giải
Bất đẳng thức có dạng thuần nhất bậc 5 ta chuẩn hoá x + y + z =
1.
Ta phải chứng minh
1
x4 ( y + z ) + y 4 ( z + x ) + z 4 ( x + y ) ≤ .
12
1
giả sử x max { x, y, z} ⇒ x ≥
Gọi P là biểu thức vế trái và= ta có
3

703
Khám phá tư duy Kỹ thuật giải bất ĐT Bài toán Max – Min – Đặng Thành Nam

(
P= x 4 ( y + z ) + yz y 3 + z 3 + x y 4 + z 4 ) ( )
( 
4
)
= x 4 ( y + z ) + yz ( y + z ) − 3 yz ( y + z ) + x ( y + z ) + 2 y 2 z 2 − 4 ( y + z ) yz 
3 2

= tx 4 + xt 4 + ( 2 x − 3t ) s 2 + t 2 ( t − 4 x ) s

t= y + z  t2 
Với  , s ∈ 0;  .
 s = yz  4 
 t2 
Xét hàm số f ( s ) = tx 4 + xt 4 + ( 2 x − 3t ) s 2 + t 2 ( t − 4 x ) s trên đoạn 0;  ta có
 4 

n
t2 2 t3
2 ( 2 x − 3t ) s + t 2 ( t − 4 x ) ≤ 2 ( 2 x − 3t ) . + t (t − 4x ) =

.v
f '( s ) = − − 3 xt 2 ≤ 0 .
4 2

om
 t2 
Do đó f(s) là hàm nghịch biến trên đoạn 0;  .
 4 

( ) ( ) ≤ 121 .
.c
Vì vậy P = f ( s ) ≤ f (0) = tx 4 + xt 4 = xt x3 + t 3 = x (1 − x ) x3 + (1 − x )
3
ok
Bất đẳng thức được chứng minh.
 3+ 6 3− 6   3− 6 3+ 6 
Đẳng thức xảy ra khi và chỉ khi ( x; y; z ) =  0; ;  ;  0; ; 
bo

 6 6   6 6 
và các hoán vị của mỗi bộ số tương ứng.
et

Ví dụ 6. Cho x,y,z là các số thực không âm. Chứng minh

(8 x 2
)( )( )
+ yz 8 y 2 + zx 8 z 2 + xy ≤ ( x + y + z ) .
6
vi
ng

Lời giải
Bất đẳng thức có dạng thuần nhất bậc 6 ta huẩn hoá x + y + z =
1.

( )(
Ta phải chứng minh 8 x 2 + yz 8 y 2 + zx 8 z 2 + xy ≤ 1 . )( )
a
kh

1
Không mất tính tổng quát= giả sử x max { x, y, z} ⇒ x ≥ .
3
Gọi P là biểu thức vế trái của bất đẳng thức ta có
P= (8 x 2
) ( )
+ yz 64 y 2 z 2 + 8 x y 3 + z 3 + x 2 yz 
 
= (8 x + yz ){64 y z + 8 x ( y + z ) − 3 yz ( y + z )  + x }
2 2 2 3 2
yz
 
= (8 x 2
+ s )( 64 s + 8 xt − 24 xts + x s )
2 3 2

704
Cty TNHH MTV DVVH Khang Việt

t= y + z  t2 
Với  , s ∈ 0;  .
 s = yz  4 
 t2 
( )( )
Xét hàm số f ( s ) = 8 x 2 + s 64 s 2 + 8 xt 3 − 24 xts + x 2 s trên đoạn 0;  .
 4 

( )(
f '( s )= 64 s 2 + 8 xt 3 − 24 xts + x 2 s + 8 x 2 + s 128s + x 2 − 24 xt )
(
f ''( s )= 128s − 24 xt + x 2 + 128s + x 2 − 24 xt + 128 8 x 2 + s )
 171 
f '''( s ) = 384 ⇒ f ''( s ) ≥ f ''(0) = 1026 x 2 − 48 xt = 48 x  x−t > 0

n
 8 

.v
 t2  t2
Do đó f(s) là hàm lồi trên đoạn 0;  do đó f(s) đạt max tại 0 hoặc .
 4  4

om
3
 x + 1 − x  2 
Ta có f (0) = 64 x t = 64  x (1 − x )  ≤ 64 
3
  = 1 ; và
3 3
 2  
.c
 t2  2 t 2  4 x 2t 2 
ok
f  =  8 x +  4 t + 2 xt 3
+ 
4 4  4 
   
 (1 − x )2  x 2 (1 − x ) 
bo

2
=  8x2 +  4 (1 − x )4 + 2 x (1 − x )3 +  ≤1
 4  4 
  
et

Vậy ta có max P bằng 1. Bất đẳng thức được chứng minh.


1 1 1  1 1
Dấu bằng đạt tại ( x; y; z ) =  ; ;  ;  0; ;  hoặc các hoán vị của mỗi bộ số
vi

3 3 3  2 2
ng

tương ứng.
Bài tập tương tự
a

Cho x,y,z là các số thực không âm thoả mãn điều kiện xy + yz + zx > 0 .
kh

1 1 1 3
Chứng minh rằng + + ≥ .
8 x 2 + yz 8 y 2 + zx 8 z 2 + xy x+ y+z

Chú ý. Bài toán này là hệ quả của bất đẳng thức trên.
Như vậy qua việc dồn biến bằng hàm số đơn giản qua tổng và tích như trên
các bài toán được xử lý khá đơn giản. Bạn đọc thử so sánh lời giải các bài toán trên
với các cách dồn biến cũng như phương khác để thấy được hiệu quả ưu việt của
phương pháp này.

705
Khám phá tư duy Kỹ thuật giải bất ĐT Bài toán Max – Min – Đặng Thành Nam

B. BÀI TẬP CHỌN LỌC

( )(
Bài 1. Cho tam giác ABC chứng minh rằng 1 + cos 2 A 1 + cos 2 B 1 + cos 2 C ≥ )( ) 125
64
.

Lời giải
π
giả sử C min { A, B, C} ⇒ 0 < C ≤
Không mất tính tổng quát= .
3
2
( 
)(
Ta chứng minh 1 + cos A 1 + cos B ≥ 1 + cos 2
2

A+ B 
2 
 .
2
)
Thật vậy bất đẳng thức đã cho tương đương với

n
A− B
6cos C − cos ( A − B ) − 1 ≥ 0 .

.v
sin 2
2 

om
1
Bất đẳng thức đúng bởi vì 6cos C − cos ( A − B ) − 1 ≥ 6. − 2 = 1 > 0 .
2
Từ đó suy ra .c 2
( 2
)( 2 
)(
1 + cos A 1 + cos B 1 + cos C ≥ 1 + cos 2
A+ B 
)
 1 + cos C .
2 2
( )
ok
 2 
Vậy ta chỉ cần chứng minh
bo

2

1 + cos

2 A+ B 
2 
(
 1 + cos C ≥
2 125
64
)
et

2
 C
(
⇔ 1 + sin 2  1 + cos 2 C ≥
125
)
vi

 2 64
2
 1 − cos C 
( )
ng

125
⇔ 1 +  1 + cos C ≥
2
 2  64

(
⇔ ( 2cos C − 1) 4cos 2 C − 20cos C + 19 ≥ 0
2
)
a
kh

⇔ ( 2cos C − 1)  4cos 2 C − 1 + 20 (1 − cos C )  ≥ 0


2
 
Bất đẳng thức cuối luôn đúng do 4cos 2 C − 1 ≥ 0;1 − cos C > 0 .
Bất đẳng thức được chứng minh.
Đẳng thức xảy ra khi và chỉ khi A= B= C= 600 .
Bài 2. Cho x,y,z là các số thực không âm thoả mãn điều kiện x + y + z =
1.

Chứng minh rằng xy + yz + zx ≥ 12 x3 + y 3 + z 3 ( )( x y + y 2 z 2 + z 2 x2 .


2 2
)

706
Cty TNHH MTV DVVH Khang Việt

Lời giải
1
giả sử x max { x, y, z} ⇒ x ≥ .
Không mất tính tổng quát=
3
Ta cần chứng minh
(
xy + yz + zx − 12 x3 + y 3 + z 3 )( x )
y + y 2 z 2 + z 2 x2 ≥ 0 .
2 2

Gọi P là biểu thức vế trái ta có


P = x ( y + z ) + yz − 12  x3 + ( y + z ) − 3 yz ( y + z )  .  x 2 ( y + z ) − 2 x 2 yz + y 2 z 2 
3 2
   
( )(
= xt + s − 12 x3 + t 3 − 3ts x 2t 2 − 2 x 2 s + s 2 )

n
t= y + z  t2 

.v
Với  , s ∈ 0;  .
 s = yz  4 

om
( )( )
Xét hàm số f ( s ) = xt + s − 12 x3 + t 3 − 3ts x 2t 2 − 2 x 2 s + s 2 liên tục trên đoạn

 t2 
0;  ta có .c
 4 

( ) ( )( )
ok
f '( s ) =1 + 36t x 2t 2 − 2 x 2 s + s 2 + 24 x3 + t 3 − 3ts x 2 − s ≥ 0 do x 2 − s ≥ 0 .

 t2 
bo

Do đó f(s) là hàm đồng biến trên đoạn 0;  .


 4 
Vì vậy
et

( )
f ( s ) ≥ f (0) = xt − 12 x3 + t 3 x 2t 2 = x (1 − x ) − 12  x3 + (1 − x )  x 2 (1 − x )

3

2
vi

=x (1 − x ) 1 − 12 x (1 − x ) (1 − 3x (1 − x ) ) 
ng

1 
= x (1 − x ) 6 x (1 − x ) − 1 ≥ 0, ∀x ∈  ;1
2

3 
a

Bất đẳng thức được chứng minh.


kh

  x (1 − x ) =
0 

6 x (1 − x ) = 1  x= 1, y= z= 0

  3+ 3 3− 3
Đẳng thức xảy ra khi và chỉ khi  yz = 0 ⇔
= = ,y =,z 0.
x 6 6
x + y + z = 1 
  3+ 3 3− 3
x ≥ 1 =
 x =
, y 0,=
z
 3 6 6

707
Khám phá tư duy Kỹ thuật giải bất ĐT Bài toán Max – Min – Đặng Thành Nam

Bài tập tương tự


Cho x,y,z là các số thực không âm. Chứng minh

( x + y + z )5 ≥
9
4 6 −9
( )
x3 + y 3 + z 3 ( xy + yz + zx ) .

Bài 3. Cho x,y,z là các số thực dương có tổng bằng 1. Chứng minh
x y z 9
1≤ + + ≤ .
1 − yz 1 − zx 1 − xy 8
Lời giải
Bất đẳng thức vế trái là hiển nhiên theo C –S thật vậy

n
x
+
y
+
z

( x + y + z) 2
=
1
≥1.

.v
1 − yz 1 − zx 1 − xy x (1 − yz ) + y (1 − zx ) + z (1 − xy ) 1 − 3xyz
Ta chứng minh bất đẳng vế phải.

om
Gọi P là biểu thức vế trái và giả sử x = max { x, y, z} ta có

=P
x
+
(
y + z − x ( y + z ) − 2 yz
2
) .c
1 − yz x 2 yz − x ( y + z ) + 1
ok
=
x
+ 2
(
t − x t 2 − 2s )
1− s
bo

x s − xt + 1
t= y + z  t2 
với  , s ∈  0;  .
 4
et

 s = yz  
t − x t 2 − 2s ( )
vi

x  t2 
Xét hàm số f=
( s) + 2 liên tục trên  0;  ta có
1− s x s − xt + 1  4 
ng

f '( s ) =2 +
x x3t − 3 x 2t + 2 x
=2 +
x (
x x 2t − 3 xt + 2
>0
)
a

(1 − s ) ( ) (1 − s ) ( )
2 2
x 2 s − xt + 1 x 2 s − xt + 1
kh

3 3 5
do x 2t − 3 xt + 2 > 2 − 3 xt ≥ 2 − ( x + t )2 = 2 − = > 0 .
4 4 4
 t2 
Vậy f(s) là hàm đồng biến trên  0;  .
 4
 

708
Cty TNHH MTV DVVH Khang Việt

xt 2
 t2  t−
x 2
Do đó f ( s ) ≤ f   = 2 + 2 2
4
  1− t x t
− xt + 1
4 4
4 (1 − x ) − 2 x (1 − x )
2
4x
= +
4 − (1 − x ) x 2 (1 − x ) − 4 x (1 − x ) + 4
2 2

3 x3 − 5 x 2 + 3 x + 3
= −2.
x 4 − 3 x3 + x 2 − x − 6
3 x3 − 5 x 2 + 3 x + 3 9
( )

n
⇔ ( 3 x − 1) x 2 + 3 x − 6 ≤ 0 .
2
Ta chứng minh −2. ≤
x − 3x + x − x − 6
4 3 2 8

.v
Bất đẳng thức luôn đúng.

om
1
Bất đẳng thức được chứng minh. Đẳng thức xảy ra khi và chỉ khi x= y= z= .
3
Bài tập tương tự
Cho x,y,z là các số thực dương thoả mãn điều kiện x 2 + y 2 + z 2 =
.c
1.
ok
9 1 1 1 27
Chứng minh rằng ≤ + + ≤ .
4 1 + xy 1 + yz 1 + zx 4 ( x + y + z )2
bo

Bài 4. Cho a,b,c là các số thực dương thỏa mãn abc = 1 . Chứng minh rằng
( a + b )( b + c )( c + a ) ≥ 4 ( a + b + c − 1) .
et

Lời giải
Không mất tính tổng quát ta có thể giả sử a ≥ b ≥ c
vi

⇒ a ≥ 1 và ta cần chứng minh


ng

( a + b )( a + c )( b + c ) − 4 ( a + b + c − 1) ≥ 0
⇔ f (a, b, c) =  a 2 + a ( b + c ) + bc  ( b + c ) − 4 ( a + b + c − 1) ≥ 0 .
a

 
kh

Xét
f (a, b, c) − f (a, bc , bc )

( ) (
= a 2 b + c − 2 bc + a ( b + c ) − 4bc  + bc b + c − 2 bc − 4 b + c − 2 bc

2
 ) ( )
( ) ( )
2
= b− c  a 2 + a b + c + 2 bc + bc − 4  ≥ 0
 

( )
a 2 + a b + c + 2 bc + bc − 4 = a 2 + 2 a + a ( b + c ) + bc − 4

≥ a 2 + 2 a + 2a bc + bc − 4 = a 2 + 4 a + bc − 4 ≥ 1 + bc > 0
709
Khám phá tư duy Kỹ thuật giải bất ĐT Bài toán Max – Min – Đặng Thành Nam

Do a ≥ 1 .
 1 
Vì vậy ta chỉ cần chứng minh f (a, bc , bc ) ≥ 0 ⇔ f  2 , x, x  ≥ 0 với x = bc
x 
 1  1   1 
2 x  2 + x  2 + x  − 4  2 x + 2 − 1 ≥ 0
x  x   x 

( ) ( )
2
⇔ x3 + 1 ≥ 2 x + 2 x 4 − x3 ⇔ x 6 − 4 x 4 + 4 x3 − 2 x + 1 ≥ 0

⇔ ( x − 1)
2
(x 4
+ 2 x3 − x 2 + 1) ≥ 0

n
2
( 
)
2
⇔ ( x − 1)  x 2 − 1 + 2 x3 + x 2  ≥ 0 (luôn đúng).

.v
 
Đẳng thức xảy ra khi và chỉ khi x =1 ⇔ a =b =c =1 .

om
Cách 2: Ta có:
( a + b )( b + c )( c + a ) = ( a + b + c )( ab + bc + ca ) − abc = ( a + b + c )( ab + bc + ca ) − 1 .
Mặt khác:
.c
( ab + bc + ca )2 ≥ 3abc ( a + b + c )= 3 ( a + b + c ) ⇒ ab + bc + ca ≥ 3( a + b + c ) .
ok
Vậy bài toán được chứng minh nếu bất đẳng thức sau đúng:
bo

( a + b + c ) 3 ( a + b + c ) − 1 ≥ 4 ( a + b + c − 1) .
Thật vậy đặt t = ( )
a + b + c , t ≥ 3 . Xét hàm số f (t ) = 3t 3 − 4t 2 + 3 với t ≥ 3 .
et

Ta có f=
'(t ) 3 3t 2 − 8t > 0, ∀t ≥ 3 nên f (t ) đồng biến trên  3; +∞ hay )
vi

f (t ) ≥ f ( 3) =
ng

0.

Do đó ( a + b + c ) 3 ( a + b + c ) − 1 ≥ 4 ( a + b + c − 1) .
a

Bất đẳng thức được chứng minh.


kh

Bài 5. Cho a,b,c là các số thực dương có tích bằng 1. Chứng minh
3 1 1 1
a 2 + b2 + c2 + 6 ≥  a + b + c + + +  .
2 a b c
Lời giải
3 1 1 1
Đặt P ( a, b, c ) = a 2 + b 2 + c 2 + 6 −  a + b + c + + +  .
2 a b c
Không mất tính tổng quát giả sử a= min {a, b, c} ⇒ x= bc ≥ 1

710
Cty TNHH MTV DVVH Khang Việt

Ta chứng minh P ( a, b, c ) ≥ P a, bc , bc ( )
Thật vậy
3 2 
(
P ( a, b, c ) − P a, bc , bc = ) ( b − c )2 − 
2
1 1
b + c − 2 bc + + −
b c 
bc 

( ) ( )
1 2 2 3
= b − c 2 b + c − 3 − 
2  bc 

( )
1 2 3
≥ b − c  8bc − 3 − 
2  bc 

( )

n
1 2
≥ b− c ( 8 − 3 − 3) ≥ 0
2

.v
Mặt khác

om
 x − 6 x + 12 x − 6 x − 3 x + 2
( )
6 5 4 3 2
 1
P a,= bc , bc P=  2 , x , x 
x  2 x4

( x − 1)2  x 2 − 2 x − 1 + x 2 + 1 ( )
2 .c
  ≥0
ok
4
2x
Bất đẳng thức được chứng minh. Đẳng thức xảy ra khi và chỉ khi a= b= c= 1 .
bo

Bài 6. Cho a,b,c là các số thực không âm có tổng bằng 3. Chứng minh
( )(
1 ≤ a 2 − a + 1 b2 − b + 1 c2 − c + 1 ≤ 7 . )( )
et

Lời giải
giả sử a max {a, b, c} ⇒ a ≥ 1, b + c ≤ 2 .
Không mất tính tổng quát=
vi

t= b + c  t2 
ng

Đặt  , s ∈ 0;  ta có
 s = bc  4 

(a )
− a + 1 b 2 c 2 − bc ( b + c ) + b 2 + c 2 + bc − ( b + c ) + 1
a

P= 2
 
kh

= (a 2
)
− a + 1 b 2 c 2 + ( b + c ) − ( b + c ) − bc (1 + b + c ) + 1

2

.

= (a 2
− a + 1)  s

2
+ t 2 − t − s ( t + 1) + 1

Xét hàm số f ( s=
) (a 2
 )
− a + 1  s 2 + t 2 − t − s ( t + 1) + 1 ta có

f '( s=
) (a 2
)
− a + 1 ( 2 s − t − 1) < 0

t2 t 2 − 2t − 2 t ( t − 2 ) − 2
vì 2s − t − 1 ≤ − t −1 ≤ = <0.
2 2 2

711
Khám phá tư duy Kỹ thuật giải bất ĐT Bài toán Max – Min – Đặng Thành Nam

 t2   t2 
Do đó f(s) là hàm nghịch biến trên đoạn 0;  . Vì vậy f   ≤ P ≤ f (0) .
 4  4
 
Ta có
f (0)= (a 2
)(
− a +1 t2 − t +1 = ) (a 2
 )
− a + 1 ( 3 − a ) − ( 3 − a ) + 1 ≤ 7 .
2

 3
2
7
Vì bất đẳng thức tương đương với: a ( a − 3)  a −  +  ≤ 0 .
 2 4 

Đúng với mọi a ∈ [ 0;3] .

n
 t2   t4 
Tương tự: f  = (a t2
− a + 1  + t 2 − t − ( t + 1) + 1
2
)

.v
4
  16 4 

om
 t4 t2 
= ( 3 − t ) − ( 3 − t ) + 1  + t 2 − t − ( t + 1) + 1 ≥ 1
2
  16
 4 
bởi vì bất đẳng thức tương đương với .c
( t − 2 )2 ( t 4 − 5t 3 + 15t 2 − 24t + 24 ) ≥ 0 (luôn đúng).
ok
Bài 7. Cho a,b,c là các số thực không âm thỏa mãn điều kiện a + b + c =3.

( )(
Chứng minh rằng a 2 + a + 1 b 2 + b + 1 c 2 + c + 1 ≤ 27 . )( )
bo

Lời giải
et

Không mất tính tổng quát giả sử a = min {a, b, c} ⇒ a ≤ 1, b + c = 3 − a ≥ 2 .


vi

2
 b + c  2 b + c 
(
Ta chứng minh b + b + 1 c + c + 1 ≤ 
2
 2 
 +)( 2
+ 1 .

)
ng

2
Thật vậy bất đẳng thức tương đương với:

( )
a

1
( b − c )2 b2 + c 2 + 6bc + 4b + 4c − 4 ≥ 0 .
kh

16
Bất đẳng thức luôn đúng vì b 2 + c 2 + 6bc + 4b + 4c − 4 ≥ 4 ( b + c ) − 4 > 0 .
2
 b + c  2 b + c 
Vậy ta chỉ cần chứng minh a + a + 1 
 2 
 +
2
( + 1 ≤ 27 .
2

)
2
 3 − a  2 3 − a 
(
⇔ a + a + 1 
2
 2 
 + )2
+ 1 ≤ 27 .


712
Cty TNHH MTV DVVH Khang Việt

( )( )
2
⇔ a 2 + a + 1 a 2 − 8a + 19 ≤ 432 .

(
⇔ ( a − 1) a 4 − 13a3 + 60a 2 − 85a − 71 ≤ 0 .
2
)
Luôn đúng vì
(
a 4 − 13a3 + 60a 2 − 85a − 71 = a 4 − 13a3 − 85a − 11 + 60 a 2 − 1 < 0, ∀a ∈ [ 0;1] . ) ( )
Đẳng thức xảy ra khi và chỉ khi a= b= c= 1 .
Nhận xét. Lời giải trên xuất phát từ cách dồn biến
P (a, b, c)= (a 2
)(
+ a + 1 b2 + b + 1 c2 + c + 1 . )( )

n
 b+c b+c
 (đúng) ta chỉ cần giả sử a = min {a, b, c} .

.v
P ( a , b, c ) ≤ P  a , ,
 2 2 

om
Bài 8. Cho a,b,c là các số thực thỏa mãn điều kiện a + b + c = 6 và a, b, c ≥ 1 .
Tìm giá trị lớn nhất của biểu thức P =a 2 + 2 b 2 + 2 c 2 + 2 . ( )( )( )
Lời giải
.c
Không mất tính tổng quát giả sử
ok
a+b+c
c = min {a, b, c} ⇒ c ≤ = 2; a + b = 6 − c ≥ 4 .
3
bo

2
 a + b  2 
Ta chứng minh a + 2 b + 2 ≤ ( 2
 2 
)(
 + 2 .

2
)
et

Thật vậy bất đẳng thức tương đương với :


vi

( )
16 a 2b 2 + 2a 2 + 2b 2 + 4 ≤ ( a + b ) + 16 ( a + b ) + 64
4 2
ng

⇔ ( a + b ) − 16a 2b 2 ≥ 16 ( a − b )
4 2

⇔ ( a + b ) − 4ab  . ( a + b ) + 4ab  ≥ 16 ( a − b )
a

2 2 2
   
kh

⇔ ( a − b ) ( a + b ) + 4ab − 16  ≥ 0
2 2
 
Bất đẳng thức cuối luôn đúng do ( a + b ) ≥ 16 .
2

2 2
 a + b  2   6 − c  2 
Do đó P ≤ c + 2 . ( 2
 2 
)
 + 2 = c + 2

2
( ) . 
 2 
 + 2

2
 6 − c  2 
Xét hàm số f (c) =
c + 2 . 
 2 
( 
2
)
 + 2  trên đoạn [ 0;2] ta có

713
Khám phá tư duy Kỹ thuật giải bất ĐT Bài toán Max – Min – Đặng Thành Nam

3 5
f '(c) =
8
(
c − 20c 4 + 156c3 − 552c 2 + 800c − 352 )
3
( )(
= ( c − 2 ) c 2 − 12c + 44 c 2 − 6c + 4 ≥ 0, ∀c ∈ [ 0;2]
8
)
Do đó f(c) là hàm đồng biến trên đoạn [ 0;2] .
216 . Đẳng thức xảy ra khi và chỉ khi a= b= c= 2 .
Suy ra P ≤ f (c) ≤ f (2) =
Vậy giá trị lớn nhất của P bằng 216 đạt tại a= b= c= 2 .
Nhận xét. Chú ý đẳng thức
2
  a + b 2  ( a − b)
2

( 2 + a )( 2 + b ) ( )

n
2 2
− 2 +    = − a 2 + 6ab + b 2 − 16 .
  2   16

.v
(
Với những bài toán có tích đối xứng k + a 2 k + b 2 k + c 2 với k > 0 ta đánh )( )( )

om
2
  a + b 2 
giá giữa k + a ( 2
)( k + b ) 2
với  k +    .
  2   .c
2
  a + b 2  ( a − b )2 a 2 + b2 + 6ab − 8k .
ok
(
Ta có : k + a k + b −  k + 
2
)(
  2  
  =− 2
)
16
( )
bo

Bài 9. Cho a,b,c là các số thực không âm thỏa mãn điều kiện a + b + c = 1.

(
Tìm giá trị lớn nhất của biểu thức P= ab a 2 + b 2 + bc b 2 + c 2 + ca c 2 + a 2 . ) ( ) ( )
et

Lời giải
vi

 t2 
Không mất tính tổng quát giả sử a ≥ b ≥ c đặt b + c = t , bc = s, s ∈ 0;  .
ng

 4 
b+c t 2
Khi đó a = 1 − t ≥ = ⇔ t ≤ và
a

2 2 3

(
P= a3 ( b + c ) + a b3 + c3 + bc b 2 + c 2 ) ( )
kh

= a3 ( b + c ) + a ( b + c ) − 3bc ( b + c )  + bc ( b + c ) − 2bc 
3 2
   
(
=a3t + a t 3 − 3st + s t 2 − 2 s ) ( )
(
Xét hàm số f ( s ) =a3t + a t 3 − 3st + s t 2 − 2 s ta có ) ( )
f '( s ) = t 2 − 4 s − 3at = t ( t − 3a ) − 4 s ≤ 0 .

714
Cty TNHH MTV DVVH Khang Việt

 t2 
Do đó f(s) là hàm nghịch biến trên đoạn 0;  .
 4 

Suy ra f ( s ) ≤ f (0) = a3t + at 3 = (1 − t ) t + (1 − t ) t 3 .


3

 2
Xét hàm số g (t ) = (1 − t ) t + (1 − t ) t 2 với t ∈ 0;  ta có
3

 3
1
f '(t ) = (1 − 2t ) ; f '(t ) = 0 ⇔ t =
3
.
2
1
Ta có f’(t) đổi dấu từ dương sang âm khi đi qua t = nên f(t) đạt cực đại tại

n
2

.v
1 1 1
t= hay P ≤ f (t ) ≤ f   = .
2 2 8

om
1
Đẳng thức xảy ra khi và chỉ khi a= b= , c= 0 hoặc các hoán vị.
2
Nhận xét. Với a,b,c không âm và a + b + c = k với chứng minh tương tự ta có có .c
( ) k4
(
ab a 2 + b 2 + bc b 2 + c 2 + ca c 2 + a 2 ≤ ) ( )
ok
.
8
Bài 10. Cho a,b,c là các số thực không âm thỏa mãn điều kiện a + b + c =
1.
bo

Tìm giá trị lớn nhất và nhỏ nhất của biểu thức
a b c
P= + + .
et

b + c +1 c + a +1 3 3 3 3
a + b3 + 1
3

Lời giải
vi

Tìm giá trị lớn nhất của P


ng

Với a,b,c không âm ta luôn có


a b c
≤ a, ≤c. ≤ b,
b + c +1
3 3
c + a +1 3
a + b3 + 13 3
a

Cộng theo vế ba bất đẳng thức trên ta có P ≤ a + b + c =


1.
kh

Đẳng thức xảy ra khi và chỉ khi một số bằng 1 và hai số bằng 0.
Tìm giá trị nhỏ nhất của P
Sử dụng bất đẳng thức C-S ta có

P=
a
+
b
+
c

( a + b + c )2
(
b3 + c 3 + 1 c 3 + a 3 + 1 a 3 + b3 + 1 a b3 + c 3 + 1 + b c 3 + a 3 + 1 + c a 3 + b3 + 1 ) ( ) ( )
1
=
( ) (
a b + c + b c + a 3 + c a 3 + b3 + 1
3 3 3
) ( )
715
Khám phá tư duy Kỹ thuật giải bất ĐT Bài toán Max – Min – Đặng Thành Nam

Vậy để tìm giá trị nhỏ nhất của P ta tìm giá trị lớn nhất của biểu thức
( ) ( ) (
M = a b3 + c 3 + b c 3 + a 3 + c a 3 + b3 . )
 t2 
Không mất tính tổng quát giả sử a ≥ b ≥ c đặt b + c = t , bc = s, s ∈ 0; 
 4 
b+c t 2
Khi đó a = 1 − t ≥ = ⇔ t ≤ và
2 2 3

( )
M= a3 ( b + c ) + a b3 + c3 + bc b 2 + c 2( )
= a3 ( b + c ) + a ( b + c ) − 3bc ( b + c )  + bc ( b + c ) − 2bc 
3 2

n
   
( ) ( )

.v
=a3t + a t 3 − 3st + s t 2 − 2 s

om
( ) (
Xét hàm số f ( s ) =a3t + a t 3 − 3st + s t 2 − 2 s ta có )
f '( s ) = t 2 − 4 s − 3at = t ( t − 3a ) − 4 s ≤ 0 . .c
 t2 
Do đó f(s) là hàm nghịch biến trên đoạn 0;  .
ok
 4 
Suy ra f ( s ) ≤ f (0) = a3t + at 3 = (1 − t ) t + (1 − t ) t 3 .
3
bo

 2
Xét hàm số g (t ) = (1 − t ) t + (1 − t ) t 2 với t ∈ 0;  ta có
3

 3
et

1
f '(t ) = (1 − 2t ) ; f '(t ) = 0 ⇔ t =
3
.
vi

2
1
ng

Ta có f’(t) đổi dấu từ dương sang âm khi đi qua t = nên f(t) đạt cực đại tại
2
1 1 1
t= hay M ≤ f (t ) ≤ f   =
a

.
2 2 8
kh

1
Đẳng thức xảy ra khi và chỉ khi a= b= , c= 0 hoặc các hoán vị.
2
1 1
Tìm được GTLN của M bằng đạt tại a= b= , c= 0 hoặc các hoán vị.
8 2
1 8
Suy ra P ≥ = .
1 9
1+
8
8 1
Vậy giá trị nhỏ nhất của P bằng đạt tại a= b= , c= 0 hoặc các hoán vị.
9 2
716
Cty TNHH MTV DVVH Khang Việt

Nhận xét. Với a,b,c không âm và a + b + c =k với chứng minh tương tự ta có có

( ) (
ab a 2 + b 2 + bc b 2 + c 2 + ca c 2 + a 2 ≤ ) ( ) k4
8
.

Bài tập tương tự


Cho a,b,c là các số thực không âm có tổng bằng 1. Chứng minh
a b c 4
+ + ≥ .
1+ b + c
2 2
1+ c + a 2 2
1+ a + b
2 2 5

Bài 11. Cho a,b,c là các số thực không âm thỏa mãn điều kiện ab + bc + ca =
1.

n
1 1 1 5
Chứng minh rằng + + ≥ .

.v
a+b b+c c+a 2
Lời giải

om
Không mất tính tổng quát giả sử a = max {a, b, c} .

t= b + c  t2  1− s
Đặt  , s ∈ 0;  ta có at + s =1 ⇔ a = .c
 s = bc  4  t
ok
Gọi P là biếu thức vế trái ta có
1 1 1 1 2a + b + c
P= + + = + 2
b + c a + b a + c b + c a + ab + bc + ca
bo

1− s
=
1 2a + t 1
+ = +
2.
t
+t
=
1 t −2s + t + 2
+
2
( )
et

t a 2 + 1 t  1 − s 2 t 2 + ( s − 1)
t 2
  + 1
 t 
vi

(
1 t −2 s + t + 2
2
)  t2 
ng

Xét hàm số f ( s )= + trên đoạn 0;  ta có


t 2 + ( s − 1)
2
t  4 
a

 t2 
2t ( s − 1) − t s 
2 2
( ) 2t 3  − s 
kh

2t a − s 3 2
f '( s ) =  = ≥ 4  ≥0

( ) (
2 2
) ( )
2 2 2
t 2 + ( s − 1) t 2 + ( s − 1) t 2 + ( s − 1)
2

 t2 
Do đó f(s) là hàm đồng biến trên đoạn 0; 
 4 

1 t t +2
Vì vậy f ( s ) ≥ f (0) = + 2
2
( )
t t +1

717
Khám phá tư duy Kỹ thuật giải bất ĐT Bài toán Max – Min – Đặng Thành Nam

1 t t +2 5
2
( )
Ta cần chứng minh
t
+ 2
t +1
≥ ⇔ ( t − 1) 2t 2 − t + 2 ≥ 0 (luôn đúng).
2
2
( )
Bất đẳng thức được chứng minh. Đẳng thức xảy ra khi và chỉ khi
a= b= 1, c= 0 hoặc các hoán vị.
a+b
giả sử c max {a, b, c} ⇒ c ≥
Cách 2: Không mất tính tổng quát= .
2
1 − ab
Thay c = vào bất đẳng thức cần chứng minh ta có
a+b
1 1 1 5 1 1 1 5
Đặt P (a, b, c= + + −= + + − .

n
)
a+b b+c c+a 2 a+b a+ 1 − ab 1 − ab 2
b+

.v
a+b a+b
 

om
1
Xét hiệu P (a, b, c) − P  a + b, ,0 
 a + b 
   
 1 1 1   1 1  .c
=  + + − +a+b+ 
 a + b a + 1 − ab b + 1 − ab   a + b + 1 a+b
ok
 a+b a+b   a+b 
a+b a+b  1 
= + − ( + )  + 
bo

a b 1
1 + a 2 1 + b2  1 + ( a + b )2 

 
et

a+b a+b 1
Ta chứng minh + − ( a + b ) 1 + ≥0
1 + a2 1 + b2  1 + ( a + b )2 
vi

1 1 1
⇔ ab  ab ( a + b ) + 2ab − 2  ≤ 0
2
⇔ + ≥1+
ng

1+ a 2
1+ b 2
1 + (a + b)
2  

⇔ ab ( a + b ) ≤ 2 (1 − ab ) (luôn đúng).
2
a

Vì 2 (1 − ab =
) 2c ( a + b ) ≥ ( a + b ) ≥ ab ( a + b )
kh

2 2

Vậy ta cần chứng minh


1 1 1 5
P ( a + b, ,0) ≥ 0 ⇔ +a+b+ ≥
a+b a+b+
1 a+b 2
a+b
1 + ( a + b)
2
a+b 5
⇔ + ≥ .
1 + ( a + b) a+b
2 2

718
Cty TNHH MTV DVVH Khang Việt

1 + (a + b)2(a + b)
2
Đặt x= ≥
= 2.
a+b a+b
1 5
Ta cần chứng minh x + ≥ ⇔ 2 x 2 − 5 x + 2 ≥ 0 ⇔ ( 2 x − 1)( x − 2 ) ≥ 0 (luôn đúng).
x 2
Đẳng thức xảy ra khi và chỉ khi

a + b = 1
  a= c= 1, b= 0
=
ab + bc + ca 1 ⇔ .
  b= c= 1, a= 0
ab  ab ( a + b ) + 2ab − 2  =
2
0

n
Bài toán được chứng minh đẳng thức xảy ra khi và chỉ khi a= b= 1, c= 0 hoặc

.v
các hoán vị.

om
Cách 3: Đặt p = a + b + c, q = ab + bc + ca, r = abc .
Bất đẳng thức được viết lại dưới dạng:
1 + p2 5
≥ ⇔ ( p − 2 )( 2 p − 1) + 5r ≥ 0 .
p−r 2
.c
ok
+ Nếu p ≥ 2 bất đẳng thức hiển nhiên đúng.
+ Nếu 3 ≤ p ≤ 2 theo bất đẳng thức Schur bậc 3, ta có
bo

r≥
(
p 4q − p 2 )=
4p − p
.
3
et

9 9
4 p − p3
Ta chỉ cần chứng minh ( p − 2 )( 2 p − 1) + 5. ≥0
vi

9
 5 p ( p + 2 ) − 9 ( 2 p − 1) 
ng

⇔ (2 − p)  ≥0
 9 
( )
⇔ ( 2 − p ) 5 p2 − 8 p + 9 ≥ 0
a
kh

Bất đẳng thức luôn đúng ta có điều phải chứng minh.


Bài 12. Cho a,b,c là các số thực không âm thoả mãn điều kiện ab + bc + ca =
1.
1 1 1 1
Chứng minh rằng + + − ≥ 2.
a+b b+c c+a a+b+c
Lời giải
Không mất tính tổng quát giả sử a = min {a, b, c} .
1 1
+ Nếu b + c > 2 ⇒ − > − , khi đó theo bài toán trên ta có
a+b+c 2

719
Khám phá tư duy Kỹ thuật giải bất ĐT Bài toán Max – Min – Đặng Thành Nam

1 1 1 5 1 1 1 1
+ + ≥ ⇒ + + − > 2.
a+b b+c c+a 2 a+b b+c c+a a+b+c
Bất đẳng thức được chứng minh.
t= b + c  t2 
Ta xét với b + c ≤ 2, khi đó đặt  , s ∈ 0;  ,0 < t ≤ 2
 s = bc  4 
1− s
Ta có at + s =1 ⇔ a = .
t
Gọi biểu thức vế trái là P ta có
1 1 1 1 1 2a + b + c 1
P= + + − = + −

n
b+c a+b a+c a+b+c b+c a +1
2 a + b+c

.v
1− s
2. +t
1 2a + t 1 1 t 1
= + 2 − = + −

om
t a +1 a + t t 1− s  2 1− s
+t
  +1 t
 t 
t 2 + 2 − 2s
1
= + t. −
t
.c
t 2 + ( s − 1) −s + t 2 + 1
t 2
ok
1 t 2 + 2 − 2s t  t2 
Xét hàm số f ( s ) = + t. − trên đoạn 0;  ta có
t + ( s − 1) −s + t 2 + 1
2
bo

t 2
 4 

2t ( s − 1) − t 2 s 

2
− t (
2t 3 a 2 − s ) t
et

f '( s ) = = − ≤0.
( ) ( ) (t ) ( −s + t + 1)
2 2 2 2 2
t + ( s − 1) −s + t + 1 + ( s − 1)
2 2 2 2 2
vi

Bởi vì a 2 − s ≤ 0 .
ng

 t2   t2 
Do đó f(s) là hàm nghịch biến trên đoạn 0;  . Vì vậy f ( s ) ≥ f  .
4
 4   
a

 t2 
kh

Ta chỉ cần chứng minh f   ≥ 2 .


4
 
t2
t  2 +2
1 2 t
Ta có: f   − 2 ≥ 0 ⇔ + t. − 2 −2≥0
4 2
  t  t 2  3t
+1
t 2 +  − 1 4
4 

( )
⇔ ( 2 − t ) 6t 4 − 11t 3 + 10t 2 − 12t + 8 ≥ 0 ⇔ t ≤ 2 (luôn đúng).

720
Cty TNHH MTV DVVH Khang Việt

Bất đẳng thức được chứng minh. Đẳng thức xảy ra khi và chỉ khi a= b= 1, c= 0
hoặc các hoán vị.
Cách 2: Đặt p = a + b + c, q = ab + bc + ca, r = abc .
Bất đẳng thức được viết lại dưới dạng:
1 + p2 1
− ≥ 2 ⇔ p 2 ( p − 2 ) + r (1 + 2 p ) ≥ 0 .
p−r p
+ Nếu p ≥ 2 bất đẳng thức hiển nhiên đúng.

+ Nếu 3 ≤ p ≤ 2 theo bất đẳng thức Schur bậc 3 ta có r ≥


(
p 4q − p 2 )=
4p − p
.
3

n
9 9

.v
Ta chỉ cần chứng minh
4 p − p3
p2 ( 2 − p ) + (1 + 2 p ) ≥ 0 ⇔ p ( 2 − p )( p − 1)2 ≥ 0 .

om
9
Bất đẳng thức luôn đúng ta có điều phải chứng minh.
Bài 13. Cho a,b,c là các số thực không thoả mãn điều kiện ab + bc + ca ≥ 3 .
.c
1
Chứng minh rằg với mọi số thực k ≥ ta có
ok
3

(a 2
)( )( )
+ k b 2 + k c 2 + k ≥ ( k + 1) .
3
bo

Lời giải
Chú ý đẳng thức
et

(a 2
)( )(
+ k b 2 + k c 2 + k= ) ( k ( a + b + c ) − abc ) 2
+ k ( ab + bc + ca − k ) .
2
vi

1
Trước tiên ta chứng minh k = bất đẳng thức đúng, thật vậy
ng

3
2 2 3
 2 1  2 1  2 1  1  1 1 1 1 
 a +  b +  c +  ≥  ab + bc + ca −  ≥  3 −  =  + 1 .
a

 3  3  3  3  3  3  3 3 
kh

1
Với k ' ≥ k ≥ ta có
3
(a 2
)( )( )
+ k ' b2 + k ' c 2 + k ' =  a 2 + k + ( k '− k )  b2 + k + ( k '− k )  c 2 + k + ( k '− k ) 
   

( )( )( )
3
≥  3 a 2 + k b2 + k c 2 + k + k '− k  ≥ (1 + k + k '− k ) = ( k '+ 1)
3 3
 
Bài 14. Cho x,y,z là các số thực không âm thoả mãn điều kiện xy + yz + zx =
1.
1 1 1 1
Chứng minh rằng + + ≥2+ .
x+ y y+z z+x 2

721
Khám phá tư duy Kỹ thuật giải bất ĐT Bài toán Max – Min – Đặng Thành Nam

Lời giải
t= y + z  t2 
Không mất tính tổng quát giả sử x = max { x, y, z} đặt  , s ∈ 0;  .
 s = yz  4 
1− s
Ta có xt + s =1 ⇒ x = .
t
Gọi P là biểu thức vế trái ta có

1 x+ y + x+z 1 2 x + y + z + 2 x2 + 1
P= + = +
y+z x 2 + xy + xz + yz y+z x2 + 1

n
1− s
+t

.v
2.
1 t 2 1 t 2 + 2 − 2s 2t
= + + = + t . + .
t 1− s 
2

2 t 2
+ ( − ) 2
+ ( − ) 2

om
2
 1 s  t 1 s t 1 s
  +1   +1
 t   t 

t 2 + 2 − 2s 2t .c  t2 
=
Xét hàm số f ( s ) t. + trên đoạn 0;  ta có
t 2 + (1 − s )
2
t 2 + (1 − s )
2  4 
ok
f '( s ) =2t.
(1 − s )2 − t 2 s +
2t (1 − s )
=
(
2t 3 x 2 − s ) +
2t (1 − s )
≥0
( ) ( )
bo

2 2  t2 + 1− s 2 
3 2
 t2 + 1− s 2 
3
t 2 + (1 − s ) ( )  t + (1 − s )
2
( ) 
2
 
   
 t2 
et

Do đó f(s) là hàm đồng biến trên đoạn 0;  .


 4 
vi

2
 t 
t2 + 2 2t
ng

Vì vậy f ( s ) ≥ f (0) =t. 2 + = t + 2  .


t +1 t2 +1  t + 1 
a

1 t 1
Do đó P ≥ + t+ 2 ≥2+ .
t +1
kh

t 2
Bài 15. Cho a,b,c là các số thực không âm thoả mãn điều kiện ab + bc + ca > 0.
1 1 1 4
Chứng minh + + ≥ .
4a 2 + bc 4b 2 + ca 4c 2 + ab a + b + c
Lời giải
Giả sử a = max {a, b, c} , chứng minh
1 1 2 a+b
+ ≥ ,t = .
4a 2 + bc 4b 2 + ca 4t 2 + tc 2

722
Cty TNHH MTV DVVH Khang Việt

Thật vậy theo bất đẳng thức AM – GM ta có


1 1 2
+ ≥ .
4a + bc
2
4b + ca
2
( 4a 2
)(
+ bc 4b 2 + ca )
( ) ( )( )
( a − b )2  c 2 + a 2 + b2 + 6ab − 3c ( a + b )  ≥ 0 .
1
2
Và 4t 2 + tc − 4a 2 + bc 4b 2 + ca =
4 
1 1
Theo AM – GM ta có ≥ .
4c + ab 2
4c + t 22

Vậy ta chỉ cần chứng minh

n
2 1 4 4
+ ≥ =

.v
4t + tc
2
4c + t
2 2 a + b + c 2t +c
 1  1

om
2 1 4 2
⇔ − + − ≥ −
   
t  2t + c t
 4t + tc t   4c + t
2 2 2

−t −4c −2
⇔ + ≥
( ) ( )
4t 2 + tc 2t + 4t 2 + tc 4c 2 + t 2 t + 4c 2 + t 2 2t + c
.c
ok
2 1 4c
⇔ − − ≥0
t ( 2t + c )
(
4t 2 + tc 2t + 4t 2 + tc )
t 4c 2 + t 2 t + 4c 2 + t 2 ( )
bo

Bất đẳng thức trên là tổng của hai bất đẳng thức sau
et

1 1
− ≥ 0 (1)
3t ( 2t + c )
(
4t + tc 2t + 4t + tc
2 2
)
vi

5 4c
− ≥ 0 (2)
( )
ng

3t ( 2t + c ) t 4c 2 + t 2 t + 4c 2 + t 2

Bất đẳng thức được chứng minh.


a

Đẳng thức xảy ra khi và chỉ khi=


a b=
kh

, c 0 hoặc các hoán vị.


Bài 16. (Việt Nam TST 2006) Cho x,y,z là các số thực thuộc đoạn [1;2]. Chứng
1 1 1  x y z 
minh ( x + y + z )  + +  ≥ 6  + + .
x y z  y+z z+x x+ y
Lời giải
Ta cần chứng minh
1 1 1  x y z 
f ( x, y , z ) = ( x + y + z )  + +  − 6  + + ≥0.
 x y z  y+z z+x x+ y

723
Khám phá tư duy Kỹ thuật giải bất ĐT Bài toán Max – Min – Đặng Thành Nam

Không mất tính tổng quát giả sử x = max { x, y, z} .


Xét hiệu:
 y + z y + z  ( x + y + z )( y − z ) 6( x + y + z )( y − z ) 2
2
f ( x, y , z ) − f  x, ,  = −
 2 2  yz ( y + z ) ( x + y )( x + z )(2 x + y + z )
( x + y + z )( y − z ) 2 [( x + y )( z + x)(2 x + y + z ) − 6 yz ( y + z )]
=
( x + y )( y + z )( z + x)(2 x + y + z )
vì x là số lớn nhất trong ba số nên ( x + y )( z + x)(2 x + y + z ) > 6 yz ( y + z ) .
y+z
=
nên f ( x, y, z ) ≥ f ( x, t , t ) với t ≥1

n
2
Vì vậy ta chỉ cần chứng minh f ( x, t , t ) ≥ 0 .

.v
Thật vậy, bất đẳng thức tương đương với:

om
1 2  x 2t  (t − x) 2 (2t − x)
( x + 2t )  +  − 6  +  ≥ 0 ⇔ ≥ 0, .
 x t   2t t + x  tx(t + x)
Bất đẳng thức cuối đúng do 2t ≥ 2 ≥ x .
Đẳng thức xảy ra khi và chỉ khi x= y= z; x= 2, y= z= 1 .
.c
ok
Bài toán được chứng minh.
bo

C. BÀI TẬP RÈN LUYỆN


Bài 1. Cho a,b,c là độ dài 3 cạnh một tam giác thoả mãn điều kiện a 2 + b 2 + c 2 =
3.
et

Chứng minh rằng a + b + c ≥ 2 + abc .


Bài 2. Cho a,b,c là các số thực không âm thoả mãn điều kiện a 2 + b 2 + c 2 =
vi

3.
Chứng minh rằng ( 2 − ab )( 2 − bc )( 2 − ca ) ≥ 1 .
ng

Bài 3. Cho a,b,c là các số thực dương có tích bằng 1. Chứng minh
1 1 1 13 25
a

+ + + ≥ .
a b c a + b + c +1 4
kh

Bài 4. Cho a,b,c là các số thực dương thỏa mãn a, b, c ≥ 1 và a + b + c =6.


Tìm giá trị lớn nhất của biểu thức P = (
1 + a 2 1 + b2 1 + c2 . )( )( )
3
Bài 5. Cho a,b,c là các số thực không âm thỏa mãn a + b + c = .
2
Tìm giá trị nhỏ nhất của biểu thức P = (
1 + a 2 1 + b2 1 + c2 . )( )( )
Bài 6. Cho a,b,c là các số thực không âm thỏa mãn a + b + c =
1.
Tìm giá trị lớn nhất của biểu thức P = ( )(
a 3 + b3 b3 + c 3 c 3 + a 3 . )( )
724
Cty TNHH MTV DVVH Khang Việt

Bài 7. Cho x,y,z là các số thực dương có tích bằng 1. Chứng minh
x 2 + y 2 + z 2 + x + y + z ≥ 2 ( xy + yz + zx ) .
Bài 8. Cho a,b,c là các số thực dương có tích bằng 1. Chứng minh
1 1 1 1
a 2 + b2 + c2 ≥  a + b + c + + +  .
2 a b c
Bài 9. Cho x,y,z là các số thực dương có tổng bằng 1. Chứng minh
 x z  21
1
xyz
( y
− 54 x3 + y 3 + z 3 −  + +  ≥ .
 yz zx xy  2
)

n
Bài 10. Cho a,b,c là các số thực không âm có tổng bằng 1. Chứng minh

(a )( 1
)( )

.v
3
+ b3 b3 + c 3 c 3 + a 3 ≤
.
256

om
Bài 11. Cho a,b,c là các số thực dương có tích bằng 1. Chứng minh
1 1 1
+ ≤ 3. +
a − a +1 b − b +1 c − c +1
2 2 2
.c
Bài 12. Cho a,b,c là các số thực không âm có tổng bằng 1. Chứng minh
ok
a b4 c
+ . + ≥
1+ b + c
2
1+ c + a
2
1+ a + b 25 2 2 2

Bài 13. Cho x,y,z là các số thực không âm thoả mãn điều kiện x + y + z =
bo

3.

( )(
Chứng minh rằng 3 x 2 + 5 3 y 2 + 5 3 y 2 + 5 ≥ 512 . )( )
et

Bài 14. Cho x,y,z là các số thực không âm. Chứng minh

( x + y + z )5 ≥
9
(
x3 + y 3 + z 3 ( xy + yz + zx ) . )
vi

4 6 −9
ng

Bài 15. Cho x,y,z là các số thực dương thoả mãn điều kiện x 2 + y 2 + z 2 =
1.
9 1 1 1 27
≤ + + ≤
a

Chứng minh rằng .


4 1 + xy 1 + yz 1 + zx 4 ( x + y + z )2
kh

Bài 16. Cho a,b,c là các số thực không âm thoả mãn điều kiện ab + bc + ca > 0 .
1 1 1 6
Chứng minh rằng + + ≥ .
a 2 + bc b 2 + ca c 2 + ab a+b+c

D. HƯỚNG DẪN GIẢI – ĐÁP SỐ


Bài 1. Bất đẳng thức cần chứng minh tương đương với
P ( a, b, c ) = a + b + c − abc − 2 ≥ 0 .
Không mất tính tổng quát gỉa sử a = max {a, b, c} ta có
725
Khám phá tư duy Kỹ thuật giải bất ĐT Bài toán Max – Min – Đặng Thành Nam

1 3
3= a 2 + b 2 + c 2 ≥ a 2 + ( b + c )2 > a 2 ⇒ 1 ≤ a < 2 .
2 2
 b2 + c2 b2 + c2 
Ta chứng minh P ( a, b, c ) ≥ P  a, , .
 2 2 
 
Thật vậy
   b2 + c2 
P ( a , b, c ) − P  a ,

b2 + c2 b2 + c2
2
,
2 
(
 = b + c − 2 b2 + c2 + a  )
 2
− bc 

   

a (b − c ) ( b − c )2
2

n
= −
( )

.v
2 b + c + 2 b2 + c2

 

om
=
( b − c )2  a − 2 
 
2 

b + c + 2 b2 + c2 ( ) 
 .c
2 
( b − c)  2 
ok
≥  a− 
2 

a + 2 3 − a2 
 ( )
bo

( b − c )2  a 2 + a
=
2


( 
)
2 3 − a 2 − 2  ≥ 0, ∀a ∈ 1; 2 )
et

Mặt khác
  a 3 − a2 ( )
vi

P  a,

b2 + c2 b2 + c2
2
,
2
 =+

a 2 3−a − 2
( 2
−2 )
 
ng

 3 − a 2  3 − a 2 
= 1 −  a + a − 2
a

 2  2 
  
kh

3 − a2
Chú ý 1 − ≥0
2

a
3 − a2
2
(
+ a − 2 ≥ 0 ⇔ a2 3 − a2 ≥ 2 ( 2 − a )
2
)
(
⇔ ( a − 1) 8 − a3 − a 2 ≥ 0 )
Luôn đúng. Bất đẳng thức được chứng minh đẳng thức xảy ra khi và chỉ khi
a= b= c= 1 .

726
Cty TNHH MTV DVVH Khang Việt

=
Cách 2: Giả sử a max {a, b, c} ⇒ a ≥ 1 và vì a,b,c là độ dài ba cạnh một tam giác
nên a ≤ b + c ≤ 2 .
Khi đó
P = a + b + c − 2 − abc = a + b 2 + c 2 + 2bc − 2 − abc = a + t + 2 s − 2 − as .
 t
Với t =b2 + c2 , s =bc, s ∈ 0;  .
 2
 t
Xét hàm số f ( s ) = a + t + 2 s − 2 − as với s ∈ 0;  ta có
 2

n
1 1
f '( s ) = − a; f ''( s ) =
− <0.
t + 2s

.v
( t + 2s )3

om
1 1
Vì vậy f '( s ) ≤ f '(0)= − a= − a < 0, ∀a ∈ [1;2] .
t 3 − a2
 t
Vì vậy f(s) nghịch biến trên đoạn 0;  suy ra
 2
.c
ok
t at
f ( s ) ≥ f   = a + 2t − 2 −
2 2
( )
bo

a 3 − a2
(
= a + 2 3− a 2
) −2− 2
et

 3 − a2  3 − a 2 
= 1 −  a + a − 2 ≥ 0
  
vi

2 2
  
giả sử a min {a, b, c} ⇒ a ≤ 1 ta có
Bài 2. Không mất tính tổng quát =
ng

P ( a, b, c ) =( 2 − ab )( 2 − ac ) =4 − 2a ( b + c ) + a 2bc .
a

 b2 + c2 b2 + c2 
Ta chứng minh P ( a, b, c ) ≥ P  a,
kh

, 
 2 2 
 
Thật vậy bất đẳng thức tương đương với

(
−2a ( b + c ) + a 2bc ≥ −2a 2 b 2 + c 2 + a 2 . ) b2 + c2
2


( )
⇔ 2  2 b 2 + c 2 − ( b + c )  ≥ ( b − c )
 2
a 2

727
Khám phá tư duy Kỹ thuật giải bất ĐT Bài toán Max – Min – Đặng Thành Nam

4 (b − c )
2
≥ a (b − c )
2

(
2 b2 + c2 + b + c )
⇔ 4 ≥ a ( b + c ) + a 2 b2 + c2 ( )
Ta có

(
4 − a ( b + c ) − a 2 b 2 + c 2 ≥ 4 − 2a 2 b 2 + c 2 ) ( )
(
4 1 − a2 2 − a2)( )
=−
4 2a 2 3 − a ( 2
) = ≥0
( )

n
2 + a 2 3 − a2

.v
Vậy ta chỉ cần chứng minh

om
2 2
 b2 + c2     
 2 − 
2  
 2 − a
b2 + c2
2 
(
 ≥ 1 ⇔ a2 + 1 ) 2− a

3 − a2
2
 ≥2.

    
.c ( a + 1)2
(
Ta có 2 a 2 + 1 − ( a + 1) = ) 2
( a − 1)2 ≥ 0 ⇒ a 2 + 1 ≥ .
ok
2
Vậy ta chỉ cần chứng minh
bo

2
 3 − a2   3 − a2 
( )
a + 1  2
2
− a  ≥ 4 ⇔ ( a + 1)  2 − a ≥2
 2   2 
   
et


⇔ a  2 − ( a + 1)
3 − a2 
≥0⇔
(
a a 4 + 2a 3 − 2a 2 − 6a + 5
≥0
)
vi

 2  − 2
  2 + ( a + 1)
3 a
ng

2
(
⇔ a ( a − 1) a 2 + 4a + 5 ≥ 0
2
)
a

Luôn đúng. Bất đẳng thức được chứng minh đẳng thức xảy ra khi và chỉ khi
kh

a= b= c= 1 .
Bài 3. Bất đẳng thức cần chứng minh có dạng
1 1 1 13 25
P ( a , b, c ) = + + + − ≥ 0.
a b c a + b + c +1 4
Không mất tính tổng quát giả sử a= max {a, b, c} ⇒ a ≥ 1, x= bc ≤ 1 .
Ta chứng minh
(
P ( a, b, c ) ≥ P a, bc , bc . )

728
Cty TNHH MTV DVVH Khang Việt

Thật vậy

(
P ( a, b, c ) − P a, bc , bc = ) 1 1
+ −
b c
2
+
13

13
bc a + b + c + 1 a + 2 bc + 1

( ) ( )
2 2
b− c 13 b− c
= −
bc ( a + b + c + 1) ( a + 2 )
bc + 1
.
 
( 1
) 13
2
=−b c  − 
 bc ( a + b + c + 1) a + 2 bc + 1
 ( ) 

2 

n
(
≥ b − c 1 − )
13
≥0
 ( 3 + 1)( 3 + 1) 

.v
(  1
)
 2 13 25

om
Mặt khác P a, bc , bc = P  2 , x, x  = x 2 + + −
x  x 2x + 1 + 1 4
2
x
( x − 1)2 (8 x 4 + 20 x3 − 18 x 2 − 9 x + 8)
≥ 0, ∀x ∈ ( 0;1] .
.c
4 x ( x + 1) ( 2 x 2 − x + 1)
ok
Từ đó suy ra điều phải chứng minh. Đẳng thức xảy ra khi và chỉ khi a= b= c= 1 .
bo

Bài 4.
2
  a + b 2 
(
Ta có 1 + a 2 1 + b 2)( ) − 1 +    =
1
− ( a − b )  a 2 + 6ab + b 2 − 8 ≤ 0 với
2
et

  2   16  
vi

mọi a, b ≥ 1 .
ng

2
  a + b 2 
(
Do đó 1 + a 2
)(1 + b )
2
≤ 1 +    .
  2  
a

2 2
  a + b 2    6 − c 2 
( ) ( )
kh

Suy ra P ≤ 1 +    1 + c = 1 + 
2
  1 + c ≤ 125 .
2
  2     2  
Đẳng thức xảy ra khi và chỉ khi a= b= c= 2 .
Vậy giá trị lớn nhất của P bằng 125 đạt tại a= b= c= 2 .
Bài 5. Không mất tính tổng quát giả sử
 1 3
c ≥ 2 , a + b = 2 − c ≤ 1

=c max {a, b, c} ⇒ 
ab ≤ ( a + b ) =
2
1
 4 4
729
Khám phá tư duy Kỹ thuật giải bất ĐT Bài toán Max – Min – Đặng Thành Nam

2
  a + b 2 
Khi đó 1 + a ( 2
)(1 + b ) 2
≥ 1 + 
  2  
  .

2
  a + b 2  ( a − b )2 a 2 + 6ab + b2 − 8 ≥ 0 .
(
Thật vậy 1 + a 1 + b − 1 + 2
)(
  2  
  =− 2
16
) ( )
Đẳng thức xảy ra khi và chỉ khi a = b .
2 2
  a + b 2    3 c 2  125
Khi đó P ≥ 1 + c ( 2
) 1 + 
  2  
  =1 + c2 ( ) 1 +  −   ≥
  4 2   64
.

n
1
Đẳng thức xảy ra khi và chỉ khi a= b= c=

.v
.
2
( )( )( )

om
Cách 2 : Ta có 1 + a 2 1 + b 2 1 + c 2

= a 2 + b 2 + c 2 + a 2b 2 + b 2 c 2 + c 2 a 2 + a 2b 2 c 2 + 1
=
.c
( a + b + c )2 − 2 ( ab + bc + ca ) + ( ab + bc + ca )2 − 2abc ( a + b + c ) + a 2b2c 2 + 1
ok
13
− 2 ( ab + bc + ca ) + ( ab + bc + ca ) + a 2b 2 c 2 − 3abc
2
=
4
2 2 2
( ab + bc + ca − 1)2 +  abc −
3   1   11  125
bo

=  ≥  +  =
 2 4  8  64
3
a+b+c 1
et

1 3
Do ab + bc + ca ≤ ( a + b + =
c)
2
; abc ≤  = .
3 4  3  8
vi

1
Đẳng thức xảy ra khi và chỉ khi a= b= c= .
ng

2
 t2 
Bài 6. Không mất tính tổng quát giả sử a ≥ b ≥ c đặt t =b + c, s =bc, s ∈ 0;  ta
a

 4 
kh

có a = 1 − t .

(
Khi đó P =  a 6 + a3 b3 + c3 + b3c3  b3 + c3
  ) ( . )
=  a 6 + a3 ( b + c ) − 3a3bc ( b + c ) + b3c3  . ( b + c ) − 3bc ( b + c ) 
3 3
   
( )
= a 6 + a3t 3 − 3a3ts + s 3 t 3 − 3ts )(
Xét hàm số f ( s ) =( a + a t − 3a ts + s )( t − 3ts ) ta có
6 3 3 3 3 3

f '( s ) = ( 3s − 3a t )( t − 3ts ) − 3t ( a + a t − 3a ts + s ) .
2 3 3 6 3 3 3 3

730
Cty TNHH MTV DVVH Khang Việt

Sử dụng bất đẳng thức AM-GM ta có a 6 + a3t 3 − 3a3ts + s 3 ≥ 0 .


Theo điều kiện suy ra
 t4 
( )( ) ( )( )
3s 2 − 3a3t t 3 − 3ts = 3t s 2 − a3t t 2 − 3s ≤ 3t  s 2 −  t 2 − 3s ≤ 0 .
 8 
( )

( 3
)
Do đó f '( s ) ≤ 0 ⇒ P= f ( s ) ≤ f (0)= t 3 a 6 + a3t 3 = t 3 (1 − t ) t 3 + (1 − t )  .

3

 2
Xét hàm số g (t )= t 3 (1 − t ) t 3 + (1 − t )  với t ∈ 0;  ta có
3 3
   3
t = 0

n
 2
0 ⇔  1 (do t ∈ 0;  ).
−3t ( t − 1) ( 2t − 1) ; f '(t ) =
2 3
g '(t ) = 2

.v
t =  3
 2

om
1
Ta có g’(t) đổi dấu từ dương sang âm khi đi qua t =
nên g(t) đạt cực đại tại
2
1 1 1 1
t = hay P ≤ g (t ) ≤ g   =. Đẳng thức xảy ra khi và chỉ khi a= b= .c ,
2  2  256 2
c = 0 hoặc các hoán vị.
ok
1 1
Vậy giá trị lớn nhất của P bằng đạt tại a= b= , c= 0 hoặc các hoán vị.
bo

256 2
Nhận xét. Với a,b,c không âm thỏa mãn a + b + c =k . Chứng minh tương tự ta có

(a )( )( ) k9
et

3
+ b3 b3 + c 3 c 3 + a 3 ≤ .
256
vi

Bài 7. Ta cần chứng minh


P ( x, y, z ) = x 2 + y 2 + z 2 + x + y + z − 2 ( xy + yz + zx ) ≥ 0 .
ng

giả sử x min { x, y, z} ⇒ x ≤ 1, yz ≥ 1 .
Không mất tính tổng quát =
a

( ) ( ) ( y + z − 2x + 1 + 2 )
2
Xét hiệu P ( x, y, z ) − P x, yz , yz= y− z yz ≥ 0 .
kh

Ta chỉ cần chứng minh P x, = (


yz , yz
1 
)
P  2 , t , t  ≥ 0 với t = yz .
 t 
Thật vậy bất đẳng thức tương đương với

1 1 4
+ + 2t − ≥ 0 ⇔
(
( t − 1) 2t 3 + 4t 2 + 2t + 1
2

≥0.
)
t4 t2 t t4
Bất đẳng thức được chứng minh. Đẳng thức xảy ra khi và chỉ khi x= y= z= 1 .

731
Khám phá tư duy Kỹ thuật giải bất ĐT Bài toán Max – Min – Đặng Thành Nam

Bài 8. Bất đẳng thức cần chứng minh tương đương với
1 1 1 1
P ( a , b, c ) = a 2 + b 2 + c 2 −  a + b + c + + +  ≥ 0 .
2 a b c
a min {a, b, c} ⇒=
Không mất tính tổng quát giả sử = t bc ≥ 1 .
Ta chứng minh P (a, b, c) ≥ P (a, t , t ) .
 1 1 
Thật vậy P (a, b, c) − P(a, t , t ) = ( b − c ) 2 ( b + c ) 2 − 1 +   .
 2  bc  
 1 
≥ ( b − c ) 2 .  4 − (1 + 1)  ≥ 0
 

n
2

.v
1 (t − 1) 2 (3t 4 + 4t 3 + 5t 2 + 4t + 2)
Mặt khác = P (a, t , t ) P=
( 2 , t, t) ≥ 0.
t 2t 4

om
Bất đẳng thức được chứng minh. Đẳng thức xảy ra khi và chỉ khi a= b= c= 1 .
Bài 9. Ta cần chứng minh
 x z  21
( )
P ( x, y , z ) =
1 y
− 54 x3 + y 3 + z 3 −  + +  − ≥ 0 .
.c
xyz  yz zx xy  2
ok
Chú ý
( x + y + z )2 − x 2 − y 2 − z 2 − 54
(x ) 21
bo

=P ( x, y , z ) 3
+ y3 + z3 −
xyz 4
.
1 1 1  21
( )
et

= 2  + + − 27 x3 + y 3 + z 3  −
x y z  2
vi

Không mất tính tổng quát giả sử x = max { x, y, z} ta chứng minh


ng

 y+z y+z
P ( x, y , z ) ≥ P  x, , .
 2 2 

( y − z )2 ( 4 − 81yz ( y + z )2 )
a

 y+z y+z
Xét hiệu P ( x, y, z ) − P  x, ,= ≥0
kh


 2 2  2 yz ( y + z )
4
 y+z y+z 
 y+z+ + 
y+z y+z
yz ( y + z ) 4 y.z. 2 2
2
Bởi vì = . ≤ 4 
2 2  4 
 

=
( y + z )4 ≤ 1  2  4 = 4
 
4 4 3 81

732
Cty TNHH MTV DVVH Khang Việt

Ta cần chứng minh


y+z y+z 1− x 1− x
P(=x, , ) P ( x, , )≥0
2 2 2 2
1 4  (1 − x )3   21 0
⇔ 2 + − 27  x3 + − ≥
 x 1− x  4  2
  
( 3x − 2 )2 ( 9 x3 + 12 x 2 − 6 x + 1)
⇔ ≥0
2 x (1 − x )

n
Bất đẳng thức cuối luôn đúng. Bất đẳng thức được chứng minh.

.v
2 1
Đẳng thức xảy ra khi và chỉ khi x= , y= z= hoặc các hoán vị.
3 6

om
Bài 12. Gọi P là biểu thức vế trái và sử dụng bất đẳng thức C –S ta có

P≥
( a + b + c )2
( ) ( ) ( )
a 1 + b2 + c2 + b 1 + c2 + a 2 + c 1 + a 2 + b2
.c
ok
1
=
( )
1 + a ( b + c ) + a b 2 + c 2 + bc ( b + c )
2
bo

Ta tìm giá trị lớn nhất của biểu thức với giả thiết a = max {a, b, c} .

( ) ( )
Q = a 2 ( b + c ) + a b 2 + c 2 + bc ( b + c ) = a 2t + a t 2 − 2 s + ts .
et

 t2 
( )
vi

Xét hàm số f ( s ) = a 2t + a t 2 − 2 s + ts trên đoạn 0;  ta có


 4 
ng

2
a+t  1
f '( s ) =t − 2a ≤ 0 ⇒ f ( s ) ≤ f (0) = a 2t + at 2 = at ≤   = .
 2  4
a

1 5
kh

Do đó P ≥ ≥ .
1+ Q 4
1
Đẳng thức xảy ra khi và chỉ khi a= b= , c= 0 hoặc các hoán vị.
2
Giả sử x max { x, y, z} ⇒ y + z ≤ 2, x ≥ 1 ta có
Bài 13. =
2
  x + y 2 
( )( )
3 y + 5 3 z + 5 − 3 
2 2
 + 5  =
3
( )
( x − y )2  40 − 3 x 2 + y 2 + 6 xy  ≥ 0 .
  2   16

733
Khám phá tư duy Kỹ thuật giải bất ĐT Bài toán Max – Min – Đặng Thành Nam

2
  x + y 2 
(
Vậy ta chứng minh 3 x + 5 3 
2
)
  2 
 + 5  ≥ 512

2
  3 − x 2 
(
⇔ 3 x + 5 3 
2
)
  2 
 + 5 − 512 ≥ 0
 .

( )
⇔ ( x − 1) 3 x 4 − 30 x3 + 144 x 2 − 306 x + 317 ≥ 0
2

Bất đẳng thức luôn đúng.


Bài toán được chứng minh đẳng thức xảy ra khi và chỉ khi x= y= z= 1 .

n
( )( )( )
Bài 16. HD: Đưa về chứng minh 64 a 2 + bc b 2 + ca c 2 + ab ≤ ( a + b + c ) .
6

.v
om
.c
ok
bo
et
vi
a ng
kh

734

You might also like